You are on page 1of 372

G.R. No.

L-24693: Ermita-Malate Hotel & Motel


Operators Assoc., Inc vs Mayor of Manila
On 13 June 1963, the Manila Municipal Board enacted Ord
4760 and thesame was approved by then acting mayor
Astorga. Ord 4760 sought toregulate hotels and motels. It
classified them into 1st class (taxed at 6k/yr) and 2ndclass
(taxed at 4.5k/yr). It also compelled hotels/motels to get
thedemographics of anyone who checks in to their rooms. It
compelled hotels/motels to have wide open spaces so as not
to conceal their identity of their patrons. Ermita-Malate
impugned the validity of thelaw averring that such is
oppressive, arbitrary and against dueprocess. The lower
court as well as the appellate court ruled in favorof ErmitaMalate.

Mayor Lim signed into law City Ordinance No. 7774


prohibiting short-time admission, short-time admission rates,
and wash-up rate schemes in hotels, motels, inns, lodging
houses, pension houses, and similar establishments in the
city of manila. White Light Corporation and other operators
of drive-in-hotels and motels in Manila complained that the
Ordinance is unconstitutional and void since it violates the
right to privacy and the freedom of movement; an invalid
exercise of police power; and an unreasonable and
oppressive interference in their business. On the other hand,
the City of Manila argued that the Ordinance is a valid police
power measure. It asserts that the subject establishments
have gained notoriety as venue of prostitution, adultery
and fornications in Manila. Thus, it became the ideal haven
for prostitutes and thrill-seekers. Is Ordinance No. 7774
constitutional?

ISSUE: Whether or not Ord 4760 is against the due process


clause.

SUGGESTED ANSWER:

No, Ordinance No. 7774 is unconstitutional. The SC ruled


that the ordinance is an arbitrary and whimsical intrusion
HELD: TheSC ruled in favor of Astorga. There is a
into the rights of the establishments as well as their patrons.
presumption that the lawsenacted by Congress (in this case
The Ordinance needlessly restrains the operation of the
Mun Board) is valid. W/o a showing ora strong foundation
businesses of the petitioners as well as restricting the rights
of invalidity, the presumption stays. As in thiscase, there was of their patrons without sufficient justification. The
only a stipulation of facts and such cannot prevailover the
Ordinance rashly equates wash rates and renting out a room
more than twice a day with immorality without
presumption. Further, the ordinance is a valid exercise
ofPolice Power. There is no question but that the challenged accommodating innocuous intentions.
ordinancewas precisely enacted to minimize certain practices
That the Ordinance prevents the lawful uses of a wash rate
hurtful to public morals. This is to minimize prostitution.
depriving patrons of a product and the petitioners of
The increase in taxes notonly discourages hotels/motels in
lucrative business ties in with another constitutional requisite
doing any business other than legalbut also increases the
for the legitimacy of the Ordinance as a police power
revenue of the lgu concerned. And taxation is avalid exercise measure. It must appear that the interests of the public
generally, as distinguished from those of a particular class,
of police power as well. The due process contention
islikewise untenable, due process has no exact definition but require an interference with private rights and the means
has reasonas a standard. In this case, the precise reason why must be reasonably necessary for the accomplishment of the
purpose and not unduly oppressive of private rights. It must
the ordinance wasenacted was to curb down prostitution in
also be evident that no other alternative for the
the city which is reasonenough and cannot be defeated by
accomplishment of the purpose less intrusive of private
mere singling out of the provisions ofthe said ordinance
rights can work.
alleged to be vague.
More importantly, a reasonable relation must exist between
the purposes of the measure and the means employed for its
(White Light Corp. vs. City of ManilaG.R. No. 122846,
accomplishment, for even under the guise of protecting the
January 20, 2009)

public interest, personal rights and those pertaining to private


property will not be permitted to be arbitrarily invaded.
Lacking a concurrence of these requisites, the police
measure shall be struck down as an arbitrary intrusion into
private rights. As held in Morfe v. Mutuc, the exercise of
police power is subject to judicial review when life, liberty
or property is affected.
The behavior which the Ordinance seeks to curtail is in fact
already prohibited and could in fact be diminished simply by
applying existing laws. Less intrusive measures such as
curbing the proliferation of prostitutes and drug dealers
through active police work would be more effective in
easing the situation. So would the strict enforcement of
existing laws and regulations penalizing prostitution and
drug use. These measures would have minimal intrusion on
the businesses of the petitioners and other legitimate
merchants.
Further, it is apparent that the Ordinance can easily be
circumvented by merely paying the whole day rate without
any hindrance to those engaged in illicit activities. (visit
fellester.blogspot.com) Moreover, drug dealers and
prostitutes can in fact collect wash rates from their
clientele by charging their customers a portion of the rent for
motel rooms and even apartments.

White Light Corp., vs City of Manila


On 3 Dec 1992, then Mayor Lim signed into law Ord 7774
entitled An Ordinance prohibiting short time admission in
hotels, motels, lodging houses, pension houses and similar
establishments in the City of Manila. White Light Corp is an
operator of mini hotels and motels who sought to have the
Ordinance be nullified as the said Ordinance infringes on the
private rights of their patrons. The RTC ruled in favor of
WLC. It ruled that the Ordinance strikes at the personal
liberty of the individual guaranteed by the Constitution. The
City maintains that the ordinance is valid as it is a valid
exercise of police power. Under the LGC, the City is
empowered to regulate the establishment, operation and
maintenance of cafes, restaurants, beerhouses, hotels, motels,

inns, pension houses, lodging houses and other similar


establishments, including tourist guides and transports. The
CA ruled in favor of the City.
ISSUE: Whether or not Ord 7774 is valid.
HELD: The SC ruled that the said ordinance is null and void
as it indeed infringes upon individual liberty. It also violates
the due process clause which serves as a guaranty for
protection against arbitrary regulation or seizure. The said
ordinance invades private rights. Note that not all who goes
into motels and hotels for wash up rate are really there for
obscene purposes only. Some are tourists who needed rest or
to wash up or to freshen up. Hence, the infidelity sought to
be avoided by the said ordinance is more or less subjected
only to a limited group of people. The SC reiterates that
individual rights may be adversely affected only to the extent
that may fairly be required by the legitimate demands of
public interest or public welfare.
City of Manila vs Judge Perfecto Laguio
Police Power
On 30 Mar 1993, Mayor Lim signed into law Ord 7783
entitled AN ORDINANCE PROHIBITING THE
ESTABLISHMENT OR OPERATION OF BUSINESSES
PROVIDING CERTAIN FORMS OF AMUSEMENT,
ENTERTAINMENT, SERVICES AND FACILITIES IN
THE ERMITA-MALATE AREA, PRESCRIBING
PENALTIES FOR VIOLATION THEREOF, AND FOR
OTHER PURPOSES. It basically prohibited establishments
such as bars, karaoke bars, motels and hotels from operating
in the Malate District which was notoriously viewed as a red
light district harboring thrill seekers. Malate Tourist
Development Corporation avers that the ordinance is invalid
as it includes hotels and motels in the enumeration of places
offering amusement or entertainment. MTDC reiterates that
they do not market such nor do they use women as tools for
entertainment. MTDC also avers that under the LGC, LGUs

can only regulate motels but cannot prohibit their operation.


The City reiterates that the Ordinance is a valid exercise of
Police Power as provided as well in the LGC. The City
likewise emphasized that the purpose of the law is to
promote morality in the City.

Due to the death of one Maricris Sioson in 1991, Cory


banned the deployment of performing artists to Japan and
other destinations. This was relaxed however with the
introduction of the Entertainment Industry Advisory Council
which later proposed a plan to POEA to screen and train
performing artists seeking to go abroad. In pursuant to the
ISSUE: Whether or not Ordinance 7783 is valid.
proposal POEA and the secretary of DOLE sought a 4 step
plan to realize the plan which included an Artists Record
HELD: The SC ruled that the said Ordinance is null and
Book which a performing artist must acquire prior to being
void. The SC noted that for an ordinance to be valid, it must deployed abroad. The Federation of Talent Managers of the
not only be within the corporate powers of the local
Philippines assailed the validity of the said regulation as it
government unit to enact and must be passed according to
violated the right to travel, abridge existing contracts and
the procedure prescribed by law, it must also conform to the rights and deprives artists of their individual rights. JMM
following substantive requirements:
intervened to bolster the cause of FETMOP. The lower court
ruled in favor of EIAC.
(1) must not contravene the Constitution or any statute;
ISSUE: Whether or not the regulation by EIAC is valid.
(2) must not be unfair or oppressive;
HELD: The SC ruled in favor of the lower court. The
(3) must not be partial or discriminatory;
regulation is a valid exercise of police power. Police power
concerns government enactments which precisely interfere
(4) must not prohibit but may regulate trade;
with personal liberty or property in order to promote the
general welfare or the common good. As the assailed
(5) must be general and consistent with public policy; and
Department Order enjoys a presumed validity, it follows that
the burden rests upon petitioners to demonstrate that the said
(6) must not be unreasonable.
order, particularly, its ARB requirement, does not enhance
the public welfare or was exercised arbitrarily or
The police power of the City Council, however broad and
unreasonably. The welfare of Filipino performing artists,
far-reaching, is subordinate to the constitutional limitations
particularly the women was paramount in the issuance of
thereon; and is subject to the limitation that its exercise must Department Order No. 3. Short of a total and absolute ban
be reasonable and for the public good. In the case at bar, the against the deployment of performing artists to high risk
enactment of the Ordinance was an invalid exercise of
destinations, a measure which would only drive recruitment
delegated power as it is unconstitutional and repugnant to
further underground, the new scheme at the very least
general laws.
rationalizes the method of screening performing artists by
requiring reasonable educational and artistic skills from them
and limits deployment to only those individuals adequately
JMM Promotion and Management vs Court of Appeals
prepared for the unpredictable demands of employment as
Police Power
artists abroad. It cannot be gainsaid that this scheme at least
lessens the room for exploitation by unscrupulous
individuals and agencies.

Lao Ichong vs Jaime Hernandez


Treaties May Be Superseded by Municipal Laws in the
Exercise of Police Power

police power which, being inherent could not be bargained


away or surrendered through the medium of a treaty. Hence,
Ichong can no longer assert his right to operate his market
stalls in the Pasay city market.

Constitution does not rule out the entry of foreign


investments, goods, and services. While it does not
encourage their unlimited entry into the country, it does not
prohibit them either. In fact, it allows an exchange on the
Lao Ichong is a Chinese businessman who entered the
basis of equality and reciprocity, frowning only on foreign
country to take advantage of business opportunities herein
Representatives Gerardo S. Espina, et al. vs. Hon.
competition that is unfair. The key, as in all economies in the
abound (then) particularly in the retail business. For some Ronaldo Zamora, Jr., et al. G.R. No. 143855, September
world, is to strike a balance between protecting local
time he and his fellow Chinese businessmen enjoyed a
21, 2010.
businesses and allowing the entry of foreign investments and
monopoly in the local market in Pasay. Until in June 1954
services. More important, Section 10, Article XII of the
when Congress passed the RA 1180 or the Retail Trade
Constitutionality; Retail Trade Liberalization Act of 2000.
1987 Constitution gives Congress the discretion to reserve to
Nationalization Act the purpose of which is to reserve to
The Court dismissed petitioners argument that Republic Act Filipinos certain areas of investments upon the
Filipinos the right to engage in the retail business. Ichong
No. 8762, known as the Retail Trade Liberalization Act of
recommendation of the National Economic and
then petitioned for the nullification of the said Act on the
200, violates the mandate of the 1987 Constitution for the
Development Authority and when the national interest
ground that it contravened several treaties concluded by the State to develop a self-reliant and independent national
requires. Thus, Congress can determine what policy to pass
RP which, according to him, violates the equal protection
economy effectively controlled by Filipinos. The provisions and when to pass it depending on the economic exigencies.
clause (pacta sund servanda). He said that as a Chinese
of Article II of the 1987 Constitution, the declarations of
It can enact laws allowing the entry of foreigners into
businessman engaged in the business here in the country
principles and state policies, are not self-executing.
certain industries not reserved by the Constitution to Filipino
who helps in the income generation of the country he should Legislative failure to pursue such policies cannot give rise
citizens. In this case, Congress has decided to open certain
be given equal opportunity.
to a cause of action in the courts. Further, while Section 19, areas of the retail trade business to foreign investments
Article II of the 1987 Constitution requires the development instead of reserving them exclusively to Filipino citizens.
ISSUE: Whether or not a law may invalidate or supersede
of a self-reliant and independent national economy
treaties or generally accepted principles.
effectively controlled by Filipino entrepreneurs, it does not
The control and regulation of trade in the interest of the
impose a policy of Filipino monopoly of the economic
public welfare is of course an exercise of the police power of
HELD: Yes, a law may supersede a treaty or a generally
environment. The objective is simply to prohibit foreign
the State. A persons right to property, whether he is a
accepted principle. In this case, there is no conflict at all
powers or interests from maneuvering our economic policies Filipino citizen or foreign national, cannot be taken from him
between the raised generally accepted principle and with RA and ensure that Filipinos are given preference in all areas of without due process of law. In 1954, Congress enacted the
1180. The equal protection of the law clause does not
development. The 1987 Constitution takes into account the Retail Trade Nationalization Act (RA 1180) that restricts the
demand absolute equality amongst residents; it merely
realities of the outside world as it requires the pursuit of a
retail business to Filipino citizens. In denying the petition
requires that all persons shall be treated alike, under like
trade policy that serves the general welfare and utilizes all
assailing the validity of such Act for violation of the
circumstances and conditions both as to privileges conferred forms and arrangements of exchange on the basis of equality foreigners right to substantive due process of law, the
and liabilities enforced; and, that the equal protection clause and reciprocity; and speaks of industries which are
Supreme Court held that the law constituted a valid exercise
is not infringed by legislation which applies only to those
competitive in both domestic and foreign markets as well as of police power. The State had an interest in preventing alien
persons falling within a specified class, if it applies alike to
of the protection of Filipino enterprises against unfair
control of the retail trade and R.A. 1180 was reasonably
all persons within such class, and reasonable grounds exist
foreign competition and trade practices. Thus, while the
related to that purpose. That law is not arbitrary. Here, to
for making a distinction between those who fall within such Constitution mandates a bias in favor of Filipino goods,
the extent that RA 8762 lessens the restraint on the
class and those who do not.
services, labor and enterprises, it also recognizes the need for foreigners right to property or to engage in an ordinarily
business exchange with the rest of the world on the bases of lawful business, it cannot be said that the law amounts to a
For the sake of argument, even if it would be assumed that a equality and reciprocity and limits protection of Filipino
denial of the Filipinos right to property and to due process
treaty would be in conflict with a statute then the statute
enterprises only against foreign competition and trade
of law. Filipinos continue to have the right to engage in the
must be upheld because it represented an exercise of the
practices that are unfair. In other words, the 1987
kinds of retail business to which the law in question has

permitted the entry of foreign investors. Certainly, it is not


within the province of the Court to inquire into the wisdom
of RA 8762 save when it blatantly violates the Constitution.
But as the Court has said, there is no showing that the law
has contravened any constitutional mandate. The Court is not
convinced that the implementation of RA 8762 would
eventually lead to alien control of the retail trade business.
Petitioners have not mustered any concrete and strong
argument to support its thesis. The law itself has provided
strict safeguards on foreign participation in that business.
United States vs Luis Toribio
Police Power
Sometime in the 1900s, Toribio applied for a license to have
his carabao be slaughtered. His request was denied because
his carabao is found not to be unfit for work. He nevertheless
slaughtered his carabao without the necessary license. He
was eventually sued and was sentenced by the trial court. His
counsel in one way or the other argued that the law
mandating that one should acquire a permit to slaughter his
carabao is not a valid exercise of police power.

contention being that it amounts to a deprivation of property


of petitioners-appellants of their means of livelihood without
due process of law. The assailed ordinance is worded thus:
It shall be prohibited for any operator of any barber shop to
conduct the business of massaging customers or other
persons in any adjacent room or rooms of said barber shop,
or in any room or rooms within the same building where the
barber shop is located as long as the operator of the barber
shop and the room where massaging is conducted is the
same person. As noted in the appealed order, petitionersappellants admitted that criminal cases for the violation of
this ordinance had been previously filed and decided. The
lower court, therefore, held that a petition for declaratory
relief did not lie, its availability being dependent on there
being as yet no case involving such issue having been filed.

Even if such were not the case, the attack against the validity
cannot succeed. As pointed out in the brief of respondentsappellees, it is a police power measure. The objectives
behind its enactment are: (1) To be able to impose payment
of the license fee for engaging in the business of massage
clinic under Ordinance No. 3659 as amended by Ordinance
4767, an entirely different measure than the ordinance
ISSUE: Whether or not the said law is valid.
regulating the business of barbershops and, (2) in order to
forestall possible immorality which might grow out of the
HELD: The SC ruled against Toribio. The SC explained
construction of separate rooms for massage of customers.
that it is not a taking of the property for public use, within
This Court has been most liberal in sustaining ordinances
the meaning of the constitution, but is a just and legitimate
based on the general welfare clause. As far back as U.S. v.
exercise of the power of the legislature to regulate and
Salaveria, a 1918 decision, this Court through Justice
restrain such particular use of the property as would be
Malcolm made clear the significance and scope of such a
inconsistent with or injurious to the rights of the publics. All clause, which delegates in statutory form the police power
property is acquired and held under the tacit condition that it to a municipality. As above stated, this clause has been given
shall not be so used as to injure the equal rights of others or wide application by municipal authorities and has in its
greatly impair the public rights and interests of the
relation to the particular circumstances of the case been
community.
liberally construed by the courts. Such, it is well to really is
the progressive view of Philippine jurisprudence. As it was
TOMAS VELASCO vs ANTONIO VILLEGAS
then, so it has continued to be. There is no showing,
This is an appeal from an order of the lower court dismissing therefore, of the unconstitutionality of such ordinance.
a suit for declaratory relief challenging the constitutionality
based on Ordinance No. 4964 of the City of Manila, the
Agustin vs Edu

Generally Accepted Principles of International Law


Agustin is the owner of a Volkswagen Beetle Car. He is
assailing the validity of Letter of Instruction No 229 which
requires all motor vehicles to have early warning devices
particularly to equip them with a pair of reflectorized
triangular early warning devices. Agustin is arguing that
this order is unconstitutional, harsh, cruel and
unconscionable to the motoring public. Cars are already
equipped with blinking lights which is already enough to
provide warning to other motorists. And that the mandate to
compel motorists to buy a set of reflectorized early warning
devices is redundant and would only make manufacturers
and dealers instant millionaires.
ISSUE: Whether or not the said is EO is valid.
HELD: Such early warning device requirement is not an
expensive redundancy, nor oppressive, for car owners whose
cars are already equipped with 1) blinking-lights in the fore
and aft of said motor vehicles, 2) battery-powered blinking
lights inside motor vehicles, 3) built-in reflectorized tapes
on front and rear bumpers of motor vehicles, or 4) welllighted two (2) petroleum lamps (the Kinke) . . . because:
Being universal among the signatory countries to the said
1968 Vienna Conventions, and visible even under adverse
conditions at a distance of at least 400 meters, any motorist
from this country or from any part of the world, who sees a
reflectorized rectangular early warning device installed on
the roads, highways or expressways, will conclude, without
thinking, that somewhere along the travelled portion of that
road, highway, or expressway, there is a motor vehicle which
is stationary, stalled or disabled which obstructs or endangers
passing traffic. On the other hand, a motorist who sees any
of the aforementioned other built-in warning devices or the
petroleum lamps will not immediately get adequate advance
warning because he will still think what that blinking light is
all about. Is it an emergency vehicle? Is it a law enforcement
car? Is it an ambulance? Such confusion or uncertainty in the

mind of the motorist will thus increase, rather than decrease,


the danger of collision.
The Letter of Instruction in question was issued in the
exercise of the police power. That is conceded by petitioner
and is the main reliance of respondents. It is the submission
of the former, however, that while embraced in such a
category, it has offended against the due process and equal
protection safeguards of the Constitution, although the latter
point was mentioned only in passing. The broad and
expansive scope of the police power which was originally
identified by Chief Justice Taney of the American Supreme
Court in an 1847 decision, as nothing more or less than the
powers of government inherent in every sovereignty was
stressed in the aforementioned case of Edu v. Ericta thus:
Justice Laurel, in the first leading decision after the
Constitution came into force, Calalang v. Williams,
identified police power with state authority to enact
legislation that may interfere with personal liberty or
property in order to promote the general welfare. Persons
and property could thus be subjected to all kinds of
restraints and burdens in order to secure the general comfort,
health and prosperity of the state. Shortly after independence
in 1948, Primicias v. Fugoso reiterated the doctrine, such a
competence being referred to as the power to prescribe
regulations to promote the health, morals, peace, education,
good order or safety, and general welfare of the people. The
concept was set forth in negative terms by Justice Malcolm
in a pre-Commonwealth decision as that inherent and
plenary power in the State which enables it to prohibit all
things hurtful to the comfort, safety and welfare of society.
In that sense it could be hardly distinguishable as noted by
this Court in Morfe v. Mutuc with the totality of legislative
power. It is in the above sense the greatest and most
powerful attribute of government. It is, to quote Justice
Malcolm anew, the most essential, insistent, and at least
illimitable powers, extending as Justice Holmes aptly
pointed out to all the great public needs. Its scope, ever
expanding to meet the exigencies of the times, even to
anticipate the future where it could be done, provides enough

room for an efficient and flexible response to conditions and


circumstances thus assuring the greatest benefits. In the
language of Justice Cardozo: Needs that were narrow or
parochial in the past may be interwoven in the present with
the well-being of the nation. What is critical or urgent
changes with the time. The police power is thus a dynamic
agency, suitably vague and far from precisely defined, rooted
in the conception that men in organizing the state and
imposing upon its government limitations to safeguard
constitutional rights did not intend thereby to enable an
individual citizen or a group of citizens to obstruct
unreasonably the enactment of such salutary measures
calculated to insure communal peace, safety, good order, and
welfare.
It was thus a heavy burden to be shouldered by petitioner,
compounded by the fact that the particular police power
measure challenged was clearly intended to promote public
safety. It would be a rare occurrence indeed for this Court to
invalidate a legislative or executive act of that character.
None has been called to our attention, an indication of its
being non-existent. The latest decision in point, Edu v.
Ericta, sustained the validity of the Reflector Law, an
enactment conceived with the same end in view. Calalang v.
Williams found nothing objectionable in a statute, the
purpose of which was: To promote safe transit upon, and
avoid obstruction on roads and streets designated as national
roads . . . As a matter of fact, the first law sought to be
nullified after the effectivity of the 1935 Constitution, the
National Defense Act, with petitioner failing in his quest,
was likewise prompted by the imperative demands of public
safety.
G.R. No. L-49112: Agustin vs Edu
Generally Accepted Principles of International Law
Agustin is the owner of a Volkswagen Beetle Car. He is
assailing the validity of Letter of Instruction No 229 which
requires all motor vehicles to have early warning devices
particularly to equip them with a pair of reflectorized

triangular early warning devices. Agustin is arguing that


this order is unconstitutional, harsh, cruel and
unconscionable to the motoring public. Cars are already
equipped with blinking lights which is already enough to
provide warning to other motorists. And that the mandate to
compel motorists to buy a set of reflectorized early warning
devices is redundant and would only make manufacturers
and dealers instant millionaires.
ISSUE: Whether or not the said is EO is valid.
HELD: Such early warning device requirement is not an
expensive redundancy, nor oppressive, for car owners whose
cars are already equipped with 1) 'blinking-lights in the fore
and aft of said motor vehicles,' 2) 'battery-powered blinking
lights inside motor vehicles,' 3) 'built-in reflectorized tapes
on front and rear bumpers of motor vehicles,' or 4) 'welllighted two (2) petroleum lamps (the Kinke) . . . because:
Being universal among the signatory countries to the said
1968 Vienna Conventions, and visible even under adverse
conditions at a distance of at least 400 meters, any motorist
from this country or from any part of the world, who sees a
reflectorized rectangular early warning device installed on
the roads, highways or expressways, will conclude, without
thinking, that somewhere along the travelled portion of that
road, highway, or expressway, there is a motor vehicle which
is stationary, stalled or disabled which obstructs or endangers
passing traffic. On the other hand, a motorist who sees any
of the aforementioned other built-in warning devices or the
petroleum lamps will not immediately get adequate advance
warning because he will still think what that blinking light is
all about. Is it an emergency vehicle? Is it a law enforcement
car? Is it an ambulance? Such confusion or uncertainty in the
mind of the motorist will thus increase, rather than decrease,
the danger of collision.
Taxicab Operators vs Board of Transportation
Police Power

Petitioner Taxicab Operators of Metro Manila, Inc.


(TOMMI) is a domestic corporation composed of taxicab
operators, who are grantees of Certificates of Public
Convenience to operate taxicabs within the City of Manila
and to any other place in Luzon accessible to vehicular
traffic.

(3) Protection against arbitrary and unreasonable


classification and standard?

ISSUE: Whether or not there is a violation of the equal


protection clause by the implementation of the said circular.

HELD

HELD: The SC held that Circ 77-42 is valid. BOTs reason


for enforcing the Circular initially in Metro Manila is that
taxicabs in this city, compared to those of other places, are
subjected to heavier traffic pressure and more constant use.
Thus is of common knowledge. Considering that traffic
conditions are not the same in every city, a substantial
distinction exists so that infringement of the equal protection
clause can hardly be successfully claimed.

As enunciated in the preambular clauses of the challenged


BOT Circular, the overriding consideration is the safety and
comfort of the riding public from the dangers posed by old
and dilapidated taxis. The State, in the exercise of its police
On October 10, 1977, respondent Board of Transportation
power, can prescribe regulations to promote the health,
(BOT) issued Memorandum Circular No. 77-42 which reads: morals, peace, good order, safety and general welfare of the
people. It can prohibit all things hurtful to comfort, safety
SUBJECT: Phasing out and Replacement of Old and
and welfare of society. [5] It may also regulate property
Dilapidated Taxis
rights. [6] In the language of Chief Justice Enrique M.
Fernando the necessities imposed by public welfare may
On January 27, 1981, petitioners filed a Petition with the
justify the exercise of governmental authority to regulate
BOT, docketed as Case No. 80-7553, seeking to nullify MC even if thereby certain groups may plausibly assert that their
No. 77-42 or to stop its implementation; to allow the
interests are disregarded.
registration and operation in 1981 and subsequent years of
taxicabs of model 1974, as well as those of earlier models
Taxicab Operators of Metro Manila Inc vs The Board of
which were phased-out, provided that, at the time of
Transportation et al
registration, they are roadworthy and fit for operation.
Equal Protection Phasing Out of Old Taxis in MM but
not Elsewhere
ISSUE
On 10 Oct 1977, BOT issued Circ 77-42 which has for its
purpose the phasing out of old and dilapidated taxis which
are 6 years older. The law is set to be immediately
implemented in Metro Manila first before it would be
implemented elsewhere. Pursuant to this, the Director of the
Bureau of Land Transportation issued Circ 52 which is the
B. Granting arguendo, that respondents did comply with the
IRR of the law in the NCR. TOMMI assailed the
procedural requirements imposed by Presidential Decree No.
constitutionality of the law. It avers, among other things, that
101, would the implementation and enforcement of the
the Circular in question violates their right to equal
assailed memorandum circulars violate the petitioners
protection of the law because the same is being enforced in
constitutional rights to.
Metro Manila only and is directed solely towards the taxi
industry. At the outset it should be pointed out that
(1) Equal protection of the law;
implementation outside Metro Manila is also envisioned in
Memorandum Circular No. 77-42.
(2) Substantive due process; and
A. Did BOT and BLT promulgate the questioned
memorandum circulars in accord with the manner required
by Presidential Decree No. 101, thereby safeguarding the
petitioners constitutional right to procedural due process?

In so far as the non-application of the assailed Circulars to


other transportation services is concerned, it need only be
recalled that the equal protection clause does not imply that
the same treatment be accorded all and sundry. It applies to
things or persons identically or similarly situated. It permits
of classification of the object or subject of the law provided
classification is reasonable or based on substantial
distinction, which make for real differences, and that it must
apply equally to each member of the class. What is required
under the equal protection clause is the uniform operation by
legal means so that all persons under identical or similar
circumstance would be accorded the same treatment both in
privilege conferred and the liabilities imposed. The
challenged Circulars satisfy the foregoing criteria.
Bautista vs Juinio
Police Power
The validity of an energy conservation measure, Letter of
Instruction No. 869, issued on May 31, 1979 the response
to the protracted oil crisis that dates back to 1974 is put in
issue in this prohibition proceeding filed by petitioners,
spouses Mary Concepcion Bautista and Enrique D. Bautista,
for being allegedly violative of the due process and equal
protection guarantees of the Constitution. The use of
private motor vehicles with H and EH plates on week-ends
and holidays was banned from [12:00] a.m. Saturday
morning to 5:00 a.m. Monday morning, or 1:00 a.m. of the

holiday to 5:00 a.m. of the day after the holiday. 2 Motor


vehicles of the following classifications are exempted: (a) S
(Service); (b) T (Truck); (c) DPL (Diplomatic); (d) CC
(Consular Corps); (e) TC (Tourist Cars

economy and sustain its developmental growth; [Whereas],


to cushion the effect of increasing oil prices and avoid fuel
supply disruptions, it is imperative to adopt a program
directed towards the judicious use of our energy resources
complemented with intensified conservation efforts and
efficient utilization thereof; . . .. What is undeniable is that
the action taken is an appropriate response to a problem that
This Court gave due course to the petition requiring
presses urgently for solution. It may not be the only
respondent to answer. There was admission of the facts as
alternative, but its reasonableness is immediately apparent.
substantially alleged except, as previously noted, that the ban Thus, to repeat, substantive due process, which is the
starts at 12:00 a.m. rather than 1:00 a.m. of a Saturday or of epitome of reasonableness and fair play, is not ignored, much
a holiday and as to the mention of a Willys Kaiser jeep
less infringed.
being registered in the name of a certain Teresita Urbina,
about which respondents had no knowledge. There was a
In the interplay between such a fundamental right and police
denial of the allegations that the classification of vehicles
power, especially so where the assailed governmental action
into heavy (H) and extra heavy (EH) on the other hand and
deals with the use of ones property, the latter is accorded
light and bantam on the other hand was violative of equal
much leeway. That is settled law. What is more, it is good
protection and the regulation as to the use of the former cars law. Due process, therefore, cannot be validly invoked. As
on the dates specified a transgression of due process. The
stressed in the cited Ermita-Malate Hotel decision: To hold
answer likewise denied that there was an undue delegation of otherwise would be to unduly restrict and narrow the scope
legislative power, reference being made to the Land
of police power which has been properly characterized as the
Transportation and Traffic Code. There was also a
most essential, insistent and the least limitable of powers,
procedural objection raised, namely, that what is sought
extending as it does to all the great public needs. It would
amounts at most to an advisory opinion rather than an
be, to paraphrase another leading decision, to destroy the
adjudication of a case or controversy.
very purpose of the state if it could be deprived or allowed
itself to be deprived of its competence to promote public
It is true, of course, that there may be instances where a
health, public morals, public safety and the general welfare.
police power measure may, because of its arbitrary,
Negatively put, police power is that inherent and plenary
oppressive or unjust character, be held offensive to the due
power in the State which enables it to prohibit all that is
process clause and, therefore, may, when challenged in an
hurtful to the comfort, safety, and welfare of society.
appropriate legal proceeding, be declared void on its face.
This is not one of them. A recital of the whereas clauses of
Mary Concepcion Bautista et al vs Alfredo Juinio et al
Equal Protection Distinction Between Heavy and Extra
the Letter of Instruction makes it clear. Thus: [Whereas],
Heavy Cars and Others
developments in the international petroleum supply situation
continue to follow a trend of limited production and
Bautista is assailing the constitutionality of LOI 869 issued
spiralling prices thereby precluding the possibility of
in 1979 which classified vehicles into Heavy and Extra
immediate relief in supplies within the foreseeable future;
heavy. The LOI further banned these vehicles during
[Whereas], the uncertainty of fuel supply availability
weekends and holidays that is from 5am Saturday until 5am
underscores a compelling need for the adoption of positive
Monday. Purpose of this law is to curb down petroleum
measures designed to insure the viability of the countrys

consumption as bigger cars consume more oil. Bautista


claimed the LOI to be discriminatory as it made an
assumption that H and EH cars are Heavy on petroleum
consumption when in fact there are smaller cars which are
also big on oil consumption. Further, the law restricts their
freedom to enjoy their car while others who have smaller
cars may enjoy theirs. Bautista avers that there is no rational
justification for the ban being imposed on vehicles classified
as Heavy (H) and extra-Heavy (EH), for precisely those
owned by them fall within such category.
ISSUE: Whether or not the LOI violates equal protection.
HELD: The SC held that Bautista was not able to make
merit out of her contention. The classification on cars on its
face cannot be characterized as an affront to reason. The
ideal situation is for the laws benefits to be available to all,
that none be placed outside the sphere of its coverage. Only
thus could chance and favor be excluded and the affairs of
men governed by that serene and impartial uniformity, which
is of the very essence of the idea of law. The actual, given
things as they are and likely to continue to be, cannot
approximate the ideal. Nor is the law susceptible to the
reproach that it does not take into account the realities of the
situation. . . . To assure that the general welfare be promoted,
which is the end of law, a regulatory measure may cut into
the rights to liberty and property. Those adversely affected
may under such circumstances invoke the equal protection
clause only if they can show that the governmental act
assailed, far from being inspired by the attainment of the
common weal was prompted by the spirit of hostility, or at
the very least, discrimination that finds no support in reason.
It suffices then that the laws operate equally and uniformly
on all persons under similar circumstances or that all persons
must be treated in the same manner, the conditions not being
different, both in the privileges conferred and the liabilities
imposed. Favoritism and undue preference cannot be
allowed. For the principle is that equal protection and
security shall be given to every person under circumstances,
which if not identical are analogous. If law be looked upon

in terms of burden or charges, those that fall within a class


should be treated in the same fashion, whatever restrictions
cast on some in the group equally binding on the rest.
G.R. No. 78742: Association of Small Landowners vs
Secretary of Agrarian Reform
Equal Protection

These are 3 cases consolidated questioning the


constitutionalityof the Agrarian Reform Act. Article XIII on
Social Justice and Human Rightsincludes a call for the
adoption by the State of an agrarian reform program.The
State shall, by law, undertake an agrarian reform program
founded on theright of farmers and regular farmworkers,
who are landless, to own directly orcollectively the lands
they till or, in the case of other farmworkers, toreceive a just
share of the fruits thereof. RA 3844, Agricultural Land
ReformCode, had already been enacted by Congress on
August 8, 1963. This wassubstantially superseded almost a
decade later by PD 27, which was promulgatedon Oct 21,
1972, along with martial law, to provide for the
compulsoryacquisition of private lands for distribution
among tenant-farmers and tospecify maximum retention
limits for landowners. On July 17, 1987, Cory issuedEO 228,
declaring full land ownership in favor of the beneficiaries of
PD 27and providing for the valuation of still unvalued lands
covered by the decreeas well as the manner of their payment.
This was followed on July 22, 1987 byPP 131, instituting a
comprehensive agrarian reform program (CARP), and EO
229,providing the mechanics for its implementation.
Afterwhich is the enactment ofRA 6657, Comprehensive
Agrarian Reform Law of 1988, which Cory signed on
June10. This law, while considerably changing the earlier
mentioned enactments,nevertheless gives them suppletory
effect insofar as they are not inconsistentwith its provisions.
Inconsidering the rentals as advance payment on
the land, the executive orderalso deprives the petitioners of

their property rights as protected by dueprocess. The equal


protection clause is also violated because the order placesthe
burden of solving the agrarian problems on the owners only
of agriculturallands. No similar obligation is imposed on the
owners of other properties.

Classification has been defined as the grouping of persons


orthings similar to each other in certain particulars and
different from eachother in these same particulars. To be
valid, it must conform to the followingrequirements:
(1) it must be based on substantial distinctions;

Thepetitioners maintain that in declaring the


(2) it must be germane to the purposes of the law;
beneficiaries under PD 27 to be theowners of the lands
occupied by them, EO 228 ignored judicial prerogatives
(3) it must not be limited to existing conditions only; and
andso violated due process. Worse, the measure would not
solve the agrarianproblem because even the small farmers
are deprived of their lands and theretention rights guaranteed (4) it must apply equally to all the members of the class.
by the Constitution.
In hiscomment the Sol-Gen asserted that the
alleged violation of the equal protectionclause, the sugar
planters have failed to show that they belong to a
differentclass and should be differently treated.

The Court finds that all these requisites have been met by
themeasures here challenged as arbitrary and discriminatory.

Equal protection simply means that all persons or


The Comment also suggests the possibilityof Congress first thingssimilarly situated must be treated alike both as to the
rights conferred andthe liabilities imposed. The petitioners
distributing public agricultural lands and scheduling
theexpropriation of private agricultural lands later. From this have not shown that they belong to adifferent class and
entitled to a different treatment. The argument that notonly
viewpoint, thepetition for prohibition would be premature.
landowners but also owners of other properties must be
made to share theburden of implementing land reform must
ISSUE: Whether or notthere was a violation of the equal
be rejected. There is a substantialdistinction between these
protection clause.
two classes of owners that is clearly visible exceptto those
who will not see. There is no need to elaborate on this
HELD: The SC ruledaffirming the Sol-Gen. The argument
matter. In anyevent, the Congress is allowed a wide leeway
of the small farmers that they have beendenied equal
in providing for a validclassification. Its decision is accorded
protection because of the absence of retention limits has
alsobecome academic under Sec 6 of RA 6657. Significantly, recognition and respect by the courtsof justice except only
they too have notquestioned the area of such limits. There is where its discretion is abused to the detriment of theBill of
Rights.
also the complaint that theyshould not be made to share the
burden of agrarian reform, an objection alsomade by the
sugar planters on the ground that they belong to a particular DECS vs San Diego
Facts:
classwith particular interests of their own. However, no
San Diego, private respondent graduate of UE (Zoology),
evidence has been submittedto the Court that the requisites
took NMAT 3 times and flunked 3 times. The fourth time he
of a valid classification have been violated.
will take NMAT, he was rejected by DECS and DCEM for
the rule that:

A student shall be allowed only three (3) chances to take


the NMAT. After three (3) successive failures, a student shall
not be allowed to take the NMAT for the fourth time.
Ramon Guevarra then went to the Regional Trial Court of
Valenzuela, Metro Manila, to compel his admission to the
test. He invoked his constitutional rights to academic
freedom and quality education.
By agreement of the parties, herein defendant was allowed to
take the NMAT scheduled on April 16, 1989, subject to the
outcome of his petition.
In an amended petition filed with leave of court, San Diego
squarely challenged the constitutionality of MECS Order
No. 12, Series of 1972, containing the above-cited rule. The
additional grounds raised were due process and equal
protection. Respondent Judge Teresita Dizon-Capulong ruled
that the MECS Order No. 12, Series of 1972 was invalid and
held that the petitioner had been deprived of his right to
pursue a medical education through an arbitrary exercise of
the police power.
DECS VS. SAN DIEGO, 180 SCRA 533

The proper exercise of the police power of the State requires


the concurrence of a lawful subject and a lawful method.
The subject of the challenged regulation is certainly within
the ambit of the police power. It is the right and indeed the
responsibility of the State to insure that the medical
profession is not infiltrated by incompetents to whom
patients may unwarily entrust their lives and health.
The method employed by the challenged regulation is not
irrelevant to the purpose of the law nor is it arbitrary or
oppressive. The thee-flunk rule is intended to insulate the
medical schools and ultimately the medical profession from
the intrusion of those not qualified to be doctors.

VILLANUEVA, ET. AL. VS CASTAEDA, JR., ET. AL.


G.R. No. L-61311 September 2l, 1987 (damnun absque
injuria)
Appeal from a decision of CFI Pampanga holding that the
land in question, being public in nature, was beyond the
commerce of man and therefore could not be the subject of
private occupancy.
CRUZ, J.:

Facts: In the vicinity of the public market of San Fernando,


Pampanga, there stands on a strip of land, a conglomeration
of vendors stalls together. The petitioners claim they have a
right to remain in and conduct business in this area by virtue
of a previous authorization (Resolution no. 28) granted to
While every person is entitled to aspire to be a doctor, he
them by the municipal government. The respondents deny
does not have a constitutional right to be a doctor. This is
this and justify the demolition of their stalls as illegal
true of any other calling in which the public interest is
constructions on public property per municipal council
involved, and the closer the line, the longer the bridge to
Resolution G.R. No. 29, which declared the subject area as
one's ambition. The State has the responsibility to harness its "the parking place and as the public plaza of the
human resources and to see to it that they are, not dissipated municipality, thereby impliedly revoking Resolution No.
or, no less worse, not used at all. These resources must be
218.
FACTS:
applied in a manner that will best promote the common good
1. Private respondent Roberto Rey San Diego graduated
Issue: WON petitioners have the right to occupy the subject
from the University of the East with a BS degree in Zoology. while also giving the individual a sense of satisfaction.
land.
2. He took the NMAT three times and flunked it as many
times.
The right to quality education is not absolute. The
Ruling: Petition Dismissed.
3. When he applied again petitioner rejected him because it
Constitution also provides that "every citizen has a right to
It is a well-settled doctrine that the town plaza cannot be
contended that under the NMAT rule: a student shall be
choose a profession or course of study, subject to fair,
used for the construction of market stalls, and that such
allowed to take 3 chances to take the NMAT. After three
structures constitute a nuisance subject to abatement
successive failures, a student shall not be allowed to take the reasonable and equitable admission and academic
requirements.
{Art.
XIV,
Sec.5
(3)}
according to law. The petitioners had no right in the first
NMAT for the fourth time.
place to occupy the disputed premises and cannot insist in
4. Private respondent went to the RTC for a petition
The
contention
that
the
challenged
rule
violates
the
equal
remaining there now on the strength of their alleged lease
mandamus invoking his right to academic freedom and
protection clause is not well taken. A law does not have to
contracts. Even assuming a valid lease of the property in
quality education.
operate with equal force on all persons or things to be
dispute, the resolution could have effectively terminated the
5. Respondent Judge Teresita Dizon-Capulong declared the
conformable to Art. III, Sec 1 of the Constitution.
agreement for it is settled that the police power cannot be
challenged order unconstitutional.
surrendered or bargained away through the medium of a
6. Respondent Judge held that San Diego has been deprived
There
would
be
unequal
protection
if
some
applicants
who
contract. Hence, the loss or damage caused to petitioners, in
of his right to pursue a medical education through an
have
passed
the
tests
are
admitted
and
others
who
have
also
the case at bar, does not constitute a violation of a legal right
arbitrary exercise of police power.
qualified are denied entrance. In other words, what the equal or amount to a legal wrong - damnum absque injuria.
protection requires is equality among equals.
PRC vs. De Guzman
ISSUE:
Is person who has failed the NMAT three times entitled to
moral of the story: Try and try until you suceed, but if you Facts: The respondents are all graduates of the Fatima
take it again.
fail deny you even tried.
College of Medicine, Valenzuela City, Metro Manila. They
passed the Physician Licensure Examination conducted in
HELD:

February 1993 by the Board of Medicine (Board). Petitioner


Professional Regulation Commission (PRC) then released
their names as successful examinees in the medical licensure
examination. Shortly thereafter, the Board observed that the
grades of the seventy-nine successful examinees from
Fatima College in the two most difficult subjects in the
medical licensure exam, Biochemistry (Bio-Chem) and
Obstetrics and Gynecology (OB-Gyne), were unusually and
exceptionally high. Eleven Fatima examinees scored 100%
in Bio-Chem and ten got 100% in OB-Gyne, another eleven
got 99% in Bio-Chem, and twenty-one scored 99% in OBGyne.
For its part, the NBI found that the questionable passing
rate of Fatima examinees in the [1993] Physician
Examination leads to the conclusion that the Fatima
examinees gained early access to the test questions.
Issue: Was the act pursuant to R.A. 2382 a valid exercise of
police power
Ruling: Yes, it is true that this Court has upheld the
constitutional right of every citizen to select a profession or
course of study subject to a fair, reasonable, and equitable
admission and academic requirements. But like all rights and
freedoms guaranteed by the Charter, their exercise may be so
regulated pursuant to the police power of the State to
safeguard health, morals, peace, education, order, safety, and
general welfare of the people. Thus, persons who desire to
engage in the learned professions requiring scientific or
technical knowledge may be required to take an examination
as a prerequisite to engaging in their chosen careers
Chavez vs. COMELEC
Fact: Petitioner Chavez, on various dates, entered into
formal agreements with certain establishments to endorse
their products. On August 18, 2003, he authorized a certain
Andrew So to use his name and image for 96 North, a
clothing company. Petitioner also signed Endorsement
Agreements with Konka International Plastics
Manufacturing Corporation and another corporation
involved in the amusement and video games business, GBox. These last two agreements were entered into on
October 14, 2003 and November 10, 2003, respectively.
Pursuant to these agreements, three billboards were set up
along the Balintawak Interchange of the North Expressway.
One billboard showed petitioner promoting the plastic
products of Konka International Plastics Manufacturing
Corporation, and the other two showed petitioner endorsing

the clothes of 96 North. One more billboard was set up


along Roxas Boulevard showing petitioner promoting the
game and amusement parlors of G-Box.
Issue: Is Section 32 of COMELEC Resolution No. 6520 an
invalid exercise of police power?
Ruling: No, Police power, as an inherent attribute of
sovereign.
Didipio Earth Savers Multipurpose Association et al vs
DENR Sec Elisea Gozun et al
Police Power Eminent Domain
In 1987, Cory rolled out EO 279 w/c empowered DENR to
stipulate with foreign companies when it comes to either
technical or financial large scale exploration or mining. In
1995, Ramos signed into law RA 7942 or the Philippine
Mining Act. In 1994, Ramos already signed an FTAA with
Arimco Mining Co, an Australian company. The FTAA
authorized AMC (later CAMC) to explore 37,000 ha of land
in Quirino and N. Vizcaya including Brgy Didipio. After the
passage of the law, DENR rolled out its implementing RRs.
Didipio petitioned to have the law and the RR to be annulled
as it is unconstitutional and it constitutes unlawful taking of
property. In seeking to nullify Rep. Act No. 7942 and its
implementing rules DAO 96-40 as unconstitutional,
petitioners set their sight on Section 76 of Rep. Act No. 7942
and Section 107 of DAO 96-40 which they claim allow the
unlawful and unjust taking of private property for private
purpose in contradiction with Section 9, Article III of the
1987 Constitution mandating that private property shall not
be taken except for public use and the corresponding
payment of just compensation. They assert that public
respondent DENR, through the Mining Act and its
Implementing Rules and Regulations, cannot, on its own,
permit entry into a private property and allow taking of land
without payment of just compensation.
Traversing petitioners assertion, public respondents argue
that Section 76 is not a taking provision but a valid exercise
of the police power and by virtue of which, the state may
prescribe regulations to promote the health, morals, peace,

education, good order, safety and general welfare of the


people. This government regulation involves the adjustment
of rights for the public good and that this adjustment curtails
some potential for the use or economic exploitation of
private property. Public respondents concluded that to
require compensation in all such circumstances would
compel the government to regulate by purchase.
ISSUE: Whether or not RA 7942 and the DENR RRs are
valid.
HELD: The SC ruled against Didipio. The SC noted the
requisites of eminent domain. They are;
(1)

the expropriator must enter a private property;

(2)
period.

the entry must be for more than a momentary

(3)
authority;

the entry must be under warrant or color of legal

(4)
the property must be devoted to public use or
otherwise informally appropriated or injuriously affected;
(5)
the utilization of the property for public use
must be in such a way as to oust the owner and deprive him
of beneficial enjoyment of the property.
In the case at bar, Didipio failed to show that the law is
invalid. Indeed there is taking involved but it is not w/o just
compensation. Sec 76 of RA 7942 provides for just
compensation as well as section 107 of the DENR RR. To
wit,
Section 76. xxx Provided, that any damage to the property of
the surface owner, occupant, or concessionaire as a
consequence of such operations shall be properly
compensated as may be provided for in the implementing
rules and regulations.

Section 107. Compensation of the Surface Owner and


Occupant- Any damage done to the property of the surface
owners, occupant, or concessionaire thereof as a
consequence of the mining operations or as a result of the
construction or installation of the infrastructure mentioned in
104 above shall be properly and justly compensated.

a public cemetery for this purpose, the city passes the burden
Held: Section 9 of the City ordinance in question is not a
to private cemeteries.
valid exercise of police power. Section 9 cannot be justified
under the power granted to Quezon City to tax, fix the
CITY GOVERNMENT OF QUEZON CITY VS.
license fee, and regulate such other business, trades, and
ERICTA [122 SCRA 759; G.R. No. L-34915; 24 Jun
occupation as may be established or practiced in the City.
1983]

Facts: Section 9 of Ordinance No. 6118, S-64, entitled


Further, mining is a public policy and the government can
"Ordinance Regulating The Establishment, Maintenance And
invoke eminent domain to exercise entry, acquisition and use Operation Of Private Memorial Type Cemetery Or Burial
of private lands.
Ground Within The Jurisdiction Of Quezon City And
Providing Penalties For The Violation Thereof" provides:
City Government of QC vs Judge Ericta & Himlayang
Sec. 9. At least six (6) percent of the total area of the
Pilipino
memorial park cemetery shall be set aside for charity burial
Police Power Not Validly Exercised
of deceased persons who are paupers and have been
Quezon City enacted an ordinance entitled ORDINANCE residents of Quezon City for at least 5 years prior to their
death, to be determined by competent City Authorities. The
REGULATING THE ESTABLISHMENT,
area so designated shall immediately be developed and
MAINTENANCE AND OPERATION OF PRIVATE
should be open for operation not later than six months from
MEMORIAL TYPE CEMETERY OR BURIAL GROUND the
date
of
approval
of
the
application.
WITHIN THE JURISDICTION OF QUEZON CITY AND
For several years, the aforequoted section of the Ordinance
PROVIDING PENALTIES FOR THE VIOLATION
THEREOF The law basically provides that at least six (6) was not enforced but seven years after the enactment of the
percent of the total area of the memorial park cemetery shall ordinance, the Quezon City Council passed a resolution to
request the City Engineer, Quezon City, to stop any further
be set aside for charity burial of deceased persons who are
selling and/or transaction of memorial park lots in Quezon
paupers and have been residents of Quezon City for at least 5 City where the owners thereof have failed to donate the
years prior to their death, to be determined by competent
required 6% space intended for paupers burial.
City Authorities. QC justified the law by invoking police
The Quezon City Engineer then notified respondent
power.
Himlayang Pilipino, Inc. in writing that Section 9 of the
ordinance
would
be
enforced.
ISSUE: Whether or not the ordinance is valid.

Bill of rights states that 'no person shall be deprived of life,


liberty or property without due process of law' (Art. Ill,
Section 1 subparagraph 1, Constitution). On the other hand,
there are three inherent powers of government by which the
state interferes with the property rights, namely-. (1) police
power,
(2)
eminent
domain,
(3)
taxation.
The police power of Quezon City is defined in sub-section
00, Sec. 12, Rep. Act 537 that reads as follows:
To make such further ordinance and regulations not
repugnant to law as may be necessary to carry into effect and
discharge the powers and duties conferred by this act and
such as it shall deem necessary and proper to provide for the
health and safety, , and for the protection of property
therein; and enforce obedience thereto with such lawful fines
or penalties as the City Council may prescribe under the
provisions of subsection (jj) of this section.

The power to regulate does not include the power to prohibit.


The power to regulate does not include the power to
confiscate. The ordinance in question not only confiscates
but also prohibits the operation of a memorial park cemetery,
because under Section 13 of said ordinance, 'Violation of the
provision thereof is punishable with a fine and/or
imprisonment and that upon conviction thereof the permit to
operate and maintain a private cemetery shall be revoked or
cancelled. The confiscatory clause and the penal provision
Respondent Himlayang Pilipino reacted by filing a petition
HELD: The SC held the law as an invalid exercise of police for declaratory relief, prohibition and mandamus with in effect deter one from operating a memorial park cemetery.
power. There is no reasonable relation between the setting
preliminary injunction seeking to annul Section 9 of the
aside of at least six (6) percent of the total area of all private Ordinance in question. Respondent alleged that the same is
Moreover, police power is defined by Freund as 'the power
contrary to the Constitution, the Quezon City Charter, the
cemeteries for charity burial grounds of deceased paupers
Local Autonomy Act, and the Revised Administrative Code. of promoting the public welfare by restraining and regulating
and the promotion of health, morals, good order, safety, or
the use of liberty and property'. It is usually exerted in order
the general welfare of the people. The ordinance is actually a
to merely regulate the use and enjoyment of property of the
taking without compensation of a certain area from a
owner. If he is deprived of his property outright, it is not
Issue: Whether or Not Section 9 of the ordinance in question
PRIVATE cemetery to benefit paupers who are charges of
is
a
valid
exercise
of
police
power. taken for public use but rather to destroy in order to promote
the
general
welfare.
the municipal corporation. Instead of building or maintaining

It seems to the court that Section 9 of Ordinance No. 6118,


Series of 1964 of Quezon City is not a mere police
regulation but an outright confiscation. It deprives a person
of his private property without due process of law, nay, even
without compensation.
Restituto Ynot vs Intermediate Appellate Court
Police Power Not Validly Exercised
There had been an existing law which prohibited the
slaughtering of carabaos (EO 626). To strengthen the law,
Marcos issued EO 626-A which not only banned the
movement of carabaos from interprovinces but as well as the
movement of carabeef. On 13 Jan 1984, Ynot was caught
transporting 6 carabaos from Masbate to Iloilo. He was then
charged in violation of EO 626-A. Ynot averred EO 626-A
as unconstitutional for it violated his right to be heard or his
right to due process. He said that the authority provided by
EO 626-A to outrightly confiscate carabaos even without
being heard is unconstitutional. The lower court ruled
against Ynot ruling that the EO is a valid exercise of police
power in order to promote general welfare so as to curb
down the indiscriminate slaughter of carabaos.

immediately condemned and punished. The conferment on


the administrative authorities of the power to adjudge the
guilt of the supposed offender is a clear encroachment on
judicial functions and militates against the doctrine of
separation of powers. There is, finally, also an invalid
delegation of legislative powers to the officers mentioned
therein who are granted unlimited discretion in the
distribution of the properties arbitrarily taken.
Vicente De La Cruz vs Edgardo Paras
Subject Shall Be Expressed in the Title Police Power Not
Validly Exercise

De La Cruz et al were club & cabaret operators. They assail


the constitutionality of Ord. No. 84, Ser. of 1975 or the
Prohibition and Closure Ordinance of Bocaue, Bulacan. De
la Cruz averred that the said Ordinance violates their right to
engage in a lawful business for the said ordinance would
close out their business. That the hospitality girls they
employed are healthy and are not allowed to go out with
customers. Judge Paras however lifted the TRO he earlier
issued against Ord. 84 after due hearing declaring that Ord
84. is constitutional for it is pursuant to RA 938 which reads
AN ACT GRANTING MUNICIPAL OR CITY BOARDS
AND COUNCILS THE POWER TO REGULATE THE
ISSUE: Whether or not the law is valid.
ESTABLISHMENT, MAINTENANCE AND OPERATION
OF CERTAIN PLACES OF AMUSEMENT WITHIN
HELD: The SC ruled that the EO is not valid as it indeed
THEIR RESPECTIVE TERRITORIAL JURISDICTIONS.
violates due process. EO 626-A ctreated a presumption based Paras ruled that the prohibition is a valid exercise of police
on the judgment of the executive. The movement of carabaos power to promote general welfare. De la Cruz then appealed
from one area to the other does not mean a subsequent
citing that they were deprived of due process.
slaughter of the same would ensue. Ynot should be given to
defend himself and explain why the carabaos are being
ISSUE: Whether or not a municipal corporation, Bocaue,
transferred before they can be confiscated. The SC found
Bulacan can, prohibit the exercise of a lawful trade, the
that the challenged measure is an invalid exercise of the
operation of night clubs, and the pursuit of a lawful
police power because the method employed to conserve the occupation, such clubs employing hostesses pursuant to Ord
carabaos is not reasonably necessary to the purpose of the
84 which is further in pursuant to RA 938.
law and, worse, is unduly oppressive. Due process is
violated because the owner of the property confiscated is
HELD: The SC ruled against Paras. If night clubs were
denied the right to be heard in his defense and is
merely then regulated and not prohibited, certainly the

assailed ordinance would pass the test of validity. SC had


stressed reasonableness, consonant with the general powers
and purposes of municipal corporations, as well as
consistency with the laws or policy of the State. It cannot be
said that such a sweeping exercise of a lawmaking power by
Bocaue could qualify under the term reasonable. The
objective of fostering public morals, a worthy and desirable
end can be attained by a measure that does not encompass
too wide a field. Certainly the ordinance on its face is
characterized by overbreadth. The purpose sought to be
achieved could have been attained by reasonable restrictions
rather than by an absolute prohibition. Pursuant to the title of
the Ordinance, Bocaue should and can only regulate not
prohibit the business of cabarets.
El Banco ESPAOL-Filipino vs Vicente Palanca
Judicial Due Process Requisites

Engracio Palanca was indebted to El Banco and he had his


parcel of land as security to his debt. His debt amounted to
P218,294.10. His property is worth 75k more than what he
owe. Due to the failure of Engracio to make his payments, El
Banco executed an instrument to mortgage Engracios
property. Engracio however left for China and he never
returned til he died. Since Engracio is a non resident El
Banco has to notify Engracio about their intent to sue him by
means of publication using a newspaper. The lower court
further orderdd the clerk of court to furnish Engracio a copy
and that itd be sent to Amoy, China. The court eventually
granted El Banco petition to execute Engracios property. 7
years thereafter, Vicente surfaced on behalf of Engracio as
his administrator to petition for the annulment of the ruling.
Vicente averred that there had been no due process as
Engracio never received the summons.
ISSUE: Whether or not due process was not observed.
HELD: The SC ruled against Palanca. The SC ruled that the
requisites for judicial due process had been met. The
requisites are;

1. There must be an impartial court or tribunal clothed


with judicial power to hear and decide the matter
before it.

HELD: The SC ruled that the ruling of the SB is bereft of


merit as there was no strong showing of Imeldas guilt. The
SC further emphasized that Imelda was deprived of due
process by reason of Garchitorena not waiting for Amores
2. Jurisdiction must be lawfully acquired over the
manifestation. Such procedural flaws committed by
person of the defendant or over the property subject
respondent Sandiganbayan are fatal to the validity of its
of the proceedings.
decision convicting petitioner. Garchitorena had already
created the Special Division of five (5) justices in view of
3. The defendant must be given the opportunity to be
the lack of unanimity of the three (3) justices in the First
heard.
Division. At that stage, petitioner had a vested right to be
4. Judgment must be rendered only after lawful
heard by the five (5) justices, especially the new justices in
hearing.
the persons of Justices Amores and del Rosario who may
have a different view of the cases against her. At that point,
Presiding Justice Garchitorena and Justice Balajadia may
Imelda Marcos vs Sandiganbayan
change their mind and agree with the original opinion of
Due Process
Justice Atienza but the turnaround cannot deprive petitioner
of her vested right to the opinion of Justices Amores and del
Imelda was charged together with Jose Dans for Graft &
Corruption for a dubious transaction done in 1984 while they Rosario. It may be true that Justice del Rosario had already
expressed his opinion during an informal, unscheduled
were officers transacting business with the Light Railway
meeting in the unnamed restaurant but as aforestated, that
Transit. The case was raffled to the 1st Division of the
opinion is not the opinion contemplated by law. But what is
Sandiganbayan. The division was headed by Justice
more, petitioner was denied the opinion of Justice Amores
Garchitorena with J Balajadia and J Atienza as associate
justices. No decision was reached by the division by reason for before it could be given, Presiding Justice Garchitorena
dissolved the Special Division.
of Atienzas dissent in favor of Imeldas innocence.
Garchitorena then summoned a special division of the SB to
G.R. No. L-47276: Emma Delgado vs Court of Appeals
include JJ Amores and Cipriano as additional members.
Due Process
Amores then asked Garchitorena to be given 15 days to send
in his manifestation. On the date of Amores request,
Delgado together with 3 others were charged for estafa
Garchitorena received manifestation from J Balajadia stating
causing the frustration of one medical student. Delgado was
that he agrees with J Rosario who further agrees with J
assisted by one Atty. Yco. The said lawyer has filed for
Atienza. Garchitorena then issued a special order to
multiple postponement of trial and one time he failed to
immediately dissolve the special division and have the issue
appear in court by reason of him being allegedly sick. No
be raised to the SB en banc for it would already be pointless
medical certificate was furnished. The court was not
to wait for Amores manifestation granted that a majority has
impressed with such actuation and had considered the same
already decided on Imeldas favor. The SB en banc ruled
as Delgados waiver of her right to trial. The lower court
against Imelda.
convicted her and the others. She appealed before the CA
and the CA sustained the lower courts rule. Delgado later
ISSUE: Whether or not due process has been observed.
found out that Yco is not a member of the IBP.

ISSUE: Whether or not due process was observed.


HELD: The SC ruled in favor of Delgado. An accused
person is entitled to be represented by a member of the bar in
a criminal case filed against her before the Regional Trial
Court. Unless she is represented by a lawyer, there is great
danger that any defense presented in her behalf will be
inadequate considering the legal perquisites and skills
needed in the court proceedings. This would certainly be a
denial of due process.
G.R. No. 179907: Pedro Consulta vs People of the
Philippines
Due Process
Consulta is charged for stealing a gold necklace worth 3.5k
owned by a certain Silvestre. He was convicted by the lower
court. The court of appeals raised before the CA the issue
that he was not properly arraigned and that he was
represented by a non lawyer.
ISSUE: Whether or not Consulta was denied of due process.
HELD: The SC ruled that Consultas claim of being
misrepresented cannot be given due course. He was assisted
by two lawyers during the proceeding. In the earlier part, he
was assisted by one Atty. Jocelyn Reyes who seemed not to
be a lawyer. Granting that she indeed is not a lawyer, her
withdrawal from the case in the earlier part of the case has
cured the defect as he was subsequently assisted by a lawyer
coming from the PAO.
G.R. No. L-46272 June 13, 1986
PEOPLE OF THE PHILIPPINES, plaintiff-appellee,
vs.
ALBERTO OPIDA y QUIAMBAO and VIRGILIO
MARCELO, accused-appellants.

CRUZ, J.:

Reading the transcript, one gathers the impression that the


judge had allied himself with the prosecution to discredit at
the outset the credibility of the witnesses for the defense.

This is an automatic review of the Decision of the Circuit


Criminal Court, Seventh Judicial District, imposing the death
penalty upon Alberto Opida and Virgilio Marcelo for the
Opida is a police character, admittedly a member of the
crime of murder.
Commando gang and with a string of convictions for
robbery, theft and vagrancy. 6 It is worth noting that the
Unlike the victim in this case, who died from only one stab
judge took special interest in his tattoos, required him to
wound, the decision under review suffers from several fatal remove his shirt so they could be examined, and even
flaws, all equally deadly. It suffices to discuss only one of
described them in detail for the record. 7
them.
Besides belaboring Opida's criminal activities and his
Time and again this Court has declared that due process
tattoos, the judge asked him if he had "ever been convicted
requires no less than the cold neutrality of an impartial
at the National Mental Hospital with what else but malice
1
judge. Bolstering this requirement, we have added that the and suggested to him that his claim of manhandling by the
judge must not only be impartial but must also appear to be police was a lie because investigators leave no mark when
impartial, to give added assurance to the parties that his
they torture a suspect. 8 This was a point that could have
2
decision will be just. The parties are entitled to no less than been validly raised by the prosecution but certainly not by
this, as a minimum guaranty of due process. This guaranty
the court. The judge also made it of record that the witness
was not observed in this case.
was gnashing his teeth, was showing signs of hostility, that
he was uneasy and that he was restless. "Now, whom do you
On July 31, 1976, in Quezon City, several persons ganged up want to fool the judge asked, "the prosecutor, your lawyer, or
on Fabian Galvan, stoned and hit him with beer bottles until the court? 9
finally one of them stabbed him to death. The actual knifewielder was identified as Mario del Mundo. 3 Nonetheless,
In the hearing of September 22, 1976, the interrogation of
Alberto Opida and Virgilio Marcelo were charged with
Virgilio Marcelo, the other accused, was conducted almost
4
murder as conspirators and, after trial, sentenced to death.
wholly by the judge who started cross-examining the witness
even before the defense counsel could ask his first question,
The basis of their conviction by the trial court was the
and took over from the prosecution the task of impeaching
testimony of two prosecution witnesses, neither of whom
Marcelo's credibility. 10 The judge asked him about his drug
positively said that the accused were at the scene of the
addiction, his membership in the Commando gang, his
crime, their extrajudicial confessions, which were secured
tattoos, his parentage, his activities, his criminal record all
without the assistance of counsel, and corroboration of the
when he was supposed to be under direct examination by his
alleged conspiracy under the theory of interlocking
own lawyer. Defense counsel could hardly put in a word
5
confession.
edgewise because the judge kept interrupting to ask his own
questions. 11
What is striking about this case is the way the trial judge
conducted his interrogation of the two accused and their lone The questions were not clarificatory but adversary; and when
witness, Lilian Layug. It was hardly judicious and certainly they were not adversary, they were irrelevant, and sometimes
far from judicial, at times irrelevant, at Worst malicious.
also cruel. At one point, the judge drew from the witness the

statement that his mother was living with another man;


forthwith he suggested that the mother was unfaithful to his
father. 12 We deplore this sadistic treatment of the witness,
especially as, for all his supposed "toughness," he could not
answer back. We fail to see what possible connection the
mother's infidelity could have had, by any stretch of the
imagination, with the instant prosecution.
But the judge was to save the best or worst of his spite for
the third witness, Lilian Layug, a waitress in the restaurant
where the appellant Opida was working as a cook. Noting at
the outset that she spoke English, he wanted to know where
she had learned it and asked in ill-concealed insinuation if
she had worked in Angeles City or Olongapo or Sangley. 13
Because she was gesturing nervously, he asked, "Are you a
conductor? 14 Of the two accused, he asked her, "They are
very proud of belonging to the Commando gang to which the
witness answered, putting him in his place, "That I do not
know, Your Honor." 15
One cannot but note the mockery in the following questions
put by the judge to the witness, who was probably
wondering what the interrogation was all about
Court
Q You are a very good
friend of Alberto Opida?
A Yes, Your Honor.
Q You have known him for
years?
A One year only, Your
Honor.
Q He always feed you with
his favorite menu?

A Yes, Your Honor.


Q He is a very good cook?

Q That is precisely one of


the reasons why you also
admire him?

Q Whenever he cooks
adobo, he was singing?
A Sometimes, Your Honor.

A Yes, Your Honor.


Q Because what he could
cook, you could not cook?

A That is also a part, Your


Honor,

A I know also how to cook,


Your Honor.

Q Whenever you request


him to cook adobo for you,
he always accommodate
you?

Q Answer my question.

A Yes, Your Honor.

A Yes, Your Honor.

Q As a matter of fact, the


moment that he starts
cooking adobo, you could
smell it already?

Q Whenever you try to cook


what he cooked, you could
not imitate it, because he is
a good cook?
A Yes, Your Honor.
Q So, your admiration
developed because of his
cooking?
A Yes, Your Honor.
Q What favorite dish does
he cook that you like, as far
as you are concerned?
A Adobo, Your Honor.
Q Most often you request
him to cook adobo for you?
A Yes, Your Honor.

Q What kind of song?


A He is singing a song with
intended for Cora, Your
Honor.
Q And you were also
affected by it?
A No, Your Honor.
Q You mean to say, you are
not very fond of emotional
songs?

A Yes, Your Honor,


Q That starts your
admiration for him.

A I am not, because Cora is


not minding him, Your
Honor.

Q And in return you


reciprocate?

Q But sometimes he sings


in the absence of Cora
because, as you said, he is
cooking adobo for you?

A Yes, Your Honor.

A Yes, Your Honor.

Q What kind of
reciprocation do you give to
Alberto Opida, whenever
you admire his cooking of
adobo for you, cooking just
for you?

Q What does he sings (sic)


for you?

A Yes, Your Honor.

A None, Your Honor.

A He sings many songs,


Your Honor.
Q For example, give the
title

A Milagro, Your Honor.


Q He also sings Diyos
Lamang Ang Nakakaalam?

Because of the smell of


adobo and his songs and it
is an admiration. Therefore,
there is that tendency to
testify in his favor?

gang even as they flaunted their tattoos as a badge of


notoriety. 21 Nevertheless, they were entitled to be presumed
innocent until the contrary was proved and had a right not to
be held to answer for a criminal offense without due process
of law. 22

A Yes, Your Honor. 16

The judge disregarded these guarantees and was in fact all


too eager to convict the accused, who had manifestly earned
his enmity. When he said at the conclusion of the trial, "You
want me to dictate the decision now?" 23, he was betraying a
pre-judgment long before made and obviously waiting only
to be formalized.

A Sometimes, Your Honor.


Q He also sings Kapantay
ay Langit?
A Yes, Your Honor.
Q He also sings Sapagkat
Tayo'y Tao Lamang?

On direct examination, Opida challenged his extrajudicial


confession, claiming it had been obtained without
observance of the rights available under Article IV, Section
20 of the Constitution, particularly the right to counsel. 17
Parenthetically, the extrajudicial confession of Marcelo was
also made without assistance of counsel. 18 Opida also
testified, under questioning from his counsel, that he had
been repeatedly hit with a "dos por dos" by a police officer
while he was being investigated. 19

The scales of justice must hang equal and, in fact, should


even be tipped in favor of the accused because of the
A I did not hear, Your
constitutional presumption of innocence. Needless to stress,
Honor.
this right is available to every accused, whatever his present
circumstance and no matter how dark and repellent his past.
Q But, you said he also
We have consistently held that the rights guaranteed during a Despite their sinister connotations in our society, tattoos are
sings even in the absence of custodial investigation are not supposed to be merely
at best dubious adornments only and surely not under our
Cora?
communicated to the suspect, especially if he is unlettered,
laws indicia of criminality. Of bad taste perhaps, but not of
but must be painstakingly explained to him so he can
crime.
A Yes, Your Honor.
understand their nature and significance. Moreover,
manhandling of any sort will vitiate any extrajudicial
In any event, convictions are based not on the mere
Q You smell adobo while he confession that may be extracted from him and renders it
appearance of the accused but on his actual commission of
cooks and sings. So, you
inadmissible in evidence against him. 20
crime, to be ascertained with the pure objectivity of the true
developed admiration also?
judge who must uphold the law for all without favor or
Those principles were given mere lip service by the judge,
malice and always with justice.
A Little only, Your Honor.
who did not bother to look deeper into the validity of the
challenged confessions.
Accused-appellants Opida and Marcelo, who have been
Q One way or another you
imprisoned since 1976, have sent us separate letters pleading
have appreciated him, but
Given the obvious hostility of the judge toward the defense, for the resolution of their death sentences one way or the
the only thing, as you know, it was inevitable that all the protestations of the accused in
other once and for all. Considering the way they were tried,
he is related to Cora in the
this respect would be, as they in fact were, dismissed. And
we now declare that they should not be detained in jail a
same way?
once the confessions were admitted, it was easy enough to
minute longer. While this is not to say that the accused are
employ them as corroborating evidence of the claimed
not guilty, it does mean that, because their constitutional
A Yes, Your Honor.
conspiracy among the accused.
rights have been violated, their guilt, if it exists, has not been
established beyond reasonable doubt and so cannot be
Q That is why you are
The accused are admittedly notorious criminals who were
pronounced. Due process has stayed the uneven hand of the
testifying in his favor?
probably even proud of their membership in the Commando quick condemnor and must set the defendants free.

WHEREFORE, the conviction of Alberto Opida and Virgilio of the detention. 2 So accused persons deprived of the
Marcelo is reversed and they are hereby ordered released
constitutional right of speedy trial have been set free. 3 And
immediately. No costs.
likewise persons detained indefinitely without charges so
much so that the detention becomes punitive and not merely
SO ORDERED.
preventive in character are entitled to regain their freedom.
The spirit and letter of our Constitution negates as contrary
Abad Santos, Yap, Narvasa, Melencio-Herrera, Alampay,
to the basic precepts of human rights and freedom that a
Gutierrez, Jr. and Paras, JJ., concur.
person be detained indefinitely without any charges."
Feria and Fernan JJ., are on leave.

Separate Opinions

TEEHANKEE, C.J., concurring:


I concur. I wish to state that some of us are not persuaded at
all that the two herein accused should be held guilty of the
single stab wound inflicted on the victim in what appears to
have been a tumultuous affray. I hail the Court's ratio
decidendi that prescinding therefrom, the accused's guilt, if it
exists in reality, cannot be pronounced because of the
violation of their basic constitutional rights of due process
and of the constitutional provision outlawing uncounselled
confessions.

I had stressed in another case that the plain mandate of the


constitutional provision expressly adopted the exclusionary
rule as the only practical means of enforcing the
constitutional injunction against uncounselled confessions
obtained in violation of one's constitutional rights by
outlawing their admission in court. The outlawing of such
confessions thereby removed the incentive on the part of
military or police officers to disregard such basic
constitutional rights, in the same manner that the
exclusionary rule bars admission of illegally seized
evidence. 4
This fundamental rule that the court that rendered the
judgment or before whom the case is pending is ousted of
jurisdiction upon showing of deprivation of a basic
constitutional right was eroded during the past authoritarian
regime. I hail its vigorous restatement in the ponencia of Mr.
Justice Isagani A. Cruz.

People vs Mortera (Right Of The Accused To An


Impartial Trial)
In my dissenting opinion in the habeas corpus case of Dr.
FACTS: This is an appeal from the January 23, 2009
Aurora Parong, 1 wrote that "the Court stands as the
Decision of the Court of Appeals which affirmed with
guarantor of the constitutional and human rights of all
modification the Decision of the Regional Trial Court in
persons within its jurisdiction and must see to it that the
criminal case which found accused Benancio Mortera guilty
rights are respected and enforced. It is settled in this
beyond reasonable doubt of the crime of murder for the
jurisdiction that once a deprivation of a constitutional right is killing of one Robelyn Rojas.
Prosecution witness Ramil Gregorio testified that one
shown to exist, the court that rendered the judgment or
before whom the case is pending is ousted of jurisdiction and afternoon, he together with other men were drinking tuba.
They have just started drinking when Benancio Mortera, Jr.
habeas corpus is the appropriate remedy to assail the legality

arrived. He wanted to hit Alberto Rojas with a Nescafe glass.


Alberto Rojas ran away. Mortera said, "Sayang." He listened
while the group of Ramil Gregorio were singing
accompanied by a guitar. Jomer Diaz, brother-in-law of
Alberto Diaz, arrived. Mortera said, "Here comes another
Rojas." Gregorio and his companions told Jomer Diaz to run
away. Mortera hurled a stone at Diaz but the latter was not
hit. Mortera left but he said that he will return. After a few
minutes, Mortera came back. When Jomer Diaz ran, Robelyn
Rojas, brother of Alberto Rojas went to Jomer. Mortera met
Robelyn at a distance of about seven meters from the place
where the group were drinking. Mortera and Robelyn
discussed with each other and later shook hands. Robelyn
turned his face and Mortera suddenly stabbed Robelyn Rojas
at the back. After stabbing Robelyn, Mortera ran away.
Robelyn Rojas tried to chase Mortera but he was not able to
catch up but he fell down mortally wounded. He was brought
to the hospital by his brother but he was pronounced DOA at
the hospital. Jovel Veales who was drinking together with
Ramil Gregorio and others, corroborated Ramil Gregorio's
testimony.
Although the accused pleaded not guilty when arraigned,
during the trial, he admitted having stabbed the victim
whom he referred to as Tonying, but claimed self-defense.
By his account he passed by a corner and saw a group of
people drinking. They were Ramil Gregorio, Jonel Veales
and Tonying. Upon seeing him, Tonying ran away and called
his brother, Alberto Rojas. When the accused was about to
reach the main road, Alberto Rojas, Tonying and a certain
"Duk" (brother-in-law of Tonying) accosted him and asked
him for liquor money. When he refused, the three men got
angry. After telling them that he had to go, Tonying hit him
with a spray gun (for painting), causing him to fall down.
While he was in a supine position, Tonying attempted to hit
him again. It was at that point that he was able to get hold of
his knife and thrust it forward and hit someone. He did not
know who got stabbed. He then immediately fled.
On January 23, 2007, the RTC rendered judgment finding
the accused guilty of murder. In rejecting the claim of selfdefense, the trial court stated that it was not worthy of belief
as it was belied by the credible testimonies of the
prosecution witnesses.
The accused appealed to the CA raising the issues of denial
of due process of law and his right to an impartial trial. He
claimed that the trial court judge, Judge Jesus Carbon, was

hostile towards him and prejudged his guilt as could be


inferred from his "prosecutor-like" conduct. The accused
likewise reiterated his claim of self-defense.
In its decision, the CA affirmed the decision of the RTC with
modification as to the civil liabilities. The CA ruled that the
trial judge did not transgress the standard of "cold neutrality"
required of a magistrate and added that the questions he
propounded were "substantially clarificatory."
Still not satisfied, the accused now comes before the SC.
ISSUE: WON the accused were denied of his right to have
an impartial trial.
HELD: As correctly pointed out by the CA, although the trial
judge might have made improper remarks and comments, it
did not amount to a denial of his right to due process or his
right to an impartial trial. Upon perusal of the transcript as a
whole, it cannot be said that the remarks were reflective of
his partiality. Not only did the accused mislead the court by
initially invoking a negative defense only to claim otherwise
during trial, he was also not candid to his own lawyer, who
was kept in the dark as to his intended defense. The
invocation of Opida did not persuade the SC. In Opida, SC
did not fail to notice the "malicious," "sadistic" and
"adversarial" manner of questioning by the trial judge of the
accused therein, including their defense witness. In Opida,
the accused never admitted the commission of the crime, and
so the burden of proof remained with the prosecution.
G.R. No. 167139

February 25, 2010

SUSIE CHAN-TAN, Petitioner,


vs.
JESSE C. TAN, Respondent.
DECISION
CARPIO, J.:
The Case
This is a petition for review1 of (i) the 17 May 2004
Resolution2 amending the 30 March 2004 Decision3 and (ii)

the 15 February 2005 Resolution4 of the Regional Trial Court


of Quezon City, Branch 107, in Civil Case No. Q-01-45743.
In its 30 March 2004 Decision, the trial court declared the
marriage between petitioner Susie Chan-Tan and respondent
Jesse Tan void under Article 36 of the Family Code.
Incorporated as part of the decision was the 31 July 2003
Partial Judgment5 approving the Compromise Agreement6 of
the parties. In its 17 May 2004 Resolution, the trial court
granted to respondent custody of the children, ordered
petitioner to turn over to respondent documents and titles in
the latter's name, and allowed respondent to stay in the
family dwelling. In its 15 February 2005 Resolution, the trial
court denied petitioner's motion for reconsideration of the 28
December 2004 Resolution7 denying petitioner's motion to
dismiss and motion for reconsideration of the 12 October
2004 Resolution,8 which in turn denied for late filing
petitioner's motion for reconsideration of the 17 May 2004
resolution.
The Facts
Petitioner and respondent were married in June of 1989 at
Manila Cathedral in Intramuros, Manila.9 They were blessed
with two sons: Justin, who was born in Canada in 1990 and
Russel, who was born in the Philippines in 1993.10
In 2001, twelve years into the marriage, petitioner filed a
case for the annulment of the marriage under Article 36 of
the Family Code. The parties submitted to the court a
compromise agreement, which we quote in full:

2. Susie Tan hereby voluntarily agrees to exclusively


shoulder and pay out of her own funds/assets whatever is the
remaining balance or unpaid amounts on said lots mentioned
in paragraph 1 hereof directly with Megaworld Properties,
Inc., until the whole purchase or contract amounts are fully
paid.
Susie Tan is hereby authorized and empowered to directly
negotiate, transact, pay and deal with the seller/developer
Megaworld Properties, Inc., in connection with the Contract
to Sell marked as Annexes "A" and "B" hereof.
The property covered by CCT No. 3754 of the Registry of
Deeds of Quezon City and located at Unit O, Richmore
Town Homes 12-B Mariposa St., Quezon City shall be
placed in co-ownership under the name of Susie Tan (1/3),
Justin Tan (1/3) and Russel Tan (1/3) to the exclusion of
Jesse Tan.
The property covered by TCT No. 48137 of the Registry of
Deeds of Quezon City and located at View Master Town
Homes, 1387 Quezon Avenue, Quezon City shall be
exclusively owned by Jesse Tan to the exclusion of Susie
Tan.
The undivided interest in the Condominium Unit in Cityland
Shaw. Jesse Tan shall exclusively own blvd. to the exclusion
of Susie Tan.

The shares of stocks, bank accounts and other properties


presently under the respective names of Jesse Tan and Susie
Tan shall be exclusively owned by the spouse whose name
1. The herein parties mutually agreed that the two (2) lots
appears as the registered/account owner or holder in the
located at Corinthian Hills, Quezon City and more
particularly described in the Contract to Sell, marked in open corporate records/stock transfer books, passbooks and/or the
court as Exhibits "H" to "H-3" shall be considered as part of one in possession thereof, including the dividends/fruits
thereof, to the exclusion of the other spouse.
the presumptive legitimes of their two (2) minor children
namely, Justin Tan born on October 12, 1990 and Russel Tan
Otherwise stated, all shares, bank accounts and properties
born on November 28, 1993. Copies of the Contract to Sell
registered and under the name and/or in the possession of
are hereto attached as Annexes "A" and "B" and made
Jesse Tan shall be exclusively owned by him only and all
integral parts hereof.

shares, accounts and properties registered and/or in the


possession and under the name of Susie Tan shall be
exclusively owned by her only.
However, as to the family corporations of Susie Tan, Jesse
Tan shall execute any and all documents transferring the
shares of stocks registered in his name in favor of Susie Tan,
or Justin Tan/Russel Tan. A copy of the list of the corporation
owned by the family of Susie Tan is hereto attached as
Annex "C" and made an integral part hereof.
The parties shall voluntarily and without need of demand
turn over to the other spouse any and all original documents,
papers, titles, contracts registered in the name of the other
spouse that are in their respective possessions and/or
safekeeping.

During the summer vacation/semestral break or Christmas


vacation of the children, the parties shall discuss the proper
arrangement to be made regarding the stay of the children
with Jesse Tan.
Neither party shall put any obstacle in the way of the
maintenance of the love and affection between the children
and the other party, or in the way of a reasonable and proper
companionship between them, either by influencing the
children against the other, or otherwise; nor shall they do
anything to estrange any of them from the other.
The parties agreed to observe civility, courteousness and
politeness in dealing with each other and shall not insult,
malign or commit discourteous acts against each other and
shall endeavor to cause their other relatives to act similarly.

The husband shall always be notified of all school activities


of the children and shall see to it that he will exert his best
effort to attend the same.
5. During the birthdays of the two (2) minor children, the
parties shall as far as practicable have one celebration.
Provided that if the same is not possible, the Husband (Jesse
Tan) shall have the right to see and bring out the children for
at least four (4) hours during the day or the day immediately
following/or after the birthday, if said visit or birthday
coincides with the school day.

6. The existing Educational Plans of the two children shall


be used and utilized for their High School and College
education, in the event that the Educational Plans are
insufficient to cover their tuition, the Husband shall shoulder
3. Thereafter and upon approval of this Compromise
4. Likewise, the husband shall have the right to bring out and the tuition and other miscellaneous fees, costs of books and
Agreement by the Honorable Court, the existing property
see the children on the following additional dates, provided educational materials, uniform, school bags, shoes and
regime of the spouses shall be dissolved and shall now be
that the same will not impede or disrupt their academic
similar expenses like summer workshops which are taken in
governed by "Complete Separation of Property". Parties
schedule in Xavier School, the dates are as follows:
Xavier School, which will be paid directly by Jesse Tan to
expressly represent that there are no known creditors that
the children's school when the same fall due. Jesse Tan, if
a.
Birthday
of
Jesse
Tan
will be prejudiced by the present compromise agreement.
necessary, shall pay tutorial expenses, directly to the tutor
concerned.
b. Birthday of Grandfather and Grandmother, first
The parties shall have joint custody of their minor children.
cousins and uncles and aunties
However, the two (2) minor children shall stay with their
The husband further undertake to pay P10,000.00/monthly
mother, Susie Tan at 12-B Mariposa St., Quezon City.
support pendente lite to be deposited in the ATM Account of
c. Father's Day
SUSIE CHAN with account no. 3-189-53867-8 Boni Serrano
The husband, Jesse Tan, shall have the right to bring out the
Branch effective on the 15th of each month. In addition Jesse
d.
Death
Anniversaries
of
immediate
members
of
the
two (2) children every Sunday of each month from 8:00 AM
Tan undertakes to give directly to his two (2) sons every
family of Jesse Tan
to 9:00 PM. The minor children shall be returned to 12-B
Sunday, the amount needed and necessary for the purpose of
Mariposa Street, Quezon City on or before 9:00 PM of every
the daily meals of the two (2) children in school.
e.
During
the
Christmas
seasons/vacation
the
herein
Sunday of each month.
parties will agree on such dates as when the children 7. This Compromise Agreement is not against the law,
can stay with their father. Provided that if the
The husband shall also have the right to pick up the two (2)
customs, public policy, public order and good morals. Parties
children stay with their father on Christmas Day
minor children in school/or in the house every Thursday of
hereby voluntarily agree and bind themselves to execute and
from December 24th to December 25th until 1:00
each month. The husband shall ensure that the children be
sign any and all documents to give effect to this Compromise
PM
the
children
will
stay
with
their
mother
on
home by 8:00 PM of said Thursdays.
Agreement.11
December 31 until January 1, 1:00 PM, or vice
versa.

On 31 July 2003, the trial court issued a partial judgment 12


approving the compromise agreement. On 30 March 2004,
the trial court rendered a decision declaring the marriage
void under Article 36 of the Family Code on the ground of
mutual psychological incapacity of the parties. The trial
court incorporated in its decision the compromise agreement
of the parties on the issues of support, custody, visitation of
the children, and property relations.

children; and petitioner failed to turn over to respondent


documents and titles in the latter's name.1avvphi1

be issued vacating all prior orders and leaving the parties at


the status quo ante the filing of the suit.

Thus, the trial court, in its 17 May 2004 resolution, awarded


to respondent custody of the children, ordered petitioner to
turn over to respondent documents and titles in the latter's
name, and allowed respondent to stay in the family dwelling
in Mariposa, Quezon City.

Meanwhile, petitioner cancelled the offer to purchase the


Corinthian Hills Subdivision Lot No. 12, Block 2. She
authorized Megaworld Corp. to allocate the amount of
P11,992,968.32 so far paid on the said lot in the following
manner:

Petitioner filed on 28 June 2004 a motion for


reconsideration14 alleging denial of due process on account
of accident, mistake, or excusable negligence. She alleged
she was not able to present evidence because of the
negligence of her counsel and her own fear for her life and
the future of the children. She claimed she was forced to
leave the country, together with her children, due to the
alleged beating she received from respondent and the
pernicious effects of the latter's supposed gambling and
womanizing ways. She prayed for an increase in
respondent's monthly support obligation in the amount of
P150,000.

In its 28 December 2004 Resolution,18 the trial court denied


both the motion to dismiss and the motion for
reconsideration filed by petitioner. It held that the 30 March
2004 decision and the 17 May 2004 resolution had become
final and executory upon the lapse of the 15-day
reglementary period without any timely appeal having been
filed by either party.

(a) P3,656,250.04 shall be transferred to fully pay


the other lot in Corinthian Hills on Lot 11, Block 2;
(b) P7,783,297.56 shall be transferred to fully pay
the contract price in Unit 9H of the 8 Wack Wack
Road Condominium project; and
(c) P533,420.72 shall be forfeited in favor of
Megaworld Corp. to cover the marketing and
administrative costs of Corinthian Hills Subdivision
Lot 12, Block 2.13

Unconvinced, the trial court, in its 12 October 2004


Resolution,15 denied petitioner's motion for reconsideration,
which was filed beyond the 15-day reglementary period. It
also declared petitioner in contempt of court for noncompliance with the partial judgment and the 17 May 2004
Petitioner authorized Megaworld Corp. to offer Lot 12,
resolution. The trial court also denied petitioner's prayer for
Block 2 of Corinthian Hills to other interested buyers. It also increase in monthly support. The trial court reasoned that
appears from the records that petitioner left the country
since petitioner took it upon herself to enroll the children in
bringing the children with her.
another school without respondent's knowledge, she should
therefore defray the resulting increase in their expenses.
Respondent filed an omnibus motion seeking in the main
custody of the children. The evidence presented by
On 4 November 2004, petitioner filed a motion to dismiss 16
respondent established that petitioner brought the children
and a motion for reconsideration17 of the 12 October 2004
out of the country without his knowledge and without prior
Resolution. She claimed she was no longer interested in the
authority of the trial court; petitioner failed to pay the
suit. Petitioner stated that the circumstances in her life had
P8,000,000 remaining balance for the Megaworld property
led her to the conclusion that withdrawing the petition was
which, if forfeited would prejudice the interest of the
for the best interest of the children. She prayed that an order

Undeterred, petitioner filed a motion for reconsideration of


the 28 December 2004 resolution, which the trial court
denied in its 15 February 2005 resolution.19 The trial court
then issued a Certificate of Finality20 of the 30 March 2004
decision and the 17 May 2004 resolution.
The Trial Court's Rulings
The 30 March 2004 Decision21 declared the marriage
between the parties void under Article 36 of the Family Code
on the ground of mutual psychological incapacity. It
incorporated the 31 July 2003 Partial Judgment22 approving
the Compromise Agreement23 between the parties. The 17
May 2004 Resolution24 amended the earlier partial judgment
in granting to respondent custody of the children, ordering
petitioner to turn over to respondent documents and titles in
the latter's name, and allowing respondent to stay in the
family dwelling in Mariposa, Quezon City. The 15 February
2005 Resolution25 denied petitioner's motion for
reconsideration of the 28 December 2004 Resolution26
denying petitioner's motion to dismiss and motion for
reconsideration of the 12 October 2004 Resolution,27 which
in turn denied for late filing petitioner's motion for
reconsideration of the 17 May 2004 resolution.
The Issue
Petitioner raises the question of whether the 30 March 2004
decision and the 17 May 2004 resolution of the trial court

have attained finality despite the alleged denial of due


process.

We held that the decision annulling the marriage had already


become final and executory when the husband failed to
appeal during the reglementary period. The husband claimed
The Court's Ruling
that the decision of the trial court was null and void for
violation of his right to due process. He argued he was
The petition has no merit.
denied due process when, after failing to appear on two
scheduled hearings, the trial court deemed him to have
Petitioner contends she was denied due process when her
waived his right to present evidence and rendered judgment
counsel failed to file pleadings and appear at the hearings for based solely on the evidence presented by private
respondent's omnibus motion to amend the partial judgment respondent. We upheld the judgment of nullity of the
as regards the custody of the children and the properties in
marriage even if it was based solely on evidence presented
her possession. Petitioner claims the trial court issued the 17 by therein private respondent.
May 2004 resolution relying solely on the testimony of
respondent. Petitioner further claims the trial court erred in
We also ruled in Tuason that notice sent to the counsel of
applying to her motion to dismiss Section 7 of the Rule on
record is binding upon the client and the neglect or failure of
the Declaration of Absolute Nullity of Void Marriages and
the counsel to inform the client of an adverse judgment
Annulment of Voidable Marriages. Petitioner argues that if
resulting in the loss of the latter's right to appeal is not a
indeed the provision is applicable, the same is
ground for setting aside a judgment valid and regular on its
unconstitutional for setting an obstacle to the preservation of face.29
the family.
In the present case, the 30 March 2004 decision and the 17
Respondent maintains that the 30 March 2004 decision and
May 2004 resolution of the trial court had become final and
the 17 May 2004 resolution of the trial court are now final
executory upon the lapse of the reglementary period to
and executory and could no longer be reviewed, modified, or appeal.30 Petitioner's motion for reconsideration of the 17
vacated. Respondent alleges petitioner is making a mockery May 2004 resolution, which the trial court received on 28
of our justice system in disregarding our lawful processes.
June 2004, was clearly filed out of time. Applying the
Respondent stresses neither petitioner nor her counsel
doctrine laid down in Tuason, the alleged negligence of
appeared in court at the hearings on respondent's omnibus
counsel resulting in petitioner's loss of the right to appeal is
motion or on petitioner's motion to dismiss.
not a ground for vacating the trial court's judgments.
The issue raised in this petition has been settled in the case
of Tuason v. Court of Appeals.28 In Tuason, private
respondent therein filed a petition for the annulment of her
marriage on the ground of her husband's psychological
incapacity. There, the trial court rendered judgment declaring
the nullity of the marriage and awarding custody of the
children to private respondent therein. No timely appeal was
taken from the trial court's judgment.

Further, petitioner cannot claim that she was denied due


process. While she may have lost her right to present
evidence due to the supposed negligence of her counsel, she
cannot say she was denied her day in court. Records show
petitioner, through counsel, actively participated in the
proceedings below, filing motion after motion. Contrary to
petitioner's allegation of negligence of her counsel, we have
reason to believe the negligence in pursuing the case was on
petitioner's end, as may be gleaned from her counsel's
manifestation dated 3 May 2004:

Undersigned Counsel, who appeared for petitioner, in the


nullity proceedings, respectfully informs the Honorable
Court that she has not heard from petitioner since Holy
Week. Attempts to call petitioner have failed.
Undersigned counsel regrets therefore that she is unable to
respond in an intelligent manner to the Motion (Omnibus
Motion) filed by respondent.31
Clearly, despite her counsel's efforts to reach her, petitioner
showed utter disinterest in the hearings on respondent's
omnibus motion seeking, among others, custody of the
children. The trial judge was left with no other recourse but
to proceed with the hearings and rule on the motion based on
the evidence presented by respondent. Petitioner cannot now
come to this Court crying denial of due process.
As for the applicability to petitioner's motion to dismiss of
Section 7 of the Rule on the Declaration of Absolute Nullity
of Void Marriages and Annulment of Voidable Marriages,
petitioner is correct. Section 7 of the Rule on the Declaration
of Absolute Nullity of Void Marriages and Annulment of
Voidable Marriages provides:
SEC. 7. Motion to dismiss. - No motion to dismiss the
petition shall be allowed except on the ground of lack of
jurisdiction over the subject matter or over the parties;
provided, however, that any other ground that might warrant
a dismissal of the case may be raised as an affirmative
defense in an answer. (Emphasis supplied)
The clear intent of the provision is to allow the respondent to
ventilate all possible defenses in an answer, instead of a mere
motion to dismiss, so that judgment may be made on the
merits. In construing a statute, the purpose or object of the
law is an important factor to be considered. 32 Further, the
letter of the law admits of no other interpretation but that the
provision applies only to a respondent, not a petitioner. Only
a respondent in a petition for the declaration of absolute
nullity of void marriage or the annulment of voidable

marriage files an answer where any ground that may warrant


a dismissal may be raised as an affirmative defense pursuant
to the provision. The only logical conclusion is that Section
7 of the Rule does not apply to a motion to dismiss filed by
the party who initiated the petition for the declaration of
absolute nullity of void marriage or the annulment of
voidable marriage.
Since petitioner is not the respondent in the petition for the
annulment of the marriage, Section 7 of the Rule does not
apply to the motion to dismiss filed by her. Section 7 of the
Rule not being applicable, petitioner's claim that it is
unconstitutional for allegedly setting an obstacle to the
preservation of the family is without basis.
Section 1 of the Rule states that the Rules of Court applies
suppletorily to a petition for the declaration of absolute
nullity of void marriage or the annulment of voidable
marriage. In this connection, Rule 17 of the Rules of Court
allows dismissal of the action upon notice or upon motion of
the plaintiff, to wit:

party. The 30 March 2004 decision and the 17 May 2004


resolution may no longer be disturbed on account of the
belated motion to dismiss filed by petitioner. The trial court
was correct in denying petitioner's motion to dismiss.
Nothing is more settled in law than that when a judgment
becomes final and executory, it becomes immutable and
unalterable. The same may no longer be modified in any
respect, even if the modification is meant to correct what is
perceived to be an erroneous conclusion of fact or law.33 The
reason is grounded on the fundamental considerations of
public policy and sound practice that, at the risk of
occasional error, the judgments or orders of courts must be
final at some definite date fixed by law. Once a judgment has
become final and executory, the issues there should be laid to
rest.34
WHEREFORE, we DENY the petition for review. We
AFFIRM the (i) 17 May 2004 Resolution amending the 30
March 2004 Decision and (ii) the 15 February 2005
Resolution of the Regional Trial Court of Quezon City,
Branch 107, in Civil Case No. Q-01-45743.

of the Municipal Trial Court in Cities (MTCC), Branch 4,


Baguio City for denial of due process relative to Criminal
Case No. 118628.
Complainant avers: On November 9, 2006, the Philippine
National Police (PNP) Quezon City Police District (QCPD)
served her a warrant of arrest dated October 13, 2006, issued
by the MTCC Baguio City, Branch 4, presided by
respondent, relative to Criminal Case No. 118628 for alleged
violation of Batas Pambansa Blg. 22. It was only then that
she learned for the first time that a criminal case was filed
against her before the court. She was detained at the Quezon
City Hall Complex Police Office and had to post bail of
P1,000.00 before the Office of the Executive Judge of the
Regional Trial Court (RTC) of Quezon City for her
temporary release. Upon verification, she learned that
respondent issued on August 8, 2006 an Order directing her
to appear before the court on October 10, 2006 for
arraignment. It was sent by mail to PNP Quezon City for
service to her. However, she did not receive any copy of the
Order and up to the present has not seen the same; hence, she
was not able to attend her arraignment. She also found out
that there was no proof of service of the Order or any notice
to her of the arraignment. This notwithstanding, respondent
issued a warrant for her arrest. Complainant alleges that she
was deeply aggrieved and embarrassed by the issuance of the
warrant for her arrest despite the fact that she was never
notified of her arraignment. Complainant prayed that the
appropriate investigation be conducted as to the undue
issuance of a warrant for her arrest. 1

Section 1. Dismissal upon notice by plaintiff. - A complaint


may be dismissed by the plaintiff by filing a notice of
dismissal at any time before service of the answer or of a
motion for summary judgment. Upon such notice being
filed, the court shall issue an order confirming the dismissal.
xxx

Costs against petitioner.

Section 2. Dismissal upon motion of plaintiff. - Except as


provided in the preceding section, a complaint shall not be
dismissed at the plaintiff's instance save upon approval of the
court and upon such terms and conditions as the court deems
proper. x x x (Emphasis supplied)

NORYN S. TAN, Petitioner,


vs.
JUDGE MARIA CLARITA CASUGA-TABIN, Municipal In her Comment2 dated July 5, 2007, respondent answered:
Trial Court in Cities, Branch 4, Baguio City, Respondent.
She issued the warrant of arrest because when the case was
called for appearance, the complainant, as accused therein,
R E S O LUTI O N
failed to appear. Prior to the issuance of the warrant of arrest,
her staff sent by registered mail the court's Order dated
AUSTRIA-MARTINEZ, J.:
August 8, 2006 addressed to complainant "through the Chief
of Police, PNP, 1104, Quezon City" directing complainant to
Noryn S. Tan (complainant) filed a Complaint dated April 2, appear on October 10, 2006 at 8:30 a.m. for the arraignment
2007 against Judge Maria Clarita Casuga-Tabin (respondent) and preliminary conference in Criminal Case No. 118628, as

However, when petitioner filed the motion to dismiss on 4


November 2004, the 30 March 2004 decision and the 17
May 2004 resolution of the trial court had long become final
and executory upon the lapse of the 15-day reglementary
period without any timely appeal having been filed by either

SO ORDERED.
A.M. No. MTJ-09-1729
January 20, 2009
(Formerly OCA I.P.I. No. 07-1910-MTJ)

proven by Registry Receipt No. 0310. It is true that the


return on the court's Order dated August 8, 2006 had not yet
been made by the QC Police on or before October 10, 2006.
Nonetheless, she issued the warrant of arrest in good faith
and upon the following grounds: (a) under Sec. 3 of Rule
1313 of the Rules of Court, the court was entitled to presume
that on October 10, 2006, after the lapse of a little over two
months, official duty had been regularly performed and a
letter duly directed and mailed had been received in the
regular course of mail; and (b) Sec. 12 of the 1983 Rule on
Summary Procedure in Special Cases provides that bail may
be required where the accused does not reside in the place
where the violation of the law or ordinance was committed.
The warrant of arrest she issued was meant to implement this
provision, which was not repealed by the 1991 Revised Rule
on Summary Procedure, since complainant is a resident of
Quezon City and not of Baguio City. If her interpretation
was erroneous, she (respondent) believes that an
administrative sanction for such error would be harsh and
unsympathetic. She has nothing personal against
complainant and did not want to embarrass or humiliate her.
She issued the warrant in the honest belief that her act was in
compliance with the rules. She prays that the case against her
be dismissed and that a ruling on the interpretation of Secs.
10 & 12, of the 1983 Rule on Summary Procedure in Special
Cases, in relation to Sec. 16 of the 1991 Revised Rule on
Summary Procedure be made for the guidance of the bench
and bar.4
The OCA, in its agenda report dated September 28, 2007,
recommended that the case be dismissed for lack of merit. It
held: Prior to the filing of the information, a preliminary
investigation was conducted by the provincial prosecutor
resulting in the Resolution dated July 11, 2006
recommending the filing of the case; it was incredulous for
complainant to claim that she came to learn for the first time
of the filing of the criminal case when the warrant of arrest
was served on her; furthermore, there was already a
complete service of notice as contemplated in Sec. 10, Rule
135 of the Rules of Court; hence the requirement of notice

was fully satisfied by the service of the Order dated August


8, 2006 and the completion of the service thereof. 6
Adopting the recommendation of the OCA, the Court on
November 12, 2007 issued a Resolution dismissing the case
for lack of merit.7

In a Resolution dated April 16, 2008, the Court required


respondent to Comment on complainant's Motion for
Reconsideration.9

Complainant filed a Comment stating: Complainant's motion


did not raise any new issue or ground that would merit the
reconsideration of the Court's November 12, 2007
Complainant filed a Motion for Reconsideration dated
Resolution; complainant failed to rebut the presumption that
January 8, 2008 alleging: The issue in this case was not
she was notified of the scheduled arraignment; what
whether complainant was aware of the criminal complaint
complainant propounded was a mere self-serving denial that
against her, but whether the issuance of a warrant of arrest
she never received the subpoena intended for her; there was
against her despite the absence of notice should be
no explanation why she would be able to receive a warrant
administratively dealt with; complainant was never notified of arrest; which was coursed in the same manner as the
of the arraignment; thus, she was not able to attend the same; subpoena, in a little less than a month, but allegedly to
respondent admitted in her Comment that no return had yet
receive the subpoena in almost two months; if complainant's
been made on or before October 10, 2006, the date
assertion was to be believed, the effect would be to paralyze
respondent ordered the warrant to be issued; her explanation the operation of courts in the provinces that had to inevitably
of good faith was therefore unjustifiable; neither could
rely on the police resources of Metro Manila; arraignments
respondent invoke the presumption of regularity of
could not proceed and trials could not go on; it was
performance of official duty, since the complainant did not
reasonable to follow as a rule that once a pleading or any
actually receive any notice; respondent in an Order dated
other official document was received in the ordinary course
March 14, 2007 admitted that since she did not usually wear of sending them, it must be presumed that others of the same
eyeglasses during hearings, she thought that the
nature were also delivered to the named addressees; to
acknowledgment receipt at the back of the Order referred to believe otherwise would be to delay justice for those residing
the copy sent to complainant; later scrutiny, however,
outside Metro Manila.10
showed that it pertained to the one sent to the prosecutor's
office; Section 10, Rule 13 of the Rules of Court did not
The Court finds the Motion for Reconsideration to be
apply to the instant case; the Order was addressed and sent to impressed with merit.
PNP Quezon City; assuming that the Order was properly
Whenever a criminal case falls under the Summary
served on the PNP, it was not equivalent to a service on
complainant; there was no actual delivery of the Order to the Procedure, the general rule is that the court shall not order
the arrest of the accused, unless the accused fails to appear
complainant; hence, there was no personal service; neither
whenever required.11 This is clearly provided in Section 16
was it served by ordinary mail or by registered mail; thus,
of the 1991 Revised Rule on Summary Procedure which
the rule on completeness of service had not been satisfied;
states:
complainant was not aware of and therefore did not attend
the preliminary investigation of her case; no proof can be
Sec. 16. Arrest of accused. - The court shall not order the
shown that she was ever notified of the said preliminary
8
arrest of the accused except for failure to appear
investigation, much less of the filing of the same.
whenever required. Release of the person arrested shall

either be in bail or on recognizance by a responsible citizen


acceptable to the court. (Emphasis supplied)

Sections 10 and 12 of the 1983 Rules on Summary


Procedure in Special Cases (As Amended) state:

does not justify the warrant of arrest issued in this case.


Section 12 talks of instances when bails are required, one of
which is when the accused does not reside in the place where
In this case, respondent claims that the issuance of a warrant Sec. 10. Duty of the Court. - On the basis of the complaint of the violation of the law or ordinance was committed. It does
for the arrest of complainant was justified, since complainant information and the affidavits accompanying the same, the
not state, however, that a warrant of arrest shall immediately
failed to appear during the arraignment in spite of an order
court shall make a preliminary determination whether to
issue even without actual notice to the accused. Respondent's
requiring her to do so. Respondent admits, however, that a
dismiss the case outright for being patently without basis or interpretation ascribes to the rules those which were not
copy of the Order dated August 8, 2006, was sent to
merit, or to require further proceedings to be taken. In the
expressly stated therein and unduly expands their meaning.
complainant "through the Chief of Police, PNP, 1104,
latter case, the court may set the case for immediate
Quezon City."
arraignment of an accused under custody, and if he pleads
The Court also notes that in an Order dated March 14, 2007,
guilty, may render judgment forthwith. If he pleads not
a copy of which was attached by complainant to her Motion
While it is true that the Rules of Court provides for
guilty, and in all other cases, the court shall issue an order,
for Reconsideration, respondent admitted that:
presumptions, one of which is that official duty has been
accompanied by copies of all the affidavits submitted by the
regularly performed, such presumption should not be the
complainant, directing the defendant(s) to appear and submit As a point of clarification, during the hearing on October
sole basis of a magistrate in concluding that a person called his counter-affidavit and those of his witnesses at a specified 10, 2006, when the case was called and the accused failed
to court has failed to appear as required, which in turn
to appear, the Court verified from the staff if the Accused
date not later than ten (10) days from receipt thereof.
justifies the issuance of a warrant for her arrest, when such
was notified to which said staff answered in the
notice was not actually addressed to her residence but to the Failure on the part of the defendant to appear whenever affirmative, showing to the Court a copy of the Order dated
police in her city. So basic and fundamental is a person's
required, shall cause the issuance of a warrant for his
August 8, 2006, setting this case for Appearance of the
right to liberty that it should not be taken lightly or brushed arrest if the court shall find that a probable cause exists after Accused on October 10, 2006. At the back of the Order
aside with the presumption that the police through which the an examination in writing and under oath or affirmation of
was an attached Acknowledgment Receipt. A quick
notice had been sent, actually served the same on
the complainant and his witnesses. (Emphasis supplied)
glance of the said receipt, and without eyeglasses of the
complainant whose address was not even specified.
Presiding Judge, as she does not usually wear one during
xxxx
Court sessions, made this Court believed that indeed,
Respondent further admitted in her Comment dated July 5,
that was the Acknowledgment Receipt proving that the
Sec. 12. Bail not required; Exception. --- No bail shall be
2007 that when she proceeded with the arraignment on
Accused was served with a copy of the said Order.
October 10, 2006 as scheduled, no return had yet been made required except when a warrant of arrest is issued in
accordance with Section 10 hereon or where the accused (a) The attention of the Court was called upon receipt of the
by the Quezon City Police.12 Nevertheless, she issued the
warrant of arrest, arguing that she did so on the presumption is a recidivist; (b) is fugitive from justice; (c) is charged with Accused's Motion for Clarification and a closer look on
that regular duty had been performed, and that the Order had physical injuries; (d) does not reside in the place where the
the Acknowledgment Receipt shows that the same was
violation
of
the
law
or
ordinance
was
committed,
or
(e)
has
been received in the regular course of mail; and since Sec.
for the City Prosecutor's Office. x x x14 (Emphasis
no known residence.
12 of the 1983 Rules on Summary Procedure provides that
supplied)
bail may be required where the accused does not reside in
Section 12 of the 1983 Rules on Summary Procedure was
From this, it can be inferred that respondent issued the
the place where the violation of the law or ordinance was
not reproduced in the 1991 Revised Rules on Summary
warrant of arrest on the mistaken belief that complainant was
committed, the warrant of arrest she issued was justified
Procedure,
while
Section
10
was
revised
and
portions
thereof
actually notified of the arraignment. A closer scrutiny of the
since complainant is a resident of Quezon City and not of
reproduced in Sections 1213 and 16 of the 1991 Rules on
records however showed that the Acknowledgment Receipt
Baguio City.
Summary Procedure. Granting, arguendo, that Sections 10
pertained to the copy of the City Prosecutor's Office and not
and 12 of the 1983 Rules on Summary Procedure in Special that of complainant's.
The Court disagrees.
Cases were not repealed by the 1991 Revised Rules, still it

Whatever the real reasons behind respondent's issuance of


complainant's warrant of arrest -- whether from the mistaken
belief that complainant was actually notified, or the
presumption that the police had served a copy of the order on
complainant or that the rules allow immediate issuance of
warrants of arrests whenever the accused does not reside in
the locality where the crime was committed -- the fact is,
respondent failed to uphold the rules, for which she should
be held administratively liable.

SO ORDERED.

December 1985, CCC petitioned before RTC Bulacan to


enjoin DBP and the Sheriff of Bulacan from foreclosing its
Sec of Justice vs Judge Ralph Lantion
assets and praying further that its loan terms with DBP be
Due Process
restructured and that the interest rate terms in the promissory
note be declared null and void. A TRO was issued in favor of
CCC. In December 1986, PP 502 was issued transferring
nonperforming assets of the govt to Asset Privatization
Mark Jimenez was charged of multiple crimes ranging from Trust. One of those transferred was CCCs account. DBP
tax evasion to wire tapping to conspiracy to defraud the
filed a petition to dismiss the pending case as it CCC could
USA. Jimenez was then wanted in the US. The US
no longer deal with DBP but rather with APT. The trial court
government, pursuant to the RP-US extradition treaty
The Court has held that a judge commits grave abuse of
denied the petition and has instead allowed APT to join the
requested
to
have
Jimenez
be
extradited
there.
Jimenez
authority when she hastily issues a warrant of arrest against
proceeding pursuant to PP 502 as amended. To determine
requested
for
a
copy
of
the
complaint
against
him
as
well
as
the accused in violation of the summary procedure rule that
CCCs indebtedness to DBP/APT, the RTC designated JC
the extradition request by the USA. The DOJ sec refused to Laya (former BSP Gov and DepEd Sec) as chair of a fact
the accused should first be notified of the charges against
provide him copy thereof advising that it is still premature to finding commission. He was given 60 days to come up with
him and given the opportunity to file his counter-affidavits
15
give him so and that it is not a preliminary investigation
and countervailing evidence.
a report and he was given a lot of extensions thereafter. After
hence he is not entitled to receive such copies. Jimenez sued several months, he was able to come up with the report. The
the DOJ Sec and the lower court ruled in favor of Jimenez.
While judges may not always be subjected to disciplinary
parties then filed their reactions to the report and during the
action for every erroneous order or decision they render, that
trial they were given a chance to cross examine each others
ISSUE:
Whether
or
not
Jimenez
is
deprived
of
due
process.
relative immunity is not a license to be negligent, abusive
witnesses. After cross examination, they were ordered to
and arbitrary in their prerogatives. If judges wantonly misuse
submit their position papers as to their calculation of the
HELD: The SC affirmed the ruling of the lower court. The
the powers vested in them by law, there will not only be
amount of indebtedness. CCCs computation is at P43.6M,
case
against
Jimenez
refer
to
an
impending
threat
of
confusion in the administration of justice but also oppressive
the Commissioners computation is at P61.6M while
deprivation of ones property or property right. No less is
disregard of the basic requirements of due process. 16 While
DBP/APTs calculation is at P2.6B. In June 1992, 3 of
this true, but even more so in the case before us, involving as CCCs witnesses were scheduled to be cross examined by
there appears to be no malicious intent on the part of
respondent, such lack of intent, however, cannot completely it does the possible deprivation of liberty, which, based on
APTs counsel as DBPs counsel had already done so. APTs
free her from liability.17 When the law is sufficiently basic, a the hierarchy of constitutionally protected rights, is placed
counsel was not able to do so raising the issue that he just
18
second
only
to
life
itself
and
enjoys
precedence
over
judge owes it to her office to know and simply apply it.
took over the case and needs time to prepare. The cross
property, for while forfeited property can be returned or
examination was reset to August 24-26, 1992 but counsel for
replaced, the time spent in incarceration is irretrievable and
Considering that this is respondent's first administrative
APT failed to appear due to Dengue. The other counsel,
beyond
recompense.
infraction in her more than 8 years of service in the
Jaime Cruz, for DBP was likewise absent; hes also a
judiciary,19 which serves to mitigate her liability, the Court
witness. On Aug 25th, the RTC ordered that due to the
Development
of
the
Bank
of
the
Philippines
vs
Court
holds the imposition of a fine in the amount of P10,000.00 to
foregoing the case is deemed submitted for decision. APT
of Appeals
be proper in this case.20
filed for a motion for reconsideration. It was denied and the
Due Process Opportunity to be Heard
RTC ruled that the indebtedness to be paid by CCC is the
WHEREFORE, Judge Maria Clarita Casuga-Tabin,
calculation came up with by the Commissioner. APT
In 1968 and 1969, Continental Cement Corp. entered into a
Municipal Trial Court in Cities, Branch 4, Baguio City is
appealed before the CA averring that it was denied due
loan contract with DBP. In 1979, CCC entered into a MOA
hereby found guilty of abuse of authority for which she is
process when it was not allowed to cross examine the
with DBP restructuring its loans. In November 1985, DBP
fined in the sum of P10,000.00.
witnesses of CCC nor was it allowed to present further
filed for a foreclosure against the assets of CCC. In

witnesses. CCC averred that by the failure of APTs counsel


to appear APT has waived such right. The CA sustained the
RTCs decision.

when he had been afforded the opportunity to present his


side.

ISSUE: Whether or not MIWPIs right to due process has


been violated.

HELD: The SC ruled in favor of MIWPI. Generally


ISSUE: Whether or not APT was denied of due process.
accepted is the principle that no man shall be affected by any
proceeding to which he is a stranger, and strangers to a case
HELD: The SC sustained the CAs ruling. Long ingrained in
not bound by judgment rendered by the court. In the same
jurisprudence is the principle that there can be no denial of
manner an execution can be issued only against a party and
In 1973, license was issued to Milagros Matuguina to
due process where a party had the opportunity to participate
not against one who did not have his day in court. There is
operate logging businesses under her group Matuguina
in the proceedings but did not do so. The withdrawal of
no basis for the issuance of the Order of Execution against
Logging Enterprises. MIWPI was established in 1974 with 7
APTs previous counsel in the thick of the proceedings
the MIWPI. The same was issued without giving MIWPI an
stockholders. Milagros Matuguina became the majority
would be a reasonable ground to seek postponement of the
opportunity to defend itself and oppose the request of
stockholder later on. Milagros later petitioned to have MLE
hearing. However, such reason necessitates a duty, nay an
DAVENCOR for the issuance of a writ of execution against
be transferred to MIWPI. Pending approval of MLEs
obligation, on the part of the new counsel to prepare himself
it. In fact, it does not appear that MIWPI was at all furnished
petition, Davao Enterprises Corporation filed a complaint
for the next scheduled hearing. The excuse that it was due to
with a copy of DAVENCORs letter requesting for the
against MLE before the District Forester (Davao) alleging
the former counsels failure to turn over the records of the
Execution of the Ministers decision against it. MIWPI was
that MLE has encroached upon the area allotted for
case to APT, shows the negligence of the new counsel to
suddenly made liable upon the order of execution by the
DAVENCORs timber concession. The Investigating
actively recover the records of the case. Mere demands are
respondent Secretarys expedient conclusions that MLE and
Committee found MLE guilty as charged and had
not sufficient. Counsel should have taken adequate steps to
MIWPI are one and the same, apparently on the basis merely
recommended the Director to declare that MLE has done so.
fully protect the interest of his client, rather than pass the
of DAVENCORs letter requesting for the Order, and
MLE appealed the case to the Ministry of Natural Resources.
blame on the previous counsel.
without hearing or impleading MIWPI. Until the issuance of
During pendency, Milagrosa withdrew her shares from
the Order of execution, MIWPI was not included or
MIWPI. Later, MNR Minister Ernesto Maceda found MLE
The due process requirement is satisfied where the parties
mentioned in the proceedings as having any participation in
guilty as charged. Pursuant to the finding, DAVENCOR and
are given the opportunity to submit position papers, as in this
the encroachment in DAVENCORs timber concession. This
Philip Co requested Maceda to order MLE and/or MIWPI to
case. Both parties, CCC and DBP/APT, were given
action of the Minister disregards the most basic tenets of due
comply with the ruling to pay the value in pesos of 2352.04
opportunity to submit their respective position papers after
process and elementary fairness. The liberal atmosphere
m3 worth of timbers. The Minister then issued a writ of
the Commissioner rendered his report. Contained in their
which pervades the procedure in administrative proceedings
execution against MIWPI. MIWPI filed a petition for
position papers were their respective comments and
does not empower the presiding officer to make conclusions
prohibition before the Davao RTC. The RTC ruled in favor
objections to the said report. Furthermore, the parties were
of fact before hearing all the parties concerned. (1996 Oct
of MIWPI and has ordered to enjoin the Minister from
also given the chance to cross-examine the Commissioner
24)
pursuing the execution of the writ. DAVENCOR appealed
and his representative. They were likewise granted
and the CA reversed the ruling of the RTC. MIWPI averred
opportunity to cross-examine the witnesses of the other
PEOPLE VS CA, 262 SCRA 452
that it is not a party to the original case (as it was MLE that
party, however, like in APTs case, they were deemed to
was sued a separate entity). That the issuance of the order Evelio Javier vs COMELEC & Arturo Pacificador
have waived their right, as previously discussed.
of execution by the Minister has been made not only without
Due Process impartial and competent court
or in excess of his authority but that the same was issued
The essence of due process is that a party be afforded a
patently without any factual or legal basis, hence, a gross
Javier and Pacificador, a member of the KBL under Marcos,
reasonable opportunity to be heard and to support any
violation of MIWPIs constitutional rights under the due
were rivals to be members of the Batasan in May 1984 in
evidence he may have in support of his defense. What the
process clause.
Antique. During election, Javier complained of massive
law prohibits is absolute absence of the opportunity to be
terrorism, intimidation, duress, vote-buying, fraud,
heard, hence, a party cannot feign denial of due process
Matuguina Integrated Wood Products Inc. vs Court of
Appeals
Due Process Not Being Party to a Case

tampering and falsification of election returns under duress,


threat and intimidation, snatching of ballot boxes perpetrated
by the armed men of Pacificador. COMELEC just referred
the complaints to the AFP. On the same complaint, the 2nd
Division of the Commission on Elections directed the
provincial board of canvassers of Antique to proceed with
the canvass but to suspend the proclamation of the winning
candidate until further orders. On June 7, 1984, the same 2nd
Division ordered the board to immediately convene and to
proclaim the winner without prejudice to the outcome of the
case before the Commission. On certiorari before the SC, the
proclamation made by the board of canvassers was set aside
as premature, having been made before the lapse of the 5day period of appeal, which the Javier had seasonably made.
Javier pointed out that the irregularities of the election must
first be resolved before proclaiming a winner. Further,
Opinion, one of the Commissioners should inhibit himself as
he was a former law partner of Pacificador. Also, the
proclamation was made by only the 2nd Division but the
Constitute requires that it be proclaimed by the COMELEC
en banc. In Feb 1986, during pendency, Javier was gunned
down. The Solicitor General then moved to have the petition
close it being moot and academic by virtue of Javiers death.

Without such confidence, there would be no point in


invoking his action for the justice they expect.

as he was still unaware of him being in default. On April


27th, J Castro ruled in favor Tecson. On May 2nd Azul,
unaware that J Castro already decided the case appealed to
Due process is intended to insure that confidence by
remove his default status. On May 7th Azul received the
requiring compliance with what Justice Frankfurter calls the decision rendered by the court on Apr 27th (but on record the
rudiments of fair play. Fair play calls for equal justice. There date of receipt was May 5th). Azul filed a motion for new
cannot be equal justice where a suitor approaches a court
trial on June 6th. The lower court denied the same on the 20th
already committed to the other party and with a judgment
of the same month. On Aug 1st, Azul filed a notice of appeal
already made and waiting only to be formalized after the
it was denied on the 3rd but was reconsidered on the 7th hence
litigants shall have undergone the charade of a formal
Azul filed his record on appeal on the 21st and J Castro
hearing. Judicial (and also extrajudicial) proceedings are not approved it on the 27th but surprisingly upon motion of
orchestrated plays in which the parties are supposed to make Tecson on the 30th, J Castro set aside its earlier decisaion on
the motions and reach the denouement according to a
the 27th. Finally, J Castro denied the appeal on the 7th of
prepared script. There is no writer to foreordain the ending.
September.
The judge will reach his conclusions only after all the
evidence is in and all the arguments are filed, on the basis of ISSUE: Whether or not Azul has been denied due process.
the established facts and the pertinent law.
HELD: The SC agreed with the Azul that he was denied
Pedro Azul vs Judge Jose Castro & Rosalinda Tecson
due process. The constitutional provision on due process
Due Process Impartial and Competent Court
commands all who wield public authority, but most
peremptorily courts of justice, to strictly maintain standards
Azul owns and operates a construction shop. To finance it he of fundamental fairness and to insure that procedural
entered a loan agreement with Tecson in the amount of
safeguards essential to a fair trial are observed at all stages of
P391k. Tecson was only able to collect P141k thus leaving
a proceeding. It may be argued that when the Azuls counsel
about P250k as a balance. She filed a petition for collection asked for a fifteen (15) day extension from April 11, 1979 to
ISSUE: Whether or not there had been due process in the
of sum of money before the Rizal RTC and the case was
file his answer, it was imprudent and neglectful for him to
proclamation of Pacificador.
given to J Sarmiento. On 27 Mar 79, Azul received the copy assume that said first extension would be granted. However,
of the complaint. On 10 Apr 79, Azul filed a motion for a 15 the records show that Atty. Camaya personally went to the
HELD: The SC ruled in favor of Javier and has overruled
the Sol-Gens tenor. The SC has repeatedly and consistently day extension to file for responsive pleading. Azul was
session hall of the court with his motion for postponement
unaware that J Sarmiento retired and was temporarily
demanded the cold neutrality of an impartial judge as the
only to be informed that J Sarmiento had just retired but that
substituted
by
J
Aover
who
granted
the
extension
but
only
indispensable imperative of due process. To bolster that
his motion would be considered submitted for resolution.
for 5 days starting the next day. But Azul only received the
requirement, we have held that the judge must not only be
Since the sala was vacant and pairing judges in Quezon City
rd
notice granting such on the 23 of the same month way
impartial but must also appear to be impartial as an added
are literally swamped with their own heavy loads of cases,
th
passed
the
5
day
period.
On
the
17
of
April,
Tecson
already
assurance to the parties that his decision will be just. The
counsel may be excused for assuming that, at the very least,
litigants are entitled to no less than that. They should be sure filed a motion to dismiss averring that Azuls 5 day
he had the requested fifteen (15) days to file his responsive
th
extension
has
already
lapsed.
On
the
18
of
the
same
month,
that when their rights are violated they can go to a judge who
pleading. It is likewise inexplicable why J Aover, who had
shall give them justice. They must trust the judge, otherwise J Castro, the permanent judge to replace J Sarmiento took
not permanently taken over the sala vacated by the retired
they will not go to him at all. They must believe in his sense office and he ordered Azul to be in default due to the lapse of judge, should suddenly rule that only a five-day extension
the 5 day extension. J Castro proceeded with the reception of would be allowed. And to compound the Azuls problems,
of fairness, otherwise they will not seek his judgment.
evidence the next day and of course without Azuls evidence the order was sent by mail and received only twelve (12)

days later or after the five-day period. Before the much


publicized Project Mercury of the Bureau of Posts, a court
should have known that court orders requiring acts to be
done in a matter of days should not be sent by mail.
Meanwhile, the petitioner was declared in default. The
motion to declare defendant in default is dated April 17,
1979. No copy was furnished the petitioner. It was acted
upon on April 18, 1979, the very first day in office of J
Castro in Quezon City.

should exercise his discretion in a way that the peoples faith of due process of law. In legal contemplation, it is as if no
in the courts of justice is not impaired. . . .
judgment has been rendered at all.
The reminder is also apropos that next in importance to the
duty of rendering a righteous judgment is that of doing it in
such a manner as will beget no suspicion of the fairness and
integrity of the judge . . . .

Anita Lorenzana vs Polly Cayetano


Due Process Hearing

Lorenzana was renting a parcel of land from the Manila


Railroad Company (later from the Bureau of Lands). She
later purchased the land (San Lazaro Estate). She had the
Filemon David vs Judge Gregorio Aquilizan et al
Due Process Hearing
property be rented to tenants occupying stalls. Due to
nonpayment of rents, she filed 12 ejectment cases against her
Mayor Miguel Paderanga vs Judge Cesar Azura
David
has
a
large
parcel
of
land
in
Polomolok,
Cotabato.
He
tenant. On the other hand, Cayetano was an occupant of a
Due Process Hostility Between the Judge and the Parties
let Felomeno Jugar and Ricardo Jugar tend and caretake
parcel of land adjacent to that of Lorenzanas land. Cayetano
Inhibition
separate portions of his land in 1971. The land is estimated
was renting the same from the Bureau of Lands. The lower
to be yielding 60-70 cavans of corn cobs an dthe share
Paderanga was the mayor of Gingoog City, Misamis
court granted Lorenzanas ejectment cases. Lorenzana then
agreed upon is 50-50. In 1973, David withdrew the land
Oriental. He petitioned that J Azura inhibits himself from
secured a writ of execution to forcibly eject her tenants but
from the brothers and has not allowed them to go back there. she included to eject Cayetanos property. Cayetano was not
deciding on pending cases brought before him on the
The brothers prayed for reinstatement but David refused to
grounds that they have lost confidence in him, that he
a party to the ejectment cases so she prayed for the lower
entertained tax suits against the city and had issued TROs on do so. David denied that the borthers were his tenants. He
court that her property be not touched. The lower court
said
that
Ricardo
was
his
tractor
driver
before
but
he
the sales of properties when it is clearly provided for by law
denied Cayetanos petition. The CA, upon appeal, favored
resigned to take care of his dad and to work for DOLE.
(Sec 74 PD 464) that the remedy to stop auction is to pay
Cayetano. Lorenzana averred that Cayetano is now a party to
Fewlomeno on the other hand surrendered the portion of the the ejectment cases as she already brought herself to the
tax, that J Azura is bias, oppressive and is abusive in his
land he was tending to continue his faith healing. J Aquilizan Courts jurisdiction by virtue of her appeal.
power.
handled the case filed by the brothers against David and after
ISSUE: Whether or not J Azura should inhibit himself from three months he rendered a decision in favor of the brothers ISSUE: Whether or not Cayetanos right to due process has
the trial.
without any hearing. David averred he was denied due
been violated.
process. J Aquilizan admitted that there was indeed no
HELD: The SC ruled that Azura must. As decided in the
HELD: The SC ruled in favor of Cayetano and has affirmed
hearing conducted but he said the decision has already
Pimentel Case (21 SCRA 160), All the foregoing
the CA. It must be noted that respondent was not a party to
become final and executory as the period for appeal has
notwithstanding, this should be a good occasion as any to
any of the 12 ejectment cases wherein the writs of
already lapsed.
draw attention of all judges to appropriate guidelines in a
demolition had been issued; she did not make her appearance
situation where their capacity to try and decide fairly and
ISSUE: Whether or not David is entitled to an appeal.
in and during the pendency of these ejectment cases.
judiciously comes to the fore by way of challenge from any
Cayetano only went to court to protect her property from
HELD: The SC ruled in favor of David. A decision rendered demolition after the judgment in the ejectment cases had
one of the parties. A judge may not be legally prohibited
without a hearing is null and void and may be attacked
from sitting in a litigation But when suggestion is made of
become final and executory. Hence, with respect to the
directly or collaterally. The decision is null and void for want judgment in said ejectment cases, Cayetano remains a third
record that he might be induced to act in favor of one party
of due process. And it has been held that a final and
or with bias or prejudice against a litigant arising out of
person to such judgment, which does not bind her; nor can
executory
judgment
may
be
set
aside
with
a
view
to
the
circumstances reasonably capable of inciting such a state of
its writ of execution be informed against her since she was
renewal of the litigation when the judgment is void for lack not afforded her day in court in said ejectment cases.
mind, he should conduct a careful self-examination. He

Zambales Chromite Mining et al vs Court of Appeals


Due Process Administrative Due Process

HELD: The SC annulled the decision of Gozon calling it as


a mockery of justice. Gozon had acted with grave abuse of
discretion. In order that the review of the decision of a
ZCM filed an administrative case before the Director of
subordinate officer might not turn out to be a farce, the
Mines Gozon to have them be declared the rightful and prior reviewing officer must perforce be other than the officer
locators and possessors of 69 mining claims in Sta. Cruz,
whose decision is under review; otherwise, there could be no
Zambales. They are asserting their claim against the group of different view or there would be no real review of the case.
Martinez and Pabiloa. Gozon decided in favor of Martinez The decision of the reviewing officer would be a biased
et al. ZCM appealed the case before the Secretary of
view; inevitably, it would be the same view since being
Agriculture and Natural Resources. During pendency, Gozon human, he would not admit that he was mistaken in his first
was assigned as the Sec of Agri. And Natural Resources. He view of the case. The SC affirmed the 2nd decision of the CA.
did not inhibit himself from deciding on the appeal but he
instead affirmed his earlier decision when he was still the
FELICIDAD ANZALDO vs JACOBO C. CLAVE et al
director of mines. ZCM then appealed before the CFI of
Due Process Administrative Due Process
Zambales. The CFI affirmed the decision of Gozon. It held
that the disqualification of a judge to review his own
Dr Anzaldo , 55, had been working in the National Institute
decision or ruling (Sec. 1, Rule 137, Rules of Court) does
not apply to administrative bodies; that there is no provision of Science and Technology for 28 years. She was holding the
position Scientist Research Associate IV when she was
in the Mining Law, disqualifying the Secretary of
Agriculture and Natural Resources from deciding an appeal appointed as Science Research Supervisor II. Her
from a case which he had decided as Director of Mines; that appointment was approved by the CSC in 1978. The position
delicadeza is not a ground for disqualification; that the ZCM was previously held by Dr Kintanar who recommended Dr
Venzon to his position. Dr Venzon contested the position. Dr
did not seasonably seek to disqualify Gozon from deciding
their appeal, and that there was no evidence that Gozon acted Afable, the one who appointed Anzaldo, averred that
arbitrarily and with bias, prejudice, animosity or hostility to Anzaldos appointment was approved by the NIST
evaluation Committee which gave 88 points to Anzalado and
ZCM. ZCM appealed the case to the CA. The CA reversed
Gozons finding and declared that ZCM had the rights earlier 66 points to Venzon. The issue was elevated to the Office of
the president by Venzon. Clave was then the Presidential
attributed to Martinez et al by Gozon. Martinez et al
Executive Assistant. Pursuant to PD 807 or the Civil Service
appealed averring that the factual basis found by Gozon as
Director of Mines be given due weight. The CA reconsidered Decree, Clave referred the issue to the CSC. Clave was also
holding the chairmanship of the CSC. Clave issued Res 1178
after realizing that Gozon cannot affirm his own decision
appointing Venzon to the contested position. After the denial
and the CA remanded the case to the Minister of Natural
of her motion for the reconsideration of that resolution, or on
Resources. Now both parties appealed urging their own
January 5, 1980, Anzaldo appealed to the Office of the
contentions; ZCM wants the CAs earlier decision to be
President of the Philippines. Since Clave was holding the
reaffirmed while Martinez et al demanded that Gozons
office of PEA he just affirmed his decision as the CSC
finding be reinstated. The CA denied both petition.
chairman.
ISSUE: Whether or not Gozon can validly affirm his earlier
ISSUE: Whether or not there is due process in the case at
decision w/o disturbing due process.
bar.

HELD: The SC ruled in favor of Anzaldo. When PEA Clave


said in his decision that he was inclined to concur in the
recommendation of the Civil Service Commission, what he
meant was that he was concurring with Chairman Claves
recommendation: he was concurring with himself. It is
evident that Anzaldo was denied due process of law when
Presidential Executive Assistant Clave concurred with the
recommendation of (himself) Chairman Clave of the Civil
Service Commission. Due process of law means fundamental
fairness. It is not fair to Anzaldo that PEA Clave should
decide whether his own recommendation as Chairman of the
CSC, as to who between Anzaldo and Venzon should be
appointed Science Research Supervisor II, should be adopted
by the President of the Philippines.
Miguel Singson vs National Labor Relations Commission
& Philippine Airlines
Due Process Dismissal of Employees
Singson was an employee of PAL. On 7 Jun 1991, a
Japanese national alleged that Singson extorted money from
her ($200.00) by accusing her of having excess baggage; and
that to settle the issue she needs to pay said amount to him.
Singson was later investigated and the investigating
committee found him guilty. PAL then dismissed Singson
from employment. Singson then filed a case before NLRC
against PAL for illegal dismissal, attys fees and damages.
Labor Arbiter Raul Aquino ruled in favor of Singson as he
found PALs side insufficient to dismiss Singson. PAL
appealed to the NLRC. The 2nd Division, composed of
Calaycay, Rayala former Arbiter Raul Aquino, of the NLRC
took cognizance of the case. NLRC reversed the decision of
Aquino. Singson moved for reconsideration which was
denied by NLRC, this time only Calaycay & Rayala voted.
ISSUE: Whether or not Singson was denied of due process.
HELD: The SC ruled that Singson was denied due process.
The SC held that Singson was denied due process when
Aquino participated, as presiding commissioner of the 2nd

Division of the NLRC, in reviewing PALs appeal. He was


reviewing his own decision as a former labor arbiter. Under
Rule VII, Section 2 (b) of the New Rules of Procedure of the
NLRC, each Division shall consist of one member from the
public sector who shall act as the Presiding Commissioner
and one member each from the workers and employers
sectors, respectively. The composition of the Division
guarantees equal representation and impartiality among its
members. Thus, litigants are entitled to a review of three (3)
commissioners who are impartial right from the start of the
process of review. Commissioner Aquino can hardly be
considered impartial since he was the arbiter who decided
the case under review. He should have inhibited himself
from any participation in this case. The infirmity of the
resolution was not cured by the fact that the motion for
reconsideration of Singson was denied by two
commissioners and without the participation of Aquino. The
right of petitioner to an impartial review of his appeal starts
from the time he filed his appeal. He is not only entitled to
an impartial tribunal in the resolution of his motion for
reconsideration. Moreover, his right is to an impartial review
of three commissioners. The denial of Singsons right to an
impartial review of his appeal is not an innocuous error. It
negated his right to due process.

NOTES:
Requisites of Procedural Due Process in Administrative
Proceedings
(1) the right to a hearing, which includes the right to present
ones case and submit evidence in support thereof;
(2) the tribunal must consider the evidence presented;
(3) the decision must have something to support itself;
(4) the evidence must be substantial;

(5) the decision must be based on the evidence presented at


the hearing, or at least contained in the record and disclosed
to the parties affected;

Alonte was accused of raping JuvieLyn Punongbayan with


accomplice Buenaventura Concepcion. It was alleged that
Concepcion befriended Juvie and had later lured her into
Alonetes house who was then the mayor of Bian, Laguna.
(6) the tribunal or body or any of its judges must act on its
The case was brought before RTC Bian. The counsel and
own independent consideration of the law and facts of the
the prosecutor later moved for a change of venue due to
controversy, and not simply accept the views of a
alleged intimidation. While the change of venue was
subordinate;
pending, Juvie executed an affidavit of desistance. The
prosecutor continued on with the case and the change of
(7) the Board or body should, in all controversial questions, venue was done notwithstanding opposition from Alonte.
render its decision in such manner that the parties to the
The case was raffled to the Manila RTC under J Savellano.
proceeding can know the various issues involved, and the
Savellano later found probable cause and had ordered the
reason for the decision rendered.
arrest of Alonte and Concepcion. Thereafter, the prosecution
presented Juvie and had attested the voluntariness of her
desistance the same being due to media pressure and that
they would rather establish new life elsewhere. Case was
Administrative Due Process Includes
then submitted for decision and Savellano sentenced both
accused to reclusion perpetua. Savellano commented that
(a) the right to notice, be it actual or constructive, of the
Alonte waived his right to due process when he did not cross
institution of the proceedings that may affect a persons legal examine Juvie when clarificatory questions were raised
right;
about the details of the rape and on the voluntariness of her
desistance.
(b) reasonable opportunity to appear and defend his rights
and to introduce witnesses and relevant evidence in his
ISSUE: Whether or not Alonte has been denied criminal due
favor;
process.
(c) a tribunal so constituted as to give him reasonable
assurance of honesty and impartiality, and one of competent
jurisdiction; and
(d) a finding or decision by that tribunal supported by
substantial evidence presented at the hearing or at least
ascertained in the records or disclosed to the parties.
Mayor Bayani Alonte vs Judge Maximo Savellano, NBI
& People of the Philippines
Due Process in Criminal Proceedings Waiver of Right to
Due Process

HELD: The SC ruled that Savellano should inhibit himself


from further deciding on the case due to animosity between
him and the parties. There is no showing that Alonte waived
his right. The standard of waiver requires that it not only
must be voluntary, but must be knowing, intelligent, and
done with sufficient awareness of the relevant circumstances
and likely consequences. Mere silence of the holder of the
right should not be so construed as a waiver of right, and the
courts must indulge every reasonable presumption against
waiver. Savellano has not shown impartiality by repeatedly
not acting on numerous petitions filed by Alonte. The case is
remanded to the lower court for retrial and the decision
earlier promulgated is nullified.

its objections, for distribution to indigenous peoples and


cultural communities.
Read full text here.

(c) The parties may then respectively present rebutting


evidence only, unless the court, in furtherance of justice,
The Facts and the Case
permits them to present additional evidence bearing upon the
Petitioner Central Mindanao University (CMU) is a
main issue.
NOTES:
chartered educational institution owned and run by the
State.1 In 1958, the President issued Presidential
Due process in criminal proceedings
Proclamation 476, reserving 3,401 hectares of lands of the
(d) Upon admission of the evidence, the case shall be
public domain in Musuan, Bukidnon, as school site for
(a) that the court or tribunal trying the case is properly
deemed submitted for decision unless the court directs the
CMU. Eventually, CMU obtained title in its name over 3,080
clothed with judicial power to hear and determine the matter parties to argue orally or to submit memoranda.
hectares of those lands under Original Certificates of Title
before it;
(OCTs) 0-160, 0-161, and 0-162. Meanwhile, the
government distributed more than 300 hectares of the
(b) that jurisdiction is lawfully acquired by it over the person
remaining untitled lands to several tribes belonging to the
of the accused;
(e) However, when the accused admits the act or omission
areas cultural communities.
charged in the complaint or information but interposes a
(c) that the accused is given an opportunity to be heard; and lawful defense, the order of trial may be modified
Forty-five years later or on January 7, 2003 President Gloria
accordingly.
Macapagal-Arroyo issued Presidential Proclamation 310 that
(d) that judgment is rendered only upon lawful hearing.
takes 670 hectares from CMUs registered lands for
G.R. No. 184869
September 21, 2010
distribution to indigenous peoples and cultural communities
in Barangay Musuan, Maramag, Bukidnon.
CENTRAL MINDANAO UNIVERSITY, Represented by
Section 3, Rule 119, of the Rules of Court
Officer-In-Charge Dr. Rodrigo L. Malunhao, Petitioner,
On April 3, 2003, however, CMU filed a petition for
vs.
prohibition against respondents Executive Secretary,
THE HONORABLE EXECUTIVE SECRETARY, THE Secretary of the Department of Environment and Natural
HONORABLE SECRETARY OF THE DEPARTMENT Resources, Chairperson and Commissioner of the National
Sec. 3. Order of trial. The trial shall proceed in the
OF ENVIRONMENT AND NATURAL RESOURCES,
Commission on Indigenous Peoples (NCIP), and Lead
following order:
THE CHAIRPERSON AND COMMISSIONERS OF
Convenor of the National Anti-Poverty Commission
THE NATIONAL COMMISSION ON INDIGENOUS
(collectively, NCIP, et al) before the Regional Trial Court
PEOPLES, and THE LEAD CONVENOR OF THE
(RTC) of Malaybalay City (Branch 9), seeking to stop the
NATIONAL ANTI-POVERTY COMMISSION,
implementation of Presidential Proclamation 310 and have it
(a) The prosecution shall present evidence to prove the
Respondents.
declared unconstitutional.
charge and, in the proper case, the civil liability.
DECISION

The NCIP, et al moved to dismiss the case on the ground of


lack of jurisdiction of the Malaybalay RTC over the action,
ABAD, J.:
pointing out that since the act sought to be enjoined relates to
(b) The accused may present evidence to prove his defense,
an official act of the Executive Department done in Manila,
and damages, if any, arising from the issuance of any
This case concerns the constitutionality of a presidential
jurisdiction lies with the Manila RTC. The Malaybalay RTC
provisional remedy in the case.
proclamation that takes property from a state university, over denied the motion, however, and proceeded to hear CMUs

application for preliminary injunction. Meanwhile,


respondents NCIP, et al moved for partial reconsideration of
the RTCs order denying their motion to dismiss.
On October 27, 2003, after hearing the preliminary
injunction incident, the RTC issued a resolution granting
NCIP, et als motion for partial reconsideration and
dismissed CMUs action for lack of jurisdiction. Still, the
RTC ruled that Presidential Proclamation 310 was
constitutional, being a valid State act. The RTC said that the
ultimate owner of the lands is the State and that CMU
merely held the same in its behalf. CMU filed a motion for
reconsideration of the resolution but the RTC denied the
same on April 19, 2004. This prompted CMU to appeal the
RTCs dismissal order to the Court of Appeals (CA)
Mindanao Station.2

1. Whether or not the CA erred in not finding that


the RTC erred in dismissing its action for prohibition
against NCIP, et al for lack of jurisdiction and at the
same time ruling that Presidential Proclamation 310
is valid and constitutional;
2. Whether or not the CA correctly dismissed
CMUs appeal on the ground that it raised purely
questions of law that are proper for a petition for
review filed directly with this Court; and
3. Whether or not Presidential Proclamation 310 is
valid and constitutional.
The Courts Rulings

One. The RTC invoked two reasons for dismissing CMUs


action. The first is that jurisdiction over the action to declare
CMU raised two issues in its appeal: 1) whether or not the
RTC deprived it of its right to due process when it dismissed Presidential Proclamation 310 lies with the RTC of Manila,
not the RTC of Malaybalay City, given that such action
the action; and 2) whether or not Presidential Proclamation
relates to official acts of the Executive done in Manila. The
310 was constitutional.3
second reason, presumably made on the assumption that the
4
Malaybalay RTC had jurisdiction over the action,
In a March 14, 2008 decision, the CA dismissed CMUs
Presidential Proclamation 310 was valid and constitutional
appeal for lack of jurisdiction, ruling that CMUs recourse
since the State, as ultimate owner of the subject lands, has
should have been a petition for review on certiorari filed
directly with this Court, because it raised pure questions law the right to dispose of the same for some purpose other than
CMUs use.
bearing mainly on the constitutionality of Presidential
Proclamation 310. The CA added that whether the trial court
can decide the merits of the case based solely on the hearings There is nothing essentially wrong about a court holding on
of the motion to dismiss and the application for injunction is the one hand that it has no jurisdiction over a case, and on
the other, based on an assumption that it has jurisdiction,
also a pure question of law.
deciding the case on its merits, both with the same results,
CMU filed a motion for reconsideration of the CAs order of which is the dismissal of the action. At any rate, the issue of
dismissal but it denied the same, 5 prompting CMU to file the the propriety of the RTC using two incompatible reasons for
dismissing the action is academic. The CA from which the
present petition for review.
present petition was brought dismissed CMUs appeal on
The Issues Presented
some technical ground.
The case presents the following issues:

Two. Section 9(3) of the Judiciary Reorganization Act of


19806 vests in the CA appellate jurisdiction over the final

judgments or orders of the RTCs and quasi-judicial bodies.


But where an appeal from the RTC raises purely questions of
law, recourse should be by a petition for review on certiorari
filed directly with this Court. The question in this case is
whether or not CMUs appeal from the RTCs order of
dismissal raises purely questions of law.
As already stated, CMU raised two grounds for its appeal: 1)
the RTC deprived it of its right to due process when it
dismissed the action; and 2) Presidential Proclamation 310
was constitutional. Did these grounds raise factual issues that
are proper for the CA to hear and adjudicate?
Regarding the first reason, CMUs action was one for
injunction against the implementation of Presidential
Proclamation 310 that authorized the taking of lands from
the university. The fact that the President issued this
proclamation in Manila and that it was being enforced in
Malaybalay City where the lands were located were facts
that were not in issue. These were alleged in the complaint
and presumed to be true by the motion to dismiss.
Consequently, the CMUs remedy for assailing the
correctness of the dismissal, involving as it did a pure
question of law, indeed lies with this Court.
As to the second reason, the CMU claimed that the
Malaybalay RTC deprived it of its right to due process when
it dismissed the case based on the ground that Presidential
Proclamation 310, which it challenged, was constitutional.
CMU points out that the issue of the constitutionality of the
proclamation had not yet been properly raised and heard.
NCIP, et al had not yet filed an answer to join issue with
CMU on that score. What NCIP, et al filed was merely a
motion to dismiss on the ground of lack of jurisdiction of the
Malaybalay RTC over the injunction case. Whether the RTC
in fact prematurely decided the constitutionality of the
proclamation, resulting in the denial of CMUs right to be
heard on the same, is a factual issue that was proper for the
CA Mindanao Station to hear and ascertain from the parties.

Consequently, the CA erred in dismissing the action on the


ground that it raised pure questions of law.
Three. Since the main issue of the constitutionality of
Presidential Proclamation 310 has been raised and amply
argued before this Court, it would serve no useful purpose to
have the case remanded to the CA Mindanao Station or to
the Malaybalay RTC for further proceedings. Ultimately, the
issue of constitutionality of the Proclamation in question will
come to this Court however the courts below decide it.
Consequently, the Court should, to avoid delay and
multiplicity of suits, now resolve the same.

Penn State University and Illinois State University, started as


small land grant colleges, with meager funding to support
their ever increasing educational programs. They were given
extensive tracts of agricultural and forest lands to be
developed to support their numerous expanding activities in
the fields of agricultural technology and scientific research.
Funds for the support of the educational programs of land
grant colleges came from government appropriation, tuition
and other student fees, private endowments and gifts, and
earnings from miscellaneous sources. It was in this same
spirit that President Garcia issued Proclamation No. 476,
withdrawing from sale or settlement and reserving for the
Mindanao Agricultural College (forerunner of the CMU) a
land reservation of 3,080 hectares as its future campus. It
was set up in Bukidnon, in the hinterlands of Mindanao, in
order that it can have enough resources and wide open
spaces to grow as an agricultural educational institution, to
develop and train future farmers of Mindanao and help
attract settlers to that part of the country.

The key question lies in the character of the lands taken from
CMU. In CMU v. Department of Agrarian Reform
Adjudication Board (DARAB),7 the DARAB, a national
government agency charged with taking both privatelyowned and government-owned agricultural lands for
distribution to farmers-beneficiaries, ordered the segregation
for this purpose of 400 hectares of CMU lands. The Court
nullified the DARAB action considering the inalienable
xxxx
character of such lands, being part of the long term functions
of an autonomous agricultural educational institution. Said
The education of the youth and agrarian reform are
the Court:
admittedly among the highest priorities in the government
socio-economic programs. In this case, neither need give
The construction given by the DARAB to Section 10
way to the other. Certainly, there must still be vast tracts of
restricts the land area of the CMU to its present needs or to a agricultural land in Mindanao outside the CMU land
land area presently, actively exploited and utilized by the
reservation which can be made available to landless
university in carrying out its present educational program
peasants, assuming the claimants here, or some of them, can
with its present student population and academic facility qualify as CARP beneficiaries. To our mind, the taking of the
overlooking the very significant factor of growth of the
CMU land which had been segregated for educational
university in the years to come. By the nature of the CMU,
purposes for distribution to yet uncertain beneficiaries is a
which is a school established to promote agriculture and
gross misinterpretation of the authority and jurisdiction
industry, the need for a vast tract of agricultural land for
granted by law to the DARAB.
future programs of expansion is obvious. At the outset, the
CMU was conceived in the same manner as land grant
The decision in this case is of far-reaching significance as far
colleges in America, a type of educational institution which as it concerns state colleges and universities whose resources
blazed the trail for the development of vast tracts of
and research facilities may be gradually eroded by
unexplored and undeveloped agricultural lands in the Midmisconstruing the exemptions from the CARP. These state
West. What we now know as Michigan State University,
colleges and universities are the main vehicles for our

scientific and technological advancement in the field of


agriculture, so vital to the existence, growth and
development of this country.8
It did not matter that it was President Arroyo who, in this
case, attempted by proclamation to appropriate the lands for
distribution to indigenous peoples and cultural communities.
As already stated, the lands by their character have become
inalienable from the moment President Garcia dedicated
them for CMUs use in scientific and technological research
in the field of agriculture. They have ceased to be alienable
public lands.1avvphi1
Besides, when Congress enacted the Indigenous Peoples
Rights Act (IPRA) or Republic Act 83719 in 1997, it
provided in Section 56 that "property rights within the
ancestral domains already existing and/or vested" upon its
effectivity "shall be recognized and respected." In this case,
ownership over the subject lands had been vested in CMU as
early as 1958. Consequently, transferring the lands in 2003
to the indigenous peoples around the area is not in accord
with the IPRA.
Furthermore, the land registration court considered the
claims of several tribes belonging to the areas cultural
communities in the course of the proceedings for the titling
of the lands in CMUs name. Indeed, eventually, only 3,080
hectares were titled in CMUs name under OCTs 0-160, 0161 and 0-162. More than 300 hectares were acknowledged
to be in the possession of and subject to the claims of those
tribes.
WHEREFORE, the Court GRANTS the petition, SETS
ASIDE the March 14, 2008 decision and September 22,
2008 resolution of the Court of Appeals in CA-G.R. SP
85456, and DECLARES Presidential Proclamation 310 as
null and void for being contrary to law and public policy.
SO ORDERED.

Romeo Erece vs Lyn Macalingay et al


Due Process Administrative Bodies

Due process of law in administrative cases is not identical


with judicial process for a trial in court is not always
essential to due process. While a day in court is a matter of
Atty Erece was the Regional Director CHR Region 1.
right in judicial proceedings, it is otherwise in administrative
Macalingay et al were Ereces subordinates. Macalingay et al proceedings since they rest upon different principles. The
were complaining that Erece had continuously denied them due process clause guarantees no particular form of
from using the company vehicle. That Erece had been
procedure and its requirements are not technical. Thus, in
receiving his Representation and Transportation Allowance
certain proceedings of administrative character, the right to a
yet he prioritizes himself in the use of the vehicle. The issue notice or hearing are not essential to due process of law. The
reached the CSc proper which found Erece guilty as charged. constitutional requirement of due process is met by a fair
Erece contends that he was denied due process as he was not hearing before a regularly established administrative agency
afforded the right to cross-examine his accusers and their
or tribunal. It is not essential that hearings be had before the
witnesses. He stated that at his instance, in order to prevent
making of a determination if thereafter, there is available
delay in the disposition of the case, he was allowed to
trial and tribunal before which all objections and defenses to
present evidence first to support the allegations in his
the making of such determination may be raised and
Counter-Affidavit. After he rested his case, respondents did considered. One adequate hearing is all that due process
not present their evidence, but moved to submit their
requires. . . .
position paper and formal offer of evidence, which motion
was granted by the CSC over his objection. Macalingay et al The right to cross-examine is not an indispensable aspect
then submitted their Position Paper and Formal Offer of
of due process. Nor is an actual hearing always
Exhibits. Erece submits that although he was allowed to
essential. . . .
present evidence first, it should not be construed as a waiver
of his right to cross-examine the complainants. Although the Datu Pax Pakung Mangudadatu vs HRET &
order of presentation of evidence was not in conformity with Angelo Montilla
Due Process Administrative Bodies HRET Service of
the procedure, still Erece should not be deemed to have lost
Summons
his right to cross-examine his accusers and their witnesses.
This may be allowed only if he expressly waived said right.
ISSUE: Whether or not Erece had been denied due process.
HELD: The SC agrees with the CA that petitioner was not
denied due process when he failed to cross-examine the
complainants and their witnesses since he was given the
opportunity to be heard and present his evidence. In
administrative proceedings, the essence of due process is
simply the opportunity to explain ones side.
Judicial Due Process vs Administrative Due Process

Datu Pax and Montilla were rivals in the Congressional


elections in 2007. Datu Pax won the elections. Montilla
contested the results of the elections before the HRET. The
Secretary of HRET the issued the summons to Datu Pax to
his quite remote residence in Sultan Kudarat. Datu Pax was
required to file a reply within ten days from receipt. The
summons was received by a certain Aileen Baldenas. 43
days past and no answer was received from Datu Pax as he
was unaware of the summons. HRET then considered such
inaction as a general denial to the protest. Datu Pax later
learned about the protest against him and he coordinated
with his lawyers to appear on behalf of him and to present
the answer as well as to file counter protest. He alleged that

he does not know of a Baldenas nor was she a part of the


household. HRET denied his motion and had proceeded to
the recount as prayed for by Montilla.
ISSUE: Whether or not Datu Pax was denied due process by
reason that he did not receive the summons personally.
HELD: The SC ruled in favor of Datu Pax. The summons to
Datu Pax should not have been delivered via registered mail
as the same is susceptible to fraud. The HRET should have
mde use of its own servers to make sure that the summons is
personally received by Datu Pax. The 1997 Rules of Civil
Procedure (which is in one way or the other adopted by the
2004 HRET rules on summons) provides that:
SEC. 6. Service in person on defendant. Whenever
practicable, the summons shall be served handling a copy
thereof to the defendant in person, or, if he refuses to
receive and sign for it, by tendering it to him.
SEC. 7. Substituted service. If, for justifiable causes, the
defendant cannot be served within a reasonable time as
provided in the preceding section, service may be effected
(a) by leaving copies of the summons at the defendants
residence with some person of suitable age and discretion
then residing therein, or (b) by leaving copies at
defendants office or regular place of business with some
competent person in charge thereof.
Indeed, if in ordinary civil cases (which involve only private
and proprietary interests) personal service of summons is
preferred and service by registered mail is not allowed on
jurisdictional and due process grounds, with more reason
should election cases (which involve public interest and the
will of the electorate) strictly follow the hierarchy of modes
of service of summons under the Rules of Court.
G.R. No. 157784: Department of Education vs Godofredo
Cuanan
Due Process Certiorari Service of Summons

In1996, Cuanan, while being a school principal in San


Antonio, Nueva Ecija, wascharged for sexual harassment.
DECS Region III created an investigatingcommittee and the
latter found Cuanan guilty as charged. Regional
DirectorLabrador the forced resignation of Cuanan. In 2000,
then Sec Gonzales affirmedthe decision of Labrador and had
denied Cuanans Motion for Reconsideration.
CUananappealed to the CSC which reversed Gonzales
decision in Jan 2003 and CSCissued a copy of the resolution
to Cuanan and Dep Ed, however, it seems thatDep Ed was
not able to receive the copy as it requested a copy thereof
again.The next month, Cuanan requested to be reinstated
which was indorsed by theSuperintendent. In Mar 2003, now
Dep Ed Sec de Jesus received a copy of theresolution. In
April, de Jesus filed a motion for reconsideration
w/ofurnishing a copy to Cuanan. In July 2003, de Jesus filed
a supplemental motionfor reconsideration; no copy was
furnished to Cuanan again. In Oct 2004, CSCreversed its
decision and found Cuanan guilty of sexual harassment.
Cuananfiled a certiorari before the CA. He averred that the
CSC decision in Jan 2003had already become final and
executory. The CA reversed the decision of theCSC. DepEd
averred that the proper remedy should have been a petition
forreview.

Terms. - x x x (l) PARTY ADVERSELY AFFECTED refers


to therespondent against whom a decision in a disciplinary
case has been rendered or tothe disciplining authority in an
appeal from a decision exonerating the saidemployee.

clearly within the 15-day reglementary period for the filing


of apetition for review. Such move would be in accordance
with the liberal spiritpervading the Rules of Court and in the
interest of substantial justice.

The remedy of an aggrieved partyfrom a resolution


issued by the CSC is to file a petition for review
thereofunder Rule 43 of the Rules of Court within fifteen
days from notice of theresolution. Recourse to a petition for
certiorari under Rule 65 rendersthe petition dismissible for
being the wrong remedy. Nonetheless, there areexceptions to
this rule, to wit: (a) when public welfare and the
advancement ofpublic policy dictates; (b) when the broader
interest of justice so requires; (c)when the writs issued are
null and void; or (d) when the questioned orderamounts to an
oppressive exercise of judicial authority. As will be
shownforthwith, exception (c) applies to the present case.

Cuananundoubtedly was denied procedural due process. He


had no opportunity toparticipate in the proceedings for the
petition for review/ reconsiderationfiled by the DepEd, since
no copy of the pleadings filed by the DepEd wereserved
upon him or his counsel; nor was he even required by the
CSC to file hiscomments thereon. Considering that pleadings
filed by the DepEd were not servedupon Cuanan, they may
be treated as mere scraps of paper which should not
havemerited the attention or consideration of the CSC.

Furthermore,while a motion for reconsideration is a


condition precedent to the filing of apetition for certiorari,
immediate recourse to the extraordinary remedyof certiorari
is warranted where the order is a patent nullity, as wherethe
court a quo has no jurisdiction; where petitioner was
deprived of dueprocess and there is extreme urgency for
relief; where the proceedings in thelower court are a nullity
ISSUE: Whether or not there is dueprocess in the case at bar. for lack of due process; where the proceeding was exparte or
one in which the petitioner had no opportunity to object.
These exceptions find application toCuanan's petition for
certiorari in the CA.
HELD: The SC affirmed the ruling ofthe CA. It noted that
DepEd has the power to file a motion for reconsiderationin
the case at bar. The SC maintainedthat the disciplining
At anyrate, Cuanan's petition for certiorari before the CA
authority qualifies as a party adversely affected by
could be treated asa petition for review, the petition having
thejudgment, who can file an appeal of a judgment of
been filed on November 22, 2004, orthirteen (13) days from
exoneration in anadministrative case. CSC Resolution
receipt on November 9, 2004 of CSC Resolution No.041147,
021600 provides: Section 2. Coverage andDefinition of

CERTIORARI, defined;

When anytribunal, board, or officer exercising judicial


functions has acted without orin excess of its or his
jurisdiction, or with grave abuse of discretion, andthere is no
appeal, nor any plain, speedy, and adequate remedy in the
ordinarycourse of law, a person aggrieved thereby may file a
verified petition forcertiorari in the proper court alleging the
facts with certainty and prayingthat judgment be rendered
annulling or modifying the proceedings, as the lawrequires,
of such tribunal, board or officer.
Ang Tibay vs Court of Industrial Relations
Due Process Admin Bodies CIR
TeodoroToribio owns and operates Ang Tibay a leather
company which supplies the Philippine Army. Due to alleged
shortage of leather, Toribio caused the lay off of members of
National Labor Union Inc. NLU averred that Toribios act is

not valid as it is not within the CBA. That there are two labor
unions in Ang Tibay; NLU and National Workers
Brotherhood. That NWB is dominated by Toribio hence he
favors it over NLU. That NLU wishes for a new trial as they
were able to come up with new evidence/documents that
they were not able to obtain before as they were inaccessible
and they were not able to present it before in the CIR.
ISSUE: Whether or not there has been a due process of law.
HELD: The SC ruled that there should be a new trial in
favor of NLU. The SC ruled that all administrative bodies
cannot ignore or disregard the fundamental and essential
requirements of due process. They are;
(1) The right to a hearing which includes the right of the
party interested or affected to present his own case and
submit evidence in support thereof.

(6) The Court of Industrial Relations or any of its judges,


therefore, must act on its or his own independent
consideration of the law and facts of the controversy, and not
simply accept the views of a subordinate in arriving at a
decision.
(7) The Court of Industrial Relations should, in all
controversial questions, render its decision in such a manner
that the parties to the proceeding can know the vario issues
involved, and the reasons for the decisions rendered. The
performance of this duty is inseparable from the authority
conferred upon it.
American Tobacco Company et al vs Director of Patents
Due Process

ATC et al filed before the Philippine Patent Office


concerning the use of trade mark and trade name. ATC et al
challenged the validity of Rule 168 of the Revised Rules of
Practice before the Philippine Patent Office in Trademark
(2) Not only must the party be given an opportunity to
present his case and to adduce evidence tending to establish Cases as amended, authorizing the Director of Patents to
the rights which he asserts but the tribunal must consider the designate any ranking official of said office to hear inter
partes proceedings. Said Rule likewise provides that all
evidence presented.
judgments determining the merits of the case shall be
personally and directly prepared by the Director and signed
(3) While the duty to deliberate does not impose the
by him. These proceedings refer to the hearing of
obligation to decide right, it does imply a necessity which
opposition to the registration of a mark or trade name,
cannot be disregarded, namely, that of having something to
interference proceeding instituted for the purpose of
support its decision. A decision with absolutely nothing to
determining the question of priority of adoption and use of a
support it is a nullity, a place when directly attached.
trade-mark, trade name or service-mark, and cancellation of
(4) Not only must there be some evidence to support a
registration of a trade-mark or trade name pending at the
finding or conclusion but the evidence must be substantial. Patent Office. Petitioners filed their objections to the
Substantial evidence is more than a mere scintilla It means
authority of the hearing officers to hear their cases, alleging
such relevant evidence as a reasonable mind might accept as that the amendment of the Rule is illegal and void because
adequate to support a conclusion.
under the law the Director must personally hear and decide
inter partes case. Said objections were overruled by the
(5) The decision must be rendered on the evidence
Director of Patents, hence, the present petition for
presented at the hearing, or at least contained in the record
mandamus, to compel the Director of Patents to personally
and disclosed to the parties affected.
hear the cases of petitioners, in lieu of the hearing officers.

ISSUE: Whether or not the hearing done by hearing officers


are within due process.
HELD: The SC ruled that the power to decide resides solely
in the administrative agency vested by law, this does not
preclude a delegation of the power to hold a hearing on the
basis of which the decision of the administrative agency will
be made. The rule that requires an administrative officer to
exercise his own judgment and discretion does not preclude
him from utilizing, as a matter of practical administrative
procedure, the aid of subordinates to investigate and report to
him the facts, on the basis of which the officer makes his
decisions. It is sufficient that the judgment and discretion
finally exercised are those of the officer authorized by law.
Neither does due process of law nor the requirements of fair
hearing require that the actual taking of testimony be before
the same officer who will make the decision in the case. As
long as a party is not deprived of his right to present his own
case and submit evidence in support thereof, and the decision
is supported by the evidence in the record, there is no
question that the requirements of due process and fair trial
are fully met. In short, there is no abnegation of
responsibility on the part of the officer concerned as the
actual decision remains with and is made by said officer. It
is, however, required that to give the substance of a hearing,
which is for the purpose of making determinations upon
evidence the officer who makes the determinations must
consider and appraise the evidence which justifies them.
G.R. No. 78763: Manila Electric Company vs National
Labor Relations Commission & Apolinario Signo
Due Process

Signowas employed in Meralco as supervisor-leadman since


Jan 1963.In 1981, hesupervised the installation of electricity
in de Laras house in Antipolo. DeLaras house was not yet
within the required 30-meter distance from the
Meralcofacility hence he is not yet within the service scope

of Meralco. As aworkaround, Signo had it be declared that a


certain sarisari store nearer thefacility be declared as de
Laras so as to facilitate the installation.Evertything would
have been smooth thereafter but due to fault of the
PowerSales Division of Meralco, de Lara was not billed for a
year. Investigation wasconducted and Meralco found out the
irregularity in Signos work on de Laraselectricity
installation. Signo was dismissed on May 18, 1983. Signo
filed acase for illegal dismissal and for backwages. The
Lanor Arbiter ruled thatthough there is a breach of trust in
the actuations of Signo dismissal is aharsh penalty as Signo
has been employed for more than 20 years by Meralco
andhas been commended twice before for honesty. The
NLRC affirmed the LaborArbiter. Meralco appealed.

ISSUE: Whether or not there has beendue process in the


dismissal of Signo.

Diosdado Guzman vs. National University


Facts:
Petitioners
Diosdado
Guzman,
Ulysses
Urbiztondo and Ariel Ramacula, students of respondent
National University, have come to this Court to seek
relief from what they describe as their school's
"continued and persistent refusal to allow them to
enrol." In their petition "for extraordinary legal and
equitable remedies with prayer for preliminary
mandatory injunction" dated August 7, 1984, they
alleged that they were denied due to the fact that they
were active participation in peaceful mass actions within
the premises of the University.
The respondents on the other hand claimed that
the petitioners failure to enroll for the first semester of
the school year 1984-1985 is due to their own fault and
not because of their alleged exercise of their
constitutional and human rights. That as regards to
Guzman, his academic showing was poor due to his
activities in leading boycotts of classes. That Guzman is
facing criminal charges for malicious mischief before the
Metropolitan Trial Court of Manila in connection with
the destruction of properties of respondent University.
The petitioners have failures in their records, and are
not of good scholastic standing.

There are withal minimum standards which must


be met to satisfy the demands of procedural due process;
and these are, that
(1) the students must be informed in writing of the
nature and cause of any accusation against them;
(2) they shag have the right to answer the charges against
them, with the assistance of counsel, if desired;
(3) they shall be informed of the evidence against
them;
(4) they shall have the right to adduce evidence in
their own behalf; and
(5) the evidence must be duly considered by the
investigating committee or official designated by the
school authorities to hear and decide the case.
THE PETITION WAS GRANTED AND THE
RESPONDENTS ARE DIRECTED TO ALLOW THE
PETITIONERS
TO
RE-ENROLL
WITHOUT
PREJUDICE
TO
ANY
DISCIPLINARY
PROCEEDINGS.
G.R. No. L-58610 September 30, 1982

BABELO BERINA, MARILOU ELAGDON, ERNESTO


ROBERTO and JESUS SORIAO, petitioners,
vs.
HELD: The SC sustained the decisionof the NLRC. WellPHILIPPINE MARITIME INSTITUTE, TOMAS
Held:
established is the principle that findings of
Immediately apparent from a reading of CLOMA and JAIME CLOMA, respondents.
administrativeagencies which have acquired expertise
respondents' comment and memorandum is the fact that
because their jurisdiction is confinedto specific matters are
generally accorded not only respect but even finality.Judicial they had never conducted proceedings of any sort to
determine whether or not petitioners-students had indeed
review by this Court on labor cases does not go so far as to
led or participated "in activities within the university ABAD SANTOS, J.:
evaluatethe sufficiency of the evidence upon which the
premises, conducted without prior permit from school
proper labor officer or officebased his or its determination
authorities, that disturbed or disrupted classes therein" 3
but is limited to issues of jurisdiction orgrave abuse of
or perpetrated acts of "vandalism, coercion and Babelo Beria, Marilou Elagdon, Ernesto Roberto and
intimidation, slander, noise barrage and other acts Jesus Soriao are students of the Philippine Maritime
discretion.Notwithstanding the existence of a valid cause
showing disdain for and defiance of University Institute, PMI for short. In their petition which is styled
fordismissal, such as breach of trust by an employee,
authority." 4 Parenthetically, the pendency of a civil case FOR EXTRAORDINARY AND EQUITABLE REMEDY
nevertheless, dismissalshould not be imposed, as it is too
for damages and a criminal case for malicious mischief WITH PRELIMINARY INJUNCTION, they claim that
severe a penalty if the latter has beenemployed for a
against petitioner Guzman, cannot, without more, PMI, five weeks after school had started, posted
considerable length of time in the service of his employer.
furnish sufficient warrant for his expulsion or debarment
sometime in August, 1981, a notice that there would be a
Reinstatementof respondent Signo is proper in the instant
from re-enrollment. Also apparent is the omission of
respondents to cite this Court to any duly published rule 15% increase in tuition fees retroactive to the start of the
case, but without the award ofbackwages, considering the
of theirs by which students may be expelled or refused current semester; that the students met and took positive
good faith of the employer in dismissing therespondent.
steps in respect of the problem; that their representatives
re-enrollment for poor scholastic standing.

held dialogues with the school administration; "that, in


reaction to these legitimate student activities and without
compliance with due process respondents commencing on
October 15, 1981 issued expulsion orders against Jesus
Soriao, Ernesto Roberto, and Babelo Berina and an
indefinite suspension against Marilou Elagdon;" that the
penalties were imposed without due process and had the
effect of negating the petitioners' right to free speech,
peaceful assembly and petition for redress of grievances.
The petitioners pray that the expulsion and suspension
orders be annulled and that while the case is pending
resolution they be restored to their status as students of
the PMI,
On November 10, 1981, We required PMI and its officers
who were included as respondents to comment on the
petition. We also issued a temporary restraining order
commanding the respondents to refrain from carrying
out the expulsion and suspension orders.
PMI filed its comment as required where it said that the
15% increase in tuition fee had been authorized by the
Ministry of Education and Culture; and denied that the
action taken against the petitioners was in response to
their activities in connection with the tuition fee increase.
The comment also advances the arguments that this
Court lacks jurisdiction to entertain the petition because
it involves "matters that are well within the competence
and jurisdiction of the lower courts to pass upon, as even
more serious matters and cases of greater consequences
are normally brought before them at the first instance
prior to any appeal to the Supreme Court, and there are
no valid and impelling excuses to warrant a direct
recourse to the Highest Tribunal in the judicial
hierarchy."

or not the petitioners were denied by the respondents


their constitutional rights to due process, free speech,
peaceful assembly and petition to redress of grievances.
Treating the petition as having been filed under Rule 65
of the Rules of Court as the petitioners assert, We have
no doubt that there is no absence of jurisdiction.
Typical of the expulsion orders is that which was issued
to petitioner Jesus Soriao on October 15, 1981, which has
been marked as Annex A of the petition and which reads
as follows:

Please be informed that C/miss


ELAGDON, Marilou is hereby suspended
from her classes for conduct unbecoming
of a Cadetee as against the rules and
regulation of the School.
Let the above-named student see the
undersigned and in the meantime she
remained suspended until clearance is
given by this office.
For your guidance.

For conduct unbecoming as a Cadet, you


are hereby dropped from the roll of
students of the School.

The comment does not positively assert that in imposing


the expulsion and suspension orders there was
observance of due process which simply means that the
That your actuations and behavior as
petitioners should have been given an opportunity to
reported and seen leave no other recourse defend themselves. It was only after the petitioners had
hence this action.
said in their reply that the respondents failed to traverse
the denial of due process that the latter invoked the legal
That on September 9th, you with another presumption "that the ordinary course of business has
student was (sic) caught inside the STC
been followed" (Sec. 5(q), Rule 131, Rules of Court)."
Building, distributing leaflets, enticing
and coercing other students to join the
It is obvious from the expulsion and suspension orders
slated demonstration.
that the petitioners were denied due process, res ipsa
loquitur. For the orders are bereft of the sides of the
In the subsequent days, you were caught petitioners. Hence the legal presumption of regularity
again by the undersigned campaigning
cannot be availed in the instant case.
and distributing leaflets, enjoining other
students to join the boycott.
WHEREFORE, the petition is granted; the expulsion
and suspension orders are hereby set aside but without
That all these actions are contrary to
prejudice to the power of the respondents to formally
MEC regulations and directives that
charge the petitioners for violation(s) of reasonable
appropriate action had to be taken.
school rules and regulations and after due notice to hear
and decide the charge. No special pronouncement as to
For your guidance.
costs.

We are not called upon to determine the validity or


The suspension order which was issued for Marilou
propriety of the tuition fee increase of 15% five weeks
after the classes for the current semester had started. The Elagdon on October 20, 1981, which has been marked
Annex C of the petition reads:
issue in this case is limited to the question as to whether

SO ORDERED.
G.R. No. 127980

December 19, 2007

DE LA SALLE UNIVERSITY, INC., EMMANUEL


SALES, RONALD HOLMES, JUDE DELA TORRE,
AMPARO RIO, CARMELITA QUEBENGCO, AGNES
YUHICO and JAMES YAP, petitioners,
vs.
THE COURT OF APPEALS, HON. WILFREDO D.
REYES, in his capacity as Presiding Judge of Branch 36,
Regional Trial Court of Manila, THE COMMISSION
ON HIGHER EDUCATION, THE DEPARTMENT OF
EDUCATION CULTURE AND SPORTS, ALVIN
AGUILAR, JAMES PAUL BUNGUBUNG, RICHARD
REVERENTE and ROBERTO VALDES, JR.,
respondents.
DECISION

Bungubung, Reverente, and Valdes, Jr.;2 (2) Resolution of


the CA dated October 15, 1996 denying the motion for
reconsideration;3 (3) Order dated January 7, 1997 of the
Regional Trial Court (RTC), Branch 36 Manila granting
private respondent Aguilar's motion to reiterate writ of
preliminary injunction;4 and (4) Resolution No. 181-96 dated
May 14, 1996 of the Commission on Higher Education
(CHED) exonerating private respondent Aguilar and
lowering the penalties for the other private respondents from
expulsion to exclusion.5

Came March 29, 1995 and the following events.


Factual Antecedents
Gleaned from the May 3, 1995 Decision of the DLSU-CSB
Joint Discipline Board, two violent incidents on March 29,
1995 involving private respondents occurred:

REYES, R.T., J.:


NAGTATAGIS sa kasong ito ang karapatang mag-aral
ng apat na estudyante na nasangkot sa away ng
dalawang fraternity at ang karapatang akademiko ng
isang pamantasan.
PRIVATE respondents Alvin Aguilar, James Paul
Bungubung, Richard Reverente and Roberto Valdes, Jr. are
members of Tau Gamma Phi Fraternity who were expelled
by the De La Salle University (DLSU) and College of Saint
Benilde (CSB)1 Joint Discipline Board because of their
involvement in an offensive action causing injuries to
petitioner James Yap and three other student members of
Domino Lux Fraternity. This is the backdrop of the
controversy before Us pitting private respondents' right to
education vis-a-vis the University's right to academic
freedom.
ASSAILED in this Petition for Certiorari, Prohibition and
Mandamus under Rule 65 of the Rules of Court are the
following: (1) Resolution of the Court of Appeals (CA) dated
July 30, 1996 dismissing DLSU's petition for certiorari
against respondent Judge and private respondents Aguilar,

Then, 5 members of the Tau Gamma Phi Fraternity


went to the tambayan of the Domino Lux Fraternity
in the campus. Among them were respondents
Bungubung, Reverente and Papio. They were
looking for a person whose description matched
James Yap. According to them, this person
supposedly "nambastos ng brod." As they could not
find Mr. Yap, one of them remarked "Paano ba iyan.
Pasensiya na lang."

x x x From the testimonies of the complaining


witnesses, it appears that one week prior to March
29, 1995, Mr. James Yap was eating his dinner alone
in Manang's Restaurant near La Salle, when he
overheard two men bad-mouthing and apparently
angry at Domino Lux. He ignored the comments of
the two. When he arrived at his boarding house, he
mentioned the remarks to his two other brods while
watching television. These two brods had earlier
finished eating their dinner at Manang's. Then, the
three, together with four other persons went back to
Manang's and confronted the two who were still in
the restaurant. By admission of respondent
Bungubung in his testimony, one of the two was a
member of the Tau Gamma Phi Fraternity. There
was no rumble or physical violence then.
After this incident, a meeting was conducted
between the two heads of the fraternity through the
intercession of the Student Council. The Tau Gamma
Phi Fraternity was asking for an apology.
"Kailangan ng apology" in the words of respondent
Aguilar. But no apology was made.

Ten minutes before his next class at 6:00 p.m., Mr.


James Yap went out of the campus using the
Engineering Gate to buy candies across Taft Avenue.
As he was about to re-cross Taft Avenue, he heard
heavy footsteps at his back. Eight to ten guys were
running towards him. He panicked. He did not know
what to do. Then, respondent Bungubung punched
him in the head with something heavy in his hands
"parang knuckles." Respondents Reverente and Lee
were behind Yap, punching him. Respondents
Bungubung and Valdes who were in front of him,
were also punching him. As he was lying on the
street, respondent Aguilar kicked him. People
shouted; guards arrived; and the group of attackers
left.
Mr. Yap could not recognize the other members of
the group who attacked him. With respect to
respondent Papio, Mr. Yap said "hindi ko nakita ang
mukha niya, hindi ko nakita sumuntok siya." What
Mr. Yap saw was a long haired guy also running with
the group.
Two guards escorted Mr. Yap inside the campus. At
this point, Mr. Dennis Pascual was at the
Engineering Gate. Mr. Pascual accompanied Yap to
the university clinic; reported the incident to the
Discipline Office; and informed his fraternity brods

at their tambayan. According to Mr. Pascual, their


head of the Domino Lux Fraternity said: "Walang
gagalaw. Uwian na lang."
Mr. Ericson Cano, who was supposed to hitch a ride
with Dennis Pascual, saw him under the clock in
Miguel Building. However, they did not proceed
directly for home. With a certain Michael Perez, they
went towards the direction of Dagonoy Street
because Mr. Pascual was supposed to pick up a book
for his friend from another friend who lives
somewhere in the area.
As they were along Dagonoy Street, and before they
could pass the Kolehiyo ng Malate Restaurant, Mr.
Cano first saw several guys inside the restaurant. He
said not to mind them and just keep on walking.
However, the group got out of the restaurant, among
them respondents Reverente, Lee and Valdes. Mr.
Cano told Mr. Lee: "Ayaw namin ng gulo." But,
respondent Lee hit Mr. Cano without provocation.
Respondent Reverente kicked Mr. Pascual and
respondent Lee also hit Mr. Pascual. Mr. Cano and
Mr. Perez managed to run from the mauling and they
were chased by respondent Lee and two others.

In fact, Mr. Cano saw respondent Valdes near Mr.


Pascual. He was almost near the corner of Leon
Guinto and Estrada; while respondent Pascual who
managed to run was stopped at the end of Dagonoy
along Leon Guinto. Respondent Valdes shouted:
"Mga putang-ina niyo." Respondent Reverente hit
Mr. Pascual for the last time. Apparently being
satisfied with their handiwork, the group left. The
victims, Cano, Perez and Pascual proceeded to a
friend's house and waited for almost two hours, or at
around 8:00 in the evening before they returned to
the campus to have their wounds treated. Apparently,
there were three cars roaming the vicinity.6
The mauling incidents were a result of a fraternity war. The
victims, namely: petitioner James Yap and Dennis Pascual,
Ericson Cano, and Michael Perez, are members of the
"Domino Lux Fraternity," while the alleged assailants,
private respondents Alvin Aguilar, James Paul Bungubung,
Richard Reverente and Roberto Valdes, Jr. are members of
"Tau Gamma Phi Fraternity," a rival fraternity.

The next day, March 30, 1995, petitioner Yap lodged a


complaint7 with the Discipline Board of DLSU charging
private respondents with "direct assault." Similar complaints 8
were also filed by Dennis Pascual and Ericson Cano against
Mr. Pascual was left behind. After respondent
Alvin Lee and private respondents Valdes and Reverente.
Reverente first kicked him, Mr. Pascual was ganged- Thus, cases entitled "De La Salle University and College of
upon by the rest. He was able to run, but the group
St. Benilde v. Alvin Aguilar (AB-BSM/9152105), James Paul
was able to catch up with him. His shirt was torn and Bungubung (AB-PSM/9234403), Robert R. Valdes, Jr. (BShe was hit at the back of his head with a lead pipe.
BS-APM/9235086), Alvin Lee (EDD/9462325), Richard
Respondent Lee who was chasing Cano and Perez,
Reverente (AB-MGT/9153837) and Malvin A. Papio (ABthen returned to Mr. Pascual.
MGT/9251227)" were docketed as Discipline Case No.
9495-3-25121.
Mr. Pascual identified respondents Reverente and
Lee, as among those who hit him. Although Mr.
The Director of the DLSU Discipline Office sent separate
Pascual did not see respondent Valdes hit him, he
notices to private respondents Aguilar, Bungubung and
identified respondent Valdez (sic) as also one of the Valdes, Jr. and Reverente informing them of the complaints
members of the group.
and requiring them to answer. Private respondents filed their
respective answers.9

As it appeared that students from DLSU and CSB 10 were


involved in the mauling incidents, a joint DLSU-CSB
Discipline Board11 was formed to investigate the incidents.
Thus, petitioner Board Chairman Emmanuel Sales sent
notices of hearing12 to private respondents on April 12, 1995.
Said notices uniformly stated as follows:
Please be informed that a joint and expanded
Discipline Board had been constituted to hear and
deliberate the charge against you for violation of
CHED Order No. 4 arising from the written
complaints of James Yap, Dennis C. Pascual, and
Ericson Y. Cano.
You are directed to appear at the hearing of the
Board scheduled on April 19, 1995 at 9:00 a.m. at
the Bro. Connon Hall for you and your witnesses to
give testimony and present evidence in your behalf.
You may be assisted by a lawyer when you give
your testimony or those of your witnesses.
On or before April 18, 1995, you are further directed
to provide the Board, through the Discipline Office,
with a list of your witnesses as well as the sworn
statement of their proposed testimony.
Your failure to appear at the scheduled hearing or
your failure to submit the list of witnesses and the
sworn statement of their proposed testimony will be
considered a waiver on your part to present evidence
and as an admission of the principal act complained
of.
For your strict compliance.13
During the proceedings before the Board on April 19 and 28,
1995, private respondents interposed the common defense of
alibi, summarized by the DLSU-CSB Joint Discipline Board
as follows:

First, in the case of respondent Bungubung, March


29, 1995 was one of the few instances when he was
picked-up by a driver, a certain Romeo S. Carillo.
Most of the time, respondent Bungubung goes home
alone sans driver. But on this particular date,
respondent Bungubung said that his dad asked his
permission to use the car and thus, his dad instructed
this driver Carillo to pick-up his son. Mr. Carillo is
not a family driver, but works from 8:00 a.m. to 5:00
p.m. for the Philippine Ports Authority where the
elder Bungubung is also employed.
Thus, attempting to corroborate the alibi of
respondent Bungubung, Mr. Carillo said that he
arrived at La Salle at 4:56 p.m.; picked-up
respondent at 5:02 p.m.; took the Roxas Blvd. route
towards respondent's house in BF Paraaque (on a
Wednesday in Baclaran); and arrived at the house at
6:15 p.m. Respondent Bungubung was dropped-off
in his house, and taking the same route back, Mr.
Carillo arrived at the South Harbor at 6:55 p.m. the
Philippine Ports Authority is located at the South
Harbor.14
xxxx
Secondly, respondent Valdes said that he was with
his friends at McDonald's Taft just before 6:00 p.m.
of March 29, 1995. He said that he left McDonald at
5:50 p.m. together to get some medicine at the
university clinic for his throat irritation. He said that
he was at the clinic at 5:52 p.m. and went back to
McDonald, all within a span of 3 or even 4 minutes.
Two witnesses, a certain Sharon Sia and the
girlfriend of respondent Valdes, a certain Jorgette
Aquino, attempted to corroborate Valdez' alibi.15
xxxx

Third, respondent Reverente told that (sic) the Board


that he was at his home at 5:00 p.m. of March 29,
1995. He said that he was given the responsibility to
be the paymaster of the construction workers who
were doing some works in the apartment of his
parents. Although he had classes in the evening, the
workers according to him would wait for him
sometimes up to 9:00 p.m. when he arrives from his
classes. The workers get paid everyday.
Respondent Reverente submitted an affidavit,
unsigned by the workers listed there, supposedly
attesting to the fact that he paid the workers at the
date and time in question.16
xxxx
Fourth, respondent Aguilar "solemnly sw[ore] that
[he] left DLSU at 5:00 p.m. for Camp Crame for a
meeting with some of the officers that we were
preparing."17
On May 3, 1995, the DLSU-CSB Joint Discipline Board
issued a Resolution18 finding private respondents guilty.
They were meted the supreme penalty of automatic
expulsion,19 pursuant to CHED Order No. 4.20 The
dispositive part of the resolution reads:
WHEREFORE, considering all the foregoing, the
Board finds respondents ALVIN AGUILAR (ABBSM/9152105), JAMES PAUL BUNGUBUNG
(AB-PSM/9234403), ALVIN LEE (EDD/94623250)
and RICHARD V. REVERENTE (ABMGT/9153837) guilty of having violated CHED
Order No. 4 and thereby orders their automatic
expulsion.
In the case of respondent MALVIN A. PAPIO (ABMGT/9251227), the Board acquits him of the
charge.

SO ORDERED.21
Private respondents separately moved for reconsideration 22
before the Office of the Senior Vice-President for Internal
Operations of DLSU. The motions were all denied in a
Letter-Resolution23 dated June 1, 1995.
On June 5, 1995, private respondent Aguilar filed with the
RTC, Manila, against petitioners a petition for certiorari and
injunction under Rule 65 of the Rules of Court with prayer
for temporary restraining order (TRO) and/or writ of
preliminary injunction. It was docketed as Civil Case No.
95-74122 and assigned to respondent Judge of Branch 36.
The petition essentially sought to annul the May 3, 1995
Resolution of the DLSU-CSB Joint Discipline Board and the
June 1, 1995 Letter-Resolution of the Office of the Senior
Vice-President for Internal Affairs.
The following day, June 6, 1995, respondent Judge issued a
TRO24 directing DLSU, its subordinates, agents,
representatives and/or other persons acting for and in its
behalf to refrain and desist from implementing Resolution
dated May 3, 1995 and Letter-Resolution dated June 1, 1995
and to immediately desist from barring the enrollment of
Aguilar for the second term of school year (SY) 1995.
Subsequently, private respondent Aguilar filed an ex parte
motion to amend his petition to correct an allegation in
paragraph 3.2125 of his original petition. Respondent Judge
amended the TRO26 to conform to the correction made in the
amended petition.27
On June 7, 1995, the CHED directed DLSU to furnish it with
copies of the case records of Discipline Case No. 9495-325121,28 in view of the authority granted to it under Section
77(c) of the Manual of Regulations for Private Schools
(MRPS).
On the other hand, private respondents Bungubung and
Reverente, and later, Valdes, filed petitions-in-intervention29

in Civil Case No. 95-74122. Respondent Judge also issued


corresponding temporary restraining orders to compel
petitioner DLSU to admit said private respondents.
On June 19, 1995, petitioner Sales filed a motion to dismiss30
in behalf of all petitioners, except James Yap. On June 20,
1995, petitioners filed a supplemental motion to dismiss31 the
petitions-in-intervention.
On September 20, 1995, respondent Judge issued an Order32
denying petitioners' (respondents there) motion to dismiss
and its supplement, and granted private respondents'
(petitioners there) prayer for a writ of preliminary injunction.
The pertinent part of the Order reads:
For this purpose, respondent, its agents,
representatives or any and all other persons acting
for and in its behalf is/are restrained and enjoined
from
1. Implementing and enforcing the
Resolution dated May 3, 1995 ordering the
automatic expulsion of petitioner and the
petitioners-in-intervention from the De La
Salle University and the letter-resolution
dated June 1, 1995, affirming the Resolution
dated May 3, 1995; and

1. Implementing and enforcing the


Resolution dated May 3, 1995 ordering the
automatic expulsion of petitioner and
petitioners-in-intervention and the LetterResolution dated June 1, 1995; and
2. Barring the enrollment of petitioner and
petitioners-in-intervention in the courses
offered at respondent (De La Salle
University) and to forthwith allow all said
petitioner and petitioners-in-intervention to
enroll and complete their respective
courses/degrees until their graduation
thereat.
The Writ of Preliminary Injunction shall take effect
upon petitioner and petitioners-in-intervention
posting an injunctive bond in the amount of
P15,000.00 executed in favor of respondent to the
effect that petitioner and petitioners-in-intervention
will pay to respondent all damages that the latter
may suffer by reason of the injunction if the Court
will finally decide that petitioner and petitioners-inintervention are not entitled thereto.

September 20, 1995. On September 25, 1995, respondent


Judge issued35 a writ of preliminary injunction, the relevant
portion of which reads:
IT IS HEREBY ORDERED by the undersigned of
the REGIONAL TRIAL COURT OF MANILA that
until further orders, you the said DE LA SALLE
University as well as your subordinates, agents,
representatives, employees and any other person
assisting or acting for or on your behalf, to
immediately desist from implementing the
Resolution dated May 3, 1995 ordering the
automatic expulsion of petitioner and the intervenors
in DLSU, and the letter-resolution dated June 1,
1995 affirming the said Resolution of May 3, 1995
and to immediately desist from barring the
enrolment of petitioner and intervenors in the
courses offered at DLSU and to allow them to enroll
and complete their degree courses until their
graduation from said school.36

On October 16, 1995, petitioner DLSU filed with the CA a


petition for certiorari37 (CA-G.R. SP No. 38719) with prayer
for a TRO and/or writ of preliminary injunction to enjoin the
enforcement of respondent Judge's September 20, 1995
The motion to dismiss and the supplement thereto is Order and writ of preliminary injunction dated September
denied for lack of merit. Respondents are directed to 25, 1995.
file their Answer to the Petition not later than fifteen
2. Barring the enrolment of petitioner and
(15) days from receipt thereof.
On April 12, 1996, the CA granted petitioners' prayer for
petitioners-in-intervention in the courses
preliminary injunction.
offered at respondent De La Salle University
SO ORDERED.33
and to immediately allow them to enroll and
On May 14, 1996, the CHED issued its questioned
complete their respective courses/degrees
Despite the said order, private respondent Aguilar was
Resolution No. 181-96, summarily disapproving the
until their graduation thereat in accordance refused enrollment by petitioner DLSU when he attempted
penalty of expulsion for all private respondents. As for
with the standards set by the latter.
to enroll on September 22, 1995 for the second term of SY
Aguilar, he was to be reinstated, while other private
1995-1996. Thus, on September 25, 1995, Aguilar filed with respondents were to be excluded.38 The Resolution states:
WHEREFORE, the ancillary remedy prayed for is
respondent Judge an urgent motion to cite petitioners
granted. Respondent, its agents, representatives, or
(respondents there) in contempt of court.34 Aguilar also
RESOLUTION 181-96
any and all persons acting for and its behalf are
prayed that petitioners be compelled to enroll him at DLSU
hereby restrained and enjoyed from:
in accordance with respondent Judge's Order dated

RESOLVED THAT THE REQUEST OF THE DE


LA SALLE UNIVERSITY (DLSU), TAFT
AVENUE, MANILA FOR THE APPROVAL OF
THE PENALTY OF EXPULSION IMPOSED ON
MR. ALVIN AGUILAR, JAMES PAUL
BUNGUBUNG, ROBERT R. VALDES, JR., ALVIN
LEE AND RICHARD V. REVERENTE BE, AS IT
IS HEREBY IS, DISAPPROVED.

enrollment this current TRIMESTER, and in order to


After considering the Opposition and for lack of
prevent further prejudice to his rights as a student of
merit, the Motion for Reconsideration is hereby
the institution, DLSU, through the proper school
denied.
authorities, is hereby directed to allow Mr. Alvin
SO ORDERED.45
Aguilar to provisionally enroll, pending the
Commission's Resolution of the instant Motion for
On October 28, 1996, petitioners requested transfer of case
Reconsideration filed by DLSU.
records to the Department of Education, Culture and Sports
41
(DECS) from the CHED.46 Petitioners claimed that it is the
SO ORDERED.
DECS, not CHED, which has jurisdiction over expulsion
RESOLVED FURTHER, THAT THE
Notwithstanding the said directive, petitioner DLSU,
cases, thus, necessitating the transfer of the case records of
COMMISSION DIRECT THE DLSU TO
Discipline Case No. 9495-3-25121 to the DECS.
IMMEDIATELY EFFECT THE REINSTATEMENT through petitioner Quebengco, still refused to allow private
respondent
Aguilar
to
enroll.
Thus,
private
respondent
OF MR. AGUILAR AND THE LOWERING OF
On November 4, 1996, in view of the dismissal of the
Aguilar's counsel wrote another demand letter to petitioner
THE PENALTY OF MR. JAMES PAUL
42
petition for certiorari in CA-G.R. SP No. 38719 and the
DLSU.
BUNGUBUNG, MR. ROBER R. VALDEZ, JR.,
automatic lifting of the writ of preliminary injunction,
(sic) MR. ALVIN LEE AND MR. RICHARD V.
Meanwhile, on June 3, 1996, private respondent Aguilar,
private respondent Aguilar filed an urgent motion to reiterate
REVERENTE FROM EXPULSION TO
39
using
CHED
Resolution
No.
181-96,
filed
a
motion
to
writ of preliminary injunction dated September 25, 1995
EXCLUSION.
43
dismiss in the CA, arguing that CHED Resolution No. 181- before respondent RTC Judge of Manila.47
96 rendered the CA case moot and academic.
Despite the directive of CHED, petitioner DLSU again
On January 7, 1997, respondent Judge issued its
prevented private respondent Aguilar from enrolling and/or
On July 30, 1996, the CA issued its questioned resolution questioned order granting private respondent Aguilar's
attending his classes, prompting his lawyer to write several
40
urgent motion to reiterate preliminary injunction. The
demand letters to petitioner DLSU. In view of the refusal of granting the motion to dismiss of private respondent
pertinent portion of the order reads:
petitioner DLSU to enroll private respondent Aguilar, CHED Aguilar, disposing thus:
wrote a letter dated June 26, 1996 addressed to petitioner
THE FOREGOING CONSIDERED, dismissal of
In light of the foregoing, petitioner Aguilar's urgent
Quebengco requesting that private respondent Aguilar be
herein petition is hereby directed.
motion to reiterate writ of preliminary injunction is
allowed to continue attending his classes pending the
hereby granted, and respondents' motion to dismiss
resolution of its motion for reconsideration of Resolution
44
SO ORDERED.
is denied.
No. 181-96. However, petitioner Quebengco refused to do
so, prompting CHED to promulgate an Order dated
On October 15, 1996, the CA issued its resolution
The writ of preliminary injunction dated September
September 23, 1996 which states:
denying petitioners' motion for reconsideration, as
25, 1995 is declared to be in force and effect.
follows:
Acting on the above-mentioned request of Mr.
Let a copy of this Order and the writ be served
Aguilar through counsel enjoining De La Salle
It
is
obvious
to
Us
that
CHED
Resolution
No.
181personally by the Court's sheriff upon the
University (DLSU) to comply with CHED
96
is
immediately
executory
in
character,
the
respondents at petitioners' expense.
Resolution 181-96 (Re: Expulsion Case of Alvin
pendency of a Motion for Reconsideration
Aguilar, et al. v. DLSU) directing DLSU to reinstate
notwithstanding.
SO ORDERED.48
Mr. Aguilar and finding the urgent request as
meritorious, there being no other plain and speedy
remedy available, considering the set deadline for

Accordingly, private respondent Aguilar was allowed to


conditionally enroll in petitioner DLSU, subject to the
continued effectivity of the writ of preliminary injunction
dated September 25, 1995 and to the outcome of Civil Case
No. 95-74122.
On February 17, 1997, petitioners filed the instant petition.

2.b Can petitioner DLSU invoke its right to


academic freedom?
2.c Was the guilt of private respondents
proven by substantial evidence?
3. Whether or not the penalty imposed by DLSU on
private respondents is proportionate to their
misdeed.

On June 15, 1998, We issued a TRO49 as prayed for by the


urgent motion for the issuance of a TRO50 dated June 4, 1998
Our Ruling
of petitioners, and enjoined respondent Judge from
implementing the writ of preliminary injunction dated
Prefatorily, there is merit in the observation of petitioners 53
September 25, 1995 issued in Civil Case No. 95-74122,
that while CHED Resolution No. 181-96 disapproved the
effective immediately and until further orders from this
expulsion of other private respondents, it nonetheless
Court.
authorized their exclusion from petitioner DLSU. However,
because of the dismissal of the CA case, petitioner DLSU is
On March 27, 2006, private respondent Aguilar filed his
manifestation51 stating that he has long completed his course now faced with the spectacle of having two different
directives from the CHED and the respondent Judge
at petitioner DLSU. He finished and passed all his enrolled
subjects for the second trimester of 1997-1998, as indicated CHED ordering the exclusion of private respondents
Bungubung, Reverente, and Valdes, Jr., and the Judge
in his transcript of records52 issued by DLSU. However,
despite having completed all the academic requirements for ordering petitioner DLSU to allow them to enroll and
complete their degree courses until their graduation.
his course, DLSU has not issued a certificate of
completion/graduation in his favor.
This is the reason We opt to decide the whole case on the
merits, brushing aside technicalities, in order to settle the
Issues
substantial issues involved. This Court has the power to take
We are tasked to resolve the following issues:
cognizance of the petition at bar due to compelling reasons,
and the nature and importance of the issues raised warrant
1. Whether it is the DECS or the CHED which has
the immediate exercise of Our jurisdiction.54 This is in
legal authority to review decisions of institutions of consonance with our case law now accorded near-religious
higher learning that impose disciplinary action on
reverence that rules of procedure are but tools designed to
their students found violating disciplinary rules.
facilitate the attainment of justice, such that when its rigid
application tends to frustrate rather than promote substantial
2. Whether or not petitioner DLSU is within its
justice, this Court has the duty to suspend their operation. 55
rights in expelling private respondents.
I. It is the CHED, not DECS, which has the
2.a Were private respondents accorded due
power of supervision and review over
process of law?
disciplinary cases decided by institutions
of higher learning.

Ang CHED, hindi ang DECS, ang may kapangyarihan


ng pagsubaybay at pagrepaso sa mga desisyong
pandisiplina ng mga institusyon ng mas mataas na pagaaral.
Petitioners posit that the jurisdiction and duty to review
student expulsion cases, even those involving students in
secondary and tertiary levels, is vested in the DECS not in
the CHED. In support of their stance, petitioners cite
Sections 4,56 15(2) & (3),57 54,58 57(3)59 and 7060 of Batas
Pambansa (B.P.) Blg. 232, otherwise known as the
"Education Act of 1982."
According to them, Republic Act (R.A.) No. 7722 did not
transfer to the CHED the DECS' power of
supervision/review over expulsion cases involving
institutions of higher learning. They say that unlike B.P. Blg.
232, R.A. No. 7722 makes no reference to the right and duty
of learning institutions to develop moral character and instill
discipline among its students. The clear concern of R.A. No.
7722 in the creation of the CHED was academic, i.e., the
formulation, recommendation, setting, and development of
academic plans, programs and standards for institutions of
higher learning. The enumeration of CHED's powers and
functions under Section 8 does not include
supervisory/review powers in student disciplinary cases. The
reference in Section 3 to CHED's "coverage" of institutions
of higher education is limited to the powers and functions
specified in Section 8. The Bureau of Higher Education,
which the CHED has replaced and whose functions and
responsibilities it has taken over, never had any authority
over student disciplinary cases.
We cannot agree.
On May 18, 1994, Congress approved R.A. No. 7722,
otherwise known as "An Act Creating the Commission on
Higher Education, Appropriating Funds Thereof and for
other purposes."

Section 3 of the said law, which paved the way for the
creation of the CHED, provides:
Section 3. Creation of the Commission on Higher
Education. In pursuance of the abovementioned
policies, the Commission on Higher Education is
hereby created, hereinafter referred to as
Commission.
The Commission shall be independent and separate
from the Department of Education, Culture and
Sports (DECS) and attached to the office of the
President for administrative purposes only. Its
coverage shall be both public and private institutions
of higher education as well as degree-granting
programs in all post secondary educational
institutions, public and private.

power of supervision/review over expulsion cases involving


institutions of higher learning.

exercise of institutions of higher learning of their right to


academic freedom.

First, the foregoing provisions are all-embracing. They


make no reservations of powers to the DECS insofar as
institutions of higher learning are concerned. They show that
the authority and supervision over all public and private
institutions of higher education, as well as degree-granting
programs in all post-secondary educational institutions,
public and private, belong to the CHED, not the DECS.

Fourth, petitioner DLSU cited no authority in its bare claim


that the Bureau of Higher Education, which CHED replaced,
never had authority over student disciplinary cases. In fact,
the responsibilities of other government entities having
functions similar to those of the CHED were transferred to
the CHED.62

Second, to rule that it is the DECS which has authority to


decide disciplinary cases involving students on the tertiary
level would render nugatory the coverage of the CHED,
which is "both public and private institutions of higher
education as well as degree granting programs in all post
secondary educational institutions, public and private." That
would be absurd.

Section 77 of the MRPS63 on the process of review in student


discipline cases should therefore be read in conjunction
with the provisions of R.A. No. 7722.

Fifth, Section 18 of R.A. No. 7722 is very clear in stating


that "[j]urisdiction over DECS-supervised or chartered
state-supported post-secondary degree-granting vocational
and tertiary institutions shall be transferred to the
The powers and functions of the CHED are enumerated in
Commission [On Higher Education]." This provision does
Section 8 of R.A. No. 7722. They include the following:
It is of public knowledge that petitioner DLSU is a private
not limit or distinguish that what is being transferred to the
educational institution which offers tertiary degree programs. CHED is merely the formulation, recommendation, setting
Sec. 8. Powers and functions of the Commission.
Hence, it is under the CHED authority.
and development of academic plans, programs and standards
The Commission shall have the following powers
for institutions of higher learning, as what petitioners would
and functions:
Third, the policy of R.A. No. 772261 is not only the
have us believe as the only concerns of R.A. No. 7722. Ubi
protection, fostering and promotion of the right of all
lex non distinguit nec nos distinguere debemus: Where the
xxxx
citizens to affordable quality education at all levels and the
law does not distinguish, neither should we.
taking of appropriate steps to ensure that education shall be
n) promulgate such rules and regulations and
accessible to all. The law is likewise concerned with
To Our mind, this provision, if not an explicit grant of
exercise such other powers and functions as may be ensuring and protecting academic freedom and with
jurisdiction to the CHED, necessarily includes the transfer
necessary to carry out effectively the purpose and
promoting its exercise and observance for the continued
to the CHED of any jurisdiction which the DECS might have
objectives of this Act; and
intellectual growth of students, the advancement of learning possessed by virtue of B.P. Blg. 232 or any other law or rule
and research, the development of responsible and effective
for that matter.
o) perform such other functions as may be necessary leadership, the education of high-level and middle-level
for its effective operations and for the continued
IIa. Private respondents were accorded due process of law.
professionals, and the enrichment of our historical and
enhancement of growth or development of higher
cultural heritage.
education.
Ang mga private respondents ay nabigyan ng tamang
It is thus safe to assume that when Congress passed R.A. No. proseso ng batas.
Clearly, there is no merit in the contention of petitioners that 7722, its members were aware that disciplinary cases
R.A. No. 7722 did not transfer to the CHED the DECS'
The Due Process Clause in Article III, Section 1 of the
involving students on the tertiary level would continue to
Constitution embodies a system of rights based on moral
arise in the future, which would call for the invocation and

principles so deeply imbedded in the traditions and feelings


of our people as to be deemed fundamental to a civilized
society as conceived by our entire history.64 The
constitutional behest that no person shall be deprived of life,
liberty or property without due process of law is solemn and
inflexible.65

"To be heard" does not only mean presentation of testimonial


evidence in court one may also be heard through pleadings
and where the opportunity to be heard through pleadings is
accorded, there is no denial of due process. 72

Private respondents were duly informed in writing of the


charges against them by the DLSU-CSB Joint Discipline
In administrative cases, such as investigations of students
Board through petitioner Sales. They were given the
found violating school discipline, "[t]here are withal
opportunity to answer the charges against them as they, in
minimum standards which must be met before to satisfy the fact, submitted their respective answers. They were also
demands of procedural due process and these are: that (1) the informed of the evidence presented against them as they
students must be informed in writing of the nature and cause attended all the hearings before the Board. Moreover, private
of any accusation against them; (2) they shall have the right respondents were given the right to adduce evidence on their
to answer the charges against them and with the assistance if behalf and they did. Lastly, the Discipline Board considered
counsel, if desired; (3) they shall be informed of the
all the pieces of evidence submitted to it by all the parties
evidence against them; (4) they shall have the right to
before rendering its resolution in Discipline Case No. 9495adduce evidence in their own behalf; and (5) the evidence
3-25121.
must be duly considered by the investigating committee or
Private respondents cannot claim that they were denied due
official designated by the school authorities to hear and
66
process when they were not allowed to cross-examine the
decide the case."
witnesses against them. This argument was already rejected
Where a party was afforded an opportunity to participate in
in Guzman v. National University73 where this Court held
the proceedings but failed to do so, he cannot complain of
that "x x x the imposition of disciplinary sanctions requires
deprivation of due process.67 Notice and hearing is the
observance of procedural due process. And it bears stressing
bulwark of administrative due process, the right to which is that due process in disciplinary cases involving students does
among the primary rights that must be respected even in
not entail proceedings and hearings similar to those
administrative proceedings.68 The essence of due process is
prescribed for actions and proceedings in courts of justice.
simply an opportunity to be heard, or as applied to
The proceedings in student discipline cases may be
administrative proceedings, an opportunity to explain one's
summary; and cross examination is not, x x x an essential
side or an opportunity to seek reconsideration of the action
part thereof."
69
or ruling complained of. So long as the party is given the
IIb. Petitioner DLSU, as an institution of higher learning,
opportunity to advocate her cause or defend her interest in
possesses academic freedom which includes determination
due course, it cannot be said that there was denial of due
70
of who to admit for study.
process.
Ang petitioner DLSU, bilang institusyon ng mas mataas
A formal trial-type hearing is not, at all times and in all
na pag-aaral, ay nagtataglay ng kalayaang akademiko na
instances, essential to due process it is enough that the
parties are given a fair and reasonable opportunity to explain sakop ang karapatang pumili ng mga mag-aaral dito.
their respective sides of the controversy and to present
supporting evidence on which a fair decision can be based. 71

Section 5(2), Article XIV of the Constitution guaranties all


institutions of higher learning academic freedom. This
institutional academic freedom includes the right of the
school or college to decide for itself, its aims and objectives,
and how best to attain them free from outside coercion or
interference save possibly when the overriding public
interest calls for some restraint.74 According to present
jurisprudence, academic freedom encompasses the
independence of an academic institution to determine for
itself (1) who may teach, (2) what may be taught, (3) how it
shall teach, and (4) who may be admitted to study.75
It cannot be gainsaid that "the school has an interest in
teaching the student discipline, a necessary, if not
indispensable, value in any field of learning. By instilling
discipline, the school teaches discipline. Accordingly, the
right to discipline the student likewise finds basis in the
freedom "what to teach."76 Indeed, while it is categorically
stated under the Education Act of 1982 that students have a
right "to freely choose their field of study, subject to existing
curricula and to continue their course therein up to
graduation,"77 such right is subject to the established
academic and disciplinary standards laid down by the
academic institution. Petitioner DLSU, therefore, can very
well exercise its academic freedom, which includes its free
choice of students for admission to its school.
IIc. The guilt of private respondents Bungubung,
Reverente and Valdes, Jr. was proven by substantial
evidence.
Ang pagkakasala ng private respondents na sina
Bungubung, Reverente at Valdes, Jr. ay napatunayan ng
ebidensiyang substansyal.
As has been stated earlier, private respondents interposed the
common defense of alibi. However, in order that alibi may
succeed as a defense, "the accused must establish by clear
and convincing evidence (a) his presence at another place at
the time of the perpetration of the offense and (b) the

physical impossibility of his presence at the scene of the


crime."78

impossible for them to have been there. Moreover, their alibi


cannot prevail over their positive identification by the
victims.

On the other hand, the defense of alibi may not be


successfully invoked where the identity of the assailant has
We hark back to this Court's pronouncement affirming the
been established by witnesses.79 Positive identification of
expulsion of several students found guilty of hazing:
accused where categorical and consistent, without any
No one can be so myopic as to doubt that the
showing of ill motive on the part of the eyewitness
immediate reinstatement of respondent students who
testifying, should prevail over the alibi and denial of
have been investigated and found guilty by the
appellants whose testimonies are not substantiated by clear
Disciplinary Board to have violated petitioner
and convincing evidence.80 Well-settled is the rule that denial
university's disciplinary rules and standards will
and alibi, being weak defenses, cannot overcome the positive
81
certainly undermine the authority of the
testimonies of the offended parties.
administration of the school. This we would be most
Courts reject alibi when there are credible eyewitnesses to
loathe to do.
the crime who can positively identify the accused.82 Alibi is
More importantly, it will seriously impair petitioner
an inherently weak defense and courts must receive it with
83
university's academic freedom which has been
caution because one can easily fabricate an alibi.
enshrined in the 1935, 1973 and the present 1987
Jurisprudence holds that denial, like alibi, is inherently weak
Constitution.87
and crumbles in light of positive declarations of truthful
witnesses who testified on affirmative matters that accused
Certainly, private respondents Bungubung, Reverente and
were at the scene of the crime and were the victim's
Valdes, Jr. do not deserve to claim a venerable institution as
assailants. As between categorical testimonies that ring of
truth on one hand and a bare denial on the other, the former their own, for they may foreseeably cast a malevolent
influence on the students currently enrolled, as well as those
must prevail.84 Alibi is the weakest of all defenses for it is
who come after them.88 It must be borne in mind that
easy to fabricate and difficult to disprove, and it is for this
universities are established, not merely to develop the
reason that it cannot prevail over the positive identification
85
intellect and skills of the studentry, but to inculcate lofty
of accused by the witnesses.
values, ideals and attitudes; nay, the development, or
The required proof in administrative cases, such as in student flowering if you will, of the total man.89
discipline cases, is neither proof beyond reasonable doubt
nor preponderance of evidence but only substantial evidence. As for private respondent Aguilar, however, We are inclined
According to Ang Tibay v. Court of Industrial Relations,86 it to give credence to his alibi that he was at Camp Crame in
Quezon City at the time of the incident in question on March
means "such reasonable evidence as a reasonable mind
29, 1995. This claim was amply corroborated by the
might accept as adequate to support a conclusion."
certification that he submitted before the DLSU-CSB Joint
Discipline Board, to wit:
Viewed from the foregoing, We reject the alibi of private
respondents Bungubung, Valdes Jr., and Reverente.1awphi1
C E R T I F I C AT I O N
They were unable to show convincingly that they were not at
the scene of the crime on March 29, 1995 and that it was

TO WHOM THIS MAY CONCERN:


We, the undersigned, hereby declare and
affirm by way of this Certification that
sometime on March 29, 1995, at about and
between 4:30 P.M. and 5:30 P.M., we were
together with Alvin A. Aguilar, at Kiangan
Hall, inside Camp Crame, Quezon City,
meeting in connection with an affair of our
class known as Class 7, Batch 89 of the
Philippine Constabulary discussing on the
proposed sponsorship of TAU GAMMA PHI
from said Batch '89 affair.
That the meeting was terminated at about 6:30 P.M.
that evening and Alvin Aguilar had asked our
permission to leave and we saw him leave Camp
Crame, in his car with the driver.
April 18, 1995, Camp Crame, Quezon City.90
The said certification was duly signed by PO3 Nicanor R.
Faustino (Anti-Organized Crime CIC, NCR), PO3 Alejandro
D. Deluviar (ODITRM, Camp Crame, Quezon City), PO2
Severino C. Filler (TNTSC, Camp Crame, Quezon City), and
PO3 Ireneo M. Desesto (Supply Center, PNPLSS). The rule
is that alibi assumes significance or strength when it is
amply corroborated by credible and disinterested witnesses. 91
It is true that alibi is a weak defense which an accused can
easily fabricate to escape criminal liability. But where the
prosecution evidence is weak, and betrays lack of credibility
as to the identification of defendant, alibi assumes
commensurate strength. This is but consistent with the
presumption of innocence in favor of accused. 92
Alibi is not always undeserving of credit, for there are times
when accused has no other possible defense for what could
really be the truth as to his whereabouts at the crucial time,
and such defense may, in fact, tilt the scales of justice in his
favor.93

III. The penalty of expulsion imposed by DLSU on private


respondents is disproportionate to their misdeed.

respondents from its rolls for being undesirable, and transfer


credentials immediately issued.

Ang parusang expulsion na ipinataw ng DLSU sa private WHEREFORE, the petition is PARTIALLY GRANTED.
respondents ay hindi angkop sa kanilang pagkakasala.
The Court of Appeals Resolutions dated July 30, 1996 and
dated October 15, 1996, and Regional Trial Court of Manila,
It is true that schools have the power to instill discipline in
Branch 36, Order dated January 7, 1997 are ANNULLED
their students as subsumed in their academic freedom and
AND SET ASIDE, while CHED Resolution 181-96 dated
that "the establishment of rules governing university-student May 14, 1996 is AFFIRMED.
relations, particularly those pertaining to student discipline,
may be regarded as vital, not merely to the smooth and
Petitioner DLSU is ordered to issue a certificate of
efficient operation of the institution, but to its very
completion/graduation in favor of private respondent
survival."94 This power, however, does not give them the
Aguilar. On the other hand, it may exclude or drop the names
untrammeled discretion to impose a penalty which is not
of private respondents Bungubung, Reverente, and Valdes,
commensurate with the gravity of the misdeed. If the
Jr. from its rolls, and their transfer credentials immediately
concept of proportionality between the offense committed
issued.
and the sanction imposed is not followed, an element of
arbitrariness intrudes. That would give rise to a due process SO ORDERED.
question.95
G.R. Nos. 164684-85 November 11, 2005
We agree with respondent CHED that under the
PHILIPPINE LONG DISTANCE TELEPHONE
circumstances, the penalty of expulsion is grossly
COMPANY, INC., Petitioner,
disproportionate to the gravity of the acts committed by
vs.
private respondents Bungubung, Reverente, and Valdes, Jr.
ANTONIO Q. TIAMSON, Respondent.
Each of the two mauling incidents lasted only for few
seconds and the victims did not suffer any serious injury.
DECISION
Disciplinary measures especially where they involve
suspension, dismissal or expulsion, cut significantly into the
future of a student. They attach to him for life and become a CALLEJO, SR., J.:
mortgage of his future, hardly redeemable in certain cases.
Being questioned in this petition for review on certiorari is
Officials of colleges and universities must be anxious to
1
protect it, conscious of the fact that, appropriately construed, the Decision of the Court of Appeals (CA) dated April 16,
a disciplinary action should be treated as an educational tool 2004 in CA-G.R. SP Nos. 51855 and 52247, and the
Resolution dated July 27, 2004 denying the motion for
rather than a punitive measure.96
reconsideration thereof.
Accordingly, We affirm the penalty of exclusion97 only, not
On April 16, 1986, the Philippine Long Distance Telephone
expulsion,98 imposed on them by the CHED. As such,
Company, Inc. (PLDT) employed Antonio Q. Tiamson as a
pursuant to Section 77(b) of the MRPS, petitioner DLSU
Radio Technician II (JG4). He was assigned at the
may exclude or drop the names of the said private
companys North Luzon Toll Network Division, Clark

Transmission Maintenance Center (Clark-TMC) in


Pampanga. After the expiration of the probationary period,
he was extended regular appointment for the same position.
In a Letter2 dated July 29, 1994, Anthony Dy Dee, the
President of the Angeles City Telephone System and
Datelcom Corporation, informed PLDT of his complaint
against its employees assigned in Clark-TMC, stating therein
that he suspected them to be in cohorts with the local
subscribers in effecting illegal overseas calls. Acting on the
letter-complaint, PLDT immediately dispatched a team of
inspectors and investigators from its Quality Control and
Inspection Department (QCID) and Security Division to
conduct surveillance operations in the area. On August 2,
1994, Vidal Busa, a radio technician, was caught in flagrante
delicto while monitoring an illegally connected overseas call
using the radio facilities of the companys Clark-TMC Radio
Room.3
The QCID, likewise, requested the Switching Network
Division at PLDTs Sampaloc National Toll Center to print
the CAMA4 tape recording of all long distance calls
originating from the PLDT Clark Exchange Traffic
for the period of July 29 to August 2, 1994. The printout
revealed that a total of 469 fraudulent overseas and local
calls were connected and completed at the PLDT ClarkTMC Radio Room for the said period. Three overseas calls
to Saudi Arabia made on August 1, 1994 were imputed to
Tiamson who appeared to be on duty from 10:00 p.m. to
6:00 a.m.5
The QCID conducted its initial investigation on August 2,
1994, where Busa readily admitted his involvement in the
illegal connection of overseas calls. In his sworn statement,
he specifically named Arnel Cayanan, his Shift Supervisor,
Antonio Tiamson and Paul Cruzada, both radio technicians,
as the other employees actively engaged in the illegal
practice. He stated that he knew about this because whenever
he would relieve them from their tour of duty, he would see
that the circuit was engaged.6

On August 3, 1994, during a confrontation between Busa and


Tiamson, the former reiterated his earlier statement that the
latter was involved in the illegal act of connecting overseas
calls.7 For his part, Tiamson admitted that he knew how to
make an overseas call using the companys radio equipment
and that he learned how to do so through hands-on
experimentation and intensive reading of operating manuals.
He, however, denied having actually made an illegal
connection of overseas calls. He declared that he knew of the
wrongdoings of Busa and even disconnected the latters
overseas telephone calls whenever he (Tiamson) was on
duty. Tiamson claimed that he failed to report the actuations
of Busa because the latter was his supervisor and was afraid
to antagonize him.8
On August 5, 1994, there was another confrontation
proceeding between Busa, Tiamson, Cruzada and Cayanan.
In their sworn statements, Busa and Cruzada testified that,
sometimes when they relieve Cayanan from his duty, they
would discover an illegal connection and an on-going
conversation in the line.9 Tiamson maintained that he
disconnected the illegal calls of Busa, while Cayanan
implicated his subordinates.

scam was complained and notified by Mr. A. Dy to Mrs. B.


G. Gendrano Clark Exchange Division Head on July 26,
1994.
3. The complainant requested assistance to NBI and PLDT
QCI to apprehend the personnel responsible for the illegal
connection.
4. A clue was provided by Mr. Anthony Dy that the illegal
overseas call was coming from Clark-TMC through taped
and equipment monitoring.
5. In the QCI investigation, you were implicated by your
fellow Radio Technician Mr. Vidal C. Busa as involved in
the case. You admitted you know how to operate the Lenkurt
26600 Signalling Test Set to initiate a call but denied doing it
for personal gain or interest but you failed to report the
anomaly to your superior as one of your supervisors was
involved in the fraudulent case.
The acts described above are in violation of the Companys
rules and regulations and is punishable with dismissal from
the service.

The QCID recommended that administrative action for


serious misconduct be instituted against the said employees.
Consequently, the company issued to Tiamson an InterOffice Memorandum dated August 12, 1994, charging him
with violation of the companys disciplinary rules and
regulations. He was, likewise, required to explain within 72
hours why he should not be dismissed, thus:

In view of the above, please explain in writing within 72


hours from receipt hereof why you should not be dismissed
from the service for the acts described above. You may elect
to be heard if you so desire. 10

Investigation of the complaint indicated hereunder disclosed


that:

On August 18, 1994, Tiamson submitted his written


explanation denying any participation in the illegal activities
at PLDTs Clark-TMC. He averred that Busas statement
against him was malicious and untrue and that he was the
one relieving Busa from his tour of duty and not the other
way around. He insisted that on August 1, 1994, his tour of
duty was from 6:00 a.m. to 10:00 p.m.12

1. Complainant Mr. Anthony Dy, President DATELCOM


Corp.
2. The decrease of toll revenue for DATELCOM
Angeles/Mabalacat Exchange due to fraudulent overseas call

Meanwhile, Tiamson was placed under preventive


suspension on August 16, 1994.11

PLDT found his explanation unsatisfactory and inadequate


in substance. Thus, it issued an Inter-Office Memo 13 dated
October 5, 1994, terminating Tiamsons employment
effective October 7, 1994 on the ground of serious
misconduct and/or fraud.
Tiamson filed a complaint against PLDT for illegal
suspension, illegal dismissal, damages and other monetary
claims, docketed as NLRC Case No. RAB-III-07-6414-95.
The Labor Arbiter resolved the case in favor of Tiamson:
WHEREFORE, premises considered, judgment is hereby
rendered declaring respondent PLDT guilty of illegal
dismissal and it is hereby ordered to reinstate complainant to
his former position without loss of seniority rights and with
full backwages reckoned from the date of his dismissal up to
his actual or payroll reinstatement at the option of the
respondent, which as of this date is in the amount of Three
Hundred Seventy-Two Thousand Eight Hundred TwentyFive and 32/100 (P372,825.32) Pesos.
Further, respondent is ordered to pay complainant attorneys
fee in the amount of Thirty-Seven Thousand Two Hundred
Eighty-Two and 53/100 (P37,282.53) Pesos.
The claims for moral and exemplary damages are dismissed
for lack of evidence.
SO ORDERED.14
The Labor Arbiter declared that the complainant could not
have made any illegal connection on August 1, 1994 from
10:00 p.m. to 6:00 a.m. because he was off-duty.
PLDT elevated the case to the National Labor Relations
Commission (NLRC). On August 31, 1998, the NLRC ruled
that while there was just cause for Tiamsons dismissal, the
penalty of dismissal was too harsh. Hence, the NLRC

ordered that Tiamson be reinstated to his former position


without loss of seniority rights, but without backwages. 15

opportunity to confute the charge that during his duty on


August 1, 1994, three overseas calls to Saudi Arabia were
recorded in the CAMA tape.19

Both parties moved to reconsider the decision, but the NLRC


denied the motions for lack of merit.16
The petitioner timely filed a motion for reconsideration,
which the CA denied in its Resolution20 dated July 27, 2004.
PLDT filed a petition for certiorari before the CA, assailing
the NLRCs order of reinstatement despite a categorical
The petitioner now comes before this Court, alleging that:
finding that Tiamson was guilty of illegal connection of
overseas calls. The petition was docketed as CA-G.R. SP No. THE COURT OF APPEALS COMMITTED SERIOUS
51855. Tiamson filed a similar petition, assailing the deletion ERROR IN REINSTATING THE DECISION OF THE
ARBITER A QUO AS SAID DECISION WAS NOT IN
of the award of backwages and attorneys fees. This was
ACCORD WITH LAW AND CONTRARY TO THE
docketed as CA-G.R. SP No. 52247. The CA, thereafter,
EVIDENCE ON RECORD.21
ordered the consolidation of the two petitions.
The petitioner submits that it has presented more than
substantial evidence to prove that the respondent was
involved in the illegal connection of overseas calls. The
WHEREFORE, the petition by the PLDT under CA-G.R.
petitioner avers that the CA erred in holding that Busas
SP No. 51855 is DENIED DUE COURSE and
sworn statement was not credible. According to the CA, it
DISMISSED while the petition by Antonio Tiamson under
would have been impossible for Busa to see the respondent
CA-G.R. SP No. 52247 is GIVEN DUE COURSE and
making an illegal connection since his tour of duty preceded
GRANTED, and the Decision dated October 15, 1997 of the that of the respondent. The petitioner, however, asserts that
Labor Arbiter which was set aside by the NLRC, is hereby
there was a rotation of the employees tour of duty such that,
REINSTATED in its fullness and without modifications.
at times, it was Busa who would take over from the
respondent; hence, Busa had the occasion to personally see
17
SO ORDERED.
the respondent connecting illegal calls. In support of this, the
petitioner proffers the copy of logbook entries from July 13
The CA held that Busas sworn statement was not worthy of to August 3, 1994, which was attached to its Memorandum
credence, a mere afterthought, the contents of which were
of Appeal filed with the NLRC. The logbook shows that on
seriously flawed. The appellate court found it difficult to
several occasions, it was Busa who took over from the
believe Busas assertion that, on several occasions when he
respondent.22
came to relieve the respondent, a circuit was in use which
the latter would turn off before leaving. In this regard, the
The petitioner further asserts that the respondent failed to
appellate court noted that Busas work shift preceded that of show that Busa was actuated and impelled by improper
the respondent, such that it would be impossible for him to
motive and bad faith in executing his sworn statement. 23 The
see the respondent make an illegal connection. 18
records show that Busa, from the very start, had
categorically and unequivocally named the respondent as
The CA likewise opined that the respondent was denied due one of those engaged in the illegal connection of overseas
process when he was not apprised of nor given the
calls.24 Moreover, Busas sworn statement had been
On April 16, 2004, the CA reinstated the decision of the
Labor Arbiter, thus:

corroborated by the printout of the CAMA tapes (which


disclosed that during the respondents August 1, 1994 duty,
three fraudulent calls to Saudi Arabia were illegally made), 25
as well as Cayanans sworn statement implicating the
respondent.26
The petitioner submits that the respondents offense was
serious in character and merits the penalty of dismissal from
employment. It contends that the respondent was accorded
the full measure of due process before he was dismissed: he
was given a notice which apprised him of the charge against
him and required him to explain why he should not be
dismissed, and later, a notice of termination. The petitioner
claims that the Labor Code simply requires that the
employee be given a written notice containing a statement of
the causes of termination. It insists that the printout of the
recording of the CAMA tapes showing that three illegal
connections were made on August 1, 1994 is a mere
evidentiary matter that need not be mentioned in the notice. 27
For his part, the respondent avers that Busas statement was
uncorroborated and hearsay for lack of cross-examination.
He insists that Busa could not have seen him make illegal
connections since the latters shift came before his. 28
The petitioner replies that an affidavit may be admissible
even if the witness is not presented during trial because
technical rules are not strictly followed in proceedings
before the Labor Arbiter and the NLRC.29
The petition has no merit.
It is a settled rule that factual findings of labor officials, who
are deemed to have acquired expertise in matters within their
respective jurisdictions, are generally accorded not only
respect but even finality.30 Moreover, in a petition for review
on certiorari under Rule 45, the Supreme Court reviews only
errors of law and not errors of facts.31 However, where there
is divergence in the findings and conclusions of the NLRC,
on the one hand, from those of the Labor Arbiter and the

Court of Appeals, on the other, the Court is constrained to


examine the evidence.32
In termination cases, the burden of proof rests upon the
employer to show that the dismissal is for just and valid
cause; failure to do so would necessarily mean that the
dismissal was illegal.33 The employers case succeeds or fails
on the strength of its evidence and not on the weakness of
the employees defense. If doubt exists between the evidence
presented by the employer and the employee, the scales of
justice must be tilted in favor of the latter.34 Moreover, the
quantum of proof required in determining the legality of an
employees dismissal is only substantial evidence.
Substantial evidence is more than a mere scintilla of
evidence or relevant evidence as a reasonable mind might
accept as adequate to support a conclusion, even if other
minds, equally reasonable, might conceivably opine
otherwise.35

complainants participation in illegal overseas connection.


Complainant also failed to refute respondents evidence that
on August 1, 1994, while he was on duty, three (3) overseas
calls to Saudi Arabia were recorded in cama tape (Annex 4,
p. 30, records).

August 12, 1994


TO : MR. ANTONIO Q. TIAMSON Radio Tech II Clark
TMC
FROM : Division Head, North Luzon Toll Network

However, we consider the penalty of dismissal too harsh


considering that respondent imposed a sixty (60)-day
suspension on Paul Cruzada, a co-employee of complainant
who submitted (sic) culpability. For where a lesser punitive
penalty would suffice, the supreme penalty of dismissal
should be visited (Almira vs. B.F. Goodrich, 58 SCRA 120).
Under the circumstances, reinstatement but without
backwages is appropriate (pp. 39-40, Rollo)

Our review of the records reveals that among the three


employees who issued sworn statements, namely, Busa,
Cayanan and Cruzada, it was only Busa who directly
implicated Tiamson and it was done inexplicably only in his
In this case, the appellate court ruled for respondent
second sworn statement. It does not inspire credence as it
Tiamson, ratiocinating as follows:
comes as an afterthought and the contents are seriously
flawed on material points. Looming large is the claim of
The issues posed by both parties involve the evaluation of
Busa that on several occasions when he came to relieve
the findings of facts by the agencies a quo. While the general Tiamson, he observed that his circuit was logged on and in
rule is that factual issues could not be properly raised and
use, and Tiamson would then put it off before leaving. This
considered in a petition for certiorari, it however admits of
is a canard because the shift of Busa was from 1:00 p.m. to
this exception that a disharmony between the factual
6:00 a.m. and of course ahead of the 6:00 a.m. to 2:00 p.m.
findings of the Labor Arbiter and those of the NLRC opens
shift of Tiamson who came in as his reliever. Their tours of
the door to review thereof by the Supreme Court (Asuncion
duty was in the converse order of what Busa claimed, and so
vs. National Labor Relations Commission, 362 SCRA 56),
he spoke with a forked tongue when he stated that Tiamson
including, of course, the Court of Appeals.
at the preceding shift had his circuit logged on and switched
this off when he left.
The crux of both petitions is whether the NLRC with its
findings quoted below, was correct in setting aside the
A no less important point is the undisputed fact that Tiamson
disposition of the Labor Arbiter:
was not given the opportunity to confute the charge that on
August 1, 1994 while he was on duty, three (3) overseas calls
We disagree that respondent failed to present evidence
to Saudi Arabia were recorded in the cama tape. This was not
linking complainant to the illegal connection scam. As
indicated in the memorandum sent to him on August 12,
pointed out by the respondent, co-employee Busa and
1994, the full text of which reads:
Cayanan in the course of their investigation implicated

SUBJECT: ADMINISTRATIVE CASE


-------------------------------------------Investigation of the complaint indicated hereunder disclosed
that:
1. Complainant Mr. Anthony Dy, President DATELCOM
Corp.
2. The decrease of toll revenue for DATELCOM
Angeles/Mabalacat Exchange due to fraudulent overseas call
scam was complained and notified by Mr. A. Dy to Mrs. H.
G. Gendrano Clark Exchange Division Head on July 26,
1994.
3. The complainant requested assistance to NBI and PLDT
QCI to apprehend the personnel responsible for the illegal
connection.
4. A clue was provided by Mr. Anthony Dy that the illegal
overseas call was coming from Clark-TMC through taped
and equipment monitoring.
5. In the QCI investigation, you were implicated by your
fellow Radio Technician Mr. Vidal C. Busa as involved in
the case. You admitted you know how to operate the Lenkurt
26600 Signalling Test Set to initiate a call but denied doing it
for personal gain or interest but you failed to report the
anomaly to your superior as one of your supervisors was
involved in the fraudulent case.

The acts described above are in violation of the Companys


rules and regulations and is punishable with dismissal from
the service.
In view of the above, please explain in writing within 72
hours from receipt hereof why you should not be dismissed
from the service for the acts described above. You may elect
to be heard if you so desire.

substantial evidence that there was just cause for the


respondents dismissal. To bolster such contention, the
petitioner adduces the following documentary evidences: (1)
the sworn statements of Vidal Busa specifically implicating
the respondent;
(2) the sworn statement of Arnel Cayanan; and (3) the
printout of the CAMA tape, recording the unauthorized
overseas calls originating from Clark-TMC during the
respondents tour of duty.

Please be informed also that you will be placed under


preventive suspension which will take effect on August 16,
1994 pending resolution of the case.

The respondent disputes the admissibility of Busas sworn


statements for being hearsay since the latter was not
presented for cross-examination. This argument, however, is
If no written explanation is received from you within the
not persuasive because the rules of evidence are not strictly
said period of 72 hours, this case will be decided on the basis observed in proceedings before administrative bodies like
of the evidence on hand. (p. 227, Rollo)
the NLRC where decisions may be reached on the basis of
position papers only.37
(SGD.)
The Court agrees with the contentions of the respondent and
ARMANDO A. ABESAMIS
the findings and rulings of the CA.

T 27 - Paano mo naman nasisiguro ito?


S - Nakikita ko po.
T 28 - Paano mo naman nakita samantalang magka-iba ang
tour of duty
ninyo?
S - Pag nag-relyebo kami ay naaabutan kong naka-engage
ang circuit at pag tinanong ko ay sinasabi nga nilang may
tawag sila at kasalukuyang nag-uusap ang magkabilang
parties.38
During the confrontation between Busa and the respondent,
the former likewise made the following statements:
T 3 - Ayon sa iyo, ginagawa rin ni Mr. Tiamson ang magkukunekta ng mga illegal na tawag overseas sa pamamagitan
ng pag-gamit ng inyong Radio Equipment. Tama ba ito?

Procedural due process requires that an employee be


apprised of the charge against him, given reasonable time to
answer the same, allowed ample opportunity to be heard
and defend himself, and assisted by a representative if the
employee so desires (Concorde Hotel vs. Court of Appeals,
362 SCRA 583; underlining supplied). Procedural due
process requires that the employer serve the employees to be
dismissed two (2) written notices before the termination of
their employment is effected: (a) the first, to apprise them of
the particular acts or omission for which their dismissal is
sought; and (b) second, to inform them of the decision of the
employer that they are being dismissed (Perpetual Help
Credit Cooperative, Inc. vs. Faburada, 366 SCRA 693;
underlining supplied). The Labor Arbiter, therefore, was
correct in ruling that Tiamson was indeed illegally dismissed
from his employment.36

S - Tama po, Sir.


The petitioner indeed failed to adduce substantial evidence
to prove that the dismissal of the respondent was for a just
T 4 - Paano mo nalaman na ginagawa rin ni Mr. Tiamson
cause. In his first sworn statement, Busa implicated the
respondent in the illegal connections of overseas calls in this ito?
manner:
S - Dahil nakikita ko siyang nagkukunekta at ilang beses ko
T 25 - Bukod sa iyo, sinu-sino pa sa mga kasamahan mo ang ring nadatnan kapag nag-relyebo kami na gumagana ang
circuit na ang ibig sabihin ay may nag-uusap. At bago siya
tinuruan ni Mr. Cayanan ng sistemang ito?
aalis ay inilalagay niya sa normal position ang linyang
ginamit niya.
S - Sina Antonio Tiamson at Paul Cruzada na pawang mga
Radio Technicians din.
T 5 - Kailan pa ito gingawa ni Mr. Tiamson kung
T 26 - Ang ibig mo sabihin, ginagawa din nina Mr. Tiamson natatandaan mo pa?
at Cruzada
S - Sa natatandaan ko ginagawa niya ito magmula noong
1992 pa.
ang magpa-patch ng mga tawag sa abroad o overseas?

The petitioner maintains that contrary to the findings and


conclusions of the appellate court, it has established through

S - Opo.

T 6 - Ayon pa rin sa iyo, alam din ni Mr. Tiamson na


ginagawa rin ni Mr. Cayanan itong mga illegal activities na

ito. Paano mo nasabi na alam ni Mr. Tiamson itong ginagawa nature) should be rejected as evidence without any rational
ni Mr. Cayanan
probative value even in administrative proceedings. 44

SO ORDERED.

(MGG Marine Services, Inc., et al. vs. NLRC and E.A.


Molina, G.R. No. 114313, July 29, 1996)
S - Kasi magkakasama kami at kaming apat lang nina Mr.
Thus, in Uichico v. National Labor Relations Commission,
Cayanan, Mr.Tiamson, Mr. Cruzada at ako ang nakaka-alam the Court elucidated the extent of the liberality of procedure As Mr. Justice Panganiban, in an en banc decision, states:
To constitute a completely valid and faultless dismissal, it is
niyang operation na iyan.39
in administrative actions:
well-settled that the employer must show not only sufficient
ground therefor, but it must also prove that it observed
On the other hand, during the confrontation among all four
It is true that administrative and quasi-judicial bodies like procedural due process by giving the employee two notices:
employees implicated in the matter, Cayanan testified that he the NLRC are not bound by the technical rules of procedure one, of the intention to dismiss, indicating therein his acts or
was aware that his "subordinates" were engaged in illegal
in the adjudication of cases. However, this procedural rule
omissions complained against, and two, notice of the
decision to dismiss; and an opportunity to answer and rebut
activities. However, he failed to specifically mention who
should not be construed as a license to disregard certain
40
the charges against him, in between such notices.
these subordinates were.
fundamental evidentiary rules. While the rules of evidence
prevailing in the courts of law or equity are not controlling
G.R. No. 111173 September 4, 1996
Although admissible in evidence, affidavits being selfin proceedings before the NLRC, the evidence presented
serving must be received with caution. This is because the
before it must at least have a modicum of admissibility for it
PHILIPPINE SAVINGS BANK, petitioner,
adverse party is not afforded any opportunity to test their
to be given some probative value. 46
vs.
veracity.41 By themselves, generalized and pro forma
affidavits cannot constitute relevant evidence which a
The decisions of this Court, while adhering to a liberal view NATIONAL LABOR RELATIONS COMMISSION and
VICTORIA T. CENTENO, respondents.
reasonable mind may accept as adequate.42 There must be
in the conduct of proceedings before administrative
some other relevant evidence to corroborate such affidavits. agencies, have nonetheless consistently required some proof
of authenticity or reliability as a condition for the admission
On this point, the petitioner submits that the printout of the
of documents.47 Absent any such proof of authenticity, the
MENDOZA, J.:
CAMA tapes corroborated Busas sworn statement. A
printout of the CAMA tape should be considered
perusal of the printout, however, shows that it is not
inadmissible, hence, without any probative weight.
This is a petition for certiorari to annul the decision
authenticated by the proper officer of the company.
of the National Labor Relations Commission in
Moreover, the name of the respondent and the other
To conclude, the petitioner has not established by substantial
NLRC Case No. RB-IV-2-1554-85, affirming the
annotations in the said printout are handwritten and
evidence that there was just cause for the respondents
decision of the Labor Arbiter, which found petitioner
unsigned.
termination from his employment. The sworn statements of
guilty of illegal dismissal, and the resolution of the
Busa and Cayanan alone are not sufficient to establish that
NLRC denying reconsideration.
The ruling in Asuncion v. National Labor Relations
the respondent was guilty of serious misconduct. In light of
Commission43 is instructive on how such document should be such finding, there is no need to delve into whether or not
The facts are as follows:
treated. In that case, the employer submitted a handwritten
the respondent was afforded due process when he was
listing and computer printouts to establish the charges
dismissed by the petitioner.
Private respondent Victoria T. Centeno started, as a
against the employee. The handwritten listing was not
bank teller of petitioner Philippine Savings Bank, on
signed, and while there was a computer-generated listing, the WHEREFORE, premises considered, the petition is
November 3, 1965. Through the years she was
entries of time and other annotations therein were also
DENIED DUE COURSE. The Decision of the Court of
promoted, becoming on February 4, 1985, assistant
handwritten and unsigned. The Court ruled that the
Appeals dated April 16, 2004, and its Resolution dated July
cashier of petitioner's Taytay branch, at a salary of
handwritten listing and unsigned computer printouts were
27, 2004 in CA-G.R. SP Nos. 51855 and 52247 are
P2,672.00 a month.
unauthenticated, hence, unreliable. Mere self-serving
AFFIRMED.
evidence (of which the listing and printouts are of that
45

From September 17, 1984 to November 15, 1984,


private respondent was acting branch cashier,
substituting for Mrs. Victoria Ubaa, who had gone
on maternity leave. As acting branch cashier, private
respondent was in charge of the cash in the vault and
the preparation of the daily cash proof sheet, which
was a daily record of the cash in the vault and was
used as basis in determining the starting balance on
the next banking day.
On November 16, 1984, Mrs. Victoria Ubaa
reported back to work. Before turning over the cash
to Mrs. Ubaa, private respondent Centeno
deposited P356,400.00 in the Metropolitan Bank and
Trust Co. (Metrobank). However, what appeared as
amount deposited in the November 16, 1984 cash
proof and batch sheets of the cashier and clearing
clerk, was P371,400.00, and not P356,400.00 as
shown in the Metrobank passbook. Petitioner later
charged that private respondent falsified the deposit
slip and made it appear that she had deposited
P371,400.00 when actually she had deposited only
P356,400.00.
On December 18, 1984, the branch accounting clerk,
Lolita Oliveros, discovered a discrepancy between
the cash deposit recorded (P371,400.00) in the cash
proof and batch sheets and the deposit actually made
(P356,400.00) as reflected in the Metrobank
passbook. She called the attention of the clearing
clerk, Alberto C. Jose, to the matter. They reviewed
the records and found that what had been attached to
the debit ticket of Jose was a deposit slip for
P356,400.00, and not for P371,400.00.
An audit team reviewed the account of the branch
and found a P15,000.00 shortage incurred on
November 16, 1984, the day private respondent
turned over her accountability to Mrs. Ubaa after
the latter's maternity leave.

A committee was formed to investigate the shortage.


Private respondent, the branch manager, Eladio C.
Laurena, the cashier, Victoria N. Ubaa, the clearing
clerk, Alberto C. Jose, and two other employees
were called to the investigation. The committee
found private respondent accountable for the
shortage. 1 Hence, on January 7, 1985, private
respondent was given a memorandum which stated:
In connection with the shortage of
P15,000.00 at Taytay Branch which has
been recently discovered by the Auditing
Department which shortage appears to have
been deliberately perpetuated through
falsifications of various documents, all of
which appear to have been done by you, you
are hereby required to submit your
explanation within seventy two (72) hours
from receipt of this memo why no
administrative and/or disciplinary action
shall be taken against you.
In the meantime, you are hereby
preventively suspended for a period of thirty
(30) days effective January 8, 1985.
(Emphasis added)
The manager, cashier, clearing clerk and a teller,
were also given "show-cause" memoranda, but only
private respondent was placed under preventive
suspension.
All those required to show cause filed their
respective answers, except private respondent.
Instead she requested the bank's vice-president,
Antonio Viray, on January 15, 1985, to give her until
January 18, 1985 within which to file her answer on
the ground that she needed to consult her lawyer.
Her request was granted but private respondent
nonetheless failed to answer the charges against her.

On February 4, 1985, private respondent was


dismissed by the bank. The memorandum to her
read:
Memorandum
To : MS. VICTORIA T. CENTENO
Assistant Cashier
Taytay Branch
This is in connection with the shortage of
P15,000.00 at Taytay Branch which was
incurred while your were in charge of the
vault. Immediately after the discovery of the
shortage, through the memorandum of the
undersigned dated January 7, 1985
addressed to you, we required you to explain
within seventy two (72) hours from receipt
of said memo why no administrative and/or
disciplinary action should be taken against
you. Despite the lapse of the extension
period you requested within which to submit
your explanation, and up this date, you have
not submitted your explanation.
After carefully evaluating the evidence
presented and considering your failure to
explain the shortage which tantamounts to
admission of guilt, we have no alternative
but to conclude, as we hereby conclude, that
you were the one who misappropriated the
shortage of P15,000.00. You have therefore
forfeited the confidence that the Bank has
reposed on you as an officer.
IN VIEW OF THE FOREGOING,
Management hereby dismisses you FOR
CAUSE effective immediately with
forfeiture of all benefits. The Bank reserves
the right to take such actions it may deem

necessary for the recovery of the


P15,000.00. (Emphasis added)
Private respondent sued petitioner for illegal
dismissal before the Labor Arbiter. Aside from
claiming that her dismissal was without basis, she
claimed that she was denied due process because she
had not been informed of the specific acts for which
she was dismissed. She claimed that during her 19
years of service in petitioner bank, she never
"[played] fast and loose with bank funds."
Petitioner alleged that private respondent was
dismissed for loss of trust and confidence as a result
of the shortage, which, according to petitioner, she
tried to conceal by falsifying the bank's cash proof
sheet and teller's vale. Petitioner claimed that private
respondent was accorded due process prior to her
dismissal.
On September 15, 1988, the Labor Arbiter found
petitioner guilty of having illegally dismissed private
respondent and of denying her due process.
Accordingly the Labor Arbiter ordered:
WHEREFORE, responsive to the foregoing,
judgment is as it is hereby entered in favor
of complainant and against respondent:
1. Considering the termination of
complainant illegal;
2. Ordering respondent to reinstate
complainant to her former position or
equivalent position with full backwages
from the time of her unlawful termination
and until actually reinstated without loss of
seniority rights and other privileges
appertaining to her position;

3. Ordering respondent to pay complainant


moral and exemplary damages in the
amounts of Fifty Thousand Pesos
(P50,000.00) and Ten Thousand Pesos
(P10,000.00), respectively; and,
4. Ordering respondent to pay complainant
attorney's fees equivalent to ten (10%) per
cent of the total award.
SO ORDERED.
On appeal, the NLRC affirmed with modification
thus:
PREMISES CONSIDERED, the Decision of
September 15, 1988 is hereby MODIFIED
with the deletion of awards representing
moral/exemplary damages and attorney's
fees. However, the award of backwages and
other benefits shall not exceed three (3)
years as laid down by the Supreme Court.
Respondent is hereby directed to pay
complainant backwages in the amount of
NINETY SIX THOUSAND ONE
HUNDRED NINETY TWO PESOS
(P96,192.00) and/or other benefits due. The
other findings stand AFFIRMED.
SO ORDERED.
Both parties move for reconsideration, but their
motions were denied by the NLRC in its resolution
on July 8, 1993.
Hence this petition. Petitioner claims that the NLRC
gravely abused its discretion in:
a) holding that private
respondent Centeno was

denied due process of law


prior to her dismissal; and
b) failing to fully discuss all
the six (6) assigned errors
raised by the petitioner in its
appeal by ignoring:
1) the valid
ground
wherein
petitioner
based its
termination
of the
service of
private
respondent,
and that is
loss of
confidence;
2) the
specific
circumstanc
es that led
the
petitioner to
lose its trust
and
confidence
on private
respondent;
and
3) the
applicable
settled law
and
jurispruden

ce that the
private
respondent,
having been
validly
dismissed,
is not
entitled to
reinstateme
nt and
backwages.
First. Contrary to the finding of the Labor Arbiter
and the NLRC, private respondent was notified of
the charge against her through a memorandum sent
to her on January 7, 1985. Indeed she knew the
reason for the "show-cause" order because before
that, she and other employees had been asked to
attend an investigation. The law requires that the
employer must furnish the worker sought to be
dismissed with two (2) written notices before
termination may be validly effected: first, a notice
apprising the employee of the particular acts or
omission for which his dismissal is sought and,
second, a subsequent notice informing the employee
of the decision to dismiss him. 2 In accordance with
this requirement, private respondent was given the
required notices, on January 7, 1985 and then on
February 4, 1985.
The NLRC ruled that an investigation should have
been conducted prior to private respondent's
dismissal. As already noted, however, private
respondent was informed of the charges against her
and given an opportunity to answer the charges.
Upon her request, she was given until January 18,
1985 within which to file her answer. But she failed
to file her answer. Of course she later tried to explain
that she did not find it necessary to do so because
"there was, after all, no ground for any action against

[her] . . . and [she] did not feel obligated, therefore,


to dispute the action which was baseless and
unfounded." 3 Furthermore, she claimed she thought
"the Committee had prejudged the case against her."
4

Whatever her reason might have been, the fact is that


petitioner waive the right to be heard in an
investigation. Due process is not violated where a
person is not heard because he has chosen not to
give his side of the case. If he chooses to be silent
when he has a right to speak, he cannot later be
heard to complain that he was silenced. 5 Private
respondent having chosen not to answer, should not
be allowed to turn the tables on her employer and
claim that she was denied due process. Indeed, the
requirement of due process is satisfied when a fair
and reasonable opportunity to explain his side of the
controversy is afforded the party. A formal or trialtype hearing is not at all times and in all
circumstances essential, especially when the
employee chooses not to speak. 6 Under the
circumstance of this case, it is too much to require
petitioner to hear private respondent before the latter
can be dismissed.
Happily, no liability was imposed on petitioner by
either the Labor Arbiter or the NLRC despite the
finding that petitioner had denied private respondent
due process. Accordingly, all that we need to do in
this case is to record our finding that petitioner fully
complied with its duty under the law to accord due
process to private respondent.
Second. Petitioner also claims that the NLRC
gravely abused its discretion in not passing upon
three (3) errors assigned by it on appeal.
We find the contention without merit. In affirming
the Labor Arbiter, the NLRC found the evidence

supporting the Labor Arbiter's factual findings to be


substantial and for this reason apparently found it
unnecessary to make a separate discussion. Factual
findings of administrative agencies are generally
accorded respect and even finality in this Court if
they are supported by substantial evidence. 7
Petitioner makes a "reconstruction" of the facts
which, according to it, shows how the shortage
incurred on November 16, 1984 was concealed. The
"reconstruction" is as follows:
A) During the turn-over of the cash in vault
by Mrs. Victoria Centeno to Mrs. Victoria
Ubaa, after counting the cash in vault, no
formal recording of how much cash was
actually turned over was done. However,
from the Cash Proof in November 16, 1984,
it could be reconstructed and determined
whether there is a shortage or not by the
following figures:
Cash Balance, Nov. 15 '84 P589,572.02
Deduct: Cash Vales of
Tellers at start of banking day
Pico of Teller No. 1 P19,207.06
Pico of Teller No. 2 P21,666.21
Pico of Teller No. 3 P21,995.25
[P62,868.52]
Balance: Paper Bills & Coins P536,703.50
Deduct: Deposit with Metrobank
P356,400.00
Balance that should have been
turned over P170,303.50
Additional Vale-Coins-Teller 3 P11.00
Balance P170,292.50

Add: Sorted Paper Bills turned


over by the tellers to the
cashiers:
Teller No. 1 P100,000.00
Teller No. 1 P40,000.00
Teller No. 2 P147,200.00
[P46,202.64] [P333,402.64]
CORRECT CASH BALANCE
November 16, 1984 P503,695.14
Cash Balance per Cash
Proof November 16, 1984 P488,695.14
SHORTAGE P15,000.00
B) The above P15,000.00 shortage was
covered up in two (2) ways or stages:
First: In the original
or untampered cash
proof, the deposit to
Metrobank was
written originally as
P371,400.00 instead
of the actual deposit
of P356,400.00. The
writing of the
P371,400.00
deposit to
Metrobank was
based on a deposit
slip for P371,400.00
given to Mrs.
Victoria Ubaa by
Mrs. Victoria
Centeno. The same
deposit slip for
P371,400.00 was
also given to Mr.
Alberto Jose, the

Clearing clerk, who


used the same to
enter the
P371,400.00
deposit with
Metrobank in the
Batch Sheet, as well
as in the preparation
of the Debit and
Credit Tickets. The
P15,000.00
shortage, which is
the difference
between
P371,400.00 and
P356,400.00 was
therefore concealed
in the P371,400.00
deposit to
Metrobank, which
actually and truly
was for
P356,400.00 only.
Second: When the
genuine deposit slip
to Metrobank for
P356,400.00 was
placed in the Cash
Proof file and the
spurious deposit
slip for P371,400.00
was removed by
whoever was
responsible for the
shortage, the
cashproof will NOT
BE BALANCED, so
that the second step
was to ADD

P15,000.00 to the
P11.00 cash vale of
Teller No. 3 to
make it appear as if
the vale was for
P15,011.00. In this
way, the cash proof
will again be
balanced, since the
decease of
P15,000.00 in
deposit with
Metrobank from
P371,400.00 to
P356,400.00 was
shifted to the
P15,011.00 vale
which was actually
P11.00 only.
Though the P371,400.00 deposit slip is now
missing, the insertion of the P15,000.00 in
the vale of Teller No. 3 is very apparent,
since the duplicate vale in the possession of
the Teller has not been tampered and
remains as P11.00. Incidentally, it has not
missed the petitioner's attention also that, by
force of habit, Teller No. 3 was accustomed
to placing a "hyphen" across the centavo
figures in her Teller's vales when there was
no centavo entry thereon; the added figures
amounting to P15,000.00 on the other hand
did not contain such a "hyphen" in the
centavo of the vale, leading us to believe
that the addition of P15,000.00 could not
have been made by the Teller concerned.
(Affidavit of Norberto Robleza date 09
October 1985, pp. 4-6)

Loss of trust and confidence is a cause for dismissing an


employee who is entrusted with fiducial matters, or with the
custody, handling or care and protection of the employer's
property. 8 There is no dispute about this. But the employer
must clearly and convincingly establish the facts and
incidents upon which its loss of confidence in the employee
may be fairly made to rest, otherwise, the dismissal will be
rendered illegal. 9
Petitioner's claim is that although private respondent
deposited only P356,400.00 in the Metrobank, she filed up a
deposit slip showing the deposit to be P371,400.00 and this
amount was recorded in the cash proof sheet and batch sheet
for November 16, 1984. But there is no evidence to show
this. The falsified deposit slip allegedly made by privates
respondent was not presented. Petitioner claimed it was
missing. But as private respondent testified the amount of
P356,400.00 which she deposited was recorded in the
Metrobank passbook. She gave this passbook to Mrs. Ubaa
on November 16, 1984. Yet the supposed discrepancy was
not noticed by Mrs. Ubaa in preparing the cash proof sheet
and the debit sheet who recorded P371,400.00 as having
been deposited in Metrobank. Petitioner's allegation that Ms.
Centeno misled the cashier and the clearing clerk into
recording P371,400.00 cannot therefore be given credence.

Reyes) was P15,011.00 when the fact was that it was


only for P11.00 as shown in the duplicate vale in the
possession of Reyes. This claim is subject to two
objections. First, it was not shown that private
respondent had custody of the vale or, if she had
access to the document, that private respondent was
the only one who had such access to it, so as to make
her the only possible author of the alteration.
Second, the fact that the altered vale of Teller 3 in
the possession of the bank was not in the teller's
customary way of recording does not necessarily
mean that the vale she had was the authentic vale
while that given to the clearing clerk was falsified.
She could have altered her usual practice of
recording.
It is noteworthy that the shortage was incurred on
the day (November 16, 1984) the branch regular
cashier, Mrs. Victoria Ubaa, reported for work. It
was she in fact who prepared the cash proof sheet.
The alteration in the cash proof sheet on that day
could not have been made by private respondent. As
an NBI handwriting expert stated under cross
examination:
WITNESS

Indeed, private respondent denied that she gave Mrs. Ubaa


a falsified deposit slip showing a deposit of P371,400.00
because after the Metrobank picked up the deposit she made,
private respondent handed to Mrs. Ubaa the deposit slip of
P356,400.00 together with the cash proof sheet of November
15, 1984 and the key to the vault. 10 Besides, Mrs. Ubaa as
already stated, had the passbook. She could not have failed
to notice that the amount deposited was P356,400.00 and not
P371,400.00 as the bank now claims it was made to
understand on November 16, 1984.
Petitioner claims that the party responsible for
concealing the shortage altered the teller's vale and
made it appear that the vale of Teller 3 (Antonette

A The supplemental report


is also an answer to the first.
The requested analysis
could center on the
handwritings of the two (2)
persons, Mrs. Victoria
Ubaa and Mrs. Victoria
Centeno. In my first report
dated December 3, 1985
may findings are as follows:
The no. 1 states that there
are existing fundamental
differences between the

questioned handwritings or
figures appearing on the
questioned document and
the standard
handwritings/figures
appearing on the standard
documents marked as "SV1" thru "SV-9" and those
standards were the
handwritings of one Victoria
Ubaa. The result of which
is that the questioned
handwritings and the
standard handwritings were
not written by one and the
same person. And then in
statement that the submitted
standards, signatures under
the specimen named
Victoria Centeno any
findings whether Victoria
Centeno or not is the writer
of the questioned
handwritings, so I made the
supplemental report to
make a definite answer that
all the figures and
handwritings appearing on
the cash proof sheet which
is being questioned were
not written by Victoria
Centeno to answer this
phrase. (Emphasis supplied)
Furthermore, the cash proof sheet and the vale were
kept in the bank's vault, the key to which was held
only by Mrs. Ubaa, as cashier of the bank. 11 Any
alteration in the documents by private respondent or
by any party could, therefore, have easily been
discovered by the cashier.

Petitioner further claims that private respondent's


accounting method did not correctly reflect the bills
from previous banking days and that taking into
account all the entries, the amount not reflected was
equivalent to the shortage. This contention is without
merit. While the accounting method adopted by
private respondent was different from the method
used by Mrs. Ubaa, private respondent's method
was nonetheless an acceptable bank procedure
according to Mrs. Robleza, petitioner's own witness.
12
The method adopted by private respondent was
accurate, otherwise it could not have been allowed
by the bank.
Indeed private respondent was acting cashier for two
months, from September 17, 1984 to November 15,
1984. During that period no shortage was ever
reported. At the time the cash in the vault was turned
over the Mrs. Ubaa, it was counted and the failure
to record its amount at that time can only mean one
thing: that the cash turned over to Mrs. Ubaa
corresponded with the amount recorded in the cash
proof sheet on November 15, 1984.
Private respondent had faithfully served petitioner
bank for 19 years. Starting as a bank teller, she
steadily rose to the position of assistant branch
cashier. Considering this fact, petitioner should have
been more careful in determining liability for the
loss rather than merely relying on what it calls
circumstantial evidence of guilt. The fact that only
private respondent did not answer the charge when
required in the memorandum of petitioner is not an
indication of her guilt. While we recognize that
petitioner has a wide latitude in dismissing a bank
officer, nonetheless, the evidence on which it acts
must be substantial.
As the dismissal of private respondent is illegal, she
is entitled to reinstatement to her former position

without loss of seniority rights and to the payment to


her of backwages. 13 The NLRC correctly limited the
award of backwages to three years, consistent with
the rule at the time of private respondent's dismissal.
14
R.A. No. 6715, which amended Art. 279 of the
Labor Code, awarding full backwages to illegally
dismissed employees, cannot be retroactively
applied to dismissals taking place before its
effectivity on March 21, 1989. 15
WHEREFORE, the petition is DISMISSED.
SO ORDERED.
G.R. No. 114290 September 9, 1996
RAYCOR AIRCONTROL SYSTEMS, INC., petitioner,
vs.
NATIONAL LABOR RELATIONS COMMISSION and
ROLANDO LAYA, et al., respondents.

PANGANIBAN, J.:
Were private respondents, employed by petitioner in its
business of installing airconditioning systems in buildings,
project employees or regular employees? And were their
dismissals "due to (petitioner's) present status" and effective
the day following receipt of notice legal? Where both the
petitioner and the respondents fail to present sufficient and
convincing evidence to prove their respective claims, how
should the case be decided?
This Court answers the foregoing questions in resolving this
petition for certiorari assailing the Decision 1 promulgated
November 29, 1993 by the National Labor Relations
Commission, 2 which set aside and reversed the decision of
the labor arbiter 3 dated 22 January 1993, as well as the

subsequent order of respondent Commission denying


petitioner's motion for reconsideration.
The Facts
Petitioner's sole line of business is installing airconditioning
systems in the building of its clients. In connection with such
installation work, petitioner hired private respondents
Roberto Fulgencio, Rolando Laya, Florencio Espina,
Romulo Magpili, Ramil Hernandez, Wilfredo Brun, Eduardo
Reyes, Crisostomo Donompili, Angelito Realingo, Hernan
Delima, Jaime Calipayan, Jorge Cipriano, Carlito de
Guzman, Susano Atienza, and Gerardo de Guzman, who
worked in various capacities as tinsmith, leadman, aircon
mechanic, installer, welder and painter. Private respondents
insist that they had been regular employees all along, but
petitioner maintains that they were project employees who
were assigned to work on specific projects of petitioner, and
that the nature of petitioner's business mere installation
(not manufacturing) of aircon systems and equipment in
buildings of its clients prevented petitioner from hiring
private respondents as regular employees. As found by the
labor arbiter, their average length of service with petitioner
exceeded one year, with some ranging from two six years
(but private respondents claim much longer tenures, some
allegedly exceeding ten years).
In 1991, private respondent Laya and fourteen other
employees of petitioner filed NLRC NCR Case No. 00-0302080-92 for their "regularization". This case was dismissed
on May 20, 1992 for want of cause of action. 4
On different dates in 1992, they were served with uniformlyworded notices of "Termination of Employment" by
petitioner "due to our present business status", which
terminations were to be effective the day following the date
of receipt of the notices. Private respondent felt they were
given their walking papers after they refused to sign a
"Contract Employment" providing for, among others, a fixed
period of employment which "automatically terminates

without necessity of further notice" or even earlier at


petitioner's sole discretion.
Because of the termination, private respondents filed three
cases of illegal dismissal against petitioner, alleging that the
reason given for the termination of their employment was
not one of the valid grounds therefor under the Labor Code.
They also claimed that the termination was without benefit
of due process.
The three separate cases filed by private respondents against
petitioner, docketed as NLRC-NCR 00-03-05930-92, NLRC
NCR 00-05-02789-92, and NLRC NCR 00-07-03699-92,
were subsequently consolidated. The parties were given
opportunity to file their respective memoranda and other
supplemental pleadings before the labor arbiter.
On January 22, 1993, the Labor Arbiter issued his decision
dismissing the complaints for lack of merit. He reasoned that
the evidence showed that the individual complainants
(private respondents) were project employees within the
meaning of Policy Instructions No. 20 (series of 1977) 5 of
the Department of Labor and Employment, having been
assigned to work on specific projects involving the
installation of air-conditioning units as covered by contracts
between their employer and the latter's clients. Necessarily,
the installation of airconditioning systems "must come to a
halt as projects come and go", and "(o)f consequence, the
[petitioner] cannot hire workers in perpetuity. And as project
employees, private respondent would not be entitled to
termination pay, separation pay, holiday premium pay, etc.;
and neither is the employer required to secure a clearance
from the Secretary of Labor in connection with such
termination.
Private respondents appealed to the respondent NLRC,
which in its November 29, 1993 Decision reversed the
arbiter and found private respondent to have been regular
employees illegally dismissed. The respondent Commission
made the following four-paragraph disquisition:

From the above rules, it can easily be


gleaned that complainants belong to a work
pool from which the respondent company
drew its manpower requirements. This is
buttressed by the fact that many of the
complainants have been employed for long
periods of time already.
We doubt respondent's assertion that
complainants were really assigned to
different projects. The "Contract
Employment" which it submitted (see pp.
32-38, record) purporting to show particular
projects are not reliable nay even appears to
have been recently typewritten. In the
"Contract Employment" submitted by
complainants (see p. 65, record), no such
name of project appears. Verily,
complainants were non-project employees.
Anent the dismissal of complainants, suffice
it to state that the same was capricious and
whimsical which as shown by the vague
reason proffered by respondent for said
dismissal which is "due to our present
business state" (should read "status") is
undoubtedly not one of the valid causes for
termination of an employment. We are thus
inclined to give credence to complainants'
allegation that they were eased out of work
for their refusal to sign the one-sided
"Contract Employment".
The fact that complainants were dismissed
merely to spite them is made more manifest
by respondent's failure to make a report of
dismissal or secure a clearance from the
Department of Labor (see pp. 196 and 197,
record) as required under P.I. No 20 and
their publication of an advertisement for

replacements for the same positions held by


complainants (see p. 198, record). Even
assuming that complainants were project
employees, their unceremonious dismissal
coupled with the attempt to replace them via
the newspaper advertisement entitles them
to reinstatement with backwages under P.I.
No. 20.
The dispositive portion followed immediately and read:
WHEREFORE, the appealed Decision is
hereby SET ASIDE and a new one entered
ordering respondent to:
1. Immediately reinstate complainants
(private respondents) to their former
positions without loss of seniority rights and
privileges; and
2. Pay them full backwages from the time
they were dismissed up to the time they are
actually reinstated.
Petitioner's motion for reconsideration was denied by public
respondent on February 23, 1994 for lack of merit. Hence,
this petition.
Issues
Petitioner charges public respondent NLRC with grave abuse
of discretion in finding private respondent to have been nonproject employees and illegally dismissed, and in ordering
their reinstatement with full backwages.
For clarity's sake, let us re-state the pivotal question involved
in the instant case as follows: whether private respondents
were project employees or regular (non-project) employees,
and whether or not they were legally dismissed.

In support of its petition, petitioner reiterates the same points


it raised before the tribunals below: that it is engaged solely
in the business of installation of airconditioning units or
systems in the building of its clients. It has no permanent
clients with continuous projects where its workers could be
assigned; neither is it a manufacturing firm. Most of its
projects last from two to three months. (The foregoing
matters were never controverted by private respondents.)
Thus, for petitioner, work is "not done in perpetuity but
necessarily comes to a halt when the installation of
airconditioning units is completed."

contrary to those of the labor arbiter, this Court may choose


to re-examine the same, as we hereby do in this case now.
The First Issue: Project Employees or Regular
Employees?
An Unfounded Conclusion

to other employers then they era project


employees employed by a construction
company in a particular project or in a phase
thereof.
A careful reading of the aforequoted and preceding
provisions establishes the fact that project employees
may or may not be members of a work pool, (that is
the employer may or may not have formed a work
pool at all), and in turn, members of a work pool
could be either project employees or regular
employees. In the instant case, respondent NLRC
did not indicate how private respondents came to be
considered members of a work pool as distinguished
from ordinary (non-work pool) employees. It did not
establish that a work pool existed in the first place.
Neither did it make any finding as to whether the
herein private respondents were indeed free to leave
anytime and offer their services to other employers,
as vigorously contended by petitioner, despite the
fact that such a determination would have been
critical in defining the precise nature of private
respondents' employment. Clearly, the NLRC's
conclusion of regular employment has no factual
support and is thus unacceptable.

We scoured the assailed Decision for any trace of


arbitrariness, capriciousness or grave abuse discretion, and
noted that the respondent Commission first cited the facts of
the case, then quoted part of the arbiter's disquisition along
On the basis of the foregoing, petitioner asserts that it could with relevant portions of Policy Instructions No. 20, after
which it immediately leapt to the conclusion that "(F)rom
not have hired private respondents as anything other than
project employees. It further insists that "(a)t the incipience the above rules, it can easily be gleaned that complainants
of hiring, private respondents were appraised (sic) that their belong to a work pool from which the respondent company
drew its manpower requirements. This is buttressed by the
work consisted only in the installation of airconditioning
fact that many of the complainants have been employed for
units and that as soon as the installation is completed, their
long periods of time already. " (emphasis supplied) By
work ceases and that they have to wait for another
reason of such "finding", respondent NLRC concluded that
installation projects (sic)." In other words, their work was
private respondents were regular (not project) employees,
co-terminous with the duration of the project, and was not
but failed to indicate the basis for such finding and
continuous or uninterrupted as claimed by them. Petitioner
conclusion. For our part, we combed the Decision in search
also claim that the private respondents signed project
contracts of employment indicating the names of the projects of such basis. However, repeated scrutiny of the provisions
of Policy Instruction No. 20 pertaining to work pools merely
or buildings they were working on. And when between
raised further questions.
projects, these project employees were free to work
Conclusion Based on
elsewhere with other establishments.
Members of a work pool from which a
Unwarranted Assumption
construction company draws its project
Private respondents controverted these assertions of
of Bad Faith
employees, if considered employees of the
petitioner, claiming that they had worked continuously for
construction company while in the work
Immediately thereafter, respondent Commission determined
petitioner for several years, some of them as long as ten
pool are non-project employees or
without sufficient basis that complainants were nonyears, and thus, by operation of law had become regular
employees for an indefinite period. If they
project employees. We quote:
employees.
are employed in a particular project, the
completion of the project or of any phase
We doubt respondent's (petitioner's)
The Court's Ruling
thereof will not mean severance of
assertion that complainants (private
Ordinarily, the findings made by the NLRC are entitled to
employer-employee relationship.
respondents) were really assigned to
great respect and are even clothed with finality and deemed
different projects. The "Contract
However, if the workers in the work pool are
binding on this Court, except that when such findings are
Employment" which it submitted (see pp.
free to leave anytime and offer their services
32-38, record) purporting to show particular

projects are not reliable nay even appears to


have been contrived. The names of the
projects clearly appears to have been
contrived. The names of the projects clearly
appear to have been recently typewritten. In
the "Contract Employment" submitted by
complainants (see p. 65, record), no such
name of project appears. Verily,
complainants were non-project employees."
(emphasis supplied)

therefore, petitioner's failure to indicate in the originals of


the contracts the name(s) of the project(s) to which private
respondents were assigned does not necessarily mean that
they could not have been project employees. (Incidentally,
we should make mention here that what is or is not stated in
a contract does not control nor change the juridical nature of
an employment relationship since the same is determined
and fixed by law. As a matter of fact, we note that there is no
requirement in Policy Instructions No. 20 that project
employees should be issued written contracts of
employment, let alone that a written contract should indicate
The basis for respondent NLRC's statement that the contracts the name of the project to which the employee concerned is
were contrived was the fact that the names of projects clearly being assigned.)
appeared to have been typed in only after the contracts had
been prepared. However, our examination of the contracts
Statutory Basis for Determining Nature of
(presented by petitioner as Annexes "A", "B", "B-1", "C",
Employment
6
"D", "E" and "F" to its Position Paper dated July 30, 1992
The parties and their respective counsel, as well as
filed with the labor arbiter)did not lead inexorably to the
respondent Commission and the Solicitor General, should
conclusion that these were "contrived". Said Annexes were
have re-read and carefully studied ALU-TUCP vs. National
photocopies of photocopies of the original "Contract
7
Labor Relations Commission, 8 which is highly instructional
Employment's", and the names of projects had been typed
on this question:
onto these photocopies, meaning that the originals of said
contracts probably did not indicate the project names. But
The law on the matter is Article 280 of the
this alone did not automatically or necessarily mean that
Labor Code which reads in full:
petitioner had committed any falsehood or fraud, or had any
intent to deceive or impose upon tribunals below, because
Article 280. Regular and
the names of the projects could have been typed/filed in
Casual Employment The
good faith, nunc pro tunc, in order to supply the data which
provisions of the written
ought to have been indicated in the originals at the time
agreement to the contrary
those were issued, but which for some reason or other were
notwithstanding and
omitted in short, the names of projects could have been filled
regardless of the oral
in simply in order to make the contracts speak the truth more
agreement of the parties, an
clearly or completely. Notably, no reason was advanced for
employment shall be
not according the petitioner the presumption of good faith.
deemed to be regular where
Respondent NLRC, then made an unwarranted assumption
the employee has been
that bad faith and fraudulent intent attended the filling in of
engaged to perform
the project names in said Annexes. In any event, it can be
activities which are usually
easily and clearly established with the use of the naked eye
necessary or desirable in the
that the dates and durations of the projects and/or work
usual business or trade of
assignments had been typed into the original contracts, and

the employer, except where


the employment has been
fixed for a specific project
or undertaking the
completion or termination
of which has been
determined at the time of
the engagement of the
employee or where the work
or services to be performed
is seasonal in nature and the
employment is for the
duration of the season.
An employment shall be
deemed to be casual if it is
not covered by the
preceding paragraph:
Provided, That, any
employee who has rendered
at least one year of service,
whether such service is
continuous or broken, shall
be considered a regular
employee with respect to
the activity in which he is
employed and his
employment shall continue
while such activity exists. . .
.
xxx xxx xxx
. . . For, as is evident from the provisions of
Article 280 of the Labor Code, quoted
earlier, the principal test for determining
whether particular employees are property
characterized as "project employees" as
distinguished from "regular employees," is
whether or not the "project employees" were

assigned to carry out a "specific project or


undertaking," the duration (and scope) of
which were specified at the time the
employees were engaged for that project.
(emphasis ours)
In the realm of business and industry, we
note that "project" could refer to . . . a
particular job or undertaking that is within
the regular or usual business of the employer
company, but which is distinct and separate
and identifiable as such, from the other
undertakings of the company. Such job or
undertaking begins and ends at determined
or determinable times. The typical example
of this . . . type of project is a particular
construction job or project of a construction
company. A construction company
ordinarily carried out two or more discrete
identifiable construction projects: e.g., a
twenty-five story hotel in Makati; a
residential condominium building in Baguio
City; and a domestic air terminal in ilolo
City Employees who are hired for the
carrying out of one of these separate
projects, the scope and duration of which
has been determined and made known to the
employees at the time of employment, are
properly treated as "project employees," and
their services may be lawfully terminated at
completion of the project.
The same decision goes on to say:

. . . The simple fact that the employment of


petitioners as project employees had gone
beyond one (1) year, does not detract from,
or legally dissolve, their status as project
employees. The second paragraph of Article
280 of the Labor Code, quoted above,

providing that an employee who has served


for at least one (1) year, shall be considered
a regular employee, relates to casual
employees, not to project employees.
In the case of Mercado, Sr. vs. National
Labor Relations Commission (201 SCRA
332 [1991]), this Court ruled that the proviso
in the second paragraph of Article 280
relates only to casual employees and is not
applicable to those who fall within the
definition of said Article's first paragraph,
i.e., project employees. . . .
Incidentally, we should that both respondent
Commission and the Solicitor General were in error
in concluding based on private respondents' claimed
length of employment (allegedly for over ten years)
that they were regular employees. Sad to state, the
Solicitor General in his arguments tried to "force-fit"
private respondents into the "regular employee"
category and completedly disregarded the critical
distinctions set forth in ALU-TUCP and earlier
cases.
Inconclusive Evidence
Based on the foregoing considerations, it is patent
that, in the instant case, there needs to be a finding
as to whether or not the duration and scope of the
projects were determined or specified and made
known to herein private respondents at the time of
their engagement. The labor arbiter tried to do this,
relying heavily on the "Contract(s) Employment"
presented in petitioner's Annexes as well as on
private respondents' own Annex "A" 10 attached to
their Position Paper, and citing the fact that the said
contracts of employment indicated the duration of
the projects to which the private respondents had
been assigned. He then held that "(t)here is no denial

that complainants were assigned to work in these


projects," 11 and concluded that they were indeed
project employees.
But the arbiter completed ignored the fact that all the
"Contract(s) Employment" presented in evidence by
both petitioner and private respondents had been
signed only by petitioner's president and general
manager, Luis F. Ortega, but not by the employees
concerned, who had precisely refused to sign them.
The said contracts therefore could in no wise be
deemed conclusive evidence. Thus, private
respondents faulted the labor arbiter for giving
credence and probative value to said contracts.
Besides, they claimed, only seven contracts in all
were presented in evidence, pertaining to seven
individual employees, while there are fifteen
employees involved in the complaints. Moreover,
these contracts, purportedly issued either in July or
December of 1991, except for one dated May 1992,
were all one-shot contracts of short duration, the
longest being for about five months. Now, inasmuch
as petitioner had not denied nor rebutted private
respondent' allegations that they had each worked
several years for the petitioner, the obvious question
is, why didn't petitioner produce in evidence similar
contracts for all the other years that private
respondents had worked as project employees? To
these points, petitioner offered no explanation
whatsoever.
Failure to Discharge Burden of Proof
For that matter, it seems self-evident to this Court
that, even if the contracts presented by petitioner had
been signed by the employees concerned, still, they
would not constitute conclusive proof of petitioner's
claim. After all, in the usual scheme of things,
contract terms are normally dictated by the employer
and simply acceded to and accepted by the

employee, who may be desperate for work and


therefore in no position to bargain freely or negotiate
terms to his liking.
In any event, petitioner in the case undoubtedly
could have presented additional evidence to buttress
its claim. For instance, petitioner could have
presented copies of its contracts with its clients, to
show the time, duration and scope of past
installation projects. The data from these contracts
could then have been correlated to the data which
could be found in petitioner's payroll records for, let
us say, the past three years or so, 12 to show that
private respondents had been working intermittently
as and when they were assigned to said projects, and
that their compensation had been computed on the
basis of such work. But petitioner did not produce
such additional evidence, and we find it failed to
discharge its burden of proof.
It is not so much that this Court cannot appreciate
petitioner's contentions about the nature of its
business and its inability to maintain a large
workforce on its permanent payroll. Private
respondents have admitted that petitioner is engaged
only in the installation (not manufacture) of aircon
systems or units in buildings, and since such a line
of business would obviously be highly (if not
wholly) dependent on the availability of buildings or
projects requiring such installation services, which
factor no businessman, no matter how savvy, can
accurately forecast from year to year, it can be easily
surmised that petitioner, aware that its revenues and
income would be unpredictable, would always try to
keep its overhead costs to a minimum, and would
naturally want to engage workers on a per-project or
per-building basis only, retaining very few
employees (if any) on its permanent payroll. It
would also have been more than glad if its
employees found other employment elsewhere, in

between projects. To our mind, it appears rather


unlikely that petitioner would keep private
respondent all fifteen of them continuously on
its permanent payroll for, say, ten or twelve years,
knowing fully well that there would be periods (of
uncertain duration) when no project can be had. To
illustrate, let us assume that private respondents
(who were each making about P118.00 to P119.50
per day in 1991) were paid only P100.00 per day. If
the fifteen were, as they claimed, regular employees
entitled to their wages regardless of whether or not
they were assigned to work on any project, the
overhead for their salaries alone computed at
P100.00/day for 30 days in a month would come
to no less than P45,000.00 a month, or P540,000.00
a year, not counting 13th month pay, Christmas
bonus, SSS/Medicare premium payments, sick
leaves and service incentive leaves, and so forth.
Even if petitioner may have been able to afford such
overhead costs, it certainly does not make business
sense for it or anyone else to do so, and is in every
sense contrary to human nature, not to mention
common business practice. On this score alone, we
believe that petitioner could have made out a strong
case. Which is why we have difficulty understanding
its failure to present clear and convincing evidence
on this point, it being doctrinal that in illegal
dismissal cases, the employer always has the burden
of proof. 13

1. As Annex "B" thereof, a Certification


dated January 28, 1992, signed by one Flora
P. Perez, Administrative/Accountant of
Raycor, certifying that " . . . Mr. Roberto B.
Fulgencio (one of the private respondents)
has been connected with the undersigned
corporation (Raycor) from August 22, 1986
to May 18, 1991 and September 01, 1990 to
January 25, 1992 as Aircon Installer";

Petitioner's problem of weak evidence was further


compounded by certain documentary evidence in the
records below which controverted petitioner's
position, or, at the very least, tended to confuse
rather than clarify matters. For instance, we noted
that in their Memorandum of Appeal dated February
17, 1993 filed with the respondent Commission,
herein private respondents had attached as annexes
thereto the following documents:

Understandably, private respondents made big


capital out of these certifications. But, while
petitioner failed utterly to offer rebutting evidence,
still and all, we are not prepared to conclude on the
basis of these certifications alone that private
respondents were indeed regular employees. First of
all, said certifications refer only to three out of the
fifteen private respondent, so what could be true of
them any not necessarily apply with respect to the
other twelve. Moreover, the certifications do not

2. As Annex "C" thereof, a Certification


dated May 7, 1985, signed by Luis F.
Ortega, President and General Manger of
herein petitioner corporation, to the effect
that ". . . Mr. Jaime Calipayan (another one
of the private respondents) has been
connected with the undersigned corporation
from June 18, 1982 up to present as a
Mechanical Installer; and
3. As Annex "D" thereof, a Certification
dated June 06, 1991, likewise signed by Luis
G. Ortega, president and general manager of
Raycor, certifying that ". . . Mr. Susano A.
Atienza (still another of the private
respondents) has been connected with the
undersigned corporation from October 10,
1983 up to present as Aircon
Mechanical/Technician".

categorically state that the three employees had been


permanent employees of Raycor. In other words,
they do not necessarily overturn petitioner's
contention that private respondents were project
employees, since it is still possible to read the
documents as saying that the named employees were
working as project employees during the period
therein specified. This is especially so since the said
certifications were prepared by non-lawyers who in
all likelihood were not aware of the potential legal
implications and ramifications of what were
ostensibly innocuous certifications. As held in one
recent case, ". . . it is however not difficult to
understand that ordinary business activities are
performed in the normal course without anticipation
nor foreknowledge of litigation, often with dispatch
and usually with a minimum of documentation." 14
Nonetheless, all things considered, the certifications,
issued by petitioner itself, tend to put its claims in
serious doubt.
This situation was still further aggravated by the
manner in which petitioner dismissed private
respondents. As found by respondent Commission,
the reason given for the dismissals, i.e., "due to our
present business status," is vague, to say the least,
and unarguably is not one of the valid or just causes
provided by law for termination of an employment,
whatever its classification. But more significantly
if indeed private respondents were project
employees, there would have been no need to
terminate them by sending them notices of
termination, inasmuch as their employment ceases
"as result of the completion of the project or any
phase thereof in which they are employed," per
Policy Instruction No. 20 itself. Thus, if petitioner
resorted to such dismissals, there is the unavoidable
inference that petitioner regarded the private
respondents as regular employees after all. But
again, this is inconclusive, since the notices of

termination were signed, and in all likelihood


prepared, by the president and general manager of
petitioner, probably sans any legal advice or
awareness of the implications of such a move.
All the aforesaid conflicting data have the net effect
of casting doubt upon and clouding the real nature of
the private respondents' employment status. And we
are mandated by law to resolve all doubts in favor of
labor. For which reason, we hereby hold that private
respondents were regular employees of the
petitioner.
Having arrived at basically the same results as
respondent NLRC with respect to private
respondents' employment status, did this Court waste
its time and effort in re-examining the instant case?
The answer is in the negative. This Court cannot
affirm a decision or judgment based on erroneous
findings and conclusions, for justice can never be
adequately dispensed to all parties if a judgment is
not grounded on the truth.
Second Issue: Terminations Illegal
On the second issue of alleged illegality of the
subject dismissals, we agree with respondent
Commission when it held, as mentioned above, that
"the same was capricious and whimsical as shown
by the vague reason proffered by respondent for said
dismissal which is 'due to our present state' (should
read 'status') is undoubtedly not one of the valid
causes for termination of an employment." True
indeed, for neither the Labor Code nor Policy
Instructions No. 20 allows termination on such
ground. Even Art. 283 of the Labor Code as
amended, which treats of retrenchments and closures
due to business losses, requires that the employer
first serve written notice on the workers and the
Department of Labor at least one month before the

intended date thereof; and in certain cases,


separation pay must be paid. And it cannot be denied
that in the instant case, petitioner did not afford them
due process thru the twin requirements of notice and
hearing, 15 as the terminations took effect the day
following receipt of the notices of termination.
Ineluctably, the said terminations are not in
accordance with law and therefore illegal.
On top of that, there is evidence of the bad faith of
petitioner in terminating the private respondents.
Petitioner placed an ad 16 in the classified ads section
of the People's Journal, sometime in June 1992, 17
which read:
WANTED IMMEDIATELY
MECHANICAL INSTALLERS
TINSMITHS
WELDERS/PIPEFITTERS
APPLY IN PERSON:
RAYCOR AIR CONTROL
SYSTEMS, INC.
RM 306 20TH CENTURY BLDG.
632 SHAW BLVD., MAND.
METRO MANILA
Unmistakably, petitioner, in placing the ad, must
have had at least one project, maybe more, "in the
pipeline" at that time, and was clearly in need of
replacements for private respondents whom it had
just fired. Thus, the dismissals could hardly have
been due to a valid cause, not even due to
petitioner's alleged "present business status". On this
court as well, the dismissals were illegal.
And lastly, we should mention that an order for
reinstatement with payment of backwages must be
based on the correct premises. This point is best

illustrated by considering the last ratiocination


utilized by public respondent: "Even assuming that
complainants were project employees, their
unceremonious dismissal coupled with the attempt to
replace them via the newspaper advertisement
entitles them to reinstatement with backwages under
P.I. No. 20." There is a world of difference between
reinstatement as project employees and
reinstatement as regular employees, but the
difference was obviously lost on the respondent
NLRC.

263 SCRA 174FACTS:


Joselito Macatuno, private respondent, was a seaman on
board theM / T F o r t u n a o f
L i b e r i a n r e g i s t r y . H e w a s
h i r e d b y W a l l e m Shipmanagament Ltd.
Thru its manning agent Wallem Maritime ServicesInc.
Macatunos contract of employment covers 10
months.W h i l e t h e v e s s e l w a s b e r t h e d a t t h e p o r t
o f K a w a s a k i , J a p a n , a n altercation took place between
Macatuno and Gurimbao, a fellow Filipinoagainst a
cadet/apprentice officer of the same nationality as the
captaino f t h e v e s s e l . T h e c a d e t / a p p r e n t i c e
o r d e r e d G u r i m b a o t o s h o v e a n d throw dirty and
oily water at the port of Japan. The latter protested
Conclusion
sinces u c h a c t i s a g a i n s t t h e l a w s o f J a p a n .
H o w e v e r , t h e c a d e t / a p p r e n t i c e insisted on his
We reiterate that this Court waded through the
orders so Gurimbao complied with it. Having finished
his j o b , G u r i m b a o s o u g h t t h e
records of this case searching for solid evidence
a i d o f M a c a t u n o t o a p p r o a c h
upon which to decide the case either way. But all
t h e cadet/apprentice about his improper and
told, neither party managed to make out a clear
unauthorized act. When thet w o F i l i p i n o s
case. Therefore, considering that in illegal dismissal
approached the cadet/apprentice, the
cases, the employer always has the burden of proof, l a t t e r r e a c t e d violently so Macatuno
and considering further that the law mandates that all LEBpushed the latter twice on his chest
doubts, uncertainties, ambiguities and insufficiencies whileGurimbao mildly hit his arm. The captain
witnessed the altercation and entered the incident
be resolved in favor of labor, we perforce rule
inthe tankers logbook. He summoned the two
against petitioner and in favor of private
Filipinos at his cabin. Thec a p t a i n t o l d t h e m t o
respondents.
pack their things as their services are
beingt e r m i n a t e d . A s a
WHEREFORE, the foregoing considered, the
consequence, the two were
assailed Decision is hereby SET ASIDE and a new
r e p a t r i a t e d t o t h e Philippines where they
one rendered holding that petitioner has failed to
lodged complaints for illegal dismissal with
discharge its burden of proof in the instant case and theP O E A . P e t i t i o n e r s
c o n t e n d
t h a t
t h e
t w o
therefore ORDERING the reinstatement of private
F
i
l
i
p
i
n
o
s
h
a
d
b
e
e
n delinquent
respondents as regular employees of petitioner,
on board the vessel as shown by the records of the
without loss of seniority rights and privileges and
captainslogbook T h e P O E A f o u n d t h a t t h e
with payment of backwages from the day they were p r i v a t e r e s p o n d e n t s M a c a t u n o
dismissed up to the time are actually reinstated. No a n d Gurimabaos dismissals were illegal. The
casts.
NLRC affirmed the decision of the POEA. Hence, the
instant petition.
ISSUE:
SO ORDERED.
Whether the dismissal of the two seamen were illegal.
RULING/DECISION:
WALLEM MARITIME SERVICES vs. NLRC

The Court upheld the decision of the


N L R C i n f i n d i n g t h a t t h e private respondents
were illegally dismissed.P e t i t i o n e r s d i d n o t s u b m i t
a s e v i d e n c e t o t h e P O E A t h e l o g b o o k itself but
was merely a typewritten collation of excerpts from what
could
be the logbook. Hence, as the typewritten excerpts from then
logbookwere the only pieces of evidence presented by
petitioners to support thed i s m i s s a l o f p r i v a t e
respondent have no probative
v a l u e a t a l l , petitioners cause must fail. That the
workers involvement in the incident was mustered
orc o n v e n e d t h e r e a f t e r b y t h e c a p t a i n i s
inconsequential. It is sufficientcompliance
with the law which, requires, as a vital
component of due process, observance of the
t w i n r e q u i r e m e n t s o f n o t i c e a n d h e a r i n g before
dismissing an employee.
G.R. No. 117582 December 23, 1996
CONRADO SAMILLANO and MYRNA V.
SAMILLANO, petitioners,
vs.
NATIONAL LABOR RELATIONS COMMISSIONS,
DAN-AG SA DAKBAYAN BROADCASTING
CORPORATION-RADIO STATION DXDD, MSGR.
JESUS DOSADO and SIMPLICIA NERI, respondents.

PADILLA, J.:p
This petition for certiorari under Rule 65 of the Rules of
Court refers to two (2) cases filed by petitioner-spouses
Conrado and Myrna Samillano against private respondents
Dan-ag sa Dakbayan Broadcasting Corporation-Radio
Station DXDD and/or Msgr. Jesus Dosado and/or Simplicia
Neri, Chairman of the Board and Manager respectively of
said respondent corporation.

The first case, filed by petitioner-spouses on 8 February


1991, is a complaint for illegal demotion while the second
complaint filed on 20 May 1991 is for illegal dismissal,
payment of backwages, commissions and other monetary
claims.
The two (2) complaints before Regional Arbitration Branch
No. 10, Cagayan de Oro City of the National Labor
Relations Commission (NLRC) docketed as NLRC RAB
Case Nos. 10-03-00195 and 10-06-00371-91 were later
consolidated since they involve the same parties and issues.
The undisputed facts of the two (2) cases are as follows:
1. Petitioner-spouses Conrado and Myrna Samillano were
hired by private respondents on 1 October 1981 and 1
August 1983 respectively;

6. In a supplemental position paper, herein petitioners


contended that their demotion and subsequent dismissal were
retaliatory acts of private respondents for their having
reported violations by private respondents of labor laws
particularly underpayment/nonpayment of salaries and other
benefits;
Labor Arbiter Noel Augusto S. Magbanua, to whom the
cases were assigned, found that sometime in July 1989, the
Department of Labor and Employment conducted an
inspection of the premises of private respondent corporation
and initially found deficiencies in wages and other benefits
given to employees.

It was further determined that in March or April 1990,


private respondents conducted meetings with their
employees seeking a compromise of the unpaid benefits.
Some employees executed waivers of further claims against
2. On 2 October 1990, Conrado Samillano was transferred to private respondents. Herein petitioners refused to sign said
the Technical Department as an SSB Operator from his
waivers.
previous position as Traffic Supervisor of private respondent
corporation. On the same day, his wife Myrna V. Samillano The labor arbiter formulated the following issues for
was transferred to the AM Production Department from her resolution: 1
position as cashier of respondent corporation;
1) whether complainants' demotion and subsequent
3. As a result of the transfers, the petitioner-spouses filed the termination of employment were retaliatory acts for
complaint for illegal demotion contending that the transfer
complainants' having allegedly reported respondents'
resulted in loss of commissions and violated their security of violations of labor laws;
tenure.
2) whether complainants' demotions were illegal; and
4. On 20 May 1991, petitioner-spouses filed the complaint
for illegal dismissal contending that private respondents
3) whether complainants' terminations from employment
terminated their employment on 23 April 1991 without any
were illegal.
lawful cause;
The labor arbiter resolved the first two (2) issues in the
5. Private respondents relied on allegations that petitionernegative. He declared that no evidence was presented to
spouses misappropriated funds of the radio station and
show that the demotions of petitioners were linked to their
committed acts of insubordination which resulted in loss of
reporting of alleged violations by private respondents of the
trust and confidence, upon which their dismissal were based; Labor Code.

The labor arbiter further upheld management's prerogative,


in the absence of bad faith, to protect its rights in relation to
the alleged offenses committed by petitioners. The
demotions of petitioners were therefore upheld.
With respect to the dismissal of petitioners from
employment, however, the labor arbiter found that the
alleged misappropriations of funds committed by petitioners
were not adequately substantiated. Hence, the dismissal of
petitioners was declared illegal.
The labor arbiter ruled however that instead of reinstatement,
it would be for the best interest of the parties considering the
strained relations between them, to award petitioners
separation pay equivalent to one (1) month salary for every
year of service. Full backwages were not awarded based on
findings that petitioners acted in an arrogant and
uncooperative manner during the investigation of their case
which could be a possible reason why private respondents
were not able to prove the formers' involvement in the
financial irregularities subject of this case. 2 Only six (6)
months backwages were awarded to each of the
complainants (herein petitioners).
Finally, the labor arbiter denied petitioners' claims for unpaid
commissions for lack of evidence.
Appeal by private respondents to the NLRC was dismissed
on 9 February 1994 for their failure to properly perfect their
appeal. The NLRC found that private respondents had filed
their notice of appeal without attaching thereto their appeal
memorandum as required by Section 3 Rule VI of the Rules
of Procedure of the NLRC. There was therefore failure to
perfect the appeal within the reglementary period of ten (10)
days from receipt of the assailed labor arbiter's decision.
On 30 June 1994, the NLRC reinstated the appeal based on
findings that while the notice of appeal and appeal
memorandum were received by the NLRC on 15 July 1993
and 20 July 1993 respectively, or way beyond the period for

appeal which expired on 3 July 1993, both pleadings were,


however, actually mailed on 2 July 1993 as evidenced by
Registry Receipt No. 77 of the Tangub Post Office. 3
On the merits of the appeal, the NLRC ruled that private
respondents have substantiated their claim of having lost
trust and confidence in petitioners due to serious
irregularities in the performance of their duties.
The NLRC held that, contrary to the findings of the labor
arbiter, an audit report submitted by a certain Domeciano
Adaya dated 17 September 1990 showed substantial
evidence of petitioners' involvement in irregularities
including misappropriations of funds, non-turnover of
collections and misuse of funds for personal purposes. The
NLRC relied on reports made by Janice Poncianos, the
Finance Department Business Head of respondent
corporation's radio station addressed to the station manager
as well as the report of the station manager to the chairman
of the board of respondent corporation on the alleged acts of
herein petitioners. 4

petitioners' right to due


process. 6
Before ruling on the merits of this petition, the Court takes
notice of a peculiar circumstance regarding the appeal of the
private respondents from the decision of the labor arbiter.
In the resolution reinstating private respondents' appeal, the
NLRC found that the notice of appeal and memorandum on
appeal were received on 15 July 1993 and 20 July 1993
respectively. The reason for reinstating the appeal was the
finding that both pleadings were actually mailed on 2 July
1993 as evidenced by Registry Receipt No. 77 postmarked
on the same date at the Tangub City Post Office.

It is unexplained however why two (2) pleadings mailed


together using a single registry receipt and presumably
contained in one (1) envelope would be received on two (2)
different dates. It should be pointed out that in the motion for
reconsideration of the resolution dismissing the appeal,
herein private respondents averred mailing only the notice of
appeal and a postal money order to cover appeal fees on 2
Based on the above findings, the NLRC set aside the assailed July 1993. Be that as it may, the Court shall proceed to
decision and ruled that petitioners were validly dismissed.
resolve this case on the merits despite the possible
However, private respondents were ordered to indemnify
technicality of the appeal being filed late with the NLRC.
petitioners the amount of P2,000.00 each for violation of the The NLRC is however reminded to be more accurate in
latters' right to due process. The NLRC agreed with the
recording the dates of mailing and receipt of pleadings filed
petitioners that there was no formal investigation wherein
before it since this is essential in the speedy and correct
the latter were given the chance to defend themselves against disposition of cases.
the charges levelled against them. 5
Petitioners do not dispute before this Court the validity of
In their petition before the Court, it is argued by petitioners
their re-assignments. It is clear that the re-assignments were
that:
a valid exercise of management prerogative pending
investigation of the alleged irregularities. The purpose of the
1. The NLRC gravely abused its discretion in holding that
re-assignments is no different from that of preventive
the dismissals of herein petitioners were valid; and
suspension which private respondents could likewise have
validly imposed on petitioners; to protect the employer's
2. The NLRC gravely abused its discretion in merely
property pending investigation of the alleged malfeasance or
imposing a sanction on private respondents for violation of
misfeasance committed by the employee. 7

In the present case, the labor arbiter correctly held that there
is no evidence to show that the transfer of petitioners to other
positions and the subsequent termination of their
employment were retaliatory acts of private respondents for
petitioners' reporting of the alleged violations by private
respondents of the Labor Code.
The legality of petitioners' dismissal would be determined
based on whether or not private respondents have proved the
basis for loss of trust and confidence upon which the
dismissals are based.
In China City Restaurant Corporation v. NLRC 8 the Court
held thus:
For loss of trust and confidence to be a valid
ground for the dismissal of employees, it
must be substantial and not arbitrary,
whimsical, capricious or concocted.
Irregularities or malpractices should not be
allowed to escape the scrutiny of this Court.
Solicitude for the protection of the rights of
the working class are of prime importance.
Although this is not a license to disregard
the rights of management, still the Court
must be wary of the ploys of management to
get rid of employees it considers as
undesirable.
The NLRC based its decision upholding petitioners'
dismissal on the conclusion that the irregularities involving
petitioners were more than sufficient to make out a case of
loss of trust and confidence. 9
Said irregularities allegedly involving petitioners were
enumerated in An Updated Report dated 17 August 1990
submitted by the Finance Department Business Head Janice
Procianos and various letter-memos to petitioners as well as

the audit report dated 17 September 1990 submitted by


Domeciano Adaya.

further investigation and verification would be necessary to


pinpoint the source of the irregularities.

But petitioners correctly argue that the above-mentioned


documents do not provide enough basis for termination of
their employment based on loss of trust and confidence.

There is thus no evidence on record to show that any further


investigation and verification were done by private
respondents. What is apparent is that petitioners were made
to answer charges of misconduct based on suspicions which
lacked adequate basis.

The Adaya audit report in part reads:


I am suggesting with a request that the
above-mentioned observations be reviewed
and confirmed by the Station Accountant,
Bookkeeper, Collector and Cashier or Cash
Custodian in my presence in fairness to
everyone before I give conclusion,
implication or opinion to these observations.
They may also give comments or raise
objections, if any. The comments or
objections may be made orally or in writing.
In this connection, as I don't have line
authority over the personnel concerned may
I request you to ask them to review and
confirm by observations.

While the law and this Court recognize the right of an


employer to dismiss an employee based on loss of trust and
confidence, the former's evidence must clearly and
convincingly establish the facts upon which the loss of trust
and confidence in the employee is based. 11
In the present case, the unsubstantiated suspicions and
baseless conclusions of private respondents do not provide
legal justification for dismissing herein petitioners. The
doubt in this case should be resolved in favor of labor
pursuant to the social justice policy of labor laws and the
Constitution.

Finally, on petitioners' right to due process, we uphold the


NLRC findings that no formal investigation was conducted
prior to dismissal of petitioners. Private respondents thus
There is no evidence to show that herein private respondents failed to adequately comply with the requirement that an
undertook to review and/or confirm the observations
employee should be given the opportunity to be heard and to
contained in the audit report as recommended by the audit
defend himself before he is dismissed. In San Antonio v.
report itself.
NLRC, 12 the Court stated that "Proper compliance with the
twin requirements of notice and hearing are conditions sine
On the contrary, even in their comment on the petition filed qua non before a dismissal may be validly effected. . . Any
with this Court, which respondents' later adopted as their
procedural shortcut, that effectively allows an employer to
memorandum, the dismissal of herein petitioners is justified assume the roles of both accuser and judge at the same time,
mainly on the basis of said audit report submitted by
should not be countenanced." (emphasis supplied).
10
Domeciano Adaya.
In the present case, the notices/memoranda to petitioners
It is, however, clear from the above-quoted portion of the
requiring explanations/answers to the charges against them
audit report that the findings contained therein do not
were plainly meant to provide a semblance of compliance
categorically find herein petitioners guilty of committing
with the due process requirement which the NLRC correctly
irregularities. The clear import of the said audit report is that ruled to be inadequate.

The Court will not be deceived by schemes to circumvent the


requirements of law and the Constitution. For failure to fully
comply with the requirements of due process, private
respondents should, as a matter of course, indemnify the
petitioners but we refrain from awarding damages on this
score since we are awarding separation pay and backwages
due to petitioners' illegal dismissal.
The above-finding that petitioners were illegally dismissed
normally requires that they be reinstated to their former or
equivalent positions with full backwages. In this case,
however, the relationships between petitioners and private
respondents have undoubtedly become very strained, hence,
separation pay in lieu of reinstatement is proper. 13 However,
as a consequence of petitioners' illegal dismissal, full
backwages from date of dismissal to the finality of this
decision are due the petitioners in line with the ruling in the
Bustamante case. 14
WHEREFORE, the decision appealed from is hereby SET
ASIDE and a new one entered:
1. DECLARING the dismissal from employment of
petitioners NULL and VOID;
2. ORDERING private respondents to pay petitioners'
separation pay at the rate of ONE-HALF (1/2) MONTH
salary for every year of service; and
3. ORDERING private respondents to pay petitioners full
backwages from date of illegal dismissal to the finality of
this decision.
SO ORDERED.
G.R. No. 117565 November 18, 1997
ARSENIO P. LUMIQUED (deceased), Regional Director,
DAR CAR, Represented by his Heirs, Francisca A.
Lumiqued, May A. Lumiqued, Arlene A. Lumiqued and

Richard A. Lumiqued, petitioners,


vs.
Honorable APOLONIO G. EXEVEA, ERDOLFO V.
BALAJADIA and FELIX T. CABADING, ALL Members
of Investigating Committee, created by DOJ Order No.
145 on May 30, 1992; HON. FRANKLIN M. DRILON,
SECRETARY OF JUSTICE, HON. ANTONIO T.
CARPIO, CHIEF Presidential Legal Adviser/Counsel;
and HON. LEONARDO A. QUISUMBING, Senior
Deputy Executive Secretary of the Office of the
President, and JEANNETTE OBAR-ZAMUDIO, Private
Respondent, respondents.

P44,172.46. Private respondent added that Lumiqued seldom pending actual receipt of two of private respondent's
made field trips and preferred to stay in the office, making it complaints. The committee granted the motion and gave him
impossible for him to consume the nearly 120 liters of
a five-day extension.
gasoline he claimed everyday.
In his counter-affidavit dated June 23, 1992, 4 Lumiqued
In her second affidavit-complaint dated November 22, 1989, alleged, inter alia, that the cases were filed against him to
2
private respondent accused Lumiqued with violation of
extort money from innocent public servants like him, and
Commission on Audit (COA) rules and regulations, alleging were initiated by private respondent in connivance with a
that during the months of April, May, July, August,
certain Benedict Ballug of Tarlac and a certain Benigno
September and October, 1989, he made unliquidated cash
Aquino III. He claimed that the apparent weakness of the
advances in the total amount of P116,000.00. Lumiqued
charge was bolstered by private respondent's execution of an
purportedly defrauded the government "by deliberately
affidavit of desistance. 5
concealing his unliquidated cash advances through the
Lumiqued admitted that his average daily gasoline
falsification of accounting entries in order not to reflect on
consumption was 108.45 liters. He submitted, however, that
'Cash advances of other officials' under code 8-70-600 of
ROMERO, J.:
such consumption was warranted as it was the aggregate
accounting rules."
consumption of the five service vehicles issued under his
3
Does the due process clause encompass the right to be
The third affidavit-complaint dated December 15, 1989,
name and intended for the use of the Office of the Regional
assisted by counsel during an administrative inquiry?
charged Lumiqued with oppression and harassment.
Director of the DAR. He added that the receipts which were
According to private respondent, her two previous
issued beyond his region were made in the course of his
Arsenio P. Lumiqued was the Regional Director of the
complaints prompted Lumiqued to retaliate by relieving her travels to Ifugao Province, the DAR Central Office in
Department of Agrarian Reform Cordillera Autonomous from her post as Regional Cashier without just cause.
Diliman, Quezon City, and Laguna, where he attended a
Region (DAR-CAR) until President Fidel V. Ramos
seminar. Because these receipts were merely turned over to
dismissed him from that position pursuant to Administrative The three affidavit-complaints were referred in due course to him by drivers for reimbursement, it was not his obligation
Order No. 52 dated May 12, 1993. In view of Lumiqued's
the Department of Justice (DOJ) for appropriate action. On
but that of auditors and accountants to determine whether
death on May 19, 1994, his heirs instituted this petition for
May 20, 1992, Acting Justice Secretary Eduardo G.
they were falsified. He affixed his signature on the receipts
certiorari and mandamus, questioning such order.
Montenegro issued Department Order No. 145 creating a
only to signify that the same were validly issued by the
committee to investigate the complaints against Lumiqued.
establishments concerned in order that official transactions
The dismissal was the aftermath of three complaints filed by The order appointed Regional State Prosecutor Apolinario
of the DAR-CAR could be carried out.
DAR-CAR Regional Cashier and private respondent
Exevea as committee chairman with City Prosecutor Erdolfo
Jeannette Obar-Zamudio with the Board of Discipline of the Balajadia and Provincial Prosecutor Felix Cabading as
Explaining why a vulcanizing shop issued a gasoline receipt,
DAR. The first affidavit-complaint dated November 16,
members. They were mandated to conduct an investigation
Lumiqued said that he and his companions were cruising
1
1989, charged Lumiqued with malversation through
within thirty days from receipt of the order, and to submit
along Santa Fe, Nueva Vizcaya on their way to Ifugao when
falsification of official documents. From May to September their report and recommendation within fifteen days from its their service vehicle ran out of gas. Since it was almost
1989, Lumiqued allegedly committed at least 93 counts of
conclusion.
midnight, they sought the help of the owner of a vulcanizing
falsification by padding gasoline receipts. He even submitted
shop who readily furnished them with the gasoline they
a vulcanizing shop receipt worth P550.00 for gasoline
The investigating committee accordingly issued a subpoena needed. The vulcanizing shop issued its own receipt so that
bought from the shop, and another receipt for P660.00 for a directing Lumiqued to submit his counter-affidavit on or
they could reimburse the cost of the gasoline. Domingo
single vulcanizing job. With the use of falsified receipts,
before June 17, 1992. Lumiqued, however, filed instead an
Lucero, the owner of said vulcanizing shop, corroborated
Lumiqued claimed and was reimbursed the sum of
urgent motion to defer submission of his counter-affidavit
this explanation in an affidavit dated June 25, 1990. 6 With

respect to the accusation that he sought reimbursement in the


amount of P660.00 for one vulcanizing job, Lumiqued
submitted that the amount was actually only P6.60. Any
error committed in posting the amount in the books of the
Regional Office was not his personal error or accountability.
To refute private respondent's allegation that he violated
COA rules and regulations in incurring unliquidated cash
advances in the amount of P116,000.00, Lumiqued presented
a certification 7 of DAR-CAR Administrative Officer
Deogracias F. Almora that he had no outstanding cash
advances on record as of December 31, 1989.
In disputing the charges of oppression and harassment
against him, Lumiqued contended that private respondent
was not terminated from the service but was merely relieved
of her duties due to her prolonged absences. While admitting
that private respondent filed the required applications for
leave of absence, Lumiqued claimed that the exigency of the
service necessitated disapproval of her application for leave
of absence. He allegedly rejected her second application for
leave of absence in view of her failure to file the same
immediately with the head office or upon her return to work.
He also asserted that no medical certificate supported her
application for leave of absence.
In the same counter-affidavit, Lumiqued also claimed that
private respondent was corrupt and dishonest because a COA
examination revealed that her cash accountabilities from
June 22 to November 23, 1989, were short by P30,406.87.
Although private respondent immediately returned the
amount on January 18, 1990, the day following the
completion of the cash examination, Lumiqued asserted that
she should be relieved from her duties and assigned to jobs
that would not require handling of cash and money matters.
Committee hearings on the complaints were conducted on
July 3 and 10, 1992, but Lumiqued was not assisted by
counsel. On the second hearing date, he moved for its
resetting to July 17, 1992, to enable him to employ the

services of counsel. The committee granted the motion, but


neither Lumiqued nor his counsel appeared on the date he
himself had chosen, so the committee deemed the case
submitted for resolution.

16, 1989 yet, justice can not be delayed


much longer.

Following the conclusion of the hearings, the investigating


committee rendered a report dated July 31, 1992, 10 finding
On August 12, 1992, Lumiqued filed an urgent motion for
Lumiqued liable for all the charges against him. It made the
8
additional hearing, alleging that he suffered a stroke on July following findings:
10, 1992. The motion was forwarded to the Office of the
After a thorough evaluation of the evidences
State Prosecutor apparently because
(sic) submitted by the parties, this
the investigation had already been terminated. In an order
committee finds the evidence submitted by
dated September 7, 1992, 9 State Prosecutor Zoila C.
the complainant sufficient to establish the
Montero denied the motion, viz:
guilt of the respondent for Gross Dishonesty
The medical certificate given show(s) that
and Grave Misconduct.
respondent was discharged from the Sacred
That most of the gasoline receipts used by
Heart Hospital on July 17, 1992, the date of
the respondent in claiming for the
the hearing, which date was upon the request
reimbursement of his gasoline expenses
of respondent (Lumiqued). The records do
were falsified is clearly established by the 15
not disclose that respondent advised the
Certified Xerox Copies of the duplicate
Investigating committee of his confinement
receipts (Annexes G-1 to G-15) and the
and inability to attend despite his discharge,
certifications issued by the different gasoline
either by himself or thru counsel. The
stations where the respondent purchased
records likewise do not show that efforts
gasoline. Annexes "G-1" to "G-15" show
were exerted to notify the Committee of
that the actual average purchase made by the
respondent's condition on any reasonable
respondent is about 8.46 liters only at a
date after July 17, 1992. It is herein noted
purchase price of P50.00, in contrast to the
that as early as June 23, 1992, respondent
receipts used by the respondent which
was already being assisted by counsel.
reflects an average of 108.45 liters at a
Moreover an evaluation of the counterpurchase price of P550.00. Here, the greed
affidavit submitted reveal(s) the sufficiency,
of the respondent is made manifest by his
completeness and thoroughness of the
act of claiming reimbursements of more than
counter-affidavit together with the
10 times the value of what he actually
documentary evidence annexed thereto, such
spends. While only 15 of the gasoline
that a judicious determination of the case
receipts were ascertained to have been
based on the pleadings submitted is already
falsified, the motive, the pattern and the
possible.
scheme employed by the respondent in
defrauding the government has,
Moreover, considering that the complaintnevertheless, been established.
affidavit was filed as far back as November

That the gasoline receipts have been


falsified was not rebutted by the respondent.
In fact, he had in effect admitted that he had
been claiming for the payment of an average
consumption of 108.45 liters/day by
justifying that this was being used by the 4
vehicles issued to his office. Besides he also
admitted having signed the receipts.
Respondent's act in defrauding the
government of a considerable sum of money
by falsifying receipts constitutes not only
Dishonesty of a high degree but also a
criminal offense for Malversation through
Falsification of Official Documents.
This committee likewise finds that the
respondent have (sic) unliquidated cash
advances in the year 1989 which is in
violation of established office and auditing
rules. His cash advances totaling to about
P116,000.00 were properly documented.
The requests for obligation of allotments and
the vouchers covering the amounts were all
signed by him. The mere certification issued
by the Administrative Officer of the DARCAR cannot therefore rebut these concrete
evidences (sic).
On the third complaint, this committee
likewise believes that the respondent's act in
relieving the complainant of her functions as
a Regional Cashier on December 1, 1989
was an act of harassment. It is noted that this
was done barely two weeks after the
complainant filed charges against her (sic).
The recommendation of Jose G. Medina of
the Commission on Audit came only on May
11, 1990 or almost six months after the
respondent's order relieving the complainant

was issued. His act in harassing a


subordinate employee in retaliation to a
complaint she filed constitute(s) Gross
Misconduct on the part of the respondent
who is a head of office.

reconsideration) considering that the matter has already been


forwarded to the Office of the President" and that their
authority under Department Order No. 145 ceased when they
transmitted their report to the
DOJ. 14 Concurring with this view, Undersecretary Esguerra
informed Lumiqued that the investigating committee could
The affidavits of Joseph In-uyay and
no longer act on his motion for reconsideration. He added
Josefina Guting are of no help to the
that the motion was also prematurely filed because the
respondent. In fact, this only show(s) that he Office of the President (OP) had yet to act on Secretary
is capable of giving bribes if only to have
Drilon's recommendation. 15
the cases against him dismissed. He could
not have given a certain Benigno Aquino III On May 12, 1993, President Fidel V. Ramos himself issued
the sum of P10,000.00 for any other
Administrative Order No. 52 (A.O. No. 52), 16 finding
purpose.
Lumiqued administratively liable for dishonesty in the
alteration of fifteen gasoline receipts, and dismissing him
Accordingly, the investigating committee recommended
from the service, with forfeiture of his retirement and other
Lumiqued's dismissal or removal from office, without
benefits. Thus:
prejudice to the filing of the appropriate criminal charges
against him.
That the receipts were merely turned over to
him by his drivers and that the auditor and
Acting on the report and recommendation, former Justice
accountant of the DAR-CAR should be the
Secretary Franklin M. Drilon adopted the same in his
ones to be held liable is untenable. The
Memorandum to President Fidel V. Ramos dated October 22,
receipts in question were signed by
1992. He added that the filing of the affidavit of desistance 11
respondent for the purpose of attesting that
would not prevent the issuance of a resolution on the matter
those receipts were validly issued by the
considering that what was at stake was not only "the
commercial establishments and were
violation of complainant's (herein private respondent's)
properly disbursed and used in the official
personal rights" but also "the competence and fitness of the
business for which it was intended.
respondent (Lumiqued) to remain in public office." He
opined that, in fact, the evidence on record could call for "a
This Office is not about to shift the blame
punitive action against the respondent on the initiative of the
for all these to the drivers employed by the
DAR."
DAR-CAR as respondent would want us to
do.
On December 17, 1992, Lumiqued filed a motion for
reconsideration of "the findings of the Committee" with the The OP, however, found that the charges of oppression and
DOJ. 12 Undersecretary Ramon S. Esguerra indorsed the
harassment, as well as that of incurring unliquidated cash
motion to the investigating committee. 13 In a letter dated
advances, were not satisfactorily established.
April 1, 1993, the three-member investigating committee
In a "petition for appeal" 17 addressed to President Ramos,
informed Undersecretary Esguerra that the committee "had
Lumiqued prayed that A.O. No. 52 be reconsidered and that
no more authority to act on the same (motion for

he be reinstated to his former position "with all the benefits


accorded to him by law and existing rules and regulations."
This petition was basically premised on the affidavit dated
May 27, 1993, of a certain Dwight L. Lumiqued, a former
driver of the DAR-CAR, who confessed to having authored
the falsification of gasoline receipts and attested to petitioner
Lumiqued's being an "honest man" who had no
"premonition" that the receipts he (Dwight) turned over to
him were "altered." 18

time within which to secure a counsel of his own. If


suspension was not possible, the committee should have
appointed a counsel de oficio to assist him.

falsification of public documents in its report and


recommendation.

Petitioners' misconception on the nature of the investigation


25
These arguments are untenable and misplaced. The right to
conducted against Lumiqued appears to have been
counsel, which cannot be waived unless the waiver is in
engendered by the fact that the DOJ conducted it. While it is
writing and in the presence of counsel, is a right afforded a
true that under the Administrative Code of 1987, the DOJ
suspect or an accused during custodial investigation. 23 It is
shall "administer the criminal justice system in accordance
not an absolute right and may, thus, be invoked or rejected in with the accepted processes thereof consisting in the
a criminal proceeding and, with more reason, in an
investigation of the crimes, prosecution of offenders and
Treating the "petition for appeal" as a motion for
administrative inquiry. In the case at bar, petitioners invoke
administration of the correctional system, 26 conducting
reconsideration of A.O. No. 52, the OP, through Senior
the right of an accused in criminal proceedings to have
criminal investigations is not its sole function. By its power
Deputy Executive Secretary Leonardo A. Quisumbing,
competent and independent counsel of his own choice.
to "perform such other functions as may be provided by
denied the same on August 31, 1993.
Lumiqued, however, was not accused of any crime in the
law," 27 prosecutors may be called upon to conduct
proceedings below. The investigation conducted by the
administrative investigations. Accordingly, the investigating
Undaunted, Lumiqued filed a second motion for
committee created by Department Order No. 145 was for the committee created by Department Order No. 145 was dutyreconsideration, alleging, among other things, that he was
purpose of determining if he could be held administratively bound to conduct the administrative investigation in
denied the constitutional right to counsel during the hearing. liable under the law for the complaints filed against him. The accordance with the rules therefor.
19
On May 19, 1994, 20 however, before his motion could be order issued by Acting Secretary of Justice Montenegro
resolved, Lumiqued died. On September 28, 1994, 21
While investigations conducted by an administrative body
states thus:
Secretary Quisumbing denied the second motion for
may at times be akin to a criminal proceeding, the fact
reconsideration for lack of merit.
In the interest of the public service and
remains that under existing laws, a party in an administrative
pursuant to the provisions of existing laws, a inquiry may or may not be assisted by counsel, irrespective
Hence, the instant petition for certiorari and mandamus
Committee to conduct the formal
of the nature of the charges and of the respondent's capacity
praying for the reversal of the Report and Recommendation
investigation of the administrative complaint to represent himself, and no duty rests on such a body to
of the Investigating Committee, the October 22, 1992,
for oppression, dishonesty, disgraceful and
furnish the person being investigated with counsel. 28 In an
Memorandum of then Justice Secretary Drilon, A.O. No. 52
immoral conduct, being notoriously
administrative proceeding such as the one that transpired
issued by President Ramos, and the orders of Secretary
undesirable and conduct prejudicial to the
below, a respondent (such as Lumiqued) has the option of
Quisumbing. In a nutshell, it prays for the "payment of
best interest of the service against Mr.
engaging the services of counsel or not. This is clear from
retirement benefits and other benefits accorded to deceased
ARSENIO P. LUMIQUED, Regional
the provisions of Section 32, Article VII of Republic Act No.
Arsenio Lumiqued by law, payable to his heirs; and the
Director, Department of Agrarian Reform,
2260 29 (otherwise known as the Civil Service Act) and
backwages from the period he was dismissed from service
Cordillera Autonomous Region, is hereby
Section 39, paragraph 2, Rule XIV (on Discipline) of the
22
24
up to the time of his death on May 19, 1994."
created . . .
Omnibus Rules Implementing Book V of Executive Order
No. 292 30 (otherwise known as the Administrative Code of
Petitioners fault the investigating committee for its failure to
As such, the hearing conducted by the investigating 1987). Excerpts from the transcript of stenographic notes of
inform Lumiqued of his right to counsel during the hearing.
committee was not part of a criminal prosecution.
the hearings attended by Lumiqued 31 clearly show that he
They maintain that his right to counsel could not be waived
This was even made more pronounced when, after
was confident of his capacity and so opted to represent
unless the waiver was in writing and in the presence of
finding Lumiqued administratively liable, it hinted at himself . Thus, the right to counsel is not imperative in
counsel. They assert that the committee should have
the filing of a criminal case for malversation through administrative investigations because such inquiries are
suspended the hearing and granted Lumiqued a reasonable
conducted merely to determine whether there are facts that

merit disciplinary measures against erring public officers and


employees, with the purpose of maintaining the dignity of
government service.

has already set a hearing,


morning and afternoon
today.

A. I think so, Sir.

Furthermore, petitioners' reliance on Resolution No. 94-0521


of the Civil Service Commission on the Uniform Procedure
in the Conduct of Administrative Investigation stating that a
respondent in an administrative complaint must be "informed
of his right to the assistance of a counsel of his choice," 32 is
inappropriate. In the first place, this resolution is applicable
only to cases brought before the Civil Service Commission.
33
Secondly, said resolution, which is dated January 25, 1994,
took effect fifteen days following its publication in a
newspaper of general circulation, 34 much later than the July
1992 hearings of the investigating committee created by
Department Order No. 145. Thirdly, the same committee was
not remiss in the matter of reminding Lumiqued of his right
to counsel. Thus, at the July 3, 1992, hearing, Lumiqued was
repeatedly appraised of his option to secure the services of
counsel:

RSP EXEVEA:

Let us make it of record that


we have been warning you
to proceed with the
assistance of counsel but
you said that you can take
care of yourself so we have
no other alternative but to
proceed. 36 (Emphasis
supplied).

RSP EXEVEA:

So, we will proceed with the


hearing even without your
counsel? You are willing to
proceed with the hearing
even without your counsel?
DIR. LUMIQUED:
Yes, I am confident. . .

Thereafter, the following colloquies transpired:

CP BALAJADIA:

CP BALAJADIA:

You are confident that you


will be able to represent
yourself?

We will suspend in the


meantime that we are
waiting for the
supplemental affidavit you
are going to present to us.
Do you have any request
from the panel of
investigators, Director
Lumiqued?

DIR. LUMIQUED:

This is an administrative
That is my concern. 35
case against Director
(Emphasis supplied)
Lumiqued. Director
Lumiqued is present. The
In the course of private respondent's damaging testimony, the
complainant is present,
investigating committee once again reminded Lumiqued of
Janet Obar-Zamudio.
his need for a counsel. Thus:
Complainant has just been
furnished with a copy of the
CP BALAJADIA:
counter-affidavit of the
respondent. Do you have a
Q. (To Director Lumiqued)
counsel, Director?
You really wish to go
through with this even
DIR. LUMIQUED:
without your counsel?
I did not bring anybody, Sir,
because when I went to see
him, he told me, Sir, that he

CP BALAJADIA:

DIRECTOR LUMIQUED:

DIRECTOR LUMIQUED:
I was not able to bring a
lawyer since the lawyer I
requested to assist me and
was the one who prepared
my counter-affidavit is
already engaged for a
hearing and according to
him he is engaged for the
whole month of July.

RSP EXEVEA:

without counsel, we will


proceed.

We cannot wait . . .
CP BALAJADIA:

next week, probably


Wednesday so we will have
good time (sic) of
presenting the affidavit.

CP BALAJADIA:
Madam Witness, will you
please submit the document
which we asked for and
Director Lumiqued, if you
have other witnesses, please
bring them but reduce their
testimonies in affidavit form
so that we can expedite with
the proceedings. 37

Why don't you engage the


services of another counsel.
The charges against you are
quite serious. We are not
saying you are guilty
already. We are just
apprehensive that you will
go through this
investigation without a
counsel. We would like you At the hearing scheduled for July 10, 1992, Lumiqued still
to be protected legally in the did not avail of the services of counsel. Pertinent excerpts
course of this investigation. from said hearing follow:
Why don't you get the
FISCAL BALAJADIA:
services of another counsel.
There are plenty here in
I notice also Mr. Chairman
Baguio . . .
that the respondent is not
being represented by a
DIRECTOR LUMIQUED:
counsel. The last time he
I will try to see, Sir . . .
was asked to invite his
lawyer in this investigation.
CP BALAJADIA:
May we know if he has a
lawyer to represent him in
Please select your date now,
this investigation?
we are only given one
month to finish the
DIR. LUMIQUED:
investigation, Director
Lumiqued.
There is none Sir because
when I went to my lawyer,
RSP EXEVEA:
he told me that he had set a
case also at 9:30 in the other
We will not entertain any
court and he told me if there
postponement. With or
is a possibility of having
this case postponed anytime

FISCAL BALAJADIA:
Are you moving for a
postponement Director?
May I throw this to the
panel. The charges in this
case are quite serious and he
should be given a chance to
the assistance of a
counsel/lawyer.
RSP EXEVEA:
And is (sic) appearing that
the supplemental-affidavit
has been furnished him only
now and this has several
documents attached to it so
I think we could grant him
one last postponement
considering that he has
already asked for an
extension.
DIR. LUMIQUED:
Furthermore Sir, I am now
being bothered by my heart
ailment. 38
The hearing was reset to July 17, 1992, the date when
Lumiqued was released from the hospital. Prior to said date,
however, Lumiqued did not inform the committee of his
confinement. Consequently because the hearing could not
push through on said date, and Lumiqued had already

submitted his counter-affidavit, the committee decided to


wind up the proceedings. This did not mean, however, that
Lumiqued was short-changed in his right to due process.

citizen is not that helpless that he cannot


analogous to property in a limited sense; hence, the right to
validly act at all except only with a lawyer at due process could rightfully be invoked. Nonetheless, the
his side.
right to security of tenure is not absolute. Of equal weight is
the countervailing mandate of the Constitution that all public
Lumiqued, a Regional Director of a major department in the In administrative proceedings, the essence of due process is officers and employees must serve with responsibility,
executive branch of the government, graduated from the
simply the opportunity to explain one's side. One may be
integrity, loyalty and efficiency. 48 In this case, it has been
University of the Philippines (Los Baos) with the degree of heard, not solely by verbal presentation but also, and perhaps clearly shown that Lumiqued did not live up to this
Bachelor of Science major in Agriculture, was a recipient of even much more creditably as it is more practicable than oral constitutional precept.
various scholarships and grants, and underwent training
arguments, through pleadings. 41 An actual hearing is not
seminars both here and abroad. 39 Hence, he could have
always an indispensable aspect of due process. 42 As long as The committee's findings pinning culpability for the charges
defended himself if need be, without the help of counsel, if
a party was given the opportunity to defend his interests in
of dishonesty and grave misconduct upon Lumiqued were
truth were on his side. This, apparently, was the thought he
due course; he cannot be said to have been denied due
not, as shown above, fraught with procedural mischief. Its
entertained during the hearings he was able to attend. In his process of law, for this opportunity to be heard is the very
conclusions were founded on the evidence presented and
statement, "That is my concern," one could detect that it had essence of due process. 43 Moreover, this constitutional
evaluated as facts. Well-settled in our jurisdiction is the
been uttered testily, if not exasperatedly, because of the
mandate is deemed satisfied if a person is granted an
doctrine that findings of fact of administrative agencies must
doubt or skepticism implicit in the question, "You are
opportunity to seek reconsideration of the action or ruling
be respected as long as they are supported by substantial
44
confident that you will be able to represent yourself?"
complained of. Lumiqued's appeal and his subsequent
evidence, even if such evidence is not overwhelming or
despite his having positively asserted earlier, "Yes, I am
filing of motions for reconsideration cured whatever
preponderant. 49 The quantum of proof necessary for a
confident." He was obviously convinced that he could ably
irregularity attended the proceedings conducted by the
finding of guilt in administrative cases is only substantial
45
represent himself. Beyond repeatedly reminding him that he committee.
evidence or such relevant evidence as a reasonable mind
could avail himself of counsel and as often receiving the
might accept as adequate to support a conclusion. 50
reply that he is confident of his ability to defend himself, the The constitutional provision on due process safeguards life,
liberty and property. 46 In the early case of Cornejo v.
Consequently, the adoption by Secretary Drilon and the OP
investigating committee could not do more. One can lead a
Gabriel and Provincial Board of
of the committee's recommendation of dismissal may not in
horse to water but cannot make him drink.
47
Rizal the Court held that a public office is not property
any way be deemed tainted with arbitrariness amounting to
The right to counsel is not indispensable to due process
within the sense of the constitutional guarantee of due
grave abuse of discretion. Government officials are
unless required by the Constitution or the law. In Nera v.
process of law for it is a public trust or agency. This
presumed to perform their functions with regularity. Strong
40
Auditor General, the Court said:
jurisprudential pronouncement has been enshrined in the
evidence is not necessary to rebut that presumption, 51 which
1987 Constitution under Article XI, Section 1, on
petitioners have not successfully disputed in the instant case.
. . . There is nothing in the Constitution that accountability of public officers, as follows:
says that a party in a non-criminal
Dishonesty is a grave offense penalized by dismissal under
proceeding is entitled to be represented by
Sec. 1. Public office is a public trust. Public Section 23 of Rule XIV of the Omnibus Rules Implementing
counsel and that, without such
officers and employees must at all times be Book V of the Administrative Code of 1987. Under Section 9
representation, he shall not be bound by
accountable to the people, serve them with
of the same Rule, the penalty of dismissal carries with it
such proceedings. The assistance of lawyers;
utmost responsibility, integrity, loyalty, and "cancellation of eligibility, forfeiture of leave credits and
while desirable, is not indispensable. The
efficiency, act with patriotism and justice,
retirement benefits, and the disqualification for
legal profession was not engrafted in the due
and lead modest lives.
reemployment in the government service." The instant
process clause such that without the
petition, which is aimed primarily at the "payment of
When
the
dispute
concerns
one's
constitutional
right
to
participation of its members, the safeguard
retirement benefits and other benefits," plus back wages
is deemed ignored or violated. The ordinary security of tenure, however, public office is deemed

from the time of Lumiqued's dismissal until his demise,


must, therefore, fail.

The case was docketed and assigned to theCOMELEC's


Second Division.

The COMELEC,
en banc
, issued an Order setting aside the preliminaryinjunction and
thereby allowing petitioner to assume as mayor of
theMunicipality of Madrilejos pending resolution of his
appeal.

reversing the lower court's judgmentawarding damages to


herein petitioner after it had earlier dismissed for beingmoot
and academic.
NO.RATIO:
WHEREFORE, the instant petition for certiorari and

mandamus is hereby DISMISSED and Administrative Order


The Omnibus Election Code provides: Actual or
no. 52 of the Office of the President is AFFIRMED. Costs
compensatorydamages may be granted in all election
against petitioners.
contests or in
quo warranto
SO ORDERED.
proceedings in accordance with law.

However, following the synchronized elections of May 11,


Provisions for actual or compensatory damages
1992, thePresiding Commissioner of the COMELEC's
under the law areembodied in various Civil Code articles
Atienza vs. Comelec
Second
Division
issued
anOrder
dated
July
18,
1992
allowing claims for damagesunder specific circumstances.
FACTS:
dismissing
petitioner's
appeal
for
being
moot
and
Thus, Article 2176 provides: Whoeverby act or omission

academic pursuant to the Commission's decision


causes damage to another, there being fault
Private respondent Antonio G. Sia was elected mayor of
inResolution No. 2494 declaring the election protest and
ornegligence, is obliged to pay for the damage done.
theMunicipality of Madrilejos, Cebu in the 1988 local
appeal
cases

as
well
as
petitions
for
special
relief

Such fault ornegligence, if there is no pre-existing


elections obtaininga plurality of 126 votes over his nearest
arising
out
of
the
January18,
1988
elections
dismissed
and
contractual relation betweenthe parties is called a
rival, herein petitioner Lou A.Atienza.
terminated as of June 30,1992.
quasi delict

, and is governed by the provisionsof this chapter.

On
January
28,
1993,
respondent
Commission

Following Sia's proclamation by the Municipal Board of


en
banc
Specifically, Article 2199 of the Civil Code mandates that:
Canvassers,petitioner filed an election protest with
released
itsquestioned
resolution,
the
dispositive
portion
of
Except asprovided by law or by stipulation, one is
the Regional Trial Courtquestioning the results of the
which states:PREMISES CONSIDERED, the Commission
entitled to an adequatecompensation only for such
elections in a number of precincts in themunicipality.
RESOLVED, as it herebyRESOLVES, that the dismissal pecuniary loss suffered by him as he hasduly proved. Such
of the appeal by the Commission(
compensation is referred to as actual orcompensatory

damages.
Second
Division
Consequently, in the revision ordered by the lower court,

)
f
o
r
b
e
i
n
g
m
o
o
t
a
n
d
a
c
a
d
e
m
i
c
b
e
c
a
u
s
e
o
f
petitionerobtained a total of 2,826 votes, a plurality
t h e expiration of the term of office of the contested Given this setting, it would appear virtually impossible for a
of 12 votes over theprivate respondent.
position did notthereby revive the vacated judgment of the party inan election protest case to recover actual or

compensatory damagesin the absence of the conditions


On April 12, 1989 the Regional Trial Court rendered Regional Trial Court, saidappealed judgment to remain
vacated,
not
having
been
resolved
onthe
merits
by
the
specified under Articles 2201 and2202 of the Civil
its decisiondeclaring petitioner the winner of the municipal
Commission
for
or
against
any
of
the
parties;
andthe
Code,
elections andordering the private respondent to reimburse
judgment
directing
theprotestee-appellant
to
reimburse
or in the absence of a law expressly providingfor situations
petitioner the amountof P300,856.19 representing
theprotestant-appellee
the
amount
of
P300,856.19
allowing for the recovery of the same
petitioner's expenses in the electionprotest.
representing
hisexpenses
in
the
election
protest,
is
hereby
. I t f o l l o w s , naturally, that in most election protest

REVERSED
cases where the monetaryclaim does not hinge on
Private respondent appealed the trial court's decision to the
,
said
judgmentnot
being
in
accordance
with
law
in
the
either a contract or
COMELECraising as errors 1) the computation of the
absence
of
any
evidence
of
a
n
y
w
r
o
n
g
f
u
l
,
o
r
quasi
number of votes receivedby the candidates; and 2) the
n
e
g
l
i
g
e
n
t
a
c
t
o
r
o
m
i
s
s
i
o
n
o
n
t
h
e
p
a
r
t
o
f
-contract or atortious act or omission,
alleged award of "excessive damages"in favor of the
t
h
e
protestee
appellant
to
justify
the
award.
the claimant must be able to point out to aspecific
petitioner.
ISSUE(S)/HELD:
provision of law authorizing a money claim for
Whether the COMELEC acted with grave abuse of
electionprotest expenses against the losing party
discretion when itissued its Resolution of January 28, 1993

. This, petitioner has beenunable to do.

Section 259 of the Omnibus Election Code merely


provides for thegranting of actual and compensatory
damages in accordance withl a w. T h a t i t w a s t h e
intent of the legislature to do away
w i t h provisions indemnifying the victorious party for
expenses incurred inan election contest in the absence
of a wrongful act or omissionclearly attributable to the
losing party cannot be gainsaid. The intent,moreover, to do
away with such provisions merely recognizes
themaxim, settled in law that a wrong without
damage or damage without wrong neither constitutes a
cause of action nor creates a civil obligation.

classification must, as an indispensable requisite, not be


arbitrary. The requisites to be complied with are;
(1) must rest on substantial distinctions;
(2) must be germane to the purposes of the law;
(3) must not be limited to existing conditions only; and
(4) must apply equally to all members of the same class.

Act No. 1639 satisfies these requirements. The classification


rests on real or substantial, not merely imaginary or
whimsical, distinctions. It is not based upon accident of
birth or parentage. The law, then, does not seek to mark the
People of the Philippines vs Cayat
Equal Protection Requisites of a Valid Classification non-Christian tribes as an inferior or less capable race. On
the contrary, all measures thus far adopted in the promotion
Bar from Drinking Gin
of the public policy towards them rest upon a recognition of
In 1937, there exists a law (Act 1639) which bars native non- their inherent right to equality in the enjoyment of those
privileges now enjoyed by their Christian brothers. But as
Christians from drinking gin or any other liquor outside of
there can be no true equality before the law, if there is, in
their customary alcoholic drinks. Cayat, a native of the
Cordillera, was caught with an A-1-1 gin in violation of this fact, no equality in education, the government has
Act. He was then charged and sentenced to pay P5.00 and to endeavored, by appropriate measures, to raise their culture
be imprisoned in case of insolvency. Cayat admitted his guilt and civilization and secure for them the benefits of their
progress, with the ultimate end in view of placing them with
but he challenged the constitutionality of the said Act. He
their Christian brothers on the basis of true equality.
averred, among others, that it violated his right to equal
protection afforded by the constitution. He said this an
G.R. No. 78742: Association of Small Landowners vs
attempt to treat them with discrimination or mark them as
inferior or less capable race and less entitled will meet with Secretary of Agrarian Reform
Equal Protection
their instant challenge. The law sought to distinguish and
classify native non-Christians from Christians.
ISSUE: Whether or not the said Act violates the equal
protection clause.
HELD: The SC ruled that Act 1639 is valid for it met the
requisites of a reasonable classification. The SC emphasized
that it is not enough that the members of a group have the
characteristics that distinguish them from others. The

These are 3 cases consolidated questioning the


constitutionalityof the Agrarian Reform Act. Article XIII on
Social Justice and Human Rightsincludes a call for the
adoption by the State of an agrarian reform program.The
State shall, by law, undertake an agrarian reform program
founded on theright of farmers and regular farmworkers,
who are landless, to own directly orcollectively the lands

they till or, in the case of other farmworkers, toreceive a just


share of the fruits thereof. RA 3844, Agricultural Land
ReformCode, had already been enacted by Congress on
August 8, 1963. This wassubstantially superseded almost a
decade later by PD 27, which was promulgatedon Oct 21,
1972, along with martial law, to provide for the
compulsoryacquisition of private lands for distribution
among tenant-farmers and tospecify maximum retention
limits for landowners. On July 17, 1987, Cory issuedEO 228,
declaring full land ownership in favor of the beneficiaries of
PD 27and providing for the valuation of still unvalued lands
covered by the decreeas well as the manner of their payment.
This was followed on July 22, 1987 byPP 131, instituting a
comprehensive agrarian reform program (CARP), and EO
229,providing the mechanics for its implementation.
Afterwhich is the enactment ofRA 6657, Comprehensive
Agrarian Reform Law of 1988, which Cory signed on
June10. This law, while considerably changing the earlier
mentioned enactments,nevertheless gives them suppletory
effect insofar as they are not inconsistentwith its provisions.

Inconsidering the rentals as advance payment on


the land, the executive orderalso deprives the petitioners of
their property rights as protected by dueprocess. The equal
protection clause is also violated because the order placesthe
burden of solving the agrarian problems on the owners only
of agriculturallands. No similar obligation is imposed on the
owners of other properties.

Thepetitioners maintain that in declaring the


beneficiaries under PD 27 to be theowners of the lands
occupied by them, EO 228 ignored judicial prerogatives
andso violated due process. Worse, the measure would not
solve the agrarianproblem because even the small farmers
are deprived of their lands and theretention rights guaranteed
by the Constitution.

the COMELEC avers that it only copied the provision from


Sec. 13 of R.A. 9369.
In hiscomment the Sol-Gen asserted that the
alleged violation of the equal protectionclause, the sugar
planters have failed to show that they belong to a
differentclass and should be differently treated.

(1) it must be based on substantial distinctions;


(2) it must be germane to the purposes of the law;

ISSUE: Whether or not the said COMELEC resolution was


valid.

HELD: NO. In the Farias case, the petitioners challenged


Sec. 14 of RA. 9006 repealing Sec. 66 of the Omnibus
Election Code (OEC) for giving undue benefit to elective
(4) it must apply equally to all the members of the class.
officials in comparison with appointive officials.
Incidentally, the Court upheld the substantial distinctions
The Comment also suggests the possibilityof Congress first
between the two and pronounced that there was no violation
distributing public agricultural lands and scheduling
of the equal protection clause. However in the present case,
theexpropriation of private agricultural lands later. From this
the Court held that the discussion on the equal protection
The Court finds that all these requisites have been met by
viewpoint, thepetition for prohibition would be premature.
clause was an obiter dictum since the issue raised therein
themeasures here challenged as arbitrary and discriminatory. was against the repealing clause. It didnt squarely
challenge Sec. 66.
(3) it must not be limited to existing conditions only; and

Sec. 13 of RA. 9369 unduly discriminated appointive and


elective officials. Applying the 4 requisites of a valid
Equal protection simply means that all persons or
classification, the proviso does not comply with the second
thingssimilarly situated must be treated alike both as to the
requirement that it must be germane to the purpose of the
rights conferred andthe liabilities imposed. The petitioners
law.
have not shown that they belong to adifferent class and
The obvious reason for the challenged provision is to prevent
entitled to a different treatment. The argument that notonly
HELD: The SC ruledaffirming the Sol-Gen. The argument
the use of a governmental position to promote ones
landowners but also owners of other properties must be
of the small farmers that they have beendenied equal
candidacy, or even to wield a dangerous or coercive
made to share theburden of implementing land reform must influence of the electorate. The measure is further aimed at
protection because of the absence of retention limits has
promoting the efficiency, integrity, and discipline of the
alsobecome academic under Sec 6 of RA 6657. Significantly, be rejected. There is a substantialdistinction between these
public service by eliminating the danger that the discharge of
they too have notquestioned the area of such limits. There is two classes of owners that is clearly visible exceptto those
official duty would be motivated by political considerations
who will not see. There is no need to elaborate on this
also the complaint that theyshould not be made to share the
rather than the welfare of the public. The restriction is also
matter. In anyevent, the Congress is allowed a wide leeway
burden of agrarian reform, an objection alsomade by the
justified by the proposition that the entry of civil servants to
sugar planters on the ground that they belong to a particular in providing for a validclassification. Its decision is accorded the electorate arena, while still in office, could result in
recognition and respect by the courtsof justice except only
classwith particular interests of their own. However, no
neglect or inefficiency in the performance of duty because
they would be attending to their campaign rather than to their
where its discretion is abused to the detriment of theBill of
evidence has been submittedto the Court that the requisites
office work.
Rights.
of a valid classification have been violated.
ISSUE: Whether or notthere was a violation of the equal
protection clause.

Quinto vs Comelec
G. R. No. 189698
Classification has been defined as the grouping of persons
orthings similar to each other in certain particulars and
different from eachother in these same particulars. To be
valid, it must conform to the followingrequirements:

FACTS: Petitioners Eleazar P. Quinto and Gerino A.


Tolentino, Jr. filed a petition for certiorari and prohibition
against the COMELEC for issuing a resolution declaring
appointive officials who filed their certificate of candidacy
as ipso facto resigned from their positions. In this defense,

Sec. 13 of RA. 9369 pertains to all civil servants holding


appointive posts without distinction as to whether they
occupy high positions in government or not. Certainly, a
utility worker in the government will also be considered as
ipso facto resigned once he files his certificate of candidacy
for the election. This scenario is absurd for, indeed, it is
unimaginable how he can use his position in the government
to wield influence in the political world.

case of Farinas, etl. al. vs. Executive Secretary, et. al.


The provision s directed to the activity any and all public
offices, whether they be partisan or non partisan in character,
whether they be in the national, municipal or barangay level.
Congress has not shown a compelling state interest to restrict
the fundamental right involved on such a sweeping scale.
MOTION FOR RECONSIDERATION
FACTS: This is a motion for reconsideration filed by the
Commission on Elections. The latter moved to question an
earlier decision of the Supreme Court declaring Section 4 (a)
of COMELEC Resolution No. 8678 unconstitutional.
Section 4 (a) of COMELEC Resolution No. 8678 provides
that, Any person holding a public appointive office or
position including active members of the Armed Forces of
the Philippines, and other officers and employees in
government-owned or controlled corporations, shall be
considered ipso facto resigned from his office upon the filing
of his certificate of candidacy. Be it noted that petitioners
of the above-entitled case are appointive officials who intend
to be elected in the previously held 2010 elections and who
felt aggrieved by the issuance of the questioned resolution.
ISSUE: Whether or not Section 4 (a) of COMELEC
Resolution No. 8678 is constitutional.
RULING: The Supreme Court overruled its previous
decision declaring the assailed resolution unconstitutional.
Here, it strongly upholds the constitutionality of the
resolution saying that it does not violate the equal protection
clause. It is settled that the equal protection clause does not
demand absolute equality; it merely requires that all persons
shall be treated alike, under like circumstances and
conditions both as to privileges conferred and liabilities
enforced. The test used is reasonableness which requires
that:
1. The classification rests on substantial distinctions;
2. It is germane to the purposes of the law;
3. It is not limited to existing conditions only; and
4. It applies equally to all members of the same class.
In the case under consideration, there is a substantial
distinction between public and elective officials which has
been rendered moot and academic by the ruling made in the

Section 4 (a) of COMELEC Resolution No. 8678 is


constitutional.
Roma Drug & Romeo Rodriguez vs RTC of Guagua,
Pampanga et al
Equal Protection Access to Medicine
Roma Drug, owned by Rodriguez, was raided by the NBI
upon request of Smithkline a pharmaceutical company
(now Glaxo Smithkline). RD is apparently one of 6
pharmacies who are directly importing 5 medicines produced
by Smithknline from abroad. RD is not purchasing those
medicines via local Smithkline the authorized distributor
of Smithkline in the Philippines. Smithkline Phil avers that
because the medicines were not purchased from a Philippine
registered counterpart of Smithkline then the products
imported by RD are considered as counterfeit or
unregistered imported drug product - as defined by RA
8203 Special Law on Counterfeit Drugs. Notwithstanding
RDs motion for reconsideration, the provincial prosecutor
recommended that Rodriguez be tried. Rodriguez assails the
constitutionality of SLCD averring, among other things, that
it has violated his right to equal protection as it banned him
access from such medicines.
ISSUE: Whether or not SLCD violates equal protection.
HELD: The SC ruled in favor of RD. The SC denounced
SLCD for it violated equal protection. It does not allow
private 3rd parties to import such medicines abroad even in
cases of life and death nor does it allow the importation by
3rd parties in cases wherein the stocks of such medicine
would run out. It discriminates at the expense of Filipinos
who cannot travel abroad to purchase such medicines yet
need them badly. Nevertheless, the flawed intention of
Congress had been abrogated by the passage of RA 9502
Universally Accessible Cheaper and Quality Medicines Act
of 2008 and its IRR. This law does not expressly repeal

SLCD but it emphasized that any medicine introduced into


the Philippines by its patent holder be accessible to anyone.
It provides that the right to import drugs and medicines shall
be available to any government agency OR ANY PRIVATE
3rd PARTY. The SC noted that this law provided and
recognized the constitutionally-guaranteed right of the public
to health.
G.R. No. L-45685: People vs Vera
Delegation of Powers

Cu Unjieng was convicted by the trial court in Manila. He


filed for reconsideration which was elevated to the SC and
the SC remanded the appeal to the lower court for a new
trial. While awaiting new trial, he appealed for probation
alleging that the he is innocent of the crime he was convicted
of. Judge Tuason of the Manila CFI directed the appeal to
the Insular Probation Office. The IPO denied the application.
However, Judge Vera upon another request by petitioner
allowed the petition to be set for hearing. The City
Prosecutor countered alleging that Vera has no power to
place Cu Unjieng under probation because it is in violation
of Sec. 11 Act No. 4221 which provides that the act of
Legislature granting provincial boards the power to provide a
system of probation to convicted person. Nowhere in the law
is stated that the law is applicable to a city like Manila
because it is only indicated therein that only provinces are
covered. And even if Manila is covered by the law it is
unconstitutional because Sec 1 Art 3 of the Constitution
provides equal protection of laws. The said law provides
absolute discretion to provincial boards and this also
constitutes undue delegation of power. Further, the said
probation law may be an encroachment of the power of the
executive to provide pardon because providing probation, in
effect, is granting freedom, as in pardon.

maintaining a foreignaccount in Switzerland from 19681991. On 21 Dec 1991, 14 more informationswere filed
ISSUE: Whether or not there is undue delegation of power. against Marcos, Benedicto and Rivera for the same offense.
InJanuary 1992, 11 more informations were filed. The RTC
consolidated the casesand Marcos was arraigned in Feb
1992. During the pendency of these cases, CBCirc 1318 and
HELD: The act of granting probation is not the same as
CB Circ 1353 (Further Liberalizing Foreign Exchange
pardon. In fact it is limited and is in a way an imposition of
Regulations)were issued which basically allowed residents,
penalty. There is undue delegation of power because there is firms, associations andcorporations to maintain foreign
no set standard provided by Congress on how provincial
exchange accounts abroad but the circularshave a saving
boards must act in carrying out a system of probation. The
clause excepting from the circular pending criminal
provincial boards are given absolute discretion which is
actionsinvolving violations of CB Circ 960. Marcos filed a
violative of the constitution and the doctrine of the non
Motion to Quash based onthe new circular. The RTC denied
delegability of power. Further, it is a violation of equity so
the Motion so did the CA hence the appeal.Marcos averred
protected by the constitution. The challenged section of Act that her right to equal protection has been violated,
No. 4221 in section 11 which reads as follows: This Act shall amongothers, as the new circular was purposedly designed to
apply only in those provinces in which the respective
preserve the criminalcases lodged against her.
provincial boards have provided for the salary of a probation
officer at rates not lower than those now provided for
provincial fiscals. Said probation officer shall be appointed
by the Secretary of Justice and shall be subject to the
ISSUE: Whether or notImeldas right to equal protection
direction of the Probation Office. This only means that only had been violated by CB Circ 1353.
provinces that can provide appropriation for a probation
officer may have a system of probation within their locality.
This would mean to say that convicts in provinces where no
HELD: The SC ruled against Imelda. The SC said
probation officer is instituted may not avail of their right to
Herlamentations that the aforementioned provisions are
probation.
discriminatory because theyare aimed at her and her coG.R. No. 126594: Imelda Marcos vs Court of Appeals,
accused do not assume the dignity of a legalargument since
Manila RTC Judge Guillermo Loja Sr., et al
they are unwarranted conjectures belied by even the text of
EqualProtection
thecirculars alone. Hence, as respondent appellate court
correctly concludes, theforegoing facts clearly disprove
petitioner's claim that her constitutionalright to equal
protection of the law was violated. Should she
Marcos was charged for violating Central Bank Circ 960
nonethelessdesire to pursue such objection, she may always
whichbanned residents, firms, associations and corporations adduce additional evidence atthe trial of these cases since
from maintaining foreignexchange accounts abroad w/o
that is the proper stage therefor, and not attheir present
permission from the CB. The circular was issued in1983.
posture.
Any violation thereof constitutes a criminal offense. In 1991,
8informations were filed against Marcos accusing her of

G.R. No. 113811: Ishmael Himagan vs People of the


Philippines & Judge Hilario Mapayo
EqualProtection Suspension of PNP Members Charged
with Grave Felonies

Himaganis a policeman assigned in Camp Catititgan, Davao


City. He was charged for themurder of Benjamin Machitar Jr
and for the attempted murder of Benjaminsyounger brother,
Barnabe. Pursuant to Sec 47 of RA 6975, Himagan was
placedinto suspension pending the murder case. The law
provides that Upon the filingof a complaint or information
sufficient in form and substance against a memberof the PNP
for grave felonies where the penalty imposed by law is six
(6) yearsand one (1) day or more, the court shall
immediately suspend the accused fromoffice until the case is
terminated. Such case shall be subject to continuoustrial and
shall be terminated within ninety (90) days from arraignment
of theaccused. Himagan assailed the suspension averring
that Sec 42 of PD 807 of the Civil Service Decree,that his
suspension should be limited to ninety (90) days. He claims
that animposition of preventive suspension of over 90 days is
contrary to the CivilService Law and would be a violation of
his constitutional right to equalprotection of laws.

ISSUE: Whether or not Sec 47, RA 6975 violates equal


protectionguaranteed by the Constitution.

HELD: The language of the first sentence of Sec 47 of RA


6975 isclear, plain and free from ambiguity. It gives no other
meaning than that thesuspension from office of the member
of the PNP charged with grave offensewhere the penalty is
six years and one day or more shall last until thetermination
of the case. The suspension cannot be lifted before the
terminationof the case. The second sentence of the same

Section providing that the trialmust be terminated within


ninety (90) days from arraignment does not qualify orlimit
the first sentence. The two can stand independently of each
other. Thefirst refers to the period of suspension. The second
deals with the time fromwithin which the trial should be
finished.

The reason why members of thePNP are treated


differently from the other classes of persons
chargedcriminally or administratively insofar as the
application of the rule onpreventive suspension is concerned
is that policemen carry weapons and thebadge of the law
which can be used to harass or intimidate witnesses
againstthem, as succinctly brought out in the legislative
discussions.

within ninety (90) days", there isnothing in RA 6975 that


suggests that the preventive suspension of the accusedwill be
lifted if the trial is not terminated within that period.
Nonetheless,the Judge who fails to decide the case within the
period without justifiablereason may be subject to
administrative sanctions and, in appropriate caseswhere the
facts so warrant, to criminal or civil liability. If the trialis
unreasonably delayed without fault of the accused such that
he is deprivedof his right to a speedy trial, he is not without a
remedy. He may ask for thedismissal of the case. Should the
court refuse to dismiss the case, the accusedcan compel its
dismissal by certiorari, prohibition or mandamus, or secure
hisliberty by habeas corpus.

others such as the executive, former executives and


theirwidows among others.

ISSUE: Whether or not there has been a violation of equal


protectionbefore the law.

HELD: The SC ruled that there is a violation of the


equalprotection clause. The judiciary needs the franking
privilege so badly as it isvital to its operation. Evident to that
need is the high expense allotted tothe judiciarys franking
needs. The Postmaster cannot be sustained incontending that
G.R. No. 105371: Philippine Judges Association et al vs
DOTC Secretary Pete Prado et al
the removal of the franking privilege from the judiciary is
EqualProtection Franking Privilege of the Judiciary
inorder to cut expenditure. This is untenable for if the
Postmaster would intendto cut expenditure by removing the
franking privilege of the judiciary, thenthey should have
removed the franking privilege all at once from all the
Areport came in showing that available data from the Postal otherdepartments. If the problem of the respondents is the
If a suspended policeman criminally charged with a
Service Office showthat from January 1988 to June 1992, the loss of revenues from thefranking privilege, the remedy is to
seriousoffense is reinstated to his post while his case is
total volume of frank mails amountedto P90,424,175.00. of withdraw it altogether from all agenciesof the government,
pending, his victim and thewitnesses against him are
obviously exposed to constant threat and thus easilycowed to this amount, frank mails from the Judiciary and
including those who do not need it. The problem is notsolved
otheragencies
whose
functions
include
the
service
of
judicial
silence by the mere fact that the accused is in uniform and
by retaining it for some and withdrawing it from others,
processes, such as theintervenor, the Department of Justice
armed. theimposition of preventive suspension for over 90
especiallywhere there is no substantial distinction between
and
the
Office
of
the
Ombudsman,
amountedto
P86,481,759.
days under Sec 47 of RA 6975does not violate the suspended
those favored, which may ormay not need it at all, and the
Frank
mails
coming
from
the
Judiciary
amounted
to
policeman's constitutional right to equalprotection of the
Judiciary, which definitely needs it. Theproblem is not
P73,574,864.00,and those coming from the petitioners
laws.
solved by violating the Constitution.
reached the total amount ofP60,991,431.00. The postmaster's
conclusion is that because of thisconsiderable volume of
mail from the Judiciary, the franking privilege must
bewithdrawn from it. Acting from this, Prado implemented
Suppose the trial is not terminated within ninety days
The equal protection clause doesnot require the
Circ. No. 9228 as theIRR for the said law. PJA assailed the
fromarraignment, should the suspension of accused be
universal application of the laws on all persons or
said
law
complaining
that
the
law
wouldadversely
impair
the
lifted?
thingswithout distinction. This might in fact sometimes
communication within the judiciary as it may impair
result in unequal protection,as where, for example, a law
thesending of judicial notices. PJA averred that the law is
prohibiting mature books to all persons,regardless of age,
discriminatory as itdisallowed the franking privilege of the
would benefit the morals of the youth but violate theliberty
Judiciary but has not disallowed thefranking privilege of
The answer is certainly no. While the law uses the
of adults. What the clause requires is equality among equals
mandatory word "shall"before the phrase "be terminated
asdetermined according to a valid classification. By

classification is meant thegrouping of persons or things


similar to each other in certain particulars anddifferent from
all others in these same particulars.

ISSUE: Whether or not Gumabon et al is entitled to the


effects of the Hernandez Doctrine.

On18 May 1995, alleged members of the Kuratong Baleleng


Gang were shot to death.The incident was later
sensationalized as a rub out. This implicated Lacsonamong
HELD: The SC ruled in favor of Gumabon et al. The
others as guilty for multiple murder. The case was raised
continued incarceration after the twelve-year period when
before theSandiganbayan. In 1996, Lacson et al filed
such is the maximum length of imprisonment in accordance separate motions questioning the jurisdiction of
In lumping the Judiciary with the other offices from
with the controlling doctrine, when others similarly
theSandiganbayan, asserting that under the amended
whichthe franking privilege has been withdrawn, Sec 35 has convicted have been freed, is fraught with implications at
informations, the cases fallwithin the jurisdiction of the RTC
placed the courts ofjustice in a category to which it does not war with equal protection. That is not to give it life. On the
pursuant to Sec 2 (par a and c) of RA 7975"An Act To
belong. If it recognizes the need ofthe President of the
contrary, it would render it nugatory. Otherwise, what would Strengthen The Functional And Structural Organization Of
Philippines and the members of Congress for the
happen is that for an identical offense, the only distinction
TheSandiganbayan, Amending For That Purpose Presidential
frankingprivilege, there is no reason why it should not
lying in the finality of the conviction of one being before the Decree 1606, As Amended.They contend that the said law
recognize a similar and in factgreater need on the part of the Hernandez ruling and the other after, a person duly
limited the jurisdiction of the Sandiganbayan tocases where
Judiciary for such privilege.
sentenced for the same crime would be made to suffer
one or more of the "principal accused" are
different penalties. If Gumabon et al would continue to
governmentofficials with Salary Grade (SG) 27 or higher, or
endure imprisonment, then this would be repugnant to equal PNP officials with the rankof Chief Superintendent
Mario Gumabon et al vs Director of the Bureau of
protection, people similarly situated were not similarly dealt (Brigadier General) or higher. The highest rankingprincipal
Prisons
with.
accused in the amended informations has the rank of only a
Equal Protection Hernandez Doctrine
ChiefInspector, and none has the equivalent of at least SG
What is required under this constitutional guarantee is the
27. In 1997, RA 8249 waspassed which basically expanded
Gumabon et al were charged for rebellion punished under
uniform operation of legal norms so that all persons under
the jurisdiction of the Sandiganbayan. The lawwas authored
Art 134 of the RPC. Their offense was complexed with
similar circumstances would be accorded the same treatment by Lagman and Neptali Gonzales. Lacson assailed the law as
multiple murder, robbery, arson, and kidnapping. They were
both in the privileges conferred and the liabilities imposed.
it was introduced by the authors thereof in badfaith as it was
all sentenced to reclusion perpetua. Their sentence had
As was noted in a recent decision: Favoritism and undue
made to precisely suit the situation in which petitionerscases
become final and executory when the Hernandez Doctrine
preference cannot be allowed. For the principle is that equal were in at the Sandiganbayan by restoring jurisdiction
was promulgated by the SC. Hernandez Doctrine simply
protection and security shall be given to every person under thereover to it,thereby violating his right to procedural due
states that murder cannot be complexed to rebellion as it is
circumstances, which if not identical are analogous. If law be process and the equal protectionclause of the Constitution.
necessarily absorbed therein. Hence, the without such
looked upon in terms of burden or charges, those that fall
Further, from the way the Sandiganbayan hasfoot-dragged
complexion the penalty must be lower than reclusion
within a class should be treated in the same fashion,
for nine (9) months the resolution of a pending incident
perpetua. Gumabon precisely assert a deprivation of a
whatever restrictions cast on some in the group equally
involvingthe transfer of the cases to the Regional Trial
constitutional right, namely, the denial of equal protection.
binding on the rest.
Court, the passage of the lawmay have been timed to
The petitioners were convicted by CFI for the very same
overtake such resolution to render the issue thereinmoot, and
rebellion for which Hernandez and others were convicted.
frustrate the exercise of petitioners vested rights under the
The law under which they were convicted is the very same
G.R. No. 147780: Panfilo Lacson vs Sandiganbayan,
oldSandiganbayan law (RA 7975).
law under which the latter were convicted. It had not and has Executive Secretary, et al
not been changed. For the same crime, committed under the
EqualProtection KBG Cases Before the
same law, how can the SC, in conscience, allow petitioners
Sandiganbayan
to suffer life imprisonment, while others can suffer only
ISSUE: Whether or not the right to equal protection by
prision mayor?
Lacson etal has been violated with the passage of RA 8249.

thatmakes real differences. In the first instance, evidence


against them were notyet presented, whereas in the latter the
HELD: The SC ruled that RA 8249 did not violate the right parties had already submitted theirrespective proofs,
ofLacson et al to equal protection. No concrete evidence and examined witness and presented documents. Since it is
convincing argumentwere presented to warrant a declaration withinthe power of Congress to define the jurisdiction of
of an act of the entire Congress andsigned into law by the
courts subject to theconstitutional limitations, it can be
highest officer of the co-equal executive department
reasonably anticipated that an alterationof that jurisdiction
asunconstitutional. Every classification made by law is
would necessarily affect pending cases, which is why ithas to
presumed reasonable.Thus, the party who challenges the law provide for a remedy in the form of a transitory provision.
must present proof of arbitrariness. Itis an established
Thus, Lacsonet al cannot claim that Secs 4 and 7
precept in constitutional law that the guaranty of the
placedthem under a different category from those similarly
equalprotection of the laws is not violated by a legislation
situated as them.
based on reasonableclassification. The classification is
reasonable and not arbitrary when thereis concurrence of
four elements, namely:
Precisely, par A of Sec 4 provides that it shall apply to"all
cases involving" certain public officials and, under
thetransitory provision in Sec 7, to "all cases pending in any
(1) it must rest on substantial distinction;
court."Contrary to petitioner and intervenors arguments, the
law is not particularlydirected only to the Kuratong Baleleng
cases. The transitory provision does notonly cover cases
which are in the Sandiganbayan but also in "any court."It
(2) it must be germane to the purpose of thelaw;
just happened that the Kuratong Baleleng cases are one of
those affected bythe law. Moreover, those cases where trial
had already begun are not affectedby the transitory provision
under Sec 7 of the new law (R.A. 8249).
(3) must not be limited to existing conditionsonly, and
G.R. No. 91649: Humberto Basco et al vs Philippine
Amusements & Gaming Corporation
EqualProtection - Gambling
(4) must apply equally to all members of thesame class

PAGCORwas created by virtue of PD 1067-A dated Jan1,


The classification between those pending cases involvingthe 1977 and was granted a franchiseunder PD 1067-B also
dated Jan 1, 1977 "to establish, operate and
concerned public officials whose trial has not yet
commenced and whosecases could have been affected by the maintaingambling casinos on land or water within the
territorial jurisdiction of thePhilippines." Its operation was
amendments of the Sandiganbayanjurisdiction under R.A.
8249, as against those cases where trial had alreadystarted as originally conducted in the well knownfloating casino
"Philippine Tourist." The operation was considered asuccess
of the approval of the law, rests on substantial distinction

for it proved to be a potential source of revenue to


fundinfrastructure and socioeconomic projects, thus, PD
1399 was passed on June 2,1978 for PAGCOR to fully attain
this objective. Subsequently, on July 11, 1983,PAGCOR was
created under PD 1869 to enable the Government to regulate
andcentralize all games of chance authorized by existing
franchise or permitted bylaw, under the following declared
policy:
Section1. Declaration of Policy. It is hereby declared to
be the policy ofthe State to centralize and integrate all games
of chance not heretoforeauthorized by existing franchises or
permitted by law.

Basco and other lawyers assailedthe validity of


PAGCOR averring among others that it violates the equal
protection clause ofthe constitution in that it legalizes
PAGCOR conducted gambling, while most other forms of
gambling are outlawed,together with prostitution, drug
trafficking and other vices.

ISSUE: Whether or not the creation of PAGCOR violates


the equalprotection clause.

HELD: The SC found Bascos petition to be devoid of


merit. Justhow PD 1869 in legalizing gambling conducted by
PAGCOR is violative of theequal protection is not clearly
explained in their petition. The mere fact thatsome gambling
activities like cockfighting (PD 449) horse racing (RA 306
asamended by RA 983), sweepstakes, lotteries and races (RA
1169 as amended by BP42) are legalized under certain
conditions, while others are prohibited, doesnot render the
applicable laws, PD. 1869 for one, unconstitutional.

"b) Any law to the contrary notwithstanding and starting


with the effectivity of this Act, tariff duty shall be imposed
and collected on imported crude oil at the rate of three
percent (3%) and imported refined petroleum products at the
rate of seven percent (7%), except fuel oil and LPG, the rate
for which shall be the same as that for imported crude oil:
Provided, That beginning on January 1, 2004 the tariff rate
on imported crude oil and refined petroleum products shall
be the same: Provided, further, That this provision may be
amended only by an Act of Congress."

Bascos posture ignores thewell-accepted meaning


of the clause "equal protection of the laws."The clause does
not preclude classification of individuals who may be
accordeddifferent treatment under the law as long as the
classification is notunreasonable or arbitrary. A law does not
have to operate in equal force on allpersons or things to be
conformable to Article III, Sec 1 of the Constitution.The
"equal protection clause" does not prohibit the Legislature
fromestablishing classes of individuals or objects upon
which different rules shalloperate. The Constitution does not
require situations which are different infact or opinion to be
The inclusion of the tariff provision in section 5(b) of R.A.
treated in law as though they were the same.
No. 8180 violates Section 26(1) Article VI of the
G.R. No. 124360: Francisco Tatad vs Secretary of Energy
Constitution requiring every law to have only one subject
One Title, One Subject Rule Oid Deregulation Law
which shall be expressed in its title. Petitioner contends that
the imposition of tariff rates in section 5(b) of R.A. No. 8180
is foreign to the subject of the law which is the deregulation
of the downstream oil industry.

Considering that oil is not endemic to this country, history


shows that the government has always been finding ways to
alleviate the oil industry. The government created laws
accommodate these innovations in the oil industry. One such
law is the Downstream Oil Deregulation Act of 1996 or RA
8180. This law allows that "any person or entity may import
or purchase any quantity of crude oil and petroleum products
from a foreign or domestic source, lease or own and operate
refineries and other downstream oil facilities and market
such crude oil or use the same for his own requirement,"
subject only to monitoring by the Department of Energy.
Tatad assails the constitutionality of the law. He claims that
section 5 (b) of R.A. No. 8180 violates the one title one rule
of Sec 26, Art 6 of the Constitution. Section 5 (b) provides:

ISSUE: Whether or not RA 8180 is constitutional.

HELD: The SC declared the unconstitutionality of RA 8180


not because it violated the one title one subject rule but
rather because it violated Sec 19 of Art 12 of the
Constitution. It violated that provision because it only
strengthens oligopoly which is contrary to free competition.
The SC emphasized that the provision of Sec 5 (b) of RA
8180 does not violate the one title one subject rule. The SC,
as a policy, has adopted a liberal construction of the one title
- one subject rule. The SC also emphasized that the title need
not mirror, fully index or catalogue all contents and minute
details of a law. A law having a single general subject
indicated in the title may contain any number of provisions,
no matter how diverse they may be, so long as they are not

inconsistent with or foreign to the general subject, and may


be considered in furtherance of such subject by providing for
the method and means of carrying out the general subject.
The SC held that section 5(b) providing for tariff differential
is germane to the subject of RA 8180 which is the
deregulation of the downstream oil industry. The section is
supposed to sway prospective investors to put up refineries
in our country and make them rely less on imported
petroleum.
Francisco Tatad et al vs Secretary of Energy
Equal Protection Oil Deregulation Law
Considering that oil is not endemic to this country, history
shows that the government has always been finding ways to
alleviate the oil industry. The government created laws
accommodate these innovations in the oil industry. One such
law is the Downstream Oil Deregulation Act of 1996 or RA
8180. This law allows that any person or entity may import
or purchase any quantity of crude oil and petroleum products
from a foreign or domestic source, lease or own and operate
refineries and other downstream oil facilities and market
such crude oil or use the same for his own requirement,
subject only to monitoring by the Department of Energy.
Tatad assails the constitutionality of the law. He claims,
among others, that the imposition of different tariff rates on
imported crude oil and imported refined petroleum products
violates the equal protection clause. Tatad contends that the
3%-7% tariff differential unduly favors the three existing oil
refineries and discriminates against prospective investors in
the downstream oil industry who do not have their own
refineries and will have to source refined petroleum products
from abroad.3% is to be taxed on unrefined crude products
and 7% on refined crude products.
ISSUE: Whether or not RA 8180 is constitutional.
HELD: The SC declared the unconstitutionality of RA 8180
because it violated Sec 19 of Art 12 of the Constitution. It
violated that provision because it only strengthens oligopoly

which is contrary to free competition. It cannot be denied


that our downstream oil industry is operated and controlled
by an oligopoly, a foreign oligopoly at that. Petron, Shell and
Caltex stand as the only major league players in the oil
market. All other players belong to the lilliputian league. As
the dominant players, Petron, Shell and Caltex boast of
existing refineries of various capacities. The tariff
differential of 4% therefore works to their immense benefit.
Yet, this is only one edge of the tariff differential. The other
edge cuts and cuts deep in the heart of their competitors. It
erects a high barrier to the entry of new players. New players
that intend to equalize the market power of Petron, Shell and
Caltex by building refineries of their own will have to spend
billions of pesos. Those who will not build refineries but
compete with them will suffer the huge disadvantage of
increasing their product cost by 4%. They will be competing
on an uneven field. The argument that the 4% tariff
differential is desirable because it will induce prospective
players to invest in refineries puts the cart before the horse.
The first need is to attract new players and they cannot be
attracted by burdening them with heavy disincentives.
Without new players belonging to the league of Petron, Shell
and Caltex, competition in our downstream oil industry is an
idle dream.
RA 8180 is unconstitutional on the ground inter alia that it
discriminated against the new players insofar as it placed
them at a competitive disadvantage vis--vis the established
oil companies by requiring them to meet certain conditions
already being observed by the latter.
Taxicab Operators vs Board of Transportation
Police Power

Petitioner Taxicab Operators of Metro Manila, Inc.


(TOMMI) is a domestic corporation composed of taxicab
operators, who are grantees of Certificates of Public

Convenience to operate taxicabs within the City of Manila


and to any other place in Luzon accessible to vehicular
traffic.

B. Granting arguendo, that respondents did comply with the


procedural requirements imposed by Presidential Decree No.
101, would the implementation and enforcement of the
assailed memorandum circulars violate the petitioners
constitutional rights to.

On October 10, 1977, respondent Board of Transportation


(BOT) issued Memorandum Circular No. 77-42 which reads:
(1) Equal protection of the law;

SUBJECT: Phasing out and Replacement of Old and


Dilapidated Taxis

(2) Substantive due process; and

(3) Protection against arbitrary and unreasonable


classification and standard?
On January 27, 1981, petitioners filed a Petition with the
BOT, docketed as Case No. 80-7553, seeking to nullify MC
No. 77-42 or to stop its implementation; to allow the
registration and operation in 1981 and subsequent years of
taxicabs of model 1974, as well as those of earlier models
which were phased-out, provided that, at the time of
registration, they are roadworthy and fit for operation.

ISSUE

A. Did BOT and BLT promulgate the questioned


memorandum circulars in accord with the manner required
by Presidential Decree No. 101, thereby safeguarding the
petitioners constitutional right to procedural due process?

HELD
As enunciated in the preambular clauses of the challenged
BOT Circular, the overriding consideration is the safety and
comfort of the riding public from the dangers posed by old
and dilapidated taxis. The State, in the exercise of its police
power, can prescribe regulations to promote the health,
morals, peace, good order, safety and general welfare of the
people. It can prohibit all things hurtful to comfort, safety
and welfare of society. [5] It may also regulate property
rights. [6] In the language of Chief Justice Enrique M.
Fernando the necessities imposed by public welfare may
justify the exercise of governmental authority to regulate
even if thereby certain groups may plausibly assert that their
interests are disregarded.

Mary Concepcion Bautista et al vs Alfredo Juinio et al


the very least, discrimination that finds no support in reason.
Equal Protection Distinction Between Heavy and Extra It suffices then that the laws operate equally and uniformly
Heavy Cars and Others
on all persons under similar circumstances or that all persons
must be treated in the same manner, the conditions not being
Bautista is assailing the constitutionality of LOI 869 issued
different, both in the privileges conferred and the liabilities
in 1979 which classified vehicles into Heavy and Extra
imposed. Favoritism and undue preference cannot be
Heavy. The LOI further banned these vehicles during
allowed. For the principle is that equal protection and
weekends and holidays that is from 5am Saturday until 5am security shall be given to every person under circumstances,
Monday. Purpose of this law is to curb down petroleum
which if not identical are analogous. If law be looked upon
consumption as bigger cars consume more oil. Bautista
in terms of burden or charges, those that fall within a class
claimed the LOI to be discriminatory as it made an
should be treated in the same fashion, whatever restrictions
assumption that H and EH cars are heavy on petroleum
cast on some in the group equally binding on the rest.
consumption when in fact there are smaller cars which are
also big on oil consumption. Further, the law restricts their
Patricio Dumlao et al vs COMELEC
freedom to enjoy their car while others who have smaller
cars may enjoy theirs. Bautista avers that there is no rational
Equal Protection Eligibility to Office after Being 65
justification for the ban being imposed on vehicles classified
as heavy (H) and extra-heavy (EH), for precisely those
Dumlao was the former governor of Nueva Vizcaya. He has
owned by them fall within such category.
retired from his office and he has been receiving retirement
benefits therefrom. He filed for reelection to the same office
ISSUE: Whether or not the LOI violates equal protection.
for the 1980 local elections. On the other hand, BP 52 was
passed (par 1 thereof) providing disqualification for the likes
HELD: The SC held that Bautista was not able to make
of Dumlao. Dumlao assailed the BP averring that it is class
merit out of her contention. The classification on cars on its legislation hence unconstitutional. His petition was joined by
face cannot be characterized as an affront to reason. The
Atty. Igot and Salapantan Jr. These two however have
ideal situation is for the laws benefits to be available to all, different issues. The suits of Igot and Salapantan are more of
that none be placed outside the sphere of its coverage. Only a taxpayers suit assailing the other provisions of BP 52
thus could chance and favor be excluded and the affairs of
regarding the term of office of the elected officials, the
men governed by that serene and impartial uniformity, which length of the campaign and the provision barring persons
is of the very essence of the idea of law. The actual, given
charged for crimes may not run for public office and that the
things as they are and likely to continue to be, cannot
filing of complaints against them and after preliminary
approximate the ideal. Nor is the law susceptible to the
investigation would already disqualify them from office. In
reproach that it does not take into account the realities of the general, Dumlao invoked equal protection in the eye of the
situation. . . . To assure that the general welfare be promoted, law.
which is the end of law, a regulatory measure may cut into
the rights to liberty and property. Those adversely affected
ISSUE: Whether or not the there is cause of action.
may under such circumstances invoke the equal protection
clause only if they can show that the governmental act
HELD: The SC pointed out the procedural lapses of this
assailed, far from being inspired by the attainment of the
case for this case would never have been merged. Dumlaos
common weal was prompted by the spirit of hostility, or at
cause is different from Igots. They have separate issues.

Further, this case does not meet all the requisites so that itd
be eligible for judicial review. There are standards that have
to be followed in the exercise of the function of judicial
review, namely: (1) the existence of an appropriate case; (2)
an interest personal and substantial by the party raising the
constitutional question; (3) the plea that the function be
exercised at the earliest opportunity; and (4) the necessity
that the constitutional question be passed upon in order to
decide the case. In this case, only the 3rd requisite was met.
The SC ruled however that the provision barring persons
charged for crimes may not run for public office and that the
filing of complaints against them and after preliminary
investigation would already disqualify them from office as
null and void.
The assertion that Sec 4 of BP 52 is contrary to the safeguard
of equal protection is neither well taken. The constitutional
guarantee of equal protection of the laws is subject to
rational classification. If the groupings are based on
reasonable and real differentiations, one class can be treated
and regulated differently from another class. For purposes of
public service, employees 65 years of age, have been validly
classified differently from younger employees. Employees
attaining that age are subject to compulsory retirement, while
those of younger ages are not so compulsorily retirable.
In respect of election to provincial, city, or municipal
positions, to require that candidates should not be more than
65 years of age at the time they assume office, if applicable
to everyone, might or might not be a reasonable
classification although, as the Solicitor General has
intimated, a good policy of the law should be to promote the
emergence of younger blood in our political elective
echelons. On the other hand, it might be that persons more
than 65 years old may also be good elective local officials.
Retirement from government service may or may not be a
reasonable disqualification for elective local officials. For
one thing, there can also be retirees from government service
at ages, say below 65. It may neither be reasonable to

disqualify retirees, aged 65, for a 65-year old retiree could be


a good local official just like one, aged 65, who is not a
retiree.
But, in the case of a 65-year old elective local official
(Dumalo), who has retired from a provincial, city or
municipal office, there is reason to disqualify him from
running for the same office from which he had retired, as
provided for in the challenged provision.

Villegas vs. Hui Chiong Tsai Pao Ho


FACTS: This case involves an ordinance prohibiting aliens
from being employed or engage or participate in any position
or occupation or business enumerated therein, whether
permanent, temporary or casual, without first securing an
employment permit from the Mayor of Manila and paying
the permit fee of P50.00. Private respondent Hiu Chiong Tsai
Pao Ho who was employed in Manila, filed a petition to stop
the enforcement of such ordinance as well as to declare the
same null and void. Trial court rendered judgment in favor of
the petitioner, hence this case.
ISSUE: WON said Ordinance violates due process of law
and equal protection rule of the Constitution.
HELD: Yes. The Ordinance The ordinance in question
violates the due process of law and equal protection rule of
the Constitution. Requiring a person before he can be
employed to get a permit from the City Mayor who may
withhold or refuse it at his will is tantamount to denying him
the basic right of the people in the Philippines to engage in a
means of livelihood. While it is true that the Philippines as a
State is not obliged to admit aliens within its territory, once
an alien is admitted, he cannot be deprived of life without
due process of law. This guarantee includes the means of
livelihood. The shelter of protection under the due process
and equal protection clause is given to all persons, both
aliens and citizens.
G.R. No. L-52304: Ramon Ceniza et al vs Commission on
Elections, COA & National Treasurer

Equal Protection - Gerrymandering

**"Gerrymandering" is a "term employed to describe an


apportionment of representative districts so contrived as to
give an unfair advantage to the party inpower." **

Pursuant to Batas Blg 51 (enacted 22 Dec 1979),


COMELEC adopted Resolution No. 1421 which effectively
bars voters in chartered cities (unless otherwise provided by
their charter), highly urbanized (those earning above P40M)
cities, and component cities (whose charters prohibit them)
from voting in provincial elections. The City of Mandaue, on
the other hand, is a component city NOT a chartered one or a
highly urbanized one. So when COMELEC added Mandaue
to the list of 20 cities that cannot vote in provincial
elections,Ceniza, in behalf of the other members of DOERS
(Democracy or Extinction: Resolved to Succeed) questioned
the constitutionality of BB 51 and the COMELEC
resolution. They said that the regulation/restriction of voting
being imposedis a curtailment of the right to suffrage.
Further, petitioners claim that political and gerrymandering
motives were behind the passage of Batas Blg. 51and
Section 96 of the Charter of Mandaue City. They contend
that the Provinceof Cebu is politically and historically
known as an opposition bailiwick and ofthe total 952,716
registered voters in the province, close to one-third (1/3)of
the entire province of Cebu would be barred from voting for
the provincialofficials of the province of Cebu. Ceniza also
said that the constituents of Mandaue never ratified their
charter. Ceniza likewise aver that Sec 3 of BB 885 insofar as
it classifies cities including Cebu City as highly urbanized as
the only basis for not allowing its electorate to vote for the
provincial officials is inherently and palpably
unconstitutional in that such classification is not based on
substantial distinctions germane to the purpose of the law
which in effect provides for and regulates the exercise of the

right of suffrage, and therefore such unreasonable


classification amounts to a denial of equal protection.

ISSUE: Whether or notthere is a violation of equal


protection.

HELD: The thrust of the1973 Constitution is towards the


fullest autonomy of local government units. Inthe
Declaration of Principles and State Policies, it is stated that
"TheState shall guarantee and promote the autonomy of local
government units to ensure their fullest development as selfreliant communities. The petitioner's allegation of
gerrymandering is of no merit, it has no factual or legal
basis. The Constitutional requirement that the creation,
division, merger, abolition, oralteration of the boundary of a
province, city, municipality, or barrio shouldbe subject to the
approval by the majority of the votes cast in a plebiscite in
the governmental unit or units affected is a new requirement
that came intobeing only with the 1973 Constitution. It is
prospective in character and therefore cannot affect the
creation of the City of Mandaue which came into existence
on 21 June 1969.

The classification of cities into highly urbanized


cities and component cities on the basis of their regular
annual income is based upon substantial distinction.The
revenue of a city would show whether or not it is capable of
existence and development as a relatively independent
social, economic, and political unit. It would also show
whether the city has sufficient economic or industrial activity
as to warrant its independence from the province where it is
geographically situated. Cities with smaller income need the
continued supportof the provincial government thus

justifying the continued participation of thevoters in the


election of provincial officials in some instances.

ISSUE: Whether or notUNIDO was denied equal protection


by virtue of COMELECs denial of their request.

The petitioners also contend that the voters in Mandaue City


aredenied equal protection of the law since the voters in
other component citiesare allowed to vote for provincial
officials. The contention is without merit. The practice of
allowing voters in one component city to vote for provincial
officials and denying the same privilege to voters in another
component city isa matter of legislative discretion which
violates neither the Constitution northe voter's right of
suffrage.

HELD: The SC ruled that UNIDO was not denied due


process nor were they not afforded equal protection. It is the
considered view of the SC that when Marcos conducted his
'pulong-pulong' or consultation with the people on March 12,
1981, he did so in his capacity asPresident/Prime Minister of
the Philippines and not as the head of any political party.
Under the Constitution, the 'Prime Minister and the Cabinet
shall be responsible . . . for the program of government and
shall determinethe guidelines of national policy'. In instances
where the head of state is atthe same time the president of
the political party that is in power, it does not necessarily
follow that he speaks with two voices when he dialogues
with the governed. The president is accorded certain
privileges that the oppositionmay not have. Further, the SC
cannot compel TV stations and radio stations to give UNIDO
free air time as they are not party to this case. UNIDO must
sought contract with these TV stations and radio stations at
their own expense.

likewise was shrunk and limited only to questions of law,


excluding a review of the facts and trial evidence; and there
is only one chance to appeal conviction, by certiorari to the
SC, instead of the traditional two chances; while all other
estafa indictees are entitled to appeal as a matter of right
covering both law and facts and to two appellate courts, i.e.,
first to the CA and thereafter to the SC.
ISSUE: Whether or not the creation of Sandiganbayan
violates equal protection insofar as appeals would be
concerned.

HELD: The SC ruled against Nuez. The 1973 Constitution


had provided for the creation of a special court that shall
have original jurisdiction over cases involving public
G.R. No. 56515: United Democratic Opposition vs
officials charged with graft and corruption. The constitution
Commission on Elections
specifically makes mention of the creation of a special court,
EqualProtection Access to Media
the Sandiganbayan, precisely in response to a problem, the
urgency of which cannot be denied, namely, dishonesty in
the public service. It follows that those who may thereafter
be tried by such court ought to have been aware as far back
In 1981, the BP proposed amendments to the 1973
as January 17, 1973, when the present Constitution came
Constitution. The amendments were to be placed to a
into force, that a different procedure for the accused therein,
plebiscite for the peoples approval. The YES vote was being
whether a private citizen as petitioner is or a public official,
advanced by KBL Marcos Party. While the NO vote was
Rufino Nuez vs Sandiganbayan & the People of is not necessarily offensive to the equal protection clause of
being advanced by UNIDO. To ensure parity and equality,
the Philippines
the Constitution. Further, the classification therein set forth
COMELEC issued Resolutions 1467-1469 w/c basically
met the standard requiring that it must be based on
provided that there be equal opportunity, equal time and
Equal Protection Creation of the Sandiganbayan
substantial distinctions which make real differences; it must
equal space on media use for campaigns for both sides. On
be germane to the purposes of the law; it must not be limited
12 Mar 1981, Marcos campaigned for the YES vote via TV Nuez assails the validity of the PD 1486 creating the
to existing conditions only, and must apply equally to each
and radio from 9:30pm to 11:30pm. Thesame was
Sandiganbayan as amended by PD 1606. He was accused
member of the class. Further still, decisions in the
broadcasted live by 26 TV stations and 248 radio stations
before the Sandoganbayan of estafa through falsification of Sandiganbayan are reached by a unanimous decision from 3
nationwide.UNIDO petitioned before the COMELEC that
public and commercial documents committed in connivance justices - a showing that decisions therein are more
they be granted the same opportunity as Marcos had
with his other co-accused, all public officials, in several
conceivably carefully reached than other trial courts.
pursuant to Resns 1467-69. COMELEC denied the demand. cases. It is the claim of Nuez that PD1486, as amended, is
UNIDO assailed the denial as a denial of equal protection
violative of the due process, equal protection, and ex post
Justice Makasiar (concurring & dissenting)
before the laws.
facto clauses of the Constitution. He claims that the
Sandiganbayan proceedings violates Nuezs right to equal
Persons who are charged with estafa or malversation of
protection, because appeal as a matter of right became
funds not belonging to the government or any of its
minimized into a mere matter of discretion; appeal
instrumentalities or agencies are guaranteed the right to

appeal to two appellate courts first, to the CA, and


thereafter to the SC. Estafa and malversation of private funds
are on the same category as graft and corruption committed
by public officers, who, under the decree creating the
Sandiganbayan, are only allowed one appeal to the SC
(par. 3, Sec. 7, P.D. No. 1606). The fact that the
Sandiganbayan is a collegiate trial court does not generate
any substantial distinction to validate this invidious
discrimination. Three judges sitting on the same case does
not ensure a quality of justice better than that meted out by a
trial court presided by one judge. The ultimate decisive
factors are the intellectual competence, industry and integrity
of the trial judge. But a review by two appellate tribunals of
the same case certainly ensures better justice to the accused
and to the people.

Equal Protection

Sison assails the validity of BP 135 w/c further amended Sec


21 of the National Internal Revenue Code of 1977. The law
provides that thered be a higher tax impost against income
derived from professional income as opposed to regular
income earners. Sison, as a professional businessman, and as
taxpayer alleges that by virtue thereof, he would be unduly
discriminated against by the imposition of higher rates of tax
upon his income arising from the exercise of his profession
vis-a-vis those which are imposed upon fixed income or
salaried individual taxpayers. He characterizes the above
section as arbitrary amounting to class legislation,
oppressive and capricious in character. There is a
transgression of both the equal protection and due process
clauses of the Constitution as well as of the rule requiring
Then again, par 3 of Sec 7 of PD 1606, by providing that the uniformity in taxation.
decisions of the Sandiganbayan can only be reviewed by the
SC through certiorari, likewise limits the reviewing power of ISSUE: Whether the imposition of a higher tax rate on
taxable net income derived from business or profession than
the SC only to question of jurisdiction or grave abuse of
on compensation is constitutionally infirm.
discretion, and not questions of fact nor findings or
conclusions of the trial court. In other criminal cases
involving offenses not as serious as graft and corruption, all HELD: The SC ruled against Sison. The power to tax, an
inherent prerogative, has to be availed of to assure the
questions of fact and of law are reviewed, first by the CA,
performance of vital state functions. It is the source of the
and then by the SC. To repeat, there is greater guarantee of
bulk of public funds. Taxes, being the lifeblood of the
justice in criminal cases when the trial courts judgment is
government, their prompt and certain availability is of the
subject to review by two appellate tribunals, which can
essence. According to the Constitution: The rule of taxation
appraise the evidence and the law with greater objectivity,
detachment and impartiality unaffected as they are by views shall be uniform and equitable. However, the rule of
uniformity does not call for perfect uniformity or perfect
and prejudices that may be engendered during the trial.
equality, because this is hardly attainable. Equality and
uniformity in taxation means that all taxable articles or kinds
Limiting the power of review by the SC of convictions by
of property of the same class shall be taxed at the same rate.
the Sandiganbayan only to issues of jurisdiction or grave
The taxing power has the authority to make reasonable and
abuse of discretion, likewise violates the constitutional
natural classifications for purposes of taxation. Where the
presumption of innocence of the accused, which
differentiation complained of conforms to the practical
presumption can only be overcome by proof beyond
dictates of justice and equity it is not discriminatory within
reasonable doubt (Sec. 19, Art. IV, 1973 Constitution).
the meaning of this clause and is therefore uniform. There
Antero Sison Jr. vs Acting BIR Commissioner Ruben
is quite a similarity then to the standard of equal protection
Ancheta et al

for all that is required is that the tax applies equally to all
persons, firms and corporations placed in similar situation.
What misled Sison is his failure to take into consideration
the distinction between a tax rate and a tax base. There is no
legal objection to a broader tax base or taxable income by
eliminating all deductible items and at the same time
reducing the applicable tax rate. Taxpayers may be classified
into different categories. In the case of the gross income
taxation embodied in BP 135, the discernible basis of
classification is the susceptibility of the income to the
application of generalized rules removing all deductible
items for all taxpayers within the class and fixing a set of
reduced tax rates to be applied to all of them. Taxpayers who
are recipients of compensation income are set apart as a
class. As there is practically no overhead expense, these
taxpayers are not entitled to make deductions for income tax
purposes because they are in the same situation more or less.
On the other hand, in the case of professionals in the practice
of their calling and businessmen, there is no uniformity in
the costs or expenses necessary to produce their income. It
would not be just then to disregard the disparities by giving
all of them zero deduction and indiscriminately impose on
all alike the same tax rates on the basis of gross income.
There is ample justification then for the Batasang Pambansa
to adopt the gross system of income taxation to
compensation income, while continuing the system of net
income taxation as regards professional and business
income.
Citizens Surety & Insurance Co., Inc. vs Judge Ricardo
Puno, Register of Deeds Manila
Equal Protection Purchase of Land Barrio Obrero
In 1956, Resolution 542 was passed by the Register of Deeds
Manila which provided that only Filipino laborers whose
wages do not exceed P180.00/month or P6.00/day and at the
same time residents of Manila may be allowed to purchase
lands in Barrio Obrero, Tondo, Manila. On 10 Oct 1966,
Maria Barcelon mortgaged her 180 sq. m. land located in

Barrio Obrero to CSICI. CSICI foreclosed the property due


to nonpayment and later bought the land. CSICI later sought
to register and consolidate the land before the Register of
Deeds but then Justice Puno denied the request pursuant to
Resn 542 as CSICI does not meet the qualification. CSICI
averred that Resn 542 is null and void. It averred: As may
be seen from Sec 4 of Resn 542, only laborers earning not
more than P180.00 a month, or P5.00 a day are qualified to
buy Lands in Barrio Obrero. Employees working in offices
or establishments and earning as much but who are not
laborers cannot buy lands in that area. Also persons who are
engaged in some calling or occupation earning as much are
not also qualified. It should not be overlooked that the
intention of the pertinent provisions of the Charter of the
City of Manila contained in Sections 97, 98 and 100 of said
Charter is to help the poor people of Manila to acquire
residential lands on easy terms. CSICI points out that there
is no substantial difference between these laborers to those
mentioned in the Resolution.

Equal Protection 1978 Election Code Block Voting

or as a candidate of a party group or aggrupation. The voter


is free to vote for the individual candidates or to vote by
Peralta was an independent candidate in the April 1978
party, group or aggrupation. The choice is his. No one can
Interim Batasang Pambansa Elections. He, along with others, compel him to do otherwise. In the case of candidates, the
assailed the constitutionality of PD 1269 or the 1978
decision on whether to run as an independent candidate or to
Election Code. Secs140 and 155, sub-paragraphs 26 to 28, of join a political party, group or aggrupation is left entirely to
the 1978 Election Code, grants the voter the option to vote
their discretion. Certainly, before filing his certificate of
either for individual candidates by filling in the proper
candidacy, a candidate is aware of the advantages under the
spaces in the ballot the names of candidates he desires to
law accruing to candidates of a political party or group. If he
elect, or to vote for all the candidates of a political party,
wishes to avail himself of such alleged advantages as an
group or aggrupation by simply writing in the space
official candidate of a party, he is free to do so by joining a
provided for in the ballot the name of the political party,
political party group or aggrupation. In other words, the
group or aggrupation (office-block ballot). Peralta was
choice is his. In making his decision, it must be assumed that
vehement in contending that the optional block voting
the candidate had carefully weighed and considered the
scheme is violative of this provision of the Constitution:
relative advantages and disadvantages of either alternative.
Bona fide candidates for any public office shall be free
So long as the application of the rule depends on his
from any form of harassment and discrimination. He
voluntary action or decision, he cannot, after exercising his
sought the shelter of its protection for himself and other
discretion, claim that he was the victim of discrimination.
independent candidates who, according to him, would be
thus made to suffer if the assailed provision is not nullified. HAWAIIAN-PHILIPPINE COMPANY vs
ASOCIACION DE HACENDEROS DE SILAYISSUE: Whether or not Resolution 542 violates equal
Essentially, in terms of individual rights, he would raise a
SARAVIA, INC., et al
protection.
due process and equal protection question. The main
Equal Protection Sugar Cane Industry
objection of Peralta against the optional straight party voting
HELD: The SC ruled against CSICI. CSICI, which is a
provided for in the Code is that an independent candidate
Hawaiian is a sugar milling company while Asociacion is a
corporation and not a lowly paid worker, is not competent to would be discriminated against because by merely writing
raise this claim. For even if the SC sustain it, no benefit can on his ballot the name of a political party, a voter would have corporation organized to represent sugar cane planters in
accrue to CSICI who will nonetheless be disqualified to
voted for all the candidates of that party, an advantage which Negros Occidental. In 1953, both had a contract which
would include 12 crop (years) until the 1963-1964 crop. For
acquire the lot. Moreover, in the absence of manifest abuse
the independent candidate does not enjoy. In effect, it is
the 1st 6 years, the sharing would be 63% to 37%, Asociacion
of power, the SC not vent to substitute their judgment for
contended that the candidate who is not a party-member is
being the highest getter. And from the next 5 years itd be
that of the City of Manila which is tasked by its Charter to deprived of the equal protection of the laws, as provided in
acquire private lands in the city and to subdivide the same
Sec 1 of Article IV, in relation to Sec 9 of Article XII, of the 63.5% to 36.5% and in the 63-64 crop year itd be 64% to
36%. In 1961, Asociacion made known its intention to buy
into home lots for sale on easy terms to residents, giving first 1973 Constitution.
out Hawaiian which was reluctant at first but later agreed at
priority to the bona-fide tenants or occupants of said lands,
and second priority to laborers and low-salaried employees. ISSUE: Whether or not the 1978 Election Code is violative a selling price of $14M. The sale was never actualized and
Asociacion initially sought to restructure the contract to a
Obviously, the questioned resolution merely seeks to
of equal protection.
70%-30% sharing. Eventually, Asociacion severed ties with
implement the Charter provision.
Hawaiian and Asocicion established its own milling
HELD: The SC ruled that the 1978 Election Code is valid.
company, the Agricultural Industrial Development Company
Before a voter prepares his ballot, the voter will be able to
of Silay-Saravia. Apparently, Asociacion is authorized by
read all the names of the candidates. No candidate will
Peralta et al vs Commission on Elections et al
receive more than one vote, whether he is voted individually law to break existing contracts by virtue of RA 809.

Hawaiian assailed the constitutionality of the said law: Secs


1, 4 and 9 of RA 809, Secs 4 and 5 of RA 1825 and Sec 3 of
RA 1072 amending Sec 9 of Act 4166, for being violative of
the constitutional guarantees against impairment of the
freedom of contracts, denial of equal protection of the laws,
taking of private property for public use without due process
and without just compensation and impairment of vested
rights and (2) validity of: aforesaid laws for being violative
of treaty commitments previously entered into by the
Government of the Republic of the Philippines.

Equal Protection

In 1964, Ormoc City passed a bill which read: There shall


be paid to the City Treasurer on any and all productions of
centrifugal sugar milled at the Ormoc Sugar Company
Incorporated, in Ormoc City a municipal tax equivalent to
one per centum (1%) per export sale to the United States of
America and other foreign countries. Though referred to as
a production tax, the imposition actually amounts to a tax
on the export of centrifugal sugar produced at Ormoc Sugar
Company, Inc. For production of sugar alone is not taxable;
ISSUE: Whether or not there has been a violation of equal
the only time the tax applies is when the sugar produced is
protection.
exported. Ormoc Sugar paid the tax (P7,087.50) in protest
averring that the same is violative of Sec 2287 of the
HELD: The SC ruled that there is no such violation. RA
Revised Administrative Code which provides: It shall not
809 is a social justice and police power measure for the
be in the power of the municipal council to impose a tax in
promotion of labor conditions in sugar plantations, hence
any form whatever, upon goods and merchandise carried into
whatever rational degree of constraint it exerts on freedom of the municipality, or out of the same, and any attempt to
contract and existing contractual obligations is
impose an import or export tax upon such goods in the guise
constitutionally permissible. RA 1825 and RA 1072
of an unreasonable charge for wharfage, use of bridges or
amending Act 4166 covering as they do the same subject, i.e. otherwise, shall be void. And that the ordinance is violative
sugar production, partake of the same nature as RA 809 and to equal protection as it singled out Ormoc Sugar As being
for the same reasons as above stated, cannot be considered
liable for such tax impost for no other sugar mill is found in
constitutionally objectionable. Sugar production is one of the the city.
great industries of our nation, sugar occupying a leading
position among its export products, that it gives employment ISSUE: Whether or not there has been a violation of equal
to thousands of laborers in field and factories, that it is a
protection.
great source of the states wealth, is one of the important
sources of foreign exchange needed by our government, and HELD: The SC held in favor of Ormoc Sugar. The SC noted
is thus pivotal in the plans of a regime committed to a policy that even if Sec 2287 of the RAC had already been repealed
by a latter statute (Sec 2 RA 2264) which effectively
of currency stability. Its promotion, protection and
authorized LGUs to tax goods and merchandise carried in
advancement therefore, redounds greatly to the general
and out of their turf, the act of Ormoc City is still violative
welfare. Hence it was competent for the legislature to find
of equal protection. The ordinance is discriminatory for it
that the general welfare demanded that the sugar industry
should be stabilized in turn, and in the wide field of its police taxes only centrifugal sugar produced and exported by the
Ormoc Sugar Company, Inc. and none other. At the time of
power, the lawmaking body could provide that the
the taxing ordinances enactment, Ormoc Sugar Company,
distribution of benefits therefrom be readjusted among its
Inc., it is true, was the only sugar central in the city of
components.
Ormoc. Still, the classification, to be reasonable, should be
Ormoc Sugar Company Inc. vs Ormoc City et al
in terms applicable to future conditions as well. The taxing

ordinance should not be singular and exclusive as to exclude


any subsequently established sugar central, of the same class
as plaintiff, from the coverage of the tax. As it is now, even if
later a similar company is set up, it cannot be subject to the
tax because the ordinance expressly points only to Ormoc
Sugar Company, Inc. as the entity to be levied upon.
Flores vs.
C
o
m
elec
Facts:The petitioner, Roque Flores, was proclaimed by theboard of
canvassers as having received the highest number of votes for kagawad
in Brgy. Poblacion, Tayum, Abra, andthus became punong barangay
pursuant to Sec 5 RA 6679.He was voted
punong barangay
during the
1
9
8
2 elections, aseparate position as that of
Kagawad.
The private respondent, Nobelito Rapisora,protected the result and filed a
protect before the MCTCTayum. He argued that the ballot, which only
indicatedFlores, should be declared stray votes and should not
bedivided equally to them.In his defense, the petitioner argued that
inaccordance with the Omnibus Election Code, the 4questioned votes
should be entitled to him under the
equity of the incumbent rule,
which states that if there are 2 or morecandidates with the same full name
and one of them is anincumbent and the ballot is written only on such full
name,the vote is counted in favor of the incumbent.The lower court
sustained the contention of theprivate respondent and subsequently
declared him as thepunong barangay.Hence this petition. The petitioner
argued that bynot following the rule stated, he is deprived of his right
toequal protection of the law since he is also an
incumbent
punong barangay running for election, thereby he should beentitled by
the rule.Issue:Was the petitioner considered an
incumbent
to beentitled under the rule?Held:No. Under the new rule Resolution 2
0

22- A passedby the Comelec, Barangay Captains who filed


their candidacy for the office of Kagawad, which is another office,shall
be deemed resigned in their former office.

crime of terrorism or the crime of conspiracy to commit


terrorism shall, without incurring any criminal liability for
delay in the delivery of detained persons to the proper
judicial authorities, deliver said charged or suspected person
In his filing of candidacy, it stated that he is runningfor
to the proper judicial authority within a period of three (3)
kagawad
days counted from the moment the said charged or suspected
and not as a punong barangay. Thus, pursuantto the resolution, he
person has been apprehended or arrested, detained, and taken
deemed to resign his position as punongbarangay when he filed for his
into custody by the said police, or law enforcement
candidacy as a kagawad.The rule cannot thus then be applied to
thepetitioner since pursuant to the resolution, he is notconsidered as an
personnel: Provided, That the arrest of those suspected of the
incumbent punong barangay, he is notwithin the same class as that of the crime of terrorism or conspiracy to commit terrorism must
incumbents.The court cannot sustain the argument of thepetitioner that
result from the surveillance under Section 7 and examination
since RA 669 speaks of 7 candidates for kagawad, the foremost of them
of bank deposits under Section 27 of this Act.
is the punong barangay, heshould be regarded as running for the same
office.
The police or law enforcement personnel concerned shall,
before detaining the person suspected of the crime of
Constitutional Law II (Bill of Rights): Case Briefs: Dennis G.
terrorism, present him or her before any judge at the
Libunao UC College of Law
17
latters residence or office nearest the place where the
There is a substantial distinction between Punongbarangay and kagawad. arrest took place at any time of the day or night. It shall be
The former is vested with executivepower and the latter legislative power the duty of the judge, among other things, to ascertain the
pursuant to the LGC.Note:In
identity of the police or law enforcement personnel and the
1
person or persons they have arrested and presented before
9
him or her, to inquire of them the reasons why they have
8
arrested the person and determine by questioning and
2 procedures, there are separate election for Punong barngay and
personal observation whether or not the suspect has been
members of the Sanguniangpambarangay.In
subjected to any physical, moral or psychological torture by
1
9
whom and why. The judge shall then submit a written report
8
of what he/she had observed when the subject was brought
9 elections, the only disputed position for direct election is the
before him to the proper court that has jurisdiction over the
Kagawad
case of the person thus arrested. the judge shall forthwith
and PB will be granted byoperation of law to the highest vote earned.
submit his/her report within three (3) calendar days from the
time the suspect was brought to his/her residence or office.
RA 9372
SEC. 18. Period of Detention Without Judicial Warrant of
Arrest. The provisions of Article 125 of the Revised
Penal Code to the contrary notwithstanding, any police or
law enforcement personnel, who, having been duly
authorized in writing by the Anti-Terrorism Council has
taken custody of a person charged with or suspected of the

The penalty of ten (10) years and one day to twelve (12)
years of imprisonment shall be imposed upon the police or
law enforcement personnel who fails to notify any judge as
provided in the preceding paragraph.
SEC. 19. Period of Detention in the Event of an Actual or
Imminent Terrorist Attack. In the event of an actual or
imminent terrorist attack, suspects may not be detained for
more than three (3) days without the written approval of a
municipal, city, provincial or regional official of a Human
Rights Commission or judge of the municipal, regional trial
court, the Sandiganbayan or a justice of the Court of Appeals
nearest the place of the arrest. If the arrest is made during
Saturdays, Sundays, holidays or after office hours, the
arresting police or law enforcement personnel shall bring the
person thus arrested to the residence of any of the officials
mentioned above that is nearest the place where the accused
was arrested. The approval in writing of any of the said
officials shall be secured by the police or law enforcement
personnel concerned within five (5) days after the date of the
detention of the persons concerned: Provided, however, That
within three (3) days after the detention the suspects, whose
connection with the terror attack or threat is not established,
shall be released immediately.

SEC. 26. Restriction on Travel. In cases where evidence of


guilt is not strong, and the person charged with the crime of
terrorism or conspiracy to commit terrorism is entitled to bail
and is granted the same, the court, upon application by the
prosecutor, shall limit the right of travel of the accused to
Immediately after taking custody of a person charged with or within the municipality or city where he resides or where the
suspected of the crime of terrorism or conspiracy to commit case is pending, in the interest of national security and public
safety, consistent with Article III, Section 6 of the
terrorism, the police or law enforcement personnel shall
Constitution. Travel outside of said municipality or city,
notify in writing the judge of the court nearest the place of
without the authorization of the court, shall be deemed a
apprehension or arrest: Provided, That where the arrest is
violation of the terms and conditions of his bail, which shall
made during saturdays, sundays, holidays or after office
then be forfeited as provided under the Rules of Court.
hours, the written notice shall be served at the residence of
the judge nearest the place where the accused was arrested.

He or she may also be placed under house arrest by order of


the court at his or her usual place of residence.
While under house arrest, he or she may not use telephones,
cellphones, e-mails, computers, the internet or other means
of communications with people outside the residence until
otherwise ordered by the court.
The restrictions abovementioned shall be terminated upon
the acquittal of the accused or of the dismissal of the case
filed against him or earlier upon the discretion of the court
on motion of the prosecutor or of the accused.

CA dismissed the appeal for a certiorari is not the proper


remedy.
ISSUE: Whether or not there was a valid search warrant
issued.
HELD: The SC ruled in favor of UPC and Uy in a way for it
ordered the return of the seized items but sustained the
validity of the warrant. The SC ruled that the search warrant
issued has not met some basic requisites of validity. A search
warrant must conform strictly to the requirements of the
foregoing constitutional and statutory provisions. These
requirements, in outline form, are:

Frank Uy & Unifish Packing Corp. vs Bureau of Internal (1) the warrant must be issued upon probable cause;
Revenue et al
Search and Seizure Requisites of a Valid Search Warrant
(2) the probable cause must be determined by the judge
himself and not by the applicant or any other person;
In Sept 1993, Rodrigo Abos, a former employee of UPC
reported to the BIR that Uy Chin Ho aka Frank Uy, manager
(3) in the determination of probable cause, the judge must
of UPC, was selling thousands of cartons of canned cartons
examine, under oath or affirmation, the complainant and
without issuing a report. This is a violation of Sec 253 & 263
such witnesses as the latter may produce; and
of the Internal Revenue Code. In Oct 1993, the BIR
requested before RTC Cebu to issue a search warrant. Judge
(4) the warrant issued must particularly describe the place to
Gozo-Dadole issued a warrant on the same day. A second
be searched and persons or things to be seized.
warrant was issued which contains the same substance but
has only one page, the same was dated Oct 1st 2003. These
warrants were issued for the alleged violation by Uy of Sec
253. A third warrant was issued on the same day for the
The SC noted that there has been inconsistencies in the
alleged violation of Uy of Sec 238 in relation to sec 263. On
description of the place to be searched as indicated in the
the strength of these warrants, agents of the BIR,
said warrants. Also the thing to be seized was not clearly
accompanied by members of the PNP, on 2 Oct 1993,
defined by the judge. He used generic itineraries. The
searched the premises of the UPC. They seized, among other
warrants were also inconsistent as to who should be
things, the records and documents of UPC. A return of said
searched. One warrant was directed only against Uy and the
search was duly made by Labaria with the RTC of Cebu.
other was against Uy and UPC. The SC however noted that
UPC filed a motion to quash the warrants which was denied
the inconsistencies wered cured by the issuance of the latter
by the RTC. They appealed before the CA via certiorari. The
warrant as it has revoked the two others.

Section 2, Article III of the Constitution guarantees the right


of the people against unreasonable searches and seizures:

The right of the people to be secure in their persons, houses,


papers, and effects against unreasonable searches and
seizures of whatever nature and for any purpose shall be
inviolable, and no search warrant or warrant of arrest shall
issue except upon probable cause to be determined
personally by the judge after examination under oath or
affirmation of the complainant and the witnesses he may
produce, and particularly describing the place to be searched
and the persons or things to be seized.

NOTES

Rule 126 of the Rules of Court provides:

SEC. 3. Requisite for issuing search warrant. A search


warrant shall not issue but upon probable cause in
connection with one specific offense to be determined
personally by the judge after examination under oath or
affirmation of the complainant and the witnesses he may
produce, and particularly describing the place to be searched
and the things to be seized.

SEC. 4. Examination of complainant; record. The judge


must, before issuing the warrant, personally examine in the
form of searching questions and answers, in writing and

under oath the complainant and any witnesses he may


produce on facts personally known to them and attach to the
record their sworn statements together with any affidavits
submitted.

foregoing constitutional and statutory provisions. These


requirements, in outline form, are:

(1) the warrant must be issued upon probable cause;


Frank Uy & Unifish Packing Corp. vs Bureau of Internal
Revenue et al
(2) the probable cause must be determined by the judge
Search and Seizure Requisites of a Valid Search Warrant
himself and not by the applicant or any other person;
In Sept 1993, Rodrigo Abos, a former employee of UPC
reported to the BIR that Uy Chin Ho aka Frank Uy, manager
of UPC, was selling thousands of cartons of canned cartons
without issuing a report. This is a violation of Sec 253 & 263
of the Internal Revenue Code. In Oct 1993, the BIR
requested before RTC Cebu to issue a search warrant. Judge
Gozo-Dadole issued a warrant on the same day. A second
warrant was issued which contains the same substance but
has only one page, the same was dated Oct 1st 2003. These
warrants were issued for the alleged violation by Uy of Sec
253. A third warrant was issued on the same day for the
alleged violation of Uy of Sec 238 in relation to sec 263. On
the strength of these warrants, agents of the BIR,
accompanied by members of the PNP, on 2 Oct 1993,
searched the premises of the UPC. They seized, among other
things, the records and documents of UPC. A return of said
search was duly made by Labaria with the RTC of Cebu.
UPC filed a motion to quash the warrants which was denied
by the RTC. They appealed before the CA via certiorari. The
CA dismissed the appeal for a certiorari is not the proper
remedy.
ISSUE: Whether or not there was a valid search warrant
issued.
HELD: The SC ruled in favor of UPC and Uy in a way for it
ordered the return of the seized items but sustained the
validity of the warrant. The SC ruled that the search warrant
issued has not met some basic requisites of validity. A search
warrant must conform strictly to the requirements of the

(3) in the determination of probable cause, the judge must


examine, under oath or affirmation, the complainant and
such witnesses as the latter may produce; and

issue except upon probable cause to be determined


personally by the judge after examination under oath or
affirmation of the complainant and the witnesses he may
produce, and particularly describing the place to be searched
and the persons or things to be seized.

NOTES

Rule 126 of the Rules of Court provides:

(4) the warrant issued must particularly describe the place to


be searched and persons or things to be seized.

The SC noted that there has been inconsistencies in the


description of the place to be searched as indicated in the
said warrants. Also the thing to be seized was not clearly
defined by the judge. He used generic itineraries. The
warrants were also inconsistent as to who should be
searched. One warrant was directed only against Uy and the
other was against Uy and UPC. The SC however noted that
the inconsistencies wered cured by the issuance of the latter
warrant as it has revoked the two others.

Section 2, Article III of the Constitution guarantees the right


of the people against unreasonable searches and seizures:

SEC. 3. Requisite for issuing search warrant. A search


warrant shall not issue but upon probable cause in
connection with one specific offense to be determined
personally by the judge after examination under oath or
affirmation of the complainant and the witnesses he may
produce, and particularly describing the place to be searched
and the things to be seized.

SEC. 4. Examination of complainant; record. The judge


must, before issuing the warrant, personally examine in the
form of searching questions and answers, in writing and
under oath the complainant and any witnesses he may
produce on facts personally known to them and attach to the
record their sworn statements together with any affidavits
submitted.
People of the Philippines vs Rosa Aruta
y Menguin

The right of the people to be secure in their persons, houses,


papers, and effects against unreasonable searches and
seizures of whatever nature and for any purpose shall be
inviolable, and no search warrant or warrant of arrest shall

Search and Seizure Informers Tip

In the morning of 13 Dec 1988, the law enforcement officers


received information from an informant named Benjie that
a certain Aling Rosa would be leaving for Baguio City on
14 Dec 1988 and would be back in the afternoon of the same
day carrying with her a large volume of marijuana; At 6:30
in the evening of 14 Dec 1988, Aruta alighted from a Victory
Liner Bus carrying a travelling bag even as the informant
pointed her out to the law enforcement officers; NARCOM
officers approached her and introduced themselves as
NARCOM agents; When asked by Lt. Abello about the
contents of her travelling bag, she gave the same to him;
When they opened the same, they found dried marijuana
leaves; Aruta was then brought to the NARCOM office for
investigation.

cause and the accused-appellant not having been lawfully


arrested. Stated otherwise, the arrest being incipiently illegal,
it logically follows that the subsequent search was similarly (c) the evidence must be immediately apparent, and
illegal, it being not incidental to a lawful arrest. The
constitutional guarantee against unreasonable search and
seizure must perforce operate in favor of accused-appellant.
(d) plain view justified mere seizure of evidence without
As such, the articles seized could not be used as evidence
against accused-appellant for these are fruits of a poisoned further search;
tree and, therefore, must be rejected, pursuant to Article III,
Sec. 3(2) of the Constitution.

ISSUE: Whether or not the conducted search and seizure is


constitutional.

NOTES:

HELD: The SC ruled in favor of Aruta and has noted that


some drug traffickers are being freed due to technicalities.
Aruta cannot be said to be committing a crime. Neither was
she about to commit one nor had she just committed a crime.
Aruta was merely crossing the street and was not acting in
any manner that would engender a reasonable ground for the
NARCOM agents to suspect and conclude that she was
committing a crime. It was only when the informant pointed
to Aruta and identified her to the agents as the carrier of the
marijuana that she was singled out as the suspect. The
NARCOM agents would not have apprehended Aruta were it
not for the furtive finger of the informant because, as clearly
illustrated by the evidence on record, there was no reason
whatsoever for them to suspect that accused-appellant was
committing a crime, except for the pointing finger of the
informant. The SC could neither sanction nor tolerate as it is
a clear violation of the constitutional guarantee against
unreasonable search and seizure. Neither was there any
semblance of any compliance with the rigid requirements of
probable cause and warrantless arrests. Consequently, there
was no legal basis for the NARCOM agents to effect a
warrantless search of Arutas bag, there being no probable

Read full text here.

3. Search of a moving vehicle. Highly regulated by the


government, the vehicles inherent mobility reduces
expectation of privacy especially when its transit in public
thoroughfares furnishes a highly reasonable suspicion
amounting to probable cause that the occupant committed a
criminal activity;

When is a warrantless search allowed?


4. Consented warrantless search;
1. Warrantless search incidental to a lawful arrest recognized
under Section 12, Rule 126 of the Rules of Court 8 and by
prevailing jurisprudence;
5. Customs search;

2. Seizure of evidence in plain view, the elements of which


6. Stop and Frisk; and
are:

7. Exigent and Emergency Circumstances.


(a) a prior valid intrusion based on the valid warrantless
arrest in which the police are legally present in the pursuit of
their official duties;
The People of the Philippines vs Ruben Montilla y
Gatdula
Political Law Search and Seizure Informers Tip
Warrantless Arrest
(b) the evidence was inadvertently discovered by the police
who had the right to be where they are;

On 19 June 1994 at about 2pm, police officers Talingting and


Clarin were informed by an asset that a drug courier would
be arriving from Baguio to Dasmarias carrying an
undetermined amount of marijuana. The next day, the
informant pointed at Montilla as the courier who was waiting
in a waiting shed Brgy Salitran, Dasmarias. Montilla was
then apprehended and he was caught in possession of a bag
and a carton worth 28 kilos of marijuana. Montilla denied
the allegation and he said he came to Cavite from Baguio for
work and he does not have any effects with him at that time
except for some pocket money. He was sentenced to death
thereafter. He averred that the search and seizure conducted
was illegal for there was no warrant and that he should have
been given the opportunity to cross examine the informant.
He said that if the informant has given the cops the
information about his arrival as early as the day before his
apprehension, the cops should have ample time to secure a
search warrant.

coming from Baguio in the early morning of June 20,


1994. Even assuming that the policemen were not pressed
for time, this would be beside the point for, under these
circumstances, the information relayed was too sketchy and
not detailed enough for the obtention of the corresponding
arrest or search warrant. While there is an indication that the
informant knew the courier, the records do not reveal that he
knew him by name.
On such bare information, the police authorities could not
have properly applied for a warrant, assuming that they
could readily have access to a judge or a court that was still
open by the time they could make preparations for applying
therefor, and on which there is no evidence presented by the
defense. In determining the opportunity for obtaining
warrants, not only the intervening time is controlling but all
the coincident and ambient circumstances should be
considered, especially in rural areas.

persons above-named and/or the premises of their offices,


warehouses and/or residences, and to seize and take
possession of the following personal property to wit:
Books of accounts, financial records, vouchers,
correspondence, receipts, ledgers, journals, portfolios, credit
journals, typewriters, and other documents and/or papers
showing all business transactions including disbursements
receipts, balance sheets and profit and loss statements and
Bobbins (cigarette wrappers).
The documents, papers, and things seized under the alleged
authority of the warrants in question may be split into (2)
major groups, namely:
(a) those found and seized in the offices of the
aforementioned corporations and
(b) those found seized in the residences of petitioners herein.

ISSUE: Whether or not the warrantless arrest conducted is


legal.

A legitimate warrantless arrest, as above contemplated,


necessarily cloaks the arresting police officer with authority
to validly search and seize from the offender

HELD: The SC ruled that the warrantless arrest is legal. Sec


(1) dangerous weapons, and
2 Art 3 of the Constitution has its exception, they are: (1)
customs searches;
(2) those that may be used as proof of the commission of an
offense.
(2) searches of moving vehicles,
(3) seizure of evidence in plain view;

Harry Stonehill et al vs DOJ Secretary Jose


Diokno et al

(4) consented searches;


(5) searches incidental to a lawful arrest;
(6) stop and frisk measures have been invariably
recognized as the traditional exceptions.
In the case at bar, it should be noted that the information
relayed by informant to the cops was that there would be
delivery of marijuana at Barangay Salitran by a courier

Search and Seizure General Warrants Abandonment of


the Moncado Doctrine
Stonehill et al and the corporation they form were alleged to
have committed acts in violation of Central Bank Laws,
Tariff and Customs Laws, Internal Revenue (Code) and
Revised Penal Code. By the strength of this allegation a
search warrant was issued against their persons and their
corporation. The warrant provides authority to search the

Stonehill averred that the warrant is illegal for:


(1) they do not describe with particularity the documents,
books and things to be seized;
(2) cash money, not mentioned in the warrants, were actually
seized;
(3) the warrants were issued to fish evidence against the
aforementioned petitioners in deportation cases filed against
them;
(4) the searches and seizures were made in an illegal manner;
and
(5) the documents, papers and cash money seized were not
delivered to the courts that issued the warrants, to be
disposed of in accordance with law.

The prosecution counters, invoking the Moncado doctrine,


that the defects of said warrants, if any, were cured by
petitioners consent; and (3) that, in any event, the effects
seized are admissible in evidence against them. In short, the
criminal cannot be set free just because the government
blunders.
ISSUE: Whether or not the search warrant issue is valid.
HELD: The SC ruled in favor of Stonehill et al. The SC
emphasized however that Stonehill et al cannot assail the
validity of the search warrant issued against their corporation
for Stonehill are not the proper party hence has no cause of
action. It should be raised by the officers or board members
of the corporation. The constitution protects the peoples
right against unreasonable search and seizure. It provides;
(1) that no warrant shall issue but upon probable cause, to be
determined by the judge in the manner set forth in said
provision; and (2) that the warrant shall particularly describe
the things to be seized. In the case at bar, none of these are
met. The warrant was issued from mere allegation that
Stonehill et al committed a violation of Central Bank
Laws, Tariff and Customs Laws, Internal Revenue (Code)
and Revised Penal Code. In other words, no specific
offense had been alleged in said applications. The averments
thereof with respect to the offense committed were abstract.
As a consequence, it was impossible for the judges who
issued the warrants to have found the existence of probable
cause, for the same presupposes the introduction of
competent proof that the party against whom it is sought has
performed particular acts, or committed specific omissions,
violating a given provision of our criminal laws. As a matter
of fact, the applications involved in this case do not allege
any specific acts performed by herein petitioners. It would
be a legal heresy, of the highest order, to convict anybody of
a violation of Central Bank Laws, Tariff and Customs
Laws, Internal Revenue (Code) and Revised Penal Code,
as alleged in the aforementioned applications without
reference to any determinate provision of said laws or codes.

The grave violation of the Constitution made in the


application for the contested search warrants was
compounded by the description therein made of the effects to
be searched for and seized, to wit:
Books of accounts, financial records, vouchers, journals,
correspondence, receipts, ledgers, portfolios, credit journals,
typewriters, and other documents and/or papers showing all
business transactions including disbursement receipts,
balance sheets and related profit and loss statements.
Thus, the warrants authorized the search for and seizure of
records pertaining to all business transactions of Stonehill et
al, regardless of whether the transactions were legal or
illegal. The warrants sanctioned the seizure of all records of
Stonehill et al and the aforementioned corporations,
whatever their nature, thus openly contravening the explicit
command of the Bill of Rights that the things to be seized
be particularly described as well as tending to defeat its
major objective: the elimination of general warrants. The
Moncado doctrine is likewise abandoned and the right of the
accused against a defective search warrant is emphasized.
Bache & Co. Inc. et al vs BIR Commissioner Vivencio
Ruiz et al
Search and Seizure Personal Examination of the Judge

his Deputy Clerk of Court to take the depositions of De Leon


and Logronio. After the session had adjourned, J Ruiz was
informed that the depositions had already been taken. The
stenographer read to him her stenographic notes; and
thereafter, J Ruiz asked respondent Logronio to take the oath
and warned him that if his deposition was found to be false
and without legal basis, he could be charged for perjury. J
Ruiz signed de Leons application for search warrant and
Logronios deposition. The search was subsequently
conducted.
ISSUE: Whether or not there had been a valid search
warrant.
HELD: The SC ruled in favor of Bache on three grounds.
1. J Ruiz failed to personally examine the complainant and
his witness.
Personal examination by the judge of the complainant and
his witnesses is necessary to enable him to determine the
existence or non-existence of a probable cause.
2. The search warrant was issued for more than one specific
offense.

The search warrant in question was issued for at least four


distinct offenses under the Tax Code. As ruled in Stonehill
Such is the seriousness of the irregularities committed in
connection with the disputed search warrants, that this Court
On 24 Feb 1970, Commissioner Vera of Internal Revenue,
deemed it fit to amend Section 3 of Rule 122 of the former
wrote a letter addressed to J Ruiz requesting the issuance of
Rules of Court that a search warrant shall not issue but upon
a search warrant against petitioners for violation of Sec 46(a)
probable cause in connection with one specific offense. Not
of the NIRC, in relation to all other pertinent provisions
satisfied with this qualification, the Court added thereto a
thereof, particularly Sects 53, 72, 73, 208 and 209, and
paragraph, directing that no search warrant shall issue for
authorizing Revenue Examiner de Leon make and file the
more than one specific offense.
application for search warrant which was attached to the
letter. The next day, de Leon and his witnesses went to CFI
3. The search warrant does not particularly describe the
Rizal to obtain the search warrant. At that time J Ruiz was
things to be seized.
hearing a certain case; so, by means of a note, he instructed

The documents, papers and effects sought to be seized are


described in the Search Warrant
Unregistered and private books of accounts (ledgers,
journals, columnars, receipts and disbursements books,
customers ledgers); receipts for payments received;
certificates of stocks and securities; contracts, promissory
notes and deeds of sale; telex and coded messages; business
communications, accounting and business records; checks
and check stubs; records of bank deposits and withdrawals;
and records of foreign remittances, covering the years 1966
to 1970.
The description does not meet the requirement in Art III,
Sec. 1, of the Constitution, and of Sec. 3, Rule 126 of the
Revised Rules of Court, that the warrant should particularly
describe the things to be seized.
A search warrant may be said to particularly describe the
things to be seized when the description therein is as specific
as the circumstances will ordinarily allow or when the
description expresses a conclusion of fact not of law by
which the warrant officer may be guided in making the
search and seizure or when the things described are limited
to those which bear direct relation to the offense for which
the warrant is being issued.

was actually for sale and when he observed them closer he


overheard that it was being offered for sale for 100,000
pesos by Rogelio Roxas; he saw the Buddha and firearms
and some bullets inside the house. By these facts, Colonel
Calano requested for a warrant from J Marcos at about 12
midnight on Apr 4, 1971. Due to the urgency he issued the
warrant. And eventually the golden Buddha and some
firearms were seized from Roxass house. Santos assailed the
warrant averring that the search warrant was not limited to
one offense covering both illegal possession of firearms and
violation of Central Bank rules and regulations; that it did
not particularly describe the property to be seized; that he
did not carefully examine under oath the applicant and his
witnesses; that articles not mentioned were taken; and that
thereafter the return and the inventory although appearing to
have been prepared on said date were not actually submitted
to respondent Judge until April 13, 1971 and the objects
seized delivered only about a week later on April 19.
ISSUE: Whether or not the search warrant issued by Judge
Marcos is valid.

HELD: The SC ruled in favor Judge Marcos and had


basically affirmed the decision of appellate Judge
Gatamaitan. Taking into consideration to nature of the
articles so described, it is clear that no other more adequate
and detailed description could be given, particularly because
it is difficult to give a particular description of the contents
DOJ Sec Vicente Abad Santos vs CFI Benguet Judge Pio thereof, The description so made substantially complies with
Marcos
the legal provisions because the officer of the law who
Search and Seizure
executed the warrant was thereby placed in a position
enabling him to Identify the articles in question, which he
did, so that here, since certainly, no one would be
On March 31, 1971, Amansec went to Baguio and passed by mistaken in Identifying the Buddha, whose image is well
known, and even the firearms and ammunition because these
a house at 47 Ledesma Street, Baguio he was attracted by
several persons inside the house; he peeped from outside the were those without permit to possess, and all located at 47
house and when the curtain was moved he saw a Buddha that Ledesma St., Baguio City, so far as description was
was inside the house; he observed what was going on inside concerned, the search warrant perhaps could not be said to
the house and he heard someone say that the golden Buddha have suffered fatal defects.

Castro vs. Pabalan (70 SCRA 477)


Posted by taxcasesdigest on Tuesday, July 14, 2009
Labels: constitutional law, search and seizure, search warrant
Facts: Judge Pabalan ordered the issuance of a search
warrant despite failure of the application of Lumang or the
warrant itself to specify the offense, to examine the applicant
as well as his witnesses on the part of the Judge, and to
describe with particularity the place to be searched and the
things to be seized. Judge never refuted the assertions when
required to answer. Application alleged that applicants wee
informed and claimed that they verified the report that Maria
Castro and Co Ling are in possession of narcotics and other
contraband in Barrio Padasil, Bangar, La Union without
specifying the particular place in the Barrio. No complete
description of the goods and inquiry was brief. Upon actual
search, it turned out that it was in Barrio Ma. Cristina and
not in Padasil.
Issue: Whether or not the search warrant is validly issued.
Held: Search warrant issued illegal for violation of the 1935
Constitution and the Rules of Court because the two basic
requirements are not complied with: (a) no warrant shall
issue but upon probable cause, (b) the warrant shall
particularly describe the things to be seized, thus, a general
warrant. However, things seized cannot be returned and shall
be destroyed, except the liquors, playing cards, distilled
water and five bottles of Streptomycin.
G.R. No. L-25232 December 20, 1973
ASIAN SURETY and INSURANCE COMPANY, INC.,
petitioner,
vs.
HON. JOSE HERRERA, as Judge, City Court of Manila,
NBI Agent CELSO J. ZOLETA, JR. and MANUEL
CUARESMA, respondents.
Astraquillo, Laquio, Brillantes and Associates, Taada,
Carmon and Taada and Alidio, Elegir, Anchete and
Catipon petitioner.

Assistant Solicitor General Pacifico P. de Castro and


Solicitor Augusto M. Amores for respondent Celso J. Zoleta,
Jr.
Antonio Barredo for respondent Manuel Cuaresma.

ESGUERRA, J.:
Petition to quash and annul a search warrant issued by
respondent Judge Jose Herrera of the City Court of Manila,
and to command respondents to return immediately the
documents, papers, receipts and records alleged to have been
illegally seized thereunder by agents of the National Bureau
of Investigation (NBI) led by respondent Celso Zoleta, Jr.
On October 27, 1965, respondent Judge Herrera, upon the
sworn application of NBI agent Celso Zoleta, Jr. supported
by the deposition of his witness, Manuel Cuaresma, issued a
search warrant in connection with an undocketed criminal
case for estafa, falsification, insurance fraud, and tax
evasion, against the Asian Surety and Insurance Co., a
corporation duly organized and existing under the laws of
the Philippines, with principal office at Room 200 Republic
Supermarket Bldg., Rizal Avenue, Manila. The search
warrant is couched in the following language:
It appearing to the satisfaction of the
undersigned, after examining under oath
NBI Agent Celso J. Zoleta, Jr. and his
witness Manuel Cuaresma that there are
good and sufficient reasons to believe that
Mr. William Li Yao or his employees
has/have in his/their control in premises No.
2nd Floor Republic Supermarket Building,
in Rizal Avenue district of Sta. Cruz, Manila,
property (Subject of the offense; stolen or
embezzled and proceeds or fruits of the
offense used or intended to be used as the

means of committing the offense) should be


seized and brought to the undersigned.
You are hereby commanded to make an
immediate search at any time in the ----- of
the premises above-described and forthwith
seize and take possession of the following
personal property to wit: Fire Registers,
Loss Bordereau, Adjusters Report including
subrogation receipt and proof of loss, Loss
Registers, Books of Accounts, including
cash receipts and disbursements and general
ledger, check vouchers, income tax returns,
and other papers connected therewith ... for
the years 1961 to 1964 to be dealt with as
the law directs.
Armed with the search warrant Zoleta and other agents
assigned to the Anti-graft Division of the NBI entered the
premises of the Republic Supermarket Building and served
the search warrant upon Atty. Alidio of the insurance
company, in the presence of Mr. William Li Yao, president
and chairman of the board of directors of the insurance firm.
After the search they seized and carried away two (2)
carloads of documents, papers and receipts.
Petitioner assails the validity of the search warrant, claiming
that it was issued in contravention of the explicit provisions
of the Constitution and the Rules of Court, particularly
Section 1, of Art. III of the 1935 Constitution, now Section
3, of Art. IV of the new Constitution, and Sections 3, 5, 8
and 10 of Rule 126 of the Rules of Court, hereunder quoted
for convenience of reference, viz:
Sec. 3 The rights of the people to be
secure in their persons, houses, papers and
effects, against unreasonable searches and
seizures shall not be violated, and no
warrant shall issue but upon probable cause
to be determined by the judge after

examination under oath or affirmation of the


complainant and the witnessed he may
produce, and particularly describing the
place to be searched, and the persons, or
things to be seized." (Art. IV, Section 3,
New Constitution)
Sec. 3 Requisites for issuing search
warrant A search warrant shall not issue
but upon probable cause in connection with
one specific offense to be determined by the
judge or justice of the peace after
examination under oath or affirmation of the
complainant and the witnesses he may
produce, and particularly describing the
place to be searched and the persons or
things to be seized.
No search warrant shall issue for more than
one specific offense. (Sec. 3, Rule 126,
Rules of Court)
Sec. 5 Issuance and form of search
warrant If the judge or justice of the
peace is thereupon satisfied of the existence
of facts upon which the application is based,
or that there is probable cause to believe that
they exist, he must issue the warrant in the
form prescribed by these rules. (Sec. 5, Rule
126)
Sec. 8 Time of making search The
warrant must direct that it be served in the
day time, unless the affidavit asserts that the
property is on the person or in the place
ordered to be searched, in which case a
direction may be inserted that it be served at
any time of the night or day. (Sec. 8, Rule
126)

Sec. 10 Receipt for property seized. The


officer seizing property under the warrant
must give a detailed receipt for the same to
the person on whom or in whose possession
it was found, or in the absence of any
person, must, in the presence of at least one
witness, leave a receipt in the place in which
he found the seized property. (Sec. 10, Rule
126) .
"Of all the rights of a citizen, few are of greater importance
or more essential to his peace and happiness than the right of
personal security, and that involves the exemption of his
private affairs, books, and papers from the inspection and
scrutiny of others. 1 While the power to search and seize is
necessary to the public welfare, still it must be exercised and
the law enforced without transgressing the constitutional
rights of the citizens, for the enforcement of no statute is of
sufficient importance to justify indifference to the basic
principles of government (People v. Elias, 147 N.E. 472)."
I.
In the case at bar, the search warrant was issued for four
separate and distinct offenses of : (1) estafa, (2) falsification,
(3) tax evasion and (4) insurance fraud, in contravention of
the explicit command of Section 3, Rule 126, of the Rules
providing that: "no search warrant shall issue for more than
one specific offense." The aforequoted provision, which is
found in the last paragraph of the same section, is something
new. "There is no precedent on this amendment
prohibition against the issuance of a search warrant for more
than one specific offense either in the American books on
Criminal procedure or in American decisions." 2 It was
applied in the celebrated case of Harry S. Stonehill v.
Secretary of Justice 3 where this Court said:
To uphold the validity of the warrants in
question would be to wipe out completely
one of the most fundamental rights

guaranteed in our Constitution, for it would


SEC. 2. A search warrant may be issued for
place the sanctity of the domicile and the
the search and seizure of the following
privacy of communication and
personal property:
correspondence at the mercy of the whims,
(a) Property subject of the offense;
caprice or passion of peace officers. This is
precisely the evil sought to be remedied by
(b) Property stolen or embezzled and other
the constitutional provision abovequoted
proceeds or fruits of the offense; and
to outlaw the so-called general warrants. It is
not difficult to imagine what would happen
(c) Property used or intended to be used as
in times of keen political strife, when the
the means of committing an offense.
party in power feels that the minority is
likely to wrest it, even though by legal
The search warrant herein involved reads in part: "...
means.
property (Subject of the offense, stolen or embezzled and
proceeds or fruits of the offense used or intended to be used
Such is the seriousness of the irregularities
as the means of committing the offense) should be seized
committed in connection with the disputed
search warrants, that this Court deemed it fit and brought to the undersigned." The claim of respondents
that by not cancelling the description of one or two of the
to amend section 3 of Rule 122 of the
classes of property contained in the form when not
former Rules of Court by providing in its
applicable to the properties sought to be seized, the
counterpart, under the Revised Rules of
respondent judge intended the search to apply to all the three
Court, that a search warrant shall not issue
but upon probable cause in connection with classes of property. This is a patent impossibility because the
one specific offense. Not satisfied with this description of the property to be searched and seized, viz:
Fire Registers, Loss Bordereau, Adjusters Report, including
qualification, the court added thereto a
subrogation receipts and proof of loss, Loss Registers, Books
paragraph, directing that no search warrant
of Accounts including cash receipts and disbursements and
shall issue for more than one specific
general ledger, etc. and the offenses alleged to have been
offense.
committed by the corporation to wit: estafa, falsification, tax
evasion and insurance fraud, render it impossible for Us to
II.
see how the above-described property can simultaneously be
Petitioner likewise contests the validity of the search warrant contraband goods, stolen or embezzled and other proceeds or
fruits of one and the same offense. What is plain and clear is
on the ground that it authorized the search and seizures of
the fact that the respondent Judge made no attempt to
personal properties so vaguely described and not
particularized, thereby infringing the constitutional mandate determine whether the property he authorized to be searched
and seized pertains specifically to any one of the three
requiring particular description of the place to be searched
classes of personal property that may be searched and seized
and the persons or things to be seized. It also assails the
under a search warrant under Rule 126, Sec. 2 of the Rules.
noncompliance with the above-requirement as likewise
The respondent Judge simply authorized search and seizure
openly violative of Section 2 of Rule 126 which provides:
under an omnibus description of the personal properties to be
seized. Because of this all embracing description which

includes all conceivable records of petitioner corporation,


which if seized (as it was really seized in the case at bar),
could possibly paralyze its business, 4 petitioner in several
motions, filed for early resolution of this case, manifested
that the seizure of TWO carloads of their papers has
paralyzed their business to the grave prejudice of not only
the company, its workers, agents, employees but also of its
numerous insured and beneficiaries of bonds issued by it,
including the government itself, and of the general public. 5
And correlating the same to the charges for which the
warrant was issued, We have before Us the infamous general
warrants of old. In the case of Uy Kheytin, et al., v. Villareal,
42 Phil. 896, cited with approval in the Bache case, supra,
We had occasion to explain the purpose of the requirement
that the warrant should particularly describe the place to be
searched and the things to be seized, to wit:

failure to give a detailed receipt of the things seized. Going


over the receipts (Annexes "B", "B-1", B-2", "B-3" and "B4" of the Petition) issued, We found the following: one
bordereau of reinsurance, 8 fire registers, 1 marine register,
four annual statements, folders described only as Bundle gm1 red folders; bundle 17-22 big carton folders; folders of
various sizes, etc., without stating therein the nature and kind
of documents contained in the folders of which there were
about a thousand of them that were seized. In the seizure of
two carloads of documents and other papers, the possibility
that the respondents took away private papers of the
petitioner, in violation of his constitutional rights, is not
remote, for the NBI agents virtually had a field day with the
broad and unlimited search warrant issued by respondent
Judge as their passport.

eventually adopted the exclusionary rule, realizing that this


is the only practical means of enforcing the constitutional
injunction against unreasonable searches and seizures. Thus
the Supreme Court of the United States declared: 9
If letters and private documents can thus be
seized and held and used in evidence against
a citizen accused of an offense the protection
of the 4th Amendment, declaring his right to
be secured against such searches and
seizures is of no value, and so far as those
thus placed are concerned, might as well be
stricken from the Constitution. The efforts of
the courts and their officials to bring the
guilty to punishment, praise-worthy as they
are, are not to be aided by the sacrifice of
those great principles established by years of
endeavor and suffering which have resulted
in their embodiment in the fundamental law
of the land.

IV.
"... Both the Jones Law (sec. 3) and General
Orders No. 58 (sec. 97) specifically require
that a search warrant should particularly
describe the place to be searched and the
things to be seized. The evident purpose and
intent of this requirement is to limit the
things to be seized to those, and only those,
particularly described in the search warrant
to leave the officers of the law with no
discretion regarding what articles they shall
seize, to the end that "unreasonable searches
and seizures" may not be made. That this is
the correct interpretation of this
constitutional provision is borne out by
American authorities."
The purpose as thus explained could, surely and effectively,
be defeated under the search warrant issued in this case.

The search warrant violated the specific injunctions of


Section 8 of Rule 126. 6 Annex "A" of the Petition which is
the search warrant in question left blank the "time" for
making search, while actual search was conducted in the
evening of October 27, 1965, at 7:30 p.m., until the wee
hours of the morning of October 28, 1965, thus causing
untold inconveniences to petitioners herein. Authorities 7 are
of the view that where a search is to be made during the
night time, the authority for executing the same at that time
should appear in the directive on the face of the warrant.
In their Memorandum 8 respondents, relying on the case of
Moncado v. Peoples Court (80 Phil. 1), argued:
Even assuming that the search warrant in
question is null and void, the illegality
thereof would not render the incriminating
documents inadmissible in evidence.

Moreover, the criminal charges filed by the NBI have all


been dismissed and/or dropped by the Court or by the office
of the City Fiscal of Manila in 1968, as manifested in the
petition filed by petitioner dated October 24, 1972, for early
resolution of this case.
V.
It has likewise been observed that the offenses alleged took
place from 1961 to 1964, and the application for search
warrant was made on October 27, 1965. The time of the
application is so far remote in time as to make the probable
cause of doubtful veracity and the warrant vitally defective.
Thus Mr. Joseph Varon, an eminent authority on Searches,
Seizures and Immunities, has this to say on this point:

III.
Moreover, as contended by petitioner, respondents in like
manner transgressed Section 10 of Rule 126 of the Rules for

This Court has reverted to the old rule and abandoned the
Moncado ruling (Stonehill case, supra). Most common law
jurisdictions have already given up this approach and

From the examination of the several cases


touching upon this subject, the following

general rules are said to apply to affidavits


for search warrants:

A
sian
S
urety vs. Herrera
(1) xxx xxx xxx
Facts:On October
1
(2) Such statement as to the time of the
965, upon a sworn application of NBIagent Celso Zoleta Jr. supported
alleged offense must be clear and definite
with the deposition of witness Manuel Cuaresma, the respondent Judge
and must not be too remote from the time of JoseHerrera, issued a search warrant against the petitioner for criminal
the making of the affidavit and issuance of
case of Estafa, falsification, insurance fraud and taxevasion.By virtue of
the search warrant.
the search warrant, NBI agents seizedthe place in the office of the
petitioner in Republi Market andcarried away two car loads of
documents, papers andreceipt.The petitioners, then filed a suit assailing
(3) There is no rigid rule for determining
whether the stated time of observation of the the validityof the SW, contending that it doesnot follow theConstitutional
and statutory requirements of a valid SW.Issue:What are the violated
offense is too remote from the time when the
procedures in the case atbar?Held:
affidavit is made or the search warrant
S
issued, but, generally speaking, a lapse of
ingle warrant single offense rule. General warrant.
time of more than three weeks will be held
The constitution requires that a SW should beissued upon a probable
not to invalidate the search warrant while a cause in connection with one singleoffense.In the case at bar, the SW was
issued for 4separate and distinct offenses. Estafa, falsification, taxevasion
lapse of four weeks will be held to be so.
and insurance fraud. Therefore it is invalid for it is ageneral warrant.
Particular description of the objects to be seized.
A good and practical rule of thumb to
The constitution mandates that objects to be seizedshould be couched not
measure the nearness of time given in the
on generic but specific terms.
affidavit as to the date of the alleged offense, S
and the time of making the affidavit is thus ection 2 provides that a
expressed: The nearer the time at which the S
observation of the offense is alleged to have W may be issued for the search and seizure of the following
personal properties.a)
been made, the more reasonable the
P
conclusion of establishment of probable
roperty subject to the offenseb)
cause. [Emphasis Ours]
P
PREMISES CONSIDERED, petition is hereby granted; the roperty stolen or embezzled and other proceeds or fruits of the offense
c)
search warrant of October 27, 1965, is nullified and set
P
aside, and the respondents are hereby ordered to return
roperty used or intended to be used as themeans of committing the
immediately all documents, papers and other objects seized offense
or taken thereunder. Without costs.
In the case at bar, the respondent judge usedall three of the description in
relation to the things to beseized in the petitioner. Thus, they are all
couched in
Makalintal, C.J., Castro, Fernandez * and Muoz Palma,
JJ., concur.
Constitutional Law II (Bill of Rights): Case Briefs: Dennis G.
Libunao UC College of Law
Makasiar, J., concurs in the result.

27
generic terms. The respondent judge did not bother tospecify the things
to be seized that would be admitted asan evidence to the offense charged.
The Rule on RR
C
that the
S
W
should be issued on dayti
m
e.
In the case at bar, the SW was conducted eveningof Oct 27,
1
965 at 7:3
0
pm until morning.
Re
m
oteness of the ti
m
e of the offense and theapplication of the
S
W
.
Joseph Varon provides rules to apply affidavits for SW.
1
) Such statement as to the time of the alleged offensemust be clear and
definite and must not be tooremote from the time of the making of the
affidavitand issuance of the search warrant2) There is no rigid rule for
determining whether thestated time of observation of the offense id
tooremote from the time when the affidavit was madeor the search
warrant issued but generally speaking,a lapse of time of less than three
weeks will be heldnot to invalidate the search warrant, while the lapseof
four weeks will be held to be so.Thus, the nearer the time at which the
observation of anoffense is alleged to have been made, the more
reasonablethe conclusion of establishment of a probable cause.In the case
at bar, the alleged commission of thecrime is from
1
96
1
to
1
964 and the application for SW ismade
1

965. thus, there can be doubt as to the establishmentof a probable cause


because of the remoteness of time.
Viduya vs. Berdiago

Whether or not respondent Judge committed grave


abuse of discretion in quashing the warrant

HELD:
73 SCRA 553 (1976)

The Court opined that except in the case of the search of a


Except in the case of the search of a dwelling house, dwelling house, persons exercising police authority under
the customs law may effect search and seizure without a
persons exercising police authority under the
customs law may effect search and seizure without a search warrant in the enforcement of customs laws. There is
search warrant in the enforcement of customs laws. justification then for the insistence on the part of private
respondent that probable cause be shown. So respondent
Judge found in issuing the search warrant.
FACTS:
Respondent Berdiago is the owner of a Rolls Royce car,
Model 1966, which arrived in the Port of Manila on January
8, 1968. However, the petitioner, Jose Viduya, then Collector
of Customs of Manila, obtained reliable intelligence that
fraudulent documents were used by Berdiago in securing the
release of the car from the Bureau of Customs, making it
appear therein that the car was a 1961 model instead of a
1966 one, thus enabling respondent to pay a much lower
customs duty.
There was, accordingly, a formal demand for the payment of
the sum to cover the deficiency, respondent manifesting his
willingness to do so but failing to live up to his promise. As
the car was kept in a dwelling house at the Yabut Compound,
two officials of the Customs Police Service as duly
authorized agents of petitioner, applied to respondent Judge
for a warrant to search said dwelling house and to seize the
Rolls Royce car found therein.
Berdiago filed a motion to quash the search warrant issued
by the court based on lack of probable cause to issue the
warrant. Collector Viduya opposed, alleging that Berdiago
could not rely on the constitutional right against
unreasonable search and seizure because it was not shown
that he owned the dwelling house which was searched.
Nonetheless, respondent Judge in the challenged order
quashed such search warrant.
Hence, this petition.
ISSUE:

Claro M. Recto for appellant.


Attorney-General Villa-Real for appellee.
Malcolm, J.:

This is an appeal from a judgment of the Court of First


Instance of Manila finding the accused, Jose Ma. Veloso,
guilty of the crime of resistance of the agents of the
authority, in violation of article 252 of the Penal Code, and
sentencing him to four months and one day imprisonment,
arresto mayor, with the accessory penalties, to pay a fine of
P200, with the corresponding subsidiary imprisonment in
Apparently, he was persuaded to quash it when he noted that case of insolvency, and to pay the costs. The errors assigned
by counsel for the accused as appellant, go to the proposition
the warrant for seizure and detention came later than its
that the resistance of the police was justifiable on account of
issuance. In thus acting, respondent Judge apparently
the illegality of the John Doe search warrant.
overlooked that long before the search warrant was applied
for, to be specific on April 15, 1968, the misdeclaration and
underpayment was already noted and that thereafter on April In May, 1923, the building located at No. 124 Calle
Arzobispo, City of Manila, was used by an organization
24, 1968, private respondent himself agreed to make good
known as the Parliamentary Club. Jose Ma. Veloso was at
the further amount due but not in the sum demanded.
that time a member of the House of Representative of the
Philippine Legislature. He was also the manager of the club.
As the car was kept in a dwelling house, petitioner through
two of his officers in the Customs Police Service applied for
and was able to obtain the search warrant. Had there been no The police of Manila had reliable information that the socalled Parliamentary Club was nothing more than a gambling
such move on the part of petitioner, the duties expressly
house. Indeed, on May 19, 1923, J. F. Townsend, the chief of
enjoined on him by law assess and collect all lawful
the gambling squad, had been to the club and verified this
revenues, to prevent and suppress smuggling and other
frauds and to enforce tariff and customs law would not have fact. As a result, on May 25, 1923, Detective Andres
Geronimo of the secret service of the City of Manila, applied
been performed.
for, and obtained a search warrant from Judge Garduo of
the municipal court. Thus provided, the police attempted to
While therefore, it is to be admitted that his warrant of
raid the Parliamentary Club a little after three in the
seizure and detention came later than the search warrant,
afternoon of the date above- mentioned. They found the
there were indubitable facts in existence at that time to call
doors to the premises closed and barred. Accordingly, one
for its issuance. Certainly there was probable cause. There
band of police including policeman Rosacker, ascended a
was evidently need for the issuance of a search warrant. It
telephone pole, so as to enter a window of the house. Other
ought not to have been thereafter quashed.
policemen, headed by Townsend, broke in the outer door.
October 20, 1925
Once inside the Parliamentary Club, nearly fifty persons
were apprehended by the police. One of them was the
G.R. No. L-23051
defendant Veloso. Veloso asked Townsend what he wanted,
THE PEOPLE OF THE PHILIPPINES ISLANDS,
and the latter showed him the search warrant. Veloso read it
plaintiff-appellant,
and told Townsend that he was Representative Veloso and
vs.
not John Doe, and that the police had no right to search the
JOSE MA. VELOSO, defendant-appellant.

house. Townsend answered that Veloso was considered as


John Doe. As Veloso's pocket was bulging, as if it contained
gambling utensils, Townsend required Veloso to show him
the evidence of the game. About five minutes was consumed
in conversation between the policemen and the accused the
policemen insisting on searching Veloso, and Veloso
insisting in his refusal to submit to the search.

There are found in the record the application for search


warrant, the affidavit for search warrant, and the search
warrant. The application reads:

used in gambling games, such as cards, dice, chips, lottery


tickets, lists of drawing and lists used in prohibited games
are kept.

UNITED STATES OF AMERICA

I, Andres Geronimo, being duly sworn, depose and say that I


have read the foregoing questions and answers and that I find
the same to correct and true to the best of my knowledge and
belief.

At last the patience of the officers was exhausted. So


policeman Rosacker took hold of Veloso only to meet with
his resistance. Veloso bit Rosacker in the right forearm, and
gave him a blow in another part of the body, which injured
the policeman quite severely. Through the combined efforts
of Townsend and Rosacker, Veloso was finally laid down on
the floor, and long sheets of paper, of reglas de monte, cards,
cardboards, and chips were taken from his pockets.

IN THE MUNICIPAL COURT OF THE CITY OF MANILA

All of the persons arrested were searched and then conducted


to the patrol wagons. Veloso again refused to obey and
shouted offensive epithets against the police department. It
was necessary for the policemen to conduct him downstairs.
At the door, Veloso resisted so tenaciously that three
policemen were needed to place him in the patrol wagon.
In the municipal court of the City of Manila, the persons
arrest in the raid were accused of gambling. All of them were
eventually acquitted in the Court of First Instance for lack of
proof, with the sole exception of Veloso, who was found
guilty of maintaining a gambling house. This case reached
the appellate court where the accused was finally sentenced
to pay a fine of P500. (No. 22163. 1 )
The foregoing are the principal facts taken mainly from the
findings of the trial judge, the Honorable Vicente
Nepomuceno. Counsel for the appellant makes no effort to
impugn these findings, except that he stresses certain points
as more favorable to the case of his client. The defense, as
previously indicated, is planted squarely on the contention
that since the name of Veloso did not appear in the search
warrant, but instead the pseudonym John Doe was used,
Veloso had a legal right to resist the police by force. The
nature of this defense makes it advisable to set forth further
facts, relating particularly to the search warrant, before
passing to the law.

PHILIPPINE ISLANDS

(Sgd.) ANDRES GERONIMO


THE PEOPLE OF THE PHILIPPINE ISLANDS, plaintiff,
vs. JOHN DOE, Defendant.
APPLICATION FOR (G)
SEARCH WARRANT
Testimony taken before Hon. L. Garduo, Judge, Municipal
Court, Manila.

Subscribed and sworn to before me this 25th day of May,


1923.
(Sgd.) L. GARDUO Judge, Municipal Court

Andres Geronimo, being duly sworn, testifies as follows:

The affidavit and the search warrant are so nearly alike that
it will suffice to copy the search warrant alone. This
document reads:

Q. What is your name, residence and occupation? A.


Andres Geronimo, No. 47 Revellin, detective.

UNITED STATES OF AMERICA

Q. Are you the applicant of this search warrant? A. Yes,


sir.
Q. Do you know the premises situated at No. 124 Calle
Arzobispo, District of W. C., City of Manila? A. Yes. sir.

PHILIPPINE ISLANDS
IN THE MUNICIPAL COURT OF THE CITY OF MANILA
THE PEOPLE OF THE PHILIPPINE ISLANDS, Plaintiff,
vs.

Q. Do you know who occupies said premises? A. I do not


know. According to the best of my information the house is JOHN DOE, Defendant.
occupied by John Doe.
SEARCH WARRANT (G)
Q . What are your reasons for applying for this search
warrant? A. It has been reported to me by a person whom The People of the Philippine Islands, to any member of the
I consider to be reliable that in said premises there are
Police Force of the City of Manila.
instruments and devices used in gambling games, such as
cards, dice, chips, lottery tickets, lists of drawing and lists
used in prohibited games kept. It has been reported to me by GREETING:
a person whom I consider to be reliable that there are or
Proof by affidavit having this day been made before me by
there will be gambling conducted in said premises. The
Andres Geronimo that he has good reason to believe and
aforesaid premises are known as gambling house. I have
does believe that John Doe has illegally in his possession in
watched the foregoing premises and believed it to be a
the building occupied by him and which is under his control,
gambling house and a place where instruments and devices

namely in the building numbered 124 Calle Arzobispo, City


of Manila, Philippines Islands, certain devices and effects
used in violation of the Gambling Law, to wit: money, cards,
chips, reglas, pintas, tables and chairs and other utensils used
in connection with the game commonly known as monte and
that the said John Doe keeps and conceals said devices and
effects with the illegal and criminal intention of using them
in violation of the Gambling Law.
Now therefore, you are hereby commanded that at any time
in the day or night within ten (10) days on or after this date
to make a search on the person of said John Doe and in the
house situated at No. 124 Calle Arzobispo, City of Manila,
Philippine Islands, in quest of the above described devices
and effects and if you find the same or any part thereof, you
are commanded to bring it forthwith before me as provided
for by law.
Given under my hand, this 25th day of May, 1923.
(Sgd.) L. GARDUO Judge, Municipal Court
Coming now to the legal aspects of the case it is first worthy
of mention that by reason of the Fourth Amendment to the
United States Constitution and the eleventh and eighteenth
paragraphs of the Philippine Bill of Rights, as found in the
present Organic Act, the security of the dwelling and the
person is guaranteed. The organic act provides "that the right
to be secured against unreasonable searches and seizures
shall not be violated." It further provides "that no warrant
shall issue but upon probable cause, supported by oath or
affirmation and particularly describing the place to be
searched and the person or things to be seized."
In the Philippine Code of Criminal Procedure are found
provisions of the same import although naturally entering
more into detail. It is therein provided, among other things,
that "a search warrant shall not issue except for probable
cause and upon application supported by oath particularly
describing the place to be searched and the person of thing to
be seized." (Section 97.) After the judge or justice shall have
examined on oath the complainant and any witnesses he may
produce, and shall have taken their depositions in writing
(section 98), and after the judge or justice is satisfied of the
existence of facts upon which the application is based, or

that there is probable cause to believe that they exist, he


must issue the warrant which must be substantially in the
following form:

xxxxxxxxx
Name and description of the accused should be inserted in
the body of the warrant and where the name is unknown
there must be such a description of the person accused as
will enable the officer to identify him when found
8UYIXlM.

. . . You are, therefore, commanded, . . . to make immediate


search on the person of ............................, or in the house
situated ...................................... (describing it or any other
place to be searched with reasonable particularity, as the case
may be) for the following property: . . . ." (Section 99.) It is x x x x x x x x x
finally provided that "a person charged with a crime may be
searched for dangerous weapons or anything which may be
Warrant for apprehension of unnamed party, or containing a
used as proof of the commission of the crime. (Section 105). wrong name for the party to be apprehended is void, except
in those cases where it contains a descriptio personae such as
A search warrant must conform strictly to the requirements
will enable the officer to identify the accused.
of the constitutional and statutory provisions under which it
is issued. Otherwise it has rightly been held, must be
xxxxxxxxx
absolutely legal, "for there is not a description of process
known to the law, the execution of which is more distressing John Doe' Warrants. It follows, on principle, from what has
to the citizen. Perhaps there is none which excites such
already been said regarding the essential requirements of
intense feeling in consequence of its humiliating and
warrants for the apprehension of persons accused, and about
degrading effect." The warrant will always be construed
blank warrants, that a warrant for the apprehension of a
strictly without, however, going the full length of requiring
person whose true name is unknown, by the name of "John
technical accuracy. No presumptions of regularity are to be
Doe" or "Richard Roe," "whose other or true name in
invoked in aid of the process when an officer undertakes to
unknown," is void, without other and further descriptions of
justify under it. (24 R. C. L., pp. 711, et seq.; Reed vs. Rice the person to be apprehended, and such warrant will not
[1829], 2 J. J. Marshall [Ky.] 44; 19 Am. Dec., 122; Smith
justify the officer in acting under it. Such a warrant must, in
vs. McDuffee [1914], 72 Ore., 276; Ann. Cas. 1916 D, 947.) addition, contain the best descriptio personae possible to be
obtained of the person or persons to be apprehended, and this
The search warrant has been likened to a warrant of arrest.
description must be sufficient to indicate clearly the proper
Although apprehending that there are material differences
person or persons upon whom the warrant is to be served;
between the two, in view of the paucity of authority
and should state his personal appearance and peculiarities,
pertaining to John Doe search warrants we propose to take
give his occupation and place of residence, and any other
into consideration the authorities relied upon by the
circumstances by means of which he can be identified.
appellant, thus following the precedent of Uy Kheytin vs.
Villareal ([1920], 42 Phil., 886), where the regularity of the
Person apprehended in act of committing a crime, under a
issuance of the search warrant was also questioned.
"John Doe" warrant, on the other hand, the apprehension will
not be illegal, or the officer liable, because under such
In the lower court, and again in this court, the attorneys for
circumstances it is not necessary that a warrant should have
the defense quoted from Wharton's Criminal Procedure. In
been issued.
that text at pages 51, 52, 54, 55, and 56 of volume 1 of the
Tenth Edition, is found the following:
The authority most often cited to sustain the text, and quoted
with approval by the United States Supreme Court, is the
Form and Sufficiency of Warrant. Technical accuracy is not case of Commonwealth vs. Crotty ([1865], 10 Allen [Mass.],
required. . . uklVBdbi.
403). It there appeared that one Peaslee had made a
complaint to the police court Lee, charging that "John Doe or

Richard Roe, whose other or true name is to your


complainant unknown," had committed an assault and
battery upon him; upon which complaint a warrant was
issued against "John Doe or Richard Roe, whose other or
true name is to your complainant unknown, named in the
foregoing complaint." Neither the complaint nor the warrant
contained any further description or means of identification
of the person to be arrested. Crotty resisted the arrest upon
the ground that the warrant was invalid. Mr. Chief Justice
Bigelow, as the organ of the Supreme Court of
Massachusetts, said:

place of his residence, or other circumstances by which he


can be identified. (1 Chit. Crim. Law, 39, 40.)
The warrant being defective and void on its face, the officer
had no right to arrest the person on whom he attempted to
serve it. He acted without warrant and was a trespasser. The
defendant whom he sought to arrest had a right to resist by
force, using no more than was necessary to resist the
unlawful acts of the officer . . .

The defendants, therefore, in resisting the officer in making


an arrest under the warrant in question, if they were guilty of
We cannot entertain a doubt that the warrant on which the
no improper or excessive force or violence, did not do an
officer attempted to arrest one of the defendant at the time of unlawful act by lawful means, or a lawful act by unlawful
the alleged riot was insufficient, illegal and void. It did not
means, and so could not be convicted of the misdemeanor of
contain the name of the defendant, nor any description or
a riot, with which they are charged in the indictment.
designation by which he could be known and identified as
the person against whom it was issued. It was in effect a
Appellant's argument, as based on these authorities, runs
general warrant, upon which any other individual might as
something like this. The law, constitutional and statutory,
well have been arrested, as being included in the description, requires that the search warrant shall not issue unless the
as the defendant himself. Such a warrant was contrary to
application "particularly" describe the person to be seized. A
elementary principles, and in direct violation of the
failure thus to name the person is fatal to the validity of the
constitutional right of the citizen, as set forth in the
search warrant. To justify search and arrest, the process must
Declaration of Rights, article 14, which declares that every
be legal. Illegal official action may be forcibly resisted
subject has a right to be secure from all unreasonable
hi3Pq6RbJJ.
searches and seizures of his person, and that all warrants,
therefore, are contrary to this right, if the order in the warrant For the prosecution, however, as the arguments are advanced
to a civil officer to arrest one or more suspected persons or to by the Attorney-General, and as the law was summarized by
seize their property be not accompanied with a special
the trial judge, there is much to be said. Careful and logical
designation of the persons or objects of search, arrest or
reflection brings forth certain points of paramount force and
seizure. This is in fact only a declaration of an ancient
exercising a decisive influence. We will now make mention
common law right. It was always necessary to express the
of them by correlating the facts and the law.
name or give some description of a party to be arrested on a
warrant; and if one was granted with the name in blank, and In the first place, the affidavit for the search warrant and the
without other designation of the person to be arrested, it was search warrant itself described the building to be searched as
void. (1 Hale P. C. 577. 2 Ib. 119. Foster, 312. 7 Dane Ab.
"the building No. 124 Calle Arzobispo, City of Manila,
248. 1 Chit. Crim. Law, 39. Mead vs. Haws, 7 Cow., 332,
Philippine Islands." This, without doubt, was a sufficient
and cases cited.)
designation of the premises to be searched. It is the
prevailing rule that a description of a place to be searched is
This rule or principle does not prevent the issue and service sufficient if the officer with the warrant can, with reasonable
of a warrant against a party whose name is unknown. In such effort, ascertain and identify the place intended. (Steele vs.
case the best description possible of the person to be arrested U. S. [1925], U. S. Supreme Court Advance Opinions 1924is to be given in the warrant; but it must be sufficient to
1925; 69 Law. ed., 757). The police officers were
indicate clearly on whom it is to be served, by stating his
accordingly authorized to break down the door and enter the
occupation, his personal appearance and peculiarities, the
premises of the building occupied by the so-called

Parliamentary Club. When inside, they then had the right to


arrest the persons presumably engaged in a prohibited game,
and to confiscate the evidence of the commission of the
crime. It has been held that an officer making an arrest may
take from the person arrested any money or property found
upon his person, which was used in the commission of the
crime or was the fruit of the crime, or which may furnish the
person arrested with the means of committing violence or of
escaping, or which may be used as evidence on the trial of
the cause, but not otherwise. (Moreno vs. Ago Chi [1909],
12 Phil., 439.)
Proceeding along a different line of approach, it is
undeniable that the application for the search warrant, the
affidavit, and the search warrant failed to name Jose Ma.
Veloso as the person to be seized. But the affidavit and the
search warrant did state that "John Doe has illegally in his
possession in the building occupied by him, and which is
under his control, namely, in the building numbered 124
Calle Arzobispo, City of Manila, Philippine Islands, certain
devices and effects used in violation of the Gambling Law."
Now, in this connection, it must not be forgotten that the
Organic Act requires a particular description of the place to
be searched, and the person or things to be seized, and that
the warrant in this case sufficiently described the place and
the gambling apparatus, and, in addition, contained a
description of the person to be seized. Under the authorities
cited by the appellant, it is invariably recognized that the
warrant for the apprehension of an unnamed party is void,
"except in those cases where it contains a description
personae such as will enable the officer to identify the
accused." The description must be sufficient to indicate
clearly the proper person upon whom the warrant is to be
served. As the search warrant stated that John Doe had
gambling apparatus in his possession in the building
occupied by him at No. 124 Calle Arzobispo, City of Manila,
and as this John Doe was Jose Ma. Veloso, the manager of
the club, the police could identify John Doe as Jose Ma.
Veloso without difficulty.
Again, it must be remembered that No. 124 Calle Arzobispo
was supposed to be used for club purposes. It was not the
home of Veloso; not the place of abode of the family, which
the law carefully protects in all of its sanctity. It was a club
partially public in nature. It was, moreover, a camouflaged

club with a high sounding name calculated to mislead the


police, but intended for nefarious practices. In a club of such
a character, unlike in the home, there would commonly be
varying occupancy, a number of John Does and Richard
Roes whose names would be unknown to the police.
It is also borne out by the authorities that, in defense of
himself, any member of his family or his dwelling, a man
has a right to employ all necessary violence. But even in the
home, and much less so in a club or public place, the person
sought to be arrested or to be searched should use no more
force than is necessary to repel the unlawful act of the
officers. To authorize resistance to the agents of the
authority, the illegality of the invasion must be clearly
manifest. Here, there was possibly a proper case for protest.
There was no case for excessive violence to enforce the
defendant's idea of a debatable legal question.
(Commonwealth vs. Crotty, supra; People vs. Chan Fook
[1921], 42 Phil., 230; 3 Groizard, Codigo Penal, pp. 456,
457.)
The trial judge deduced from the searched warrant that the
accused Veloso was sufficiently identified therein. Mention
was made by his Honor of the code provision relating to a
complaint or information, permitting a fictitious name to be
inserted in the complaint or information, in lieu of the true
name. The Attorney-General adds to this the argument that
the police were authorized to arrest without a warrant since a
crime was being committed. We find it unnecessary to
comment on this contention.
John Doe search warrants should be the exception and not
the rule. The police should particularly describe the place to
be searched and the person or things to be seized, wherever
and whenever it is feasible. The police should not be
hindered in the performance of their duties, which are
difficult enough of performance under the best of conditions,
by superficial adherence to technicality or far fetched
judicial interference 9G8mA7jwJm.
We agree with the trial judge and with the Attorney-General
in their conclusions to the effect that the search warrant was
valid, and that the defendant has been proved guilty beyond
a reasonable doubt, of the crime of resistance of the agents
of the authority.

The information alleges that at the time of the commission of


the crime, the accused was a member of the House of
Representatives. The trial court was led to consider this
allegation in relation with the facts as an aggravating
circumstance, and to sentence the accused accordingly. We
doubt, however, that advantage was taken by the offender of
his public position when he resisted the officers of the law.
The offender did not necessarily make use of the prestige of
his office as a means to commit a crime. Undoubtedly, Jose
Ma. Veloso, as Juan de la Cruz, would have resisted the
police just as stoutly, as the Honorable Jose Ma. Veloso did.
The penalty, accordingly, falls within the medium of that
provided by the Penal Code.
Finding present no reversible error, agreeing in all respects
with the findings of facts as made by the trial judge, and
concurring with the trial judge in his legal conclusion, with
one exception, it results that the judgment appealed from
must be, as it is hereby, affirmed, with the sole modification
that the defendant and appellant shall be sentenced to two
months and one day imprisonment, arresto mayor, with the
costs of this instance against him. Let the corresponding
order to carry this judgment into effect issue.
Avancea, C.J., Street, Villamor, Ostrand, Johns, and
Romualdez, JJ., concur.
Villa-Real, JJ., took no part.
G.R. No. 141176
May 27, 2004
ELI LUI and LEO ROJAS, petitioners,
vs.
SPOUSES EULOGIO and PAULINA MATILLANO,
respondents.
DECISION
CALLEJO, SR., J.:
This is a petition for review on certiorari of the Decision 1 of
the Court of Appeals in CA-G.R. CV No. 44768 which

reversed and set aside the decision of the Regional Trial


Court of Bansalan, Davao del Sur, Branch 21.2
The Antecedents
Sometime in September 1987, then seventeen-year-old
Elenito Lariosa visited his aunt, his fathers older sister,
Paulina Lariosa Matillano, at Lily Street, Poblacion
Bansalan, Davao del Sur. On May 2, 1988, Lariosawas
employed as a laborer at the Davao United Products
Enterprise store, with a monthly salary of P800.00. The store
was owned by Leong Shiu Ben and King Kiao and was
located at the corner of Monteverde and Gempesaw Streets,
Davao City. Lariosa was tasked to close the store during
lunchtime and after store hours in the afternoon. Ben himself
opened the store in the mornings and after lunchtime.
Adjacent to the said store was another store owned by Kiaos
son, Eli Lui, who also happened to be Bens nephew. Aside
from Lariosa, Ben and Kiao employed Maximo Pagsa and
Rene Malang.
Lariosa chose to live in the house of Kiao. Lariosa fed the
dogs of his employer every morning before going to work
and in the afternoon, in exchange for free meals and lodging.
There were occasions when Lariosa stayed in the house of
Pagsa and Malang and left some of his things with them.
Lariosa deposited his savings with the Mindanao Savings
Bank in Bansalan.
On October 17, 1988, Lariosa was taken ill and was
permitted to take the day off. He went to the house of his
aunt, Paulina Matillano, and her husband Eulogio Matillano
in Bansalan City, where he rested until the next day, October
18, 1988. Lariosa reported for work the day after, or on
October 19, 1988, but Kiao told him that his employment
was terminated. Lariosa was not paid his salary for the
month of October. Kiao warned Lariosa not to report the
matter to the Department of Labor. Lariosa decided to return
to Bansalan without retrieving his things from Kiaos house.

On October 27, 1988, Lariosa returned to Davao City and


was able to collect his backwages from Ben in the amount of
P500.00. Lariosa withdrew his savings from the Mindanao
Savings Bank in Bansalan City and on November 1, 1988,
applied for a job at his cousins place, at Quimpo Boulevard,
Davao City. He bought a radio cassette for P2,500.00 and a
pair of Rayban sunglasses for P900.00.
On November 3, 1988, Lariosa went to the house of his
fiancee, Nancy, at New Matina, Davao City, but returned to
Bansalan on the same day. On November 4, 1988, he
returned to Nancys house and stayed there until the next
day, November 5, 1988.
That day, Ben informed his nephew, Eli Lui, that he had lost
P45,000.00 in cash at the store. Ben reported the matter to
NBI Senior Agent Ruperto Galvez, and forthwith executed
an affidavit wherein he alleged that after Lariosas
employment was terminated on October 19, 1988, he
discovered that he had lost P45,000.00 in cash. He suspected
that Lariosa was the culprit because the latter, as a former
employee, had a duplicate key to the side door of the United
Products Enterprise Store.
At 9:00 a.m. on November 6, 1988, a Sunday, Lariosa went
to the house of Pagsa and Malang to retrieve his things. The
two invited Lariosa to go with them to the beach, and when
Lariosa agreed, they borrowed Luis Ford Fierra for their
transportation. The vehicle stopped at the Almendras Hall
where Pagsa alighted on the pretext that he was going to buy
fish. Lariosa, Rene, and his wife remained in the Fierra.
Pagsa contacted Lui and informed the latter that Lariosa was
with him.
After about an hour, Lui arrived on board a vehicle. With
him were Pagsa and two others, Alan Mendoza and Henry
Tan. Lui told Lariosa that he wanted to talk, and asked the
latter to go with him. Pagsa urged Lariosa to go along with
Lui. Lariosa agreed and boarded Luis vehicle. The car
stopped in front of Luis house, where the latter alighted and

went inside, while his companions and Lariosa remained in


the car. When Lui returned, he was armed with a 9 mm.
caliber gun and poked Lariosa with the weapon. He warned
Lariosa not to run, otherwise, he would be killed. The group
went to Bens house to get the keys to the store. Ben joined
them as they drove towards the store.

The policemen brought Lariosa there, where they asked


Nancy if Lariosa had left anything while he slept thereat.
Nancy replied that Lariosa had left a radio cassette and a pair
of sunglasses. The policemen took these and brought Lariosa
back to the Metrodiscom headquarters where Lui and his two
companions were waiting.

Lui mauled Lariosa and tried to force the latter to admit that
he had stolen Bens money. Lariosa refused to do so. Lui
then brought Lariosa to the comfort room of the store and
pushed his face into the toilet bowl, in an attempt to force
him into confessing to the crime. Lariosa still refused to
admit to anything. Lui then made a telephone call to the
Metrodiscom (PNP) based in Davao City.

Lui asked Lariosa where he stayed when he went to


Bansalan, and Lariosa replied that he used to stay in the
house of his aunt and uncle, the Spouses Matillano, in Lily
Street, Poblacion Bansalan. Rojas and Lui then brought
Lariosa, with his hands still handcuffed, to a car. Luis
companions, Alan Mendoza and Henry Tan boarded another
car and proceeded to the Matillano residence.

Sgt. Alberto Genise of the Metrodiscom (PNP) issued


Mission Order No. MRF-A-004-88 dated November 6, 1988,
directing Pat. Leo Rojas "to follow up a theft case committed
in Davao City from 12:30 p.m. to 5:00 p.m." Rojas was
directed to coordinate with the nearest PNP headquarters
and/or stations. He was authorized to carry his firearm for
the mission. He then left the police station on board a police
car and proceeded to the corner of Magsaysay and
Gempesaw Streets.

Without prior coordination with the Bansalan PNP, Rojas,


who was in civilian clothes, Lui, Tan and Mendoza arrived at
the house of the Spouses Matillano at about 3:00 p.m, with
the handcuffed Lariosa in tow. With handguns drawn, they
kicked the door to the kitchen and gained entry into the
house. They then proceeded to the sala where they found
Lariosas aunt, Paulina Matillano. In the adjacent room were
Julieta, Lariosas sister, Paulinas daughter-in-law, Virginia,
the latters sister, Erlinda, and a seven-month-old baby.
Paulina was shocked. Rojas told Paulina, "Mrs., we are
authorities. We are here to get something." Paulina
remonstrated, "Why are you meddling (manghilabot)?"

In the meantime, a police car arrived at the store with two


policemen on board. One of them handcuffed Lariosa at
gunpoint and ordered him to open the store with the use of
the keys. As Lariosa opened the lock as ordered, one of Luis
companions took his picture. Another picture was taken as
Lariosa held the door knob to open the door. Lariosa was
then boarded in the police car and brought to the corner of
Magsaysay and Gemphesaw Streets where he was
transferred to the police car driven by Rojas. He was brought
to the Metrodiscom headquarters. Lui once more mauled
Lariosa, still trying to force the latter to confess that he stole
P45,000.00 from his uncle and to reveal what he did with the
money. When a policeman asked him where he slept the
night before, Lariosa replied that he spent the night in the
house of his girlfriends parents at New Matina, Davao City.

Lui poked his gun at Paulina and warned her not to talk
anymore because something might happen. He then said,
"All right, where is your aparador because we are getting
something." Paulina told Lui to wait for her husband
Eulogio. Lui ignored her protest and told her that they were
in a hurry. Paulina was then impelled to bring Lui and his
two companions, Mendoza and Tan, to the second floor
where her aparador was located. Rojas and the handcuffed
Lariosa remained in the sala. Lui and his two companions
then took two mats and two pairs of ladies shoes belonging
to Paulina and Eulogio, two pairs of pants, leather shoes, two
t-shirts and two polo shirts which belonged to the latters

children. They also ordered Paulina to open a chest and


when she did, Lui and his companions took her old Bulova
wristwatch, necklace, ring and old coins. Lui and his two
companions then went down to the ground floor. When
Julieta went out of the room, one of Luis companions
recognized her as Lariosas sister. Lui and his companions
brought her along with them as they left the house.

the complaint of Paulina, docketed as Barangay Case No.


168.5
On November 8, 1988, Lariosa executed an uncounselled
confession where he stated that he stole P40,000.00 on
October 15, 1988 from the Davao United Products, and that
he used part of the money to buy appliances, a Sony cassette
tape-recorder, two pairs of ladies shoes, a Seiko wristwatch,
two pairs of maong pants, Rayban sunglasses and floor
mats.6

Matillano, accuseds uncle, is hereby allowed to get


or to retrieve exhibits "H," "I," "J," "K," "L," and
"M," consisting of Sony Cassette with serial no.
W3658; Rayban sunglasses; two (2) bundles of floor
mat; two (2) pairs of pants; two (2) pairs of ladies
shoes; and Seiko Actus wristwatch.10

Meanwhile, Paulina Matillano filed a criminal complaint for


Paulina was so unnerved by the incident. Her vision blurred,
robbery against Lui, Peter Doe, John Doe and Alan
her stomach ached and she was on the verge of losing
Mendoza. An Information was, thereafter, filed against them
consciousness. Concerned, Erlinda massaged Paulinas
in the Municipal Circuit Trial Court of Bansalan, Davao del
On November 16, 1988, an Information was filed in the
stomach. However, Erlinda had to leave because she was
Sur, and the case was docketed as Criminal Case No. 880-B.
Regional
Trial
Court
of
Davao
City,
charging
Lariosa
with
worried about her mother. Paulina then went to the kitchen,
On December 13, 1988, the court issued a warrant for the
robbery with force upon things. The case was docketed as
prepared hot water and put a soothing ointment on her
arrest of the accused therein. Upon reinvestigation, however,
7
Criminal Case No. 17,136,88. The trial court rendered
stomach to relieve the pain.
the Provincial Prosecutor issued a Resolution dated March
judgment on June 14, 1989, acquitting Lariosa of the crime
31, 1989, recommending that the case be dismissed for
In the meantime, Lui and his companions proceeded to the
charged on reasonable doubt. The trial court held that Lui
insufficiency of evidence, but that the charges be forwarded
Bansalan Police Station and caused an entry in the police
procured Lariosas confession through force and
to the Judge Advocate Generals Office for possible
blotter at 3:20 p.m. that he had recovered the following items intimidation, in connivance with police authorities. 8 The trial administrative sanctions against Rojas.
from the Matillano residence -- one pair of colored blue
court, likewise, found that Lui had an ulterior motive for
pants valued at P89.00; one floor mat costing P290.00; a pair charging Lariosa of robbery:
WHEREFORE, in view of the foregoing, it is respectfully
of black ladies shoes worth P126.00; and another pair of
recommended that the complaint against the respondents Eli
What would have been the possible motive of
ladies shoes worth P69.00.
Lui be dismissed for insufficiency of evidence. Considering
complainant in putting the burden of this charged
that Pat. Leo Rojas is a member of the Integrated National
At 4:30 p.m., Paulina reported to the barangay captain that
against the accused despite want of any appreciable Police, this office is without jurisdiction to entertain the
persons identifying themselves as policemen had gained
evidence, can be gathered in the record, as indicating complaint against him pursuant to Presidential Decree No.
entry into their house and took the following: two polo
the fear of complainant, that the accused will file a
1850. Therefore, let the complaint against Pat. Leo Rojas,
shirts; two t-shirts; two pairs of pants; two floor mats; two
complaint against him in the Department of Labor
together with its annexes, including a copy of the resolution
pairs of ladies shoes; one Bulova wristwatch; one necklace;
for illegally dismissing him in his employment,
of the undersigned, be forwarded to the Judge Advocate
3
one ring; and old coins.
without any sufficient legal grounds and basis. This Generals Office at Camp Catitipan, Davao City, for
unfounded complaint was intended to support
whatever action it may take.11
At 7:35 p.m., Eulogio Matillano made an entry in the
complainants ground against any possible
Bansalan police blotter that earlier that day, at 4:00 p.m.,
complaint, the accused might file against him with
The complaint was docketed as Administrative Case No. 92Rojas took the following from his house: two polo shirts;
the Department of Labor by way of anticipation. 9
0020. The National Police Commission, thereafter, rendered
two t-shirts; 2 pairs of pants; two floor mats; two pairs of
a decision exonerating Rojas of administrative liability for
ladies shoes; 1 Bulova wristwatch; 1 necklace; one ring;
On motion of Lariosa, the trial court ordered the return of the the complainants failure to substantiate the charges. 12 The
and, old coins, without his and his wifes consent and
following exhibits:
Commission held that Rojas was merely complying with the
4
without a search warrant. In the meantime, Doroteo
mission order issued to him when he accompanied Lui and
Accordingly and conformably with the judgment of the latters two companions to the Matillano residence.
Barawan, officer-in-charge of the Office of the Barangay
this court dated June 14, 1989, one Eulogio
Captain, filed a complaint against Kim Kiao, et al., based on

In a Resolution dated August 25, 1989, then Secretary of


Justice Silvestre H. Bello III dismissed the petition for
review of the Provincial Prosecutors resolution filed by
Paulina Matillano. The Secretary of Justice, likewise, denied
a motion for reconsideration thereon.
In a parallel development, Lariosas parents, as well as
Paulina Matillano, filed a complaint for robbery, violation of
domicile, unlawful arrest and/or arbitrary detention against
Leo Rojas, Eli Lui, et al., with the Commission of Human
Rights docketed as CHR Case No. RFO No. 88-0207-DS. In
a Resolution dated December 4, 1989, the Regional Office of
the Commission recommended, thus:
WHEREFORE, premises considered, we are
recommending that there is sufficient prima facie
evidence:
1. to indict Eli Lui for unlawful arrest as
defined under Art. 369 of the Revised Penal
Code, as amended; and
2. to indict both Eli Lui and Pat. Leo Rojas
liable for Violation of Domicile, as defined
under Art. 128 of the same code.13
The Proceedings in the Trial Court
On January 11, 1990, the spouses Eulogio and Paulina
Matillano filed a civil complaint for damages in the Regional
Trial Court of Davao del Sur against Eli Lui, Leo Rojas,
Alan Mendoza and Henry Tan. The case was docketed as
Civil Case No. G-XXI-47(90). The plaintiffs therein alleged
the following:
3. That plaintiffs are merchants by occupation and
have been residing in Bansalan, Davao del Sur, for
several years now. They are law-abiding and
peaceful citizens in the community;

4. That at about 3:00 oclock in the afternoon of


jointly and severally exemplary damages in the
November 6, 1988, while plaintiff husband was
amount of P300,000.00;
away from his residential house at Lily St.,
9. That in the protection of their rights, plaintiffs
Bansalan, Davao del Sur, and plaintiff wife was
engaged the services of counsel for an agreed
there tending the house, defendants, without any
attorneys fees equivalent to 25% of the total award
lawful search warrant, arrived and thru intimidation
plus per diem of P1,000.00 per court appearance;
succeeded in searching the house owned by the
plaintiff after which they brought with them two
10. That plaintiffs are bound to incur litigation
floor mats, two pairs of ladies shoes, two pairs of
expenses in an amount not less than P10,000.00;14
pants, two polo shirts, two T-shirts, one Relova wrist
watch, one necklace (sinubong), one ring (sinubong)
They prayed that, after due proceedings, judgment be
and several old coins, without the consent of the
plaintiffs and without even giving any receipt for the rendered in their favor, viz:
items taken;
WHEREFORE, it is most respectfully prayed that
after hearing judgment issue ordering the defendants
5. That the defendants allegedly wanted to recover
to jointly and severally pay plaintiffs:
the items taken by one Elinito Lariosa but
defendants thru the use of naked power and brute
1. P500,000.00 as moral damages;
force, illegally searched the house of the herein
plaintiffs in gross violation of plaintiffs
2. P300,000.00 as exemplary damages;
constitutional rights;
6. That what defendants did in conspiring and
confederating to illegally search the house of
plaintiffs and then taking with them the items
mentioned above without even the benefit of any
receipt is not only violative of Article 19 in relation
to Article 21 of the Civil Code but also of Article 32
of the Civil Code;
7. That because of what defendants did, plaintiffs
suffered mental anguishes, wounded feelings,
deprivation of the properties taken, besmirched
reputation, and fright for which reason defendants
should be made to jointly and severally pay moral
damages in the amount of P500,000.00;
8. That in order to deter others similarly bent and
minded and by way of example or correction for the
public good, defendants should be made to pay

3. Litigation expenses of P10,000.00;


4. Attorneys fees equivalent to 25% of the
total award;
5. Per diems to be proved during the trial of
this case.
Plaintiffs pray for other reliefs consistent with
equity.15
In their Answer to the complaint, the defendants therein
alleged, inter alia, that they did not conduct a search in the
house of the plaintiffs and that plaintiff Paulina Matillano
allowed them to enter the house and even brought out pairs
of pants. They added that the other items were brought out
by Lariosas sister and that they took only one (1) floor mat,

two (2) pairs of ladies shoes, and one (1) pair of blue
pants.16

2. THE LOWER COURT ERRED IN FINDING


THAT MRS. PAULINA MATILLANO WAS THE
ONE WHO REPORTED THE MATTER TO THE
BANSALAN POLICE STATION.

The defendants adduced evidence that plaintiff Paulina


Matillano allowed them to enter their house, and with
Lariosas sister, voluntarily turned over the items declared in
3. THE LOWER COURT ERRED IN DISMISSING
the complaint. They testified that no violence, threats or
THE COMPLAINT DESPITE CLEAR
intimidation were even committed by them against Paulina
PREPONDERANCE OF EVIDENCE AGAINST
Matillano. Defendant Rojas further testified that he was
THE DEFENDANTS APPELLEES.17
merely complying with the Mission Order issued to him
when he entered the house of the plaintiffs in the company of On April 22, 1999, the Court of Appeals rendered judgment
reversing the decision of the RTC. The decretal portion of
the other defendants, and that he remained in the ground
the decision reads:
floor while the other defendants retrieved the goods from
plaintiff Matillano in the second floor of the house.
IN VIEW OF ALL THE FOREGOING, the decision
appealed from is hereby REVERSED and SET
On August 18, 1993, the RTC rendered judgment, ordering
ASIDE and a new one entered ordering defendantsthe dismissal of the complaint for plaintiffs failure to prove
appellees jointly and severally:
their claims. The trial court also dismissed the defendants
counterclaims. The trial court gave credence to the collective
1. To pay plaintiffs-appellants the amount of
testimonies of the defendants, that plaintiff Paulina Matillano
Fifty Thousand Pesos (P50,000.00) as moral
voluntarily allowed them to enter her house, and that the
damages and Fifteen Thousand Pesos
latter voluntarily turned over the subject items to them. The
(P15,000.00) as exemplary damages; and
trial court took into account the findings of the Provincial
Prosecutor, the Secretary of Justice, the National Police
2. Ten Thousand Pesos (P10,000.00), as
Commission, as well as the order of the Municipal Circuit
attorneys fees; and
Trial Court of Bansalan, dismissing Criminal Case No. 880B.
3. To pay the costs.
The Case on Appeal
SO ORDERED.18
The decision of the trial court was elevated to the Court of
The appellate court denied the appellees motion for
Appeals where the appellants contended, thus:
reconsideration of the said decision. The appellees Mendoza
and Tan no longer appealed the decision.
1. THE LOWER COURT ERRED IN FINDING
THAT APPELLANT PAULINA MATILLANO
Petitioners Eli Lui and Leo Rojas now assail the decision of
VOLUNTARILY ALLOWED APPELLEES TO
the Court of Appeals contending that:
ENTER THE HOUSE BECAUSE OF THE
PRESENCE OF HER NEPHEW ELINITO
I. THE HONORABLE COURT OF APPEALS
LARIOSA WHO WAS HANDCUFFED;
DISREGARDED THE TIME-HONORED

DOCTRINE LAID DOWN BY THIS


HONORABLE COURT THAT FINDINGS OF
TRIAL COURT ARE BINDING AND
CONCLUSIVE AND DESERVE A HIGH DEGREE
OF RESPECT, WHEN IT SET ASIDE THE
FINDINGS OF FACTS AND ASSESSMENT OF
THE REGIONAL TRIAL COURT THAT TRIED
THE CASE;
II. THE HONORABLE COURT OF APPEALS
ERRONEOUSLY CONCLUDED THAT AN
ILLEGAL SEARCH WAS CONDUCTED IN MRS.
MATILLANOS RESIDENCE, IN DISREGARD
OF THE EXCULPATORY FINDINGS OF THE
TRIAL COURT THAT MRS. MATILLANO HAD
VOLUNTARILY ALLOWED PETITIONERS
ENTRY INTO HER HOUSE.19
The Issues
The issues in this case may be synthesized, thus: (a) whether
or not respondent Paulina Matillano consented to the
petitioners entry into her house, as well as to the taking of
the clothes, shoes and pieces of jewelry owned by her and
her family; (b) whether or not the petitioners are liable for
damages to the respondents; and, (c) if so, the extent of the
petitioners liability to the respondents.
Considering that the assignments of errors are interrelated,
this Court shall delve into and resolve them simultaneously.
The Courts Ruling
The petition has no merit.
Admittedly, the issues in the case at bar are factual. Under
Rule 45 of the Rules of Court, only questions of law may be
raised in this Court in a petition for review on certiorari.
However, the rule admits of some exceptions, such as a case
where the findings of facts of the trial court are substantially

different from those of the appellate court, and the resolution respondent Paulina Matillano was. Over her vehement
of such issues are determinative of the outcome of the
protests, and because of petitioner Luis warning that she
petition.20
might be harmed, respondent Paulina Matillano was forced
to accompany the petitioner and his cohorts to the second
The petitioners aver that the Court of Appeals committed a
floor of their house. The foregoing was testified to by
reversible error in discarding the factual findings of the trial respondent Paulina Matillano, thus:
court. Contrary to the disquisitions of the appellate court, the
petitioners assert that the inconsistencies between the
ATTY. SUARIO:
testimonies of Rojas and Lui are peripheral. Lui did not
Q Mrs. Matillano, do you know the person of Eli
conduct any search in the second floor of the respondents
Lui?
house and even if he did so, respondent Paulina Matillano
waived her right against unreasonable search when she
A I know him.
allowed the petitioners to enter. According to the petitioners,
the respondents failed to prove that they forced their way
Q Why do you know Eli Lui?
into the house of the respondents, and that the facts and
circumstances which the appellate court found the trial court
A Because he is from Bansalan.
to have overlooked are not, in fact, substantial enough to
warrant a reversal of the factual findings of the court a quo.
Q On November 6, 1988, where were you, Mrs.
According to the petitioners, the appellate court failed to
Matillano?
discern that the action filed by the respondents with the trial
court was merely a leverage to the charge of robbery against
Lariosa, the respondents nephew.

A I was in our house.

On the other hand, the Court of Appeals gave credence and


full probative weight to the evidence of the respondents. It
stated in its decision that the trial court erred in giving
credence and probative weight to the testimonies of the
petitioners (the appellants therein). Moreover, the appellate
court found that the trial court had overlooked facts and
circumstances of substance, which, if considered, would
have altered the courts decision. The appellate court gave
weight to the findings of the trial court in Criminal Case No.
17,136,88.21

Q At about 3:00 oclock in the afternoon of


November 6, 1988, did you notice any unusual
incident that took place in your house?

A I was in our sala.


Q Now, what did you do when you saw these five
(5) persons entered (sic) your house?
A I was afraid.
Q Aside from fear, what did you do?
A One of them suddenly said, "Mrs., we are
authorities."
ATTY. TAN:
Not responsive to the question, Your Honor.
ATTY. SUARIO:
She is responding the question because my question
is, "Aside from fear, what did you do?" and
according to this witness, she was not able to do
anything because one of those who entered(not
continued)
COURT:

A There was.

I think the answer is not responsive. Just reform the


question.

Q What incident was that, Mrs. Matillano?

ATTY. SUARIO:

A There were five (5) persons who suddenly went


inside our house.

Q What did these persons do when they entered your


house?

We agree with the Court of Appeals.

Q Where did they enter?

A One of them said, "Mrs., we are authorities. We


are here to get something from your house."

The evidence of the respondents show that the petitioners,


Tan and Mendoza, guns drawn and with the handcuffed
Lariosa in tow, kicked the kitchen door and barged into the
house of the respondents. They proceeded to the sala where

A They entered through the kitchen.


Q Now, where were you when they entered suddenly
in your house?

Q Do you know who this person was, this person


who was talking that they were persons in authority?

A That person when he first went to our house, I do


not know him yet, but I know (sic) him later to be
Leo Rojas.

A Yes, because they said that they are taking some of


our things and I said why are they doing that
(manghilabot)?

Q Why do you know him later to be Leo Rojas?

Q When you said those remarks, what else


happened?

A When the case was already being tried, he


introduced himself as Leo Rojas.
Q What was Leo Rojas wearing at that time?

A It was Eli Lui who answered, "Mrs., do not answer


anymore because something might happen." (Basig
madisgrasya).

ATTY. TAN:
Q Now, you said on November 6, 1988, five (5) men
suddenly entered your house. When you said
suddenly, will you please describe how did they
enter the house?
A They passed through the kitchen and suddenly
appeared inside the house.

A He was in civilian clothes.

ATTY. SUARIO:

Q You mean to say that they did not knock at the


door?

Q Aside from Leo Rojas, who were the other persons


who entered your house?

"Madisgrasya," Your Honor, is more than something.

A They did not.

ATTY. SUARIO:

Q Who first entered the house among the five (5)?

Q When you heard those words from Eli Lui, what


else transpired?

A What I first saw was that they immediately


converged in the sala and whom I recognized was
Eli Lui and my nephew who was in handcuffs.

A Aside from the two (2) persons whom I do not


know, my nephew was also with them in the name of
Elinito Lariosa.
Q Who else, Mrs. Matillano?
A Eli Lui.

A He said, "All right, where is your aparador


because we are getting something." And I even told
him that we should wait for my husband but they did
not agree because they said they are in a hurry.

Q Was your door opened at that time?


A It was closed but it was not locked. It can be
kicked open.

Q And after that, what else happened?


ATTY. SUARIO:

Q But you can open it without kicking the door?


A I accompanied him upstairs.

At least, may we ask, Your Honor, that the word


"manghilabot" be incorporated.

A Yes, sir.
Q You accompanied him upstairs, who are you
referring to that you accompanied upstairs.

COURT:

Q Now, you said that you were afraid, why were you
afraid?

A Eli Lui and his other two (2) companions.


So, the word is "interfering" or "meddling." You
record the word "manghilabot."

Q These two (2) companions whom you said you do


not know their names?

ATTY. SUARIO:

A Why would you not be afraid when they were


armed?
Q Who were armed among the five (5)?

22

A Yes, sir.
Q When you said "manghilabot," what do you mean,
Mrs. Matillano?

A All of them except the one who was in handcuffs.

Q You are very sure of that?

Q One (1) Bulova wristwatch?

A I think it was in the month of November.

A I am very sure.23

A Yes.

Q 1988?

Q One (1) necklace?

A 1988.

A Yes.

Q And who owns these two (2) polo-shirts?

Q Two (2) pairs of lady (sic) shoes?

A My children.

A Yes.

Q What are the names of your children?

Q Two (2) pairs of pants?

A Allan and Danilo.

A Yes.

Q Where is Allan residing?

Q One (1) ring?

A During the incident, Allan was still schooling in


Tacloban.

Respondent Paulina Matillano, likewise, testified that


petitioner Lui and his cohorts took her personal things, and
those of her familys, from the second floor of the house:
Q Now, while you and Eli Lui with two (2) other
companions were upstairs, what happened upstairs?
A Upon reaching upstairs, they immediately rolled
the two (2) floor mats, the pair of leather shoes, 2
pairs of pants, two (2) polo-shirts. They also let me
open the chest and when it was already open they
rummaged through it and they got my old Bulova
watch, my necklace, my ring and a coinsita, old gold
coins.
Q When you said "coinsita," what is "coinsita"?
A Old coins.

A Yes.
Q Who owns these two (2) pairs of ladys (sic)
shoes?

Q So, you mean to say, on November 6, 1988, he


was no longer residing in Bansalan?
A No more.

Q After taking all of these things, what else


happened?
A They went downstairs.24

A That was mine.


Q What were the color of the shoes?
A Black and dirty white (referring to the color of the
rostrum).

Q How about Danilo, where was he residing in


November 6, 1988?
A He was living in Sta. Cruz.
Q He has a family of his own at Sta. Cruz?

Q Now, you mentioned in this affidavit that several


properties were taken from your house, do you
confirm that there were two (2) polo-shirts that were
taken?
A Yes.
Q And there were also two (2) floor mats?
A Yes, that is true.

Q Where did you buy that shoes?


A He was still single then.
A In Davao City.
Q But he was residing in Sta. Cruz?
Q What store in Davao City?
A Yes.
A NCCC.
Q What particular date when you bought that shoes?

Q How about these two (2) pairs of pants, who owns


these pants?

A My children also.

Q These polo shirts were still new?

Q You are referring to Allan and Danilo?

A Already used.

A No, because I still have so many children.

Q How about the pants?

Q So, who owns these two (2) pants?

A The other one is already used and the other one is


new.

A Also my children, Eulogio, Jr. and Allan.


Q Now, Eulogio, Jr. where is (sic) he residing on
November 6, 1988?
A In our house.

Q And inspite (sic) the fact that there were more than
three (3) dozens of clothes, pants, polo shirts and tshirts only these two (2) pants, two (2) polo shirts
and two (2) t-shirts w ere taken?
A Only those things because they only selected the
ones which were still usable the good ones.

Q How about the floor mats?

Q Now, you mentioned also in your affidavit that the


group also searched your trunk?

A That is mine.

A I was ordered to open the trunk.

Q Now, you claimed that these clothes were taken


from the cabinet or aparador, is that correct?

Q Who particularly ordered you to open the trunk?

Q How about these two (2) t-shirts?

A Eli Lui.25
A Yes, that is true.

A Also owned by my children.


Q Are you referring to Allan and Danilo?

Q Inside your aparador, how many pieces of clothes


were stored therein?

A They used to wear that.

A Many.

Q How come that Allan has a polo-shirt in your


house when you said he was then residing in
Tacloban?

Q Could you say one (1) dozen?

ATTY. SUARIO:

Q Could you say three (3) dozens?

May we manifest, Your Honor, that he was schooling


in Tacloban.

A It is really full of dress.

A It cannot be counted.

Q Would you say it is more than three (3) dozens?


COURT:
A More.
All right.
A They used to have a vacation during December
and March and usually they left some of their
clothes inside our aparador.

Q And these more than three (3) dozens consists of


polo shirts, t-shirts and pants?
A Yes.

The respondents immediately reported the matter to the


Office of the Barangay Captain26 and filed a complaint
against petitioner Lui and his cohorts.27
The petitioners claim that respondent Paulina Matillano
allowed them and their cohorts inside the house and
voluntarily gave their personal belongings is belied by the
unshaken testimony of respondent Paulina Matillano,
corroborated by Erlinda Clarin.
The petitioners attempt to project themselves to have acted
with civility and courtesy to respondent Paulina Matillano is
implausible, taking into account petitioner Luis state of
mind before he and petitioner Rojas and their cohorts left the
Metrodiscom Headquarters in Davao City, and proceeded to
the house of the respondents in Bansalan. Before they left
Davao City, Lui sadistically mauled Lariosa with the
acquiescence of the police authorities, and forced him to give
an uncounselled extrajudicial confession. This was the
finding of the RTC in Criminal Case No. 17,136,88:
Despite being mauled by Eli Lui and drowned in a
toilet bowl, accused denied having anything to do

with the lost money of the complainant. Later, he


was turned over to the police for investigation and
there without affording accused with his right to
counsel, he was interrogated orally and was forced
to admit that out of the money he stole, he bought
items which the police later recovered at Bansalan.
They also returned the accused to the complainants
establishment and forced to do re-enactment of the
act of robbery, without accused again afforded the
right to counsel. Pictures were taken during the reenactment while accused was handcuffed, as shown
in the pictures taken by the police.
Finally, the accused was forced to admit and sign his
extrajudicial statement (Exhibit A), no longer able to
bear the pain of the mauling to him by Eli Lui, who
has the temerity of maltreating the accused even in
the presence of the guards in the jail and seriously
threatening accused to admit ownership of the
recovered items at Bansalan and at New Matina,
SIR, Davao City, otherwise he will be salvaged,
along with the serious threatening words of
accuseds companion in the jail, that if he will refuse
to sign his alleged confession, he will be salvaged as
directed by Eli Lui with the police.
Indeed, in the records, it can be deduced with
sufficient basis, that Eli Lui seems to have an open
hand in the prosecution of accused. He was the one
who called the police to arrest him, even without a
warrant of arrest. Before his statement was obtained,
policeman relied on him in the investigation and the
filing of proper charges against accused. They rode
in a car of Eli Lui, in taking accused from the
Metrodiscom to the establishment of complainant
during the re-enactment in going to Bansalan, to
recover the items allegedly bought by accused out of
the money allegedly stolen; all of these incidents
shows (sic) [that] the police despite justification, that
they do not have enough facilities (sic), [had] gone

astray in conducting an impartial investigation, by


submitting to any possible indiscretion of Eli Lui of
making the scale of justice bend in his favor, by
manifesting control over the police power of
investigation highly and seriously pre-judicial to the
rights, and interests of the accused.28

the penalty shall be prision correccional in its medium and


maximum periods.

Although petitioner Rojas did not follow petitioner Lui and


his cohorts to the second floor of the respondents house and
himself conduct a search therein, he allowed them to search
the premises without a warrant. The petitioners and their
If petitioner Lui was so brazen as to have mauled Lariosa in cohorts were not authorized to conduct a search in the house
the presence of police authorities, he would not have cared a of the respondents, much less divest the latter of their
whit in barging into the respondents house with petitioner
personal belongings. As a police officer, it was petitioner
Rojas, a policeman of Davao City, and his cohorts, and
Rojas duty to prevent the commission of crimes in his
divesting the respondents of their belongings. The petitioners presence, and to arrest the persons committing such crimes.
and their cohorts wanted to insure that their caper would
The trial court rejected the testimony of respondent Paulina
succeed. Hence, they did not coordinate with the Bansalan
Matillano on the following grounds: (a) she had known
Police Station when they went to the respondents house
petitioner Lui for ten years as a businessman doing business
with their intention to divest them of their belongings.
in Bansalan; (b) the occupants of the respondents house
Petitioner Rojas reliance on Mission Order No. MRF-Awhen the petitioners and their cohorts arrived were all
004-98 issued to him by Sergeant Alberto Genise is
women; (c) the respondents failed to report the incident to
misplaced. It bears stressing that the petitioner was merely
the Bansalan police authorities; and, (d) the provincial
tasked in the said order to "follow up a theft case within the prosecutors resolution recommending the dismissal of
area of responsibility of the Metrodiscom, Davao City." The Criminal Case No. 880-B for robbery against the petitioners,
petitioner was not authorized, under the said order, to
which was sustained by the Secretary of Justice, and the
commit or tolerate the commission of a crime, such as
ruling of the National Police Commission exonerating
violation of domicile as defined in Article 128 of the Revised petitioner Rojas from any liability.
Penal Code, viz:
We find that the Court of Appeals was correct in overruling
ART. 128. Violation of domicile The penalty of
the trial court.
prision correccional in its minimum period shall be
First. Respondent Paulina Matillano testified that petitioner
imposed upon any public officer or employee who,
Lui did not stay permanently in Bansalan. He went there
not being authorized by judicial order, shall enter
only to collect money from a certain Matura and other
any dwelling against the will of the owner thereof,
search papers or other effects found therein without businessmen.29 She also testified that there were many cases
against the petitioner, one of which was for arson. The case
the previous consent of such owner, or, having
was dismissed, but one of her neighbors was rendered
surreptitiously entered said dwelling, and being
required to leave the premises, shall refuse to do so. missing.30 If the petitioner, a businessman for ten years or so,
had no qualms in torturing Lariosa under the very noses of
If the offense be committed in the nighttime, or if any papers police officers, he would, likewise, have no qualms about
or effects not constituting evidence of a crime be not
intimidating respondent Paulina Matillano and divesting her
returned immediately after the search made by the offender, of her personal belongings. It must be stressed that petitioner

Lui was in the company of petitioner Rojas, a police officer


from Davao City.
Second. The petitioners and their cohorts had no
foreknowledge that the occupants of the respondents house
were all women. They must have believed that there were
male occupants; hence, barged into the house with drawn
guns.
Third. As shown clearly in respondent Paulina Matillanos
sworn statement before the Bansalan Police Station, she
declared that the petitioners were armed with guns. They
threatened her life and, without any search warrant therefor,
divested her and her family of their personal belongings
against their will.31

recover the properties from the house of the respondents


which petitioner Lui perceived to have been acquired by
Lariosa with money stolen from his uncle, Ben.36 The
decision of the National Police Commission absolving
petitioner Rojas of grave misconduct was anchored on its
finding that the petitioner was merely performing his duty as
ordered by his superior officer.37 It was inevitable for the
City Prosecutor to dismiss the complaint for violation of
domicile filed against petitioner Rojas in I.S. No. 91-1488
because the crime of violation of domicile was committed in
Bansalan and not in Davao City.38 In contrast, the
Commission on Human Rights recommended the indictment
of petitioner Lui for unlawful arrest and of petitioner Rojas
for violation of domicile.39

Sixth. Under Articles 19 and 32, in relation to Article 21 of


Fourth. In her complaint before the Office of the Barangay
the New Civil Code, the dismissal of the complaint against
Captain, respondent Paulina Matillano declared that the
the petitioners by the Provincial and City Prosecutors, the
petitioners entered their house, that petitioner Lui pointed a Municipal Trial Court and the National Police Commission
gun at her, and that the petitioners and their cohorts searched are of no relevance to the civil complaint for damages filed
the house and carted away their personal belongings. 32 That by the respondents against the petitioners. The action of the
the report made before the Barangay Captain and petitioner respondents against the petitioners may still proceed despite
Paulina Matillanos sworn statement are not as complete as
the dismissal of the criminal and administrative actions
her testimony before the trial court is understandable.
against them.
Affidavits are usually taken ex parte and are almost always
incomplete and inaccurate, but they do not detract from the
The petitioners contention that respondent Paulina Matillano
credibility of the witness.33 An entry in the police blotter is
waived her right against unreasonable search and seizure
usually incomplete and inaccurate for want of suggestions or deserves scant consideration. Under Article III, Section 2 of
inquiries, without the aid of which the victim may be unable the Constitution, "the right of the people to be secure in their
to recall the connected collateral circumstances necessary for persons, houses, papers and effects against unreasonable
the correction of the first suggestion of his memory, and for searches and seizures of whatever nature and for any purpose
his accurate recollection of all that pertain to the subject. 34
shall be inviolable." This provision protects not only those
The same principle applies to entries in the barangay blotter. who appear to be innocent but also those who appear to be
guilty, who must nevertheless be presumed innocent until the
Fifth. As correctly held by the trial court, the findings of
contrary is proved.40 The general rule is that a search and
administrative and quasi-administrative agencies are not
seizure must be carried through or with judicial warrant;
binding on the courts. In the present case, the Office of the
otherwise, such a search and seizure becomes
Provincial Prosecutor, as affirmed by the Secretary of
unconstitutional within the context of the constitutional
Justice,35 found no probable cause for robbery against the
provision41 because a warrantless search is in derogation of a
petitioners because they had no intent to rob, but merely to
constitutional right. Peace officers who effect a warrantless

search cannot invoke regularity in the performance of


official functions.42
The right against unreasonable searches and seizures is a
personal right which may be waived expressly or impliedly.
But a waiver by implication cannot be presumed.43 There
must be clear and convincing evidence of an actual intention
to relinquish the right to constitute a waiver of a
constitutional right. There must be proof of the following: (a)
that the right exists; (b) that the person involved had
knowledge, either actual or constructive, of the existence of
such right; and, (c) that the said person had an actual
intention to relinquish the right.44 The waiver must be made
voluntarily, knowingly and intelligently. The Court indulges
every reasonable presumption against any waiver of
fundamental constitutional rights.45 The fact that the
aggrieved person did not object to the entry into her house
by the police officers does not amount to a permission to
make a search therein.46 A peaceful submission to search and
seizure is not a consent or an invitation thereto, but is merely
a demonstration of regard for the supremacy of the law.47
In this case, the petitioners failed to prove, with clear and
convincing evidence, that respondent Paulina Matillano
waived her right against unreasonable search and seizure by
consenting thereto, either expressly or impliedly. Admittedly,
respondent Paulina Matillano did not object to the opening
of her wooden closet and the taking of their personal
properties. However, such failure to object or resist did not
amount to an implied waiver of her right against
unreasonable search and seizure. The petitioners were armed
with handguns; petitioner Lui threatened and intimidated her.
Respondent Eulogio Matillano, her husband, was out of the
house when the petitioner and his cohorts conducted the
search and seizure. He could, thus, not have waived his
constitutional right.
Furthermore, the petitioners claim that respondent Paulina
Matillano voluntarily handed over the articles to petitioner
Lui is incredible. There is no evidence that there was

foreknowledge on the part of the petitioners of the articles


they wanted to retrieve from the respondents house. Even if
respondent Paulina Matillano did hand over the articles to
the petitioner, it was only because the petitioner and his
cohorts had earlier threatened and intimidated her into doing
so.
We agree with the ruling of the Court of Appeals that the
petitioners are liable to the respondents for moral and
exemplary damages in the amounts respectively awarded by
it. Petitioner Rojas, a policeman of Davao City, conspired
with petitioner Lui and, with drawn guns, gained entry into
the respondents house, and threatened and intimidated
respondent Paulina Matillano. Although petitioner Rojas did
not himself conduct the search, he assented thereto by
allowing petitioner Lui and his cohorts to go up to the
second floor and divest the respondents of their belongings.
The petitioners even left together after the incident.
In MHP Garments, Inc. vs. Court of Appeals,48 we had the
occasion to state:
In the case of Lim vs. Ponce de Leon, we ruled for
the recovery of damages for violation of
constitutional rights and liberties from public officer
or private individual, thus:
"ART. 32. Any public officer or employee, or any
private individual, who directly or indirectly
obstructs, defeats, violates or in any manner impedes
or impairs any of the following rights and liberties of
another person shall be liable to the latter for
damages.
"x x x

"x x x
"The indemnity shall include moral damages.
Exemplary damages may also be adjudged."
"ART 2219. Moral damages may be recovered in the
following and analogous cases:
"x x x
"(6) Illegal search;

"[T]he decisive factor in this case, in our view, is the


language of Article 32. The law speaks of an officer
or employee or person directly or indirectly
responsible for the violation of the constitutional
rights and liberties of another. Thus, it is not the
actor alone (i.e., the one directly responsible) who
must answer for damages under Article 32; the
person indirectly responsible has also to answer for
the damages or injury caused to the aggrieved party.
xxx

"(1) Acts and actions referred to in Articles 21, 26,


27, 28, 29, 30, 32, 34, and 35.
"Pursuant to the foregoing provisions, a person
whose constitutional rights have been violated or
impaired is entitled to actual and moral damages
from the public officer or employee responsible
therefor. In addition, exemplary damages may also
be awarded."
xxx
"The very nature of Article 32 is that the wrong may
be civil or criminal. It is not necessary therefore that
there should be malice or bad faith. To make such a
requisite would defeat the main purpose of Article
32 which is the effective protection of individual
rights. Public officials in the past have abused their
powers on the pretext of justifiable motives or good
faith in the performance of their duties. Precisely, the
object of the Article is to put an end to official abuse
by plea of the good faith. In the United States this
remedy is in the nature of a tort." (emphasis
supplied)

"(9) the rights to be secure in ones persons, house,


papers and effects against unreasonable searches and In the subsequent case of Aberca vs. Ver, the Court En Banc
explained the liability of persons indirectly responsible, viz:
seizures.

"While it would certainly be too nave to expect that


violators of human rights would easily be deterred
by the prospect of facing damage suits, it should
nonetheless be made clear in no uncertain terms that
Article 32 of the Civil Code makes the persons who
are directly, as well as indirectly, responsible for the
transgression joint tortfeasors.
xxx
[N]either can it be said that only those shown to
have participated directly should be held liable.
Article 32 of the Civil Code encompasses within the
ambit of its provisions those directly, as well as
indirectly, responsible for its violations." (emphasis
supplied)
Applying the aforecited provisions and leading cases, the
respondent court correctly granted damages to private
respondents. Petitioners were indirectly involved in
transgressing the right of private respondents against
unreasonable search and seizure. Firstly, they instigated the
raid pursuant to their covenant in the Memorandum
Agreement to undertake the prosecution in court of all illegal
sources of scouting supplies. As correctly observed by
respondent court:

"Indeed, the acts committed by the PC soldiers of


unlawfully seizing appellees (respondents)
merchandise and of filing the criminal complaint for
unfair competition against appellees (respondents)
were for the protection and benefit of appellant
(petitioner) corporation. Such being the case, it is,
thus, reasonably fair to infer from those acts that it
was upon appellant (petitioner) corporations
instance that the PC soldiers conducted the raid and
effected the illegal seizure. These circumstances
should answer the trial courts query posed in its
decision now under consideration as to why the PC
soldiers immediately turned over the seized
merchandise to appellant (petitioner) corporation."
The raid was conducted with the active participation of their
employee. Larry de Guzman did not lift a finger to stop the
seizure of the boy and girl scout items. By standing by and
apparently assenting thereto, he was liable to the same extent
as the officers themselves. So with the petitioner corporation
which even received for safekeeping the goods unreasonable
seized by the PC raiding team and de Guzman, and refused
to surrender them for quite a time despite the dismissal of its
complaint for unfair competition.49
IN LIGHT OF ALL THE FOREGOING, the petition is
DISMISSED. The Decision of the Court of Appeals is
AFFIRMED in toto. Costs against the petitioners.
SO ORDERED.
G.R. No. 182677

August 3, 2010

JOSE ANTONIO C. LEVISTE, Petitioner,


vs.
HON. ELMO M. ALAMEDA, HON. RAUL M.
GONZALEZ, HON. EMMANUEL Y. VELASCO,
HEIRS OF THE LATE RAFAEL DE LAS ALAS,
Respondents.

DECISION

Meantime, petitioner filed an Urgent Ex-Parte Manifestation


and Motion before the trial court to defer acting on the
CARPIO MORALES, J.:
public prosecutors recommendation on the proper offense
until after the appellate court resolves his application for
Jose Antonio C. Leviste (petitioner) assails via the present
injunctive reliefs, or alternatively, to grant him time to
petition for review filed on May 30, 2008 the August 30,
comment on the prosecutors recommendation and thereafter
1
2
2007 Decision and the April 18, 2008 Resolution of the
set a hearing for the judicial determination of probable
Court of Appeals in CA-G.R. SP No. 97761 that affirmed the cause.10 Petitioner also separately moved for the inhibition of
trial courts Orders of January 24, 31, February 7, 8, all in
Judge Alameda with prayer to defer action on the admission
2007, and denied the motion for reconsideration,
of the Amended Information.11
respectively.
The trial court nonetheless issued the other assailed orders,
Petitioner was, by Information3 of January 16, 2007, charged viz: (1) Order of February 7, 200712 that admitted the
with homicide for the death of Rafael de las Alas on January Amended Information13 for murder and directed the issuance
12, 2007 before the Regional Trial Court (RTC) of Makati
of a warrant of arrest; and (2) Order of February 8, 200714
City. Branch 150 to which the case was raffled, presided by which set the arraignment on February 13, 2007. Petitioner
Judge Elmo Alameda, forthwith issued a commitment order 4 questioned these two orders via supplemental petition before
against petitioner who was placed under police custody
the appellate court.
while confined at the Makati Medical Center.5
The appellate court dismissed petitioners petition, hence, his
After petitioner posted a P40,000 cash bond which the trial
present petition, arguing that:
6
court approved, he was released from detention, and his
arraignment was set on January 24, 2007.
PRIVATE RESPONDENT DID NOT HAVE THE RIGHT
TO CAUSE THE REINVESTIGATION OF THE
The private complainants-heirs of De las Alas filed, with the CRIMINAL CASE BELOW WHEN THE CRIMINAL
conformity of the public prosecutor, an Urgent Omnibus
INFORMATION HAD ALREADY BEEN FILED WITH
7
Motion praying, inter alia, for the deferment of the
THE LOWER COURT. HENCE, THE COURT OF
proceedings to allow the public prosecutor to re-examine the APPEALS COMMITTED A GRAVE ERROR IN FINDING
evidence on record or to conduct a reinvestigation to
THAT RESPONDENT JUDGE DID NOT ACT WITH
determine the proper offense.
GRAVE ABUSE OF DISCRETION IN GRANTING SUCH
REINVESTIGATION DESPITE HAVING NO BASIS IN
The RTC thereafter issued the (1) Order of January 24, 20078 THE RULES OF COURT[;]
deferring petitioners arraignment and allowing the
prosecution to conduct a reinvestigation to determine the
RESPONDENT JUDGE ACTED WITH GRAVE ABUSE
proper offense and submit a recommendation within 30 days OF DISCRETION IN ADMITTING STATE PROSECUTOR
from its inception, inter alia; and (2) Order of January 31,
VELASCOS AMENDED INFORMATION, ISSUING A
20079 denying reconsideration of the first order. Petitioner
WARRANT OF ARREST, AND SETTING THE CASE
assailed these orders via certiorari and prohibition before the BELOW FOR ARRAIGNMENT, CONSIDERING THAT
Court of Appeals.
THE VALIDITY AND LEGALITY OF HIS ORDERS
DATED 24 AND 31 JANUARY 2007, WHICH LED TO

THE QUESTIONABLE REINVESTIGATION AND


ILLEGAL AMENDED INFORMATION[,] ARE YET TO
BE RESOLVED BY THIS HONORABLE COURT (sic);
[AND]

which this Court, in G.R. No. 189122, affirmed by Decision


of March 17, 2010.

enters his plea and participates during trial, without


previously invoking his objections thereto."19 There must be
clear and convincing proof that petitioner had an actual
The Office of the Solicitor General (OSG) later argued that
intention to relinquish his right to question the existence of
the present petition had been rendered moot since the
probable cause. When the only proof of intention rests on
CONSIDERING THAT PROSECUTOR VELASCOS
presentation of evidence, wherein petitioner actively
what a party does, his act should be so manifestly consistent
18
FINDINGS IN HIS RESOLUTION DATED 2 FEBRUARY participated, had been concluded.
with, and indicative of, an intent to voluntarily and
2007 ARE BLATANTLY BASED ON MERE
unequivocally relinquish the particular right that no other
SPECULATIONS AND CONJECTURES, WITHOUT ANY Waiver on the part of the accused must be distinguished from explanation of his conduct is possible.20
mootness of the petition, for in the present case, petitioner
SUBSTANTIAL OR MATERIAL NEW EVIDENCE
BEING ADDUCED DURING THE REINVESTIGATION, did not, by his active participation in the trial, waive his
From the given circumstances, the Court cannot reasonably
stated objections.
RESPONDENT JUDGE SHOULD HAVE AT LEAST
infer a valid waiver on the part of petitioner to preclude him
ALLOWED PETITIONERS MOTION FOR A HEARING
from obtaining a definite resolution of the objections he so
Section 26, Rule 114 of the Rules of Court provides:
FOR JUDICIAL DETERMINATION OF PROBABLE
timely invoked. Other than its allegation of active
15
CAUSE. (emphasis in the original omitted)
participation, the OSG offered no clear and convincing proof
SEC. 26. Bail not a bar to objections on illegal arrest, lack
that petitioners participation in the trial was unconditional
Records show that the arraignment scheduled on March 21, of or irregular preliminary investigation. An application
with the intent to voluntarily and unequivocally abandon his
for
or
admission
to
bail
shall
not
bar
the
accused
from
2007 pushed through during which petitioner refused to
petition. In fact, on January 26, 2010, petitioner still moved
challenging the validity of his arrest or the legality of the
plead, drawing the trial court to enter a plea of "not guilty"
for the early resolution of the present petition. 21
warrant issued therefor, or from assailing the regularity or
for him.
questioning the absence of a preliminary investigation of the Whatever delay arising from petitioners availment of
charge against him, provided that he raises them before
Prior thereto or on February 23, 2007, petitioner filed an
remedies against the trial courts Orders cannot be imputed
entering
his
plea.
The
court
shall
resolve
the
matter
as
early
Urgent Application for Admission to Bail Ex Abundanti
to petitioner to operate as a valid waiver on his part. Neither
as practicable but not later than the start of the trial of the
Cautela16 which the trial court, after hearings thereon,
can the non-issuance of a writ of preliminary injunction be
17
case.
granted by Order of May 21, 2007, it finding that the
deemed as a voluntary relinquishment of petitioners
evidence of guilt for the crime of murder is not strong. It
principal prayer. The non-issuance of such injunctive relief
accordingly allowed petitioner to post bail in the amount of By applying for bail, petitioner did not waive his right to
only means that the appellate court did not preliminarily find
challenge
the
regularity
of
the
reinvestigation
of
the
charge
P300,000 for his provisional liberty.
any exception22 to the long-standing doctrine that injunction
against him, the validity of the admission of the Amended
will not lie to enjoin a criminal prosecution.23 Consequently,
Information, and the legality of his arrest under the Amended the trial of the case took its course.
The trial court, absent any writ of preliminary injunction
from the appellate court, went on to try petitioner under the Information, as he vigorously raised them prior to his
Amended Information. By Decision of January 14, 2009, the arraignment. During the arraignment on March 21, 2007,
The petition is now moot, however, in view of the trial
petitioner refused to enter his plea since the issues he raised courts rendition of judgment.
trial court found petitioner guilty of homicide, sentencing
were still pending resolution by the appellate court, thus
him to suffer an indeterminate penalty of six years and one
day of prision mayor as minimum to 12 years and one day of prompting the trial court to enter a plea of "not guilty" for
A moot and academic case is one that ceases to present a
him.
reclusion temporal as maximum. From the Decision,
justiciable controversy by virtue of supervening events, so
petitioner filed an appeal to the appellate court, docketed as
that a declaration thereon would be of no practical use or
The principle that the accused is precluded after arraignment value.24
CA-G.R. CR No. 32159, during the pendency of which he
from questioning the illegal arrest or the lack of or irregular
filed an urgent application for admission to bail pending
preliminary investigation applies "only if he voluntarily
appeal. The appellate court denied petitioners application

The judgment convicting petitioner of homicide under the


Amended Information for murder operates as a supervening
event that mooted the present petition. Assuming that there is
ground25 to annul the finding of probable cause for murder,
there is no practical use or value in abrogating the concluded
proceedings and retrying the case under the original
Information for homicide just to arrive, more likely or even
definitely, at the same conviction of homicide. Mootness
would have also set in had petitioner been convicted of
murder, for proof beyond reasonable doubt, which is much
higher than probable cause, would have been established in
that instance.

investigation, the complaint or information may be filed by a


prosecutor without need of such investigation provided an
inquest has been conducted in accordance with existing
rules. In the absence or unavailability of an inquest
prosecutor, the complaint may be filed by the offended party
or a peace officer directly with the proper court on the basis
of the affidavit of the offended party or arresting officer or
person.

It is imperative to first take a closer look at the predicament


of both the arrested person and the private complainant
during the brief period of inquest, to grasp the respective
remedies available to them before and after the filing of a
complaint or information in court.

BEFORE THE FILING OF COMPLAINT OR


INFORMATION IN COURT, the private complainant may
proceed in coordinating with the arresting officer and the
Before the complaint or information is filed, the person
inquest officer during the latters conduct of inquest.
arrested may ask for a preliminary investigation in
Meanwhile, the arrested person has the option to avail of a
accordance with this Rule, but he must sign a waiver of the
15-day preliminary investigation, provided he duly signs a
provisions of Article 125 of the Revised Penal Code, as
waiver of any objection against delay in his delivery to the
Instead, however, of denying the petition outright on the
amended, in the presence of his counsel. Notwithstanding
proper judicial authorities under Article 125 of the Revised
ground of mootness, the Court proceeds to resolve the legal the waiver, he may apply for bail and the investigation must Penal Code. For obvious reasons, this remedy is not
issues in order to formulate controlling principles to guide
be terminated within fifteen (15) days from its inception.
available to the private complainant since he cannot waive
26
the bench, bar and public. In the present case, there is
what he does not have. The benefit of the provisions of
After the filing of the complaint or information in court
compelling reason to clarify the remedies available before
Article 125, which requires the filing of a complaint or
without a preliminary investigation, the accused may, within information with the proper judicial authorities within the
and after the filing of an information in cases subject of
five (5) days from the time he learns of its filing, ask for a
inquest.
applicable period,32 belongs to the arrested person.
preliminary investigation with the same right to adduce
After going over into the substance of the petition and the
evidence in his defense as provided in this Rule.
The accelerated process of inquest, owing to its summary
assailed issuances, the Court finds no reversible error on the (underscoring supplied)
nature and the attendant risk of running against Article 125,
part of the appellate court in finding no grave abuse of
ends with either the prompt filing of an information in court
A preliminary investigation is required before the filing of a or the immediate release of the arrested person. 33 Notably,
discretion in the issuance of the four trial court Orders.
complaint or information for an offense where the penalty
the rules on inquest do not provide for a motion for
In his first assignment of error, petitioner posits that the
prescribed by law is at least four years, two months and one reconsideration.34
prosecution has no right under the Rules to seek from the
day without regard to fine.28 As an exception, the rules
trial court an investigation or reevaluation of the case except provide that there is no need for a preliminary investigation Contrary to petitioners position that private complainant
through a petition for review before the Department of
in cases of a lawful arrest without a warrant 29 involving such should have appealed to the DOJ Secretary, such remedy is
Justice (DOJ). In cases when an accused is arrested without a type of offense, so long as an inquest, where available, has
not immediately available in cases subject of inquest.
warrant, petitioner contends that the remedy of preliminary
been conducted.30
Noteworthy is the proviso that the appeal to the DOJ
investigation belongs only to the accused.
Inquest is defined as an informal and summary investigation Secretary is by "petition by a proper party under such rules
The contention lacks merit.
conducted by a public prosecutor in criminal cases involving as the Department of Justice may prescribe."35 The rule
persons arrested and detained without the benefit of a
referred to is the 2000 National Prosecution Service Rule on
Section 6,27 Rule 112 of the Rules of Court reads:
warrant of arrest issued by the court for the purpose of
Appeal,36 Section 1 of which provides that the Rule shall
determining whether said persons should remain under
"apply to appeals from resolutions x x x in cases subject of
When a person is lawfully arrested without a warrant
custody and correspondingly be charged in court.31
preliminary investigation/ reinvestigation." In cases subject
involving an offense which requires a preliminary
of inquest, therefore, the private party should first avail of a

preliminary investigation or reinvestigation, if any, before


elevating the matter to the DOJ Secretary.

x x x Since no evidence has been presented at that stage, the


error would appear or be discoverable from a review of the
records of the preliminary investigation. Of course, that fact
In case the inquest proceedings yield no probable cause, the may be perceived by the trial judge himself but, again,
private complainant may pursue the case through the regular realistically it will be the prosecutor who can initially
course of a preliminary investigation.
determine the same. That is why such error need not be
manifest or evident, nor is it required that such nuances as
ONCE A COMPLAINT OR INFORMATION IS FILED IN offenses includible in the offense charged be taken into
COURT, the rules yet provide the accused with another
account. It necessarily follows, therefore, that the
opportunity to ask for a preliminary investigation within five prosecutor can and should institute remedial
days from the time he learns of its filing. The Rules of Court measures[.]42 (emphasis and underscoring supplied)
and the New Rules on Inquest are silent, however, on
whether the private complainant could invoke, as respondent The prosecution of crimes appertains to the executive
heirs of the victim did in the present case, a similar right to
department of the government whose principal power and
ask for a reinvestigation.
responsibility is to see that our laws are faithfully executed.
A necessary component of this power to execute our laws is
The Court holds that the private complainant can move for
the right to prosecute their violators. The right to prosecute
reinvestigation, subject to and in light of the ensuing
vests the prosecutor with a wide range of discretion the
disquisition.
discretion of what and whom to charge, the exercise of
which depends on a smorgasbord of factors which are best
All criminal actions commenced by a complaint or
appreciated by prosecutors.43
information shall be prosecuted under the direction and
control of the public prosecutor.37 The private complainant in The prosecutions discretion is not boundless or infinite,
a criminal case is merely a witness and not a party to the
however.44 The standing principle is that once an information
case and cannot, by himself, ask for the reinvestigation of
is filed in court, any remedial measure such as a
the case after the information had been filed in court, the
reinvestigation must be addressed to the sound discretion of
proper party for that being the public prosecutor who has the the court. Interestingly, petitioner supports this view.45
control of the prosecution of the case.38 Thus, in cases where Indeed, the Court ruled in one case that:
the private complainant is allowed to intervene by counsel in
the criminal action,39 and is granted the authority to
The rule is now well settled that once a complaint or
prosecute,40 the private complainant, by counsel and with the information is filed in court, any disposition of the case,
conformity of the public prosecutor, can file a motion for
whether as to its dismissal or the conviction or the acquittal
reinvestigation.
of the accused, rests in the sound discretion of the court.
Although the prosecutor retains the direction and control of
In fact, the DOJ instructs that before the arraignment of the
the prosecution of criminal cases even when the case is
accused, trial prosecutors must "examine the Information
already in court, he cannot impose his opinion upon the
vis--vis the resolution of the investigating prosecutor in
tribunal. For while it is true that the prosecutor has the quasiorder to make the necessary corrections or revisions and to
judicial discretion to determine whether or not a criminal
ensure that the information is sufficient in form and
case should be filed in court, once the case had already been
substance."41

brought therein any disposition the prosecutor may deem


proper thereafter
should be addressed to the court for its consideration and
approval. The only qualification is that the action of the
court must not impair the substantial rights of the accused or
the right of the People to due process of law.
xxxx
In such an instance, before a re-investigation of the case may
be conducted by the public prosecutor, the permission or
consent of the court must be secured. If after such reinvestigation the prosecution finds a cogent basis to
withdraw the information or otherwise cause the dismissal of
the case, such proposed course of action may be taken but
shall likewise be addressed to the sound discretion of the
court.46 (underscoring supplied)
While Abugotal v. Judge Tiro47 held that to ferret out the
truth, a trial is to be preferred to a reinvestigation, the Court
therein recognized that a trial court may, where the interest
of justice so requires, grant a motion for reinvestigation of a
criminal case pending before it.
Once the trial court grants the prosecutions motion for
reinvestigation, the former is deemed to have deferred to the
authority of the prosecutorial arm of the Government.
Having brought the case back to the drawing board, the
prosecution is thus equipped with discretion wide and far
reaching regarding the disposition thereof,48 subject to the
trial courts approval of the resulting proposed course of
action.
Since a reinvestigation may entail a modification of the
criminal information as what happened in the present case,
the Courts holding is bolstered by the rule on amendment of
an information under Section 14, Rule 110 of the Rules of
Court:

A complaint or information may be amended, in form or


in substance, without leave of court, at any time before
the accused enters his plea. After the plea and during the
trial, a formal amendment may only be made with leave of
court and when it can be done without causing prejudice to
the rights of the accused.

Considering the general rule that an information may be


amended even in substance and even without leave of court
at any time before entry of plea, does it mean that the
conduct of a reinvestigation at that stage is a mere
superfluity?
It is not.

However, any amendment before plea, which downgrades


the nature of the offense charged in or excludes any accused
from the complaint or information, can be made only upon
motion by the prosecutor, with notice to the offended party
and with leave of court. The court shall state its reasons in
resolving the motion and copies of its order shall be
furnished all parties, especially the offended party.

jurisdiction of the court. All other matters are merely of


form. The following have been held to be mere formal
amendments: (1) new allegations which relate only to the
range of the penalty that the court might impose in the event
of conviction; (2) an amendment which does not charge
another offense different or distinct from that charged in the
original one; (3) additional allegations which do not alter the
prosecutions theory of the case so as to cause surprise to the
accused and affect the form of defense he has or will
assume; (4) an amendment which does not adversely affect
any substantial right of the accused; and (5) an amendment
that merely adds specifications to eliminate vagueness in the
information and not to introduce new and material facts, and
merely states with additional precision something which is
already contained in the original information and which adds
nothing essential for conviction for the crime charged.

Any remedial measure springing from the reinvestigation


be it a complete disposition or an intermediate modification 53
of the charge is eventually addressed to the sound
discretion of the trial court, which must make an
independent evaluation or assessment of the merits of the
case. Since the trial court would ultimately make the
determination on the proposed course of action, it is for the
If it appears at any time before judgment that a mistake has
prosecution to consider whether a reinvestigation is
been made in charging the proper offense, the court shall
necessary to adduce and review the evidence for purposes of
dismiss the original complaint or information upon the filing buttressing the appropriate motion to be filed in court.
The test as to whether a defendant is prejudiced by the
of a new one charging the proper offense in accordance with
amendment is whether a defense under the information as it
More importantly, reinvestigation is required in cases
section 11, Rule 119, provided the accused would not be
originally stood would be available after the amendment is
involving a substantial amendment of the information. Due
placed in double jeopardy. The court may require the
made, and whether any evidence defendant might have
process of law demands that no substantial amendment of an would be equally applicable to the information in the one
witnesses to give bail for their appearance at the trial.
information may be admitted without conducting another or form as in the other. An amendment to an information which
(emphasis supplied)
a new preliminary investigation. In Matalam v. The 2nd
does not change the nature of the crime alleged therein does
54
In fine, before the accused enters a plea, a formal or
Division of the Sandiganbayan, the Court ruled that a
not affect the essence of the offense or cause surprise or
substantial amendment of the complaint or information may substantial amendment in an information entitles an accused deprive the accused of an opportunity to meet the new
be made without leave of court.49 After the entry of a plea,
to another preliminary investigation, unless the amended
averment had each been held to be one of form and not of
only a formal amendment may be made but with leave of
information contains a charge related to or is included in the substance.55 (emphasis and underscoring supplied)
court and only if it does not prejudice the rights of the
original Information.
accused. After arraignment, a substantial amendment is
Matalam adds that the mere fact that the two charges are
50
The
question
to
be
resolved
is
whether
the
amendment
of
the
proscribed except if the same is beneficial to the accused.
related does not necessarily or automatically deprive the
Information from homicide to murder is considered a
accused of his right to another preliminary investigation.
It must be clarified though that not all defects in an
substantial amendment, which would make it not just a right Notatu dignum is the fact that both the original Information
information are curable by amendment prior to entry of plea. but a duty of the prosecution to ask for a preliminary
and the amended Information in Matalam were similarly
An information which is void ab initio cannot be amended to investigation.
charging the accused with violation of Section 3(e) of the
51
obviate a ground for quashal. An amendment which
Anti-Graft and Corrupt Practices Act.
The Court answers in the affirmative.
operates to vest jurisdiction upon the trial court is likewise
impermissible.52
In one case,56 it was squarely held that the amendment of the
A substantial amendment consists of the recital of facts
Information from homicide to murder is "one of substance
constituting the offense charged and determinative of the with very serious consequences."57 The amendment involved

in the present case consists of additional averments of the


circumstances of treachery, evident premeditation, and
cruelty, which qualify the offense charged from homicide to
murder. It being a new and material element of the offense,
petitioner should be given the chance to adduce evidence on
the matter. Not being merely clarificatory, the amendment
essentially varies the prosecutions original theory of the
case and certainly affects not just the form but the weight of
defense to be mustered by petitioner.

Petitioner did not, however, make much of the opportunity to


present countervailing evidence on the proposed amended
charge. Despite notice of hearing, petitioner opted to merely
observe the proceedings and declined to actively participate,
even with extreme caution, in the reinvestigation. Mercado
v. Court of Appeals states that the rules do not even require,
as a condition sine qua non to the validity of a preliminary
investigation, the presence of the respondent as long as
efforts to reach him were made and an opportunity to
controvert the complainants evidence was accorded him. 62

injudicious performance of functions. For ones prompt


dispatch may be anothers undue haste. The orderly
administration of justice remains as the paramount and
constant consideration, with particular regard of the
circumstances peculiar to each case.

The presumption of regularity includes the public officers


official actuations in all phases of work. Consistent with
such presumption, it was incumbent upon petitioners to
present contradictory evidence other than a mere tallying of
The Court distinguishes the factual milieus in Buhat v. CA58
days or numerical calculation. This, petitioners failed to
59
and Pacoy v. Cajigal, wherein the amendment of the
In his second assignment of error, petitioner basically assails discharge. The swift completion of the Investigating Panels
caption of the Information from homicide to murder was not the hurried issuance of the last two assailed RTC Orders
initial task cannot be relegated as shoddy or shady without
considered substantial because there was no real change in
despite the pendency before the appellate court of the
discounting the presumably regular performance of not just
the recital of facts constituting the offense charged as alleged petition for certiorari challenging the first two trial court
one but five state prosecutors.68
in the body of the Information, as the allegations of
Orders allowing a reinvestigation.
There is no ground for petitioners protestations against the
qualifying circumstances were already clearly embedded in
DOJ Secretarys sudden designation of Senior State
the original Information. Buhat pointed out that the original The Rules categorically state that the petition shall not
Information for homicide already alleged the use of superior interrupt the course of the principal case unless a temporary Prosecutor Emmanuel Velasco as Acting City Prosecutor of
retraining order or a writ of preliminary injunction has been Makati City for the present case69 and the latters conformity
strength, while Pacoy states that the averments in the
issued.63 The appellate court, by Resolution of February 15, to the motion for reinvestigation.
amended Information for murder are exactly the same as
2007,64 denied petitioners application for a temporary
those already alleged in the original Information for
In granting the reinvestigation, Judge Alameda cannot
restraining order and writ of preliminary injunction.
homicide. None of these peculiar circumstances obtains in
choose the public prosecutor who will conduct the
Supplementary efforts to seek injunctive reliefs proved
the present case.
65
reinvestigation or preliminary investigation.70 There is a
futile. The appellate court thus did not err in finding no
Considering that another or a new preliminary investigation grave abuse of discretion on the part of the trial court when it hierarchy of officials in the prosecutory arm of the executive
branch headed by the Secretary of Justice71 who is vested
is required, the fact that what was conducted in the present
proceeded with the case and eventually arraigned the
case was a reinvestigation does not invalidate the substantial accused on March 21, 2007, there being no injunction order with the prerogative to appoint a special prosecutor or
designate an acting prosecutor to handle a particular case,
amendment of the Information. There is no substantial
from the appellate court. Moreover, petitioner opted to
which broad power of control has been recognized by
distinction between a preliminary investigation and a
forego appealing to the DOJ Secretary, a post-inquest
jurisprudence.72
reinvestigation since both are conducted in the same manner remedy that was available after the reinvestigation and
66
and for the same objective of determining whether there
which could have suspended the arraignment. 1avvphi1
As for the trial courts ignoring the DOJ Secretarys
exists sufficient ground to engender a well-founded belief
uncontested statements to the media which aired his opinion
Regarding
petitioners
protestations
of
haste,
suffice
to
state
that a crime has been committed and the respondent is
60
that if the assailant merely intended to maim and not to kill
probably guilty thereof and should be held for trial. What is that the pace in resolving incidents of the case is not per se
the victim, one bullet would have sufficed the DOJ
an indication of bias. In Santos-Concio v. Department of
essential is that petitioner was placed on guard to defend
Secretary reportedly uttered that "the filing of the case of
Justice,67 the Court held:
himself from the charge of murder61 after the claimed
homicide against ano against Leviste lintek naman eh I told
circumstances were made known to him as early as the first
Speed
in
the
conduct
of
proceedings
by
a
judicial
or
quasiyou to watch over that case there should be a report about
motion.
judicial officer cannot per se be instantly attributed to an
the ballistics, about the paraffin, etc., then thats not a

complete investigation, thats why you should use that as a


ground" no abuse of discretion, much less a grave one,
can be imputed to it.

It is a function that properly pertains to the public prosecutor


who is given a broad discretion to determine whether
probable cause exists and to charge those whom he believes
to have committed the crime as defined by law and thus
The statements of the DOJ Secretary do not evince a
should be held for trial. Otherwise stated, such official has
"determination to file the Information even in the absence of the quasi-judicial authority to determine whether or not a
probable cause."73 On the contrary, the remarks merely
criminal case must be filed in court. Whether that function
underscored the importance of securing basic investigative
has been correctly discharged by the public prosecutor, i.e.,
reports to support a finding of probable cause. The original
whether he has made a correct ascertainment of the existence
Resolution even recognized that probable cause for the crime of probable cause in a case, is a matter that the trial court
of murder cannot be determined based on the evidence
itself does not and may not be compelled to pass upon.77
obtained "[u]nless and until a more thorough investigation is
conducted and eyewitness/es [is/]are presented in
The judicial determination of probable cause is one made by
evidence[.]"74
the judge to ascertain whether a warrant of arrest should be
issued against the accused. The judge must satisfy himself
The trial court concluded that "the wound sustained by the
that based on the evidence submitted, there is necessity for
victim at the back of his head, the absence of paraffin test
placing the accused under custody in order not to frustrate
and ballistic examination, and the handling of physical
the ends of justice. If the judge finds no probable cause, the
evidence,"75 as rationalized by the prosecution in its motion, judge cannot be forced to issue the arrest warrant. 78
are sufficient circumstances that require further inquiry.
Paragraph (a), Section 5,79 Rule 112 of the Rules of Court
outlines the procedure to be followed by the RTC.
That the evidence of guilt was not strong as subsequently
assessed in the bail hearings does not affect the prior
To move the court to conduct a judicial determination of
determination of probable cause because, as the appellate
probable cause is a mere superfluity, for with or without such
court correctly stated, the standard of strong evidence of
motion, the judge is duty-bound to personally evaluate the
guilt which is sufficient to deny bail to an accused is
resolution of the public prosecutor and the supporting
markedly higher than the standard of judicial probable cause evidence. In fact, the task of the presiding judge when the
which is sufficient to initiate a criminal case. 76
Information is filed with the court is first and foremost to
determine the existence or non-existence of probable cause
In his third assignment of error, petitioner faults the trial
for the arrest of the accused.80
court for not conducting, at the very least, a hearing for
judicial determination of probable cause, considering the
What the Constitution underscores is the exclusive and
lack of substantial or material new evidence adduced during personal responsibility of the issuing judge to satisfy himself
the reinvestigation.
of the existence of probable cause. But the judge is not
required to personally examine the complainant and his
Petitioners argument is specious.
witnesses. Following established doctrine and procedure, he
shall (1) personally evaluate the report and the supporting
There are two kinds of determination of probable cause:
documents submitted by the prosecutor regarding the
executive and judicial. The executive determination of
existence of probable cause, and on the basis thereof, he may
probable cause is one made during preliminary investigation. already make a personal determination of the existence of

probable cause; and (2) if he is not satisfied that probable


cause exists, he may disregard the prosecutors report and
require the submission of supporting affidavits of witnesses
to aid him in arriving at a conclusion as to the existence of
probable cause.81 (emphasis and underscoring supplied)
The rules do not require cases to be set for hearing to
determine probable cause for the issuance of a warrant of
arrest of the accused before any warrant may be issued. 82
Petitioner thus cannot, as a matter of right, insist on a
hearing for judicial determination of probable cause.
Certainly, petitioner "cannot determine beforehand how
cursory or exhaustive the [judge's] examination of the
records should be [since t]he extent of the judges
examination depends on the exercise of his sound discretion
as the circumstances of the case require." 83 In one case, the
Court emphatically stated:
The periods provided in the Revised Rules of Criminal
Procedure are mandatory, and as such, the judge must
determine the presence or absence of probable cause within
such periods. The Sandiganbayans determination of
probable cause is made ex parte and is summary in nature,
not adversarial. The Judge should not be stymied and
distracted from his determination of probable cause by
needless motions for determination of probable cause
filed by the accused.84 (emphasis and underscoring
supplied)
Petitioner proceeds to discuss at length evidentiary matters,
arguing that no circumstances exist that would qualify the
crime from homicide to murder.
The allegation of lack of substantial or material new
evidence deserves no credence, because new pieces of
evidence are not prerequisites for a valid conduct of
reinvestigation. It is not material that no new matter or
evidence was presented during the reinvestigation of the
case. It should be stressed that reinvestigation, as the word
itself implies, is merely a repeat investigation of the case.

New matters or evidence are not prerequisites for a


reinvestigation, which is simply a chance for the prosecutor
to review and re-evaluate its findings and the evidence
already submitted.85
Moreover, under Rule 45 of the Rules of Court, only
questions of law may be raised in, and be subject of, a
petition for review on certiorari since this Court is not a trier
of facts. The Court cannot thus review the evidence adduced
by the parties on the issue of the absence or presence of
probable cause, as there exists no exceptional circumstances
to warrant a factual review.86
In a petition for certiorari, like that filed by petitioner before
the appellate court, the jurisdiction of the court is narrow in
scope. It is limited to resolving only errors of
jurisdiction.1avvphi1 It is not to stray at will and resolve
questions and issues beyond its competence, such as an error
of judgment.87 The courts duty in the pertinent case is
confined to determining whether the executive and judicial
determination of probable cause was done without or in
excess of jurisdiction or with grave abuse of discretion.
Although it is possible that error may be committed in the
discharge of lawful functions, this does not render the act
amenable to correction and annulment by the extraordinary
remedy of certiorari, absent any showing of grave abuse of
discretion amounting to excess of jurisdiction.88

WONG FONG FUI, petitioners,


vs.
THE COURT OF APPEALS, THE HON. MAXIMIANO
ASUNCION, in his capacity as the Presiding Judge of the
Regional Trial Court, Quezon City, Branch 104, HON.
APOLINARIO G. EXEVEA, HON. HENRICK F.
GINGOYON, and HON. PHILIP A. AGUINALDO, in
their capacities as Members of the Department of Justice
"349" Committee, and the CITY PROSECUTOR OF
QUEZON CITY, respondents.
J. ROBERT DELGADO, petitioner-Intervenor.

DAVIDE, JR., J.:p

I
Respondent Judge acted with grave abuse of
discretion when he ordered the arrest of the
petitioners without examining the record of
the preliminary investigation and in
determining for himself on the basis thereof
the existence of probable cause.
II
The Department of Justice "349" Committee
acted with grave abuse of discretion when it
refused to review the City Prosecutor's Joint
Resolution and dismissed petitioner's appeal
therefrom.

We are urged in this petition to set aside (a) the decision of


III
the Court of Appeals of 28 September 1993 in CA-G.R. SP
The Court of Appeals acted with grave
No. 31226, 1 which dismissed the petition therein on the
abuse of discretion when it upheld the
ground that it has been "mooted with the release by the
subject order directing the issuance of the
Department of Justice of its decision . . . dismissing
warrants of arrest without assessing for itself
petitioners' petition for review"; (b) the resolution of the said
2
whether based on such records there is
court of 9 February 1994 denying the petitioners' motion to
3
probable cause against petitioners.
reconsider the decision; (c) the order of 17 May 1993 of
respondent Judge Maximiano C. Asuncion of Branch 104 of
IV
the Regional Trial Court (RTC) of Quezon City in Criminal
Case No. Q-93-43198 denying petitioners' motion to suspend
The facts on record do not establish prima
proceedings and to hold in abeyance the issuance of the
WHEREFORE, the petition is DENIED. The assailed
facie probable cause and Criminal Case No.
Decision and Resolution of the Court of Appeals in CA-G.R. warrants of arrest and the public prosecutor's motion to defer
Q-93-43198 should have been dismissed. 5
arraignment; and (d) the resolutions of 23 July 1993 and 3
SP No. 97761 are AFFIRMED.
February 1994 4 of the Department of Justice (DOJ)
The antecedents of this petition are not disputed.
SO ORDERED.
dismissing petitioners' petition for the review of the Joint
Resolution of the Assistant City Prosecutor of Quezon City
Several thousand holders 6 of "349" Pepsi crowns in
G.R. No. 113930 March 5, 1996
and denying the motion to reconsider the dismissal,
connection with the Pepsi Cola Products Phils., Inc.'s
respectively.
(PEPSI's) Number Fever Promotion 7 filed with the Office of
PAUL G. ROBERTS, JR., RODOLFO C. SALAZAR,
LUIS LORENZO, SR., LUIS LORENZO, JR.,
The petitioners rely on the following grounds for the grant of the City Prosecutor of Quezon City complaints against the
petitioner's in their respective capacities as Presidents or
AMAURY R. GUTIERREZ, BAYANI N. FABIC, JOSE the reliefs prayed for in this petition:
Chief Executive Officers, Chairman of the Board, ViceYULO, JR., ESTEBAN B. PALANNUAYAN, and

Chairman of the Board, and Directors of PEPSI, and also


against other officials of PEPSI. The complaints respectively
accuse the petitioners and the other PEPSI officials of the
following crimes: (a) estafa; (b) violation of R.A. No. 7394,
otherwise known as the Consumer Act of the Philippines; (c)
violation of E.O. No. 913; 8 and (d) violation of Act No.
2333, entitled "An Act Relative to Untrue, Deceptive and
Misleading Advertisements," as amended by Act No. 3740. 9
After appropriate proceedings, the investigating prosecutor,
Ramon M. Gerona, released on 23 March 1993 a Joint
Resolution 10 where he recommended the filing of an
information against the petitioners and others for the
violation of Article 318 of the Revised Penal Code and the
dismissal of the complaints for the violation of Article 315,
2(d) of the Revised Penal Code; R.A. No. 7394; Act No.
2333, as amended by Act No. 3740; and E.O. No. 913. The
dispositive portion thereof reads as follows:
In view of all the foregoing, it is
recommended that:
1. The attached information be filed against
respondents Paul G. Roberts, Jr., Rodolfo C.
Salazar, Rosemarie R. Vera, Luis F. Lorenzo,
Sr., Luis P. Lorenzo, Jr., J. Roberto Delgado,
Amaury R. Gutierrez, Bayani N. Fabic, Jose
Yulo, Jr., Esteban B. Pacannuayan, Jr., Wong
Fong Fui, Quintin J. Gomez, Jr. and Chito V.
Gutierrez for estafa under Article 318,
Revised Penal Code, while the complaint for
violation of Article 315, 2(d), Revised Penal
Code against same respondents Juanito R.
Ignacio, R. Sobong, R.O. Sinsuan, M.P.
Zarsadias, L.G. Dabao, Jr., R.L. Domingo,
N.N. Bacsal, Jesus M. Manalastas, Janette P.
Pio de Roda, Joaquin W. Sampaico,
Winefreda O. Madarang, Jack Gravey, Les
G. Ham, Corazon Pineda, Edward S.
Serapio, Alex O. Caballes, Sandy Sytangco,

Jorge W. Drysdale, Richard Blossom, Pablo


de Borja, Edmundo L. Tan, Joseph T. Cohen,
Delfin Dator, Zosimo B. San Juan, Joaquin
Franco, Primitivo S. Javier, Jr., Luisito
Guevarra, Asif H. Adil, Eugenio
Muniosguren, James Ditkoff and Timothy
Lane be dismissed;
2. The complaints against all respondents for
violation of R.A. 7394 otherwise known as
the Consumer Act of the Philippines and
violation of Act 2333 as amended by Act
3740 and E.O. 913 be also dismissed for
insufficiency of evidence, and
3. I.S. Nos. 92-7833; 92-8710 and 92-P1065 involving Crowns Nos. 173; 401; and
117, 425, 703 and 373, respectively, alleged
to be likewise winning ones be further
investigated to afford respondents a chance
to submit their counter-evidence. 11

GUTIERREZ, BAYANI N. FABIC, JOSE


YULO, JR., ESTEBAN B.
PACANNUAYAN, JR. and WONG FONG
FUI, of the crime of ESTAFA, committed as
follows:
That in the month of February, 1992, in
Quezon City, Philippines and for sometime
prior and subsequent thereto, the abovenamed
accused
Paul G. Roberts, Jr. ) being then the
Presidents
Rodolfo G. Salazar ) and Executive Officers
Luis F. Lorenzo, Sr. ) being then the
Chairman
of the Board of Directors

On 6 April 1993, City Prosecutor Candido V. Rivera


approved the recommendation with the modification that
Rosemarie Vera, Quintin Gomez, Jr., and Chito Gonzales be
excluded from the charge on the ground of insufficiency of
evidence. 12

Luis P. Lorenzo, Jr. ) being then the Vice

The information for estafa attached to the Joint Resolution


was approved (on 7 April 1993) by Ismael P. Casabar, Chief
of the Prosecution Division, upon authority of the City
Prosecutor of Quezon City, and was filed with the RTC of
Quezon City on 12 April 1993. It was docketed as Criminal
Case No. Q-93-43198. 13 The information reads as follows:

Amaury R. Gutierrez ) being then Members


of

The undersigned 1st Assistant City


Prosecutor accuses PAUL G. ROBERTS,
JR. RODOLFO C. SALAZAR, LUIS F.
LORENZO, SR., LUIS P. LORENZO, JR.,
J. ROBERTO DELGADO, AMAURY R.

Chairman of the Board


J. Roberto Delgado )

Bayani N. Fabic ) the Board


Jose Yulo, Jr. )
Esteban B. Pacannuayan, )
Jr. and
Wong Fong Fui )

OF THE PEPSI COLA PRODUCTS


PHILIPPINES, INC., CONSPIRING with
one another, with intent of gain, by means of
deceit, fraudulent acts or false pretenses,
executed prior to or simultaneously with the
commission of the fraud, did then and there
willfully, unlawfully and feloniously defraud
the private complainants whose names with
their prizes claimed appear in the attached
lists marked as Annexes "A" to "A-46"; "B"
to "-33"; "C" to "C-281"; "D" to "D-238";
"E" to "E-30" and "F" to "F-244" in the
following manner: on the date and in the
place aforementioned, said accused pursuant
to their conspiracy, launched the Pepsi Cola
Products Philippines, Inc. "Number Fever
Promotion" from February 17 to May 8,
1992 later extended to May 11-June 12,
1992 and announced and advertised in the
media that "all holders of crowns and/or
caps of Pepsi, Mirinda, Mountain Dew and
Seven-up bearing the winning 3-digit
number will win the full amount of the prize
printed on the crowns/caps which are
marked with a seven-digit security code as a
measure against tampering or faking of
crowns and each and every number has its
own unique matching security code",
enticing the public to buy Pepsi softdrinks
with aforestated alluring and attractive
advertisements to become millionaires, and
by virtue of such representations made by
the accused, the said complainants bought
Pepsi softdrinks, but, the said accused after
their TV announcement on May 25, 1992
that the winning number for the next day
was "349", in violation of their aforecited
mechanics, refused as they still refuse to
redeem/pay the said Pepsi crowns and/or

caps presented to them by the complainants,


who, among others, were able to buy Pepsi
softdrinks with crowns/caps bearing number
"349" with security codes L-2560-FQ and L3560-FQ, despite repeated demands made
by the complainants, to their damage and
prejudice to the extent of the amount of the
prizes respectively due them from their
winning "349" crowns/caps, together with
such amounts they spent in going to and
from the Office of Pepsi to claim their prizes
and such other amounts used in buying
Pepsi softdrinks which the complainants
normally would not have done were it not
for the false, fraudulent and deceitful posters
of Pepsi Cola Products Philippines, Inc.
CONTRARY TO LAW.
On 14 April 1993, the petitioners filed with the Office of the
City Prosecutor a motion for the reconsideration of the Joint
Resolution 14 alleging therein that (a) there was neither fraud
in the Number Fever Promotion nor deviation from or
modification of the promotional rules approved by the
Department of Trade and Industry (DTI), for from the start
of the promotion, it had always been clearly explained to the
public that for one to be entitled to the cash prize his crown
must bear both the winning number and the correct security
code as they appear in the DTI list; (b) the complainants
failed to allege, much less prove with prima facie evidence,
the specific overt criminal acts or omissions purportedly
committed by each of the petitioners; (c) the compromise
agreement entered into by PEPSI is not an admission of
guilt; and (d) the evidence establishes that the promo was
carried out with utmost good faith and without malicious
intent.
On 15 April 1993, the petitioners filed with the DOJ a
Petition for Review 15 wherein, for the same grounds adduced
in the aforementioned motion for reconsideration, they

prayed that the Joint Resolution be reversed and the


complaints dismissed. They further stated that the approval
of the Joint Resolution by the City Prosecutor was not the
result of a careful scrutiny and independent evaluation of the
relevant facts and the applicable law but of the grave threats,
intimidation, and actual violence which the complainants had
inflicted on him and his assistant prosecutors.
On that same date, the petitioners filed in Criminal Case No.
Q-93-43198 Motions to Suspend Proceedings and to Hold in
Abeyance Issuance of Warrants of Arrest on the ground that
they had filed the aforesaid Petition for Review. 16
On 21 April 1993, acting on the Petition for Review, Chief
State Prosecutor Zenon L. de Guia issued a 1st Indorsement,
1
7 directing the City Prosecutor of Quezon City to inform the
DOJ whether the petitioners have already been arraigned,
and if not, to move in court for the deferment of further
proceedings in the case and to elevate to the DOJ the entire
records of the case, for the case is being treated as an
exception pursuant to Section 4 of Department Circular No.
7 dated 25 January 1990.
On 22 April 1993, Criminal Case No. Q-93-41398 was raffle
to Branch 104 of the RTC of Quezon City. 18
In the morning of 27 April 1993, private prosecutor Julio
Contreras filed an Ex-Parte Motion for Issuance of Warrants
of Arrest. 19
In the afternoon of that same day, petitioner Paul Roberts, Jr.,
filed a Supplemental Urgent Motion to Hold in Abeyance
Issuance of Warrant of Arrest and to Suspend Proceedings. 20
He stressed that the DOJ had taken cognizance of the
Petition for Review by directing the City Prosecutor to
elevate the records of I.S. No. P-4401 and its related cases
and asserted that the petition for review was an essential part
of the petitioners' right to a preliminary investigation.

The next day, respondent Judge Asuncion, Presiding Judge


of Branch 104 of the RTC of Quezon City, issued an order
advising the parties that his court would "be guided by the
doctrine laid down by the Supreme Court in the case of
Crespo vs. Mogul, 151 SCRA 462 and not by the resolution
of the Department of Justice on the petition for review
undertaken by the accused." 21
On 30 April 1993, Assistant City Prosecutor Tirso M. Gavero
filed with the trial court a Motion to Defer Arraignment
wherein he also prayed that "further proceedings be held in
abeyance pending final disposition by the Department of
Justice." 22
23

On 4 May 1993, Gavero filed an Amended Information,


accompanied by a corresponding motion 24 to admit it. The
amendments merely consist in the statement that the
complainants therein were only "among others" who were
defrauded by the accused and that the damage or prejudice
caused amounted "to several billions of pesos, representing
the amounts due them from their winning '349'
crowns/caps." The trial court admitted the amended
information on the same date. 25
Later, the attorneys for the different private complainants
filed, respectively, an Opposition to Motion to Defer
Arraignment, 26 and Objection and Opposition to Motion to
Suspend Proceedings and to Hold in Abeyance the Issuance
of Warrants of Arrest. 27
On 14 May 1993, the petitioners filed a Memorandum in
Support of their Motion to Suspend Proceedings and to Hold
in Abeyance the Issuance of the Warrants of Arrest. 28
On 17 May 1993, respondent Judge Asuncion issued the
challenged order (1) denying the petitioners' Motion to
Suspend Proceedings and to Hold in Abeyance Issuance of
Warrants of Arrest and the public prosecutor's Motion to
Defer Arraignment and (2) directing the issuance of the
warrants of arrest "after June 1993" and setting the

arraignment on 28 June 1993. 29 Pertinent portions of the


order read as follows:
In the Motion filed by the accused, it is
alleged that on April 15, 1993, they filed a
petition for review seeking the reversal of
the resolution of City Prosecutor of Quezon
City approving the filing of the case against
the accused, claiming that:
1. The resolution
constituting [sic] force and
duress;
2. There was no fraud or
deceit therefore there can be
no estafa;
3. No criminal overt acts by
respondents were proved;
4. Pepsi nor the accused
herein made no admission
of guilt before the
Department of Trade and
Industry;
5. The evidence presented
clearly showed no malicious
intent on the part of the
accused.
Trial Prosecutor Tirso M. Gavero in his
Motion to Defer Arraignment averred that
there is a pending petition for review with
the Department of Justice filed by the
accused and the Office of the City
Prosecutor was directed, among other
things, to cause for the deferment of further

proceedings pending final disposition of said


Petition by the Department of Justice.
The motions filed by the accused and the
Trial Prosecutor are hereby DENIED.
This case is already pending in this Court for
trial. To follow whatever opinion the
Secretary of Justice may have on the matter
would undermine the independence and
integrity of this Court. This Court is still
capable of administering justice.
The Supreme Court in the case of Crespo vs.
Mogul (SCRA 151, pp. 471-472) stated as
follows:
In order therefor to avoid
such a situation whereby the
opinion of the Secretary of
Justice who reviewed the
action of the fiscal may be
disregarded by the trial
court, the Secretary of
Justice should, as far as
practicable, refrain from
entertaining a petition for
review or appeal from the
action of the fiscal, when
the complaint or
information has already
been filed in Court. The
matter should be left
entirely for the
determination of the Court.
WHEREFORE, let warrant of arrest be
issued after June 21, 1993, and arraignment
be set on June 28, 1993, at 9:30 in the
morning.

On 7 June 1993, the petitioners filed with the Court of


Appeals a special civil action for certiorari and prohibition
with application for a temporary restraining order, 30 which
was docketed as CA-G.R. SP No. 31226. They contended
therein that respondent Judge Asuncion had acted without or
in excess of jurisdiction or with grave abuse of discretion in
issuing the aforementioned order of 17 May 1993 because

Quezon City to elevate the original records of Criminal Case


No. 4-93-43198. 33

Upon receipt of the original records of the criminal case, the


Court of Appeals found that a copy of the Joint Resolution
had in fact been forwarded to, and received by, the trial court
on 22 April 1993, which fact belied the petitioners' claim that
the respondent Judge had not the slightest basis at all for
I. RESPONDENT JUDGE FAILED TO
determining probable cause when he ordered the issuance of
EXAMINE THE RECORD OF
warrants of arrest. It ruled that the Joint Resolution "was
PRELIMINARY INVESTIGATION
sufficient in itself to have been relied upon by respondent
BEFORE ORDERING THE ARREST OF
Judge in convincing himself that probable cause indeed
PETITIONERS.
exists for the purpose of issuing the corresponding warrants
of arrest"; and that the "mere silence of the records or the
II. THERE IS NO PROBABLE CAUSE TO absence of any express declaration" in the questioned order
HOLD PETITIONERS CRIMINALLY
as to the basis of such finding does not give rise to an
LIABLE FOR ESTAFA, OTHER
adverse inference, for the respondent Judge enjoys in his
DECEITS, OR ANY OTHER OFFENSE.
favor the presumption of regularity in the performance of his
official duty. The Court of Appeals then issued a resolution 34
III. THE PROCEEDINGS BELOW
denying the application for a writ of preliminary injunction.
SHOULD HAVE BEEN SUSPENDED TO
AWAIT THE SECRETARY OF JUSTICE'S On 8 June 1993, the petitioners filed a motion to reconsider 35
RESOLUTION OF PETITIONERS'
the aforesaid resolution. The Court of Appeals required the
APPEAL, AND
respondents therein to comment on the said motion. 36
IV. THERE IS NO OTHER PLAIN,
SPEEDY AND ADEQUATE REMEDY IN
THE ORDINARY COURSE OF LAW.

On 15 June 1993, the Court of Appeals issued a temporary


restraining order to maintain the status quo. 31 In view
thereof; respondent Judge Asuncion issued an order on 28
June 1993 32 postponing indefinitely the arraignment of the
petitioners which was earlier scheduled on that date.
On 28 June 1993, the Court of Appeals heard the petitioners'
application for a writ of preliminary injunction, granted the
motion for leave to intervene filed by J. Roberto Delgado,
and directed the Branch Clerk of Court of the RTC of

On 3 August 1993, the counsel for the private complainants


filed in CA-G.R. SP No. 31226 a Manifestation 37 informing
the court that the petitioners' petition for review filed with
the DOJ was dismissed in a resolution dated 23 July 1993. A
copy 38 of the resolution was attached to the Manifestation.
On 21 September 1993, the public respondents filed in CAG.R. SP No. 31226 a motion to dismiss the petition 39 on the
ground that it has become moot and academic in view of the
dismissal by the DOJ of the petitioners' petition to review the
Joint Resolution. The dismissal by the DOJ is founded on the
following exposition:
You questioned the said order of the RTC
before the Court of Appeals and prayed for

the issuance of a writ of preliminary


injunction to restrain the Trial Judge from
issuing any warrant of arrest and from
proceeding with the arraignment of the
accused. The appellate court in a resolution
dated July 1, 1993, denied your petition.
In view of the said developments, it would
be an exercise in futility to continue
reviewing the instant cases for any further
action on the part of the Department would
depend on the sound discretion of the Trial
Court. The denial by the said court of the
motion to defer arraignment filed at our
instance was clearly an exercise of its
discretion. With the issuance of the order
dated May 17, 1993, Trial Court was in
effect sending a signal to this Department
that "the determination of the case is within
its exclusive jurisdiction and competence."
The rule is that ". . . once a complaint or
information is filed in Court, any disposition
of the case as to dismissal or the conviction
or acquittal of the accused rests in the sound
discretion of the Court. Although the fiscal
retains the direction and control of the
prosecution of criminal cases even while the
case is already in Court, he cannot impose
his opinion on the trial court. The court is
the best and sole judge on what to do with
the case before it. . . ." (Crespo vs. Mogul,
151 SCRA 462). 40
On 28 September 1993, the Court of Appeals promulgated a
decision 41 dismissing the petition because it had been
"mooted with the release by the Department of Justice of its
decision . . . dismissing petitioners' petition for review by
inerrantly upholding the criminal court's exclusive and
unsupplantable authority to control the entire course of the

case brought against petitioners, reiterating with approval the In the resolution of 24 November 1994, the Court en banc
dictum laid down in the 'Crespo' case."
accepted the referral.
The petitioners filed a motion to reconsider the DOJ's
dismissal of the petition citing therein its resolutions in other
similar cases which were favorable to the petitioners and
adverse to other "349" Pepsi crowns holders.

On 10 October 1995, after deliberating on the motion for


reconsideration and the subsequent pleadings in relation
thereto, the Court en banc granted the motion for
reconsideration; reconsidered and set aside the resolution of
19 September 1994; and reinstated the petition. It then
In its resolution of 3 February 1994, the DOJ, through its
considered the case submitted for decision, "since the parties
"349" Committee, denied the motion and stated: "The instant have exhaustively discussed the issues in their pleadings, the
petition is different from the other petitions resolved by this original records of Criminal Case No. Q-93-43198 and of
Department in similar cases from
CA-G.R. SP No. 31226 had been elevated to this Court, and
the provinces. In the latter petitions, the complaints against
both the petitioners and the Office of the Solicitor General
herein respondents [sic] 42 were dismissed inasmuch as the
pray, in effect, that this Court resolve the issue of probable
informations have not yet been filed or even if already filed cause on the basis thereof."
in court, the proceedings have been suspended by the courts
to await the outcome of the appeal pending with this
The pleadings of the parties suggest for this Court's
43
Department."
resolution the following key issues:
The petitioners likewise filed a motion to reconsider 44 the
aforesaid Court of Appeals' decision, which the said court
denied in its resolution 45 of 9 February 1994. Hence, the
instant petition.
The First Division of this Court denied due course to this
petition in its resolution of 19 September 1994. 46
On 7 October 1994, the petitioners filed a motion for the
reconsideration 47 of the aforesaid resolution. Acting
thereon, the First Division required the respondents to
comment thereon.
Later, the petitioners filed a supplemental motion for
reconsideration 48 and a motion to refer this case to the Court
en banc. 49 In its resolution of 14 November 1994, 50 the First
Division granted the latter motion and required the
respondents to comment on the supplemental motion for
reconsideration.

application for a writ of preliminary


injunction and (b) of public respondent
Asuncion's denial of the abovementioned
motions.
4. Whether public respondent Court of
Appeals committed grave abuse of
discretion (a) in denying the motion for a
writ of preliminary injunction solely on the
ground that public respondent Asuncion had
already before him the Joint Resolution of
the investigating prosecutor when he
ordered the issuance of the warrants of
arrest, and (b) in ultimately dismissing the
petition on the ground of mootness since the
DOJ had dismissed the petition for review.
5. Whether this Court may determine in this
proceedings the existence of probable cause
either for the issuance of warrants of arrest
against the petitioners or for their
prosecution for the crime of estafa.

1. Whether public respondent Judge


Asuncion committed grave abuse of
discretion in denying, on the basis of Crespo
vs. Mogul, the motions to suspend
proceedings and hold in abeyance the
We resolve the first four issues in the affirmative and the
issuance of warrants of arrest and to defer
fifth, in the negative.
arraignment until after the petition for
I.
review filed with the DOJ shall have been
resolved.
There is nothing in Crespo vs. Mogul 51 which bars the DOJ
from taking cognizance of an appeal, by way 'of a petition
2. Whether public respondent Judge
for review, by an accused in a criminal case from an
Asuncion committed grave abuse of
unfavorable ruling of the investigating prosecutor. It merely
discretion in ordering the issuance of
advised the DOJ to, "as far as practicable, refrain from
warrants of arrest without examining the
entertaining a petition for review or appeal from the action of
records of the preliminary investigation.
the fiscal, when the complaint or information has already
3. Whether the DOJ, through its "349"
been filed in Court." More specifically, it stated:
Committee, gravely abused its discretion in
In order therefore to avoid such a situation
dismissing the petition for review on the
whereby the opinion of the Secretary of
following bases: (a) the resolution of public
Justice who reviewed the action of the fiscal
respondent Court of Appeals denying the

may be disregarded by the trial court, the


Secretary of Justice should, as far as
practicable, refrain from entertaining a
petition for review or appeal from the action
of the fiscal, when the complaint or
information has already been filed in Court.
The matter should, be left entirely for the
determination of the Court. 52
In Marcelo vs. Court of Appeals, 53 this Court explicitly
declared:
Nothing in the said ruling forecloses the
power or authority of the Secretary of
Justice to review resolutions of his
subordinates in criminal cases. The
Secretary of Justice is only enjoined to
refrain as far as practicable from
entertaining a petition for review or appeal
from the action of the prosecutor once a
complaint or information is filed in court. In
any case, the grant of a motion to dismiss,
which the prosecution may file after the
Secretary of Justice reverses an appealed
resolution, is subject to the discretion of the
court.
Crespo could not have intended otherwise without doing
violence to, or repealing, the last paragraph of Section 4,
Rule 112 of the Rules of Court 54 which recognizes the
authority of the Secretary of Justice to reverse the resolution
of the provincial or city prosecutor or chief state prosecutor
upon petition by a proper party.

only resolutions dismissing a criminal complaint may be


appealed to the Secretary of Justice. Its Section 4, 55
however, provided an exception, thus allowing, upon a
showing of manifest error or grave abuse of discretion,
appeals from resolutions finding probable cause, provided
that the accused has not been arraigned.

the following pronouncement in Crespo did not yet truly


become relevant or applicable:

There is nothing in Department Order No. 223 which would


warrant a recall of the previous action of the DOJ giving due
course to the petitioners' petition for review. But whether the
DOJ would affirm or reverse the challenged Joint Resolution
is still a matter of guesswork. Accordingly, it was premature
for respondent Judge Asuncion to deny the motions to
suspend proceedings and to defer arraignment on the
following grounds:

The rule therefore in this jurisdiction is that


once a complaint or information is filed in
Court any disposition of the case as its
dismissal or the conviction or acquittal of
the accused rests in the sound discretion of
the court. Although the fiscal retains the
direction and control of the prosecution of
criminal cases even while the case is already
in court he cannot impose his opinion on the
trial court. The court is the best and sole
judge on what to do with the case before it.
The determination of the case is within its
exclusive jurisdiction and competence. A
motion to dismiss the case filed by the fiscal
should be addressed to the Court who has
the option to grant or deny the same. It does
not matter if this is done before or after the
arraignment of the accused or that the
motion was filed after a reinvestigation or
upon instructions of the Secretary of Justice
who reviewed the records of the
investigation. 57

This case is already pending in this Court for


trial. To follow whatever opinion the
Secretary of Justice may have on the matter
would undermine the independence and
integrity of this Court. This Court is still
capable of administering justice.

However, once a motion to dismiss or withdraw the


information is filed the trial judge may grant or deny
it, not out of subservience to the Secretary of Justice,
but in faithful exercise of judicial prerogative. This
Court pertinently stated so in Martinez vs. Court of
Appeals: 58

The DOJ gave due course to the petitioners' petition for


review as an exception pursuant to Section 4 of Circular No.
7.
Meanwhile, the DOJ promulgated on 30 June 1993
Department Order No. 223 56 which superseded Circular No.
7. This Order, however, retained the provisions of Section 1
of the Circular on appealable cases and Section 4 on the nonappealable cases and the exceptions thereto.

The real and ultimate test of the independence and integrity


of his court is not the filing of the aforementioned motions at
Pursuant to the said provision, the Secretary of Justice had
that stage of the proceedings but the filing of a motion to
promulgated the rules on appeals from resolutions in
dismiss or to withdraw the information on the basis of a
preliminary investigation. At the time the petitioners filed
resolution of the petition for review reversing the Joint
their petition for the review of the Joint Resolution of the
investigating prosecutor, the governing rule was Circular No. Resolution of the investigating prosecutor. Before that time,
7, dated 25 January 1990. Section 2 thereof provided that

Whether to approve or disapprove the stand


taken by the prosecution is not the exercise
of discretion required in cases like this. The
trial judge must himself be convinced that
there was indeed no sufficient evidence
against the accused, and this conclusion can
be arrived at only after an assessment of the

evidence in the possession of the


prosecution. What was imperatively
required was the trial judge's own
assessment of such evidence, it not being
sufficient for the valid and proper exercise
of judicial discretion merely to accept the
prosecution's word for its supposed
insufficiency.

cases filed with them after appropriate preliminary


investigations conducted by officers authorized to do so
other than judges of MeTCs, MTCs and MCTCs. 62

As to the first, a warrant can issue only if the judge is


satisfied after an examination in writing and under oath of
the complainant and the witnesses, in the form of searching
questions and answers, that a probable cause exists and that
there is a necessity of placing the respondent under
As aptly observed the Office of the Solicitor immediate custody in order not to frustrate the ends of
General, in failing to make an independent
justice.
finding of the merits of the case and merely
As to the second, this Court held in Soliven vs. Makasiar 63
anchoring the dismissal on the revised
that the judge is not required to personally examine the
position of the prosecution, the trial judge
complainant and the witnesses, but
relinquished the discretion he was duty
bound to exercise. In effect, it was the
[f]ollowing established doctrine and
prosecution, through the Department of
procedure, he shall: (1) personally evaluate
Justice which decided what to do and not the
the report and supporting documents
court which was reduced to a mere rubber
submitted by the fiscal regarding the
stamp in violation of the ruling in Crespo vs.
existence of probable cause and, on the basis
Mogul.
thereof; issue a warrant of arrest; or (2) if on
the basis thereof he finds no probable cause,
II.
he may disregard the fiscal's report and
Section 2, Article III of the present Constitution provides
require the submission of supporting
that no search warrant or warrant of arrest shall issue except
affidavits of witnesses to aid him in arriving
upon probable cause to be determined personally by the
at a conclusion as to the existence of
judge after examination under oath or affirmation of the
probable cause. 64
complainant and the witnesses he may produce.
Sound policy supports this procedure, "otherwise
Under existing laws, warrants of arrest may be issued (1) by
judges would be unduly laden with the preliminary
the Metropolitan Trial Courts (MeTCs) except those in the
examination and investigation of criminal
National Capital Region, Municipal Trial Courts (MTCs),
complaints instead of concentrating on hearing and
and Municipal Circuit Trial Courts (MCTCs) in cases falling
deciding cases filed before their courts." It must be
59
within their exclusive original jurisdiction; in cases
emphasized that judges must not rely solely on the
covered by the rule on summary procedure where the
report or resolution of the fiscal (now prosecutor);
accused fails to appear when required; 60 and in cases filed
they must evaluate the report and the supporting
with them which are cognizable by the Regional Trial Courts
document. In this sense, the aforementioned
(RTCs); 61 and (2) by the Metropolitan Trial Courts in the
requirement has modified paragraph 4(a) of Circular
National Capital Region (MeTCs-NCR) and the RTCs in
No. 12 issued by this Court on 30 June 1987

prescribing the Guidelines on Issuance of Warrants


of Arrest under Section 2, Article III of the 1987
Constitution, which provided in part as follows:
4. In satisfying himself of the existence of a
probable cause for the issuance of a warrant
of arrest, the judge, following established
doctrine and procedure, may either:
(a) Rely upon the fiscal's
certification of the existence
of probable cause whether
or not the case is cognizable
only by the Regional Trial
Court and on the basis
thereof, issue a warrant of
arrest. . . .
This requirement of evaluation not only of the report or
certification of the fiscal but also of the supporting
documents was further explained in People vs. Inting, 65
where this Court specified what the documents may consist
of, viz., "the affidavits, the transcripts of stenographic notes
(if any), and all other supporting documents behind the
Prosecutor's certification which are material in assisting the
Judge to make his determination" of probable cause. Thus:
We emphasize the important features of the
constitutional mandate that ". . . no search
warrant or warrant of arrest shall issue
except upon probable cause to be
determined personally by the judge . . ."
(Article III, Section 2, Constitution).
First, the determination of probable cause is
a function of the Judge. It is not for the
Provincial Fiscal or Prosecutor nor the
Election Supervisor to ascertain. Only the
Judge and the Judge alone makes this
determination.

Second, the preliminary inquiry made by a


Prosecutor does not bind the Judge. It
merely assists him to make the
determination of probable cause. The Judge
does not have to follow what the Prosecutor
presents to him. By itself, the Prosecutor's
certification of probable cause is ineffectual.
It is the report, the affidavits, the transcripts
of stenographic notes (if any), and all other
supporting documents behind the
Prosecutor's certification which are material
in assisting the Judge to make his
determination.
In adverting to a statement in People vs. Delgado 66 that the
judge may rely on the resolution of the Commission on
Elections (COMELEC) to file the information by the same
token that it may rely on the certification made by the
prosecutor who conducted the preliminary investigation in
the issuance of the warrant of arrest, this Court stressed in
Lim vs. Felix 67 that
Reliance on the COMELEC resolution or
the Prosecutor's certification presupposes
that the records of either the COMELEC or
the Prosecutor have been submitted to the
Judge and he relies on the certification or
resolution because the records of the
investigation sustain the recommendation.
The warrant issues not on the strength of the
certification standing alone but because of
the records which sustain it.
And noting that judges still suffer from the inertia of
decisions and practice under the 1935 and 1973
Constitutions, this Court found it necessary to restate
the rule "in greater detail and hopefully clearer
terms." It then proceeded to do so, thus:

We reiterate the ruling in Soliven vs.


Makasiar that the Judge does not have to
personally examine the complainant and his
witnesses. The Prosecutor can perform the
same functions as a commissioner for the
taking of the evidence. However, there
should be a report and necessary documents
supporting the Fiscal's bare certification.
All of these should be before the Judge.
The extent of the Judge's personal
examination of the report and its annexes
depends on the circumstances of each case.
We cannot determine beforehand how
cursory or exhaustive the Judge's
examination should be. The Judge has to
exercise sound discretion for, after all, the
personal determination is vested in the Judge
by the Constitution. It can be as brief as or
detailed as the circumstances of each case
require. To be sure, the Judge must go
beyond the Prosecutor's certification and
investigation report whenever necessary. He
should call for the complainant and
witnesses themselves to answer the court's
probing questions when the circumstances
of the case so require.
This Court then set aside for being null and void the
challenged order of respondent Judge Felix directing
the issuance of the warrants of arrest against
petitioners Lim, et al., solely on the basis of the
prosecutor's certification in the informations that
there existed probable cause "without having before
him any other basis for his personal determination of
the existence of a probable cause."
In Allado vs. Diokno, 68 this Court also ruled that
"before issuing a warrant of arrest, the judge must
satisfy himself that based on the evidence submitted

there is sufficient proof that a crime has been


committed and that the person to be arrested is
probably guilty thereof."
In the recent case of Webb vs. De Leon, 69 this Court rejected
the thesis of the petitioners of absence of probable cause and
sustained the investigating panel's and the respondent
Judge's findings of probable cause. After quoting extensively
from Soliven vs. Makasiar, 70 this Court explicitly pointed
out:
Clearly then, the Constitution, the Rules of
Court, and our case law repudiate the
submission of petitioners that respondent
judges should have conducted "searching
examination of witnesses" before issuing
warrants of arrest against them. They also
reject petitioners' contention that a judge
must first issue an order of arrest before
issuing a warrant of arrest. There is no law
or rule requiring the issuance of an Order of
Arrest prior to a warrant of arrest.
In the case at bar, the DOJ Panel submitted
to the trial court its 26-page report, the two
(2) sworn statements of Alfaro and the
sworn statements of Carlos Cristobal and
Lolita Birrer as well as the counteraffidavits of the petitioners. Apparently, the
painstaking recital and analysis of the
parties' evidence made in the DOJ Panel
Report satisfied both judges that there is
probable cause to issue warrants of arrest
against petitioners. Again, we stress that
before issuing warrants of arrest, judges
merely determine personally the probability,
not the certainty of the guilt of an accused.
In doing so, judges do not conduct a de novo
hearing to determine the existence of
probable cause. They just personally review

the initial determination of the prosecutor


finding a probable cause to see if it is
supported by substantial evidence. The
sufficiency of the review process cannot be
measured by merely counting minutes and
hours. The fact that it took the respondent
judges a few hours to review and affirm the
probable cause determination of the DOJ
Panel does not mean they made no personal
evaluation of the evidence attached to the
records of the case. (emphasis supplied)
The teachings then of Soliven, Inting, Lim, Allado, and Webb
reject the proposition that the investigating prosecutor's
certification in an information or his resolution which is
made the basis for the filing of the information, or both,
would suffice in the judicial determination of probable cause
for the issuance of a warrant of arrest. In Webb, this Court
assumed that since the respondent Judges had before them
not only the 26-page resolution of the investigating panel but
also the affidavits of the prosecution witnesses and even the
counter-affidavits of the respondents, they (judges) made
personal evaluation of the evidence attached to the records
of the case.
Unfortunately, in Criminal Case No. Q-93-43198, nothing
accompanied the information upon its filing on 12 April
1993 with the trial court. As found by the Court of Appeals
in its resolution of 1 July 1993, a copy of the Joint
Resolution was forwarded to, and received by, the trial court
only on 22 April 1993. And as revealed by the certification 71
of Branch Clerk of Court Gibson Araula, Jr., no affidavits of
the witnesses, transcripts of stenographic notes of the
proceedings during the preliminary investigation, or other
documents submitted in the course thereof were found in the
records of Criminal Case No. Q-93-43198 as of 19 May
1993. Clearly, when respondent Judge Asuncion issued the
assailed order of 17 May 1993 directing, among other things,
the issuance of warrants of arrest, he had only the
information, amended information, and Joint Resolution as

bases thereof. He did not have the records or evidence


supporting the prosecutor's finding of probable cause. And
strangely enough, he made no specific finding of probable
cause; he merely directed the issuance of warrants of arrest
"after June 21, 1993." It may, however, be argued that the
directive presupposes a finding of probable cause. But then
compliance with a constitutional requirement for the
protection of individual liberty cannot be left to
presupposition, conjecture, or even convincing logic.

Hence, the DOJ committed grave abuse of discretion when it


executed on 23 July 1993 a unilateral volte-face, which was
even unprovoked by a formal pleading to accomplish the
same end, by dismissing the petition for review. It dismissed
the petition simply because it thought that a review of the
Joint Resolution would be an exercise in futility in that any
further action on the part of the Department would depend
on the sound discretion of the trial court, and that the latter's
denial of the motion to defer arraignment filed at the instance
of the DOJ was clearly an exercise of that discretion or was,
III.
in effect, a signal to the Department that the determination of
the case is within the court's exclusive jurisdiction and
As earlier stated, per its 1st Indorsement of 21 April 1993,
competence. This infirmity becomes more pronounced
the DOJ gave due course to the petitioners' petition for
because the reason adduced by the respondent Judge for his
review pursuant to the exception provided for in Section 4 of denial of the motions to suspend proceedings and hold in
Circular No. 7, and directed the Office of the City Prosecutor abeyance issuance of warrants of arrest and to defer
of Quezon City to forward to the Department the records of arraignment finds, as yet, no support in Crespo.
the cases and to file in court a motion for the deferment of
the proceedings. At the time it issued the indorsement, the
IV.
DOJ already knew that the information had been filed in
court, for which reason it directed the City Prosecutor to
If the only issue before the Court of Appeals were the denial
inform the Department whether the accused have already
of the petitioners' Motion to Suspend Proceedings and to
been arraigned and if not yet arraigned, to move to defer
Hold in Abeyance Issuance of Warrants of Arrest and the
further proceedings. It must have been fully aware that,
public prosecutor's Motion to Defer Arraignment, which
pursuant to Crespo vs. Mogul, a motion to dismiss a case
were both based on the pendency before the DOJ of the
filed by the prosecution either as a consequence of a
petition for the review of the Joint Resolution, the dismissal
reinvestigation or upon instructions of the Secretary of
of CA-G.R. SP No. 31226 on the basis of the dismissal by
Justice after a review of the records of the investigation is
the DOJ of the petition for review might have been correct.
addressed to the trial court, which has the option to grant or However, the petition likewise involved the issue of whether
to deny it. Also, it must have been still fresh in its mind that respondent Judge Asuncion gravely abused his discretion in
a few months back it had dismissed for lack of probable
ordering the issuance of warrants of arrest despite want of
cause other similar complaints of holders of "349" Pepsi
basis. The DOJ's dismissal of the petition for review did not
72
crowns. Thus, its decision to give due course to the petition render moot and academic the latter issue.
must have been prompted by nothing less than an honest
In denying in its resolution of 1 July 1993 the petitioners'
conviction that a review of the Joint Resolution was
necessary in the highest interest of justice in the light of the application for a writ of preliminary injunction to restrain
respondent Judge Asuncion from issuing warrants of arrest,
special circumstances of the case. That decision was
the Court of Appeals justified its action in this wise:
permissible within the "as far as practicable" criterion in
Crespo.

The Joint Resolution was sufficient in itself


to have been relied upon by respondent
judge in convincing himself that probable
cause indeed exists for the purpose of
issuing the corresponding warrants of arrest.
The mere silence of the records or the
absence of any express declaration in the
questioned Order of May 17, 1993 as to
where the respondent Judge based his
finding of probable cause does not give rise
to any adverse inference on his part. The
fact remains that the Joint Resolution was at
respondent Judge's disposal at the time he
issued the Order for the issuance of the
warrants of arrest. After all, respondent
Judge enjoys in his favor the presumption of
regularity in the performance of official
actuations. And this presumption prevails
until it is overcome by clear and convincing
evidence to the contrary. Every reasonable
intendment will be made in support of the
presumption, and in case of doubt as to an
officer's act being lawful or unlawful it
should be construed to be lawful. (31 C.J.S.,
808-810. See also Mahilum, et al. vs. Court
of Appeals, 17 SCRA 482; People vs.
Cortez, 21 SCRA 1228; Government of the
P.I. vs. Galarosa, 36 Phil. 338).

directed the issuance of warrants of arrest only "after June


21, 1993." If he did read the Joint Resolution and, in so
reading, found probable cause, there was absolutely no
reason at all to delay for more than one month the issuance
of warrants of arrest. The most probable explanation for such
delay could be that the respondent Judge had actually wanted
to wait for a little while for the DOJ to resolve the petition
for review.
It is, nevertheless, contended in the dissenting opinion of Mr.
Justice Reynato S. Puno that whatever doubts may have
lingered on the issue of probable cause was dissolved when
no less than the Court of Appeals sustained the finding of
probable cause made by the respondent Judge after an
evaluation of the Joint Resolution. We are not persuaded
with that opinion. It is anchored on erroneous premises. In
its 1 July 1993 resolution, the Court of Appeals does not at
all state that it either sustained respondent Judge Asuncion's
finding of probable cause, or found by itself probable cause.
As discussed above, it merely presumed that Judge Asuncion
might have read the Joint Resolution and found probable
cause from a reading thereof. Then too, that statement in the
dissenting opinion erroneously assumes that the Joint
Resolution can validly serve as sufficient basis for
determining probable cause. As stated above, it is not.
V.

and the same proceeding, there should be no


confusion about the objectives. The
determination of probable cause for the
warrant of arrest is made by the Judge. The
preliminary investigation proper whether
or not there is reasonable ground to believe
that the accused is guilty of the offense
charged and, therefore, whether or not he
should be subjected to the expense, rigors
and embarrassment of
trial is the function of the Prosecutor.
....
We reiterate that preliminary investigation
should be distinguished as to whether it is an
investigation for the determination of a
sufficient ground for the filing of the
information or it is an investigation for the
determination of a probable cause for the
issuance of a warrant of arrest. The first kind
of preliminary investigation is executive in
nature. It is part of the prosecution's job. The
second kind of preliminary investigation
which is more properly called preliminary
examination is judicial in nature and is
lodged with the judge. . . .

In criminal prosecutions, the determination of probable


Ordinarily, the determination of probable cause is not lodged
cause may either be an executive or a judicial prerogative. In with this Court. Its duty in an appropriate case is confined to
We are unable to agree with this disquisition, for it merely
People vs. Inting, 73 this Court aptly stated:
the issue of whether the executive or judicial determination,
assumes at least two things: (1) that respondent Judge
as the case may be, of probable cause was done without or in
Asuncion had read and relied on the Joint Resolution and (2)
And third, Judges and Prosecutors alike
excess of jurisdiction or with grave abuse of discretion
he was convinced that probable cause exists for the issuance
should
distinguish
the
preliminary
inquiry
amounting to want of jurisdiction. This is consistent with the
of the warrants of arrest against the petitioners. Nothing in
which determines probable cause for the
general rule that criminal prosecutions may not be restrained
the records provides reasonable basis for these assumptions.
issuance of a warrant of arrest from a
or stayed by injunction, preliminary or final. There are,
In his assailed order, the respondent Judge made no mention
preliminary
investigation
proper
which
however, exceptions to this rule. Among the exceptions are
of the Joint Resolution, which was attached to the records of
ascertains whether the offender should be
enumerated in Brocka vs. Enrile 74 as follows:
Criminal Case No. Q-93-43198 on 22 April 1993. Neither
held for trial or released. Even if the two
did he state that he found probable cause for the issuance of
inquiries are conducted in the course of one
warrants of arrest. And, for an undivinable reason, he

a. To afford adequate protection to the


constitutional rights of the accused
(Hernandez vs. Albano, et al., L-19272,
January 25, 1967, 19 SCRA 95);
b. When necessary for the orderly
administration of justice or to avoid
oppression or multiplicity of actions
(Dimayuga, et al. vs. Fernandez, 43 Phil.
304; Hernandez vs. Albano, supra; Fortun
vs. Labang, et al., L-38383, May 27, 1981,
104 SCRA 607);

vs. Castelo, 18 L.J. [1953], cited in Raoa


vs. Alvendia, CA-G.R. No. 30720-R,
October 8, 1962; Cf. Guingona, et al. vs.
City Fiscal, L-60033, April 4, 1984, 128
SCRA 577); and
j. When there is clearly no prima facie case
against the accused and a motion to quash
on that ground has been denied (Salonga vs.
Pao, et al., L- 59524, February 18, 1985,
134 SCRA 438).

much of their attention, time, and energy, which they could


devote to other equally, if not more, important cases. Such a
frightful scenario would seriously affect the orderly
administration of justice, or cause oppression or multiplicity
of actions a situation already long conceded by this Court
to be an exception to the general rule that criminal
prosecutions may not be restrained or stayed by injunction. 76

We shall not, however, reevaluate the evidence to determine


if indeed there is probable cause for the issuance of warrants
of arrest in Criminal Case No. Q-93-43298. For, as earlier
stated, the respondent Judge did not, in fact, find that
7. Preliminary injunction has been issued by probable cause exists, and if he did he did not have the basis
c. When there is a pre-judicial question
the Supreme Court to prevent to threatened therefor as mandated by Soliven, Inting, Lim, Allado, and
which is sub judice (De Leon vs. Mabanag,
unlawful arrest of petitioners (Rodriguez vs. even Webb. Moreover, the records of the preliminary
70 Phil. 202);
Castelo, L- 6374, August 1, 1953). (cited in investigation in Criminal Case No. Q-93-43198 are not with
Regalado, Remedial Law Compendium, p.
this Court. They were forwarded by the Office of the City
d. When the acts of the officer are without or
188, 1988 Ed.)
Prosecutor of Quezon City to the DOJ in compliance with
in excess of authority (Planas vs. Oil, 67
the latter's 1st Indorsement of 21 April 1993. The trial court
Phil. 62);
In these exceptional cases, this Court may ultimately and the DOJ must be required to perform their duty.
resolve the existence or non-existence of probable
e. Where the prosecution is under an invalid
cause by examining the records of the preliminary
WHEREFORE, the instant petition is GRANTED and the
75
law, ordinance or regulation (Young vs.
investigation, as it did in Salonga vs. Pao, Allado, following are hereby SET ASIDE:
Rafferty, 33 Phil. 556; Yu Cong Eng vs.
and Webb.
Trinidad, 47 Phil. 385, 389);
(a) Decision of 28 September 1993 and
There can be no doubt that, in light of the several thousand
Resolution of 9 February 1994 of respondent
f. When double jeopardy is clearly apparent private complainants in Criminal Case No. Q-93-43198 and
Court of Appeals in CA-G.R. SP No. 31226;
(Sangalang vs. People and Avendia, 109
several thousands more in different parts of the country who
Phil. 1140);
are similarly situated as the former for being holders of
(b) The Resolutions of the "349" Committee
"349" Pepsi crowns, any affirmative holding of probable
of the Department of Justice of 23 July 1993
g. Where the court has no jurisdiction over
cause in the said case may cause or provoke, as justly feared
dismissing the petitioners' petition for
the offense (Lopez vs. City Judge, L-25795, by the petitioners, the filing of several thousand cases in
review and of 3 February 1994 denying the
October 29, 1966, 18 SCRA 616);
various courts throughout the country. Inevitably, the
motion to reconsider the dismissal; and
petitioners would be exposed to the harassments of warrants
h. Where it is a case of persecution rather
(c) The Order of respondent Judge
of arrest issued by such courts and to huge expenditures for
than prosecution (Rustia vs. Ocampo, CAMaximiano C. Asuncion of 17 May 1993 in
premiums on bailbonds and for travels from one court to
G.R. No. 4760, March 25, 1960);
Criminal Case No. Q-93-43198.
another throughout the length and breadth of the archipelago
for their arraignments and trials in such cases. Worse, the
i. Where the charges are manifestly false and filing of these staggering number of cases would necessarily The Department of Justice is DIRECTED to resolve on the
motivated by the lust for vengeance (Recto affect the trial calendar of our overburdened judges and take merits, within sixty (60) days from notice of this decision,

the petitioners' petition for the review of the Joint Resolution


of Investigating Prosecutor Ramon Gerona and thereafter to
file the appropriate motion or pleading in Criminal Case No.
Q-93-43198, which respondent Judge Asuncion shall then
resolve in light of Crespo vs. Mogul, Soliven vs. Makasiar,
People vs. Inting, Lim vs. Felix, Allado vs. Diokno, and
Webb vs. De Leon.
In the meantime, respondent Judge Asuncion is DIRECTED
to cease and desist from further proceeding with Criminal
Case No. Q-93-43198 and to defer the issuances of warrants
of arrest against the petitioners.
No pronouncement as to costs.
SO ORDERED.

PEOPLE VS. DEL ROSARIO [234 SCRA 246; G.R. NO.


109633; 20 JUL 1994]
Wednesday, February 04, 2009 Posted by Coffeeholic Writes
Labels: Case Digests, Political Law
Facts: Accused was charged and convicted by the trial court
of illegal possession of firearms and illegal possession and
sale of drugs, particularly methamphetamine or shabu. After
the issuance of the search warrant, which authorized the
search and seizure of an undetermined quantity of
methamphetamine and its paraphernalias, an entrapment
was planned that led to the arrest of del Rosario and to the
seizure of the shabu, its paraphernalias and of a .22 caliber
pistol
with
3
live
ammunition.
Issue: Whether or Not the seizure of the firearms was
proper.
Held: No. Sec 2 art. III of the constitution specifically
provides that a search warrant must particularly describe the
things to be seized. In herein case, the only objects to be

seized that
the
warrant
determined was
the 1. Personally evaluate the report and thesupporting documents submitted
methamphetamine and the paraphernalias therein. The by the prosecutor regarding the existence of probablecause, and in basis
seizure
of
the
firearms
was
unconstitutional. thereof, issue arrest warrant and
Wherefore the decision is reversed and the accused is
acquitted.
Probable Cause,
Defined
Such facts and Circumstances which would lead areasonably prudent
man to believe that an offense has beencommitted and the objects sought
in connection with theoffense are in the place sought to be searched
(20 thCentury Fox Film. Corp. vs. CA)
De Los Santos vs. Montesa
Facts:This case came from the Criminal Case for themurder of Reyes et.
al in San Juan Del Monte Bulacan. After the submission of an
investigation by the StateProsecutor, RTC of Malolos Bulacan issued a
warrant of Arrest. First, the accused filed a petition to grant bail and
toReduce Bail, subsequently withdrawn and rather filed anurgent motion
to quash warrants of arrest for lack of existence of probable cause. After
an examination of the lower court of the caseand documents forwarded
to him by the prosecution, the trialcourt found the existence of
probable cause
, but instead of issuing of correspondent Arrest Warrant for
acquiring jurisdiction of the persons involved in the crime, therespondent
judge granted the petition for bail despite of theits withdrawal and lack of
hearing where the prosecutionwould have been accorded the right to
present evidenceshowing the evidence of guilt is strong.Thus the
petitioner filed an administrative complaintcharging the respondent judge
with gross ignorance of thelaw and evident dishonesty in his exercise of
his function.Issue:What should be the procedure in the determinationof
the existence of a
probable cause?
Held:What the respondent judge had in mind in the caseat bar is that,
since he believed that the evidence agisnt theaccused are purely
circumstantial and weak, he resolved togranting the petition of the
accused to grant bail in order for the court to acquire jurisdiction over
these persons instead of issuing warrant of arrest and set hearing for the
prosecutionspresentation of evidence.He is thus then confused in the
procedure of determining the existence of probable cause in the
issuanceof arrest warrant and proceedings for admission to bail.What the
court is given prerogative by theconstitution in accordance to Sec.
2 Art III is that, it issufficient that he personally evaluates the report
andsupporting documents submitted by the prosecution indetermining
probable cause, hearing is not necessary. Insatisfying the existence of
probable cause, the judge shalleither

2.
If there is no sufficient establishment of probable cause, he may
disregard the prosecutors certification and require thesubmission of the
supporting affidavits of witnesses to aid him in arriving at a conclusionas
to the existence of probable cause
.The judge is not tasked to review in detail theevidence submitted during
the preliminary investigation.Thus, once a judge found the probable
cause, whathe should have to do is to issue warrants of arrest
andadmission to bail shall only be granted once persons areapprehended
and are under their jurisdiction.
Lim vs. Felix
Facts:The petitioners. Lim et al, was charged of the crimeof multiple
murder and frustrated murder of CongressmanEspinosa of Masbate
among others.Private respondent, Alfane was designated toreview the
case and was raffled to RTC Makati Br. 56 of therespondent judge,
Nemesio Felix. After transmittal of the case, the respondent Judgeissued
warrant of Arrest against the accused by virtue of theprosecutors
certification in each submitted informationrecommending the existence
of a probable cause.Issue:Whether or not a judge may issue a warrant
of arrest without bail by simply relying on the prosecutionscertification
and recommendation that a probable causeexists.Held:Yes. But by
itself, it does not bind judges to comeout with the warrant of
arrest.Issuance of warrants calls for the exercise of judicialdiscretion on
the part of the issuing judge. If the judge issatisfied from the preliminary
examination conducted by himor by the investigating officer than an
offense complained of has been committed and that there is a reasonable
groundsto believe that the accused has committed it, he must issue
awarrant or order for an arrest. A judge is not required to personally
examine thecomplainants and witnesses, what the constitution
mandatesin satisfying the existence of probable cause, the judge
shalleither;
1. Personally evaluate the report and thesupporting documents submitted
by the prosecutor regarding the existence of probablecause, and in basis
thereof, issue arrest warrant and
2.
If there is no sufficient establishment of probable cause, he may
disregard the prosecutors certification and require thesubmission of the
supporting affidavits of witnesses to aid him in arriving at a conclusionas
to the existence of probable cause

.Moreover, the constitution pursuant to Sec 2 Art IIIalso mandates that x


x x probable cause should bepersonally determined by the judgex x x.
This means that
1
. The determination of probable cause is a function of the judge.2.
Preliminary inquiry made by a prosecutor does notbind the judge.

) The affidavit of Jubair is hearsay and does notpossess gravity for the
establishment of theexistence of probable cause.2) So as the certification
of the petitioner fiscal wontwarrant the existence of probable
cause.Therefore, there can be no prima facie evidence asto necessity for
the issuance of warrant of arrest against theaccused.Thus, the petitioner
filed a petitioner for certiorariand mandamus contending that petitioner
has alreadyconducted a preliminary examination and thus it is
3. Judges and prosecutors alike should distinguish thepreliminary inquiry, ministerialfunction for the respondent to issue arrest warrants.Upon the
other hand, the respondent judge arguesthat the issuance of a warrant of
which determines probablecause for the issuance of a warrant of arrest
arrest involves a judicialpower which necessarily imposes upon him the
fromthe preliminary investigation proper, whichascertains whether the
legal duty of first satisfying himself that there is probable
offender should be held for trial or release.In the case at bar, the only
basis of the respondent judgein issuing warrants of arrest is only cause,independently of and notwithstanding the preliminaryinvestigation
the certification of theprosecutor, without personally examining the made by the provincial fiscal under Republic ActNo. 732; and to that end
he may require the fiscal to submitsuch evidence as may be sufficient to
information(
show at least a primafacie case.Issue:Whether or not the certification of a
which still in Masbate, and wherein the respondent denied the motion
for transmittal of such records of the cases in theground that certification prosecutor issufficient to issue a warrant of arrest.Held:No.The
id enough ground for thedetermination of probable cause and issuance of constitution mandates that the determination of probable cause depends
upon the judgment and discretion of the judge or magistrate in issuing
warrant
warrant of arrest.It simply means that sufficient facts must bepresented to
).Thus, there is no personal examination conducted by the judge to
the judge or magistrate issuing the warrant toconvince him, not that the
establish the existence of probable cause, thereby,the respondent
particular person has committedthe crime, but that there is probable cause
committed abuse of discretion.
for believing thatthe person whose arrest is sought committed the
Note:
Preliminary investigation for the determination of sufficient ground for crimechargedIn the case at bar, the petitioners certification thathe had
already conducted a preliminary investigation in thecase does not
filing of information
sufficiently warrant the existence of probablecause. Nor the single
and
affidavit submitted to the respondent asit is not enough for the respondent
investigation for the determination of a probable cause for the issuance
judge to exercise his judicial function to determine the existence of
of awarrant of arrest
probablecause.However, the petition is granted to continue thehearing of
,
the case in the ground of lack of prosecution andthat refusal of the
Distinguished.
prosecution to submit additional affidavit isnot a valid ground for the
T he former is executive in nature and part of aPROSECU T ORS
dismissal of a case.
JOB. While the latter aka preliminary examination
is judicial in nature and is lodged to the JUDGE.
20th Century Fox vs. CA
Amargavs. Abbas
Facts:The respondent Judge, Macapanton Abbas, after receiving;
1
) An information with a certification stating that the petitioner fiscal,
Amarga has conducted a sufficient preliminaryinvestigation pursuant to
the provision of RA 732, and2) One supporting affidavit of one witness
(Jubair) statingthat he saw the deceased Dugusan Paspasan was shot
andkilled by three gunmen,Dismissed the criminal handled by the
petitioner against Appang et. al on the ground that;
1

The probable cause is wanting in theissuance of the search warrant. As


ingrained by law and jurisprudence, probablecause is defined as
such facts and circumstances whichwould lead a reasonably discreet and
prudent man tobelieve that an offense has been committed and that
theobjects sought in connection with the offenses are in the place sought
to be searched.
The existence of probable cause requires that
thereshould be a personal knowledge by the complainant and witnesses
of the facts upon which the issuance of asearch warrant may be justified
.
In the case at bar, the lower court lifted the issuedsearch warrant on the
ground of the discovery that the NBIwitnesses does not have personal
knowledge that the crimeof Piracy has been committed. The deposition
of suchwitnesses cannot provide a sufficient existence of probablecause
necessary for the issuance of search warrant.Moreover, in accordance
with PD 49 (Anti FilmPiracy), the essence of such law is the similarity of
thepirated and the copyrighted work. Thus, the applicants mustpresent
the court the copyrighted films and compare themwith the evidence of
the video tapes allegedly pirated todetermine whether or not the crim has
been violated.In the case at bar, the applicants does not provideany
evidences 9master tapes and pirated tapes) that wouldconvince the judge
that violation of PD 49 has been
Constitutional Law II (Bill of Rights): Case Briefs: Dennis G.
Libunao UC College of Law
19
committed, and therefore for the existence of probablecause.The articles
included in the search warrant iscouched in general term, making it a
general warrant which isprohibited under the Constitution.
Note:
S
earch and
S
eizure, guaranteed rights.
Protects a citizen against unreasonable searchersinvasion of his privacy
and liberty as to his person, houses,papers and effects. The privacy of a
person must not bedisturbed except in case of overriding social need and
onlyunder stringent procedural safeguards.Thus, the government so as
not to make arrest,search and seizure unreasonable should strictly
followconstitutional and statutory guidelines.

Facts: After surveillance and investigation, petitioner, withthe NBI


apllied for three separate search warrants againstthe video outlets owned
by private respondents, Barretto etal, and was subsequently granted and
issued by the lower court.The applicants did not present the master
tapes andthe pirated tapes upon their application of search
warrant.Instead, only the deposition of two NBI witnesses served asthe
basis for the existence of probable cause.Subsequently, the lower court
lifted the three issuedsearch warrants in the ground that the articles seized,
whichwere still under the custody of NBI, could not be a basis for any
Quintero vs. NBI (
criminal prosecution.Issue:Was there an establishment of probable cause
for the issuance of search warrant against the respondent?Held:None. M

arcos
m
anipulation)
Facts: After the expose of the petitioner, Eduardo Quinteroof the
1
st
district of Leyte in the
1
97
1
Con-con alleging thatsome delegates, including him, in the Con-con are
under thepayroll of the first lady and then President Marcos, NBIagents
raided the houses of the petitioner by virtue of thesearch warrant issued
by the respondent Judge Elias Asuncion. The raid confiscated an
amount of moneyamounting to 379K.The said search warrant was base
from theapplication of two persons. One is from the affidavit
of Congressman Artemio Mate, also from
1
st
district of Leyteand an NBI Agent Samuel Castro, alleging that the
petitioner committed the crime of bribery.The affidavit of the latter
applicant however showedthat he has no personal knowledge about the
allegationsagainst the petitioner.The affidavit of the statements of the
Congressmanalso shows that his allegations are anchored
uponspeculations.No sufficient evidence is presented to therespondent
judge.Issue:Was there an establishment of the existence of aprobable
cause?Held:No. The allegations and the statements of theapplicants are
merely anchored on hearsay andspeculations. As ingrained under
jurisprudence (Roan vs.Gonzales as cited), in application for a search
warrant, if based on hearsay, sannot, standing alone justify theissuance of
search warrant. Thus, it is indispensable that theapplicants should have
personal knowledge of the crimecommitted.In the case at bar, the
deposition of the NBI agent isbased on hearsay in which only and
anchored on theinformation given by Cong. Mata.Moreover, the
deposition of Cong. Mata cannotestablish that he has a direct personal
knowledge of thealleged bribery of the petitioner since his affidavit
shows thatit is only based on speculation. The element of directnessand
definiteness is wanting so as to establish his personalknowledge. Not to
mention, there is no concrete evidencethat would support their
accusation, so as to validly establishprobable cause.Irregularities:
1
) Moreover, there is also irregularity in the printing of the search warrant,
wherein the crime of bribery Art2
10
of RPC was superimposed by ink, which wasoriginally Art. 2

8
2.In the case at bar, the search warrantpresented was in the case of grave
threatsdirected against the nephew of thepetitioner (nephew), thus, the
confiscationof he money is not related to the articlesseized.2) The search
team also violated statutory guidelinesfor a lawful search since there is no
members of thehousehold present while others are searching
thepremises. Thus it is planted and orchestratedsearch.3) Also the
respondents also violated the statutoryguidelines that they should issue a
detailed receiptof articles seized.The lifting of the respondent judge of the
search warrantare null and void.

regular courts, other thanthe higher tribunals ---- the Court of Appeals and
this Court.Held:Quasi-judicial body has been defined as "an organof
government other than a court and other than a legislature,which affects
the rights of private parties through either adjudication or rule making."
Thus, these bodies has thebasic function to adjudicate claims and/or to
determine rights,unless its decisions are appealed to proper
reviewingauthorities.In the case at bar, the PADS is not meant toexercise
quasi judicial function, that is to decide and tryclaims and execute its
judgments, its only task is to handlethe prosecution of salting or black
marketing activities andnothing more.

Presidential
A
ntiD
ollar vs.
C
A
(quasi-judicial body daw)
Facts:The petitioner, PADS, is the presidents armassigned to investigate
and prosecute :dollar-saltingactivities in the country pursuant to PD
1
936.Sometime
1
9
8
5, PADS issued a search warrantagainst respondent Karamfil ImportExport Co. et al. Atty.Gatmaitan of Bureau of Customs applied for a
Searchwarrant, a deputized member of PADS with attached affidavitby
Castro, an investigator and operative of PADS. After the search
procedure, the respondentcontested the search warrant and subsequently
declared bythe lower court null and void.The respondent CA initially
favored the petitioner stating that it is a quasi judicial body that ranks
with RTC andthat, lower courts has no jurisdiction to declare the
issuedsearch warrants of the petitioners null and void. A motion for
reconsideration was filed by thepetitioner Karamfil and subsequently,
November of the sameyear, the respondent CA reversed itself.Thus, the
petitioner filed an appeal alleging that therespondent CA committed
grave abuse of discretion andacted in excess of its appellate jurisdiction
by validating therestraining of the lawful orders or decrees issued by
thepetitioner as a quasi judicial body by the lower Court. This isso since
the petitioner contends that they are quasi-judicialbody that ranks with
the RTC.Issue: As we have observed, the question is whether or not the
Presidential Anti-Dollar Salting Task Force is, in thefirst place, a quasijudicial body, and one whose decisionsmay not be challenged before the

Constitutional Law II (Bill of Rights): Case Briefs: Dennis G.


Libunao UC College of Law
20
However, the enabling statute of the petitioner empowers them to
determine the existence of probablecause and therefore power to issue
warrants of arrest or search and seizure however it does not make them
co-equalwith RTC nor make this agency semi-court.
Note:
K
inds of quasi-Judicial agencies:
(
1
)
Agencies created to function in situations wherein thegovernment is
offering some gratuity, grant, or special privilege, like the defunct
Philippine Veterans Board, Board on Pensions for Veterans, and
NARRA, and PhilippineVeterans Administration.
(2
) Agencies set up to function in situations wherein thegovernment is
seeking to carry on certain government functions, like the Bureau of
Immigration, the Bureau of Internal Revenue, the Board of Special
Inquiry and Board of Commissioners, the Civil Service Commission,
the Central Bank of the Philippines.
(3
) Agencies set up to function in situations wherein thegovernment is
performing some business service for the public, like the Bureau of
Posts, the Postal Savings Bank,Metropolitan Waterworks & Sewerage
Authority, PhilippineNational Railways, the Civil Aeronautics
Administration.
(4
) Agencies set up to function in situations wherein thegovernment is
seeking to regulate business affected with public interest, like the Fiber
Inspections Board, thePhilippine Patent Office, Office of the
InsuranceCommissioner.

(
5
) Agencies set up to function in situations wherein thegovernment is
seeking under the police power to regulate private business and
individuals, like the Securities &Exchange Commission, Board of Food
Inspectors, the Board of Review for Moving Pictures, and the
Professional Regulation Commission.
(
6
) Agencies set up to function in situations wherein thegovernment is
seeking to adjust individual controversiesbecause of some strong social
policy involved, such as theNational Labor Relations Commission, the
Court of AgrarianRelations, the Regional Offices of the Ministry of
Labor, theSocial Security Commission, Bureau of Labor
Standards,Women and Minors Bureau.
3
1
P
D
1936, the enabling statute of P
A
DS
isunconstitutional, reason.
Under the constitution, the existence of
probablecause
is under the sole responsibility and discretion of a judge, who, must be
neutral and prudent enough for hisexercise of conducting preliminary
examination of the factsand circumstances of the case submitted by the
fiscal.Presidential Anti-Dollar Salting Task Forceexercises, or was meant
to exercise, prosecutorial powers,and on that ground, it cannot be said to
be a neutral anddetached "judge" to determine the existence of
probablecause for purposes of arrest or search. Unlike a magistrate,
aprosecutor is naturally interested in the success of his case. Although his
office "is to see that justice is done and notnecessarily to secure the
conviction of the person accused,"he stands, invariably, as the accused's
adversary and hisaccuser. To permit him to issue search warrants and
indeed,warrants of arrest,
is to make him both judge and jury in hisown right, when he is neither
. That makes, to our mind and tothat extent, Presidential Decree No.
1
936 as amended byPresidential Decree No. 2
00
2, unconstitutional.
S

oliven vs.
M
akasiar (
A
controversial case)
T
his case softens the doctrine laid down by the B
A
CHE case where the court states that the judge should personally
depose the complainants and witnessesunder oath and in writing in
determining the existence of probable cause.
T
his is a responsibility that should not be delegated to clerk of court or
other authority.
Facts:This is a consolidated petition for certiorari
andprohibition to review the decision of the respondent JudgeRamon
Makasiar.In the case filed by Beltran, he alleged that therespondent judge
committed grave abuse of discretionamounting to lack or excess of
jurisdiction when therespondent judge issued a warrant of arrest against
thepetitioner for the crime of libel, without the respondent
judgepersonally examining the complainant and witnesses for
thedetermination of probable cause.The petitioner contend that the
constitution requiresthat the judge should personally examine the
complainantand/or witness for the determination of probable cause
andtherefore issue an arrest warrant.Issue:Was the contention correct?
Held:No. (Sadly)What the Constitution underscores is the
exclusiveand personal responsibility of the issuing judge to satisfyhimself
the existence of probable cause. In satisfying himself of the existence of
probable cause for the issuance of awarrant of arrest, the judge is not
required to personallyexamine the complainant and his witnesses.
Followingestablished doctrine and procedure, he shall:(
1
) Personally evaluate the report and the supportingdocuments submitted
by the fiscal regarding theexistence of probable cause and, on the
basisthereof, issue a warrant of arrest; or (2) If on the basis thereof he
finds no probable cause,he may disregard the fiscal's report and
requirethe submission of supporting affidavits of witnesses to aid him in
arriving at a conclusion asto the existence of probable cause.Sound
policy dictates this procedure, otherwise judges would be unduly laden
with the preliminaryexamination and investigation of criminal
complaints insteadof concentrating on hearing and deciding cases filed
beforetheir courts.On June 3
0
,
1

9
8
7, the Supreme Court unanimouslyadopted Circular No.
1
2, setting down guidelines for theissuance of warrants of arrest. The
procedure thereinprovided is reiterated and clarified in this resolution.It
has not been shown that respondent judge hasdeviated from the
prescribed procedure. Thus, with regard tothe issuance of the warrants of
arrest, a finding of graveabuse of discretion amounting to lack or excess
of jurisdictioncannot be sustained.
Pendon vs.
C
A
(
S
i
m
ilar
C
ase with the 20
th
C
enturyFox)
Facts:
Constitutional Law II (Bill of Rights): Case Briefs: Dennis G.
Libunao UC College of Law
21
Sometime February of
1
9
8
7,
1
st
Lt. Felipe Rojas,Officer in charge of the Philippine Constabulary CriminalInvestigation Service (PC-CIS), Bacolod City, filed
anapplication for a search warrant, alleging that KENERTrading is the
possession of some NAPOCOR Properties,contrary to anti-fencing
law.His application was subscribed before JudgeDemosthenes
Magallanes of MTC Bacolod City supported bythe joint deposition of
two (2) witnesses, Ignacio L. Reyes, anemployee of NAPOCOR
(National Power Corporation) andIAI Eduardo Abaja of the CIS of

Bacolod City.Search warrant was issued by the lower court andthen


proceeded the search operation, seizing some articlesfrom the
premises.The petitioner, Pendon, filed an application for thereturn of the
articles seized by the PC-CIS contending thatthe search warrant is
illegally issued, which, however deniedby the lower court.The petitioner
subsequently filed a petition for mandamus certiorari and prohibition in
the CA assailing thelegality of the Search Warrant, which also was
denied by theCA stating that there is an existence of a probable cause.The
petitioner contend that the applicants for theassailed warrants has no
personal knowledge that thearticles sought to be seized were
stolen.Moreover, he also alleged that there is irregularitieswith the
contents of the joint deposition of the two witnessesand that there is no
personal examination conducted by the judge as required by the law and
the rules.Issue:Whether or not the search warrant is illegally
issued.Held:Yes. The assailed search warrant was illegallyissued.The
constitution provides that search warrants areissued based solely on
probable cause. And in determiningprobable cause, it is required that
1
)
T
he judge
(
or) officer must examine thewitnesses personally;2)
T
he examination must be under oath; and 3)
T
he examination must be reduced to writing inthe form of searching
questions and answers
And also, such finding of probable cause should besubstantiated by the
records.In the case at bar, We find that the requirementmandated by the
law and the rules that the
judge must personally examine the applicant and his witnesses in
theform of searching questions and answers before issuing thewarrant
, was not sufficiently complied with. The applicanthimself was not asked
any searching question by JudgeMagallanes. The records disclose that
the only part played bythe applicant,
Lieutenant Rojas was to subscribe theapplication before Judge
Magallanes
.
T
he applicationcontained pre-typed questions, none of which stated
that applicant had personal knowledge of a robbery or a theft and that
the proceeds thereof are in the possession and control of the person
against whom the search warrant was sought tobe issued

. Affidavits of the complainants and witness are notsufficient for


determining a probable cause.Pursuant to the Anti fencing law, the
records alsoshows that the questions asked during the deposition
cannotsupport the finding of the probable cause
T
here was also nostatement in the joint deposition that the articles sought
to beseized were derived from the proceeds of the crime of robbery or a
theft or that applicants have any knowledge that a robbery or theft was
committed and the articles sought tobe seized were the proceeds thereof
.Moreover, the items to be confiscated listed in theSW is couched on
general terms, therefore considered as aprohibited general warrants.
(
Galvanized bolts, aluminumwires and other Napocor tower and line
parts and accessories)
The items confiscated does not also fall under thepersonal properties that
are allowed to be seized under thelaw.Thus the petition is granted and
the articles areordered to be return since possession thereof is
notprohibited by the law.
Note:
A
nti Fencing Law, commission of, in the case at bar,
The offense which petitioner was sought to becharged was violation of
the anti-fencing law which
punishesthe act of any person who, with intent to gain for himself or for
another, shall buy, receive, possess, keep, acquire,conceal, sell or dispose
of, or shall buy or sell, or in any other manner deal in any article, item,
object or anything of valuewhich he knows, or should have known to
him, to have beenderived from the proceeds of the crime of robbery or
theft
(
Sec.
2
a, P.D. 1
6
1
2
)
P
ersonal properties that may be confiscated inaccordance with the law,
enumerate
Section 2, Rule
1
26 of the Rules on Criminal Procedure
a)

T
he subject of an offense;b)
S
tolen or embezzled property and other proceeds or fruits of an offense;
and c) Used or intended to be used as a means of committing an offense.
If the law does not prohibit the possession of the articlessought, it should
be returned to the owner.
People vs. Inting (si
m
ilar to
D
eLos
S
antos case)Preli
m
inary inquiry of the prosecutor does not bind the judge for the deter
m
ination of probable cause
Facts:Sometime
1
9
88
, Editha Barba filed a letter complaint against OIC Mayor of Tanjay,
Negros oriental withthe Comelec for transferring her to a remote
barangaywithout prior clearance from the Comelec.The complaint was
handled by the provincialelection Supervisor of Dumaguete City, atty.
GerardoLituanas . after his investigation, he found
prima facie
evidence and then filed to the respondent Trial court under Judge
Enrique Inting a criminal case in violation of Sec. 26
1
of the Omnibus Election Code against the OIC Mayor. An arrest
warrant was issued but later set aside onthe ground that Atty. Lituanas is
not authorized to determinethe existence of probable cause. Hence this
petition.Issue:Does the Provincial Election Supervisor of theComelec has
the jurisdiction to find the existence of aprobable cause?Held:No. the
phrase under Sec 2 Art II of the Constitutionwhich reads
and such other responsible officer as may beauthorized by law
has been deleted, making thedetermination for the existence of a
probable cause under the sole responsibility and discretion of a Judge.
Constitutional Law II (Bill of Rights): Case Briefs: Dennis G.
Libunao UC College of Law
22

It should be distinguished that preliminaryinvestigation for the


determination of a probable cause isvested with the authority of Judges
(J
udicial in nature)
andwhile preliminary investigation in determining whether or notthe
accused is guilty of the offense charged and thereforeshould be subjected
to litigation is vested to the authority of the prosecutor
(
executive in nature).
The separate kinds of preliminary investigation hasbeen delineated and
that the latter kind has been deleted tothe authorities of judges
(
preliminary investigation proper).
What the constitution vested the Comelec is toinvestigate and prosecute
cases in violation of election laws(Art. IX Sec 2), however, this does not
mean that the power to determine the existence of probable cause is
within thescope of their authority, but rather, they are empowered onlyin
purpose in assisting the Judge to determine the probablecause and for
filing for an information.
Note:
The Prosecutor cannot assume roles in theprosecution of election
offenses, if he has, it is because he isdeputized by the Comelec to handle
such election offenses.
U
m
il vs. Ra
m
os
Facts:This is a consolidated petition for
habeas corpus
where the petitioners alleged that their detention is illegal andunlawful as
their arrests were made without warrant and thatno preliminary
investigation was first conducted, making theinformations filed against
them are null and void.The respondents contends otherwise.In this
consolidated case, all of the petitioners arecharged under the Anti
Subversion Law, with an exception tothe case of Enrile vs. Lim (Inciting
to sedition) and Nazarenovs. Station Commander.The rest are charged
guilty of rebellion, a crimeagainst the State, and is a
continuing crimes
in nature. Theywere found of the possession of unlicensed firearms
andammunitions as well as subversive documents.Issue:Was warrantless
arrest in the case at bar illegal, asthe arrest was not made pursuant to the
constitutional andstatutory guidelines for the issuance of warrantless

arrest?Held:Warrantless arrest conducted in the case at bar


islawful.Pursuant to 5 Rule
11
3 of RRC, arrest of a personwithout a warrant of arrest or previous
complaint isrecognized by law. The instances where a valid
warrantlessarrest may be effected are the following.
S
ec5.
A
rrest without Warrant: When lawful
A
peaceofficer or a private person may, without warrant, arrest a person:
(
a) When in his presence, the person to be arrested has committees, is
actually committing, or isattempting to commit an offense.
(
b) When an offense has in fact just beencommitted, and he has personal
knowledge of the facts indicating that the person to bearrested has
committed it; and
(c)
When the person to be arrested is a prisoner who has escaped from a
penal establishment or place where he is serving final judgment
or temporarily confined while his case is pending,or has escaped while
being transferred from oneconfinement or another.
Thus, pursuant to the RRC, warrantless arrest is justifiedwhen a person
arrested is caught in
flagranti delicto

In the case at bar, the petitioners had freshlycommitted or were actually


committing an offense.In the case of the petitioners who were charged
of rebellion and inciting to sedition, the court held that they arelawfully
detained and the informations filed against them arevalid.This is since
the crime in which they are arrestedare continuous crime which is against
the State, thus, thecontinued possession of subversive materials and
unlicensedfirearms and ammunitions, even without
preliminaryinvestigation and without warrant, as long as the
authoritieshave confiscated such prohibited materials under
their possession and that in the
case of Enrile vs. Lim
, they hadactually done what is prohibited by law as long as they
arepositively identified by a witness having a personalknowledge of the
committed crime, their arrest are lawful.This is justified since under the
doctrine of Garcia-Padilla vs. Enrile, persons arrested of rebellion does
notneed to follow strict procedures,
since their crimescommitted are violence against the
S
tate, whichconcerns the very survival of the society and government.
In the case of Nazareno vs. station Commander, thewarrantless arrest is
justified since there was a prior information filed against the petitioner for
the same offense,and that he was positively identified by a witness who
has apersonal knowledge about the crime he committed. He wasarrested
while he was at large.
Paderangs vs.
D
rilon
Facts:Felizardo Roxas, was included as a co-accused inan amended
information for the murder of Bucag et. al.The respondent state
or inthe act of co
prosecutor Henrick Gingoyanwas designated by the DOJ to conduct the
mm
preliminaryinvestigation and directed to include the petitioner Roxas asa
itting an offense; or when an offense has just been co
co-accused in the criminal case.Contending that the preliminary
mm
investigation wasnot yet completed so as to include Roxas as a Coitted and the person
accused inthe case and that he was deprived of his right to present
m
acorresponding counter-affidavit and additional evidencecrucial to the
aking the arresthas personal knowledge of the facts indicating that
determination of the allege linkage to the crimecharged, the petitioner
theperson arrested has co
filed a Motion for reconsideration, butwas later denied. A petition for
mm
Review was subsequentlyfiled in the DOJ but was also denied.Thus, the
itted it.
petitioner filed a suit before the SCcontending that preliminary
The rational behind the concept o warrantless arrestis laid down in the
investigation is not yet completedthereby, there is no existing prima facie
PPI vs. Malasugui, that to sustain thatarresting a person without warrant evidence or probablecause that would justify the petitioners inclusion to
illegal
the crimecharged.Issue:Whether or not the preliminary investigation is
would leave thesociety, into a large extent, at the mercy of the
theproper forum to present evidence to prove or disprove theguilt of the
shrewdest,the most expert and the most depraved of criminals,facilitating party.Held:No. A preliminary investigation is defined as
their escape in many instances.
an inquiry or proceeding for the purpose of determining whether there is

Constitutional Law II (Bill of Rights): Case Briefs: Dennis G.


Libunao UC College of Law
23
sufficient ground to engender a well founded belief that acrime
cognizable by the Regional
T
rial Court has beencommitted and that the respondent is probably guilty
thereof,and should be held for trial.
The quantum of evidence now required inpreliminary investigation is
such evidence sufficient to"engender a well founded belief" as to the fact
of thecommission of a crime and the respondent's probable guiltthereof.
A preliminary investigation is not the occasion for the full and
exhaustive display of the parties' evidence; it isfor the presentation of
such evidence only as may engender a well-grounded belief that an
offense has been committed and that the accused is probably guilty
thereof
Preliminary investigation is generally inquisitorial,and it is often the only
means of discovering the persons whomay be reasonably charged with a
crime, to enable the fiscalto prepare his complaint or information.
It is not a trial of thecase on the merits and has no purpose except
that of determining whether a crime has been committed andwhether
there is probable cause to believe that the accusedis guilty thereof, and it
does not place the person againstwhom it is taken in jeopardy.
It does not mean absolute certainty. Innocenceof a party, whether
merely proclaimed or realdoes not necessarily preclude a finding of
PC.
[DOH v. Sy Chi Siong (1989)
]
People vs. Villanueva
A clear difference between the 197
3
and the 1987 Constitutional provision on Sec
2
Art III
(
responsible officers authorized by law)
Facts:Sometime
1
9

80
, the petitioners, city fiscal andassistant City fiscal filed an information
against Rogelim Yeewith serious slander by deed. The petitioners
certified thatthey have already conducted a preliminary investigation
andfind probable cause.Instead of issuing an arrest warrant, the
respondentJudge, Napoleon Villanueva conducted an
ex parte
preliminary examination for scanning the records todetermine the
existence of probable cause. After such examination, the respondent
judge foundout that the crime committed may wither be slander by
deedor slight physical injury. However, since the information wasfiled 64
days after the commission of the crime, the judgedismissed the case by
prescription.Hence, this petition for certiorari and mandamus onthe
ground that the respondent committed grave abuse of discretion
amounting to lack or excess of jurisdiction.In his defense, the respondent
judge contends thathe was not satisfied in the existence of probable cause
andthat he needed to conduct a separate examination for itsdetermination
and that, the case was dismissed because of prescription.Issue:Whether
or not the judge may dismiss the caseoutright upon the absence of
probable cause.Held:No.Under the
1
973 constitution, the judge has thepower and legal duty to determine the
existence of probablecause, also, in cases where he is not satisfied with
thecertification of the prosecutor in the information, he mayconduct
preliminary investigation authorized under Sec 6Rule
11
2 of the RRC.But that power does not include the authority todismiss
outright the information if the judge believes thatthere is no probable
cause. The judge should require thefiscal to present additional evidence
to show probable cause.If the fiscal refuses to do so, then the case may be
dismissedfor "lack of prosecution" as also stated under the case
of Amarga vs. Abbas.The fiscal is a "responsible officer authorized
bylaw" within the meaning of Section 3 of the Bill of Rights.
Hisdetermination of probable cause is a sufficient justification for the
issuance of a warrant of arrest.Thus, it was held that "in a clash of views
betweenthe judge who did not investigate and the fiscal who did,
or between the fiscal and the offended party or the defendant,those of the
fiscal's should normally prevail"Thus, during the 73 constitution, it is a
rule that thecertification that probable cause exist by the
preliminaryinvestigation of the prosecutor is a sufficient ground for
a judge to issue probable cause. The judge, therefore shouldntconduct
another examination.Note:Fiscals certification is sufficient, reason:The
time-saving practice has been for the judge(municipal, city or Court
of First Instance) to rely on thepreliminary investigation conducted
by the fiscal as the basisfor issuing the warrant of arrest. That practice is

supported bythe presumption that the fiscal performed his duties


regularlyand competently (Edillon vs. Narvios, Administrative CaseNo.
1
753, August 2
1
,
1
9
80
, 99 SCRA
1
74). And that practiceexisted even under the old Constitution when
Section
1
(3) of the Bill of Rights did not contain the terms "warrant of arrest"and
"such other responsible officer as may be authorized bylaw."We hold
that, as a rule, a trial judge should not holdanother preliminary
examination to determine probable causein case the fiscal has filed an
information and certified that hehas conducted the requisite preliminary
investigation. Thatcertification means that there is a prima facie case
againstthe accused and that the issuance of a warrant of arrest is justified.
Placer vs. Villanueva
Conflicting decision as far as the fiscals certification in theexistence of
probable cause is concerned
Facts:The petitioners, Fiscal and assistant City Fiscal, intheir submission
of ten information in the court of therespondent, provides certification
that probable cause existsand that there is a reasonable ground to believe
that crimehas been committed and the accused are probably guiltythereof
and therefore be subjected to arduous litigationprocess.(existence of
probable cause and
prima facie
evidence)Following receipt of said informations, respondent judge
issued an order setting on April 5,
1
9
8
2 the hearing of said criminal cases for the purpose of determining
thepropriety of issuing the corresponding warrants of arrest. After said
hearing, respondent issued the questioned ordersdated April
1
3,
1
5,
1

6 and
1
9,
1
9
8
2,
requiring petitioners tosubmit to the court the affidavits of the prosecution
witnesses
Constitutional Law II (Bill of Rights): Case Briefs: Dennis G.
Libunao UC College of Law
28
and other documentary evidence in support of theinformations to aid him
in the exercise of his power of judicial review of the findings of probable
cause by petitioners.
Petitioners filed two separate motions for reconsideration of said orders,
contending that under P.D.Nos. 77 and 9
11
, they are authorized to determine theexistence of a probable cause in a
preliminaryexamination/investigation, and that their findings as to
theexistence thereof constitute sufficient basis for the issuanceof warrants
of arrest by the court. On April 2
8
,
1
9
8
2,respondent judge denied said motions and reiterated hisorder to
petitioners to submit the supporting affidavits andother documents within
five (5) days from notice.Hence this petition for certiorari and mandamus
wasfiled.Issue:Whether or not the respondent city judge may, for the
purpose of issuing a warrant of arrest, compel the fiscalto submit to the
court the supporting affidavits and other documentary evidence
presented during the preliminaryinvestigation.Held:NO. As dictated in
the Amarga case, the certification of the fiscal may relied upon by the
judge bit however notbinding for an automatic issuance of arrest warrant,
this is so,since the issuance of warrants is not ministerial function, itcalls
for the exercise of judicial discretion on the part of theissuing
magistrate.Under Section 6, Rulle
11
2 of RRC
the judge must satisfy himself of the existence of probable cause
beforeissuing a warrant or order of arrest

.
I
f on the face of theinformation the judge finds no probable cause
, he maydisregard the fiscal's certification and require the submissionof
the affidavits of witnesses to aid him in arriving at aconclusion as to the
existence of a probable cause. This hasbeen the rule since
U.
S
. vs. Ocampo and
A
marga vs.
A
bbas
. And this evidently is the reason for the issuance byrespondent of the
questioned orders of April
1
3,
1
5,
1
6,
1
9,
1
9
8
2 and July
1
3,
1
9
8
2. Without the affidavits of theprosecution witnesses and other evidence
which, as a matter of long-standing practice had been attached to
theinformations filed in his sala, respondent found theinformations
inadequate bases for the determination of probable cause. For as the
ensuing events would show, after petitioners had submitted the required
affidavits, respondentwasted no time in issuing the warrants of arrest in
the caseswhere he was satisfied that probable cause existed.
Note:
C
ases subjected to
S
u

mm
ary Procedures, reason for requiring the sub
m
ission of affidavits of the co
m
plainantand witnesses
To enable the court to determine whether to dismissthe case or require
further proceedings.Under the Rule on Summary Procedure in
SpecialCases, the respondent judge has the power to order theoutright
dismissal of the charge if, from the information andthe affidavits attached
thereto, he finds the same to bepatently without basis or merit.
I
m
portant: Re
m
edial Law;
C
ri
m
inal ProcedureOn Probable cause:To be deter
m
ined by the Judge,
The probable cause here to bedetermine by the judge is to
whether to issuea search or arrest warrant
or not
. Notehowever that, though the judge finds probablecause,
this does not mean that he shouldautomatically issue the
same, in cases of anarrest warrant, he should determine
whether there is a necessity to arrest the accused
soas not to frustrate the ends of justice.
Otherwise, the judge may refuse to issue anarrest warrant.
To be deter
m
ined by the prosecutor
The probable cause here is todetermine whether or not there
is a crimecommitted and that the accused is probablyguilty
thereof. This is not tantamount to thedetermination of guilt
of the accused, what isimportant is that he is
probably guiltythereof.
Tolentino vs. Villaluz
Facts:Sometime
1

973, the respondent Fiscal Mojica fileda complaint against the petitioners
Bayot, Parra and Castillobefore the Circuit Criminal Court Court of
Pasig under therespondent Judge, Onofre Villaluz, for violation of the
Anti-Graft and Corrupt Practices Act. After preliminary examination
and investigation, therespondent judge issued a resolution stating that;
1
. There exists a
prima facie
case against thepetitioners.2. Ordering as arrest warrant against the
accused-petitioners.3. Ordering respondent Fiscal to conduct
apreliminary examination and investigation in thiscase to determine the
criminal liability of all themembers of the said City Council and
thereafter tofile the corresponding information in the court of competent
jurisdiction, if evidence so warrant. A subsequent motion to Dismiss was
filed by thepetitioners but however was denied by the respondent
court.Hence a petition for certiorari was filed by the petitionersenjoining
the respondent Judge to take cognizance in thecriminal case filed against
them and to declare all theproceedings undertaken and orders issued by
the lower courtnull and void.They contend that the respondent judge has
limited jurisdiction as a judge of the Circuit Criminal Court to try
anddecided specific criminal cases. They allege that therespondent has
no authority to conduct preliminaryinvestigations pursuant to RA 5
1
9.Issue:Does the limited jurisdiction of the respondentJudge also limits
its authority as to the issuance of warrantsof arrest, determination of
probable cause and conductingpreliminary investigation?Held:No.What
is limited by RA 5
1
79 is only the scope of thecases tat maybe tried by Circuit Criminal
Courts.These courts cannot try all criminal cases fallingunder the
jurisdiction of the CFIs as courts of general jurisdiction. They can only
try cases provided under section
1
of the said law. However, these does not follow that judgesunder these
courts also has limited power and authority.They have the same authority
and powers as those conferredupon regular CFIs.
Constitutional Law II (Bill of Rights): Case Briefs: Dennis G.
Libunao UC College of Law
29
Thus, the judges under these courts can alsoconduct preliminary
investigations for the determination of prima facie case and of probable
cause for the issuance of warrants.
Note:Reason for the establishment of Circuit Criminal Courts.

To alleviate the burden of the CFI and expedite thedisposition of case.


Limiting the powers conferred to judgesof these courts would defeat
such purpose, as the judges of the CFIs will still carry the burden to try
and decide thesecases.
C
ruz vs. Gatan
Facts:Serafin G. Cruz was arrested by PC agents on August 3
0
,
1
976, at the Baguio Checkpoint along KennonRoad, Baguio City, and
brought to Camp Olivas, SanFernando, Pampanga, under the command
of respondentGen. Romeo Gatan, for custodial interrogation, where he
ispresently detained.On October 22,
1
976, a petition for the issuance of a writ of habeas corpus was filed in his
behalf wherein it wasclaimed that the said Serafin Cruz is held
incommunicado;that he is restrained of his liberty without due process of
lawand is in the custody of the respondent not by virtue of a judgment or
court order; that he is not a member of anysubversive organization
covered by Proclamation No.
1081
and falls within the clays of persons to whom the privilege of the writ of
habeas corpus has not been suspended.The petitioner is said to be an
over-all Commander and Contractor General of the Bataan Defenders
Command,an unregistered veterans outfit. He is thus allegedly
violated Art.
1
47 of the RPC (Illegal Associations).Subsequently, the counsel for the
petitioner however calimed after a discussion with the petitioner claimed
that his continued detention is the free will andvolition of the petitioner
who expressed fears that he mightbe harmed or injured by some
members of the "BataanDefenders Command" if he were free from
custody while themastermind and legal counsel of the association, one
Atty.Cecilio Baylon Buenafe, has not yet been arrested.Issue:Was the
petitioner illegally detained?Held:No. The privilege of writ of habeas
corpus wassuspended by virtue of the declaration of PD
1081
.Moreover, the case (petition for habeas corpus) becamemoot and
academic since the petitioner voluntarily give itsconsent for its detention
due to threats.The petitioner in the instant case was arrested anddetained
by virtue of an Arrest, Search, and Seizure Order issued by the Secretary
of National Defense for violation of Article
1

47 of the Revised Penal Code pursuant to theaforequoted General Order


No. 2-A, as amended; hence, hisarrest and continued detention is legal.
The declaration of martial law and the consequent suspension of the
privilege of the writ of habeas corpus with respect to persons
reasonablybelieved or charged to be engaged in the disorder or
infomenting it having been settled in the case of Aquino, Jr. vs.Ponce
Enrile etc., et al., any inquiry by this Court into thecontinued detention of
the petitioner would be purposeless.
Geroni
m
ovs.Ra
m
os
Facts:On January
8
,
1
9
80
, private respondent JulianPendre filed a petition with the Commission
on Elections(COMELEC) to disqualify petitioner Meliton C.
Geronimofrom running as a candidate for the mayorship of Baras,
Rizalon the ground of political turncoatism. After hearing, the Comelec
declared his adversaryfor the position of mayor in the said town Bayani
Ferrera asduly elected mayor.The decision of the Comelec resulted into
anger anddiscontent from the supporters of the petitioners, theyentered
the municipal hall and stayed until May
1
3,
1
9
8
2.Subsequently, Pendre filed a petition before the SCpraying to cite the
petitioner in contempt. After hearing, theComelec granted the petition,
sentencing the petitioner tosuffer 5-month imprisonment with fine. As a
result, the PC forcibly arrest the petitioner, acommotion between the PC
and supporters of the petitionershappened on May
1
4,
1
9
8
2. The incident resulted to theissuance of 5 warrants of arrest against the
supporters of thepetitioner for the following violations:

Usurpation of Authority [Art. 177, Revised Penal Code


(
RPC)]; Violation of Usurpation of Authority of Official
(
sic) [Art. 177, Revised Penal Code
(
RPC)];
T
umultuous Affray [Art. 1
5
3
, RPC];Sedition
(
Art. 1
3
9, RPC); Illegal Possession of Firearms;Disobedience to a Person in
Authority or the Agent of suchPerson
(
Art. 1
5
1, RPC) and Alarm and Scandal
(
Art. 1
55
,RPC).
Thus three separate petitions were filed by thepetitioner. One of these
prays for the dismissal of the 75complaints filed against the petitioner and
his supporters.Issue:Was the issuance of 75 warrant in a single day
validas a matter of procedure?Held:No.It is impossible for one Judge to
finish thepreliminary investigation/examination of 75 persons in oneday.
Similar caution is warranted for the issuance of warrants of arrest. In the
case at bar and especially considering the background circumstances
which led to thefiling of charges, we find it highly improbable for the
judge tobe able to determine the existence of reasonable grounds
tobelieve that the offenses have been committed and that eachand
everyone of the seventy-six
(
7
6
) persons are probably guilty thereof in a matter of a few hours and to
proceed withthe issuance of the warrants of arrest also on the same day.It
should be remembered and the judge should have takeninto account that
all the offenses which were allegedly committed were only the product
and result of the outburst of the feelings and emotions of the people of

Baras due to thehighly tense situation in the municipality, which


culminated with the May 1
4
, 198
2
incident.
T
he judge, therefore, inconducting his preliminary investigation should
haveascertained with double care if, indeed, there was ampleevidence to
warrant the issuance of arrest warrants and eventually the filing of
criminal informations against such abig number of persons, most of
whom were impelled by different motivations and whose respective
participationswere of varying natures and degrees. One of the
crimescharged was sedition, a particularly grave offense not to belightly
treated by any prosecuting officer or judge.
T
he possibility of prolonged detention because of the chargeshould have
been considered.
Thus, such a hasty and manifestly haphazardmanner of conducting the
preliminary examination todetermine probable cause for the issuance of
the warrants of
Constitutional Law II (Bill of Rights): Case Briefs: Dennis G.
Libunao UC College of Law
30
arrest and eventually for the filing of the necessaryinformation cannot be
sanctioned by the Supreme Court.
It is an undisputed fact that all the criminal chargeswere the result of the
events that transpired before and until the May 1
4
, 198
2
incident, when Mr. Geronimo was forcibly taken out of the municipal
building of Baras by the military.
T
he charges were also filed almost successively: one onMarch
20
; one on April 1
2
; one on April 1
4
; two on May
4
,two on May 1

4
, and one on May 19, - and with the samecourt and presided over by the
same judge. In one of thecriminal complaints wherein about 7
5
people were charged,the warrants of arrest were issued on the same day
that the preliminary examination was conducted.
A judge
(
Ricardo Javier in the case at bar)
must firstsatisfy himself of the existence of probable cause
beforeissuing a warrant or order of arrest. The requirements arestrict. The
examination must be legitimate and not a feignedone intended to justify a
course of action alreadypredetermined.Thus, because of the inherent
impossibility of a judge to conduct preliminary examination to all 75
complaintsand finding probable cause to all of it resulting to hisissuance
of warrants is hasty and haphazard.
Note
(V
ery
I
mportant):
Existing Ruling on determination of probable cause:
Issuance of;
S
earch and
S
eizure
W
arrant:
The judge should personally examine the witnessesand complainant
under oath or affirmance for thedetermination of probable cause. This is
the doctrineembodied under Sec 2 Art III of the Constitution.
A
rrest
W
arrant:
The judge is not required to personally examine thecomplainant and
witnesses to determine the probable cause.It is enough that(a) He
examines the report and supporting documentsprovided by the fiscal in
the determination of probable cause, and in basis thereof, issue
arrestwarrant.(b) If there is no sufficient establishment of probablecause,
he may disregard the prosecutorscertification and require the
submission of thesupporting affidavits of witnesses to aid him inarriving
at a conclusion as to the existence of probable cause.

Juan Ponce Enrile vs Judge Salazar


Habeas Corpus Right to Bail Rebellion SC Cannot
Change Law

In February 1990, Sen Enrile was arrested. He was charged


together with Mr. & Mrs. Panlilio, and Honasan for the
crime of rebellion with murder and multiple frustrated
murder which allegedly occurred during their failed coup
attempt. Enrile was then brought to Camp Karingal. Enrile
later filed for the habeas corpus alleging that the crime being
charged against him is nonexistent. That he was charged
with a criminal offense in an information for which no
complaint was initially filed or preliminary investigation was
conducted, hence was denied due process; denied his right to
bail; and arrested and detained on the strength of a warrant
issued without the judge who issued it first having personally
determined the existence of probable cause.
ISSUE: Whether or not the court should affirm the
Hernandez ruling.
HELD: Enrile filed for habeas corpus because he was
denied bail although ordinarily a charge of rebellion would
entitle one for bail. The crime of rebellion charged against
him however is complexed with murder and multiple
frustrated murders the intention of the prosecution was to
make rebellion in its most serious form so as to make the
penalty thereof in the maximum. The SC ruled that there is
no such crime as Rebellion with murder and multiple
frustrated murder. What Enrile et al can be charged of would
be Simple Rebellion because other crimes such as murder or
all those that may be necessary to the commission of
rebellion is absorbed hence he should be entitiled for bail.
The SC however noted that a petition for habeas corpus was
not the proper remedy so as to avail of bail. The proper step
that should have been taken was for Enrile to file a petition

to be admitted for bail. He should have exhausted all other


efforts before petitioning for habeas corpus. The Hernandez
ruling is still valid. All other crimes committed in carrying
out rebellion are deemed absorbed. The SC noted, however,
that there may be a need to modify the rebellion law.
Considering that the essence of rebellion has been lost and
that it is being used by a lo t of opportunists to attempt to
grab power.

PEOPLE V. MENGOTE [210 SCRA 174; G.R. NO.


87059; 22 JUN 1992]
Friday, February 06, 2009 Posted by Coffeeholic Writes
Labels: Case Digests, Political Law
Facts: The Western Police District received a telephone call
from an informer that there were three suspicious looking
persons at the corner of Juan Luna and North Bay Boulevard
in Tondo, Manila. A surveillance team of plainclothesmen
was forthwith dispatched to the place. The patrolmen saw
two men looking from side to side, one of whom holding his
abdomen. They approached the persons and identified
themselves as policemen, whereupon the two tried to run but
unable to escape because the other lawmen surrounded them.
The suspects were then searched. One of them the accusedappellant was found with a .38 caliber with live ammunitions
in it, while his companion had a fan knife. The weapons
were taken from them and they were turned over to the
police headquarters for investigation. An information was
filed before the RTC convicting the accused of illegal
possession of firearm arm. A witness testified that the
weapon was among the articles stolen at his shop, which he
reported to the police including the revolver. For his part,
Mengote made no effort to prove that he owned the fire arm
or that he was licensed to possess it but instead, he claimed
that the weapon was planted on him at the time of his arrest.
He was convicted for violation of P.D.1866 and was
sentenced to reclusion perpetua. In his appeal he pleads that
the weapon was not admissible as evidence against him
because it had been illegally seized and therefore the fruit of
a
poisonous
tree.
Issue: Whether or not the warrantless search and arrest was
illegal.
Held: An evidence obtained as a result of an illegal search
and seizure inadmissible in any proceeding for any purpose
as provided by Art. III sec 32 of the Constitution. Rule 113
sec.5 of the Rules of Court, provides arrest without warrant
lawful when: (a) the person to be arrested has committed, is
actually committing, or is attempting to commit an offense,
(b) when the offense in fact has just been committed, and he
has personal knowledge of the facts indicating the person

arrested has committed it and (c) the person to be arrested


has escaped from a penal establishment or a place where he
is serving final judgment or temporarily confined while his
case is pending, or has escaped while being transferred from
one
confinement
to
another.
These requirements have not been established in the case at
bar. At the time of the arrest in question, the accused
appellant was merely looking from side to side and holding
his abdomen, according to the arresting officers themselves.
There was apparently no offense that has just been
committed or was being actually committed or at least being
attempt by Mengote in their presence. Moreover a person
may not be stopped and frisked in a broad daylight or on a
busy
street
on
unexplained
suspicion.
Judgment is reversed and set aside. Accused-appellant is
acquitted.
Warrantless Search and
Seizure-Whenvalid or not?
People vs. Go
Facts: After an information was relayed to the intelligenceand Follow
Up Unit of Calamba Police that shabu or
methamphetamine hydrochloride
is supplied in Crossing,Calamba Laguna, agents went in the place for
follow up. A police agent, Panuringan, then reported that hesaw the
accused, Luisito Go, enter a disco House with twowomen with a gun
tucked in the accused waist.Three policemen entered the disco House for
the
Operation Bakal
to search for illegally possessed firearms.Thus, when the accused was
ordered to stand-up, policeagents saw a revolver tucked in his waist, they
asked for itslicensed but the accused never showed it, instead, he gavehis
drivers license bearing other name. The police agentsconfiscated the gun
with
10 live ammunitions.The agents also confiscated glass toothers and
tinfoils in the accused-appellants car together with an allegedshabu
wrapped in cellophane. The accused was arrestedwithout search and
arrest warrants.Thus, two information were filed against theaccused, one
in violation of the Dangerous Drugs Act (RA6452) and violation to
PD1866 or the Illegal Possession of firearms and ammunitions.The
accused assails the validity of his arrest and hissubsequent convictions for
two crimes.Issue:Was the warrantless arrest valid?Held:Yes. It is
valid.Rules of Court and Jurisprudence recognizeexceptional cases
where an arrest may be effected withoutwarrant, to wit;(a) In the

presence of a peace officer, the person to bearrested has committed, is


actually committing, or isattempting to commit an offense(b) When an
offense has in fact just been committedand the arresting officer has
personal knowledge of facts indicating that the person to be arrested
hascommitted it.(c) When the accused is a fugitive from law
whileserving his sentence from a final conviction.In the case at bar, it is
clear that the policemen sawa gun tucked in his waist when he was
arrested. The gunis plainly visible and no corresponding license
wasfound. Thus, it is a valid search under the RRC (
Plainview doctrine)
no arrest warrant is necessary.The case also falls under the recognize
instancesprovided under the RRC (
Search incidental to lawfularrest),
thus, the police has the authority to search theaccused for dangerous
drugs or weapons that is used asproof to the commission of the offense.
The incidentaldiscovery and confiscation of the drug paraphernalia tohis
car is lawful.
Note: Bail;
Criminal Procedure:
Admission for bail does not preclude the accused toraise question on the
validity of his arrest, as long as heraised the same before he enters plea,
otherwise, it will bar him from questioning the same as he place himself
alreadyunder the jurisdiction of the court. Absence of Preliminary
investigation is not a groundto quash the information.

PITA VS CA

become unreasonable and subject to challenge. In Burgos v


Chief of Staff (133 SCRA 800) , the SC countermanded the
orders of the RTC authorizing the serach of the premises WE
Forum and Metropolitan Mail, two Metro Manila Dailies, by
reason of a defective warrant. There is a greater reason in
this case to reprobate the questioned raid, in the complete
absence of a warrant, valid or invalid. The fact that the
instant case involves an obscenity rap makes it no different
from Burgos, a political case, because speech is speech,
whether political or "obscene". The authorities must apply
for the issuance of the a search warrant from the judge , if in
their opinion, an obscenity rap is in order. They must
convince the court that the materials sought to be seized are
"obscene" and pose a clear and present danger of an evil
substantive enough to warrant State interference and action.
The judge must determine WON the same are indeed
"obscene": the question is to be resolved on a case-to-case
basis and on the judge's sound discretion. If probable cause
exist, a search warrant will issue.
G.R. No. 163087 February 20, 2006
SILAHIS INTERNATIONAL HOTEL, INC. and JOSE
MARCEL PANLILIO, Petitioners,
vs.
ROGELIO S. SOLUTA, JOSELITO SANTOS, EDNA
BERNATE, VICENTA DELOLA, FLORENTINO
MATILLA, and GLOWHRAIN-SILAHIS UNION
CHAPTER, Respondents.

FACTS: Loida Somacera (Loida), a laundrywoman of the


hotel, stayed overnight at the female locker room at the
basement of the hotel. At dawn, she heard pounding sounds
outside, she saw five men in barong tagalog whom she failed
to recognize but she was sure were not employees of the
hotel, forcibly opening the door of the union office. In the
morning, as union officer Soluta was trying in vain to open
the door of the union office, Loida narrated to him what she
had witnessed at dawn.
Soluta immediately lodged a complaint before the Security
Officer. And he fetched a locksmith. At that instant, men in
barong tagalog armed with clubs arrived and started hitting
Soluta and his companions. Panlilio thereupon instructed
Issue: W/N the search and seizure was illegal
Villanueva to force open the door, and the latter did. Once
HELD: YES. It is basic that searches and seizure may be inside, Panlilio and his companions began searching the
done only through a judicial warrant , otherwise, they office, over the objection of Babay who even asked them if
FACTS: Pursuant to the Anti-Smut Campaign of Mayor
Ramon Bagatsng, policemen seized and confiscated from
dealers, distributors, newsstand owners and peddlers along
Manila sidewalks, magazines, publications and other reading
materials believed to be obscene, pornographic, and indecent
and later burned the seized materials in public. Among the
publications seized and later burned was "Pinoy Playboy"
magazines published and co-edited by plaintiff Leo Pita.
After his injunctive relief was dismissed by the RTC and his
appeal rejected by CA, he seeks review with SC, invoking
the guaranty against unreasonable searches and seizure.

they had a search warrant. A plastic bag was found


containing marijuana flowering tops.
As a result of the discovery of the presence of marijuana in
the union office and after the police conducted an
investigation of the incident, a complaint against the 13
union officers was filed before the Fiscals Office of Manila.
RTC acquitted the accused. On appeal, the CA affirmed with
modification the decision of the trial court.

damages to herein individual respondents in accordance with


the earlier-quoted pertinent provision of Article 32, in
relation to Article 2219(6) and (10) of the Civil Code which
provides:

HELD:
1. No. The case at bar assumes a peculiar character since the
evidence sought to beexcluded was primarily discovered and
obtained by a private person, acting in a private capacity and
without the intervention and participation of State
Art. 2219. Moral damages may be recovered in the following authorities.Under the circumstances, can accused/appellant
and analogous cases, among others, (6) Illegal search and
validly claim that his constitutionalright against
(10) Acts and action referred to in Articles 21, 26, 27, 28, 29, unreasonable searches and seizure has been violated. Stated
30, 32, 34 and 35.
otherwise,may an act of a private individual, allegedly in
ISSUE: Whether respondent individual can recover damages
violation of appellant's constitutionalrights, be invoked
for violation of constitutional rights.
DECISION: Denied.
against the State. In the absence of governmental
interference, theliberties guaranteed by the Constitution
RULING: Article 32, in relation to Article 2219(6) and (10) PEOPLE OF THE PHILIPPINES, plaintiff-appellee vs
cannot be invoked against the State. It wasMr. Job Reyes, the
of the Civil Code, allows so.
proprietor of the forwarding agency, who made
ANDRE MARTI, accused-appellant
search/inspectionof the packages. Said inspection was
. G.R.No.81561 January 18, 1991
ART. 32. Any public officer or employee, or any private
reasonable and a standard operating procedureon the part of
FACTS:
individual, who directly or indirectly obstructs, defeats,
Accused-appellant went to a forwarding agency to send four Mr. Reyes as a precautionary measure before delivery of
violates or in any manner impedes or impairs any of the
packages tothe Bureau of Customs or the Bureau of Posts.
packages to a friend inZurich. Initially, the accused was
following rights and liberties of another person shall be
Second, the mere presence of the NBIagents did not convert
asked by the proprietress if the packages can beexamined.
liable to the latter for damages: x x x x
However, he refused. Before delivering said packages to the the reasonable search effected by Reyes into a
Bureau of Customs and the Bureau of Posts, the husband of warrantlesssearch and seizure proscribed by the Constitution.
In the present case, petitioners had, by their own claim,
the proprietress opened said boxes for final inspection. From Merely to observe and look at thatwhich is in plain sight is
already received reports in late 1987 of illegal activities and that inspection, included in the standard operating procedure not a search. Having observed that which is open, where
Maniego conducted surveillance. Yet, in the morning of
andout of curiosity, he took several grams of its contents.He notrespass has been committed in aid thereof, is not search.
January 11, 1988, petitioners and their companions barged
2.
brought a letter and the said sample to the National Bureau
into and searched the union office without a search warrant, of Investigation. When the NBI was informed that the rest of
despite ample time for them to obtain one.
No. The law enforcers testified that accused/appellant was
the shipment was still in his office, three agents went back
informed of hisconstitutional rights. It is presumed that they
with him. In their presence, the husband totally opened the
The course taken by petitioners and company stinks in
have regularly performed their duties(See. 5(m), Rule 131)
packages. Afterwards,the NBI took custody of said
illegality. Petitioners violation of individual respondents
and their testimonies should be given full faith and
packages. The contents, after examination by
constitutional right against unreasonable search thus
credence,there being no evidence to the contrary.3.
forensicchemists, were found to be marijuana flowering
furnishes the basis for the award of damages under Article
tops.The appellant, while claiming his mail at the Central
32 of the Civil Code. For respondents, being the lawful
Post Office, was invited by theagents for questioning. Later
No. Appellant signed the contract as the owner and shipper
occupants of the office had the right to raise the question of on, the trial court found him guilty of violation of
thereof giving moreweight to the presumption that things
validity of the search and seizure.
theDangerous Drugs Act.
which a person possesses, or exercises acts of ownership
over, are owned by him (Sec. 5 [j], Rule 131). At this point,
ISSUES:
Article 32 speaks of an officer or employee or person
1.Whether or not the items admitted in the searched illegally appellant istherefore estopped to claim otherwise.
"directly or indirectly" responsible for the violation of the
searched and seized.2.
constitutional rights and liberties of another. Hence, it is not
G.R. No. 185011
December 23, 2009
the actor alone who must answer for damages under Article Whether or not custodial investigation was not properly
32; the person indirectly responsible has also to answer for
applied.3.
PEOPLE OF THE PHILIPPINES, Plaintiff-Appellee,
the damages or injury caused to the aggrieved party. Such
vs.
being the case, petitioners, together with Maniego and
Whether or not the trial court did not give credence to the
SPO3 SANGKI ARA y MIRASOL, MIKE TALIB y
Villanueva, the ones who orchestrated the illegal search, are explanation of the appellanton how said packages came to
jointly and severally liable for actual, moral and exemplary
his possession.

MAMA, and JORDAN MUSA y BAYAN, AccusedAppellants.


DECISION

Court, the above-named accused, without being authorized


by law, willfully, unlawfully and consciously had in his
possession and control one (1) plastic sachet of
Methamphetamine Hydrochloride or "shabu," weighing
0.3559 gram, which is a dangerous drug.

VELASCO, JR., J.:


CONTRARY TO LAW.2
This is an appeal from the December 13, 2007 Decision of
the Court of Appeals (CA) in CA-G.R. CR-H.C. No. 00025B
entitled People of the Philippines v. SPO3 Sangki Ara y
Mirasol, Mike Talib y Mama, Jordan Musa y Bayan, which
affirmed the Decision of the Regional Trial Court (RTC),
Branch 9 in Davao City, convicting accused-appellants of
violation of Republic Act No. (RA) 9165 or the
Comprehensive Dangerous Drugs Act of 2002.
The Facts
Three Informations charged accused-appellants Sangki Ara,
Mike Talib, and Jordan Musa, as follows:
Criminal Case No. 51,471-2002 against Ara
That on or about December 20, 2002, in the City of Davao,
Philippines and within the jurisdiction of this Honorable
Court, the above-named accused, without being authorized
by law, willfully, unlawfully and consciously traded,
transported and delivered 26.6563 grams of
Methamphetamine Hydrochloride or "shabu," which is a
dangerous drug, with the aggravating circumstance of
trading, transporting and delivering said 26.6563 grams of
"shabu" within 100 meters from [the] school St. Peter's
College of Toril, Davao City.
CONTRARY TO LAW.1
Criminal Case No. 51,472-2002 against Talib
That on or about December 20, 2002, in the City of Davao,
Philippines and within the jurisdiction of this Honorable

Criminal Case No. 51,473-2002 against Musa


That on or about December 20, 2002, in the City of Davao,
Philippines, and within the jurisdiction of this Honorable
Court, the above-mentioned accused, without being
authorized by law, willfully, unlawfully and consciously had
in his possession and control five (5) big plastic sachet[s] of
Methamphetamine Hydrochloride or "shabu" weighing
14.2936 grams, which is a dangerous drug.
CONTRARY TO LAW.3
During their arraignment, accused-appellants all gave a "not
guilty" plea.
Version of the Prosecution
At the trial, the prosecution presented the following
witnesses: Forensic Chemist Noemi Austero, PO2 Ronald
Lao, SPO1 Bienvenido Furog, PO1 Enrique Ayao, Jr., SPO4
Rodrigo Mallorca, and PO2 Jacy Jay Francia.
In the morning of December 20, 2002, a confidential
informant (CI) came to the Heinous Crime Investigation
Section (HCIS) of the Davao City Police Department and
reported that three (3) suspected drug pushers had contacted
him for a deal involving six (6) plastic sachets of shabu. He
was instructed to go that same morning to St. Peter's College
at Toril, Davao City and look for an orange Nissan Sentra
car.4

Police Chief Inspector Fulgencio Pavo, Sr. immediately


formed a buy-bust team composed of SPO3 Reynaldo
Capute, SPO4 Mario Galendez, SPO3 Antonio Balolong,
SPO2 Arturo Lascaos, SPO2 Jim Tan, SPO1 Rizalino
Aquino, SPO1 Bienvenido Furog, PO2 Vivencio Jumawan,
Jr., PO2 Ronald Lao, and PO1 Enrique Ayao, Jr., who would
act as poseur-buyer.5
The team proceeded to the school where PO1 Ayao and the
CI waited by the gate. At around 8:45 a.m., an orange Nissan
Sentra bearing plate number UGR 510 stopped in front of
them. The two men approached the vehicle and the CI talked
briefly with an old man in the front seat. PO1 Ayao was then
told to get in the back seat as accused-appellant Mike Talib
opened the door. The old man, later identified as accusedappellant SPO3 Ara, asked PO1 Ayao if he had the money
and the latter replied in the positive. Ara took out several
sachets with crystalline granules from his pocket and handed
them to PO1 Ayao, who thereupon gave the pre-arranged
signal of opening the car door. The driver of the car, later
identified as accused-appellant Jordan Musa, tried to drive
away but PO1 Ayao was able to switch off the car engine in
time. The back-up team appeared and SPO1 Furog held on to
Musa while PO2 Lao restrained Talib. PO1 Ayao then asked
Ara to get out of the vehicle.6
Recovered from the group were plastic sachets of white
crystalline substance: six (6) big sachets, weighing 26.6563
grams, from Ara by PO1 Ayao; five (5) big sachets, weighing
14.2936 grams, from Musa by SPO1 Furog; and a small
sachet, weighing 0.3559 gram, from Talib by PO2 Lao.7
The three suspects were brought to the HCIS and the seized
items indorsed to the Philippine National Police (PNP)
Crime Laboratory for examination. Forensic Chemist
Austero, who conducted the examination, found that the
confiscated sachets all tested positive for shabu. 8
Version of the Defense

The defense offered the sole testimony of Ara, who said that
he had been a member of the PNP for 32 years, with a
spotless record. On December 20, 2002, SPO3 Ara was in
Cotabato City, at the house of his daughter Marilyn, wife of
his co-accused Musa. He was set to go that day to the
Ombudsman's Davao City office for some paperwork in
preparation for his retirement on July 8, 2003. He recounted
expecting at least PhP 1.6 million in retirement benefits. 9
Early that morning, past three o'clock, he and Musa headed
for Davao City on board the latter's car. As he was feeling
weak, Ara slept in the back seat.
Upon reaching Davao City, he was surprised to see another
man, Mike Talib, in the front seat of the car when he woke
up. Musa explained that Talib had hitched a ride on a bridge
they had passed.10

The dispositive portion of the RTC Decision reads:


WHEREFORE, premised on the foregoing the Court finds
the following:

LIFE IMPRISONMENT and FINE of FOUR HUNDRED


THOUSAND PESOS (PhP 400,000) with all the accessory
penalties corresponding thereto.
SO ORDERED.12

In Criminal Case No. 51,471-2002, the accused herein


SANGKI ARA Y MASOL, Filipino, 55 years old, widower,
a resident of Kabuntalan, Cotabato City, is hereby found
GUILTY beyond reasonable doubt, and is CONVICTED of
the crime of violation of Sec. 5, 1st paragraph of Republic
Act 9165. He is hereby imposed the DEATH PENALTY and
FINE of TEN MILLION PESOS (PhP 10,000,000) with all
the accessory penalties corresponding thereto, including
absolute perpetual disqualification from any public office, in
view of the provision of section 28 of RA 9165 quoted
above.

As the death penalty was imposed on Ara, the case went on


automatic review before this Court. Conformably with
People v. Mateo,13 we, however, ordered the transfer of the
case to the CA.
The Ruling of the Appellate Court

Contesting the RTC Decision, accused-appellants filed


separate appeals before the CA. Talib claimed that it was
erroneous for the trial court to have used the complaining
witnesses' affidavits as basis for ruling that their arrest was
valid. He also cited as erroneous the trial court's refusal to
Since
the
prosecution
proved
beyond
reasonable
doubt
that
When they arrived in Toril, Ara noticed the car to be
the crime was committed in the area which is only five (5) to rule that the prosecution's evidence was inadmissible. Lastly,
overheating, so they stopped. Ara did not know that they
were near St. Peter's College since he was not familiar with six (6) meters away from the school, the provision of section he questioned the failure of the buy-bust team to follow the
requirements of RA 9165 on proper inventory of seized
5 paragraph 3 Article II of RA 9165 was applied in the
the area. Talib alighted from the car and Ara transferred to
drugs.
imposition of the maximum penalty against the herein
the front seat. While Talib was getting into the back seat,
accused.
PO1 Ayao came out of nowhere, pointed his .45 caliber
Ara and Musa filed a joint brief, alleging the following: (1)
pistol at Ara even if he was not doing anything, and ordered
the trial court erred in denying the Motion to Suppress
In Criminal Case No. 51,472-2002, the accused herein
him to get off the vehicle. He saw that guns were also
and/or exclude illegally obtained evidence; (2) the trial court
MIKE
TALIB
y
MAMA,
Filipino,
of
legal
age,
single
and
a
pointed at his companions. As the group were being arrested,
erred in denying the Demurrer to Evidence; (3) the trial court
resident of Parang, Cotabato, is found GUILTY beyond
he told PO1 Ayao that he was also a police officer. Ara
failed to consider that the criminal informations did not
insisted that he was not holding anything and that the shabu reasonable doubt, and is CONVICTED of the crime of
allege conspiracy among the accused; and (4) the trial court
violation of Sec. 11, 3rd paragraph, Article II of Republic
taken from him was planted. He asserted that the only time
11
erred in ruling that the "intercept operation" was valid.
Act 9165. He is hereby imposed a penalty of Imprisonment
he saw shabu was on television.
of SIXTEEN (16) YEARS and a fine of THREE HUNDRED
The CA affirmed the trial court's decision with some
THOUSAND PESOS (PhP 300,000) with all the accessory
The Ruling of the Trial Court
modifications on the penalty imposed. It ruled that a
penalties corresponding thereto.
majority of the errors raised in the appeal referred to
The RTC pronounced accused-appellants guilty of the crimes
technicalities in the conduct of buy-bust operations that did
charged. In its Decision dated March 1, 2003, the trial court In Criminal Case No. 51,473-2002 the accused herein
not invalidate the police officers' actions. On the issue of the
JORDAN
MUSA
Y
BAYAN,
Filipino,
30
years
old,
married
held that the prosecution was able to establish the quantum
evidence presented, the CA held that the presumption that
and a resident of Cotabato City, is hereby found GUILTY
of proof showing the guilt of accused-appellants beyond
police officers performed their duties in a regular manner
beyond reasonable doubt and is CONVICTED of the crime
reasonable doubt. It further ruled that the "intercept
for Violation of Sec. 11, 1st paragraph, Article II of Republic was not overturned.
operation" conducted by the buy-bust team was valid.
Act No. 9165. He is hereby sentenced to suffer a penalty of

The appellate court resolved the issue of the validity of the


buy-bust operation by stating that the law requires no
specific method of conducting such an operation. It ruled
that to require a warrant of arrest would not accomplish the
goal of apprehending drug pushers in flagrante delicto. The
CA's Decision emphasized that all the elements necessary
for the prosecution of illegal sale of drugs were established.
The fallo of the December 13, 2007 CA Decision reads:

III
Whether the Court of Appeals erred in refusing to consider
the suppression or exclusion of evidence
IV
Whether the Court of Appeals erred in not holding that the
prosecution miserably failed to prove the guilt of the accused
beyond reasonable doubt

WHEREFORE, premises foregoing, the appeal is hereby


Talib also raises the following grounds for his acquittal:
DISMISSED and the appealed March 1, 2003 Decision is
hereby AFFIRMED subject to the modification insofar as the
I
death penalty imposed upon accused SPO3 Sangki Ara is
concerned. Accordingly, his penalty is hereby reduced to life
Whether the arrest of Talib was illegal and the evidence
imprisonment pursuant to Republic Act No. 9346.
confiscated from him illegally obtained
14
SO ORDERED.
II
On December 17, 2008, this Court required the parties to
Whether the police officers who conducted the illegal search
submit supplemental briefs if they so desired. The parties,
and arrest also deliberately failed and/or violated the
save for Musa, manifested their willingness to forego the
provisions of RA 9165
filing of additional briefs.
The Issues
Reiterating the matters raised before the CA, accusedappellants alleged the following:
I

III
Whether the trial court erred in ruling that the "intercept
operation" was valid
Accused-appellant Musa also avers that the CA erred in
convicting him since the prosecution failed to prove the
corpus delicti of the offense charged.
The Ruling of this Court
What are mainly raised in this appeal are (1) whether the
buy-bust conducted was valid; (2) whether the crimes of
illegal sale and illegal possession of drugs were sufficiently
established; and (3) whether the chain of custody over the
shabu was unbroken.
Warrantless Arrest and Seizure Valid
In calling for their acquittal, accused-appellants decry their
arrest without probable cause and the violation of their
constitutional rights. They claim that the buy-bust team had
more than a month to apply for an arrest warrant yet failed to
do so.

III
Whether the testimonies of the prosecution's witnesses and
their respective affidavits were gravely inconsistent
Ara and Musa additionally raise the following issues:

I
Whether the Court of Appeals erred in holding that the arrest
of the accused-appellants was valid based on the affidavits of
Whether the trial court erred in denying the Demurrer to
the complaining witnesses
Evidence
II
II
Whether the Court of Appeals erred in disregarding the
Whether the trial court failed to consider that the criminal
apparent defects and inconsistencies in the affidavits of the
informations did not allege conspiracy among the accused
complaining witnesses

Owing to the special circumstances surrounding the drug


trade, a buy-bust operation has long been held as a legitimate
method of catching offenders. It is a form of entrapment
employed as an effective way of apprehending a criminal in
the act of commission of an offense.15 We have ruled that a
buy-bust operation can be carried out after a long period of
planning. The period of planning for such operation cannot
be dictated to the police authorities who are to undertake
such operation.16 It is unavailing then to argue that the
operatives had to first secure a warrant of arrest given that
the objective of the operation was to apprehend the accusedappellants in flagrante delicto. In fact, one of the situations
covered by a lawful warrantless arrest under Section 5(a),
Rule 113 of the Rules of Court is when a person has
committed, is actually committing, or is attempting to

commit an offense in the presence of a peace officer or


private person.
It is erroneous as well to argue that there was no probable
cause to arrest accused-appellants. Probable cause, in
warrantless searches, must only be based on reasonable
ground of suspicion or belief that a crime has been
committed or is about to be committed. There is no hard and
fast rule or fixed formula for determining probable cause, for
its determination varies according to the facts of each case. 17
Probable cause was provided by information gathered from
the CI and from accused-appellants themselves when they
instructed PO1 Ayao to enter their vehicle and begin the
transaction. The illegal sale of shabu inside accusedappellants' vehicle was afterwards clearly established. Thus,
as we have previously held, the arresting officers were
justified in making the arrests as accused-appellants had just
committed a crime when Ara sold shabu to PO1 Ayao.18 Talib
and Musa were also frisked for contraband as it may be
logically inferred that they were also part of Ara's drug
activities inside the vehicle. This inference was further
strengthened by Musa's attempt to drive the vehicle away
and elude arrest.
Moreover, the trial court correctly denied the Motion to
Suppress or Exclude Evidence. We need not reiterate that the
evidence was not excluded since the buy-bust operation was
shown to be a legitimate form of entrapment. The pieces of
evidence thus seized therein were admissible. As the
appellate court noted, it was within legal bounds and no
anomaly was found in the conduct of the buy-bust operation.
There is, therefore, no basis for the assertion that the trial
court's order denying said motion was biased and committed
with grave abuse of discretion.
Prosecution Established Guilt Beyond Reasonable Doubt
For the successful prosecution of the illegal sale of shabu,
the following elements must be established: (1) the identity
of the buyer and the seller, the object of the sale, and the

consideration; and (2) the delivery of the thing sold and its
payment. What is material is the proof that the transaction or
sale actually took place, coupled with the presentation in
court of the corpus delicti as evidence.19 All these requisites
were met by the prosecution.

shabu. The shabu was then presented before the trial court.
The non-presentation of the marked money may, thus, be
overlooked as a peripheral matter.

Talib further contends that it is incredible that a shabu


transaction would be carried out in a very open and public
In contrast, Ara, the sole defense witness, could only proffer place. Contrary to Talib's claim, however, judicial experience
the weak defenses of denial and alibi. He expressed surprise has shown that drug transactions have been conducted
at having Talib in his car and claimed he was framed and that without much care for an inconspicuous location.
the shabu confiscated from him was planted. According to
Thus, we observed in People v. Roldan:
the trial court, however, Ara's lying on the witness stand
"was so intense as he tried very hard in vain to win the
Drug pushing when done on a small level x x x belongs to
Court's sympathy."20
that class of crimes that may be committed at anytime and at
any place. After the offer to buy is accepted and the
Given the prosecution's evidence, we rule that the
exchange is made, the illegal transaction is completed in a
presumption of regularity in the performance of official
few minutes. The fact that the parties are in a public place
duties has not been overturned. The presumption remains
and in the presence of other people may not always
because the defense failed to present clear and convincing
discourage them from pursuing their illegal trade these
evidence that the police officers did not properly perform
21
their duty or that they were inspired by an improper motive. factors may even serve to camouflage the same. Hence, the
Court has sustained the conviction of drug pushers caught
Ara could not explain why his fellow police officers, who
selling illegal drugs in a billiard hall, in front of a store,
did not know him prior to his arrest, would frame him for
along a street at 1:45 p.m., and in front of a house.23
such a serious offense.
It is also argued as impossible to believe that even if there
was already a deal between the informant and accusedLikewise questioned by the defense in the affidavits of the
appellants, it was the apprehending police officer who acted
police officers was the allegation that there was a legitimate as the buyer and that he requested to see the shabu first
buy-bust operation. No marked money was presented to back before showing the money. These claims by Talib are
up the police officers' claims. This argument lacks basis,
similarly undeserving of consideration. First, there is no
however. There are requirements that must be complied with uniform method by which drug pushers and their buyers
in proving the legitimacy of drug buy-bust operations.
operate. Second, the choice of effective ways to apprehend
Nevertheless, this Court has ruled that presentation of the
drug dealers is within the ambit of police authority. Police
marked money used is not such a requirement. In the
officers have the expertise to determine which specific
prosecution for the sale of dangerous drugs, the absence of
approaches are necessary to enforce their entrapment
marked money does not create a hiatus in the evidence for
operations.24 Third, as long as they enjoy credibility as
the prosecution, as long as the sale of dangerous drugs is
witnesses, the police officers' account of how the buy-bust
adequately proved and the drug subject of the transaction is operation transpired is entitled to full faith and credit. 25
presented before the court.22 In the instant case, the police
Lastly, these arguments are merely incidental and do not
officers' testimonies adequately established the illegal sale of
Validity of Buy-Bust Operation

affect the elements of the crime which have been, in the


instant case, sufficiently established.

Atty. Javines

evidence to sustain the indictment or to support a verdict of


guilt.

Q Ayao did not arrest [Ara] inside the vehicle?


Talib also alleges that during his testimony, SPO1 Furog was
not certain as to the reason he was apprehending Musa.
Another claim is that SPO1 Furog, when examined by the
prosecutor and two different defense lawyers, allegedly
made relevant inconsistencies in his testimony. The pertinent
exchange reads:

A Only I rushed to the vehicle. I don't know if he directly


arrested him when he saw the substance and [got] out of the
vehicle but I saw him get out from the vehicle.27

Allegation of Conspiracy in Information Not Necessary

The alleged inconsistencies in SPO1 Furog's "reason for


apprehending Musa" are, however, insignificant and do not
merit much consideration as well. The questioned parts in
Direct Examination of SPO1 Furog:
the testimony of SPO1 Furog do not dent the totality of
Prosecutor Weis:
evidence against accused-appellants. To repeat, the elements
of the crime of illegal sale of drugs and illegal possession of
Q What was your basis for stopping [Musa] from letting the drugs were both sufficiently established. Although SPO1
car go?
Furog was not categorical in explaining his basis for
apprehending Musa, the arrest of the latter must be
A I made him [stop] the car[.] [W]e [had] to check them first considered as part of a legitimate buy-bust operation which
because I think Ayao saw [that] Ara [had] the suspected
was consummated. Musa's arrest came after the pre-arranged
shabu.
signal was given to the back-up team and this served as basis
for the police officers to apprehend all those in the vehicle,
Cross-Examination of SPO1 Furog:
including Musa.
Atty. Estrada
Q When you arrested Musa as you said, it was because he
attempted to drive the car away, that was it?
A The most, when SPO3 Sangki Ara told us that he was a
PNP member and when we saw the substances from the two
of them first.
xxxx
Q You are referring to Musa and Ara?
A Yes sir.26
xxxx

Here, the trial court found competent and sufficient evidence


to support a conviction of all three accused-appellants. We
see no reason to overturn the trial court's finding.

We find no merit in accused-appellants' insistence that


conspiracy should have been alleged in the separate
Informations indicting them. We agree with the appellate
court, which succinctly stated that conspiracy was not
alleged "precisely because they were charged with different
offenses for the distinct acts that each of them committed.
One's possession of an illegal drug does not need to be
conspired by another who, on his part, also possessed an
illegal drug."30 The three separate indictments against Ara,
Musa, and Talib do not need to allege conspiracy, for the act
of conspiring and all the elements of the crime must be set
forth in the complaint or information only when conspiracy
is charged as a crime.31

Denial of Demurrer to Evidence

Requirements of RA 9165 on Proper Inventory

Although alleged by accused-appellants Ara and Musa, no


reason was given in the appeal as to why the trial court erred
in denying their Demurrer to Evidence. Whatever their basis
may be, an action on a demurrer or on a motion to dismiss
rests on the sound exercise of judicial discretion. 28 In Gutib
v. CA,29 we explained that:

Musa contends that since the markings on the seized items


were only made at the police station, there is a great
possibility that these were replaced. The result, he argues,
would be a lack of guarantee that what were inventoried and
photographed at the crime laboratory were the same
specimens confiscated from the accused.

A demurrer to evidence is an objection by one of the parties


in an action, to the effect that the evidence which his
adversary produced is insufficient in point of law, whether
true or not, to make out a case or sustain the issue. The party
demurring challenges the sufficiency of the whole evidence
to sustain a verdict. The court, in passing upon the
sufficiency of the evidence raised in a demurrer, is merely
required to ascertain whether there is competent or sufficient

As recently highlighted in People v. Cortez32 and People v.


Lazaro, Jr.,33 RA 9165 and its subsequent Implementing
Rules and Regulations (IRR) do not require strict compliance
as to the chain of custody rule. The arrest of an accused will
not be invalidated and the items seized from him rendered
inadmissible on the sole ground of non-compliance with Sec.
21, Article II of RA 9165. We have emphasized that what is
essential is "the preservation of the integrity and the

evidentiary value of the seized items, as the same would be


utilized in the determination of the guilt or innocence of the
accused."

Musa were marked Exhibits "A-1" to "A-5," while the sachet hydrochloride or "shabu" is ten (10) grams or more but less
seized from Talib was marked Exhibit "B." The six (6)
than fifty (50) grams;
sachets taken from Ara were marked Exhibits "B1-B6."
Musa was sentenced to life imprisonment and a fine of PhP
Briefly stated, non-compliance with the procedural
We are, thus, satisfied that the prosecution was able to
400,000.
requirements under RA 9165 and its IRR relative to the
preserve the integrity and evidentiary value of the shabu in
Criminal Case No. 51,471-2002 against Ara
custody, photographing, and drug-testing of the apprehended all three criminal cases against accused-appellants.
persons, is not a serious flaw that can render void the
The crime of illegal sale of shabu is penalized by Sec. 5, Art.
The rest of the arguments interposed are evidently without
seizures and custody of drugs in a buy-bust operation. 34
11 of RA 9165:
merit and do not warrant discussion.
The chain of custody in the instant case did not suffer from
SEC. 5. Sale, Trading, Administration, Dispensation,
Penalties Imposed
serious flaws as accused-appellants argue. The recovery and
Delivery, Distribution and Transportation of Dangerous
handling of the seized drugs showed that, as to Ara, first,
Criminal Case No. 51,472-2002 against Talib
Drugs and/or Controlled Precursors and Essential
PO1 Ayao recovered six plastic sachets of white crystalline
Chemicals. - The penalty of life imprisonment to death and a
substance from Ara and marked them with both his and Ara's
The crime of illegal possession of drugs is punishable by
fine ranging from Five Hundred Thousand Pesos
initials. Second, the sachets were likewise signed by
Sec. 11 of RA 9165, as follows:
(P500,000.00) to Ten Million Pesos (P10,000,000.00) shall
property custodian PO3 Pelenio. Third, PO1 Ayao signed a
be imposed upon any person, who, unless authorized by law,
Request for Laboratory Examination then personally
Sec. 11. Possession of Dangerous Drugs. - x x x
shall sell, trade, administer, dispense, deliver, give away to
delivered the sachets to the PNP Crime Laboratory for
another, distribute, dispatch in transit or transport any
examination. Fourth, SPO4 Mallorca then received the
xxxx
dangerous drug, including any and all species of opium
sachets at the crime laboratory.
poppy regardless of the quantity and purity involved, or shall
3)
Imprisonment
of
twelve
(12)
years
and
one
(1)
day
to
act as a broker in any of such transactions.
As to Musa, first, SPO1 Furog seized the sachets from Musa
and marked each with his own initials. Second, an Inventory twenty (20) years and a fine ranging from Three hundred
thousand pesos (P300,000.00) to Four hundred thousand
The same section contains the following provision:
of Property Seized was then made by SPO4 Galendez.
Lastly, SPO1 Furog later submitted a Request for Laboratory pesos (P400,000.00), if the quantities of dangerous drugs are
less than five (5) grams of x x x methamphetamine
If the sale, trading, administration, dispensation, delivery,
Examination of the five (5) sachets weighing a total of
hydrochloride
x
x
x.
distribution or transportation of any dangerous drug and/or
14.2936 grams to the PNP Crime Laboratory.
controlled precursor and essential chemical transpires within
Talib
was
sentenced
to
imprisonment
of
sixteen
(16)
years
one hundred (100) meters from the school, the maximum
As to Talib, first, PO2 Lao seized a small sachet from Talib
penalty shall be imposed in every case.
during the buy-bust operation. Second, PO2 Lao delivered a and a fine of PhP 300,000.
Request for Laboratory Examination of one (1) sachet of
Criminal Case No. 51,473-2002 against Musa
Since the sale of shabu was within five (5) to six (6) meters
suspected shabu weighing 0.3559 gram. Third, SPO4
from St. Peter's College, the maximum penalty of death
Mallorca also received the items at the PNP Crime
The
provision
Musa
was
charged
of
violating
provides
the
should be imposed on Ara. Pursuant to RA 9346 or "An Act
Laboratory.
following penalty:
Prohibiting the Imposition of Death Penalty in the
Philippines," however, only life imprisonment and a fine
Forensic Chemist Noemi Austero's examination of the
(1)
Life
imprisonment
and
a
fine
ranging
from
Four
hundred
shall be meted on him.
sachets confiscated from all accused-appellants showed that
thousand
pesos
(P400,000.00)
to
Five
hundred
thousand
these were positive for shabu. During trial, the seized items
pesos (P500,000.00), if the quantity of methamphetamine
were identified in court. The five (5) sachets taken from

Ara was sentenced to life imprisonment and a fine of PhP


10,000,000. He, however, is no longer eligible for parole.
What distinguishes this case from others is that one of the
accused-appellants was a police officer himself who should
have known better than to break the law he was duty-bound
to enforce. What is more, he is charged with the crime of
selling illegal drugs, an offense so horrendous for destroying
the lives of its victims and their families that the penalty of
death used to be imposed on its perpetrators. No one could
have been more deserving of such a punishment than
someone who should be enforcing the law but caught
pushing drugs instead. As it was, the death penalty was
indeed originally imposed on SPO3 Ara, who had been in the
service for more than 30 years.1avvphi1

G.R. No. 175604

April 10, 2008

THE PEOPLE OF THE PHILIPPINES, appellee,


vs.
SALVADOR PEAFLORIDA, JR., Y CLIDORO,
appellant.
DECISION
TINGA, J.:
Subject of this appeal is the Decision1 of the Court of
Appeals in CA-G.R. CR No. 01219, dated 31 July 2006,
affirming in toto the judgment2 of the Regional Trial Court
of Camarines Sur, Branch 30, in Criminal Case No. T-1476.
The trial court found appellant Salvador Peaflorida y
Clidoro guilty of transporting marijuana and sentenced him
to suffer the penalty of reclusion perpetua and to pay a fine
of one million pesos.

The ill effects of the use of illegal drugs are too repulsive
and shocking to enumerate. Thus, once the charges of sale
and possession of said drugs are established in cases such as
this, any errors or technicalities raised by the suspects should
not be allowed to invalidate the actions of those involved in The Information against appellant reads:
curtailing their illegal activities. The punishments given to
That on or about the 7th day of June, 1994, in the
drug pushers should serve as deterrent for others not to
afternoon thereat, at Barangay Huyon-huyon,
commit the same offense. No price seems high enough for
Municipality of Tigaon, Province of Camarines Sur,
drug dealers to pay; it is just unfortunate that the penalty of
Philippines and within the jurisdiction of this
death can no longer be imposed because it has been
Honorable Court, the above-named accused, with
abolished.
intent to sell, possess and to deliver with the use of a
bicycle, did then and there, willfully, unlawfully and
As the penalties meted out to all three accused-appellants are
feloniously have in his possession, control and
within the range provided by RA 9165, we affirm the CA's
custody, [o]ne bundle estimated to be one (1) kilo
sentence.
more or less, of dried marijuana leaves (Indian
Hemp) without the necessary license, permit or
WHEREFORE, the appeal is DENIED. The CA Decision in
authority to sell, administer, deliver, give away to
CA-G.R. CR-H.C. No. 00025B entitled People of the
another, distribute, dispatch in transit or transport
Philippines v. SPO3 Sangki Ara y Mirasol, Mike Talib y
any prohibited drug from a competent officer as
Mama, Jordan Musa y Bayan is AFFIRMED with the
required by law.
modification that accused-appellant Sangki Ara is not
eligible for parole.
ACTS CONTRARY TO LAW.3
SO ORDERED.

Upon arraignment, appellant pleaded not guilty. Trial ensued.


Two police officers and one forensic chemist testified for the
prosecution.
SPO3 Vicente Competente (Competente) narrated that in his
capacity as chief of the Investigation and Operation Division
of the Philippine National Police (PNP) station in Tigaon,
Camarines Sur, that he received a tip from an asset that a
bundle of marijuana was being transported by appellant to
Huyon-huyon from another barangay in Tigaon, Camarines
Sur.4 Major Domingo Agravante (Agravante), chief of police
of Tigaon, then organized a team composed of Competente
as team leader, SPO2 Ricardo Callo (Callo), SPO1 Portugal,
PO3 Pillos and PO2 Edgar Latam. The team boarded the
police mobile car and proceeded to Sitio Nasulan in
Barangay Huyon-huyon.5 They overtook appellant who was
on a bicycle. The police officers flagged appellant down and
found marijuana wrapped in a cellophane and newspaper
together with other grocery items. The amount of P1550.00
was also found in appellant's possession. The police officers
confiscated these items and took photographs thereof.
Appellant was then brought to the headquarters where he
was booked. 6
Callo, who was the chief intelligence officer of Tigaon PNP,
recounted that at around 1:00 p.m. on 7 June 1994, he was
called by Competente and was briefed about the operation.
While they were in Nasulan, the members of the police team
caught a man riding a bicycle who turned out to be appellant.
Callo saw the marijuana wrapped in a cellophane and
newspaper in the bicycle of appellant so the latter was
brought to the police headquarters and turned over to the
desk officer. 7
Major Lorlie Arroyo (Arroyo), a forensic chemist at the PNP
Crime Laboratory Regional Office No. V, was presented as
an expert witness to identify the subject marijuana leaves.
She related that after taking a representative sample from the
928-gram confiscated dried leaves, the same was tested

positive of marijuana. The findings were reflected in


Chemistry Report No. D-26-94 dated 9 June 1994.8
Appellant denied the accusations against him. Appellant,
who is a resident of Huyon-huyon, Tigaon, Camarines Sur,
testified that in the morning of 7 June 1994, he first went to
the house of Igmidio Miranda (Miranda) in Sagnay,
Camarines Sur. The latter accompanied appellant to the
house of Arnel Dadis in San Francisco, Tigaon to buy a dog.
They, however, failed to get the dog; prompting them to
leave. On their way home, they met Boyet Obias (Obias)
who requested appellant to bring a package wrapped in a
newspaper to Jimmy Gonzales (Gonzales).9 Appellant placed
it in the basket in front of his bicycle and Gonzales
proceeded to the Tiagon town proper. He and Miranda parted
ways when they reached the place. Appellant dropped by the
grocery store and the blacksmith to get his scythe. On his
way home, he was flagged down by the police and was
invited to go with them to the headquarters. Upon inspection
of the package in his bicycle, the police discovered the
subject marijuana. Appellant tried to explain that the package
was owned by Obias but the police did not believe him. He
was sent to jail.10

WHEREFORE, the accused Salvador


Peaflorida[,Jr.] is hereby sentenced to suffer the
penalty of imprisonment of reclusion perpetua and
to pay a fine of One Million (P1,000,000.00) Pesos,
with subsidiary imprisonment in accordance with
law, in case of insolvency for the fine and for him to
pay the costs.
The accused Salvador Peaflorida[,Jr.] shall be
entitled to full credit of his preventive imprisonment
if he agreed to abide with the rules imposed upon
convicted person, otherwise, he shall be entitled to
four-fifth (4/5) credit thereof.
The subject marijuana consisting of 928 grams,
possession thereof being mala prohibita, the court
hereby orders its confiscation in favor of the
Government to be destroyed in accordance with law.

his four (4) other co-police officers apprehended the


accused Salvador Peaflorida[,Jr.] on the roadside at
Nasulan, Huyon-huyon, Tigaon, Camarines Sur [,]
then riding on his bicycle and placed on the still
structure at its front, a thing wrapped in a newspaper
and found to be 928 grams of marijuana. No illmotive has been presented by the defense against the
police officers Vicente Competente and companions
by falsely testifying against the accused Salvador
Peaflorida, Jr. So, the conclusion is inevitable that
the presumption that the police officers were in the
regular performance of their duties apply. The
confiscation of the marijuana subject of the instant
case and the arrest of the accused Salvador
Peaflorida[,Jr.] by the said police officers being
lawful, having been caught in flagrante delicto, there
is no need for the warrant for the seizure of the fruit
of the crime, the same being incidental to the lawful
arrest. Rightly so, because a person caught illegally
possessing or transporting drugs is subject to the
warrantless search. Besides, object in the "plain
view" of an officer who has the right to be in the
position to have that view are subject to seizure and
may be presented as evidence.13

This court, however, hereby recommends to His


Excellency, the President of the Philippines, through
the Honorable Secretary of Justice to commute the
above penalty herein imposed, being too harsh;
accordingly, the said penalty imposed to accused
Salvador Peaflorida[,Jr] shall be six (6) years of
Miranda corroborated the testimony of appellant that the two
prision correccional, as minimum, to eight (8) years In view of the penalty imposed, the case was directly
of them went to San Francisco, Tigaon, Camarines Sur in the
and one (1) day of prision mayor, as maximum.
morning of 7 June 1994 to buy a dog. On their way back to
appealed to this Court on automatic review. Pursuant to our
the town proper of Tigaon, they met Obias who requested
decision in People v. Mateo,14 however, this case was
12
SO ORDERED.
appellant to bring a package, which Miranda thought
referred to the Court of Appeals. The appellate court
contained cookies, to Gonzales. Upon reaching the town
affirmed appellant's conviction on 31 July 2006.
11
In
convicting
appellant,
the
trial
court
lent
credence
to
the
proper, they parted ways.
testimonies of the police officers, thus:
In a Resolution15 dated 14 February 2007, the parties were
On 26 October 1998, the trial court rendered judgment
given to file their supplemental briefs, if they so desire. Both
Now going over the evidence adduced, the court is
finding appellant guilty beyond reasonable doubt of
parties manifested their intention not to file any
convinced that the accused Salvador Peaflorida[,Jr.] supplemental brief since all the issues and arguments have
transporting a prohibited drug, a violation of Section 4,
committed the offense of illegal possession of 928
Article II of Republic Act (R.A.) No. 6425, otherwise known
already been raised in their respective briefs. 16
grams of marijuana, if not, of transporting it, as
as The Dangerous Drugs Act of 1972, as amended by R.A.
charged. This is so, because it appears undisputed
No. 7659. The dispositive portion of the decision reads:
Hence, the instant case is now before this Court on automatic
that on June 7, 1994, at about 1:00 o'clock in the
review.
afternoon police officers Vicente Competente and

In assailing his conviction, appellant submits that there is


doubt that he had freely and consciously possessed
marijuana. First, he claims that the alleged asset did not
name the person who would transport the marijuana to
Huyon-huyon. In view of the "vague" information supplied
by the asset, the latter should have been presented in court.
Second, upon receipt of the information from the asset, the
police officers should have first investigated and tried to
obtain a warrant of arrest against appellant, instead of
arbitrarily arresting him. Third, appellant maintains that he is
not aware of the contents of the package. Fourth, upon
arrival at the headquarters, the police did not determine the
contents and weight of the package. Fifth, appellant argues
that the findings of the forensic expert are questionable
because there is doubt as to the identity of the package
examined.17
Prefatorily, factual findings of the trial courts, including their
assessment of the witness' credibility are entitled to great
weight and respect by this Court, particularly when the Court
of Appeals affirm the findings.18 Indeed, the trial court is in
the best position to assess the credibility of witnesses since it
has observed firsthand their demeanor, conduct and attitude
under grilling examination.19 After a review of the records of
this case, we find no cogent reason to disregard this timehonored principle.
We shall retrace the series of events leading to the arrest of
appellant and resolve the issues raised by him.
Acting on an asset's tip, a police team was organized to
apprehend appellant who was allegedly about to transport
the subject marijuana. Appellant is wrong in concluding that
the asset did not name appellant. As early as 16 November
1996, appellant through counsel had already conceded in his
Memorandum20 filed with the trial court that based on the tip,
he was about to transport the contraband. It further cited
excerpts from the result of the preliminary investigation
conducted by the judge on Competente, and we quote:

Q: Did your [a]sset tell you the place and the person
or persons involved?
A: Yes[,]sir.

A: Yes[,] sir[.] That he would deliver marijuana.


Q: So, at the time that you form[ed] a team, Salvador
was nowhere to be seen, you have not seen the
shadow of Salvador?

Q: Where and who?


A: He said that marijuana is being transported from
Tigaon town to Bgy. Huyon-huyon by Salvador
Peaflorida, Jr.21

A: When the tip was given to us[,] I have not seen


him[.] [B]ut the tip is he will deliver from Tigaon
to Huyon-huyon, that is why we chased him.22
[Emphasis supplied]

Moreover, on cross-examination, the defense counsel even


assumed that according to the asset's tip it was appellant who
was assigned to deliver the contraband. And the witness
under cross-examination affirmed it was indeed appellant
who would be making the delivery according to the tip:

Prescinding from the above argument, appellant insists that


the asset should have been presented in court. He invoked
the court ruling in People v. Libag,23 wherein the nonpresentation of the informant was fatal to the case of the
prosecution. Libag cannot find application in this case. In
that case, the crime charged was the sale of shabu where the
Q: Will you inform this Honorable Court who has
informant himself was a poseur-buyer and a witness to the
given you the tip that the accused was going to
transaction. His testimony as a poseur-buyer was
deliver that marijuana[?] [W]ho is [this] person?
indispensable because it could have helped the trial court in
determining whether or not the appellant had knowledge that
A: It was a confidential tip.
the bag contained marijuana, such knowledge being an
essential ingredient of the offense for which he was
Q: Now, but [sic] on June 1 you were in your office? convicted.24 In this case, however, the asset was not present
in the police operation. The rule is that the presentation of an
A: Yes[,] sir[.] I was in the office.
informant in an illegal drugs case is not essential for
conviction nor is it indispensable for a successful
Q: Since your office is just near the Municipal Trial prosecution because his testimony would merely be
Court of Tigaon and you were given a tip that
corroborative and cumulative. Informants are generally not
Salvador Peaflorida[,Jr.] will be delivering
presented in court because of the need to hide their identity
marijuana, why did you not get a [w]arrant of
and preserve their invaluable service to the police. 25
[a]rrest?
xxx
Q: The tip that was given to you that it was
Salvador Peaflorida [who] will be dealing
marijuana on that date and according to you
Salvador was to travel from a certain town to
Tigaon, is that the tip?

Competente testified that his team caught up with appellant


who was riding a bicycle. He saw the marijuana in a package
which appellant was carrying inside his basket, thus:
Q: And so as the team leader x x x and in connection
with the instruction of Chief Domingo Agravante,
what did you do?

A: We used the mobile and proceeded to the place,


to the route where the marijuana was being
transported.

Callo also confirmed that he saw appellant transporting and


in possession of the subject marijuana:
Q: When you reached there[,] what happened next?

Q: When you said we to whom are you referring to?


A: The team.

A: We have not reached yet [sic] the Huyon-huyon


proper. [W]e are in Nasulan when we met the man
who had with him the marijuana.

Q: Were you able to go to the place as you said?


xxx
A: Yes, sir.
Q: So, upon reaching the place, [sic] what place was
that?

Q: After you talked with the person with


marijuana[,] what happened next?
A: We saw on his bicycle a wrap[ped] marijuana.

A: Sitio Nasulan, Barangay Huyon-huyon, Tigaon,


Camarines Sur.

presumption of regularity accorded to them and the lack of


motive on their part to falsely testify against appellant.
Appellant resorts to a challenge on the validity of his arrest
predicated on lack of a warrant of arrest. The OSG correctly
justifies the failure to apply for an arrest warrant because at
that point, time was of the essence in appellant's
apprehension, noting in the same breath that there is no law
requiring investigation and surveillance upon receipt of tips
from assets before conducting police operations. 28 The police
officers succinctly testified on this point when crossexamined, viz:
Q: Will you inform this Honorable Court who has
given you the tip that the accused was going to
deliver that marijuana, who is that person?

Q: Who was in possession of that?


A: It was a confidential tip.

Q: And upon reaching the place together with the


other member of the team, what did you find if you
found any?
A: We overtook our suspect while riding in a bicycle
and we stopped him.
Q: And did the suspect stop?

A: Salvador Peaflorida[,] Jr.


Q: Now, but [sic] on June 1 you were in your office?
Q: How is that person related to the accused in this
case now?
A: He is the one, sir.
Q: Kindly describe to us the marijuana that you are
able to tell that it was marijuana?

A: Yes[,] sir.
Q: Tell us the name of your suspect?
A: Salvador Peaflorida[,] Jr. y Clidoro.
Q: And after stopping the accused in this case, what
else did you do[,] if any[,] together with the team?
A: When we saw the marijuana and other groceries
in his bicycle we invited him to the headquarters.26

A: It was wrapped on [cellophane] and newspaper.


We saw the edges of the marijuana.
Q: For the [record], kindly describe to us the edges
of the marijuana[;] its appearance and color.
A: It was like a shape of ream of coupon bond and
the color is green.27
These positive and categorical declarations of two police
officers deserve weight and credence in light of the

A: Yes[,] sir[.] I was in the office.


Q: Since your office is just near the Municipal Trial
Court of Tigaon and you were given a tip that
Salvador Peaflorida[,Jr.] will be delivering
marijuana, why did you not get a [w]arrant of
[a]rrest from the court?
A: There was no time to apply for a search warrant
because just after the information was received, we
proceeded.
xxx
Q: If that is true, Mr. Competente that you were
given a tip, the most that you will do is first see the
Judge of Tigaon in as much as you have not seen yet
[sic] the said person carrying marijuana?

A: There was no time for us to apply, because the


marijuana is being delivered so we have no more
time to see the Judge.

FISCAL SOLANO: Conclusion of law.

The police was tipped off at around 1:00 p.m. that appellant
was transporting marijuana to Huyon-huyon. Certainly, they
had no time to secure an arrest warrant as appellant was
already in transit and already committing a crime. The arrest
was effected after appellant was caught in flagrante delicto.
He was seen riding his bicycle and carrying with him the
contraband, hence, demonstrating that a crime was then
already being committed. Under the circumstances, the
police had probable cause to believe that appellant was
committing a crime. Thus, the warrantless arrest is justified.

A: Yes, sir[.] [I]f it is illegally confiscated it cannot


be used in court.

Article II, Section 4 of R.A. No. 6425, as amended by R.A.


No. 7659, states:

xxx
Q: Are you aware of the law that illegally
confiscated marijuana cannot be used in court?

ATTY. CLEDERA: Despite that prohibition under


the rules[,] you insisted in apprehending Salvador
Peaflorida[,Jr.] without warrant of arrest inspite of
the fact that you know that restriction?
A: Our apprehension was in plain view.
Q: How can you see that it was in open view when
according to you the house of Salvador is 120
meters[?] [H]ow can you see that distance?
A: I could see that because the marijuana was
carried in his bicycle, we have seen it.
Q: In what street?
A: Huyon-huyon[,] Sitio Nasulan, Tigaon,
Camarines Sur.
Q: About what time did you see him?
A: 1:00 o'clock sir.
x x x29

SEC. 4. Sale, Administration, Delivery, Distribution


and Transportation of Prohibited Drugs. The
penalty of reclusion perpetua to death and a fine
ranging from five hundred thousand pesos to ten
million pesos shall be imposed upon any person
who, unless authorized by law, shall sell, administer,
deliver, give away to another, distribute, dispatch in
transit or transport any prohibited drug, or shall act
as broker in any of such transactions. x x x.
Jurisprudence defines "transport" as "to carry or convey
from one place to another."30 In the instant case, appellant
was riding his bicycle when he was caught by the police. He
admitted that he was about to convey the package, which
contained marijuana, to a certain Jimmy Gonzales.
Appellant, however, denies any knowledge that the package
in his possession contained marijuana. But the trial court
rejected his contention, noting that it was impossible for
appellant not to be aware of the contents of the package
because "marijuana has a distinct sweet and unmistakable
aroma x x x which would have alarmed him."31
Taking one step further, the appellate court went on to
declare that being mala prohibita, one commits the crime
under R.A. No. 6425 by mere possession of a prohibited

drug without legal authority. Intent, motive or knowledge


thereof is not necessary.32
Appellant, in the main, asserts that he did not freely and
consciously possess marijuana.33 In criminal cases involving
prohibited drugs, there can be no conviction unless the
prosecution shows that the accused knowingly possessed the
prohibited articles in his person, or that animus possidendi is
shown to be present together with his possession or control
of such article. Animus possidendi is only prima facie. It is
subject to contrary proof and may be rebutted by evidence
that the accused did not in fact exercise power and control
over the thing in question, and did not intend to do so. The
burden of evidence is thus shifted to the possessor to explain
absence of animus possidendi.34
Knowledge refers to a mental state of awareness of a fact.
Since courts cannot penetrate the mind of an accused and
thereafter state its perceptions with certainty, resort to other
evidence is necessary. Animus possidendi, as a state of mind,
may be determined on a case-to-case basis by taking into
consideration the prior or contemporaneous acts of the
accused, as well as the surrounding circumstances. Its
existence may and usually must be inferred from the
attendant events in each particular case. 35
Appellant failed to satisfactorily establish his lack of
knowledge of possession in the instant case. First, the
marijuana was found in the bicycle he himself was driving.
Second, the police officers first readily saw in plain view the
edges of the marijuana leaves jutting out of the package.
Third, it is incredulous that appellant did not ask Obias what
the package contained when the latter requested him to do
the delivery errand since the package was wrapped in a
newspaper and weighed almost one kilogram. The same
observation was reached by the trial court:
Finally, it is very hard for the court to accept the
claim of the accused Salvador Peaflorida[,Jr.] that
he does not know that the thing wrapped in a

newspaper which Boyet Obias, now dead, requested


the accused Peaflorida[,Jr.] would deliver to a
certain Jimmy Gonzales whose present whereabouts
is not known, was a marijuana. Its odor is different
especially from tobacco. This was observed by the
court during the trial of the case, everytime the
wrapper containing the subject marijuana with a
volume of 928 grams is brought to court its odor is
noticeable. For the accused Peaflorida[,Jr.], not to
notice it is hard to believe. Rightly so, because
marijuana has a distinct sweet and unmistakable
aroma very different from (and not nauseating)
unlike tobacco. This aroma would have alarmed
him.36
Furthermore, it appeared from the cross-examination of
appellant that Obias was an acquaintance. In the ordinary
course of things, one is expected to inquire about the
contents of a wrapped package especially when it is a mere
acquaintance who requests the delivery and, more so, when
delivery is to a place some distance away.

Despite intense grilling from the defense counsel, Arroyo


never faltered and was in fact consistent in declaring that she
received the specimen from Agravante on 9 June 1994 and
immediately conducted the laboratory test.
Finally, the lower courts correctly sentenced appellant to
suffer the penalty of reclusion perpetua and to pay a fine of
one million pesos by virtue of the amendment to Section 4,
R.A. No. 6425 by R.A. No. 7659.38
WHEREFORE, in view of the foregoing, the decision of the
Regional Trial Court of San Jose, Camarines Sur, Branch 30
in Criminal Case No. T-1476, finding appellant Salvador
Peaflorida y Clidoro guilty beyond reasonable doubt of
violation of Section 4, Article II of R.A. No. 6425
(Dangerous Drugs Act) as amended, and sentencing him to
suffer the penalty of reclusion perpetua and to pay a fine of
One Million Pesos (P1,000,000.00), is AFFIRMED in toto.
SO ORDERED.

Morenovs.AgoChi
Facts:Sometime
May1904, the defendant (Ago Chi),represented by the
Anent appellant's claim that the package examined by
plaintiff
was
charged
and was convictedby the CFI Manila in the crime
Arroyo was not the one confiscated from him, the appellate
of assassination. He wassentenced to capital punishment but was later
court had this to say:
reduced to 20 years of reclusion temporal upon appeal in the SC.Upon
the defendants arrest, the arresting officer confiscated his money
SPO3 Competente testified that marijuana was
amounting to P700and was depositedin the clerk of court.The plaintiff
confiscated from appellant. The pictures of
Moreno, filed a petition to the CFIManila under Judge Quintero praying
for the court to pay theplaintiff of his legal services to the defendant in the
appellant, together with the items seized from him,
depict a package containing dry leaves suspected to amount of P600that will be taken from the confiscated money of
thedefendant.The lower court only granted P50as legal servicesto the
be marijuana. On the other hand, Forensic Chemist
plaintiff. Hence this petition.
Arroyo testified that the specimen she examined was
Issue:Whether or not the search and confiscation of thedefendant is a
delivered to her by Major Agravante on June 9, 1994 valid search.Can the plaintiff validly claim any lien from theconfiscated
or two days after the apprehension. From these
money?Held:No.Under the Rules of Criminal procedure, an
series of events, it can be inferred that the package
officer making an arrest may take from the person arrested anymoney or
property found upon his person which was used inthe commission of the
confiscated from appellant and the specimen
delivered to Forensic Chemist Arroyo for laboratory crime or was the fruit of the crime or which might furnish the prisoner
with the means of committing violence or of escaping, or which may be
examination were one and the same.37
used asevidence in the trial of the cause.In the case at bar, the defendant

was convicted inthe crime of assassination, thus, the confiscation of


hismoney amounting to P7
00
cannot in no way be connectedwith the crime where he was charged or
in any casesmentioned.To deprive the defendant of his money or
propertyunder other circumstances than those mentioned above is
todeprive him, perhaps, of the lawful means of defense.Thus, the money
is illegally searched andconfiscated and cannot be admitted as evidence.
Since itwas illegally confiscated, third persons cannot validly claimany
liens from the confiscated money and should remainunder the custody of
the defendant.Case remanded to the lower court.
People vs. Ang ChunKit
Facts: ANG CHUN KIT, a Chinese national and reputed tobe a member
of a Hong Kong-based drug syndicateoperating in Metro Manila, was
collared by NARCOMoperatives in a buy-bust operation after he sold to
anundercover agent for P4
00
,
000
.
00
a kilo of methamphetamine hydrochloride known as shabu. His
car also yielded more of the regulated drug neatly tucked in aKleenex
box. After tipping an information from a ConfidentialInformer, Chief
investigator Avelino Razon arranged a buy-bust operation using
operatives and marked money,scheduled to be operated in Cardinal
Santos Hospital.The buy-bust operation was successful, the moneywas
confiscated and nearly a kilo of
shabu
was seized in thedefendants car.He was then brought to the Camp
Crame, chargedwith violation to the Dangerous drugs Act. He was
convictedby RTC Pasig Br.
1
55 in violation of RA 6425, Sec
1
5 Art III.Hence This petition.He argued that the confiscated articles
areinadmissible as evidence since it was procured through anillegal
search and seizure and that the drugs are not found inhis person but in his
car.Issue:Was the defendats contention correct?Held:No. It was a search
and seizure incidental to a validarrest.Under the constitutional and
statutory provision, avalid warrantless arrest may be effected when the
accused isactually committed, committing or about to commit a
crime. And that a warrantless search and seizure can be effectedwhen the
search is incidental to a valid arrest. Moreover, awarrantless search and
seizure as an incident to a lawfularrest may extend beyond the person of

the one arrested toinclude the premises or surroundings under his


immediatecontrol.In the case at bar, RA 6425 actually punishes evena
mere possession of regulated drugs, thus the accused is inthe process of
committing a crime during the buy bustoperation. It is thus then a valid
warrantless arrest.Since it is a valid, the warrantless search mayextend to
the accused surrounding under his immediatecontrol, thus the search
seizure of shabu in his car is a validseizure.
Note:
In the case at bar . the Court however agree withthe accused that his
signature on the receipt or lists of itemsconfiscated from him is
inadmissible in evidence as there isno showing that he was then assisted
by counsel. In Peoplev. Mauyao we said that "conformance to these
documentsare declarations against interest and tacit admissions of
thecrime charged, since merely unexplained possession of prohibited
drugs is punished by law. They have beenobtained in violation of his
right as a person under custodialinvestigation for the commission of an
offense, there beingnothing in the records to show that he was assisted
bycounsel."With regard to the Booking Sheet and ArrestReport, we
already said in People v. Morico that "when anarrested person signs a
Booking Sheet and Arrest Report ata police station he does not admit the
commission of anoffense nor confess to any incriminating circumstance.
TheBooking Sheet is merely a statement of the accused's beingbooked
and of the date which accompanies the fact of anarrest. It is a police
report and may be useful in charges of arbitrary detention against the
police themselves. It is not anextra-judicial statement and cannot be the
basis of a judgment of conviction."
People vs. Lua
Facts:The defendant, Rolando
Chekwa
Lua, was arrestedby a buy bust operation under the Oplan Saturn by
theCaloocan police operatives in Bagong Silang Caloocan City.In the
buy bust operation, after theinformant/operative successfully bought
some marijuanafrom the accused, the police subsequently acted
andarrested Lua, confiscated the marked money, the regulateddrug
marijuana
and a .3
8
cal
paltik
and live bulletswhich wasfound in the accused waistline. These articles
are soughtoutside the accuseds houseThe police operatives also found
and confiscated abrick of marijuana inside the accuseds house.The RTC
Br.
1

24 of Caloocan City convicted theaccused in violation of RA 6425.


Hence this petition.Issue:Was there a valid warrantless search and
seizure?Held:Yes. The search and seizure is lawful, that being
awarrantless search and seizure incidental to a lawful arrest,however, the
articles (brick of marijuana) seized inside theaccuseds house is
inadmissible evidence.Having settled the issues raised by appellant,
theequally important matter as regards admissibility of theevidence
should likewise be passed upon. The buy-bustoperation conducted by
the police operatives is a form of entrapment allowed by law. The arrest
of the appellant waslawful having been caught in
flagrante delicto
. Consequently,there is no need for a warrant for the seizure of the 3 tea

and sister, to acquiesce to a search of thehouse. The search yielded a .45


caliber pistol, a magazine,seven live ammunitions, and a match box
containing analuminum foil package with "shabu." Confronted,
Figueroadenied ownership of the items. An inventory was conductedby
the PC team, attested to by Barangay Captain Bigornia, of the seized
items.The accused questions the admissibility in evidenceof the firearm
and confiscated ammunition for it wasdiscovered during a warrantless
search.Issue:Was their an unlawful warrantless search
andseizure.Held:No. the search and seizure of the articles sought isa valid
being a search incidental to an arrest.The .45 caliber pistol, magazine and
rounds of ammunition were not unlawfully obtained. While the SC
mightconcede difficulty in readily accepting the statement of
theprosecution that the search was conducted with consentfreely given
by appellant and members of his household, itshould be pointed out, in
Constitutional Law II (Bill of Rights): Case Briefs: Dennis G.
any case, that the search andseizure was done admittedly on the occasion
Libunao UC College of Law
of a lawfularrest. A significant exception from the necessity for asearch
32
bags of marijuana (5.3934 grams) the same being the fruit of the crime. warrant is when the search and seizure is effected asan incident to a
With respect to the body search made by Puno,the same was valid being lawful arrest. As a doctrine in jurisprudence, the warrantlesssearch and
seizure, as an incident to a suspect's lawfularrest, may extend beyond the
incidental to a lawful arrest.Therefore, the .3
person of the one arrested toinclude the premises or surrounding under
8
his immediatecontrol. Objects in the `plain view' of an officer who has
cal. paltik and the two (2) live bullets andthe empty shell found in the
theright to be in the position to have that view are subject toseizure and
cylinder are admissible inevidence. As regards the brick of marijuana
may be presented as evidence."
found inside theappellant's house, the trial court correctly ignored
itapparently in view of its inadmissibility.While initially the arrest as well Nolasco vs. Pano
Conflicting views between the ponente Justice Melencio-Herrera and
as the body searchwas lawful, the warrantless search made inside
Justices
appellant'shouse became unlawful since the police operatives were
Teehankee and Cuevas concerning thevalidity place of arrest and search
notarmed with a search warrant. Such search cannot fall under "search
made incidental to a lawful arrest," the same beinglimited to body search of the accused inconnection with Lawful Search Incidental to an Arrest
Facts:The petitioners, Nolasco, Aguilar-Roque andTolentino were
and to that point within reach or control of the person arrested, or that
charged of the crime of Rebellion, Subversionand/or Conspiracy o
which may furnish himwith the means of committing violence or of
commit Rebellion/Subversion and wasarrested by Constabulary Security
escaping. In thecase at bar, appellant was admittedly outside his house
whenhe was arrested. Hence, it can hardly be said that the inner portion Group (CSG) on August6 that11:30AM at the intersection of Mayon
St. and P.Margali St, Quezon City. At
of his house was within his reach or control.In sum, this court finds
1
accused-appellant RolandoLua guilty beyond reasonable doubt of
2 noon of that same day, a search wasconducted at the residence of the
violating Sec. 4, Art.II, of R.A. 6425, as amended, under which the
petitioner Aguilar-Roque at239 B Mayon St. QC. The CSG confiscated
penalty of lifeimprisonment to death and a fine ranging from
42
twentythousand to thirty thousand pesos shall be imposed.
8
writtendocuments, typewriter and 2 wooden boxes.The petitioners
People vs. Figueroa
Facts:The accused was charged with Illegal Possession of Firearms and alleged that the search warrant andarrest warrants issued was void in the
ground that there is noestablished existing probable cause and that the
Ammunitions and and of RA 645 andsubsequently convicted by the
warrant is ageneral in nature, violative of their constitutional
RTC Br. 23 of Trece Martiresin Cavite.While serving the warrant of
right.Moreover, they contend that the articles confiscatedin inadmissible
arrest, the officersnoticed, strewn around, aluminum foil packages of
as evidence as the search is illegal not beinga search incidental to an
differentsizes in the sala. Suspecting thus the presence of "shabu" inthe
premises, the arresting officers requested appellant, aswell as his brother

arrest.Issue:Was the search cannot be qualified as a lawfulsearch


incidental to lawful arrest?Held:No. The Search is lawful.UNDER Sec
12, rule126 of the RRC,Section 12
. Search without warrant of personarrested. - Aperson charged with an
offense may besearched for dangerous weapons or anything which may
beused as proof of the commission of the offense."
The provision is declaratory in the sense that it isconfined to the search,
without a search warrant, of a personwho had been arrested. It is also a
general rule that, as anincident of an arrest, the place or premises where
the arrestwas made can also be search without a search warrant. Inthis
latter case, "the extent and reasonableness of the searchmust be decided
on its own facts and circumstances, and ithas been stated that, in the
application of general rules, thereis some confusion in the decisions as to
what constitutes theextent of the place or premises which may be
searched"."What must be considered is the balancing of the
individual'sright to privacy and the public's interest in the prevention
of crime and the apprehension of criminals."Considering that
AGUILAR-ROQUE has beencharged with Rebellion, which is a crime
against public order;that the warrant for her arrest has not been served for
aconsiderable period of time; that she was arrested within thegeneral
vicinity of her dwelling; and that the search of her dwelling was made
within a half hour of her arrest, we are of the opinion that, in her respect,
the search at No. 239-BMayon Street, Quezon City,
did not need a search warrant
;this, for possible effective results in the interest of publicorder.The
search in the residence of the petitioner Aguilar-Roque is valid since in
relation with the nature of thecrime of rebellion, which is a crime
against public order, it is asearch incidental to an arrest. The
immediate vicinity of thearrest is not controlling as far as the publics
interest in theprevention of the crime and apprehension of the criminals
areconcerned.
Separate and Dissenting opinion of justices
T
eehankeeand Cuevas
Justice Teehankee
The exception of Rule
1
26, sec.
1
2 which allows awarrantless search of a person who is lawfully arrested
isabsolutely l
imited to his person, at the time of and incident tohis arrest and to
"dangerous weapons or anything which may be used as proof of the

commission of the offense." Suchwarrantless search obviously cannot be


made in a placeother than the place of arrest
. In this case, petitioner Aguilar-Roque was arrested at
11
:3
0
a.m. on board a public vehicleon the road (at Mayon and P. Margall
Streets). To hold thather dwelling could "later on the same day" be
searchedwithout warrant is to sanction an untenable violation, if
notnullification, of the cited basic constitutional rights
againstunreasonable searches and seizures.
Justice
C
uevas
The lawful arrest being the sole justification for thevalidity of the
warrantless search under the statutoryprovision (Sec.
1
2, Rule
1
26) the same must be limited toand circumscribed by,
the subject, time, and place of said arrest
.
A
s to subject
, the warrantless search is sanctionedonly with respect to the person of the
suspect, and things thatmay be seized from him are limited to "dangerous
weapons"or "anything which may be used as proof of the commissionof
the offense.""An officer making an arrest may take from theperson
arrested any money or property found upon hisperson which was used in
the commission of the crime or might furnish the prisoner with the
means of committingviolence or escaping or which may be used as
evidence inthe trial of the cause.
W
ith respect to the ti
m
e and place of thewarrantless search allowed by law
, it must becontemporaneous with the lawful arrest. Stated otherwise,
tobe valid, the search must have been conducted at about thetime of the
arrest or immediately thereafter and only at theplace where the suspect
was arrested."The right without a search warrant contemporaneously
tosearch a person lawfully arrested while committing a crimeand to
search the place where the arrest is made in order tofind and seize things
connected with the crime as its fruits or as the means by which it was

committed, as well as weaponsor other things to effect an escape from


custody is not to bedoubted.
But the right does not extend to other places
The second element which must exist in order tobring the case within the
exception to the general rule is that,in addition to a lawful arrest, t
he search
m
ust be incident tothe arrest.
"The search
m
ust be
m
ade at the place of thearrest, otherwise, it is not incident to the arrest.
AGNELLO vs. U.S. supra. In this latter case, 269 U.S. 2
0
at3
0
, it is said that the officers have a right to make a
searchcontemporaneously with the arrest. And if the purpose of
theofficers in making their entry is not to make an arrest, but
tomake a search to obtain evidence for some future arrest,then search is
not incidental to arrest. BYARS vs. U.S. 273U.S., 2
8
ET AL." (Papani vs. U.S.,
8
4F2d
1
6
0
,
1
63)In the instant case, petitioners were arrested at theintersection of
Mayon St. and P. Margall St. at
11
:3
0
A.M. of August 6,
1
976. The search, on the other hand, wasconducted after the arrest, that
was at around
1
2:
00

noon of the same day or "late that same day (as respondents claim intheir
"COMMENT") at the residence of petitioner AGUILAR-ROQUE in
239B Mayon St., Quezon City. How far or howmany kilometers is that
place from the place where petitioner was arrested do not appear shown
by the record, But whatappears undisputed is that the search was made in
a placeother than the place of arrest and, not on the occasion of
nor immediately after the arrest. It cannot be said, therefore, thatsuch a
search was incidental to the arrest of the petitioners.Not being an incident
of a lawful arrest, the search of thepremises at 239B Mayon St., Quezon
City
WITHOUT A VALIDSEARCHWARRANT is ILLEGAL and
violative of the constitutional rights of the respondent
. The thingsand properties seized on the occasion of said illegal searchare
therefore INADMISSIBLE in evidence under theexclusionary rule.
G.R. No. 188611

June 16, 2010

PEOPLE OF THE PHILIPPINES, Appellee,


vs.
BELEN MARIACOS, Appellant.
DECISION
NACHURA, J.:
Before this Court is an appeal from the Decision1 of the
Court of Appeals (CA) in CA-G.R. CR-HC No. 02718,
which affirmed the decision2 of the Regional Trial Court
(RTC), Branch 29, San Fernando City, La Union, in Criminal
Case No. 7144, finding appellant Belen Mariacos guilty of
violating Article II, Section 5 of Republic Act (R.A.) No.
9165, or the Comprehensive Dangerous Drugs Act of 2002.
The facts of the case, as summarized by the CA, are as
follows:
Accused-appellant Belen Mariacos was charged in an
Information, dated November 7, 2005 of violating Section 5,
Article II of Republic Act [No.] 9165, allegedly committed
as follows:

"That on or about the 27th day of October, 2005, in the


Municipality of San Gabriel, Province of La Union,
Philippines, and within the jurisdiction of this Honorable
Court, the above-named accused, did then and there
willfully, unlawfully and feloniously transport, deliver
7,030.3, (sic) grams of dried marijuana fruiting tops without
the necessary permit or authority from the proper
government agency or office.
CONTRARY TO LAW."
When arraigned on December 13, 2005, accused-appellant
pleaded not guilty. During the pre-trial, the following were
stipulated upon:
"1. Accused admits that she is the same person
identified in the information as Belen Mariacos;
2. That accused is a resident of Brgy. Lunoy, San
Gabriel, La Union;
3. That at the time of the arrest of the accused,
accused had just alighted from a passenger jeepney;
4. That the marijuana allegedly taken from the
possession of the accused contained in two (2) bags
were submitted for examination to the Crime Lab;
5. That per Chemistry Report No. D-109-2005, the
alleged drug submitted for examination gave
positive result for the presence of marijuana;
6. That the drugs allegedly obtained from the
accused contained (sic) and submitted for
examination weighed 7,030.3 grams;
7. The Prosecutor admits the existence of a counteraffidavit executed by the accused; and

8. The existence of the affidavits executed by the


witnesses of the accused family (sic): Lyn Punasen,
Mercedes Tila and Magdalena Carino."
During the trial, the prosecution established the following
evidence:
On October 26, 2005, in the evening, the San Gabriel Police
Station of San Gabriel, La Union, conducted a checkpoint
near the police station at the poblacion to intercept a
suspected transportation of marijuana from Barangay
Balbalayang, San Gabriel, La Union. The group at the
checkpoint was composed of PO2 Lunes B. Pallayoc ("PO2
Pallayoc"), the Chief of Police, and other policemen. When
the checkpoint did not yield any suspect or marijuana, the
Chief of Police instructed PO2 Pallayoc to proceed to
Barangay Balbalayang to conduct surveillance operation
(sic).
At dawn on October 27, 2005, in Barangay Balbalayang,
PO2 Pallayoc met with a secret agent of the Barangay
Intelligence Network who informed him that a baggage of
marijuana had been loaded on a passenger jeepney that was
about to leave for the poblacion. The agent mentioned three
(3) bags and one (1) blue plastic bag. Further, the agent
described a backpack bag with an "O.K." marking. PO2
Pallayoc then boarded the said jeepney and positioned
himself on top thereof. While the vehicle was in motion, he
found the black backpack with an "O.K." marking and
peeked inside its contents. PO2 Pallayoc found bricks of
marijuana wrapped in newspapers. He then asked the other
passengers on top of the jeepney about the owner of the bag,
but no one knew.
When the jeepney reached the poblacion, PO2 Pallayoc
alighted together with the other passengers. Unfortunately,
he did not notice who took the black backpack from atop the
jeepney. He only realized a few moments later that the said
bag and three (3) other bags, including a blue plastic bag,
were already being carried away by two (2) women. He

caught up with the women and introduced himself as a


owner of the bags and that she did not know what were
immediately after seizure or confiscation, to have the same
policeman. He told them that they were under arrest, but one contained in the bags. At the police station (sic) she executed physically inventoried and photographed in the presence of
of the women got away.
a Counter-Affidavit.3
appellant or her representative, who shall be required to sign
copies of the inventory. The failure to comply with this
PO2 Pallayoc brought the woman, who was later identified
On January 31, 2007, the RTC promulgated a decision, the
directive, appellant claimed, casts a serious doubt on the
as herein accused-appellant Belen Mariacos, and the bags to dispositive portion of which states:
identity of the items allegedly confiscated from her. She,
the police station. At the police station, the investigators
likewise, averred that the prosecution failed to prove that the
WHEREFORE, the Court finds the accused Belen Mariacos items allegedly confiscated were indeed prohibited drugs,
contacted the Mayor of San Gabriel to witness the opening
GUILTY as charged and sentences here (sic) to suffer the
of the bags. When the Mayor arrived about fifteen (15)
and to establish the chain of custody over the same.
penalty of life imprisonment and to pay a fine of
minutes later, the bags were opened and three (3) bricks of
P500,000.00.
marijuana wrapped in newspaper, two (2) round bundles of
On the other hand, the People, through the Office of the
marijuana, and two (2) bricks of marijuana fruiting tops, all
Solicitor General (OSG), argued that the warrantless arrest
The
7,030.3
grams
of
marijuana
are
ordered
confiscated
and
wrapped in a newspaper, were recovered.
of appellant and the warrantless seizure of marijuana were
turned over to the Philippine Drug Enforcement Agency for valid and legal,8 justified as a search of a moving vehicle. It
destruction in the presence of the Court personnel and
Thereafter, the investigators marked, inventoried and
averred that PO2 Pallayoc had reasonable ground to believe
forwarded the confiscated marijuana to the crime laboratory media.
that appellant had committed the crime of delivering
for examination. The laboratory examination showed that the
dangerous drugs based on reliable information from their
4
SO
ORDERED.
stuff found in the bags all tested positive for marijuana, a
agent, which was confirmed when he peeked into the bags
dangerous drug.
and smelled the distinctive odor of marijuana.9 The OSG also
Appellant appealed her conviction to the CA. She argued
argued that appellant was now estopped from questioning the
that the trial court erred in considering the evidence of the
When it was accused-appellants turn to present evidence,
illegality of her arrest since she voluntarily entered a plea of
prosecution despite its inadmissibility.5 She claimed that her "not guilty" upon arraignment and participated in the trial
she testified that:
right against an unreasonable search was flagrantly violated and presented her evidence.10 The OSG brushed aside
by Police Officer (PO)2 Pallayoc when the latter searched
On October 27, 2005, at around 7:00 in the morning,
appellants argument that the bricks of marijuana were not
accused-appellant, together with Lani Herbacio, was inside a the bag, assuming it was hers, without a search warrant and photographed and inventoried in her presence or that of her
with no permission from her. She averred that PO2
passenger jeepney bound for the poblacion. While the
counsel immediately after confiscation, positing that
jeepney was still at the terminal waiting for passengers, one Pallayocs purpose for apprehending her was to verify if the physical inventory may be done at the nearest police station
Bennie Lao-ang ("Lao-ang"), her neighbor, requested her to bag she was carrying was the same one he had illegally
or at the nearest office of the apprehending team, whichever
searched earlier. Moreover, appellant contended that there
carry a few bags which had been loaded on top of the
was practicable.11
6
was no probable cause for her arrest.
jeepney. At first, accused-appellant refused, but she was
persuaded later when she was told that she would only be
In a Decision dated January 19, 2009, the CA dismissed
Further, appellant claimed that the prosecution failed to
carrying the bags. When they reached the poblacion, Laoappellants appeal and affirmed the RTC decision in toto. 12 It
7
prove the corpus delicti of the crime. She alleged that the
ang handed accused-appellant and her companion, Lani
held that the prosecution had successfully proven that
Herbacio, the bags, and then Lao-ang suddenly ran away. A apprehending police officers violated Dangerous Drugs
appellant carried away from the jeepney a number of bags
Board Regulation No. 3, Series of 1979, as amended by
few moments later, PO2 Pallayoc was upon them, arresting
which, when inspected by the police, contained dangerous
Board
Regulation
No.
2,
Series
of
1990,
which
prescribes
them. Without explanation, they were brought to the police
drugs. The CA ruled that appellant was caught in flagrante
station. When they were at the police station, Lani Herbacio the procedure in the custody of seized prohibited and
delicto of "carrying and conveying" the bag that contained
regulated drugs, instruments, apparatuses, and articles. The
disappeared. It was also at the police station that accusedthe illegal drugs, and thus held that appellants warrantless
appellant discovered the true contents of the bags which she said regulation directs the apprehending team having initial arrest was valid. The appellate court ratiocinated:
custody and control of the drugs and/or paraphernalia,
was asked to carry. She maintained that she was not the

It must be stressed that PO2 Pallayoc had earlier ascertained


the contents of the bags when he was aboard the jeep. He
saw the bricks of marijuana wrapped in newspaper. That said
marijuana was on board the jeepney to be delivered to a
specified destination was already unlawful. PO2 Pallayoc
needed only to see for himself to whom those bags belonged.
So, when he saw accused-appellant carrying the bags, PO2
Pallayoc was within his lawful duty to make a warrantless
arrest of accused-appellant.
xxxx
Firstly, this Court opines that the invocation of Section 2,
Article III of the Constitution is misplaced. At the time,
when PO2 Pallayoc looked into the contents of the
suspicious bags, there was no identified owner. He asked the
other passengers atop the jeepney but no one knew who
owned the bags. Thus, there could be no violation of the
right when no one was entitled thereto at that time.

Once again, we are asked to determine the limits of the


powers of the States agents to conduct searches and
seizures. Over the years, this Court had laid down the rules
on searches and seizures, providing, more or less, clear
parameters in determining which are proper and which are
not.1avvphi1
Appellants main argument before the CA centered on the
inadmissibility of the evidence used against her. She claims
that her constitutional right against unreasonable searches
was flagrantly violated by the apprehending officer.
Thus, we must determine if the search was lawful. If it was,
then there would have been probable cause for the
warrantless arrest of appellant.
Article III, Section 2 of the Philippine Constitution provides:

Section 2. The right of the people to be secure in their


persons, houses, papers, and effects against unreasonable
searches and seizures of whatever nature and for any purpose
Secondly, the facts of the case show the urgency of the
shall be inviolable, and no search warrant or warrant of
situation. The local police has been trying to intercept the
transport of the illegal drugs for more than a day, to no avail. arrest shall issue except upon probable cause to be
Thus, when PO2 Pallayoc was tipped by the secret agent of determined personally by the judge after examination under
oath or affirmation of the complainant and the witnesses he
the Barangay Intelligence Network, PO2 Pallayoc had no
may produce, and particularly describing the place to be
other recourse than to verify as promptly as possible the tip
searched and the persons or things to be seized.
and check the contents of the bags.
Thirdly, x x x the search was conducted in a moving vehicle. Law and jurisprudence have laid down the instances when a
warrantless search is valid. These are:
Time and again, a search of a moving vehicle has been
justified on the ground that the mobility of motor vehicles
1. Warrantless search incidental to a lawful arrest
makes it possible for the vehicle to move out of the locality
recognized under Section 12 [now Section 13], Rule
or jurisdiction in which the warrant must be sought. Thus,
126 of the Rules of Court and by prevailing
under the facts, PO2 Pallayoc could not be expected to
jurisprudence;
secure a search warrant in order to check the contents of the
bags which were loaded on top of the moving jeepney.
2. Seizure of evidence in "plain view," the elements
Otherwise, a search warrant would have been of no use
13
of which are:
because the motor vehicle had already left the locality.
Appellant is now before this Court, appealing her conviction.

(a) a prior valid intrusion based on the valid


warrantless arrest in which the police are
legally present in the pursuit of their official
duties;
(b) the evidence was inadvertently
discovered by the police who had the right
to be where they are;
(c) the evidence must be immediately
apparent[;] and;
(d) "plain view" justified mere seizure of
evidence without further search.
3. Search of a moving vehicle. Highly regulated by
the government, the vehicle's inherent mobility
reduces expectation of privacy especially when its
transit in public thoroughfares furnishes a highly
reasonable suspicion amounting to probable cause
that the occupant committed a criminal activity;
4. Consented warrantless search;
5. Customs search;
6. Stop and Frisk; and
7. Exigent and Emergency Circumstances.14
Both the trial court and the CA anchored their respective
decisions on the fact that the search was conducted on a
moving vehicle to justify the validity of the search.
Indeed, the search of a moving vehicle is one of the
doctrinally accepted exceptions to the Constitutional
mandate that no search or seizure shall be made except by
virtue of a warrant issued by a judge after personally
determining the existence of probable cause. 15

In People v. Bagista,16 the Court said:


The constitutional proscription against warrantless searches
and seizures admits of certain exceptions. Aside from a
search incident to a lawful arrest, a warrantless search had
been upheld in cases of a moving vehicle, and the seizure of
evidence in plain view.

offense has been committed, and that the items, articles or


objects sought in connection with said offense or subject to
seizure and destruction by law are in the place to be
searched.19

obtain a warrant. Indeed, he only had enough time to board


the vehicle before the same left for its destination.

It is well to remember that on October 26, 2005, the night


before appellants arrest, the police received information that
The grounds of suspicion are reasonable when, in the
marijuana was to be transported from Barangay Balbalayang,
absence of actual belief of the arresting officers, the
and had set up a checkpoint around the area to intercept the
suspicion that the person to be arrested is probably guilty of suspects. At dawn of October 27, 2005, PO2 Pallayoc met
With regard to the search of moving vehicles, this had been committing the offense is based on actual facts, i.e.,
the secret agent from the Barangay Intelligence Network,
justified on the ground that the mobility of motor vehicles
supported by circumstances sufficiently strong in themselves who informed him that a baggage of marijuana was loaded
makes it possible for the vehicle to be searched to move out to create the probable cause of guilt of the person to be
on a passenger jeepney about to leave for the poblacion.
of the locality or jurisdiction in which the warrant must be
arrested. A reasonable suspicion therefore must be founded
Thus, PO2 Pallayoc had probable cause to search the
sought.
on probable cause, coupled with good faith on the part of the packages allegedly containing illegal drugs.
peace officers making the arrest.20
This in no way, however, gives the police officers unlimited
This Court has also, time and again, upheld as valid a
discretion to conduct warrantless searches of automobiles in Over the years, the rules governing search and seizure have warrantless search incident to a lawful arrest. Thus, Section
the absence of probable cause. When a vehicle is stopped
been steadily liberalized whenever a moving vehicle is the
13, Rule 126 of the Rules of Court provides:
and subjected to an extensive search, such a warrantless
object of the search on the basis of practicality. This is so
search has been held to be valid only as long as the officers
SEC. 13. Search incident to lawful arrest.A person
considering that before a warrant could be obtained, the
conducting the search have reasonable or probable cause to
place, things and persons to be searched must be described to lawfully arrested may be searched for dangerous weapons or
believe before the search that they will find the
anything which may have been used or constitute proof in
the satisfaction of the issuing judge a requirement which
instrumentality or evidence pertaining to a crime, in the
borders on the impossible in instances where moving vehicle the commission of an offense without a search warrant. 23
vehicle to be searched.
is used to transport contraband from one place to another
For this rule to apply, it is imperative that there be a prior
with impunity.21
It is well to remember that in the instances we have
valid arrest. Although, generally, a warrant is necessary for a
recognized as exceptions to the requirement of a judicial
valid arrest, the Rules of Court provides the exceptions
This exception is easy to understand. A search warrant may
warrant, it is necessary that the officer effecting the arrest or readily be obtained when the search is made in a store,
therefor, to wit:
seizure must have been impelled to do so because of
dwelling house or other immobile structure. But it is
SEC. 5. Arrest without warrant; when lawful.A peace
probable cause. The essential requisite of probable cause
impracticable to obtain a warrant when the search is
must be satisfied before a warrantless search and seizure can conducted on a mobile ship, on an aircraft, or in other motor officer or a private person may, without a warrant, arrest a
be lawfully conducted.17 Without probable cause, the articles vehicles since they can quickly be moved out of the locality person:
seized cannot be admitted in evidence against the person
or jurisdiction where the warrant must be sought. 22
(a) When, in his presence, the person to be arrested
arrested.18
has committed, is actually committing, or is
Given the discussion above, it is readily apparent that the
attempting to commit an offense;
Probable cause is defined as a reasonable ground of
search in this case is valid. The vehicle that carried the
suspicion supported by circumstances sufficiently strong in
contraband or prohibited drugs was about to leave. PO2
(b) When an offense has just been committed and he
themselves to induce a cautious man to believe that the
Pallayoc had to make a quick decision and act fast. It would
has probable cause to believe based on personal
person accused is guilty of the offense charged. It refers to
be unreasonable to require him to procure a warrant before
knowledge of facts or circumstances that the person
the existence of such facts and circumstances that can lead a conducting the search under the circumstances. Time was of
to be arrested has committed it; and
reasonably discreet and prudent man to believe that an
the essence in this case. The searching officer had no time to

(c) When the person to be arrested is a prisoner who


has escaped from a penal establishment or place
where he is serving final judgment or is temporarily
confined while his case is pending, or has escaped
while being transferred from one confinement to
another.
In cases falling under paragraphs (a) and (b) above, the
person arrested without a warrant shall be forthwith
delivered to the nearest police station or jail and shall be
proceeded against in accordance with section 7 of Rule
112.24
Be that as it may, we have held that a search substantially
contemporaneous with an arrest can precede the arrest if the
police has probable cause to make the arrest at the outset of
the search.25

thousand pesos (P500,000.00) shall be imposed upon any


person who, unless authorized by law, shall sell, trade,
administer, dispense, deliver, give away to another,
distribute, dispatch in transit or transport any controlled
precursor and essential chemical, or shall act as a broker in
such transactions.
In her defense, appellant averred that the packages she was
carrying did not belong to her but to a neighbor who had
asked her to carry the same for him. This contention,
however, is of no consequence.
When an accused is charged with illegal possession or
transportation of prohibited drugs, the ownership thereof is
immaterial. Consequently, proof of ownership of the
confiscated marijuana is not necessary.26

immaterial whether or not the place of destination is


reached.32
Moreover, appellants possession of the packages containing
illegal drugs gave rise to the disputable presumption 33 that
she is the owner of the packages and their contents. 34
Appellant failed to rebut this presumption. Her
uncorroborated claim of lack of knowledge that she had
prohibited drug in her possession is insufficient.
Appellants narration of facts deserves little credence. If it is
true that Bennie Lao-ang merely asked her and her
companion to carry some baggages, it is but logical to first
ask what the packages contained and where these would be
taken. Likewise, if, as appellant said, Lao-ang ran away after
they disembarked from the jeepney, appellant and her
companion should have ran after him to give him the bags he
had left with them, and not to continue on their journey
without knowing where they were taking the bags.

Appellants alleged lack of knowledge does not constitute a


Given that the search was valid, appellants arrest based on
valid defense. Lack of criminal intent and good faith are not
that search is also valid.
exempting circumstances where the crime charged is malum
prohibitum, as in this case.27 Mere possession and/or delivery Next, appellant argues that the prosecution failed to prove
Article II, Section 5 of the Comprehensive Dangerous Drugs of a prohibited drug, without legal authority, is punishable
the corpus delicti of the crime. In particular, she alleged that
28
Act of 2002 states:
under the Dangerous Drugs Act.
the apprehending police officers failed to follow the
procedure in the custody of seized prohibited and regulated
SEC. 5 Sale, Trading, Administration, Dispensation,
Anti-narcotics laws, like anti-gambling laws, are regulatory drugs, instruments, apparatuses, and articles.
Delivery, Distribution and Transportation of Dangerous
statutes. They are rules of convenience designed to secure a
Drugs and/or Controlled Precursors and Essential Chemicals. more orderly regulation of the affairs of society, and their
In all prosecutions for violation of the Dangerous Drugs Act,
The penalty of life imprisonment to death and a fine
violation gives rise to crimes mala prohibita. Laws defining the existence of all dangerous drugs is a sine qua non for
ranging from Five hundred thousand pesos (P500,000.00) to crimes mala prohibita condemn behavior directed not against conviction. The dangerous drug is the very corpus delicti of
Ten million pesos (P10,000,000.00) shall be imposed upon
particular individuals, but against public order.29
that crime.35
any person, who, unless authorized by law, shall sell, trade,
administer, dispense, deliver, give away to another,
Jurisprudence defines "transport" as "to carry or convey
Thus, Section 21 of R.A. No. 9165 prescribes the procedure
30
distribute, dispatch in transit or transport any dangerous
from one place to another." There is no definitive moment for custody and disposition of seized dangerous drugs, to
drug, including any and all species of opium poppy
when an accused "transports" a prohibited drug. When the
wit:
regardless of the quantity and purity involved, or shall act as circumstances establish the purpose of an accused to
a broker in any of such transactions.
transport and the fact of transportation itself, there should be Section 21. Custody and Disposition of Confiscated, Seized,
and/or Surrendered Dangerous Drugs, Plant Sources of
no question as to the perpetration of the criminal act. 31 The
The penalty of imprisonment ranging from twelve (12) years fact that there is actual conveyance suffices to support a
Dangerous Drugs, Controlled Precursors and Essential
and one (1) day to twenty (20) years and a fine ranging from finding that the act of transporting was committed and it is
Chemicals, Instruments/Paraphernalia and/or Laboratory
One hundred thousand pesos (P100,000.00) to Five hundred
Equipment. The PDEA shall take charge and have custody

of all dangerous drugs, plant sources of dangerous drugs,


controlled precursors and essential chemicals, as well as
instruments/paraphernalia and/or laboratory equipment so
confiscated, seized and/or surrendered, for proper disposition
in the following manner:

place where the search warrant is served; or at the nearest


police station or at the nearest office of the apprehending
officer/team, whichever is practicable, in case of warrantless
seizures; Provided, further, that non-compliance with these
requirements under justifiable grounds, as long as the
integrity and the evidentiary value of the seized items are
(1) The apprehending team having initial custody and control properly preserved by the apprehending officer/team, shall
of the drugs shall, immediately after seizure and
not render void and invalid such seizures of and custody
confiscation, physically inventory and photograph the same over said items.
in the presence of the accused or the person/s from whom
such items were confiscated and/or seized, or his/her
PO2 Pallayoc testified that after apprehending appellant, he
representative or counsel, a representative from the media
immediately brought her to the police station. At the station,
and the Department of Justice (DOJ), and any elected public the police requested the Mayor to witness the opening of the
official who shall be required to sign the copies of the
bags seized from appellant. When the Mayor arrived, he
inventory and be given a copy thereof.
opened the bag in front of appellant and the other police
officers. The black bag yielded three bricks of marijuana
The Implementing Rules and Regulations (IRR) of R.A. No. wrapped in newspaper, while the plastic bag yielded two
9165 further provides:
bundles of marijuana and two bricks of marijuana fruiting
tops.36 PO2 Pallayoc identified the bricks. He and PO3
SECTION 21. Custody and Disposition of Confiscated,
Stanley Campit then marked the same. Then the seized items
Seized and/or Surrendered Dangerous Drugs, Plant
were brought to the PNP Crime Laboratory for examination.
Sources of Dangerous Drugs, Controlled Precursors and
Essential Chemicals, Instruments/Paraphernalia and/or It is admitted that there were no photographs taken of the
Laboratory Equipment. The PDEA shall take charge and drugs seized, that appellant was not accompanied by
have custody of all dangerous drugs, plant sources of
counsel, and that no representative from the media and the
dangerous drugs, controlled precursors and essential
DOJ were present. However, this Court has already
chemicals, as well as instruments/paraphernalia and/or
previously held that non-compliance with Section 21 is not
laboratory equipment so confiscated, seized and/or
fatal and will not render an accuseds arrest illegal, or make
surrendered, for proper disposition in the following manner: the items seized inadmissible. What is of utmost importance
is the preservation of the integrity and evidentiary value of
(a) The apprehending officer/team having initial custody and the seized items.37
control of the drugs shall, immediately after seizure and
confiscation, physically inventory and photograph the same Based on the testimony of PO2 Pallayoc, after appellants
in the presence of the accused or the person/s from whom
arrest, she was immediately brought to the police station
such items were confiscated and/or seized, or his/her
where she stayed while waiting for the Mayor. It was the
representative or counsel, a representative from the media
Mayor who opened the packages, revealing the illegal drugs,
and the Department of Justice (DOJ), and any elected public which were thereafter marked and sent to the police crime
official who shall be required to sign the copies of the
laboratory the following day. Contrary to appellants claim,
inventory and be given a copy thereof: Provided, that the
the prosecutions evidence establishes the chain of custody
physical inventory and photograph shall be conducted at the

from the time of appellants arrest until the prohibited drugs


were tested at the police crime laboratory.
While it is true that the arresting officer failed to state
explicitly the justifiable ground for non-compliance with
Section 21, this does not necessarily mean that appellants
arrest was illegal or that the items seized are inadmissible.
The justifiable ground will remain unknown because
appellant did not question the custody and disposition of the
items taken from her during the trial.38 Even assuming that
the police officers failed to abide by Section 21, appellant
should have raised this issue before the trial court. She could
have moved for the quashal of the information at the first
instance. But she did not. Hence, she is deemed to have
waived any objection on the matter.
Further, the actions of the police officers, in relation to the
procedural rules on the chain of custody, enjoyed the
presumption of regularity in the performance of official
functions. Courts accord credence and full faith to the
testimonies of police authorities, as they are presumed to be
performing their duties regularly, absent any convincing
proof to the contrary.39
In sum, the prosecution successfully established appellants
guilt. Thus, her conviction must be affirmed.
WHEREFORE, the foregoing premises considered, the
appeal is DISMISSED. The Decision of the Court of Appeals
in CA-G.R. CR-HC No. 02718 is AFFIRMED.
SO ORDERED.

Espano vs.Courtof Appeals


Facts:Sometime July of 1991
, the narcotics division of theWPD conducted a by bust operation in
Zamora andPandacan Streets in Manila after the confirmation of
drugpushing reports in the same area.During the operation in the area,
after the policeofficers saw the accused selling something to a buyer,

theyapproached Espano and frisked him. The officers seized twoplastic


tea bags of marijuana from the accused. They thenlater asked the accused
if there are more marijuana in him,the Espano replied that there are more
in his house. Thus,the arresting officers went to Espanos house
andsubsequently confiscated ten more plastic tea bags
of marijuana.Espano was brought to the police, and charged inviolation
with RA 6425. The RTC Br.1of Manila convictedhim in the charges
whish was later affirmed
in totoby therespondent appellate court. Hence, this petition for
review. As defense, the petitioner contends that the drugsconfiscated are
inadmissible evidence against him beingsearch and seized
illegally.Issue:Was the contention correct?Held:Partly, the drugs
confiscated in his person isadmissible, however, the articles sought in his
residencecannot be admitted as evidence by the prosecution
beingunlawfully seized.Rule
113 Section 5(a) of the Rules of Court providesthat an officer may
without a warrant arrest a person when inhis presence:(a) Has
committed(b) Is actually committing(c) Or is about to commit a crimeIn
the case at bar, petitioner's arrest falls squarely under the aforecited rule.
He was caught in
flagranti
as a result of abuy-bust operation conducted by police officers on the
basisof information received regarding the illegal trade of drugswithin the
area of Zamora and Pandacan Streets, Manila.The police officer saw
petitioner handing over something toan alleged buyer. After the buyer
left, they searched him anddiscovered two cellophanes of marijuana. His
arrest was,therefore, lawful and the two cellophane bags of
marijuanaseized were admissible in evidence, being the fruits of
thecrime. As for the ten cellophane bags of marijuana found
atpetitioner's residence, however, the same inadmissiblein
evidence.The1987 Constitution guarantees freedom
againstunreasonable searches and seizures under Article III,Section 2
which provides:

surroundings under hisimmediate control. In this case, the ten cellophane


bags of marijuana seized at petitioner's house after his arrest atPandacan
and Zamora Streets do not fall under the saidexceptions.The articles
seized from petitioner during his arrestwere valid under the doctrine of
search made incidental to alawful arrest. The warrantless search made in
his house,however, which yielded ten cellophane bags of
marijuanabecame unlawful since the police officers were not armedwith
a search warrant at the time. Moreover, it was beyondthe reach and
control of petitioner.
Carroll v. United States, 267 U.S. 132 (1925), was a
decision by the United States Supreme Court which upheld
that the warrantless search of an automobile is known as the
automobile exception. The case has also been used to
increase the scope of warrantless searches.
Federal prohibition officers arranged an undercover buy of
liquor from George Carroll, an illicit dealer under
investigation, but the transaction was not completed. They
later saw Carroll and one Kiro driving on the highway from
Detroit to Grand Rapids, which they regularly patrolled.
They gave chase, pulled them over, and searched the car,
finding illegal liquor behind the rear seat.
The National Prohibition Act provided that officers could
make warrantless searches of vehicles, boats, or airplanes
when they had reason to believe illegal liquor was being
transported.[1]
Opinion of the court

The Court noted that Congress early obviated the need for a
warrant in border search situations[2], and Congress always
The right of the people to be secure in their persons,houses, papers and
recognized a necessary difference between searches of
effects against unreasonablesearches and seizures of whatever nature and buildings and vehicles for contraband goods, where it is not
for anypurposes shall be inviolable, and no search warrant or warrant of
practical to secure a warrant, because the vehicle can be
arrest shall issue except upon probable causeto be determined personally
quickly moved out of the locality or jurisdiction in which the
by the judge after examination under oath or affirmation of the
warrant must be sought.[3] The warrantless search under
complainantand the witnesses he may produce, and
particularlydescribing the place to be searched and the persons or things these circumstances was thus valid.
to be seized." An exception to the said rule is a warrantless
searchincidental to a lawful arrest for dangerous weapons or anything,
The Court held, however, that
which may be used as proof of the commission of an offense. It may
extend beyond the person of the onearrested to include the premises or

[i]t would be intolerable and unreasonable if a


prohibition agent were authorized to stop every
automobile on the chance of finding liquor, and thus
subject all persons lawfully using the highways to
the inconvenience and indignity of such a search... .
[T]hose lawfully within the country, entitled to use
the public highways, have a right to free passage
without interruption or search unless there is known
to a competent official, authorized to search,
probable cause for believing that their vehicles are
carrying contraband or illegal merchandise. [4]
The Court added that where the securing of a warrant is
reasonably practicable, it must be used.[5]
This became known as the Carroll doctrine: a vehicle could
be searched without a warrant if there was probable cause to
believe that evidence is present in the vehicle, coupled with
exigent circumstances to believe that the vehicle could be
removed from the area before a warrant could be obtained.
People v. Lo Ho Wing, 193 SCRA 122 F: Peter Lo , together
with co-accused Lim Cheng Huat alias Antonio Lim and
Reynaldo Tia, were charged with a violation of the
Dangerous Drugs Act, for the transport of metamphetamine
hydrochloride, otherwise known as "shabu". The drug was
contained in tea bags inside tin cans which were placed
inside their luggages. Upon arrival from Hongkong, they
boarded the taxis at the airport which were apprehended by
CIS operatives. Their luggages were subsequently searched
where the tea bags were opened and found to contain shabu.
Only Lo and Lim were convicted. Tia was discharged as a
state witness, who turned out to be a " deep penetration
agent" of the CIS in its mission to bust the drug syndicate .
Issue: W/N the search and seizure was legal. HELD: YES
That search and seizure must be supported by a valid warrant
is not an absolute rule. One of the exceptions thereto is a
search of a moving vehicle. The circumstance of the case
clearly show that the serach in question was made as regards
a moving vehicle.
Ads by Google
Full-Text Online Library Research online. Books,
journals, articles at Questia Online Library.
www.Questia.com
Buy Bag-kok Online Bags of Top International Brands Free

Shipping and 7 Days Return!


www.zalora.com.ph/Bag-kok
Therefore, a valid warrant was not necessary to effect the
search on appellant and his co-accused. It was firmly
established from the factual findings of the court that the
authorities had reasonable ground to believe that appellant
would attempt to bring in contraband and transport within
the country. The belief was based on intelligence reports
gathered from surveillance activities on the suspected
syndicate, of which appellant was touted to be amember.
Aside from this, they were also certain as to the expected
date and time of arrival of the accused from China via
Hongkong. But such knowledge was insufficient to enable
them to fulfill the requiremnents for the issuance of a search
warrant. Still and all, the important thing is that there was
probable cause to conduct the warrantless search, which
must still be present in the case.

evidence against him.Issue:Were the arrest, search and seizure illegal?


Held:No. It is legal, being one of the instances providedby statutory
provision that warrantless arrest and seizure canbe effected. It was search
pursuant to a lawful arrest andsearch in a moving vehicle. Accused was
searched and arrested whiletransporting prohibited drugs (hashish). A
crime was actuallybeing committed by the accused and he was caught in
flagrante delicto
. Thus, the search made upon his personaleffects falls squarely under
paragraph (
1
) of the foregoingprovisions of law (
committed a crime)
, which allow awarrantless search incident to a lawful arrest.Note:
Dissenting Opinions of Justices Narvasa and Cruz
Justice Narvasa:
If, on the other, a person is searched without awarrant, or under
circumstances other than those justifyingan arrest without warrant in
accordance with law, supra,merely on suspicion that he is engaged in
Source: http://www.shvoong.com/law-and-politics/1767269- some feloniousenterprise, and in order to discover if he has
case-digest-people-vs-lo/#ixzz1sFwkqx6K
indeedcommitted a crime, it is not only the arrest which is illegal butalso,
the search on the occasion thereof, as being "the fruit of the poisonous
tree." In that event, any evidence taken, evenif confirmatory of the initial
People vs.Malsmedt
suspicion, is inadmissible "for anypurpose in any proceeding." They are
With substantial dissenting opinions of Justices Narvasa
merely fishing for evidence to corroborate their speculations.The search
(
was not made by virtue of a warrant or as an incident of a lawful
CJ)and Cruz
warrantless arrest, i.e., under circumstances sufficient to engender a
reasonable belief thatsome crime was being or about to be committed, or
Facts:Upon reports that vehicles coming from Sagada aretransporting
marijuana and other prohibited drugs, the Firstregional NARCOM was had justbeen committed. There was no intelligent and intentionalwaiver
of the right against unreasonable searches andseizure.The search was
ordered to set-up a temporarycheckpoint area to monitor all vehicle
therefore illegal, since the lawrequires that there first be a lawful arrest of
coming from Sagada.The accused, Mikael Malmstedt, a Swedish
an individualbefore a search of his body and his belongings may licitly
nationalwas boarding in the rear end of the bus when the officers
bemade. The process cannot be reversed, i.e., a search be firstundertaken,
areinspecting. One officer saw a bulging object in the waist of the
and then an arrest effected, on the strength of the evidence yielded by the
accused and asked Malmstedt to show his passport andother
identification documents. The accused failed to complyand was asked to search. An arrest made in thatcase would be unlawful, and the search
undertaken as anincident of such an unlawful arrest, also
bring out whatever the object bulging in hisstomach. It turns out to be
unlawful.Search may extend to the area "within his immediate
haishish
a derivative of marijuana.The accused was invited to step out of the bus control,defined:The area from which said person arrested mightgain
possession of a weapon or destructible evidence.
for questioning. Before doing so, the accused first get his twotraveling
bags from the luggage carrier of the bus. Theofficers inspected the bags
and saw a suspicious teddy bear,which turned out to be containing the
same materialsconfiscated from the accused.The RTC of La Trinidad
Mustang Lumber vs.CA
Benguet later convicted theaccused with the violation of RA 6425. hence A consolidated petitionFacts:First Case:On1April1990
this petition for reversal.The accused contended that his arrest and
, acting on an information that ahuge stockpile of narra flitches, shorts,
seizureof his personal effects are illegal there being conductedwithout
and slabs were seeninside the lumberyard of the petitioner in Valenzuela,
warrant, therefore follows that articles confiscated isinadmissible
MetroManila, the SAID organized a team of foresters andpolicemen and

sent it to conduct surveillance at the saidlumberyard. In the course


thereof, the team members sawcoming out from the lumberyard the
petitioner's truck, withPlate No. CCK-322, loaded with lauan and
almaciga lumber of assorted sizes and dimensions. Since the driver
could notproduce the required invoices and transport documents,
theteam seized the truck together with its cargo and impoundedthem at
the DENR compound at Visayas Avenue, QuezonCity. The team was
not able to gain entry into the premisesbecause of the refusal of the
owner. 2The special Action and nvestigation Division of DENr also
procured a search warrant from Jusge AdrianoOsorio of RTC
Valenzuela, by virtue of the warrant, the teamseized for truckloads of
narra woods including almaciagaand supa. Moreover, the lumberyard of
the petitioner wasalso placed under administrative seizure. For failure
toproduce certificates of lumber origins, auxiliary invoices, tallysheets
and delivery receipts.Subsequently, the Sec of DENR Factoran issued
anorder confiscating the woods seized in the truck of thepetitioner as well
as those found in their lumberyard.
Thus, the petitioner filed a petitioner for certiorariand prohibition
contending that the search and seizureoperation by the respondents is a
violation under Sec 2 Art IIIof the Constitution for not having a valid
search warrant.
Second case:
PP vs.
C
apulongetal
This case deals with whether the Forestry Codewhere the petitioner
allegedly violated refers to either
timber
or
lumber
Issue:Was the warrantless search and seizure invalid?Held:No. It is a
valid warrantless search being one of thestatutory instances that
accepted.Search of a moving vehicle is one of the fivedoctrinally
accepted exceptions to the constitutional mandatethat no search and
seizure shall be made except by a virtueof a warrant. Thus a search could
be lawfully conducted on amoving vehicle without a search warrant.In
the case at bar, the conducted search andseizure is indeed without a valid
warrant, however, it wasconducted to search the materials that can be
found in amoving vehicle, which is the truck of the first case.
Papa vs.Mago
Facts:Petitioner Martin Alagao, head of the counter-intelligence unit of
the Manila Police Department, acting upona reliable information
received on November 3,1966 to theeffect that a certain shipment of

personal effects, allegedlymisdeclared and undervalued, would be


released thefollowing day from the customs zone of the port of
Manilaand loaded on two trucks, and upon orders of petitioner Ricardo
Papa, Chief of Police of Manila and a duly deputizedagent of the Bureau
of Customs, conducted surveillance atgate No.1of the customs
zone.When the trucks left gate No.1at about 4:30in theafternoon of
November 4,1966, elements of the counter-intelligence unit went after
the trucks and intercepted them atthe Agrifina Circle, Ermita, Manila.
The load of the two trucks,consisting of nine bales of goods, and the two
trucks, wereseized on instructions of the Chief of Police.
Uponinvestigation, a person claimed ownership of the goods andshowed
to the policemen a "Statement and Receipts of Duties Collected on
Informal Entry No.147-5501", issued bythe Bureau of Customs in the
name of a certain BienvenidoNaguit.The respondent Mago, filed a
petition for mandamusand certiorari before the CFI Manila contending
that thesearch and seizure is illegal for lack of a valid warrant.Moreover,
she also contends that such articles sought fromher is not included by the
law for prohibited importation andthat it no longer under the control of
the Tariff and Customscode for it (articles) were already sold to the
petitioner.She also contends that the search seizureconducted by the
respondents are illegally being madeoutside the jurisdiction of the BOC
and that the subsequentsearch warrant issued by the collector of customs
is not validbeing not issued by a judge.The respondent Mago filed an exparte motion torelease the confiscated articles upon her posting a
bond.This motion was then granted by the respondent
JudgeJarencio.Issue:Was the seizure of the goods unlawful? And that
theBOC has no jurisdiction over the articles sought?Held:No. it is a valid
seizure.The Chief of the Manila Police Department, RicardoG. Papa,
having been deputized in writing by theCommissioner of Customs,
could, for the purposes of theenforcement of the customs and tariff laws,
effect searches,seizures, and arrests, and it was his duty to make
seizure,among others, of any cargo, articles or other movableproperty
when the same may be subject to forfeiture or liablefor any fine imposed
under customs and tariff laws. He couldlawfully open and examine any
box, trunk, envelope or other container wherever found when he had
reasonable cause tosuspect the presence therein of dutiable articles
introducedinto the Philippines contrary to law; and likewise to
stop,search and examine any vehicle, beast or person
reasonablysuspected of holding or conveying such article as aforesaid.It
cannot be doubted, therefore, that petitioner Ricardo G.Papa, Chief of
Police of Manila, could lawfully effect thesearch and seizure of the goods
in question. The Tariff andCustoms Code authorizes him to demand
assistance of anypolice officer to effect said search and seizure, and the
latter has the legal duty to render said assistance. This was whathappened
precisely in the case of Lt. Martin Alagao who, withhis unit, made the

search and seizure of the two trucksloaded with the nine bales of goods in
question at the Agrifina Circle. He was given authority by the Chief of
Policeto make the interception of the cargo.Petitioner Martin Alagao and
his companionpolicemen had authority to effect the seizure without
anysearch warrant issued by a competent court.The Tariff and Customs
Code does not require saidwarrant in the instant case. The Code
authorizes personshaving police authority under Section 2203 of the
Tariff andCustoms Code to enter, pass through or search any
land,inclosure, warehouse, store or building, not being a dwellinghouse;
and also to inspect, search and examine any vesselor aircraft and any
trunk, package, box or envelope or anyperson on board, or stop and
search and examine anyvehicle, beast or person suspected of holding or
conveyingany dutiable or prohibited article introduced into
thePhilippines contrary to law, without mentioning the need of asearch
warrant in said cases.It is our considered view, therefore, that except inthe
case of the search of a dwelling house, personsexercising police authority
under the customs law may effectsearch and seizure without a search
warrant in theenforcement of customs laws.
Note:
The Bureau of Customs has the duties, powers and jurisdiction, among
others,
(1)
to assess and collect all lawfulrevenues from imported articles, and all
other dues, fees,charges, fines and penalties, accruing under the tariff
andcustoms laws;
(2)
to prevent and suppress smuggling andother frauds upon the customs;
and
(3)
to enforce tariff andcustoms laws.The goods in question were imported
from Hongkong, asshown in the "Statement and Receipts of Duties
Collected onInformal Entry." As long as the importation has not
beenterminated the imported goods remain under the jurisdictionof the
Bureau of Customs.
I
m
portation
is deemed terminated only upon thepayment of the duties, taxes and
other charges upon thearticles, or secured to be paid, at the port of entry
and thelegal permit for withdrawal shall have been granted.
The payment of the duties, taxes, fees and other charges must be in full.
Pacis vs. Pa

m
aran
Facts:The petitioner, Pedro Pacis, acting Collector of Customs in the Port
of Manila, issued a warrant of seizureand detention for an automobile
(Mercury
1
957) owned bythe respondent Ricardo Santos, who, by the records of
hisoffice, have not fully paid the customs duty collectible fromthe car.The
respondent Santos filed a suit against thepetitioner on the ground
usurpation of justice, for thepetitioner has no power granted by the
constitution to issue awarrant, which, is s judicial function of a
judge.Issue:Whether or not the petitioner is clothe with jurisdiction to
issue a warrant.Held:Yes. The Collector of customs may order seizure
of untaxed goods being without being liable for usurpation of judicial
function as provided under the tariff and customsCode.
Hizon vs.
C
A
Facts:Sometime
1
992, the Maritime Command of the PNPPalawan arrested the petitioner
who are allegedly conducting
muro-ami
a prohibited system of fishing while on board FBrobinson.The PNP filed
a complaint against the petitioners(3
1
of 35) charging them in violation of the Fishery Laws inthe Philippines,
by using sodium Cyanide in catching fishes.The RTC Puerto princesa
convicted the petitionersand was affirmed by the CA., hence this
petition. As defense, the petitioners alleged that the searchand seizure is
illegal for the absence of a warrant and thefishes sought cannot be
admitted as evidence against them.Issue:Was the arrest, search and
seizure invalid?Held:No. it is a valid being a search in a moving
motor vehicle.Our Constitution proscribes search and seizure andthe
arrest of persons without a judicial warrant.
1
6 As ageneral rule, any evidence obtained without a judicial warrantis
inadmissible for any purpose in any proceeding. The ruleis, however,
subject to certain exceptions. Some of theseare:(
1
) A search incident to a lawful of arrest;(2) Seizure of evidence in plain
view;(3) Search of a moving motor vehicle;and(4) Search in violation of
customs laws.Search and seizure without search warrant of vessels and

aircrafts for violations of customs laws havebeen the traditional exception


to the constitutionalrequirement of a search warrant. It is rooted on
therecognition that a vessel and an aircraft, like motor vehicles,can be
quickly moved out of the locality or jurisdiction inwhich the search
warrant must be sought and secured.Yielding to this reality, judicial
authorities have not required asearch warrant of vessels and aircrafts
before their searchand seizure can be constitutionally effected.The same
exception ought to apply to seizures of fishing vessels and boats
breaching our fishery laws. Thesevessels are normally powered by highspeed motors thatenable them to elude arresting ships of the Philippine
Navy,the Coast Guard and other government authorities enforcingour
fishery laws.Thus, in the case at bar, the warrantless search onthe F/B
Robinson, a fishing boat suspected of havingengaged in illegal fishing is
valid. The fish and other evidenceseized in the course of the search were
properly admitted bythe trial court.
People vs. Que
Facts: After receiving information that a truck loader withplate number
PAD 54
8
contains illegally cut lumber and willpass through Ilocos Norte, the PNP
then proceeded andpatrol along the vicinity of General Segundo Ave.
Laoag City.Sometime March
1
994, the officers saw the truckwith the plate number indicated, they
followed and thenapprehended in Marcos bridge. The officers then
arrested thepetitioners for failure to show the necessary documents
suchas (
1
) Certificate of Lumber Origin (2) Certificate of Transport Agreement
(3) Auxiliary invoice and others.The petitioners were charged for
violating theForestry Code and was convicted by RTC Laoag.The
petitioners filed an appeal before the SCcontending that the search and
seizure is illegal and theconfiscated articles cannot be used as evidence
againstthem being fruits of a poisonous tree.Issue:Was there an illegal
search?Held:No. The search is legal being conducted in searchof a
moving vehicle. As held in the PP vs. Bagista explaining theexception
and rational of a validity in search of a movingvehicle;With regard to the
search of moving vehicles, thishad been justified on the ground that the
mobility of motor vehicles makes it possible for the vehicle to be
searched tomove out of the locality or jurisdiction in which the
warrantmust be sought.This in no way, however, gives the police
officersunlimited discretion to conduct warrantless searches
of automobiles in the absence of probable cause. When avehicle is
stopped and subjected to an extensive search,such a warrantless search
has been held to be valid as longas the officers conducting the search

have reasonable or probable cause to believe before search that they will
find theinstrumentality or evidence pertaining to a crime, in thevehicle to
be searched. (citations omitted; emphasissupplied)The police officers in
the case at bar had probablecause to search appellant's truck. Amember
of the ProvincialTask Force on Illegal Logging received a reliable
informationthat a ten-wheeler truck bearing plate number PAD-54
8
loaded with illegal lumber would pass through Ilocos Norte.Two weeks
later, while members of the Provincial Task Forcewere patrolling along
General Segundo Avenue, they saw theten-wheeler truck described by
the informant. When theyapprehended it at the Marcos Bridge, accusedappellant, theowner of the truck and the cargo, admitted that there
weresawn lumber in between the coconut slabs. When the policeofficers
asked for the lumber's supporting documents,accused-appellant could
not present any. The foregoingcircumstances are sufficient to prove the
existence of probable cause which justified the extensive search
of appellant's truck even without a warrant. Thus, the 25
8
pieces of tanguile lumber were lawfully seized and were thus
Constitutional Law II (Bill of Rights): Case Briefs: Dennis G.
Libunao UC College of Law
37
properly admitted as evidence to prove the guilt of accused-appellant.
G.R. No. 190889

January 10, 2011

ELENITA C. FAJARDO, Petitioner,


vs.
PEOPLE OF THE PHILIPPINES, Respondent.
DECISION

The facts:
Petitioner, Elenita Fajardo, and one Zaldy Valerio (Valerio)
were charged with violation of P.D. No. 1866, as amended,
before the RTC, Branch 5, Kalibo, Aklan, committed as
follows:
That on or about the 28th day of August, 2002, in the
morning, in Barangay Andagao, Municipality of Kalibo,
Province of Aklan, Republic of the Philippines, and within
the jurisdiction of this Honorable Court, the above-named
accused, conspiring, confederating and mutually helping one
another, without authority of law, permit or license, did then
and there, knowingly, willfully, unlawfully and feloniously
have in their possession, custody and control two (2)
receivers of caliber .45 pistol, [M]odel [No.] M1911A1 US
with SN 763025 and Model [No.] M1911A1 US with
defaced serial number, two (2) pieces short magazine of M16
Armalite rifle, thirty-five (35) pieces live M16 ammunition
5.56 caliber and fourteen (14) pieces live caliber .45
ammunition, which items were confiscated and recovered
from their possession during a search conducted by members
of the Provincial Intelligence Special Operation Group,
Aklan Police Provincial Office, Kalibo, Aklan, by virtue of
Search Warrant No. 01 (9) 03 issued by OIC Executive
Judge Dean Telan of the Regional Trial Court of Aklan.3
When arraigned on March 25, 2004, both pleaded not guilty
to the offense charged.4 During pre-trial, they agreed to the
following stipulation of facts:

NACHURA, J.:

1. The search warrant subject of this case exists;

At bar is a Petition for Review on Certiorari under Rule 45


of the Rules of Court, seeking the reversal of the February
10, 2009 Decision1 of the Court of Appeals (CA), which
affirmed with modification the August 29, 2006 decision2 of
the Regional Trial Court (RTC), Branch 5, Kalibo, Aklan,
finding petitioner guilty of violating Presidential Decree
(P.D.) No. 1866, as amended.

2. Accused Elenita Fajardo is the same person


subject of the search warrant in this case who is a
resident of Sampaguita Road, Park Homes,
Andagao, Kalibo, Aklan;

3. Accused Zaldy Valerio was in the house of Elenita Petitioner was seen tucking a .45 caliber handgun between
Fajardo in the evening of August 27, 2002 but does her waist and the waistband of her shorts, after which, she
not live therein;
entered the house and locked the main door.

3. Fourteen (14) pieces of live ammos of Caliber 45


pistol.

Since petitioner and Valerio failed to present any documents


4. Both accused were not duly licensed firearm
To prevent any violent commotion, the policemen desisted
showing their authority to possess the confiscated firearms
holders;
from entering petitioners house but, in order to deter Valerio and the two recovered receivers, a criminal information for
from evading apprehension, they cordoned the perimeter of violation of P.D. No. 1866, as amended by Republic Act
5. The search warrant was served in the house of
the house as they waited for further instructions from P/Supt. (R.A.) No. 8294, was filed against them.
accused Elenita Fajardo in the morning of August
Mendoza. A few minutes later, petitioner went out of the
28, 2002; and
house and negotiated for the pull-out of the police troops. No For their exoneration, petitioner and Valerio argued that the
issuance of the search warrant was defective because the
agreement materialized.
6. The accused Elenita Fajardo and Valerio were not
allegation contained in the application filed and signed by
arrested immediately upon the arrival of the military At around 2:00 a.m. and 4:00 a.m. of August 28, 2002,
SPO1 Tan was not based on his personal knowledge. They
personnel despite the fact that the latter allegedly
Senior Police Officer 2 Clemencio Nava (SPO2 Nava), who quoted this pertinent portion of the application:
saw them in possession of a firearm in the evening
was posted at the back portion of the house, saw Valerio
of August 27, 2002.5
emerge twice on top of the house and throw something. The That this application was founded on confidential
information received by the Provincial Director, Police Supt.
discarded objects landed near the wall of petitioners house
As culled from the similar factual findings of the RTC and
and inside the compound of a neighboring residence. SPO2 Edgardo Mendoza.7
the CA,6 these are the chain of events that led to the filing of Nava, together with SPO1 Teodoro Neron and Jerome T.
the information:
They further asserted that the execution of the search warrant
Vega (Vega), radio announcer/reporter of RMN DYKR, as
witness, recovered the discarded objects, which turned out to was infirm since petitioner, who was inside the house at the
In the evening of August 27, 2002, members of the
time of the search, was not asked to accompany the
be two (2) receivers of .45 caliber pistol, model no.
Provincial Intelligence Special Operations Group (PISOG)
M1911A1 US, with serial number (SN) 763025, and model policemen as they explored the place, but was instead
were instructed by Provincial Director Police Superintendent no. M1911A1 US, with a defaced serial number. The
ordered to remain in the living room (sala).
Edgardo Mendoza (P/Supt. Mendoza) to respond to the
recovered items were then surrendered to SPO1 Nathaniel A.
complaint of concerned citizens residing on Ilang-Ilang and Tan (SPO1 Tan), Group Investigator, who utilized them in
Petitioner disowned the confiscated items. She refused to
Sampaguita Roads, Park Homes III Subdivision, Barangay
sign the inventory/receipt prepared by the raiding team,
applying for and obtaining a search warrant.
Andagao, Kalibo, Aklan, that armed men drinking liquor at
because the items allegedly belonged to her brother, Benito
the residence of petitioner were indiscriminately firing guns. The warrant was served on petitioner at 9:30 a.m. Together
Fajardo, a staff sergeant of the Philippine Army.
with a barangay captain, barangay kagawad, and members of
Along with the members of the Aklan Police Provincial
the media, as witnesses, the police team proceeded to search Petitioner denied that she had a .45 caliber pistol tucked in
Office, the elements of the PISOG proceeded to the area.
her waistband when the raiding team arrived. She averred
petitioners house. The team found and was able to
Upon arrival thereat, they noticed that several persons
that such situation was implausible because she was wearing
confiscate the following:
scampered and ran in different directions. The responding
garterized shorts and a spaghetti-strapped hanging blouse. 8
team saw Valerio holding two .45 caliber pistols. He fired
1. Two (2) pieces of Short Magazine of M16
Ruling of the RTC
shots at the policemen before entering the house of
Armalite Rifle;
petitioner.
The RTC rejected the defenses advanced by accused, holding
2. Thirty five (35) pieces of live M16 ammos 5.56
that the same were already denied in the Orders dated
Caliber; and
December 31, 2002 and April 20, 2005, respectively denying

the Motion to Quash Search Warrant and Demurrer to


Evidence. The said Orders were not appealed and have thus
attained finality. The RTC also ruled that petitioner and
Valerio were estopped from assailing the legality of their
arrest since they participated in the trial by presenting
evidence for their defense. Likewise, by applying for bail,
they have effectively waived such irregularities and defects.

of a prohibited weapon be its owner or a borrower. To


accomplish the object of this law[,] the proprietary concept
of the possession can have no bearing whatsoever.

Consequently, petitioner and Valerio were convicted of


illegal possession of firearms and explosives, punishable
under paragraph 2, Section 1 of P.D. No. 1866, as amended
by R.A. No. 8294, which provides:

xxxx

The penalty of prision mayor in its minimum period and a


x x x. [I]n order that one may be found guilty of a violation
fine of Thirty thousand pesos (P30,000.00) shall be imposed
of the decree, it is sufficient that the accused had no
if the firearm is classified as high powered firearm which
authority or license to possess a firearm, and that he intended includes those with bores bigger in diameter than .38 caliber
In finding the accused liable for illegal possession of
to possess the same, even if such possession was made in
firearms, the RTC explained:
and 9 millimeter such as caliber .40, .41, .44, .45 and also
good faith and without criminal intent.
lesser calibered firearms but considered powerful such as
Zaldy Valerio, the bodyguard of Elenita Fajardo, is a former
caliber .357 and caliber .22 center-fire magnum and other
soldier, having served with the Philippine Army prior to his x x x x
firearms with firing capability of full automatic and by burst
separation from his service for going on absence without
of two or three: Provided, however, That no other crime was
To
convict
an
accused
for
illegal
possession
of
firearms
and
leave (AWOL). With his military background, it is safe to
committed by the person arrested.
explosive under P.D. 1866, as amended, two (2) essential
conclude that Zaldy Valerio is familiar with and
elements must be indubitably established, viz.: (a) the
knowledgeable about different types of firearms and
Both were sentenced to suffer the penalty of imprisonment
existence
of
the
subject
firearm
ammunition
or
explosive
ammunitions. As a former soldier, undoubtedly, he can
of six (6) years and one (1) day to twelve (12) years of
which may be proved by the presentation of the subject
assemble and disassemble firearms.
prision mayor, and to pay a fine of P30,000.00.
firearm or explosive or by the testimony of witnesses who
It must not be de-emphasize[d] that the residence of Elenita saw accused in possession of the same, and (b) the negative On September 1, 2006, only petitioner filed a Motion for
fact that the accused has no license or permit to own or
Fajardo is definitely not an armory or arsenal which are the
Reconsideration, which was denied in an Order dated
possess
the
firearm,
ammunition
or
explosive
which
fact
usual depositories for firearms, explosives and ammunition.
October 25, 2006. Petitioner then filed a Notice of Appeal
Granting arguendo that those firearms and ammunition were may be established by the testimony or certification of a
with the CA.
representative of the PNP Firearms and Explosives Unit that
left behind by Benito Fajardo, a member of the Philippine
Ruling of the CA
army, the fact remains that it is a government property. If it is the accused has no license or permit to possess the subject
so, the residence of Elenita Fajardo is not the proper place to firearm or explosive (Exhibit G).
The CA concurred with the factual findings of the RTC, but
store those items. The logical explanation is that those items
The judicial admission of the accused that they do not have
disagreed with its conclusions of law, and held that the
are stolen property.
permit or license on the two (2) receivers of caliber .45
search warrant was void based on the following
pistol, model M1911A1 US with SN 763025 and model
observations:
xxxx
M1911A1 of M16 Armalite rifle, thirty-five (35) pieces live
The rule is that ownership is not an essential element of
M16 ammunition, 5.56 caliber and fourteen (14) pieces live [A]t the time of applying for a search warrant, SPO1
illegal possession of firearms and ammunition. What the law caliber .45 ammunition confiscated and recovered from their Nathaniel A. Tan did not have personal knowledge of the fact
that appellants had no license to possess firearms as required
requires is merely possession which includes not only actual possession during the search conducted by members of the
by law. For one, he failed to make a categorical statement on
physical possession but also constructive possession or the
PISOG, Aklan Police Provincial Office by virtue of Search
subjection of the thing to ones control and management.
Warrant No. 01 (9) 03 fall under Section 4 of Rule 129 of the that point during the application. Also, he failed to attach to
the application a certification to that effect from the Firearms
This has to be so if the manifest intent of the law is to be
Revised Rules of Court.9
and Explosives Office of the Philippine National Police. x x
effective. The same evils, the same perils to public security,
x, this certification is the best evidence obtainable to prove
which the law penalizes exist whether the unlicensed holder

that appellant indeed has no license or permit to possess a


firearm. There was also no explanation given why said
certification was not presented, or even deemed no longer
necessary, during the application for the warrant. Such vital
evidence was simply ignored.10

M16 Armalite rifle, thirty-five (35) pieces of live M16


ammunition 5.56 caliber, and fourteen (14) pieces of live
caliber .45 ammunition is punishable under paragraph 2 of
the said section, viz.:

The penalty of prision mayor in its minimum period and a


Resultantly, all firearms and explosives seized inside
fine of Thirty thousand pesos (P30,000.00) shall be imposed
petitioners residence were declared inadmissible in
if the firearm is classified as high powered firearm which
evidence. However, the 2 receivers recovered by the
includes those with bores bigger in diameter than .38 caliber
policemen outside the house of petitioner before the warrant and 9 millimeter such as caliber .40, 41, .44, .45 and also
was served were admitted as evidence, pursuant to the plain lesser calibered firearms but considered powerful such as
view doctrine.
caliber .357 and caliber .22 center-fire magnum and other
firearms with firing capability of full automatic and by burst
Accordingly, petitioner and Valerio were convicted of illegal of two or three: Provided, however, That no other crime was
possession of a part of a firearm, punishable under paragraph committed by the person arrested.14
1, Section 1 of P.D. No. 1866, as amended. They were
sentenced to an indeterminate penalty of three (3) years, six On the other hand, illegal possession of the two (2) receivers
(6) months, and twenty-one (21) days to five (5) years, four of a .45 caliber pistol, model no. M1911A1 US, with SN
(4) months, and twenty (20) days of prision correccional, and 763025, and Model M1911A1 US, with a defaced serial
ordered to pay a P20,000.00 fine.
number, is penalized under paragraph 1, which states:

old P.D. No. 1866 which set a standard penalty for the illegal
possession of any kind of firearm. Section 1 of the old law
reads:
Section 1. Unlawful Manufacture, Sale, Acquisition,
Disposition or Possession of Firearms or Ammunition or
Instruments Used or Intended to be Used in the Manufacture
of Firearms of Ammunition. The penalty of reclusion
temporal in its maximum period to reclusion perpetua shall
be imposed upon any person who shall unlawfully
manufacture, deal in, acquire dispose, or possess any
firearms, part of firearm, ammunition, or machinery, tool or
instrument used or intended to be used in the manufacture of
any firearm or ammunition. (Emphasis ours.)

By virtue of such changes, an information for illegal


possession of firearm should now particularly refer to the
paragraph of Section 1 under which the seized firearm is
classified, and should there be numerous guns confiscated,
each must be sorted and then grouped according to the
categories stated in Section 1 of R.A. No. 8294, amending
11
Petitioner moved for reconsideration, but the motion was
Sec. 1. Unlawful manufacture, sale, acquisition, disposition P.D. No. 1866. It will no longer suffice to lump all of the
12
denied in the CA Resolution dated December 3, 2009.
or possession of firearms or ammunition or instruments used seized firearms in one information, and state Section 1, P.D.
Hence, the present recourse.
or intended to be used in the manufacture of firearms or
No. 1866 as the violated provision, as in the instant case, 16
ammunition. The penalty of prision correccional in its
because different penalties are imposed by the law,
At the onset, it must be emphasized that the information filed maximum period and a fine of not less than Fifteen thousand depending on the caliber of the weapon. To do so would
against petitioner and Valerio charged duplicitous offenses
pesos (P15,000.00) shall be imposed upon any person who
result in duplicitous charges.
contrary to Section 13 of Rule 110 of the Rules of Criminal shall unlawfully manufacture, deal in, acquire, dispose, or
Procedure, viz.:
possess any low powered firearm, such as rimfire handgun, . Ordinarily, an information that charges multiple offenses
380 or .32 and other firearm of similar firepower, part of
merits a quashal, but petitioner and Valerio failed to raise
Sec. 13. Duplicity of offense. A complaint or information
firearm, ammunition, or machinery, tool or instrument used this issue during arraignment. Their failure constitutes a
must charge but one offense, except only in those cases in
or intended to be used in the manufacture of any firearm or
waiver, and they could be convicted of as many offenses as
15
which existing laws prescribe a single punishment for
ammunition: Provided, That no other crime was committed. there were charged in the information.17 This accords
various offenses.
propriety to the diverse convictions handed down by the
This is the necessary consequence of the amendment
courts a quo.
A reading of the information clearly shows that possession of introduced by R.A. No. 8294, which categorized the kinds of
the enumerated articles confiscated from Valerio and
firearms proscribed from being possessed without a license, Further, the charge of illegal possession of firearms and
petitioner are punishable under separate provisions of
according to their firing power and caliber. R.A. No. 8294
ammunition under paragraph 2, Section 1 of P.D. No. 1866,
Section 1, P.D. No. 1866, as amended by R.A. No. 8294.13
likewise mandated different penalties for illegal possession
as amended by R.A. No. 8294, including the validity of the
Illegal possession of two (2) pieces of short magazine of
of firearm according to the above classification, unlike in the search warrant that led to their confiscation, is now beyond

the province of our review since, by virtue of the CAs


Decision, petitioner and Valerio have been effectively
acquitted from the said charges. The present review is
consequently only with regard to the conviction for illegal
possession of a part of a firearm.
The Issues

affirmation of the complainant and the witnesses he may


produce, and particularly describing the place to be searched
and the persons or things to be seized.

Tested against these standards, we find that the seizure of the


two receivers of the .45 caliber pistol outside petitioners
house falls within the purview of the plain view doctrine.

Complementing this provision is the exclusionary rule


embodied in Section 3(2) of the same article

First, the presence of SPO2 Nava at the back of the house


and of the other law enforcers around the premises was
justified by the fact that petitioner and Valerio were earlier
seen respectively holding .45 caliber pistols before they ran
inside the structure and sought refuge. The attendant
circumstances and the evasive actions of petitioner and
Valerio when the law enforcers arrived engendered a
reasonable ground for the latter to believe that a crime was
being committed. There was thus sufficient probable cause
for the policemen to cordon off the house as they waited for
daybreak to apply for a search warrant.

(2) Any evidence obtained in violation of this or the


Petitioner insists on an acquittal and avers that the discovery preceding section shall be inadmissible for any purpose in
of the two (2) receivers does not come within the purview of any proceeding.
the plain view doctrine. She argues that no valid intrusion
There are, however, several well-recognized exceptions to
was attendant and that no evidence was adduced to prove
the foregoing rule. Thus, evidence obtained through a
that she was with Valerio when he threw the receivers.
warrantless search and seizure may be admissible under any
Likewise absent is a positive showing that any of the two
receivers recovered by the policemen matched the .45 caliber of the following circumstances: (1) search incident to a
lawful arrest; (2) search of a moving motor vehicle; (3)
pistol allegedly seen tucked in the waistband of her shorts
when the police elements arrived. Neither is there any proof search in violation of custom laws; (4) seizure of evidence in Secondly, from where he was situated, SPO2 Nava clearly
that petitioner had knowledge of or consented to the alleged plain view; and (5) when the accused himself waives his
saw, on two different instances, Valerio emerge on top of the
right against unreasonable searches and seizures. 18
throwing of the receivers.
subject dwelling and throw suspicious objects. Lastly,
considering the earlier sighting of Valerio holding a pistol,
Under the plain view doctrine, objects falling in the "plain
Our Ruling
SPO2 Nava had reasonable ground to believe that the things
view" of an officer, who has a right to be in the position to
thrown might be contraband items, or evidence of the
We find merit in the petition.
have that view, are subject to seizure and may be presented
offense they were then suspected of committing. Indeed,
as evidence.19 It applies when the following requisites
when subsequently recovered, they turned out to be two (2)
First, we rule on the admissibility of the receivers. We hold
concur: (a) the law enforcement officer in search of the
receivers of .45 caliber pistol.
that the receivers were seized in plain view, hence,
evidence has a prior justification for an intrusion or is in a
admissible.
position from which he can view a particular area; (b) the
The pertinent portions of SPO2 Navas testimony are
discovery of the evidence in plain view is inadvertent; and
elucidating:
No less than our Constitution recognizes the right of the
(c) it is immediately apparent to the officer that the item he
people to be secure in their persons, houses, papers, and
observes may be evidence of a crime, contraband, or
Q When you arrived in that place, you saw
effects against unreasonable searches and seizures. This right otherwise subject to seizure. The law enforcement officer
policemen?
is encapsulated in Article III, Section 2, of the Constitution, must lawfully make an initial intrusion or properly be in a
which states:
A Yes, sir.
position from which he can particularly view the area. In the
course of such lawful intrusion, he came inadvertently across
Sec. 2. The right of the people to be secure in their persons, a piece of evidence incriminating the accused. The object
Q What were they doing?
houses, papers, and effects against unreasonable searches
must be open to eye and hand, and its discovery
and seizures of whatever nature and for any purpose shall be inadvertent.20
A They were cordoning the house.
inviolable, and no search warrant or warrant of arrest shall
Q You said that you asked your assistant team leader
issue except upon probable cause to be determined
Deluso about that incident. What did he tell you?
personally by the judge after examination under oath or

A Deluso told me that a person ran inside the house


carrying with him a gun.
Q And this house you are referring to is the house
which you mentioned is the police officers were
surrounding?

xxxx
Q Can you tell the Honorable Court who was that
person who threw that something outside the house?
A It was Zaldy Valerio.

A Yes, sir.

COURT: (to witness)

Q Now, how long did you stay in that place, Mr.


Witness?

Q Before the incident, you know this person Zaldy


Valerio?

A I stayed there when I arrived at past 10:00 oclock


up to 12:00 oclock the following day.

A Yes, sir.

A I saw there the lower [part] of the receiver of cal.


45.
xxxx
Q Mr. Witness, at around 4:00 oclock that early
morning of August 28, 2002, do you recall another
unusual incident?
A Yes, sir.
Q And can you tell us what was that incident?

Q Why do you know him?


Q At about 2:00 oclock in the early morning of
August 28, 2002, can you recall where were you?

A Because we were formerly members of the Armed


Forces of the Philippines.

A Yes, sir.

A I saw a person throwing something there and the


one that was thrown fell on top of the roof of another
house.
Q And you saw that person who again threw
something from the rooftop of the house?

xxxx
Q Where were you?

A Yes, sir.
PROS. PERALTA:

A I was at the back of the house that is being


cordoned by the police.

Q Did you recognize him?


Q When you saw something thrown out at the top of
the house, did you do something if any?

A Yes, sir.

Q While you were at the back of this house, do you


recall any unusual incident?

A I shouted to seek cover.

Q Who was that person?

A Yes, sir.

xxxx

A Zaldy Valerio again.

Q Can you tell the Honorable Court what was that


incident?

Q So, what else did you do if any after you shouted,


"take cover?"

xxxx

A Yes, sir. A person went out at the top of the house


and threw something.

A I took hold of a flashlight after five minutes and


focused the beam of the flashlight on the place
where something was thrown.

Q And did you see the person who threw something


out of this house?
A Yes, sir.

Q What did you see if any?

Q Where were you when you saw this Zaldy Valerio


thr[o]w something out of the house?
A I was on the road in front of the house.
Q Where was Zaldy Valerio when you saw him
thr[o]w something out of the house?

A He was on top of the house.


xxxx
Q Later on, were you able to know what was that
something thrown out?
A Yes, sir.

sighting the seized contraband be identified and known to be


so. The law merely requires that the law enforcer observes
that the seized item may be evidence of a crime, contraband,
or otherwise subject to seizure.
Hence, as correctly declared by the CA, the two receivers
were admissible as evidence. The liability for their
possession, however, should fall only on Valerio and not on
petitioner.1avvphil

offender has the intent to perpetrate the act prohibited by the


special law. Intent to commit the crime and intent to
perpetrate the act must be distinguished. A person may not
have consciously intended to commit a crime; but he did
intend to commit an act, and that act is, by the very nature of
things, the crime itself. In the first (intent to commit the
crime), there must be criminal intent; in the second (intent to
perpetrate the act) it is enough that the prohibited act is done
freely and consciously.

Q What was that?


The foregoing disquisition notwithstanding, we find that
petitioner is not liable for illegal possession of part of a
firearm.

In the present case, a distinction should be made between


criminal intent and intent to possess. While mere possession,
without criminal intent, is sufficient to convict a person for
xxxx
illegal possession of a firearm, it must still be shown that
In dissecting how and when liability for illegal possession of there was animus possidendi or an intent to possess on the
Q And what did he tell you?
firearms attaches, the following disquisitions in People v. De part of the accused. Such intent to possess is, however,
Gracia22 are instructive:
without regard to any other criminal or felonious intent
A It [was] on the wall of another house and it [could]
which the accused may have harbored in possessing the
be seen right away.
The rule is that ownership is not an essential element of
firearm. Criminal intent here refers to the intention of the
illegal possession of firearms and ammunition. What the law accused to commit an offense with the use of an unlicensed
xxxx
requires is merely possession which includes not only actual firearm. This is not important in convicting a person under
physical possession but also constructive possession or the
Presidential Decree No. 1866. Hence, in order that one may
Q What did you do if any?
subjection of the thing to one's control and management.
be found guilty of a violation of the decree, it is sufficient
This has to be so if the manifest intent of the law is to be
that the accused had no authority or license to possess a
A We waited for the owner of the house to wake up. effective. The same evils, the same perils to public security, firearm, and that he intended to possess the same, even if
which the law penalizes exist whether the unlicensed holder such possession was made in good faith and without criminal
xxxx
of a prohibited weapon be its owner or a borrower. To
intent.
accomplish the object of this law the proprietary concept of
Q Who opened the fence for you?
the possession can have no bearing whatsoever.
Concomitantly, a temporary, incidental, casual, or harmless
possession or control of a firearm cannot be considered a
A It was a lady who is the owner of the house.
But is the mere fact of physical or constructive possession
violation of a statute prohibiting the possession of this kind
sufficient to convict a person for unlawful possession of
of weapon, such as Presidential Decree No. 1866. Thus,
Q When you entered the premises of the house of the firearms or must there be an intent to possess to constitute a
although there is physical or constructive possession, for as
lady, what did you find?
violation of the law? This query assumes significance since long as the animus possidendi is absent, there is no offense
the offense of illegal possession of firearms is a malum
committed.23
A We saw the lower receiver of this .45 cal. (sic) 21
prohibitum punished by a special law, in which case good
faith and absence of criminal intent are not valid defenses.
Certainly, illegal possession of firearms, or, in this case, part
The ensuing recovery of the receivers may have been
of a firearm, is committed when the holder thereof:
deliberate; nonetheless, their initial discovery was
When the crime is punished by a special law, as a rule, intent
indubitably inadvertent. It is not crucial that at initial
to commit the crime is not necessary. It is sufficient that the
(1) possesses a firearm or a part thereof
A Another lower receiver of a cal. 45.

(2) lacks the authority or license to possess the


firearm.24
We find that petitioner was neither in physical nor
constructive possession of the subject receivers. The
testimony of SPO2 Nava clearly bared that he only saw
Valerio on top of the house when the receivers were thrown.
None of the witnesses saw petitioner holding the receivers,
before or during their disposal.
At the very least, petitioners possession of the receivers was
merely incidental because Valerio, the one in actual physical
possession, was seen at the rooftop of petitioners house.
Absent any evidence pointing to petitioners participation,
knowledge or consent in Valerios actions, she cannot be
held liable for illegal possession of the receivers.
Petitioners apparent liability for illegal possession of part of
a firearm can only proceed from the assumption that one of
the thrown receivers matches the gun seen tucked in the
waistband of her shorts earlier that night. Unfortunately, the
prosecution failed to convert such assumption into concrete
evidence.

that the receivers later on discarded were components of the


two (2) pistols seen with Valerio.

radio announcer Vega, who witnessed the recovery of the


receivers.28

These findings also debunk the allegation in the information


that petitioner conspired with Valerio in committing illegal
possession of part of a firearm. There is no evidence
indubitably proving that petitioner participated in the
decision to commit the criminal act committed by Valerio.

Anent the lack of authority, SPO1 Tan testified that, upon


verification, it was ascertained that Valerio is not a duly
licensed/registered firearm holder of any type, kind, or
caliber of firearms.29 To substantiate his statement, he
submitted a certification30 to that effect and identified the
same in court.31 The testimony of SPO1 Tan, or the
certification, would suffice to prove beyond reasonable
doubt the second element.32

Hence, this Court is constrained to acquit petitioner on the


ground of reasonable doubt. The constitutional presumption
of innocence in her favor was not adequately overcome by
the evidence adduced by the prosecution.
The CA correctly convicted Valerio with illegal possession
of part of a firearm.
In illegal possession of a firearm, two (2) things must be
shown to exist: (a) the existence of the subject firearm; and
(b) the fact that the accused who possessed the same does
not have the corresponding license for it.26

WHEREFORE, premises considered, the February 10, 2009


Decision of the Court of Appeals is hereby REVERSED with
respect to petitioner Elenita Fajardo y Castro, who is hereby
ACQUITTED on the ground that her guilt was not proved
beyond reasonable doubt.
SO ORDERED.

PP. vs. Valdez


Facts: After a tip from a source hat there is a plantation of marijuana
owned by the appellant at Sitio Bulan VillaverdeNV, the PNP Nueva
Vizcaya immediately formed anoperation to verify the report with an
Mere speculations and probabilities cannot substitute for
instruction to uprootsaid marijuana plants and arrest the cultivator of the
(a) the existence of the part of the firearm; and
proof required to establish the guilt of an accused beyond
same.Upon reaching their destination, the police sayseven five-foot
reasonable doubt. The rule is the same whether the offenses
height marijuana flowering tops situatedapproximately 25 meters from
(b) the accused who possessed the same does not
are punishable under the Revised Penal Code, which are
the accused. Valdez admittedthat the plants are his, thus, the police
have the license for the firearm to which the seized
uprooted the plantsand subject it to forensic analysis which was later
mala in se, or in crimes, which are malum prohibitum by
25
foundpositive to be a marijuana. The accused was arrestedThe lower
part/component
corresponds.
virtue of special law. The quantum of proof required by law
court was convicted and sentenced tosuffer the capital punishment of
was not adequately met in this case in so far as petitioner is
death. Hence this automaticappeal.The petitioner through counsel
In the instant case, the prosecution proved beyond
concerned.
contended that thesearch is unlawful since the officers has ample time to
reasonable doubt the
securea search warrant, thereby the evidences cannot be admittedas
The gun allegedly seen tucked in petitioners waistband was
evidence against him being a fruit of a proverbial poisonedtree.The
elements of the crime. The subject receivers - one with the
not identified with sufficient particularity; as such, it is
respondent, through the solicitor General aversthat the search is lawful
markings "United States Property" and the other bearing
since it is one of the considered asplain view doctrine/
impossible to match the same with any of the seized
search.Issue:Whether the search is in accordance with the plainview
Serial No. 763025 - were duly presented to the court as
receivers. Moreover, SPO1 Tan categorically stated that he
doctrine.Held:The search is unlawful and cannot be classified as aplain
Exhibits E and E-1, respectively. They were also identified
saw Valerio holding two guns when he and the rest of the
by SPO2 Nava as the firearm parts he retrieved af ter Valerio view search. As ingrained in jurisprudence, if the arrestingofficers has
PISOG arrived in petitioners house. It is not unlikely then
ample time to secure a arrest warrant, speciallywhen they all have the
discarded them.27 His testimony was corroborated by DYKR
By analogy then, a successful conviction for illegal
possession of part of a firearm must yield these requisites:

information needed to identify theplace and the person sought to be


searched and/or arrested,search warrant is needed. However, statutory
andconstitutional provisions also allows instances wherewarrants are
dispensable. One of which is search of plainview or the plain view
doctrine.For the doctrine to apply, the following requisitesshould be
present.(a) Aprior valid intrusion based on thevalid warrantless arrest in
which thepolice are legally present in the pursuitof their official duties;(b)
The evidence was inadvertentlydiscovered by the police who have
theright to be where they are; and(c) The evidence must be
immediatelyapparent; and(d) Plain view justified mere seizure
of evidence without further search.Thus, the plain view doctrine applies
only where thepolice officer is not searching for evidence against
theaccused, but inadvertently comes across an incriminatingobject.In the
case at bar, their discovery of the plant is notinadvertent. It is clear from
the records and the orders of theofficer that they have to uproot the
cannabis plants whenseen and that the officers testified that they first
located themarijuana before the arrest of Valdez.Thus, the search is illegal
and that the confiscatedplants cannot be admitted as evidence being a
fruit of apoisonous tree.
Note:
The guarantee refers to "the right of personalsecurity" of the individual.
As appellant correctly points out,what is sought to be protected against
the State's unlawfulintrusion are persons, not places. To conclude
otherwisewould not only mean swimming against the stream, it
wouldalso lead to the absurd logic that for a person to be immuneagainst
unreasonable searches and seizures, he must be inhis home or office,
within a fenced yard or a private place.The Bill of Rights belongs as
much to the person in the streetas to the individual in the sanctuary of his
bedroom.
Harris v. United States, 390 U.S. 234 (1968)
Harris v. United States
No. 92
Argued January 18, 1968
Decided March 5, 1968
390 U.S. 234
Syllabus

Pursuant to a departmental regulation, a police officer


searched a impounded car held as evidence of a robbery. The
search completed, the officer opened the car door for the
purpose of rolling up a window and thus protecting the car
and its contents. On opening the door, the officer saw,
exposed to plain view, the automobile registration card
belonging to the victim of the robbery. This card was used as
evidence in petitioner's trial. Petitioner's conviction was
affirmed by the Court of Appeals over his contention that the
card had been illegally seized following a warrantless search.

because of absence of search warrant and evenprobable cause for the


issuance of the same.In the testimony of the witness by the
prosecution,he has no personal knowledge making its
testimonieshearsay and weak to establish the existence of a
probablecause.Moreover, the search conducted is not one beingqualified
as a valid search without warrant(1) Search incidental to an arrest.(2)
Customs search.(3) Consented search(4) Search of a moving vehicle(5)
Stop frisk(6) Plain view search.(Apply theMendoza doctrineincidental discovery of incriminating evidence to qualify as a plain view
search)Moreover, the rights granted under the Bill of Rights ispersonal
and that cannot be waived by anyone else rather than the person whose
rights is invaded or one whoexpressly to do so in his behalf.In the case at
bar, the prosecution did notestablished that Luz Tancianco was
Held: The card was subject to seizure and introducible in
authorized by theappellant to allow the officers to enter the appellants
evidence, since it was not discovered by means of a search in houseand seized the effects of the appellant upon seeing it inpresumption
the technical sense, but was plainly visible to the officer who that it is connected with the crime of subversion.Thereby violating the
appellants rights of privacy and securityof house and effects.Thus, the
had a right to be in a position of viewing it.
prosecutions evidences are weak andthat the search being illegal, the
articles sought areinadmissible as evidences being a fruit of poisonous
125 U.S.App.D.C. 231, 370 F.2d 477, affirmed.
tree.
People vs. Velasco
Facts: A close surveillance by the Narcotics unit of theWPD confirmed
that the Accused, Yolanda Velasco akaShabu Queen is indeed engage in
peedling shabu, aregulated drug, along Quirada St. in Manila. After the
confirmation, the WPD organized a buy-bust operation involving a
poseur and marked money. Theoperation was successful and the
appellant was caught inflagranti delicto, was arrested and brought to
precinct for aninvestigation. The confiscated five decks of shabu
werefound in the accuseds pockets.Hence this petition, arguing that the
articles soughtcannot be used as an evidence since the search was
illegalbeing obtained through a warrantless arrest. She also allegethat she
People vs. Damaso
was framed-up by the officers.Issue:Was there an illegal warrantless
Facts: After a sequence of arrest among members of theNPA, the
arrest?Held:No. it being one of the exceptions provided bystatutory
Philippine Constabulary officers of LingayenPangasinan, the
provision.The RRC provides that an officer, in his presence,can arrest
apprehended NPA(Luz Tancianco)members pointed out to the PC the
even without a search or arrest warrant when theperson arrested actually
house of the AppellantDamaso where the same is leasing.Being pointed committed, committing or attemptingto commit a crime.In the case at
out that the appellant is the lessee of the house, the police went inside and bar, the accused was caught inflagranti delicto (in the act of committing
eventually saw M-14rifles, radio sets, subversive materials and
the crime), thus it justified the warrantless arrest conducted by the officer,
pamphlets, maps,computer machines, bullets and ammunitions. These
thus,debunk the petitioner contention of frame up andinadmissibility of
articlesare confiscated to use as evidence against the accused inthe crime evidence against her.No evidence presented by the petitioner that
of Subversion.He was then convicted by the RTC Dagupan in thecrime thearresting officers are engage in unscrupulous discharge of their duty to
of subversion, hence this appeal.The appellant contends that the seizure back up her contention of a frame up.
in his houseis illegal for absence of a search warrant.Issue:Was the search
People vs. Leangsiri
unlawful?Held:Yes. The search n the house of the appellant isillegal
Official Supreme Court caselaw is only found in the print
version of the United States Reports. Justia caselaw is
provided for general informational purposes only, and may
not reflect current legal developments, verdicts or
settlements. We make no warranties or guarantees about the
accuracy, completeness, or adequacy of the information
contained on this site or information linked to from this site.
Please check official sources.

Facts:Suchinda Leangsiri was arrested in the NAIA in theact of bringing


into the country approximately more than * kiloof heroin. In his arrest, he
informed the arresting officers thatthe heroin is meant to deliver to three
other people in LasPalmas Hotel in Manila.Immediately, the NARCOM
formed a group for afollow up operation in the said Hotel. In the
accusedscooperation, he was allowed to check in to Room 5
0
4, wherethe others will meet him to give the drugs. Around
10
pm, Amidu two other co-appellantsentered Room 5
0
4 and Leagsiri gave them the drugs, beforethe appellants leave the room,
the NARCOM officers bargedin and arrested the appellants. Amidu,
told the officers that he is staying in Rm 4
1
3and that the two others are in royal Palm Hotel. The officersthen went
to the room of Amidu, searching for evidence andsubsequently
confiscated a telephone address bearing thename of Leangsiri, other
possessions and documents of Amidu were also confiscated.In the case
of the two other, the police confiscated asuit case and masking tape and
empty transparent bag,allegedly will be use in transporting the drugs.The
appellants were charged and was convicted inconspiring to transport
heroin violative of RA 6425. Hencethis petition, alleging that the search
is illegal beingconducted not in the direct premises of the
arrest.Issue:Whether the articles sought in the other room andhotel,
outside the direct premises of the arrest admissible asevidence?Held:No.
those article are inadmissible as evidence as itwas obtained not in plan
view nor within the direct premisesof the arrest.The plain view doctrin
applies to OBJECTS OF THEPLAIN VIEW OF AN OFFICER WHO
HAS THE RIGHT TOBE IN THE POSITION TO HAVE THAT
VIEW ARESUBJECT TO SEIZURE AND MAYBE PRESENTED
ASEVIDENCE.Thus, what can be admitted are evidences seizedwithin
the direct premise where the accused has animmediate control which
should only be Rm. 5
0
9.In the case at bar, appellants were arrested inRoom 5
0
4 of the Las Palmas Hotel. The piece of paper bearing Leangsiri's name
was obtained through awarrantless search of Room 4
1
3 of the same hotel, andfound tucked within the pages of appellant
Amidu's telephoneand address book. Clearly, the warrantless search is
illegaland the piece of paper bearing Leangsiri's name cannot beadmitted
as evidence against appellants. The inadmissibilityof this evidence will
not, however, exculpate appellants. Itsexclusion does not destroy the

prosecution's case againstappellants. The remaining evidence still


established their guiltbeyond reasonable doubt.
Note:Plain view" doctrine
is usually applied where apolice officer is not, searching for evidence
against theaccused, but nonetheless inadvertently comes across
anincriminating object.Furthermore, the U.S. Supreme Court stated
thefollowing limitations on the application of the doctrine.
W
hat the "plain view" cases have in co
mm
on i
sthat the police officer in each of them had a prior justificationfor an
intrusion in the course of which he came inadvertentlyacross a piece of
evidence incriminating the accused. Thedoctrine serves to supplement
the prior justification whether it be a warrant for another object, hot
pursuit, search incidentto lawful arrest, or some other legitimate reason
for beingpresent unconnected with a search directed against theaccused
and permits the warrantless seizure. Of course, theextension of the
original justification is legitimate only whereit is immediately apparent to
the police that they haveevidence before them; the "plain view" doctrine
may not beused to extend a general exploratory search from one objectto
another until something incriminating at last emerges.
W
hen there is waiver of right or gives hisconsent
A Valid waiver of right in Sec 2 art III, elements(
1
) The right to be waived is existing(2) The person waiving it had
knowledge, actual or constructive(3) He or she has actual intention to
relinquish the right.

articlessought (bloodstained bolo, shirt and short pants) cannot


beadmitted as evidence against the accused since it wasseized without a
valid search and seizure warrant.Issue:Whether the articles are validly
seized even withouta valid search warrant and therefore admitted as
evidence inthe case at bar.Whether the articles when with consent given
to theofficers can be admitted as an evidence?Held:The articles are
unlawfully searched and seized. A search incidental to a valid arrest is
one of thestatutory exceptions to the constitutional mandate that nosearch
and seizure shall be effected without a valid warrant.In this instance, the
arrest should be lawful before searchand seizure by the arresting officer
would be conducted. Awarrantless arrest may be effected by the arresting
officer when in his presence the person arrested is have
committed,committing or attempting to commit the crime. It cannot
bereversed; otherwise, it would unlawful and unconstitutionaland the
seized article would be inadmissible evidence.In the case at bar, Accusedappellants were notbeing arrested at the time that the subject articles
wereallegedly taken from them but were just being questioned bythe
police officers conducting the investigation about thedeath of Patrocinia
Caburao. The investigating officers hadno personal knowledge of facts
indicating that the accusedhad committed the crime. Being in no position
to effect awarrantless arrest, the police officers were thus likewisebarred
from effecting a warrantless search and seizure. An illegal search cannot
be undertaken and then anarrest effected on the strength of the evidence
yielded by thatsearch.The Court finds it less than credible the stance
of the prosecution that the polo shirt and short pants have
beenvoluntarily given. An alleged consent to a warrantless searchand
seizure cannot be based merely on the presumption of regularity in the
performance of duty. This presumption, byitself, cannot prevail against
the constitutionally protectedrights of an individual, and zeal in the
pursuit of criminalscannot ennoble the use of arbitrary methods that
theConstitution itself abhors.Thus, the bloodstained polo, bolo and shorts
areinadmissible as evidence.
People vs. Baula
Veroy vs. Layaguwe
Facts: After the gruesome killing of Patronicia Caburao inthe
Facts:The petitioners, Leopoldo Veroy has a residentialhouse in K-8th
municipality of Sual, Pangasinan, the investigating policewent to the
St. Quezon City where it is under the care of two houseboys and a
residence of the accused-appellant, Baula et al.In the process of
certain Soquilon. The bedrooms in thishouse are constantly locked and
questioning the appellants, thepolice saw bloodstained bolo, short pants,
access to the kitchen is theonly key to the caretakers.Sometime1990
polo shirts andwas subsequently confiscated without search warrant
, a directive issue was ordered toCapt. Obrero to search the house of the
anddirected to the NBI for forensic exams. The exam resultedthat the
petitioner in thereport that their residence is a hideout and recruiting site
bloods found in the confiscated articles bears thesame blood type O as
of rebel soldiers.They were able to enter the yard with the help of
that of the victim.
thecaretakers but did not enter the house since the owner wasnot present
and they did not have a search warrant.Petitioner Ma. Luisa was
Thus, the accused were arrested, charged and wasconvicted in the crime
contacted by telephone in her Quezon City residence by Capt. Obrero to
of murder by the RTC Lingayen andsentenced to suffer RP. Hence this
ask permission tosearch the house in Davao City as it was reportedly
appeal for review on thedecision of the lower court in the ground that the
beingused as a hideout and recruitment center of rebel soldiers.Petitioner

Ma. Luisa Veroy responded that she is flying toDavao City to witness the
search but relented if the searchwould not be conducted in the presence
of Major ErnestoMacasaet, an officer of the PC/INP, Davao City and a
longtime family friend of the Veroys.The authority given by Ma. Luisa
Veroy was relayedby Capt. Obrero to Major Macasaet who answered
that Ma.Luisa Veroy has called him twice by telephone on the
matter and that the permission was given on the condition that thesearch
be conducted in his presence.The officers in the following day with Maj.
Macasaet,by virtue of the authority granted by the petitioner, gained
anentrance to the house and first search the masters andchildrens
bedroom, where they saw and confiscated .45 calhandgun with live
bullets, printed materials of RAM-SFP,traveling bag with assorted
clothes, telescope, map, a book,medicines and religious pamphlets.The
petitioners were then charged with a criminalcomplaint in the crime of
illegal possession of Firearms and Ammunitions in furtherance with
Rebellion, and that theprosecution recommended no bail. Due to stress
and anxiety,the petitioners were confined in a hospital and filed
Motionsfor Bail and Hospital Confinement under the respondent judge
Layague, which was denied.Then an order was issued by the respondent
judgeto transfer the petitioners from St Lukes to Camp Crame.Hence this
petition. The petitioners question the admissibilityin evidence of the
articles seized in violation of their constitutional right against
unreasonable searches andseizure.Petitioners aver that while they
concede that Capt.Obrero had permission from Ma. Luisa Veroy to
break openthe door of their residence, it was merely for the purpose
of ascertaining thereat the presence of the alleged "rebel"soldiers. The
permission did not include any authority toconduct a room-to-room
search once inside the house. Theitems taken were, therefore, products of
an illegal search,violative of their constitutional rights. As such, they
areinadmissible in evidence against them.Issue:Does the articles seized
unlawful and cannot beadmitted as evidence? or Does the consent given
by the petitioners make thesearch and seizure lawful?Held:The articles
are illegally seized thereforeinadmissible as evidence.The Constitution
guarantees the right of the peopleto be secure in their persons, houses,
papers and effectsagainst unreasonable searches and seizures (Article
III,Section 2 of the1987 Constitution).However, the rule that searches
and seizures mustbe supported by a valid warrant is not an absolute
one. Among the recognized exceptions thereto are:
(1)a searchincidental to an arrest;
(2)a search of a moving vehicle; and
(3)seizure of evidence in plain viewNone of these exceptions pertains to
the case atbar. The reason for searching the house of herein petitioner is
that it was reportedly being used as a hideout andrecruitment center for
rebel soldiers. While Capt. Obrero wasable to enter the compound, he
did not enter the housebecause he did not have a search warrant and the

ownerswere not present. This shows that he himself recognized theneed


for a search warrant, hence, he did not persist inentering the house but
rather contacted the Veroys to seekpermission to enter the
same.Permission was indeed granted by Ma. Luisa Veroyto enter the
house but only to ascertain the presence of rebelsoldiers. Under the
circumstances it is undeniable that the
police officers had ample time to procure a search warrantbut did not.In a
number of cases decided by this ,Court,(Guazon v. De Villa, supra.;
People v. Aminnudin),
warrantless searches were declared illegal because theofficials
conducting the search had every opportunity tosecure a search warrant
. The objects seized, beingproducts of illegal searches, were inadmissible
in evidence inthe criminal actions subsequently instituted against
theaccused-appellantsUndeniably, the offense of illegal possession
of firearms is
malum prohibitum
but it does not follow that thesubject thereof is necessarily illegal per
se.Motive is immaterial in mala prohibita
but thesubjects of this kind of offense may not be summarily
seizedsimply because they are prohibited. A search warrant is
stillnecessary. Hence, the rule having been violated and noexception
being applicable, the articles seized wereconfiscated illegally and are
therefore protected by theexclusionary principle. They cannot be used as
evidenceagainst the petitioners in the criminal action against them
for illegal possession of firearms.
De Garcia vs. Locsin
Facts:Mariano Almeda, an agent of Anti-usury Boardapplied for a search
warrant before the respondent judgeagainst the petitioner, believing that
there is a probablecause that the petitioner, Leona De Garcia, keeps
andconseals in her house and store at Victoria Tarlac certainbooks, lists,
chits, receipts and documents relating to her activities as usurer, contrary
to law.The search warrant was granted and with officers of the law,
Almeda et al search the person and house/store of the petitioner to seized
evidence related to usury.The search proceeded without the presence of
thepetitioner, and thus, packages of records were confiscated,turned over
to the fiscal by the Anti-Usury Board and waslater filed six separate
complaint against the petitioner for violation of the Anti-usury law.Thus,
a motion was filed by the petitioner demanding the respondent Board to
return the articles seizedin the search in the ground of its invalidity. The
respondent judge denied such motion and contends that even though
thesearch warrant is invalid, the articles are still admissible asevidence
since there is a waiver of right on the part of thepetitioner.Issue:Was there
a waiver of right?If there is, would it make the search valid andtherefore

articles seized are admissible evidence?Held:No. There is no waiver of


right and that the articlesare inadmissible evidence.The constitutional
immunity against unreasonablesearches and seizures is a personal right
which may bewaived.The waiver may be either express or implied.In the
case at bar, no express waiver has beenmade. It is urged, however, that
there has been a waiver byimplication. It is well-settled that to constitute a
waiver of aconstitutional right, it must appear,(
1
) First, that the right exists;(2) Secondly, that the person involved
hadknowledge, either actual or constructive, of theexistence of such right;
and,(3) Lastly, that said person had an actual intentionto relinquish the
right.It is true that the petitioner did not object to thelegality of the search
when it was made. She could not haveobjected because she was sick and
was not present whenthe warrant was served upon Alfredo Salas.
Certainly, theconstitutional immunity from unreasonable searches
andseizures, being a personal one, cannot be waived by anyoneexcept
the person whose rights are invaded or one who isexpressly authorized to
do so in his or her behalf.Of course, the petitioner came to know later of
theseizure of some of her papers and documents. But this wasprecisely
the reason why she sent her attorneys to the officeof the Anti-Usury
Board to demand the return of thedocuments seized. In any event, the
failure on the part of thepetitioner and her bookkeeper to resist or object
to theexecution of the warrant does not constitute an impliedwaiver of
constitutional right. As the constitutional guaranty is not dependentupon
any affirmative act of the citizen, the courts do not placethe citizen in the
position of either contesting an officer'sauthority by force, or waiving his
constitutional rights; butinstead they hold that a peaceful submission to a
search or seizure is not a consent or an invitation thereto, but is merelya
demonstration of regard for the supremacy of the law.Moreover, the
invalidity of the search warrant notonly anchored in the mentioned
ground, but also, therespondent judge did not follow the
constitutionalrequirement in determining probable cause to issue
warrants,such that, it is the applicant that determines the probablecause in
the case at bar. In 35 constitution, it mandated the judge to personally
determine the existence of probablecause and no other.
Lopez and Velasco vs. Commissioner of Customs
Confusing case where it is adverse to later jurisprudence that a waiver of
right can only be waived by the person whoseright against unreasonable
search and seizure was invaded.
Facts: After the surveillance conducted by the respondentsNBI officers,
NBI and PC Davao officers went to the room(Rm 22
0
) rented by the petitioner, Tomas Velasco, to searchand seized articles
papers and documents including a .45 calpistol, that became evidence
that commodities confiscated inthe wharf (MV Jojo Lema) allegedly

smuggled fromIndonesia to the country (sacks of coffee beans and


copra).The search was without a search warrant, however,the officers
have successfully confiscated the articles byvirtue of the consent of the
petitioners wife (Teofila ibanez)who also is an occupant of the room
rented by the petitioner.The petitioner contends that the consent given
byIbanez cannot be regarded since she was not the legal wifeof the
petitioner Velasco, but a certain Corazon Velasco.The decision of CTAis
adverse by the petitioner,affirmed by the SC.Issue:Was the consent valid
to justify the warrantlesssearch and seizure?Held: As far as the decision
is concerned, yes, theconsent given by Ibanez is a valid so as to dispense
thenecessity of a search warrant.The court ruled that the mere fact that
Ibanez ispresent in a room rented by the petitioner, her consent
wouldlead to belief that her consent as an alleged wife of thepetitioner
and that it would be an act on behalf of thepetitioner.

SUSAN ESQUILLO Y ROMINES, Petitioner,


vs.
PEOPLE OF THE PHILIPPINES, Respondent.
DECISION
CARPIO MORALES, J.:

Via petition erroneously captioned as one for Certiorari,


Susan Esquillo y Romines (petitioner) challenges the
November 27, 2007 Decision1 of the Court of Appeals in
CA-G.R. CR No. 27894 which affirmed the July 28, 2003
Decision of Branch 116 of the Regional Trial Court (RTC) of
Pasay City in Criminal Case No. 02-2297 convicting Susan
Note:
Esquillo y Romines (petitioner) for violating Section 11,
Exact decision penned by J. LaurelThere was an attempt on the part of
Article II of Republic Act (R.A.) No. 9165 (the
petitioners tocounteract the force of the above recital by an affidavit of
oneCorazon Y. Velasco, who stated that she is the legal wife of petitioner Comprehensive Dangerous Drugs Act of 2002) possession
Tomas Velasco, and another by such petitioner himself reiterating such a of methamphetamine hydrochloride or shabu.
fact and that the person who waspresent at his hotel room was one
Teofila Ibaez, "amanicurist by occupation." Their effort appurtenant
The accusatory portion of the Information dated December
thereto isdoomed to failure. If such indeed were the case, then it ismuch
12, 2002 indicting petitioner reads:
more easily understandable why that person, TeofilaIbaez, who could
be aptly described as the wrong person atthe wrong place and at the
That on or about the 10th day of December, 2002 in Pasay
wrong time, would have signifiedher consent readily and
immediately.Under the circumstances, that was the most prudentcourse City, Metro Manila, Philippines and within the jurisdiction
of action. It would save her and even petitioner Velasco himself from
of this Honorable Court, the above-named accused, without
any gossip or innuendo. Nor could theofficers of the law be blamed if
authority of law, did then and there willfully, unlawfully and
they would act on theappearances. There was a person inside who from
feloniously have in her possession, custody and control
allindications was ready to accede to their request. Evencommon
2
courtesy alone would have precluded them frominquiring too closely as 0.1224 gram of Methylamphetamine Hydrochloride (shabu).
(underscoring supplied)
to why she was there. Under all thecircumstances, therefore, it can
readily be concluded thatthere was consent sufficient in law to dispense
with the needfor a search warrant. The petition cannot, therefore, prevail. At the trial, petitioner admitted the genuineness and due
execution of the documentary evidence of the prosecution,
particularly the Dangerous Drugs and Toxicology Reports
Stop and Frisk Doctrine
issued by National Bureau of Investigation (NBI) Forensic
Defined:
Chemist Antonino de Belen (de Belen),3 subject to her
Stop-and-frisk was defined as the vernacular designation of the right of a
defenses, to thus dispense with the testimony of de Belen.
police officer to stop a citizen onthe street, interrogate him, and pat him
for weapon
G.R. No. 182010

August 25, 2010

De Belen recorded the results of the laboratory examination


of the contents of the sachet in Dangerous Drugs Report No.
DD-02-613,4 viz:

xxxx
SPECIMEN:
White crystalline substance contained in a heat-sealed
transparent plastic sachet marked "SRE" and further placed
in bigger marked transparent plastic sachet.
xxxx
F I N D I N G S:
Net Weight of specimen = 0.1224 gram
Examinations conducted on the above-mentioned specimen
gave POSITIVE RESULTS for METHAMPHETAMINE
HYDROCHLORIDE, a dangerous drug. x x x
x x x x (emphasis and underscoring supplied)
With respect to the examination of the urine of petitioner, de
Belen recorded the results thereof in Toxicology Report No.
TDD-02-41285 reading:
xxxx
SPECIMEN:
Urine of one SUSAN ESQUILLO Y ROMINES. 37 y/o,
married, jobless, of no. 1159 Bo. Bayanihan, Maricaban,
Pasay City.
xxxx
F I N D I N G S:
Volume of urine = 60 mL.
pH of urine = 5.0

Appearance = yellow orange, turbid


Examinations conducted on the above-mentioned specimen
gave POSITIVE RESULTS for the presence of
METHAMPHETAMINE HYDROCHLORIDE, and its
metabolite AMPHETAMINE. x x x
x x x x (emphasis and underscoring supplied)
Based on its documentary evidence and the testimony of
PO1 Alvin Cruzin (PO1 Cruzin),6 a member of the Pasay
City Police Station Special Operations Group (SOG), the
prosecution established its version as follows:
On the basis of an informants tip, PO1 Cruzin, together with
PO2 Angel Aguas (PO2 Aguas), proceeded at around 4:00
p.m. on December 10, 2002 to Bayanihan St., Malibay,
Pasay City to conduct surveillance on the activities of an
alleged notorious snatcher operating in the area known only
as "Ryan."
As PO1 Cruzin alighted from the private vehicle that brought
him and PO2 Aguas to the target area, he glanced in the
direction of petitioner who was standing three meters away
and seen placing inside a yellow cigarette case what
appeared to be a small heat-sealed transparent plastic sachet
containing white substance. While PO1 Cruz was not sure
what the plastic sachet contained, he became suspicious
when petitioner started acting strangely as he began to
approach her. He then introduced himself as a police officer
to petitioner and inquired about the plastic sachet she was
placing inside her cigarette case. Instead of replying,
however, petitioner attempted to flee to her house nearby but
was timely restrained by PO1 Cruzin who then requested her
to take out the transparent plastic sachet from the cigarette
case.
After apprising petitioner of her constitutional rights, PO1
Cruzin confiscated the plastic sachet7 on which he marked
her initials "SRE." With the seized item, petitioner was

brought for investigation to a Pasay City Police Station


where P/Insp. Aquilino E. Almanza, Chief of the Drug
Enforcement Unit, prepared a memorandum8 dated
December 10, 2002 addressed to the Chief Forensic Chemist
of the NBI in Manila requesting for: 1) a laboratory
examination of the substance contained in the plastic sachet
to determine the presence of shabu, and 2) the conduct of a
drug test on the person of petitioner. PO1 Cruzin and PO2
Aguas soon executed a Joint Affidavit of Apprehension9
recounting the details of their intended surveillance and the
circumstances leading to petitioners arrest.

WHEREFORE, in light of the foregoing premises and


considerations, this Court hereby renders judgment finding
the accused Susan Esquillo y Romines GUILTY beyond
reasonable doubt of the crime of Violation of par. 3 of
Section 11, Article II of R. A. 9165, otherwise known as the
Comprehensive Dangerous Drugs Act of 2002, and absent
any modifying circumstance to either aggravate or mitigate
the criminal liability of the same accused, and furthermore,
applying the provisions of the Indeterminate Sentence Law,
the same accused is hereby sentenced to suffer the penalty of
imprisonment ranging from Eight (8) years and One (1) day,
as minimum, to Fourteen (14) years, Eight (8) months and
10
Repudiating the charges, petitioner gave the following tale: One (1) day, as maximum, and to pay a fine of P350,000.00,
Philippine Currency, plus costs.
At around 1:00 to 2:00 p.m. of the date in question, while
she was sick and resting at home, several policemen in
The 0.1224 gram of Methylamphetamine Hydrochloride or
civilian garb with guns tucked in their waists barged in and
"Shabu" involved in this case is declared forfeited in favor of
asked her whether she knew one named "Ryan" who they
the Government and ordered to be turned over to the
claimed was a notorious snatcher operating in the area, to
Philippine Drug Enforcement Agency (PDEA) for proper
which she replied in the negative. The police officers then
and appropriate disposition in accordance with the
forced her to go with them to the Pasay City Police Station- provisions of the law.14 (underscoring supplied)
SOG office where she was detained.
Before the Court of Appeals, appellant questioned as illegal
While she was under detention, the police officers were
her arrest without warrant to thus render any evidence
toying with a wallet which they claimed contained shabu and obtained on the occasion thereof inadmissible.
recovered from her.
In its challenged Decision affirming petitioners conviction,
In fine, petitioner claimed that the evidence against her was the appellate court, citing People v. Chua,15 held that the
"planted," stemming from an all too obvious attempt by the police officers had probable cause to search petitioner under
police officers to extort money from her and her family.
the "stop-and-frisk" concept, a recognized exception to the
general rule prohibiting warrantless searches. 16
Two other witnesses for the defense, petitioners daughter
Josan Lee11 and family friend Ma. Stella Tolentino,12
Brushing aside petitioners defense of frame-up, the
corroborated petitioners account. They went on to relate that appellate court noted that petitioner failed to adduce
the police officers never informed them of the reason why
evidence that the arresting officers were impelled by any evil
they were taking custody of petitioner.
motive to falsely charge her, and that she was even found
positive for substance abuse.1wphi1
13
By Decision of July 28, 2003, the trial court found
petitioner guilty of illegal possession of Methylamphetamine In her present petition, petitioner assails the appellate courts
Hydrochloride or shabu, disposing as follows:
application of the "stop-and-frisk" principle in light of PO1

Cruzins failure to justify his suspicion that a crime was


being committed, he having merely noticed her placing
something inside a cigarette case which could hardly be
deemed suspicious. To petitioner, such legal principle could
only be invoked if there were overt acts constituting unusual
conduct that would arouse the suspicion.17
Respondent, through the Office of the Solicitor General,
prays for the affirmance of the appealed decision but seeks a
modification of the penalty to conform to the pertinent
provisions of R.A. No. 9165.
Appellants conviction stands.
Petitioner did not question early on her warrantless arrest
before her arraignment. Neither did she take steps to quash
the Information on such ground. Verily, she raised the issue
of warrantless arrest as well as the inadmissibility of
evidence acquired on the occasion thereof for the first time
only on appeal before the appellate court.18 By such
omissions, she is deemed to have waived any objections on
the legality of her arrest.19
Be that as it may, the circumstances under which petitioner
was arrested indeed engender the belief that a search on her
was warranted. Recall that the police officers were on a
surveillance operation as part of their law enforcement
efforts. When PO1 Cruzin saw petitioner placing a plastic
sachet containing white crystalline substance into her
cigarette case, it was in his plain view. Given his training as
a law enforcement officer, it was instinctive on his part to be
drawn to curiosity and to approach her. That petitioner
reacted by attempting to flee after he introduced himself as a
police officer and inquired about the contents of the plastic
sachet all the more pricked his curiosity.

(1) consented searches; (2) as an incident to a lawful arrest;


(3) searches of vessels and aircraft for violation of
immigration, customs, and drug laws; (4) searches of
moving vehicles; (5) searches of automobiles at borders or
constructive borders; (6) where the prohibited articles are in
"plain view;" (7) searches of buildings and premises to
enforce fire, sanitary, and building regulations; and (8) "stop
and frisk" operations.20 (emphasis underscoring supplied)
In the instances where a warrant is not necessary to effect a
valid search or seizure, the determination of what constitutes
a reasonable or unreasonable search or seizure is purely a
judicial question, taking into account, among other things,
the uniqueness of the circumstances involved including the
purpose of the search or seizure, the presence or absence of
probable cause, the manner in which the search and seizure
was made, the place or thing searched, and the character of
the articles procured.21
Elucidating on what includes "stop-and-frisk" operation and
how it is to be carried out, the Court in People v. Chua22
held:
. . . the act of a police officer to stop a citizen on the street,
interrogate him, and pat him for weapon(s) or contraband.
The police officer should properly introduce himself and
make initial inquiries, approach and restrain a person who
manifests unusual and suspicious conduct, in order to check
the latters outer clothing for possibly concealed weapons.
The apprehending police officer must have a genuine reason,
in accordance with the police officers experience and the
surrounding conditions, to warrant the belief that the person
to be held has weapons (or contraband) concealed about him.
It should therefore be emphasized that a search and seizure
should precede the arrest for this principle to apply.

That a search may be conducted by law enforcers only on the This principle of "stop-and-frisk" search was invoked by the
strength of a valid search warrant is settled. The same,
Court in Manalili v. Court of Appeals. In said case, the
however, admits of exceptions, viz:
policemen chanced upon the accused who had reddish eyes,
walking in a swaying manner, and who appeared to be high

on drugs. Thus, we upheld the validity of the search as akin


to a "stop-and-frisk." In People v. Solayao, we also found
justifiable reason to "stop-and-frisk" the accused after
considering the following circumstances: the drunken
actuations of the accused and his companions, the fact that
his companions fled when they saw the policemen, and the
fact that the peace officers were precisely on an intelligence
mission to verify reports that armed persons w[h]ere
roaming the vicinity. (emphasis and underscoring supplied;
citations omitted)1wphi1
What is, therefore, essential is that a genuine reason must
exist, in light of the police officers experience and
surrounding conditions, to warrant the belief that the person
who manifests unusual suspicious conduct has weapons or
contraband concealed about him. Such a "stop-and-frisk"
practice serves a dual purpose: (1) the general interest of
effective crime prevention and detection, which underlies the
recognition that a police officer may, under appropriate
circumstances and in an appropriate manner, approach a
person for purposes of investigating possible criminal
behavior even without probable cause; and (2) the more
pressing interest of safety and self-preservation which permit
the police officer to take steps to assure himself that the
person with whom he deals is not armed with a deadly
weapon that could unexpectedly and fatally be used against
the police officer.23
From these standards, the Court finds that the questioned act
of the police officers constituted a valid "stop-and-frisk"
operation. The search/seizure of the suspected shabu initially
noticed in petitioners possession - later voluntarily
exhibited24 to the police operative - was undertaken after she
was interrogated on what she placed inside a cigarette case,
and after PO1 Cruzin introduced himself to petitioner as a
police officer. And, at the time of her arrest, petitioner was
exhibiting suspicious behavior and in fact attempted to flee
after the police officer had identified himself.

It bears recalling that petitioner admitted the genuineness


and due execution of the Dangerous Drugs and Toxicology
Reports, subject, however, to whatever available defenses
she would raise. While such admissions do not necessarily
control in determining the validity of a warrantless search or
seizure, they nevertheless provide a reasonable gauge by
which petitioners credibility as a witness can be measured,
or her defense tested.
It has not escaped the Courts attention that petitioner seeks
exculpation by adopting two completely inconsistent or
incompatible lines of defense. On one hand, she argues that
the "stop-and-frisk" search upon her person and personal
effects was unjustified as it constituted a warrantless search
in violation of the Constitution. In the same breadth,
however, she denies culpability by holding fast to her
version that she was at home resting on the date in question
and had been forcibly dragged out of the house by the police
operatives and brought to the police station, for no apparent
reason than to try and extort money from her. That her two
witnesses a daughter and a friend who were allegedly
present at the time of her arrest did not do anything to report
it despite their claim that they were not informed why she
was being arrested, should dent the credibility of their
testimony.

court with respect to the credibility of witnesses prevail over Section 1 of the Indeterminate Sentence Law provides that
that of petitioner.25
when the offense is punished by a law other than the Revised
Penal Code, "the court shall sentence the accused to an
A word on the penalty.
indeterminate sentence, the maximum term of which shall
not exceed the maximum fixed by law and the minimum
While the appellate court affirmed the trial courts decision, shall not be less than the minimum term prescribed by the
it overlooked the error in the penalty imposed by the trial
same."
court. The trial court, applying the provisions of the
Indeterminate Sentence Law, sentenced petitioner to "suffer The prayer of the Office of the Solicitor General for a
the penalty of imprisonment ranging from Eight (8) years
modification of the penalty is thus in order.
and One (1) day, as minimum, to Fourteen (14) years, Eight
(8) months and One (1) day, as maximum."
The Court, therefore, imposes on petitioner the penalty of
imprisonment of twelve (12) years and one (1) day, as
Article II, Section 11 of R.A. No. 9165 provides, however:
minimum, to fourteen (14) years, as maximum.
Section 11. Possession of Dangerous Drugs.

Otherwise, if the quantity involved is less than the foregoing


quantities, the penalties shall be graduated as follows:

WHEREFORE, the assailed decision of the Court of Appeals


is AFFIRMED, with the MODIFICATION that the penalty
of imprisonment shall be twelve (12) years and one (1) day,
as minimum, to fourteen (14) years, as maximum. In all
other respects, the decision of the RTC in Criminal Case No.
02-2297 is AFFIRMED.

xxxx

SO ORDERED.

xxxx

(3) Imprisonment of twelve (12) years and one (1) day to


twenty (20) years and a fine ranging from Three hundred
thousand pesos (P300,000) to Four hundred thousand
Courts have tended to look with disfavor on claims of
accused, such as those of petitioners, that they are victims of pesos (P400,000), if the quantities of dangerous drugs are
less than five (5) grams of opium, morphine, heroin,
a frame-up. The defense of frame-up, like alibi, has been
cocaine or cocaine hydrochloride, marijuana resin or
held as a shop-worn defense of the accused in drug-related
cases, the allegation being easily concocted or contrived. For marijuana resin oil, metamphetamine hydrochloride or
"shabu" or other dangerous drugs such as, but not limited to
this claim to prosper, the defense must adduce clear and
MDMA or "ecstacy," PMA, TMA, LSD, GHB and those
convincing evidence to overcome the presumption of
similarly designed or newly introduced drugs and their
regularity of official acts of government officials. This it
derivatives, without having any therapeutic value or if the
failed to do.
quantity possesses is far behind therapeutic requirements; or
less than three hundred (300) grams of marijuana. (emphasis
Absent any proof of motive to falsely accuse petitioner of
and underscoring supplied)
such a grave offense, the presumption of regularity in the
performance of official duty and the findings of the trial

People vs. Posadas


Facts:On October 16,1986 at about10:00o'clock in themorning Pat.
Ursicio Ungab and Pat. Umbra Umpar, bothmembers of the Integrated
National Police (INP) of theDavao Metrodiscom assigned with the
Intelligence TaskForce, were conducting a surveillance along
MagallanesStreet, Davao City. While they were within the premises
of the Rizal Memorial Colleges they spotted petitioner carryinga "buri"
bag and they noticed him to be acting suspiciously.They approached the
petitioner and identifiedthemselves as members of the INP. Petitioner
attempted toflee but his attempt to get away was thwarted by the
twonotwithstanding his resistance.They then checked the "buri" bag of
the petitioner where they found one (1) caliber .38Smith &
Wessonrevolver with Serial No. 770196 two (2) rounds of
liveammunition for a .38caliber gun, a smoke (tear gas) grenadea
and two (2) live ammunitions for a .22 caliber gun.They brought the
petitioner to the police station for further investigation. In the course of

the same, the petitioner was asked to show the necessary license or
authority topossess firearms and ammunitions found in his possessionbut
he failed to do so.He was prosecuted for illegal possession of firearmsand
ammunitions in the Regional Trial Court of Davao Citywherein after a
plea of not guilty and trial on the merits adecision was rendered on
October 8,1987 finding petitioner guilty of the offense. The CA
affirmed the decision in toto.Thus, this appeal was filed by the appellant
in theground that the search and seizure is unlawful and thearticles sought
are inadmissible evidence against him.Issue:Was the warrantless search
valid?Held:Yes. The warrantless search is valid.Under the statutory
provision (RRC) warrantlessarrest t may be effected by a peace officer or
private person,among others, when in his presence the person to
bearrested has committed, is actually committing, or isattempting to
commit an offense; or when an offense has infact just been committed,
and he has personal knowledge of the facts indicating that the person
arrested has committed it.The Solicitor General argues that when the
twopolicemen approached the petitioner, he was actuallycommitting or
had just committed the offense of illegalpossession of firearms and
ammunitions in the presence of the police officers and consequently the
search and seizureof the contraband was incidental to the lawful arrest
inaccordance with Section12, Rule126 of the1985 Rules onCriminal
ProcedureHowever, in the case at bar, the record does notshow that when
the officers arrested the petitioner whoattempted to flee, there is no
probable cause nor a personalknowledge by the officers that the appellant
is actuallycommitted or committing a crime, the apprehension was
onlydue to the fact of suspicion by the officers that he is hidingsomething
in the bag. Thus, it does justified a warrantlessarrest.However, this does
not make the arrest invalid asthe case at bar falls with the exclusionary
rule of stop andfrisk doctrine.Jurisprudence has already dictate that
warrantlessarrest, search and seizure by checkpoints of military andpolice
is justified by the stop and frisk method.Thus, as between a warrantless
search and seizureconducted at military or police checkpoints and the
searchthereat in the case at bar, there is no question that, indeed,the latter
is more reasonable considering that unlike in theformer, it was effected
on the basis of a probable cause.The probable cause is that when the
petitioner acted suspiciously and attempted to flee with the buri bagthere
was a probable cause that he was concealingsomething illegal in the bag
and it was the right and duty of the police officers to inspect the same.It
would be useless and too late for the officers if they should first procure a
warrant before searching the bagof a suspicious person.
Manalili vs. People
Facts:Policemen from the Anti-Narcotics Unit of theCaloocan City
Police Station were conducting a surveillancealong A. Mabini street,
Caloocan City, in front of theCaloocan City Cemetery.The surveillance
was being made because of information that drug addicts were roaming

the area in frontof the Caloocan City Cemetery.Upon reaching the


Caloocan City Cemetery, thepolicemen alighted from their vehicle. They
then chancedupon a male person in front of the cemetery who
appearedhigh on drugs. The male person was observed to havereddish
eyes and to be walking in a swaying manner. Whenthis male person tried
to avoid the policemen, the latter approached him and introduced
themselves as policeofficers. The policemen then asked the male person
what hewas holding in his hands. The male person tried to resist.Pat.
Romeo Espiritu asked the male person if hecould see what said male
person had in his hands. The latter showed the wallet and allowed Pat.
Romeo Espiritu toexamine the same. Pat. Espiritu took the wallet and
examined it. He found suspected crushed marijuana residueinside. He
kept the wallet and its marijuana contents.The confiscated residue was
tested positive fir marijuana.However, the petitioner rebutted the
information innarrating a different facts in the case. The petitioner filed
anappeal questioning the admissibility of the evidencepresented by the
prosecution.Issue:Was the search qualified as a stop and friskmeasure
therefore justified as a valid warrantless arrest andsearch?Held:Yes. The
search is valid being conducted through astop and frisk method.Stopand-frisk was defined as the vernacular designation of the right of a
police officer to stop a citizen onthe street, interrogate him, and pat him
for weapon. Thismethod allows police officer to approach a person,
inappropriate circumstances and manner, for purposes of investigating
possible criminal behavior even though there isinsufficient probable
cause to make an actual arrest.In the case at hand, Patrolman Espiritu and
hiscompanions observed during their surveillance that appellanthad red
eyes and was wobbling like a drunk along theCaloocan City Cemetery,
which according to policeinformation was a popular hangout of drug
addicts.From his experience as a member of the Anti-Narcotics Unit of
the Caloocan City Police, such suspiciousbehavior was characteristic of
drug addicts who were "high."The policemen therefore had sufficient
reason tostop petitioner to investigate if he was actually high on
drugs.During such investigation, they found marijuana inpetitioner's
possession.
Issue on waiver of rights:
The Sc also ruled for the Solicitor General'scontention that petitioner
effectively waived the inadmissibilityof any evidence illegally obtained
when he filed to raise thisissue or to object thereto during the trial. A
valid waiver of a right, more particularly of theconstitutional right against
unreasonable search, requires theconcurrence of the following
requirements:(
1
) The right to be waived existed;(2) The person waiving it had
knowledge, actual or constructive, thereof; and(3) He or she had an
actual intention to relinquishthe right.Otherwise, the Courts will indulge

every reasonablepresumption against waiver of fundamental safeguards


andwill not deduce acquiescence from the failure to exercise
thiselementary right.In the present case, however, petitioner is deemedto
have waived such right for his failure to raise its violationbefore the trial
court. In petitions under Rule 45, asdistinguished from an ordinary
appeal of criminal caseswhere the whole case is opened for review, the
appeal isgenerally limited to the errors assigned by petitioner. Issuesnot
raised below cannot be pleaded for the first time onappeal.
Malacat vs.CA
Facts:The arresting officers conducted a foot patrol due tothe report that a
grioup of Muslim extremists was going toexplode a grenade somewhere
on the vicinity of PlazaMiranda.While on patrol, they chanced upon two
groups of Muslim-looking men, with each group, comprised of three
tofour men, posted at opposite sides of the corner of QuezonBoulevard
near the Mercury Drug Store. These men wereacting suspiciously with
"[t]heir eyes. . . moving very fast."Yu and his companions positioned
themselves atstrategic points and observed both groups for about
thirtyminutes. The police officers then approached one group of men,
who then fled in different directions. As the policemen gave chase, Yu
caught up withand apprehended petitioner. Upon searching petitioner,
Yufound a fragmentation grenade tucked inside petitioner's"front waist
line." Yu's companion, police officer RogelioMalibiran, apprehended
Abdul Casan from whom a .38 caliber revolver was recovered.
Petitioner and Casan werethen brought to Police Station No. 3 where Yu
placed an "X"mark at the bottom of the grenade and thereafter gave it
tohis commander.The lower court then charged the petitioners andwas
convicted holding that the search is valid being a searchincidental to a
lawful arrest.The petitioner filed an appeal in the ground that theCA erred
in ruling that the search is valid since they areabout to or attempting to
commit a crime due to their attemptto flee when the officers approached
them.Issue:Was the search qualified as a search incidental tovalid arrest
or stop frisk measure?Held:None. The lower and appellate court erred in
thedecision in such a way the the case at bar is neither a lawfulsearch
incidental to valid arrest nor a valid warrantlesssearch by stop frisk
methodThe search is not qualified as a search incidental toa valid arrest.In
the instant petition, the trial court validated thewarrantless search as a
"stop and frisk" with "the seizure of the grenade from the accused [as an
appropriate incident tohis arrest," hence necessitating a brief discussion
on thenature of these exceptions to the warrant requirement. At the
outset, we note that the trial court confusedthe concepts of a "stop-andfrisk" and of a search incidentalto a lawful arrest. These two types of
warrantless searchesdiffer in
terms of the requisite quantum of proof before they may be validly
effected and in their allowable scope

.
In a search incidental to a lawful arrest
, as theprecedent arrest determines the validity of the incidentalsearch,
the legality of the arrest is questioned in a largemajority of these cases,
e.g., whether an arrest was merelyused as a pretext for conducting a
search. 36 In thisinstance, the law requires that there first be a lawful
arrestbefore a search can be made the process cannot bereversed.
At bottom, assuming a valid arrest, the arrestingofficer may search the
person of the arrestee and the areawithin which the latter may reach for a
weapon or for evidence to destroy, and seize any money or property
foundwhich was used in the commission of the crime, or the fruit of the
crime, or that which may be used as evidence, or whichmight furnish the
arrestee with the means of escaping or committing violence.Here, there
could have been no valid in
flagrantedelicto
or hot pursuit arrest preceding the search in light of the lack of personal
knowledge on the part of Yu, thearresting officer, or an overt physical act,
on the part of petitioner, indicating that a crime had just been
committed,was being committed or was going to be committed.
Having thus shown the invalidity of thewarrantless arrest in this case,
plainly, the searchconducted on petitioner could not have been
oneincidental to a lawful arrest.
On the other hand, stop and frisk is limitedprotective search of outer
clothing for weapons or any other incriminating evidence against the
arrestee, which does notrequire probable cause but the existence of a
genuinereason. A "stop-and-frisk" serves a two-fold interest:(
1
) The general interest of effective crimeprevention and detection, which
underlies therecognition that a police officer may, under appropriate
circumstances and in an appropriatemanner, approach a person for
purposes of investigating possible criminal behavior evenwithout
probable cause; and(2) The more pressing interest of safety andselfpreservation which permit the police officer totake steps to assure himself
that the person withwhom he deals is not armed with a deadly
weaponthat could unexpectedly and fatally be used againstthe police
officer.The court did not qualify the case at bar as a validwarrantless
search through stop and frisk method. For thefollowing reasons:
First
, we harbor grave doubts as to Yu's claim thatpetitioner was a member of
the group which attempted tobomb Plaza Miranda two days earlier.This
claim is neither supported by any police reportor record nor corroborated
by any other police officer whoallegedly chased that group. Aside from
impairing Yu'scredibility as a witness, this likewise diminishes
theprobability that a genuine reason existed so as to arrest andsearch

petitioner.If only to further tarnish the credibility of Yu'stestimony,


contrary to his claim that petitioner and hiscompanions had to be chased
before being apprehended,the affidavit of arrest (Exh. "A") expressly
declares otherwise,i.e., upon arrival of five (5) other police officers,
petitioner andhis companions were "immediately collared."
Second, there was nothing in petitioner's behavior or conduct which
could have reasonably elicited even meresuspicion other than that his
eyes were "moving very fast"an observation which leaves us incredulous
since Yu and histeammates were nowhere near petitioner and it was
already6:30
p.m., thus presumably dusk. Petitioner and hiscompanions were merely
standing at the corner and were notcreating any commotion or trouble.
Third
, there was at all no ground, probable or otherwise, to believe that
petitioner was armed with a deadlyweapon. None was visible to Yu, for
as he admitted, thealleged grenade was "discovered" "inside the front
waistline"of petitioner, and from all indications as to the distancebetween
Yu and petitioner, any telltale bulge, assuming thatpetitioner was indeed
hiding a grenade, could not have beenvisible to Yu. In fact, as noted by
the trial court:When the policemen approached the accused andhis
companions, they were not yet aware that a handgrenade was tucked
inside his waistline. They did not seeany bulging object in [sic] his
person.What is unequivocal then in this case are blatantviolations of
petitioner's rights solemnly guaranteed inSections 2 and12(1) of Article
III of the Constitution.Lastly, the search is illegal since the officers
hasample time to procure a valid search and arrest warrant.
In the issue of admissibility of conducted testimony andinvestigation by
NBI:
Serapio conducted the custodial investigation onpetitioner the day
following his arrest. No lawyer was presentand Serapio could not have
requested a lawyer to assistpetitioner as no PAO lawyer was then
available. Thus, even if petitioner consented to the investigation and
waived hisrights to remain silent and to counsel, the waiver was invalidas
it was not in writing, neither was it executed in thepresence of
counsel.Contrary to Sec12 Art III involving right to acompetent counsel
and that any admission or admissionobtained in violation of this or
Section17 shall beinadmissible in evidence against him.

(Tondo Manila) were unconstitutional. They alleged that


there is no specific target house to be search and that there is
no search warrant or warrant of arrest served. Most of the
policemen are in their civilian clothes and without
nameplates or identification cards. The residents were rudely
rouse from their sleep by banging on the walls and windows
of their houses. The residents were at the point of highpowered guns and herded like cows. Men were ordered to
strip down to their briefs for the police to examine their
tattoo marks. The residents complained that they're homes
were ransacked, tossing their belongings and destroying their
valuables. Some of their money and valuables had
disappeared after the operation. The residents also reported
incidents of maulings, spot-beatings and maltreatment.
Those who were detained also suffered mental and physical
torture to extract confessions and tactical informations. The
respondents said that such accusations were all lies.
Respondents contends that the Constitution grants to
government the power to seek and cripple subversive
movements for the maintenance of peace in the state. The
aerial target zoning were intended to flush out subversives
and criminal elements coddled by the communities were the
said drives were conducted. They said that they have
intelligently and carefully planned months ahead for the
actual operation and that local and foreign media joined the
operation to witness and record such event.
Issue: Whether or Not the saturation drive committed
consisted
of
violation
of
human
rights.

Held: It is not the police action per se which should be


prohibited rather it is the procedure used or the methods
which "offend even hardened sensibilities" .Based on the
facts stated by the parties, it appears to have been no
impediment to securing search warrants or warrants of arrest
before any houses were searched or individuals roused from
GUANZON VS. DE VILLA [181 SCRA 623; G.R. 80508; sleep were arrested. There is no showing that the objectives
sought to be attained by the "aerial zoning" could not be
30 JAN 1990]
achieved even as th rights of the squatters and low income
Friday, February 06, 2009 Posted by Coffeeholic Writes
families are fully protected. However, the remedy should not
Labels: Case Digests, Political Law
be brought by a tazpaer suit where not one victim complaints
Facts: The 41 petitioners alleged that the "saturation drive" and not one violator is properly charged. In the
or "aerial target zoning" that were conducted in their place circumstances of this taxpayers' suit, there is no erring

soldier or policeman whom the court can order prosecuted. committed in furtherance therefore in connection therewith After making a preliminary investigation based on the
In the absence of clear facts no permanent relief can be constitute direct assaults against the state and are in the affidavits of the complainant and her witnesses and countergiven.
nature of continuing crimes.
affidavits of the respondent and his witnesses, as provided in
Section 3, Rule 112 of the 1985 Rules on Criminal
In the meantime where there is showing that some abuses
Procedure, Judge Samulde transmitted the records of the
were committed, the court temporary restraint the alleged G.R. No. 78606 September 26, 1988
case to Provincial Fiscal Ramon Salvani with his finding that
violations which are shocking to the senses. Petition is
"there is prima facie evidence of robbery as charge in the
remanded to the RTC of Manila.
GELACIO V. SAMULDE, in his official capacity as
complaint" (Annex A of Answer).
Municipal Judge, petitioner,
UMIL VS. RAMOS [187 SCRA 311; G.R. NO. 81567; 3
vs.
OCT 1991]
The fiscal returned the records to Judge Samulde on the
Wednesday, February 04, 2009 Posted by Coffeeholic Writes RAMON M. SALVANI, Jr., in his official capacity as
ground that the transmittal of the records to his office was
Provincial fiscal of Antique, substituted by LEOPOLDO
"premature" because Judge Samulde failed to include the
Labels: Case Digests, Political Law
O. VILLAVERT, respondent.
warrant of arrest against the accused as provided in Section
5, Rule 112 of the 1985 Rules on Criminal Procedure.
Facts: On 1 February 1988, military agents were dispatched
to the St. Agnes Hospital, Roosevelt Avenue, Quezon City,
to verify a confidential information which was received by
Judge Samulde sent back the records to Fiscal Salvani. He
their office, about a "sparrow man" (NPA member) who had GRIO-AQUINO, J.:
pointed out that under Section 6, Rule 112, he may issue a
been admitted to the said hospital with a gunshot wound.
warrant of arrest if he is satisfied "that a probable cause
That the wounded man in the said hospital was among the This case involves a disagreement between an investigating
five (5) male "sparrows" who murdered two (2) Capcom judge and the provincial fiscal on whether it is mandatory for exists and that there is a necessity of placing the respondent
mobile patrols the day before, or on 31 January 1988 at the former to issue a warrant for the arrest of the accused in under immediate custody in order not to frustrate the ends of
justice, " implying thereby that, although he found that a
about 12:00 o'clock noon, before a road hump along view of his finding, after conducting a preliminary
Macanining St., Bagong Barrio, Caloocan City. The
probable cause existed, he did not believe that Arangale
wounded man's name was listed by the hospital management investigation, that there exists prima facie evidence that the should be immediately placed under custody so as not to
as "Ronnie Javellon," twenty-two (22) years old of Block 10, accused committed the crime charged.
frustrate the ends of justice. Hence, he refused to issue a
Lot 4, South City Homes, Bian, Laguna however it was
warrant of arrest.
disclosed later that the true name of the wounded man was Municipal Judge Gelacio Samulde of Patnogon, Antique,
Rolando Dural. In view of this verification, Rolando Dural conducted a preliminary investigation of Pelayo Arangale
On October 9, 1986, a special civil action of mandamus was
was transferred to the Regional Medical Servicesof the upon a complaint for robbery filed on October 29, 1985 by
filed in the Regional Trial Court of Antique by Provincial
CAPCOM, for security reasons. While confined thereat, he Maria Magbanua, alleging that Arangale harvested palay
was positively identified by the eyewitnesses as the one who
Fiscal Salvani against Judge Samulde to compel the latter to
murdered
the
2
CAPCOM
mobile
patrols. from a portion of her land directly adjoining Arangale's land issue a warrant for the arrest of Arangale.
(Crim. Case No. 2046-B, entitled "People of the Philippines
vs. Pelayo Arangale").
During the pendency of the case in the lower court, Fiscal
Issue: Whether or Not Rolando was lawfully arrested.
Salvani was replaced by Fiscal Leopoldo Villavert as
provincial fiscal of Antique. Believing that the procedural
question involved is important, Fiscal Villavert manifested to
Held: Rolando Dural was arrested for being a member of the
NPA, an outlawed subversive organization. Subversion being
the Court that there is need to continue the case begun by his
a continuing offense, the arrest without warrant is justified as
predecessor.
it can be said that he was committing as offense when
arrested. The crimes rebellion, subversion, conspiracy or
On February 12, 1987, the Regional Trial Judge Pedro
proposal to commit such crimes, and crimes or offenses
Icamina dismissed the petition for mandamus on the ground

that "the petitioner had not shown that he has a clear, legal
right to the performance of the act to be required of
respondent and that the latter had an imperative duty to
perform (it)," citing this Court's decision in Felix Morada vs.
Hon. Hermogenes Caluag, 5 SCRA 1128. Nevertheless, he
ordered Judge Samulde to issue a warrant for the arrest of
Arangale in Crim. Case No. 2046-B in view of his (Judge
Samulde's) resolution dated May 22, 1986, and to transmit
the warrant, if the arrest is by virtue of a warrant, to the
Provincial Fiscal for appropriate action in accordance with
the provisions of Section 5, Rule 112 of the 1985 Rules on
Criminal Procedure. He further advised the Municipal Judge
"that henceforth he adheres to the same rule in similar cases
where he conducts a preliminary investigation with a finding
of a prima facie or probable cause." (pp. 23-32, Records.)
Unconvinced, Judge Samulde appealed to this Court. The
issue posed by this case necessitates an examination of the
history and development of the rule on preliminary
investigation. Section 13 of General Orders No. 58 dated
April 23, 1900 of the U.S. Military Governor in the
Philippines was the original source of the rule on preliminary
investigation. It provided
SEC. 13. When a complaint or information
alleging the commission of a crime is laid
before a magistrate, he must examine, on
oath, the informant or prosecutor and the
witnesses produced, and take their
depositions in writing, causing them to be
subscribed by the parties making them. If
the magistrate be satisfied from the
investigation that the crime complained of
has been committed, and that there is
reasonable ground to believe that the party
charged has committed it, he must issue an
order for his arrest. If the offense be
bailable, and the defendant offers a
sufficient security, he shall be admitted to
bail; otherwise he shall be committed to

prison. (General Orders & Circulars, issued


by the Office of U.S. Military Governor in
the Philippine Islands, 1900, p. 3, Emphasis
supplied.)
It was amended by Act 194 of the Public Laws enacted by
the Philippine Commission with Amendments indicated, Vol.
I, p. 527, which authorized every justice of the peace to
conduct such investigation and order the arrest of the
accused if he believed the complaint to be well founded.
SEC. 1. Every justice of the peace in the
Philippine Islands is hereby invested with
authority to make preliminary investigation
of any crime alleged to have been
committed within his municipality,
jurisdiction to hear and determine which is
by law now vested in the judges of Courts of
First Instance. It shall be the duty of every
justice of the peace, when written complaint
under oath has been made to him that a
crime has been committed within his
municipality and there is reason to believe
that any person has committed the same,
which complaint the justice believes to be
well founded, or when he has knowledge of
facts tending to show the commission of a
crime within his municipality by any person,
to issue an order for the arrest of the
accused and have him brought before the
justice of the peace for such preliminary
examination. (Emphasis supplied.)

Under Rule 112 of the 1964 Rules of Court, the provincial


fiscal was added to the enumeration of persons authorized to
conduct a preliminary investigation.
In the 1985 Rules on Criminal Procedure, Section 2, Rule
112, the list grew even longer to include: (a) provincial or
city fiscals and their assistants; (b) Judges of the Municipal
Trial Courts and Municipal Circuit Trial Courts; (c) National
and Regional state prosecutors; and (d) other officers
authorized by law. *
Both the 1940 and 1964 Rules of Court provided for two (2)
stages of the preliminary investigation, to wit: (1) the
"previous inquiry or examination" of the complainant and his
witnesses to determine whether a warrant of arrest should
issue against the defendant, and (2) the preliminary
investigation proper of the defendant himself to determine if
he should be held for trial. Thus, the preliminary
investigation was defined as:
... a previous inquiry or examination made
before the arrest of the defendant by the
judge . . . for the purpose of determining
whether there is a reasonable ground to
believe that an offense has been committed
and the defendant is probably guilty thereof,
so as to issue a warrant of arrest and to
hold him for trial. (Sec. 1, Rule 108, 1940
Rules of Court. Emphasis supplied.)

In Section 1, Rule 112 of the 1964 Rules of Court, the


distinction between a preliminary examination and
preliminary investigation was more clearly defined by using
The rule was substantially unchanged under Rule 108 of the the term "preliminary examination" in Section I of the Rule
Rules of Court except that municipal judges and city fiscals to differentiate the first stage of the preliminary investigation
were also authorized to conduct the preliminary investigation (where only the testimonies of the complainant and his
witnesses were taken), from the second stage where, after the
of offenses committed within their municipality or city
arrest of the defendant, he was informed of the complaint
cognizable by the Court of First Instance.
against him and given a chance to testify and present his
evidence (Sec. 10, Rule 112, 1964 Rules of Court). The

purpose of the preliminary examination was still to


determine "whether there is a reasonable ground to believe
that an offense has been committed and the accused is
probably guilty thereof, so that a warrant of arrest may be
issued and the accused held for trial. " (Sec. 1, Rule 112,
1964 Revised Rules of Court.)
In both the 1940 and 1964 Rules of Court, it was mandatory
upon the investigating judge to issue a warrant for the arrest
of the accused, if he was satisfied that the offense charged
was committed and that the accused probably committed it.
Accordingly, Section 7, Rule 108 of the 1940 Rules of Court
provided:

(Sec. 13, Rule 108,1940 Rules of Court; Sec. 12, Rule


112,1964 Revised Rules of Court).
However, the rule on preliminary investigation underwent
some modifications in the 1985 Rules on Criminal
Procedure, which is the applicable rule in this case. Under
Section 1 of the present rule, the definition of the purpose of
a preliminary investigation, does not contemplate the
issuance of a warrant of arrest by the investigating judge or
officer:
SECTION 1. Definition. Preliminary
investigation is an inquiry or proceeding for
the purpose of determining whether there is
sufficient ground to engender a well founded
belief that a crime cognizable by the
Regional Trial Court has been committed
and that the respondent is probably guilty
thereof, and should be held for trial.

SEC. 7. Warrant of arrest, when issued.


If the judge be satisfied from the preliminary
investigation conducted by him that the
offense complained of has been committed
and that there is reasonable ground to
believe that the defendant has committed it,
The mandatory provision that the investigating judge "must
he must issue a warrant or order for his
issue a warrant of arrest" if he finds probable cause that the
arrest. (Emphasis ours.)
respondent committed the crime charged, found in all
Section 6, Rule 112 of the 1964 Rules of Court similary
previous rules of criminal procedure, from General Orders
provided:
No. 58 down to Rule 112 of the 1964 Revised Rules of
Court, is absent in Section 1 of the 1985 Rules on Criminal
SEC. 6. Warrant of arrest, when issued.
Procedure.
If the judge be satisfied from the preliminary
examination conducted by him or by the
Another significant change is that under the 1985 Rules on
investigating officer that the offense
Criminal Procedure there is only one (1) way of conducting
complained of has been committed and that a preliminary investigation, and that is by affidavits and
there is reasonable ground to believe that the counter-affidavits submitted by the parties to the
accused has committed it, he must issue a
investigating judge under Section 3, Rule 112. On the basis
warrant or order for his arrest. (Emphasis
of the affidavits, the investigating judge shall "determine
supplied.)
whether or not there is sufficient ground to hold the
respondent for trial' (subpar. f ). Gone is the requirement in
Because the arrest of the accused was mandatory, the records the 1940 and 1964 Rules of Court that "he must issue a
to be transmitted by the investigating judge to the clerk of
warrant or order" for the arrest of the defendant.
the Court of First Instance upon the conclusion of the
preliminary investigation, included the warrant of arrest

To determine whether a warrant of arrest should issue


against the accused, the investigating judge must examine
the complainant and his witnesses "in writing and under oath
... in the form of searching questions and answers." When he
is "satisfied that a probable cause exists, and that there is a
necessity of placing the respondent under immediate custody
in order not to frustrate the ends of justice," he may issue the
warrant as provided in Section 6, par. b, of the 1985 Rules on
Criminal Procedure.
SEC. 6 When warrant of arrest may issue.

(a) By the Regional Court.....


(b) By the Municipal Trial Court. -If the
municipal trial judge conducting the
preliminary investigation is satisfied after an
examination in writing and under oath of the
complainant and his witnesses in the form of
searching questions and answers, that a
probable cause exists and that there is a
necessity of placing the respondent under
immediate custody in order not to frustrate
the ends of justice, he shall issue a warrant
of arrest.
As correctly argued by the petitioner Judge Samulde, three
(3) conditions must concur for the issuance of the warrant of
arrest. The investigating judge must:
(a) have examined in writing and under oath
the complainant and his witnesses by
searching questions and answers;
(b) be satisfied that a probable cause exists;
and

(c) that there is a need to place the


The fiscal's speedy and adequate remedy, if he believes that
respondent under immediatecustody in order the accused should be immediately placed under custody so
not to frustrate the ends of justice.
as not to frustrate the ends of justice, is not to file a
mandamus action (which may take two years or more to
It is an entirely new rule, and it is plain to see that it is not
finally resolve, as happened in this case), but as sensibly
obligatory, but merely discretionary, upon the investigating
indicated by the petitioner, to immediately file the
judge to issue a warrant for the arrest of the accused, even
information so that the Regional Trial Court may issue a
after having personally examined the complainant and his
warrant for the arrest of the accused (Sec. 6, par. a, Rule
witnesses in the form of searching questions and answers,
112,1985 Rules on Criminal Procedure).
for the determination of whether a probable cause exists and
whether it is necessary to arrest the accused in order not to
WHEREFORE, the appealed decision in Civil Case No.
frustrate the ends of justice, is left to his sound judgment or 2145 is set aside. No costs.
discretion.
SO ORDERED.
In this particular case, since the robbery charge was the
Narvasa, Cruz, Gancayco and Medialdea JJ., concur.
offshoot of a boundary dispute between two property
owners, the investigating judge did not believe there was any
danger of the accused absconding before the filing of the
information against him by the fiscal, hence, he found no
People vs Court of Appeals (291 SCRA 400)
need to place him under immediate custody.
FACTS
A petition for certiorari has been filed to invalidate the order
The provincial fiscal anchored his action for mandamus on
of Judge Casanova which quashed search warrant issued by
Section 5, Rule 112 of the 1985 Rules on Canal Procedure
Judge Bacalla and declared inadmissible for any purpose the
which provides that upon the termination of the preliminary items seized under the warrant.
>An application for a search warrant was made by S/Insp
investigation, the investigating judge should transmit to the
provincial fiscal (instead of the clerk of Court of the CFI as Brillantes against Mr. Azfar Hussain who had allegedly in
his possession firearms and explosives at Abigail Variety
provided in the 1940 and 1964 Rules of Court) the warrant
Store, Apt 1207 Area F. Bagon Buhay Avenue, Sarang Palay,
of arrest and other records of the preliminary investigation.
San Jose Del Monte, Bulacan. The following day Search
From that he deduced that the investigating judge must issue Warrant No. 1068 was issued but was served not at Abigail
a warrant for the arrest of the accused upon the conclusion of Variety Store but at Apt. No. 1, immediately adjacent to
Abigail Variety Store resulting in the arrest of 4 Pakistani
the preliminary investigation. That inference is not correct.
The provision of Section 5, Rule 112 simply means that the nationals and the seizure of a number of different explosives
and firearms.
warrant of arrest, if one was issued, shall be transmitted to
the fiscal with the records of the preliminary investigation. If
ISSUE: WON a search warrant was validly issued as regard
the investigating judge, in the exercise of his sound
the apartment in which private respondents were then
discretion, decides not to issue a warrant of arrest, then none actually residing, or more explicitly, WON that particular
need be transmitted to the fiscal, and he may not be
apartment had been specifically described in the warrant.
compelled by mandamus to issue it (Vda. de Crisologo vs.
Court of Appeals, 137 SCRA 231; Pio vs. Marcos, 56 SCRA HELD:
The ambiguity lies outside the instrument, arising from the
725; PAL Employees Assn. vs. PAL, Inc., III SCRA 215).

absence of a meeting of minds as to the place to be searched


between the applicants for the warrant and the Judge issuing
the same; and what was done was to substitute for the place
that the Judge had written down in the warrant, the premises
that the executing officers had in their mind. This should not
have been done. It is neither fair nor licit to allow police
officers to search a place different from that stated in the
warrant on the claim that the place actually searched
although not that specified in the warrant is exactly what
they had in view when they applied for the warrant and had
demarcated in their supporting evidence. What is material in
determining the validity of a search is the place stated in the
warrant itself, not what the applicants had in their thoughts,
or had represented in the proofs they submitted to the court
issuing the warrant.
The place to be searched, as set out in the warrant, cannot be
amplified or modified by the officers'
own personal knowledge of the premises, or the evidence
they adduced in support of their application for the warrant.
Such a change is proscribed by the Constitution which
requires inter alia the search warrant to particularly describe
the place to be searched as well as the persons or things to be
seized. It would concede to police officers the power of
choosing the place to be searched, even if it not be that
delineated in the warrant. It would open wide the door to
abuse of the search process, and grant to officers executing a
search warrant that discretion which the Constitution has
precisely removed from them. The particularization of the
description of the place to be searched may properly be done
only by the Judge, and only in the warrant itself; it cannot be
left to the discretion of the police officers conducting the
search.
Prudente vs. Dayrit, GR No.82870
(Constitutional Law Search Warrant, Probable Cause)
Facts: Based on an information from a verified source that
petitioner violated the Dangerous Drug Act of 1972, police
officers applied for a search warrant which was subsequently
issued by respondent judge. Petitioner assailed the validity of
the search warrant on the ground that there was no probable
cause for the issuance of search warrant.

Issue: Whether or not a warrant issued based on verified


information to the police is valid.
Held: No. Probable cause must be shown to be within the
personal knowledge of the complainant or the witnesses and
not simply on hearsay.
G.R. No. 82870 December 14, 1989
DR. NEMESIO E. PRUDENTE, petitioner,
vs.
THE HON. EXECUTIVE JUDGE ABELARDO M.
DAYRIT, RTC Manila, Branch 33 and PEOPLE OF
THE PHILIPPINES, respondents.
Francisco SB Acejas III, Oscar S. Atencio, Rodolfo M.
Capocyan, Ernesto P. Fernandez, Romulo B. Macalintal,
Rodrigo H. Melchor, Rudegelio D. Tacorda Virgilio L. Valle
and Luciano D. Valencia for petitioner.

PADILLA, J.:
This is a petition for certiorari to annul and set aside the
order of respondent Judge dated 9 March 1988 which denied
the petitioner's motion to quash Search Warrant No. 87-14,
as well as his order dated 20 April 1988 denying petitioner's
motion for reconsideration of the earlier order.
It appears that on 31 October 1987, P/Major Alladin
Dimagmaliw, Chief of the Intelligence Special Action
Division (ISAD) of the Western Police District (WPD) filed
with the Regional Trial Court (RTC) of Manila, Branch 33,
presided over by respondent Judge Abelardo Dayrit, now
Associate Justice of the Court of Appeals. an application 1
for the issuance of a search warrant, docketed therein as
SEARCH WARRANT NO. 87-14, for VIOLATION OF PD
NO. 1866 (Illegal Possession of Firearms, etc.) entitled
"People of the Philippines, Plaintiff, versus Nemesis E.

Prudente, Defendant." In his application for search warrant,


P/Major Alladin Dimagmaliw alleged, among others, as
follows:
1. That he has been informed and has good
and sufficient reasons to believe that
NEMESIO PRUDENTE who may be found
at the Polytechnic University of the
Philippines, Anonas St. Sta. Mesa,
Sampaloc, Manila, has in his control or
possession firearms, explosives
handgrenades and ammunition which are
illegally possessed or intended to be used as
the means of committing an offense which
the said NEMESIO PRUDENTE is keeping
and concealing at the following premises of
the Polytechnic University of the
Philippines, to wit:
a. Offices of the Department
of Military Science and
Tactics at the ground floor
and other rooms at the
ground floor;
b. Office of the President,
Dr. Nemesio Prudente at
PUP, Second Floor and
other rooms at the second
floor;
2. That the undersigned has verified the
report and found it to be a fact, and
therefore, believes that a Search Warrant
should be issued to enable the undersigned
or any agent of the law to take possession
and bring to this Honorable Court the
following described properties:
a. M 16 Armalites with ammunitions;

b. .38 and .45 Caliber handguns and pistols;


c. explosives and handgrenades; and,
d. assorted weapons with ammunitions.
In support of the application for issuance of search warrant,
P/Lt. Florenio C. Angeles, OIC of the Intelligence Section of
(ISAD) executed a "Deposition of Witness" dated 31
October 1987, subscribed and sworn to before respondent
Judge. In his deposition, P/Lt. Florenio Angeles declared,
inter alia, as follows:
Q: Do you know P/Major
Alladin Dimagmaliw, the
applicant for a Search
Warrant?
A: Yes, sir, he is the Chief,
Intelligence and Special
Action Division, Western
Police District.
Q: Do you know the
premises of Polytechnic
University of the
Philippines at Anonas St.,
Sta. Mesa, Sampaloc,
Manila
A: Yes, sir, the said place
has been the subject of our
surveillance and observation
during the past few days.
Q: Do you have personal
knowledge that in the said
premises is kept the
following properties subject
of the offense of violation of

PD No. 1866 or intended to


be used as a means of
committing an offense:
a. M 16 Armalites with ammunitions;
b. .38 and 45 Caliber handguns and pistols;
c. explosives and handgrenades; and d.
Assorted weapons with ammunitions?
A: Yes sir.
Q: Do you know who is or
who are the person or
persons who has or have
control of the abovedescribed premises?
A: Yes sir, it is Dr. Nemesio
Prudente, President of the
Polytechnic University of
the Philippines.
Q: How do you know that
said property is subject of
the offense of violation of
Pres. Decree No. 1866 or
intended to be used as the
means of committing an
offense?
A: Sir, as a result of our
continuous surveillance
conducted for several days,
we gathered information
from verified sources that
the holder of said firearms
and explosives as well as
ammunitions aren't licensed

to possess said firearms and


ammunition. Further, the
premises is a school and the
holders of these firearms are
not students who were not
supposed to possess
firearms, explosives and
ammunition.
On the same day, 31 October 1987, respondent Judge issued
Search Warrant No. 87-14, 3 the pertinent portions of which
read as follows:
It appearing to the satisfaction of the
undersigned, after examining under oath
applicant ALLADIN M. DIMAGMALIW
and his witness FLORENIO C. ANGELES
that there are good and sufficient reasons to
believe (probable cause) that NEMESIO
PRUDENTE has in his control in the
premises of Polytechnic University of the
Philippines, Anonas St., Sta. Mesa,
Sampaloc, Manila, properties which are
subject of the above offense or intended to
be used as the means of committing the said
offense.
You are hereby commanded to make an
immediate search at any time in the day or
night of the premises of Polytechnic
University of the Philippines, more
particularly (a) offices of the Department of
Military Science and Tactics at the ground
floor and other rooms at the ground floor;
(b) office of the President, Dr. Nemesio
Prudente at PUP, Second Floor and other
rooms at the second floor, and forthwith
seize and take possession of the following
personal properties, to wit:

a. M 16 Armalites with ammunition;


b. .38 and .45 Caliber handguns and pistols;
c. explosives and hand grenades; and
d. assorted weapons with ammunitions.
and bring the above described properties to
the undersigned to be dealt with as the law
directs.
On 1 November 1987, a Sunday and All Saints Day, the
search warrant was enforced by some 200 WPD operatives
led by P/Col. Edgar Dula Torre, Deputy Superintendent,
WPD, and P/Major Romeo Maganto, Precinct 8 Commander.
In his affidavit, 4 dated 2 November 1987, Ricardo Abando y
Yusay, a member of the searching team, alleged that he
found in the drawer of a cabinet inside the wash room of Dr.
Prudente's office a bulging brown envelope with three (3)
live fragmentation hand grenades separately wrapped with
old newspapers, classified by P/Sgt. J.L. Cruz as follows (a)
one (1) pc.M33 Fragmentation hand grenade (live); (b)
one (11) pc.M26 Fragmentation hand grenade (live); and
(c) one (1) pc.PRB423 Fragmentation hand grenade
(live).
On 6 November 1987, petitioner moved to quash the search
warrant. He claimed that (1) the complainant's lone witness,
Lt. Florenio C. Angeles, had no personal knowledge of the
facts which formed the basis for the issuance of the search
warrant; (2) the examination of the said witness was not in
the form of searching questions and answers; (3) the search
warrant was a general warrant, for the reason that it did not
particularly describe the place to be searched and that it
failed to charge one specific offense; and (4) the search
warrant was issued in violation of Circular No. 19 of the
Supreme Court in that the complainant failed to allege under

oath that the issuance of the search warrant on a Saturday


was urgent. 5
The applicant, P/Major Alladin Dimagmaliw thru the Chief,
Inspectorate and Legal Affairs Division, WPD, opposed the
motion. 6 After petitioner had filed his reply 7 to the
opposition, he filed a supplemental motion to quash. 8

the personal knowledge of the complainant or the witnesses


he may produce and not based on mere hearsay. 16

Petitioner assails the validity of Search Warrant No. 87-14


on the ground that it was issued on the basis of facts and
circumstances which were not within the personal
knowledge of the applicant and his witness but based on
hearsay evidence. In his application for search warrant,
Thereafter, on 9 March 1988, respondent Judge issued an
P/Major Alladin Dimagmaliw stated that "he has been
9
order, denying the petitioner's motion and supplemental
informed" that Nemesio Prudente "has in his control and
motion to quash. Petitioner's motion for reconsideration 10
possession" the firearms and explosives described therein,
11
was likewise denied in the order dated 20 April 1988.
and that he "has verified the report and found it to be a fact."
On the other hand, in his supporting deposition, P/Lt.
Hence, the present recourse, petitioner alleging that
Florenio C. Angeles declared that, as a result of their
respondent Judge has decided a question of substance in a
continuous surveillance for several days, they "gathered
manner not in accord with law or applicable decisions of the informations from verified sources" that the holders of the
Supreme Court, or that the respondent Judge gravely abused said fire arms and explosives are not licensed to possess
his discretion tantamount to excess of jurisdiction, in issuing them. In other words, the applicant and his witness had no
the disputed orders.
personal knowledge of the facts and circumstances which
became the basis for issuing the questioned search warrant,
For a valid search warrant to issue, there must be probable
but acquired knowledge thereof only through information
cause, which is to be determined personally by the judge,
from other sources or persons.
after examination under oath or affirmation of the
complainant and the witnesses he may produce, and
While it is true that in his application for search warrant,
particularly describing the place to be searched and the
applicant P/Major Dimagmaliw stated that he verified the
persons or things to be seized. 12 The probable cause must be information he had earlier received that petitioner had in his
in connection with one specific offense 13 and the judge
possession and custody the t there is nothing in the record to
must, before issuing the warrant, personally examine in the
show or indicate how and when said applicant verified the
form of searching questions and answers, in writing and
earlier information acquired by him as to justify his
under oath, the complainant and any witness he may
conclusion that he found such information to be a fact. He
produce, on facts personally known to them and attach to the might have clarified this point if there had been searching
record their sworn statements together with any affidavits
questions and answers, but there were none. In fact, the
14
submitted.
records yield no questions and answers, whether searching or
not, vis-a-vis the said applicant.
The "probable cause" for a valid search warrant, has been
defined "as such facts and circumstances which would lead a What the records show is the deposition of witness, P/Lt.
reasonably discreet arid prudent man to believe that an
Angeles, as the only support to P/Major Dimagmaliw's
offense has been committed, and that objects sought in
application, and the said deposition is based on hearsay. For,
connection with the offense are in the place sought to be
it avers that they (presumably, the police authorities) had
searched." 15 This probable cause must be shown to be within conducted continuous surveillance for several days of the

suspected premises and, as a result thereof, they "gathered


information from verified sources" that the holders of the
subject firearms and explosives are not licensed to possess
them.
In Alvarez vs. Court of First Instance, 17 this Court laid the
following test in determining whether the allegations in an
application for search warrant or in a supporting deposition,
are based on personal knowledge or not
The true test of sufficiency of a deposition
or affidavit to warrant issuance of a search
warrant is whether it has been drawn in a
manner that perjury could be charged
thereon and the affiant be held liable for
damage caused. The oath required must refer
to the truth of the facts within the personal
knowledge of the applicant for search
warrant, and/or his witnesses, not of the
facts merely reported by a person whom one
considers to be reliable.
Tested by the above standard, the allegations of the witness,
P/Lt. Angeles, in his deposition, do not come up to the level
of facts of his personal knowledge so much so that he cannot
be held liable for perjury for such allegations in causing the
issuance of the questioned search warrant.
In the same Alvarez case, 18 the applicant stated that his
purpose for applying for a search warrant was that: "It had
been reported to me by a person whom I consider to be
reliable that there are being kept in said premises books,
documents, receipts, lists, chits and other papers used by him
in connection with his activities as a money lender,
challenging usurious rate of interests, in violation of law."
The Court held that this was insufficient for the purpose of
issuing a search warrant.
In People vs. Sy Juco, 19 where the affidavit contained an
allegation that there had been a report to the affiant by a

person whom lie considered reliable that in said premises


were "fraudulent books, correspondence and records," this
was likewise held as not sufficient for the purpose of issuing
a search warrant. Evidently, the allegations contained in the
application of P/ Major Alladin Dimagmaliw and the
declaration of P/Lt. Florenio C. Angeles in his deposition
were insufficient basis for the issuance of a valid search
warrant. As held in the Alvarez case:
The oath required must refer to the truth of
the facts within the personal knowledge of
the petitioner or his witnesses, because the
purpose thereof is to convince the
committing magistrate, not the individual
making the affidavit and seeking the
issuance of the warrant, of the existence of
probable cause.
Besides, respondent Judge did not take the deposition of the
applicant as required by the Rules of Court. As held in Roan
v. Gonzales, 20 "(m)ere affidavits of the complainant and his
witnesses are thus not sufficient. The examining Judge has to
take depositions in writing of the complainant and the
witnesses he may produce and attach them to the record."

search warrant. The Court also notes post facto that the
search in question yielded, no armalites, handguns, pistols,
assorted weapons or ammunitions as stated in the application
for search warrant, the supporting deposition, and the search
warrant the supporting hand grenades were itself Only three
(3) live fragmentation found in the searched premises of the
PUP, according to the affidavit of an alleged member of the
searching party.

Petitioner next attacks the validity of the questioned warrant,


on the ground that it was issued in violation of the rule that a
search warrant can be issued only in connection with one
specific offense. The search warrant issued by respondent
judge, according to petitioner, was issued without any
reference to any particular provision of PD No. 1866 that
was violated when allegedly P.D. No. 1866 punishes several
offenses.

The Court avails of this decision to reiterate the strict


requirements for determination of "probable cause" in the
valid issuance of a search warrant, as enunciated in earlier
cases. True, these requirements are stringent but the purpose
is to assure that the constitutional right of the individual
against unreasonable search and seizure shall remain both
meaningful and effective.

In Stonehill vs. Diokno, 23 Where the warrants involved were


issued upon applications stating that the natural and juridical
persons therein named had committed a "violation of Central
Bank Laws, Tariff and Customs Laws, Internal Revenue
Code and Revised Penal Code," the Court held that no
specific offense had been alleged in the applications for a
search warrant, and that it would be a legal hearsay of the
highest order to convict anybody of a "Violation of Central
Bank Laws, Tariff and Customs Laws, Internal Revenue
Code and Revised Penal Code" without reference to any
determinate provision of said laws and codes.

Petitioner also assails the validity of the search warrant on


the ground that it failed to particularly describe the place to
be searched, contending that there were several rooms at the
ground floor and the second floor of the PUP.

In the present case, however, the application for search


The rule is, that a description of a place to be searched is
warrant was captioned: "For Violation of PD No. 1866
sufficient if the officer with the warrant can, with reasonable (Illegal Possession of Firearms, etc.) While the said decree
effort, ascertain and Identify the place intended . 22 In the
punishes several offenses, the alleged violation in this case
Moreover, a perusal of the deposition of P/Lt. Florenio
case at bar, the application for search warrant and the search was, qualified by the phrase "illegal possession of firearms,
Angeles shows that it was too brief and short. Respondent
warrant itself described the place to be searched as the
etc." As explained by respondent Judge, the term "etc."
Judge did not examine him "in the form of searching
premises of the Polytechnic University of the Philippines,
referred to ammunitions and explosives. In other words, the
questions and answers." On the contrary, the questions asked located at Anonas St., Sta. Mesa, Sampaloc, Manila more
search warrant was issued for the specific offense of illegal
were leading as they called for a simple "yes" or "no"
particularly, the offices of the Department of Military
possession of firearms and explosives. Hence, the failure of
21
answer. As held in Quintero vs. NBI," the questions
Science and Tactics at the ground floor, and the Office of the the search warrant to mention the particular provision of PD
propounded by respondent Executive Judge to the applicant's President, Dr. Nemesio Prudente, at PUP, Second Floor and No. 1-866 that was violated is not of such a gravity as to call
witness are not sufficiently searching to establish probable
other rooms at the second floor. The designation of the
for its invalidation on this score. Besides, while illegal
cause. Asking of leading questions to the deponent in an
places to be searched sufficiently complied with the
possession of firearms is penalized under Section 1 of PD
application for search warrant, and conducting of
constitutional injunction that a search warrant must
No. 1866 and illegal possession of explosives is penalized
examination in a general manner, would not satisfy the
particularly describe the place to be searched, even if there
under Section 3 thereof, it cannot be overlooked that said
requirements for issuance of a valid search warrant."
were several rooms at the ground floor and second floor of
decree is a codification of the various laws on illegal
the PUP.
possession of firearms, ammunitions and explosives; such
Manifestly, in the case at bar, the evidence failed to show the
illegal possession of items destructive of life and property
existence of probable cause to justify the issuance of the
are related offenses or belong to the same species, as to be

subsumed within the category of illegal possession of


firearms, etc. under P.D. No. 1866. As observed by
respondent Judge: 24
The grammatical syntax of the phraseology
comparative with the title of PD 1866 can
only mean that illegal possession of
firearms, ammunitions and explosives, have
been codified under Section 1 of said
Presidential Decree so much so that the
second and third are forthrightly species of
illegal possession of firearms under Section
(1) thereof It has long been a practice in the
investigative and prosecution arm of the
government, to designate the crime of illegal
possession of firearms, ammunitions and
explosives as 'illegal possession of firearms,
etc.' The Constitution as well as the Rules of
Criminal Procedure does not recognize the
issuance of one search warrant for illegal
possession of firearms, one warrant for
illegal possession of ammunitions, and
another for illegal possession of explosives.
Neither is the filing of three different
informations for each of the above offenses
sanctioned by the Rules of Court. The usual
practice adopted by the courts is to file a
single information for illegal possession of
firearms and ammunitions. This practice is
considered to be in accordance with Section
13, Rule 110 of the 1985 Rules on Criminal
Procedure which provides that: 'A complaint
or information must charge but one offense,
except only in those cases in which existing
laws prescribe a single punishment for
various offenses. Describably, the servers
did not search for articles other than
firearms, ammunitions and explosives. The
issuance of Search Warrant No. 87-14 is
deemed profoundly consistent with said rule

and is therefore valid and enforceable.


(Emphasis supplied)

HUBERT J. P. WEBB, petitioner,


vs.
HONORABLE RAUL E. DE LEON, the Presiding Judge
Finally, in connection with the petitioner's contention that the of the Regional Trial Court of Paraaque, Branch 258,
failure of the applicant to state, under oath, the urgent need
HONORABLE ZOSIMO V. ESCANO, the Presiding
for the issuance of the search warrant, his application having Judge of the Regional Trial Court of Paraaque, Branch
been filed on a Saturday, rendered the questioned warrant
259, PEOPLE OF THE PHILIPPINES, ZENON L. DE
invalid for being violative of this Court's Circular No. 19,
GUIA, JOVENCITO ZUO, LEONARDO GUIYAB,
dated 14 August 1987, which reads:
JR., ROBERTO LAO, PABLO FORMARAN, and
NATIONAL BUREAU OF INVESTIGATION, and
3. Applications filed after office hours,
HONORABLE AMELITA G. TOLENTINO, the
during Saturdays, Sundays and holidays
Presiding Judge of the Regional Trial Court of
shall likewise be taken cognizance of and
Paraaque, Branch 274, respondents, LAURO
acted upon by any judge of the court having VIZCONDE, intervenor.
jurisdiction of the place to be searched, but
in such cases the applicant shall certify and G.R. No. 121245 August 23, 1995
state the facts under oath, to the satisfaction
of the judge, that the issuance is urgent.
MICHAEL A. GATCHALIAN, petitioner,
vs.
it would suffice to state that the above section of the circular HONORABLE RAUL E. DE LEON, the Presiding Judge
merely provides for a guideline, departure from which would of the Regional Trial Court of Paraaque, Branch 258,
not necessarily affect the validity of an otherwise valid
HONORABLE ZOSIMO V. ESCANO, the Presiding
search warrant.
Judge of the Regional Trial Court of Paraaque, Branch
259, PEOPLE OF THE PHILIPPINES, ZENON L. DE
WHEREFORE, all the foregoing considered, the petition is GUIYAB, JR., ROBERTO LAO, PABLO FORMARAN,
GRANTED. The questioned orders dated 9 March 1988 and and NATIONAL BUREAU OF INVESTIGATION, and
20 April 1988 as well as Search Warrant No. 87-14 are
HONORABLE AMELITA G. TOLENTINO, the
hereby ANNULLED and SET ASIDE.
Presiding Judge of the Regional Trial Court of
Paraaque, Branch 274, respondents.
The three (3) live fragmentation hand grenades which,
according to Ricardo Y. Abando, a member of the searching G.R. No. 121297 August 23, 1995
team, were seized in the washroom of petitioner's office at
the PUP, are ordered delivered to the Chief, Philippine
ANTONIO L. LEJANO, petitioner,
Constabulary for proper disposition.
vs.
HONORABLE RAUL E. DE LEON, the Presiding Judge
SO ORDERED.
of the Regional Trial Court of Paraaque, Branch 258,
HONORABLE ZOSIMO V. ESCANO, the Presiding
Judge of the Regional Trial Court of Paraaque, Branch
G.R. No. 121234 August 23, 1995
259, PEOPLE OF THE PHILIPPINES, ZENON L. DE
GUIA, JOVENCITO ZUO, LEONARDO GUIYAB,

JR., ROBERTO LAO, PABLO FORMARAN, and


NATIONAL BUREAU OF INVESTIGATION, and
HONORABLE AMELITA G. TOLENTINO, the
Presiding Judge of the Regional Trial Court of
Paraaque, Branch 274, respondents.

PUNO, J.:
Before the Court are petitions for the issuance of the
extraordinary writs of certiorari, prohibition and mandamus
with application for temporary restraining order and
preliminary injunction to: (1) annul and set aside the
Warrants of Arrest issued against petitioners by respondent
Judges Raul E. de Leon and Amelita Tolentino in Criminal
Case No. 95-404; (2) enjoin the respondents from
conducting any proceeding in the aforementioned criminal
case; and (3) dismiss said criminal case or include Jessica
Alfaro as one of the accused therein. 1
From the records of the case, it appears that on June 19,
1994, the National Bureau of Investigation (NBI) filed with
the Department of Justice a letter-complaint charging
petitioners Hubert Webb, Michael Gatchalian, Antonio J.
Lejano and six (6) other persons, 2 with the crime of Rape
with Homicide. Forthwith, the Department of Justice formed
a panel of prosecutors headed by Assistant Chief State
Prosecutor Jovencio R. Zuo to conduct the preliminary
investigation 3 of those charged with the rape and killing on
June 30, 1991 of Carmela N. Vizconde; 4 her mother
Estrellita Nicolas-Vizconde, 5 and her sister Anne Marie
Jennifer 6 in their home at Number 80 W. Vinzons, St., BF
Homes, Paraaque, Metro Manila.
During the preliminary investigation, the NBI presented the
following: (1) the sworn statement dated May 22, 1995 of
their principal witness, Maria Jessica M. Alfaro who
allegedly saw the commission of the crime; 7 (2) the sworn
statements of two (2) of the former housemaids of the Webb

family in the persons of Nerissa E. Rosales and Mila S.


Gaviola; 8 (3) the sworn-statement of Carlos J. Cristobal
who alleged that on March 9, 1991 he was a passenger of
United Airlines Flight No. 808 bound for New York and who
expressed doubt on whether petitioner Webb was his copassenger in the trip; (4) the sworn statement of Lolita
Birrer, a former live-in partner of Gerardo Biong, who
narrated the manner of how Biong investigated and tried to
cover up the crime at bar; 9 (5) the sworn statements of Belen
Dometita and Teofilo Minoza, two of the Vizconde maids,
and the sworn statements of Normal White, a security guard
and Manciano Gatmaitan, an engineer. The autopsy reports
of the victims were also submitted and they showed that
Carmela had nine (9) stab wounds, Estrellita twelve (12) and
Jennifer nineteen (19). 10 The genital examination of Carmela
confirmed the presence of spermatozoa. 11
Before submitting his counter-affidavit, petitioner Webb
filed with the DOJ Panel a Motion for Production And
Examination of Evidence and Documents for the NBI to
produce the following:
(a) Certification issued by the U.S. Federal Bureau
of Investigation on the admission to and stay of
Hubert Webb in the United States from March 9,
1991 to October 22, 1992;
(b) Laboratory Report No. SN-91-17 of the Medico
Legal Officer, Dr. Prospero A. Cabanayan, M.D.;
(c) Sworn Statements of Gerardo C. Biong (other
than his Sworn Statement dated October 7, 1991);
(d) Photographs of fingerprints lifted from the
Vizconde residence taken during the investigation;
(e) Investigation records of NBI on Engr. Danilo
Aguas, et al.;

(f) List of names of 135 suspects/persons


investigated by the NBI per Progress Report dated
September 2, 1991 submitted by Atty. Arlis Vela,
Supervising Agent;
(g) Records of arrest, interview, investigation and
other written statements of Jessica Alfaro (other than
the May 22, 1995 Sworn Statement) conducted by
the NBI and other police agencies;
(h) transmittal letter to the NBI, including the report
of the investigation conducted by Superintendent
Rodolfo C. Sison, Regional Deputy Director, NCRC;
(i) The names of NBI officials/agents composing the
Task Force Jecares, including their respective
positions and duties;
(j) Statements made by other persons in connection
with the crime charged.
The motion was granted by the DOJ Panel and the NBI
submitted photocopies of the documents. It alleged it lost the
original of the April 28, 1995 sworn statement of Alfaro.
This compelled petitioner Webb to file Civil Case No.
951099 in the Regional Trial Court (RTC) of Makati, Br. 63,
for the purpose, among others, of obtaining the original of
said sworn statement. He succeeded, for in the course of its
proceedings, Atty. Arturo L. Mercader, Jr., produced a copy
of said original in compliance with a subpoena duces tecum.
The original was then submitted by petitioner Webb to the
DOJ Panel together with his other evidence. It appears,
however, that petitioner Webb failed to obtain from the NBI
the copy of the Federal Bureau of Investigation (FBI) Report
despite his request for its production.
Petitioner Webb claimed during the preliminary investigation
that he did not commit the crime at bar as he went to the
United States on March 1, 1991 and returned to the
Philippines on October 27, 1992. 12 His alibi was

corroborated by Honesto Aragon, Lecinia Edrosolano, Sylvia


Climaco, Gina Roque, Sonia Rodriguez, Edgardo Venture
and Pamela Francisco. 13 To further support his defense, he
submitted documentary evidence that he bought a bicycle
and a 1986 Toyota car while in the United States on said
dates 14 and that he was issued by the State of California
Driver's License No. A8818707 on June 14, 1991. 15
Petitioner Webb likewise submitted the letter dated July 25,
1995 of Mr. Robert Heafner, Legal Attache of the US
Embassy, citing certain records tending to confirm, among
others, his arrival at San Francisco, California on March 9,
1991 as a passenger in United Airlines Flight No. 808.

voluntarily inhibited himself from the case to avoid any


suspicion about his impartiality considering his employment
with the NBI before his appointment to the bench. The case
was re-raffled to Branch 274, presided by Judge Amelita
Tolentino who issued new warrants of arrest against the
petitioners and their co-accused. On August 11, 1995,
petitioner Webb voluntarily surrendered to the police
authorities at Camp Ricardo Papa Sr., in Bicutan, Taguig.
Petitioners Gatchalian and Lejano likewise gave themselves
up to the authorities after filing their petitions before us.

In their petitions at bar, petitioners contend: (1) respondent


Judges de Leon and Tolentino gravely abused their discretion
The other respondents Hospicio "Pyke" Fernandez,
when they failed to conduct a preliminary examination
Michael Gatchalian, Antonio "Tony Boy" Lejano, Peter
before issuing warrants of arrest against them: (2) the DOJ
Estrada, Miguel Rodriguez and Gerardo Biong submitted Panel likewise gravely abused its discretion in holding that
sworn statements, responses, and a motion to dismiss
there is probable cause to charge them with the crime of rape
denying their complicity in the rape-killing of the Vizcondes. with homicide; (3) the DOJ Panel denied them their
16
Only the respondents Joey Filart and Artemio "Dong"
constitutional right to due process during their preliminary
Ventura failed to file their counter-affidavits though they
investigation; and (4) the DOJ Panel unlawfully intruded
17
were served with subpoena in their last known address. In into judicial prerogative when it failed to charge Jessica
his sworn statement, petitioner Gatchalian alleged that from Alfaro in the Information as an accused.
11 o'clock in the evening of June 29, 1991 until 3 o'clock in
the morning of the following day, he was at the residence of We find the petitions bereft of merit.
his friends, Carlos and Andrew Syyap, at New Alabang
I
Village, Muntinlupa watching video tapes. He claimed that
his co-petitioner Lejano was with him.
Petitioners fault the DOJ Panel for its finding of
probable cause. They insist that the May 22, 1995
On August 8, 1995, the DOJ Panel issued a 26-page
sworn statement of Jessica Alfaro is inherently weak
Resolution "finding probable cause to hold respondents for
and uncorroborated. They hammer on alleged
trial" and recommending that an Information for rape with
material inconsistencies between her April 28, 1995
homicide be filed against petitioners and their co18
and May 22, 1995 sworn statements. They assail her
respondents, On the same date, it filed the corresponding
19
credibility for her misdescription of petitioner
Information against petitioners and their co-accused with
Webb's hair as semi-blonde. They also criticize the
the Regional Trial Court of Paraaque. The case was
procedure followed by the DOJ Panel when it did
docketed as Criminal Case No. 95-404 and raffled to Branch
not examine witnesses to clarify the alleged
258 presided by respondent judge Zosimo V. Escano. It was,
incredulities and inconsistencies in the sworn
however, the respondent judge Raul de Leon, pairing judge
statements of the witnesses for the NBI.
of Judge Escano, who issued the warrants of arrest against
the petitioners. On August 11, 1995, Judge Escano

We start with a restatement of the purpose of a


preliminary investigation. Section 1 of Rule 112
provides that a preliminary investigation should
determine " . . . whether there is a sufficient ground
to engender a well-grounded belief that a crime
cognizable by the Regional Trial Court has been
committed and that the respondent is probably guilty
thereof, and should be held for trial." Section 3 of
the same Rule outlines the procedure in conducting a
preliminary investigation, thus:
Sec. 3. Procedure. Except as provided for
in Section 7 hereof, no complaint or
information for an offense cognizable by the
Regional Trial Court shall be filed without a
preliminary investigation having been first
conducted in the following manner:
(a) The complaint shall state the known
address of the respondent and be
accompanied by affidavits of the
complainant and his witnesses as well as
other supporting documents, in such number
of copies as there are respondents, plus two
(2) copies for the official file. The said
affidavits shall be sworn to before any fiscal,
state prosecutor or government official
authorized to administer oath, or, in their
absence or unavailability, a notary public,
who must certify that he personally
examined the affiants and that he is satisfied
that they voluntarily executed and
understood their affidavits.
(b) Within ten (10) days after the filing of
the complaint, the investigating officer shall
either dismiss the same if he finds no ground
to continue with the inquiry, or issue a
subpoena to the respondent, attaching
thereto a copy of the complaint, affidavits

and other supporting documents. Within ten


(10) days from receipt thereof, the
respondent shall submit counter-affidavits
and other supporting documents. He shall
have the right to examine all other evidence
submitted by the complainant.
(c) Such counter-affidavits and other
supporting evidence submitted by the
respondent shall also be sworn to and
certified as prescribed in paragraph (a)
hereof and copies thereof shall be furnished
by him to the complainant.

Section 4 of Rule 112 then directs that "if the


investigating fiscal finds cause to hold the
respondent for trial, he shall prepare the resolution
and corresponding information. He shall certify
under oath that he, or as shown by the record, an
authorized officer, has personally examined the
complainant and his witnesses, that there is
reasonable ground to believe that a crime has been
committed and that the accused is probably guilty
thereof . . ."

The need to find probable cause is dictated by the Bill of


Rights which protects "the right of the people to be secure in
their persons . . . against unreasonable searches and seizures
(d) If the respondent cannot be subpoenaed, of whatever nature . . ." 20 An arrest without a probable cause
or if subpoenaed, does not submit counteris an unreasonable seizure of a person, and violates the
affidavits within the ten (10) day period, the privacy of persons which ought not to be intruded by the
investigating officer shall base his resolution State. 21 Probable cause to warrant arrest is not an opaque
on the evidence presented by the
concept in our jurisdiction. Continuing accretions of case
complainant.
law reiterate that they are facts and circumstances which
would lead a reasonably discreet and prudent man to believe
(e) If the investigating officer believes that
that an offense has been committed by the person sought to
there are matters to be clarified, he may set a be arrested. 22 Other jurisdictions utilize the term man of
hearing to propound clarificatory questions reasonable caution 23 or the term ordinarily prudent and
to the parties or their witnesses, during
cautious man. 24 The terms are legally synonymous and their
which the parties shall be afforded an
reference is not to a person with training in the law such as a
opportunity to be present but without the
prosecutor or a judge but to the average man on the street. 25
right to examine or cross-examine. If the
It ought to be emphasized that in determining probable
parties so desire, they may submit questions cause, the average man weighs facts and circumstances
to the investigating officer which the latter
without resorting to the calibrations of our technical rules of
may propound to the parties or witnesses
evidence of which his knowledge is nil. Rather, he relies on
concerned.
the calculus of common sense of which all reasonable men
have an abundance.
(f) Thereafter, the investigation shall be
deemed concluded, and the investigating
Applying these basic norms, we are not prepared to
officer shall resolve the case within ten (10)
rule that the DOJ Panel gravely abused its discretion
days therefrom. Upon the evidence thus
when it found probable cause against the petitioners.
adduced, the investigating officer shall
Petitioners belittle the truthfulness of Alfaro on two
determine whether or not there is sufficient
(2) grounds: (a) she allegedly erroneously described
ground to hold the respondent for trial.
petitioner Webb's hair as semi-blond and (b) she

committed material inconsistencies in her two (2)


sworn statement, thus: 26
xxx xxx xxx
To illustrate, the following are some
examples of inconsistencies in the two
sworn statements of Alfaro:
On whether Alfaro knew Carmela before the
incident in question
First Affidavit: She had
NOT met Carmela before
June 29, 1991.
Second Affidavit: "I met her
in a party sometime in
February, 1991."
On whether Alfaro saw the dead bodies
First Affidavit: She did not
see the three dead persons
on that night. She just said
"on the following day I read
in the newspaper that there
were three persons who
were killed . . ."
Second Affidavit: "I peeped
through the first door on the
left. I saw two bodies on top
of the bed, bloodied, and in
the floor, I saw Hubert on
top of Carmela."
On the alleged rape of Carmela Vizconde

First Affidavit: She did not


see the act of rape.
Second Affidavit: She saw
Hubert Webb "with bare
buttocks, on top of Carmela
and pumping, her mouth
gagged and she was
moaning and I saw tears on
her eyes."
On how Webb, Lejano, and Ventura entered
the Vizconde house
First Affidavit: "by jumping
over the fence, which was
only a little more than a
meter high."
Second Affidavit: They
"entered the gate which was
already open."
On whether Alfaro entered the Vizconde
house
First Affidavit: She never
entered the house.
Second Affidavit: "I
proceeded to the iron grill
gate leading to the dirty
kitchen."
In its Resolution, the DOJ Panel ruled that these
alleged misdescription and inconsistencies did not
erode the credibility of Alfaro. We quote the
pertinent ruling, viz.: 27
xxx xxx xxx

As regards the admissibility of Alfaro's


statements, granting for purposes of
argument merely that she is a co-conspirator,
it is well to note that confessions of a coconspirator may be taken as evidence to
show the probability of the co-conspirator's
participation in the commission of the crime
(see People vs. Lumahang, 94 Phil. 1084).

On August 7, 1995, another counsel for


respondent Webb submitted his
memorandum suggesting that the instant
complaint "should not be decided within the
month to give time to the NBI to coordinate
with the FBI on the latter's inquiry into the
whereabouts of Hubert Webb . . . and to
check on our U.S.-based witnesses."

Furthermore, it is a well-established doctrine


that conspiracy need not be proved by direct
evidence of prior agreement to commit the
crime. Indeed, "only rarely would such a
prior agreement be demonstrable since, in
the nature of things, criminal undertakings
are only rarely documented by agreements
in writing. Thus, conspiracy may be inferred
from the conduct of the accused before,
during and after the commission of the
crime, showing that the several accused had
acted in concert or in unison with each other,
evincing a common purpose or design."
(Angelo vs. Court of Appeals, 210 SCRA
402 [1992], citations omitted; People vs.
Molleda, 86 SCRA 699).

In said memorandum, counsel for


respondent Webb calls for the application of
the maxim falsus in uno, falsus in omnibus
arising from the inconsistencies of Alfaro's
statements, among others. This is untenable.
As held in Angelo:

Neither can we discredit Alfaro merely


because of the inconsistencies in her two
sworn statements. In Angelo, the Court
refused to discredit the testimony of a
witness accusing therein petitioner for the
slaying of one Gaviano Samaniego even
though said witness failed to name Angelo
in his affidavit which was executed five (5)
months earlier. Granting, the Court
continued, that a part of the witness'
testimony is untrue, such circumstance is not
sufficient to discredit the entire testimony of
the witness.

There is no rule of law


which prohibits a court from
crediting part of the
testimony of a witness as
worthy of belief and from
simultaneously rejecting
other parts which the court
may find incredible or
dubious. The maxim falsus
in uno, falsus in omnibus is
not a rule of law, let alone a
general rule of law which is
universally applicable. It is
not a legal presumption
either. It is merely a latinism
describing the conclusion
reached by a court in a
particular case after
ascribing to the evidence
such weight or lack of
weight that the court
deemed proper.

In the case before us, complainant reasoned


out that Alfaro was then having reservations
when she first executed the first statement
and held back vital information due to her
natural reaction of mistrust. This being so,
the panel believes that the inconsistencies in
Alfaro's two sworn statements have been
sufficiently explained especially specially so
where there is no showing that the
inconsistencies were deliberately made to
distort the truth. Consequently, the probative
value of Alfaro's testimony deserves full
faith and credit. As it has been often noted,
ex parte statements are generally incomplete
because they are usually executed when the
affiant's state of mind does not give her
sufficient and fair opportunity to
comprehend the import of her statement and
to narrate in full the incidents which
transpired (People vs. Sarellana, 233 SCRA
31 [1994]; Angelo vs. Court of Appeals,
supra). In the case at bar, there is no dispute
that a crime has been committed and what is
clear before us is that the totality of the
evidence submitted by the complainant
indicate a prima facie case that respondents
conspired in the perpetration of the imputed
offense.
We note that the May 22, 1995 sworn statement of Alfaro
was given with the assistance of counsel 28 and consists of six
(6) pages, in single space reciting in rich details how the
crime was planned and then executed by the petitioners. In
addition, the DOJ Panel evaluated the supporting sworn
statements of Nerissa Rosales and Mila Gaviola, former
housemaids of the Webbs, Carlos J. Cristobal, a passenger in
United Airlines Flight No. 808 and Lolita Birrer, a paramour
of Gerardo Biong. The Panel assayed their statements as
follows: 29

xxx xxx xxx


According to Nerissa E. Rosales, a former
housemaid of the Webb family, on June 29,
1991, between 7:00 o'clock and 8:00 o'clock
in the evening, Hubert was at home inside
his room with two male visitors. She knew it
because she and her co-housemaid, Loany,
were instructed by Hubert to bring them
three glasses of juice. It was the last time she
saw Hubert and was later told by then
Congressman Webb that Hubert was in the
United States.
While Mila S. Gaviola, another former
housemaid of the Webb family and who
served as a laundry woman, claims, aside
from corroborating the statement of Nerissa
Rosales, that on June 30, 1991, she woke up
at around 4:00 in the morning and as what
she used to do, she entered the rooms of the
Webbs to get their clothes to be washed. As
a matter of fact, in that early morning, she
entered Hubert's room and saw Hubert, who
was only wearing his pants, already awake
and smoking while he was sitting on his bed.
She picked up Hubert's scattered clothes and
brought them together with the clothes of
the other members of the family to the
laundry area. After taking her breakfast, she
began washing the clothes of the Webbs. As
she was washing the clothes of Hubert
Webb, she noticed fresh bloodstains in his
shirt. After she finished the laundry, she
went to the servant's quarters. But feeling
uneasy, she decided to go up to the
stockroom near Hubert's room to see what
he was doing. In the said stockroom, there is
a small door going to Hubert's room and in
that door there is a small opening where she

used to see Hubert and his friends sniffing


on something. She observed Hubert was
quite irritated, uneasy, and walked to and
from inside his room.
On that day, she noticed Hubert left the
house at around 1:00 in the afternoon and
came back at around 4:00 in the same
afternoon and went inside his room using
the secret door of the house. It was the last
time that she saw Hubert until she left the
Webb family.
On the other hand, Carlos J. Cristobal
alleged that on March 9, 1991, at about
10:00 in the morning, he was at the Ninoy
Aquino International Airport as he was then
scheduled to take the United Airlines Flight
No. 808 at 2:00 in the afternoon for New
York. At the airport's lobby, he saw then
Congressman Freddie Webb with a male
companion. He greeted him and Webb
answered: "Mabuti naman, at ito, ihahatid
ko ang anak ko papuntang Florida." He
knew Freddie Webb because he often
watched him then in a television show
"Chicks to Chicks." He observed that the
man whom Freddie Webb referred to as his
son, was of the same height as Freddie. The
son referred to has fair complexion with no
distinguishing marks on his face. He (son of
Webb) was then wearing a striped white
jacket. When he and his children were
already inside the plane, he did not see
Freddie anymore, but he noticed his son was
seated at the front portion of the economy
class. He never noticed Freddie Webb's son
upon their arrival in San Francisco. He
claims that, while watching the television
program "DONG PUNO LIVE" lately, he

saw the wife of Freddie Webb with her


lawyer being interviewed, and when she
described Hubert as "moreno" and small
built, with a height of five feet and seven
inches tall, and who was the one who left for
United States on March 9, 1991, he nurtured
doubts because such description does not fit
the physical traits of the son of Freddie, who
left with him for United States on the same
flight and date.
Lolita Birrer, alleged that she know Gerardo
Biong because she had an affair with him for
almost three (3) years and in fact, she had a
child with him who is now four (4) years
old. Their relationship started in February,
1991 until she broke up with him in
September 1993. She recalls that on June 29,
1991, at around 6:00 p.m., Biong invited her
to play mahjong at the canteen of a certain
Aling Glo located at the back of the
Paraaque Municipal Hall.
At about 2:30, in the early morning of
January 30, 1991, the radio operator of the
Paraaque police told Biong that he has a
phone call. Before Biong went to the radio
room, she was instructed to take him over
and after somebody won the game, she
followed Biong at the radio room where she
overheard him uttering, "Ano?, Saan?
Mahirap yan, Paano, o sige, aantayin kita,
O ano?, dilaw na taxi, o sige." When he put
the phone down, Biong told her, "Mayroon
lang akong rerespondehan, ikaw muna ang
maupo" and then, he went outside the
canteen apparently waiting for somebody.
Twenty minutes later, a taxi, colored yellow,
arrived with a male passenger sitting at the
backseat and parked near the canteen. After

it made some signals by blinking its


headlight, Biong rode thereat at the front
seat beside the driver and then, they left. She
was not able to recognize the male
passenger because the window of the taxi
was tinted. Biong came back at around 7:00
of the same morning and when he arrived,
he immediately washed his hands and face,
and took his handkerchief from his pocket
which he threw at the trash can. She asked
him why he threw his handkerchief and he
answered, "Hmp . . . amoy tae." She
inquired what happened in BF Homes and
he replied, "Putang inang mga batang iyon,
pinahirapan nila ako."
Biong later invited her for breakfast, but
they first went to his office where she
observed him doing something in his steel
cabinet while he appeared to be uneasy.
Moments later, Galvan, another policeman
of Paraaque, arrived and said, "Oy Biong,
may tatlong patay sa BF, imbestigahan mo"
to which Biong answered, "Oo susunod na
ako." Biong went to the office of Capt. Don
Bartolome who offered to accompany him
and with whom she asked permission to go
with them. Before they proceeded to the
place where the killings happened, she asked
Biong if he knew the exact address and the
latter immediately responded, "Alam ko na
yon." She was surprised because Galvan
never told him the place of the incident.
As soon as they arrived at the Vizconde's
residence, Biong instructed the housemaids
to contact the victim's relatives, while the
security guard fetched the barangay
chairman and the president of the
Homeowners Association. When all these

persons were already in the house, Biong


started recording the wounds of the victim.
Inside the master's bedroom, she saw Biong
took a watch from the jewelry box. Because
she could not tolerate the foul odor, she and
Capt. Bartolome went out of the room and
proceeded to the dining area. On top of the
dining table, she saw the scattered contents
of a shoulder bag. Moments later, Biong
came out from the room and proceeded to
the front door to remove the chain lock;
asked the keys from the housemaid and it
was only then that the main door was
opened. Biong noticed a stone in front of the
broken glass of the door and requested Capt.
Bartolome to go inside the servant's quarters
as he doubted the housemaids' claim that
they heard nothing unusual. Using the
handle of his gun, Biong broke the
remaining glass of the door panel.
Bartolome then came out of the room and
told Biong that he can hear the sound of the
glass being broken. At the garage, Biong
also noticed same marks on the hood of the
car.
On the following day, at around 12:00 noon,
Biong arrived in her house together with the
Vizconde housemaids. When Biong was
preparing to take a bath, she saw him
remove from his pocket the things she also
saw from Vizconde's residence, to wit:
calling cards, driver's license, ATM card, a
crossed check worth P80,000.00, earrings, a
ring, bracelet, necklace, and the watch he
took from the jewelry box inside the room of
the Vizcondes. These jewelry items were
later pawned by Biong for P20,000.00 at a
pawnshop in front of Chow-Chow restaurant
in Santos Avenue, Paraaque. The next day,

she saw Biong took from his locker at the


Paraaque Police Station an imported brown
leather jacket, which the latter claimed to
have been given to him by the person who
called him up in the early morning of June
30, 1991.
Since then, Biong has been wearing said
jacket until they broke up sometime in 1993.
She observed that Biong seemed not
interested in pursuing the investigation of
the Vizconde case. In fact, when Biong and
this group picked up Mike Gatchalian and
brought him to the Paraaque Police Station,
she was surprised that Biong halted the
investigation when Gatchalian was profusely
sweating while being interrogated. After the
father of Gatchalian talked to Colonel
Pureza, the latter called up and instructed
Biong to bring Gatchalian to him (Colonel
Pureza) and that was the last thing she
remembered regarding this case.
The DOJ Panel then weighed these inculpatory evidence
against the exculpatory evidence of petitioners. It ruled: 30

relatives (People vs. Apolonia, 235 SCRA


124 [1994]; People vs. Lucas, 181 SCRA
316 and a long line of cases).
Similarly, denial is a self-serving negative
which cannot be given greater evidentiary
weight than the declaration of a credible
witness who testified on affirmative matters
(People vs. Carizo, 233 SCRA 687 [1994]).
Indeed, denial, like alibi, is weak and
becomes even more weaker when arrayed
against the positive identification by the
witness for the prosecution (People vs.
Onpaid, 233 SCRA 62 [1994]).
Surprisingly, Gatchalian's defense of alibi
was not corroborated by Lejano, whom he
claimed was with him watching video tapes
at the Syyap residence. Other than claiming
that he "was not and could not have been at
or near the area of the Vizconde residence at
the time of the alleged commission of the
crime," respondent Lejano proffered no
evidence to substantiate his claim of alibi.
xxx xxx xxx

xxx xxx xxx


The voluminous number of exhibits
submitted by respondent Webb to support his
defense of denial and alibi notwithstanding,
the panel, after a careful and thorough
evaluation of the records, believes that they
cannot outweigh the evidence submitted by
the complainant. Alibi cannot prevail over
the positive identification made by a
prosecution witness. Verily, alibi deserves
scant consideration in the face of positive
identification especially so where the claim
of alibi is supported mainly by friends and

On the other hand, respondent Webb seeks


to enhance the acceptability of his alibi in
the form of documents tending to show that
he was thousands of miles away when the
incident occurred. We have carefully
deliberated and argued on the evidence
submitted by respondent Webb in support of
his absence from the country since March 9,
1991 to October 26, 1992 and found the
same wanting to exonerate him of the
offense charged. The material dates in this
case are June 29 and 30, 1991. While
respondent Webb may have submitted proof

tending to show that he was issued a


California driver's license on June 14, 1991,
there is no showing that he could not have
been in the country on the dates above
mentioned. Neither do we find merit in the
allegation that respondent Webb personally
bought a bicycle on June 30, 1991 in
California in view of his positive
identification by Alfaro and the two (2)
househelps of the Webb family who testified
that he was here in the country on said dates.
Additionally, the issuance of receipt
evidencing the purchase of a bicycle in
California is no conclusive proof that the
name appearing thereon was the actual
buyer of the merchandise.
Given these conflicting pieces of evidence of the
NBI and the petitioners, we hold that the DOJ Panel
did not gravely abuse its discretion when it found
probable cause against the petitioners. A finding of
probable cause needs only to rest on evidence
showing that more likely than not a crime has been
committed and was committed by the suspects.
Probable cause need not be based on clear and
convincing evidence of guilt, neither on evidence
establishing guilt beyond reasonable doubt and
definitely, not on evidence establishing absolute
certainty of guilt. As well put in Brinegar v. United
States, 31 while probable cause demands more than
"bare suspicion," it requires "less than evidence
which would justify . . . conviction." A finding of
probable cause merely binds over the suspect to
stand trial. It is not a pronouncement of guilt.
Considering the low quantum and quality of
evidence needed to support a finding of probable
cause, we also hold that the DOJ Panel did not,
gravely abuse its discretion in refusing to call the
NBI witnesses for clarificatory questions. The

decision to call witnesses for clarificatory questions


is addressed to the sound discretion of the
investigator and the investigator alone. If the
evidence on hand already yields a probable cause,
the investigator need not hold a clarificatory hearing.
To repeat, probable cause merely implies probability
of guilt and should be determined in a summary
manner. Preliminary investigation is not a part of
trial and it is only in a trial where an accused can
demand the full exercise of his rights, such as the
right to confront and cross-examine his accusers to
establish his innocence. In the case at bar, the DOJ
Panel correctly adjudged that enough evidence had
been adduced to establish probable cause and
clarificatory hearing was unnecessary.
II
We now come to the charge of petitioners that
respondent Judge Raul de Leon and, later,
respondent Judge Amelita Tolentino issued warrants
of arrest against them without conducting the
required preliminary examination. Petitioners
support their stance by highlighting the following
facts: (1) the issuance of warrants of arrest in a
matter of few hours; (2) the failure of said judges to
issue orders of arrest; (3) the records submitted to
the trial court were incomplete and insufficient from
which to base a finding of probable cause; and (4)
that even Gerardo Biong who was included in the
Information as a mere accessory had a "NO BAIL"
recommendation by the DOJ Panel. Petitioners
postulate that it was impossible to conduct a
"searching examination of witnesses and evaluation
of the documents" on the part of said judges.
The issuance of a warrant of arrest interferes with
individual liberty and is regulated by no less than the
fundamental law of the land. Section 2 of Article III
of the Constitution provides:

Sec. 2. The right of the people to be secure


in their persons, houses, papers, and effects
against unreasonable searches and seizures
of whatever nature and for any purpose shall
be inviolable, and no search warrant or
warrant of arrest shall issue except upon
probable cause to be determined personally
by the judge after examination under oath or
affirmation of the complainant and the
witnesses he may produce and particularly
describing the place to be searched and the
persons or things to be seized.
The aforequoted provision deals with the
requirements of probable cause both with respect to
issuance of warrants of arrest or search warrants.
The similarities and differences of their requirements
ought to be educational. Some of them are pointed
out by Professors LaFave and Israel, thus: 32 "It is
generally assumed that the same quantum of
evidence is required whether one is concerned with
probable cause to arrest or probable cause to search.
But each requires a showing of probabilities as to
somewhat different facts and circumstances, and
thus one can exist without the other. In search cases,
two conclusions must be supported by substantial
evidence: that the items sought are in fact seizable
by virtue of being connected with criminal activity,
and that the items will be found in the place to be
searched. It is not also necessary that a particular
person be implicated. By comparison, in arrest cases
there must be probable cause that a crime has been
committed and that the person to be arrested
committed it, which of course can exist without any
showing that evidence of the crime will be found at
premises under that person's control." Worthy to
note, our Rules of Court do not provide for a similar
procedure to be followed in the issuance of warrants
of arrest and search warrants. With respect to
warrants of arrest, section 6 of Rule 112 simply

provides that "upon filing of an information, the


Regional Trial Court may issue a warrant for the
arrest of the accused." In contrast, the procedure to
be followed in issuing search warrants is more
defined. Thus, Sections 3, 4 and 5 of Rule 126
provide:
xxx xxx xxx
Sec. 3. Requisites for issuing search
warrant. A search warrant shall not issue
but upon probable cause in connection with
one specific offense to be determined
personally by the judge after examination
under oath or affirmation of the complainant
and the witnesses he may produce, and
particularly describing the place to be
searched and the things to be seized.
Sec. 4. Examination of complainant; record.
The judge must, before issuing the
warrant, personally examine in the form of
searching questions and answers, in writing
and under oath the complainant and any
witnesses he may produce on facts
personally known to them and attach to the
record their sworn statements together with
any affidavits submitted.
Sec. 5. Issuance and form of search warrant.
If the judge is thereupon satisfied of the
facts upon which the application is based, or
that there is probable cause to believe that
they exist, he must issue the warrant, which
must be substantially in the form prescribed
by these Rules.
We discussed the difference in the Procedure of
issuing warrants of arrest and search warrants in
Soliven vs. Makasiar, 33 thus:

xxx xxx xxx


The second issue, raised by Beltran, calls for
an interpretation of the constitutional
provision on the issuance of warrants of
arrest. The pertinent provision reads:
Art. III, Sec. 2. The right of
the people to be secure in
their persons, houses,
papers and effects against
unreasonable searches and
seizures of whatever nature
and for any purpose shall be
inviolable, and no search
warrant or warrant of arrest
shall issue except upon
probable cause to be
determined personally by
the judge after examination
under oath or affirmation of
the complainant and the
witnesses he may produce,
and particularly describing
the place to be searched and
the persons or things to be
seized.
The addition of the word "personally" after
the word "determined" and the deletion of
the grant of authority by the 1973
Constitution to issue warrants to "other
responsible officers as may be authorized by
law," has apparently convinced petitioner
Beltran that the Constitution now requires
the judge to personally examine the
complainant and his witnesses in his
determination of probable cause for the
issuance of warrants of arrest. This is not an
accurate interpretation.

What the Constitution underscores is the


exclusive and personal responsibility of the
issuing judge to satisfy himself of the
existence of probable cause. In satisfying
himself of the existence of probable cause
for the issuance of a warrant of arrest, the
judge is not required to personally examine
the complainant and his witnesses.
Following established doctrine and
procedure, he shall: (1) personally evaluate
the report and the documents submitted by
the fiscal regarding the existence of probable
cause and, on the basis thereof, issue a
warrant; or (2) if on the basis thereof he
finds no probable cause, he may disregard
the fiscal's report and require the submission
of supporting affidavits of witnesses to aid
him in arriving at a conclusions as to the
existence of probable cause.
Sound policy dictates this procedure,
otherwise judges would be unduly laden
with the preliminary examination and
investigation of criminal complaints instead
of concentrating on hearing and deciding
cases filed before their courts.
Clearly then, the Constitution, the Rules of Court,
and our case law 34 repudiate the submission of
petitioners that respondent judges should have
conducted "searching examination of witnesses"
before issuing warrants of arrest against them. They
also reject petitioners' contention that a judge must
first issue an order of arrest before issuing a warrant
of arrest. There is no law or rule requiring the
issuance of an Order of Arrest prior to a warrant of
arrest.
In the case at bar, the DOJ Panel submitted to the
trial court its 26-page report, the two (2) sworn

statements of Alfaro and the sworn statements of


Carlos Cristobal and Lolita Birrer 35 as well as the
counter-affidavits of the petitioners. Apparently, the
painstaking recital and analysis of the parties'
evidence made in the DOJ Panel Report satisfied
both judges that there is probable cause to issue
warrants of arrest against petitioners. Again, we
stress that before issuing warrants of arrest, judges
merely determine personally the probability, not the
certainty of guilt of an accused. In doing so, judges
do not conduct a de novo hearing to determine the
existence of probable cause. They just personally
review the initial determination of the prosecutor
finding a probable cause to see if it is supported by
substantial evidence. The sufficiency of the review
process cannot be measured by merely counting
minutes and hours. The fact that it took the
respondent judges a few hours to review and affirm
the probable cause determination of the DOJ Panel
does not mean they made no personal evaluation of
the evidence attached to the records of the case. 36
Petitioners' reliance on the case of Allado vs. Diokno
37
is misplaced. Our Allado ruling is predicated on
the utter failure of the evidence to show the
existence of probable cause. Not even the corpus
delicti of the crime was established by the evidence
of the prosecution in that case. Given the clear
insufficiency of the evidence on record, we stressed
the necessity for the trial judge to make a further
personal examination of the complainant and his
witnesses to reach a correct assessment of the
existence or non-existence of probable cause before
issuing warrants of arrest against the accused. The
case at bar, however, rests on a different factual
setting. As priorly discussed, the various types of
evidence extant in the records of the case provide
substantial basis for a finding of probable cause
against the petitioner. The corpus delicti of the crime
is a given fact. There is an eyewitness account of the

imputed crime given by Alfaro. The alibi defense of


petitioner Webb is also disputed by sworn statements
of their former maids. It was therefore unnecessary
for the respondent judges to take the further step of
examining ex parte the complainant and their
witnesses with searching questions.
III
Petitioners also complain about the denial of their
constitutional right to due process and violation of
their right to an impartial investigation. They decry
their alleged hasty and malicious prosecution by the
NBI and the DOJ Panel. They also assail the
prejudicial publicity that attended their preliminary
investigation.
We reject these contentions. The records will show
that the DOJ Panel did not conduct the preliminary
investigation with indecent haste. Petitioners were
given fair opportunity to prove lack of probable
cause against them. The fairness of this opportunity
is well stressed in the Consolidated Comment of the
Solicitor General, viz.:
Again, there is no merit in this contention.
Petitioners were afforded all the
opportunities to be heard. Petitioner Webb
actively participated in the preliminary
investigation by appearing in the initial
hearing held on June 30, 1995 and in the
second hearing on July 14, 1995; and by
filing a "Motion for Production and
Examination of Evidence and Documents"
on June 27, 1995 (p. 4, Petition), a "Reply to
the compliance and Comment/Manifestation
to the Motion for Production and
Examination of Evidence" on July 5, 1995
(p. 6, Petition), a "Comment and
Manifestation" on July 7, 1995 (p. 6,

Petition), his "Counter-Affidavit" on July 14,


1995 (pp. 6-7, Petition) and a "Motion to
Resolve" on August 1, 1995. Numerous
letter-requests were also sent by the
petitioner Webb's counsel to the DOJ Panel
requesting the latter to furnish him a copy of
the reports prepared by the FBI concerning
the petitioner's whereabouts during the
material period (Annexes "L", "L-1" and "L2" of the Supplemental Petition dated
August 14, 1995). In fact, not satisfied with
the decision of the DOJ Panel not to issue
subpoena duces tecum to Atty. Arturo L.
Mercader, Jr., petitioner Webb filed a
"Petition for Injunction, Certiorari,
Prohibition and Mandamus" with the
Regional Trial Court, Branch 63 of Makati
in order to compel said Atty. Mercader, Jr. to
produce the first sworn statement of Alfaro
for submission to the DOJ Panel. (p. 4,
Petition) The said court dismissed the
petition after Mercader produced and
submitted to the DOJ Panel the first sworn
statement of Alfaro, without ruling on the
admissibility and credence of the two (2)
conflicting and inconsistent sworn
statements of the principal witness, Alfaro.
(Attached hereto is a copy of the order of
Judge Ruben A. Mendiola, RTC-Makati,
Branch 63 dated July 28, 1995) marked as
Annex "F."
It must also be pointed out that despite the
declaration by the DOJ Panel that the
preliminary investigation was to be
terminated after the hearing held on July 14,
1995, the panel continued to conduct further
proceedings, e.g. comparison of the photocopies of the submitted documents with the
originals on July 17, 1995. (p. 7, Petition)

The panel even entertained the "Response"


submitted by accused Miguel Rodriguez on
July 18, 1995. (p. 17 Resolution) In addition
to these, the panel even announced that any
party may submit additional evidence before
the resolution of the case. (p. 8, Petition)
From the time the panel declared the
termination of the preliminary investigation
on July 14, 1995, twenty-seven (27) days
elapsed before the resolution was
promulgated, and the information eventually
filed in the Regional Trial Court of
Paraaque on August 10, 1995. This
notwithstanding the directive of Section 3(f)
Rule 112 of the Revised Rules of Court that
the investigating officer shall resolve the
case within ten (10) days from the
termination of the preliminary investigation.
The DOJ Panel precisely allowed the parties
to adduce more evidence in their behalf and
for the panel to study the evidence submitted
more fully. This directly disputes the
allegation of the petitioners that the
resolution was done with indecent haste in
violation of the rights of the petitioners.
During the period of twenty-seven (27) days,
the petitioners were free to adduce and
present additional evidence before the DOJ
Panel.
Verily, petitioners cannot now assert that
they were denied due process during the
conduct of the preliminary investigation
simply because the DOJ Panel promulgated
the adverse resolution and filed the
Information in court against them.
Petitioners cannot also assail as premature the filing
of the Information in court against them for rape
with homicide on the ground that they still have the

right to appeal the adverse resolution of the DOJ


Panel to the Secretary of Justice. The filing of said
Information is in accord with Department of Justice
Order No. 223, series of 1993, dated June 25, 1993.
We quote its pertinent sections, viz.:
Sec. 4. Non-Appealable Cases; Exceptions.
No appeal may be taken from a
resolution of the Chief State
Prosecutor/Regional State
Prosecutor/Provincial or City Prosecutor
finding probable cause except upon showing
of manifest error or grave abuse of
discretion. Notwithstanding the showing of
manifest error or grave abuse of discretion,
no appeal shall be entertained where the
appellant had already been arraigned. If the
appellant is arraigned during the pendency
of the appeal, said appeal shall be dismissed
motu propio by the Secretary of Justice.
An appeal/motion for reinvestigation from a
resolution finding probable cause, however,
shall not hold the filing of the information in
court.
Sec. 2. When to appeal. The appeal must
be filed within a period of fifteen (15) days
from receipt of the questioned resolution by
the party or his counsel. The period shall be
interrupted only by the filing of a motion for
reconsideration within ten (10) days from
receipt of the resolution and shall continue
to run from the time the resolution denying
the motion shall have been received by the
movant or his counsel. (Emphasis supplied)
Without doubt then, the said DOJ Order No. 223
allows the filing of an Information in court after the
consummation of the preliminary investigation even

if the accused can still exercise the right to seek a


review of the prosecutor's recommendation with the
Secretary of Justice.
Next, petitioners fault the DOJ Panel for not
including Alfaro in the Information considering her
alleged conspiratorial participation in the crime of
rape with homicide. The non-inclusion of Alfaro is
anchored on Republic Act
No. 6981, entitled "An Act Providing For A Witness
Protection, Security And Benefit Program And For
Other Purposes" enacted on April 24, 1991. Alfaro
qualified under its Section 10, which provides:

(f) he has not at anytime been convicted of


any crime involving moral turpitude.
An accused discharged from an information
or criminal complaint by the court in order
that he may be a State Witness pursuant to
Sections 9 and 10 of Rule 119 of the
Revised Rules of Court may upon his
petition be admitted to the Program if he
complies with the other requirements of this
Act. Nothing in this Act shall prevent the
discharge of an accused so that he can be
used as a Witness under Rule 119 of the
Revised Rules of Court.

xxx xxx xxx


Sec. 10. State Witness. Any person who
has participated in the commission of a
crime and desires to a witness for the State,
can apply and, if qualified as determined in
this Act and by the Department, shall be
admitted into the Program whenever the
following circumstances are present:
(a) the offense in which his testimony will
be used is a grave felony as defined under
the R.P.C. or its equivalent under special
laws;
(b) there is absolute necessity for his
testimony;
(c) there is no other direct evidence
available for the proper prosecution of the
offense committed;
(d) his testimony can be substantially
corroborated on its material points;
(e) he does not appear to be most guilty; and

Upon qualification of Alfaro to the program, Section


12 of the said law mandates her non-inclusion in the
criminal Complaint or Information, thus:
xxx xxx xxx
Sec. 12. Effect of Admission of a State
Witness into the Program. The
certification of admission into the Program
by the Department shall be given full faith
and credit by the provincial or city
prosecutor who is required NOT TO
INCLUDE THE WITNESS IN THE
CRIMINAL COMPLAINT OR
INFORMATION and if included therein, to
petition the court for his discharge in order
that he can be utilized as a State Witness.
The court shall order the discharge and
exclusion of the said accused from the
information.
Admission into the Program shall entitle
such State Witness to immunity from
criminal prosecution for the offense or
offenses in which his testimony will be

given or used and all the rights and benefits


provided under Section 8 hereof.
The validity of these provisions is challenged by
petitioner Webb. It is urged that they constitute ". . .
an intrusion into judicial prerogative for it is only the
court which has the power under the Rules on
Criminal Procedure to discharge an accused as a
state witness." The argument is based on Section 9,
Rule 119 38 which gives the court the prerogative to
approve the discharge of an accused to be a state
witness. Petitioner's argument lacks appeal for it lies
on the faulty assumption that the decision whom to
prosecute is a judicial function, the sole prerogative
of courts and beyond executive and legislative
interference. In truth, the prosecution of crimes
appertains to the executive department of
government whose principal power and
responsibility is to see that our laws are faithfully
executed. A necessary component of this power to
execute our laws is the right to prosecute their
violators. The right to prosecute vests the prosecutor
with a wide range of discretion the discretion of
whether, what and whom to charge, the exercise of
which depends on a smorgasbord of factors which
are best appreciated by prosecutors. We thus hold
that it is not constitutionally impermissible for
Congress to enact R.A. No. 6981 vesting in the
Department of Justice the power to determine who
can qualify as a witness in the program and who
shall be granted immunity from prosecution. 39
Section 9 of Rule 119 does not support the
proposition that the power to choose who shall be a
state witness is an inherent judicial prerogative.
Under this provision, the court, is given the power to
discharge a state witness only because it has already
acquired jurisdiction over the crime and the accused.
The discharge of an accused is part of the exercise of
jurisdiction but is not a recognition of an inherent
judicial function. Moreover, the Rules of Court have

never been interpreted to be beyond change by


legislation designed to improve the administration of
our justice system. R.A. No. 6981 is one of the much
sought penal reform laws to help government in its
uphill fight against crime, one certain cause of which
is the reticence of witnesses to testify. The rationale
for the law is well put by the Department of Justice,
viz.: "Witnesses, for fear of reprisal and economic
dislocation, usually refuse to appear and testify in
the investigation/prosecution of criminal
complaints/cases. Because of such refusal, criminal
complaints/cases have been dismissed for
insufficiency and/or lack of evidence. For a more
effective administration of criminal justice, there
was a necessity to pass a law protecting witnesses
and granting them certain rights and benefits to
ensure their appearance in investigative
bodies/courts." 40 Petitioner Webb's challenge to the
validity of R.A. No. 6981 cannot therefore succeed.
Further, petitioners charge the NBI with violating
their right to discovery proceedings during their
preliminary investigation by suppressing the April
28, 1995 original copy of the sworn statement of
Alfaro and the FBI Report. The argument is novel in
this jurisdiction and as it urges an expansive reading
of the rights of persons under preliminary
investigation it deserves serious consideration. To
start with, our Rules on Criminal Procedure do not
expressly provide for discovery proceedings during
the preliminary investigation stage of a criminal
proceeding. 41 Sections 10 and 11 of Rule 117 do
provide an accused the right to move for a bill of
particulars and for production or inspection of
material evidence in possession of the prosecution. 42
But these provisions apply after the filing of the
Complaint or Information in court and the rights are
accorded to the accused to assist them to make an
intelligent plea at arraignment and to prepare for
trial. 43

This failure to provide discovery procedure during


preliminary investigation does not, however, negate
its use by a person under investigation when
indispensable to protect his constitutional right to
life, liberty and property. Preliminary investigation is
not too early a stage to guard against any significant
erosion of the constitutional right to due process of a
potential accused. As aforediscussed, the object of a
preliminary investigation is to determine the
probability that the suspect committed a crime. We
hold that the finding of a probable cause by itself
subjects the suspect's life, liberty and property to real
risk of loss or diminution. In the case at bar, the risk
to the liberty of petitioners cannot be understated for
they are charged with the crime of rape with
homicide, a non-bailable offense when the evidence
of guilt is strong.
Attuned to the times, our Rules have discarded the
pure inquisitorial system of preliminary
investigation. Instead, Rule 112 installed a quasijudicial type of preliminary investigation conducted
by one whose high duty is to be fair and impartial. 44
As this Court emphasized in Rolito Go vs. Court of
Appeals, 45 "the right to have a preliminary
investigation conducted before being bound over for
trial for a criminal offense, and hence formally at
risk of incarceration or some other penalty, is not a
mere formal or technical right; it is a substantive
right." A preliminary investigation should therefore
be scrupulously conducted so that the constitutional
right to liberty of a potential accused can be
protected from any material damage. We uphold the
legal basis of the right of petitioners to demand from
their prosecutor, the NBI, the original copy of the
April 28, 1995 sworn statement of Alfaro and the
FBI Report during their preliminary investigation
considering their exculpatory character, and hence,
unquestionable materiality to the issue of their
probable guilt. The right is rooted on the

constitutional protection of due process which we


rule to be operational even during the preliminary
investigation of a potential accused. It is also
implicit in section (3) (a) of Rule 112 which requires
during the preliminary investigation the filing of a
sworn complaint, which shall ". . . state the known
address of the respondent and be accompanied by
affidavits of the complainant and his witnesses as
well as other supporting documents . . ."
In laying down this rule, the Court is not without
enlightened precedents from other jurisdictions. In
the 1963 watershed case of Brady v. Maryland 46
the United States Supreme Court held that
"suppression of evidence favorable to an accused
upon request violates due process where the
evidence is material to guilt or punishment,
irrespective of the good faith or bad faith of the
prosecution." Its progeny is the 1935 case of
Mooney v. Holohan 47 which laid down the
proposition that a prosecutor's intentional use of
perjured testimony to procure conviction violates
due process. Thus, evolved jurisprudence firming up
the prosecutor's duty to disclose to the defense
exculpatory evidence in its possession. 48 The
rationale is well put by Justice Brennan in Brady 49
"society wins not only when the guilty are
convicted but when criminal trials are fair." Indeed,
prosecutors should not treat litigation like a game of
poker where surprises can be sprung and where gain
by guile is not punished.
But given the right of petitioners to compel the NBI
to disclose exculpatory evidence in their favor, we
are not prepared to rule that the initial nonproduction of the original sworn statement of Alfaro
dated April 28, 1995 could have resulted in the
reasonable likelihood that the DOJ Panel would not
have found probable cause. To be sure, the NBI, on
July 4, 1995, upon request of petitioners, submitted a

photocopy of Alfaro's April 28, 1995 sworn


statement. It explained it cannot produce the original
as it had been lost. Fortunately, petitioners, on July
28, 1995, were able to obtain a copy of the original
from Atty. Arturo Mercader in the course of the
proceedings in Civil Case No. 951099. 50 As
petitioners admit, the DOJ Panel accepted the
original of Alfaro's April 28, 1995 sworn statement
as a part of their evidence. 51 Petitioners thus had the
fair chance to explain to the DOJ Panel then still
conducting their preliminary investigation the
exculpatory aspects of this sworn statement.
Unfortunately for petitioners, the DOJ Panel still
found probable cause to charge them despite the
alleged material discrepancies between the first and
second sworn statements of Alfaro. For reasons we
have expounded, this finding of probable cause
cannot be struck down as done with grave abuse of
discretion. 52 On the other hand, the FBI Report
while corroborative of the alibi of petitioner Webb
cannot by itself reverse the probable cause finding of
the DOJ Panel in light of the totality of evidence
presented by the NBI.
Finally, we come to the argument of petitioner that
the DOJ Panel lost its impartiality due to the
prejudicial publicity waged in the press and
broadcast media by the NBI.
Again, petitioners raise the effect of prejudicial
publicity on their right to due process while
undergoing preliminary investigation. We find no
procedural impediment to its early invocation
considering the substantial risk to their liberty while
undergoing a preliminary investigation.
In floating this issue, petitioners touch on some of
the most problematic areas in constitutional law
where the conflicting demands of freedom of speech
and of the press, the public's right to information,

and an accused's right to a fair and impartial trial


collide and compete for prioritization. The process
of pinpointing where the balance should be struck
has divided men of learning as the balance keeps
moving either on the side of liberty or on the side of
order as the tumult of the time and the welfare of the
people dictate. The dance of balance is a difficult act
to follow.
In democratic settings, media coverage of trials of
sensational cases cannot be avoided and oftentimes,
its excessiveness has been aggravated by kinetic
developments in the telecommunications industry.
For sure, few cases can match the high volume and
high velocity of publicity that attended the
preliminary investigation of the case at bar. Our
daily diet of facts and fiction about the case
continues unabated even today. Commentators still
bombard the public with views not too many of
which are sober and sublime. Indeed, even the
principal actors in the case the NBI, the
respondents, their lawyers and their sympathizers
have participated in this media blitz. The possibility
of media abuses and their threat to a fair trial
notwithstanding, criminal trials cannot be
completely closed to the press and the public. In the
seminal case of Richmond Newspapers, Inc. v.
Virginia, 53 it was wisely held:
xxx xxx xxx
(a) The historical evidence of the evolution
of the criminal trial in Anglo-American
justice demonstrates conclusively that at the
time this Nation's organic laws were
adopted, criminal trials both here and in
England had long been presumptively open,
thus giving assurance that the proceedings
were conducted fairly to all concerned and
discouraging perjury, the misconduct of

participants, or decisions based on secret


bias or partiality. In addition, the significant
community therapeutic value of public trials
was recognized: when a shocking crime
occurs, a community reaction of outrage and
public protest often follows, and thereafter
the open processes of justice serve an
important prophylactic purpose, providing
an outlet for community concern, hostility,
and emotion. To work effectively, it is
important that society's criminal process
"satisfy the appearance of justice," Offutt v.
United States, 348 US 11, 14, 99 L Ed 11, 75
S Ct 11, which can best be provided by
allowing people to observe such process.
From this unbroken, uncontradicted history,
supported by reasons as valid today as in
centuries past, it must be concluded that a
presumption of openness inheres in the very
nature of a criminal trial under this Nation's
system of justice, Cf., e.g., Levine v. United
States, 362 US 610, 4 L Ed 2d 989, 80 S Ct
1038.
(b) The freedoms of speech, press, and
assembly, expressly guaranteed by the First
Amendment, share a common core purpose
of assuring freedom of communication on
matters relating to the functioning of
government. In guaranteeing freedoms such
as those of speech and press, the First
Amendment can be read as protecting the
right of everyone to attend trials so as to
give meaning to those explicit guarantees;
the First Amendment right to receive
information and ideas means, in the context
of trials, that the guarantees of speech and
press, standing alone, prohibit government
from summarily closing courtroom doors
which had long been open to the public at

the time the First Amendment was adopted.


Moreover, the right of assembly is also
relevant, having been regarded not only as
an independent right but also as a catalyst to
augment the free exercise of the other First
Amendment rights with which it was
deliberately linked by
the draftsmen. A trial courtroom is a public
place where the people generally and
representatives of the media have a right
to be present, and where their presence
historically has been thought to enhance the
integrity and quality of what takes place.
(c) Even though the Constitution contains no
provision which by its terms guarantees to
the public the right to attend criminal trials,
various fundamental rights, not expressly
guaranteed, have been recognized as
indispensable to the enjoyment of
enumerated rights. The right to attend
criminal trials is implicit in the guarantees of
the First Amendment; without the freedom
to attend such trials, which people have
exercised for centuries, important aspects of
freedom of speech and of the press could be
eviscerated.
Be that as it may, we recognize that pervasive and
prejudicial publicity under certain circumstances can
deprive an accused of his due process right to fair
trial. Thus, in Martelino, et al. vs. Alejandro, et al., 54
we held that to warrant a finding of prejudicial
publicity there must be allegation and proof that the
judges have been unduly influenced, not simply that
they might be, by the barrage of publicity. In the
case at bar, we find nothing in the records that will
prove that the tone and content, of the publicity that
attended the investigation of petitioners fatally
infected the fairness and impartiality of the DOJ

Panel. Petitioners cannot just rely on the subliminal


effects of publicity on the sense of fairness of the
DOJ Panel, for these are basically unbeknown and
beyond knowing. To be sure, the DOJ Panel is
composed of an Assistant Chief State Prosecutor and
Senior State Prosecutors. Their long experience in
criminal investigation is a factor to consider in
determining whether they can easily be blinded by
the klieg lights of publicity. Indeed, their 26-page
Resolution carries no indubitable indicia of bias for
it does not appear that they considered any extrarecord evidence except evidence properly adduced
by the parties. The length of time the investigation
was conducted despite its summary nature and the
generosity with which they accommodated the
discovery motions of petitioners speak well of their
fairness. At no instance, we note, did petitioners seek
the disqualification of any member of the DOJ Panel
on the ground of bias resulting from their
bombardment of prejudicial publicity.
It all remains to state that the Vizconde case will
move to a more critical stage as petitioners will now
have to undergo trial on the merits. We stress that
probable cause is not synonymous with guilt and
while the light of publicity may be a good
disinfectant of unfairness, too much of its heat can
bring to flame an accused's right to fair trial. Without
imposing on the trial judge the difficult task of
supervising every specie of speech relating to the
case at bar, it behooves her to be reminded of the
duty of a trial judge in high profile criminal cases to
control publicity prejudicial to the fair
administration of justice. 55 The Court reminds
judges that our ability to dispense impartial justice is
an issue in every trial and in every criminal
prosecution, the judiciary always stands as a silent
accused. More than convicting the guilty and
acquitting the innocent, the business of the judiciary
is to assure fulfillment of the promise that justice

shall be done and is done and that is the only way of the Hongkong & Shanghai Banking Corporation (HSBC).
The search for and seizure of said articles were made with
for the judiciary to get an acquittal from the bar of
the opposition of Alvarez who stated his protest below the
public opinion.
inventories on the ground that the agents seized even the
originals of the documents. As the articles had not been
IN VIEW WHEREOF, the petitions are dismissed
brought immediately to the judge who issued the search
for lack of showing of grave abuse of discretion on
warrant, Alvarez, through his attorney, filed a motion on 8
the part of the respondents. Costs against petitioners. June 1936, praying that the agent Emilio L. Siongco, or any
other agent, be ordered immediately to deposit all the seized
articles in the office of the clerk of court and that said agent
SO ORDERED.
be declared guilty of contempt for having disobeyed the
order of the court. On said date the court issued an order
** Alvarez v CFI 64 PHIL 33 (1937)
directing Siongco to deposit all the articles seized within 24
Facts: On 3 June 1936, the chief of the secret service of the hours from the receipt of notice thereof and giving him a
Anti-Usury Board, of the Department of Justice, presented to period of 5 days within which to show cause why he should
Judge Eduardo Gutierrez David then presiding over the not be punished for contempt of court. On 10 June, Attorney
Court of First Instance of Tayabas, an affidavit alleging that Arsenio Rodriguez, representing the Anti-Usury Board, filed
according to reliable information, Narciso Alvarez kept in a motion praying that the order of the 8th of said month be
his house in Infanta, Tayabas, books, documents, receipts, set aside and that the Anti-Usury Board be authorized to
lists, chits and other papers used by him in connection with retain the articles seized for a period of 30 days for the
his activities as a moneylender, charging usurious rates of necessary investigation. On June 25, the court issued an
interest in violation of the law. In his oath at the end of the order requiring agent Siongco forthwith to file the search
affidavit, the chief of the secret service stated that his warrant and the affidavit in the court, together with the
answers to the questions were correct to the best of his proceedings taken by him, and to present an inventory duly
knowledge and belief. He did not swear to the truth of his verified by oath of all the articles seized. On July 2, the
statements upon his own knowledge of the facts but upon the attorney for the petitioner filed a petition alleging that the
information received by him from a reliable person. Upon search warrant issued was illegal and that it had not yet been
the affidavit the judge, on said date, issued the warrant which returned to date together with the proceedings taken in
is the subject matter of the petition, ordering the search of connection therewith, and praying that said warrant be
the Alvarezs house at any time of the day or night, the cancelled, that an order be issued directing the return of all
seizure of the books and documents and the immediate the articles seized to Alvarez, that the agent who seized them
delivery thereof to him to be disposed of in accordance with be declared guilty of contempt of court, and that charges be
the law. With said warrant, several agents of the Anti-Usury filed against him for abuse of authority. On September 10,
Board entered Alvarezs store and residence at 7:00 p.m. of 4 the court issued an order holding: that the search warrant
June 1936, and seized and took possession of the following was obtained and issued in accordance with the law, that it
articles: internal revenue licenses for the years 1933 to 1936, had been duly complied with and, consequently, should not
1 ledger, 2 journals, 2 cashbooks, 9 order books, 4 be cancelled, and that agent Siongco did not commit any
notebooks, 4 check stubs, 2 memorandums, 3 bankbooks, 2 contempt of court and must, therefore, be exonerated, and
contracts, 4 stubs, 48 stubs of purchases of copra, 2 ordering the chief of the Anti-Usury Board in Manila to
inventories, 2 bundles of bills of lading, 1 bundle of credit show cause, if any, within the unextendible period of 2 days
receipts, 1 bundle of stubs of purchases of copra, 2 packages from the date of notice of said order, why all the articles
of correspondence, 1 receipt book belonging to Luis seized appearing in the inventory should not be returned to
Fernandez, 14 bundles of invoices and other papers, many Alvarez. The assistant chief of the Anti-Usury Board of the
documents and loan contracts with security and promissory Department of Justice filed a motion praying, for the reasons
notes, 504 chits, promissory notes and stubs of used checks stated therein, that the articles seized be ordered retained for

the purpose of conducting an investigation of the violation of


the Anti-Usury Law committed by Alvarez. On October 10,
said official again filed another motion alleging that he
needed 60 days to examine the documents and papers seized,
which are designated on pages 1 to 4 of the inventory by
Nos. 5, 10, 16, 23, 25-27, 30-31 , 34, 36-43 and 45, and
praying that he be granted said period of 60 days. In an order
of October 16, the court granted him the period of 60 days to
investigate said 19 documents. Alvarez, herein, asks that the
search warrant as well as the order authorizing the agents of
the Anti-Usury Board to retain the articles seized, be
declared illegal and set aside, and prays that all the articles in
question be returned to him.
Issue: Whether the search warrant issued by the court is
illegal because it has been based upon the affidavit of agent
Almeda in whose oath he declared that he had no personal
knowledge of the facts which were to serve as a basis for the
issuance of the warrant but that he had knowledge thereof
through mere information secured from a person whom he
considered reliable, and that it is illegal as it was not
supported by other affidavits aside from that made by the
applicant.
Held: Section 1, paragraph 3, of Article III of the
Constitution and Section 97 of General Orders 58 require
that there be not only probable cause before the issuance of a
search warrant but that the search warrant must be based
upon an application supported by oath of the applicant and
the witnesses he may produce. In its broadest sense, an oath
includes any form of attestation by which a party signifies
that he is bound in conscience to perform an act faithfully
and truthfully; and it is sometimes defined as an outward
pledge given by the person taking it that his attestation or
promise is made under an immediate sense of his
responsibility to God. The oath required must refer to the
truth of the facts within the personal knowledge of the
petitioner or his witnesses, because the purpose thereof is to
convince the committing magistrate, not the individual
making the affidavit and seeking the issuance of the warrant,
of the existence of probable cause. The true test of
sufficiency of an affidavit to warrant issuance of a search
warrant is whether it has been drawn in such a manner that
perjury could be charged thereon and affiant be held liable
for damages caused. The affidavit, which served as the
exclusive basis of the search warrant, is insufficient and
fatally defective by reason of the manner in which the oath

was made, and therefore, the search warrant and the


subsequent seizure of the books, documents and other papers
are illegal. Further, it is the practice in this jurisdiction to
attach the affidavit of at least the applicant or complainant to
the application. It is admitted that the judge who issued the
search warrant in this case, relied exclusively upon the
affidavit made by agent Almeda and that he did not require
nor take the deposition of any other witness. Neither the
Constitution nor General Orders 58 provides that it is of
imperative necessity to take the depositions of the witnesses
to be presented by the applicant or complainant in addition
to the affidavit of the latter. The purpose of both in requiring
the presentation of depositions is nothing more than to
satisfy the committing magistrate of the existence of
probable cause. Therefore, if the affidavit of the applicant or
complainant is sufficient, the judge may dispense with that
of other witnesses. Inasmuch as the affidavit of the agent
was insufficient because his knowledge of the facts was not
personal but merely hearsay, it is the duty of the judge to
require the affidavit of one or more witnesses for the purpose
of determining the existence of probable cause to warrant the
issuance of the search warrant. When the affidavit of the
applicant or complainant contains sufficient facts within his
personal and direct knowledge, it is sufficient if the judge is
satisfied that there exists probable cause; when the
applicants knowledge of the facts is mere hearsay, the
affidavit of one or more witnesses having a personal
knowledge of the facts is necessary. Thus the warrant issued
is likewise illegal because it was based only on the affidavit
of the agent who had no personal knowledge of the facts.

The petitioner asks that the warrant of June 3, 1936, issued


by the Court of First Instance of Tayabas, ordering the search
of his house and the seizure, at any time of the day or night,
of certain accounting books, documents and papers
belonging to him in his residence situated in Infanta,
Province of Tayabas, as well as the order of a later date,
authorizing the agents of the Anti-Usury Board to retain the
articles seized, be declared illegal and set aside, and prays
that all the articles in question be returned to him.

correspondence, one receipt book belonging to Luis


Fernandez, fourteen bundles of invoices and other papers
many documents and loan contracts with security and
promissory notes, 504 chits, promissory notes and stubs of
used checks of the Hongkong & Shanghai Banking
Corporation. The search for and a seizure of said articles
were made with the opposition of the petitioner who stated
his protest below the inventories on the ground that the
agents seized even the originals of the documents. As the
articles had not been brought immediately to the judge who
On the date above-mentioned, the chief of the secret service issued the search warrant, the petitioner, through his
of the Anti-Usury Board, of the Department of Justice,
attorney, filed a motion on June 8, 1936, praying that the
presented to Judge Eduardo Gutierrez David then presiding agent Emilio L. Siongco, or any other agent, be ordered
over the Court of First Instance of Tayabas, an affidavit
immediately to deposit all the seized articles in the office of
alleging that according to reliable information, the petitioner the clerk of court and that said agent be declared guilty of
kept in his house in Infanta, Tayabas, books, documents,
contempt for having disobeyed the order of the court. On
receipts, lists, chits and other papers used by him in
said date the court issued an order directing Emilio L.
connection with his activities as a money-lender charging
Siongco to deposit all the articles seized within twenty-four
usurious rates of interest in violation of the law. In his oath at hours from the receipt of notice thereof and giving him a
the and of the affidavit, the chief of the secret service stated period of five (5) days within which to show cause why he
that his answers to the questions were correct to the best of
should not be punished for contempt of court. On June 10th,
his knowledge and belief. He did not swear to the truth of his Attorney Arsenio Rodriguez, representing the Anti-Usury
statements upon his own knowledge of the facts but upon the Board, filed a motion praying that the order of the 8th of said
information received by him from a reliable person. Upon
month be set aside and that the Anti-Usury Board be
the affidavit in question the Judge, on said date, issued the
authorized to retain the articles seized for a period of thirty
warrant which is the subject matter of the petition, ordering (30) days for the necessary investigation. The attorney for
the search of the petitioners house at nay time of the day or the petitioner, on June 20th, filed another motion alleging
night, the seizure of the books and documents abovethat, notwithstanding the order of the 8th of said month, the
mentioned and the immediate delivery thereof to him to be
officials of the Anti-Usury Board had failed to deposit the
January 29, 1937
disposed of in accordance with the law. With said warrant,
articles seized by them and praying that a search warrant be
several agents of the Anti-Usury Board entered the
G.R. No. L-45358
issued, that the sheriff be ordered to take all the articles into
petitioners
store
and
residence
at
seven
oclock
on
the
night
NARCISO ALVAREZ, petitioner,
his custody and deposit of the Anti-Usury Board be punished
of June 4, 1936, and seized and took possession of the
vs.
for contempt of court. Said attorney, on June 24th, filed an
THE COURT OF FIRST INSTANCE OF TAYABAS and following articles: internal revenue licenses for the years
ex parte petition alleging that while agent Emilio L. Siongco
1933 to 1936, one ledger, two journals, two cashbooks, nine had deposited some documents and papers in the office of
THE ANTI-USURY BOARD, respondents.
order books, four notebooks, four checks stubs, two
the clerk of court, he had so far failed to file an inventory
Godofredo Reyes for petitioner.
memorandums, three bankbooks, two contracts, four stubs,
duly verified by oath of all the documents seized by him, to
Adolfo N. Feliciano for respondents Anti-Usury Board.
forty-eight stubs of purchases of copra, two inventories, two return the search warrant together with the affidavit it
bundles of bills of lading, one bundle of credit receipts, one presented in support thereof, or to present the report of the
Imperial, J.:
bundle of stubs of purchases of copra, two packages of

proceedings taken by him; and prayed that said agent be


directed to filed the documents in question immediately. On
the 25th of said month the court issued an order requiring
agent Emilio L. Siongco forthwith to file the search warrant
and the affidavit in the court, together with the proceedings
taken by him, and to present an inventory duly verified by
oath of all the articles seized. On July 2d of said year, the
attorney for the petitioner filed another petition alleging that
the search warrant issue was illegal and that it had nit yet
been returned to date together with the proceedings taken in
connection therewith, and praying that said warrant be
cancelled, that an order be issued directing the return of all
the articles seized to the petitioner, that the agent who seized
them be declared guilty of contempt of court, and that
charges be filed against him for abuse of authority. On
September 10, 1936, the court issued an order holding: that
the search warrant was obtained and issued in accordance
with the law, that it had been duly complied with and,
consequently, should not be cancelled, and that agent Emilio
L. Siongco did not commit any contempt of court and must,
therefore, be exonerated, and ordering the chief of the AntiUsury Board in Manila to show case, if any, within the
unextendible period of two (2) days from the date of notice
of said order, why all the articles seized appearing in the
inventory, Exhibit 1, should not be returned to the petitioner.
The assistant chief of the Anti-Usury Board of the
Department of Justice filed a motion praying, for the reasons
stated therein, that the articles seized be ordered retained for
the purpose of conducting an investigation of the violation of
the Anti-Usury Law committed by the petitioner. In view of
the opposition of the attorney for the petitioner, the court, on
September 25th, issued an order requiring the Anti-Usury
Board to specify the time needed by it to examine the
documents and papers seized and which of them should be
retained, granting it a period of five (5) days for said
purpose. On the 30th of said month the assistant chief of the
Anti-Usury Board filed a motion praying that he be granted
ten (10) days to comply with the order of September 25th
and that the clerk of court be ordered to return to him all the

documents and papers together with the inventory thereof.


The court, in an order of October 2d of said year, granted
him the additional period of ten(10) days and ordered the
clerk of court to send him a copy of the inventory. On
October 10th, said official again filed another motion
alleging that he needed sixty (60) days to examine the
documents and papers seized, which are designated on pages
1 to 4 of the inventory by Nos. 5, 1016, 23, 25, 26, 27, 30,
31, 34, 36, 37, 38, 39, 40, 41, 42, 43 and 45, and praying that
he be granted said period of sixty (60) days. In an order of
October 16th, the court granted him the period of sixty (60)
days to investigate said nineteen (19) documents. The
petitioner alleges, and it is not denied by the respondents,
that these nineteen (19)documents continue in the possession
of the court, the rest having been returned to said petitioner.

individual, to prevent stealthy encroachment upon, or


gradual depreciation on, the rights secured by them(State vs.
Custer County, 198 Pac., 362; State vs. McDaniel, 231 Pac.,
965; 237 Pac., 373). Since the proceeding is a drastic one, it
is the general rule that statutes authorizing searches and
seizure or search warrants must be strictly construed (Rose
vs. St. Clair, 28 Fed., [2d], 189; Leonard vs. U. S., 6 Fed.
[2d], 353; Perry vs. U. S. 14 Fed. [2d],88; Cofer vs. State,
118 So., 613).

III. The petitioner claims that the search warrant issued by


the court is illegal because it has been based upon the
affidavit of agent Mariano G. Almeda in whose oath he
declared that he had no personal knowledge of the facts
which were to serve as a basis for the issuance of the warrant
but that he had knowledge thereof through mere information
I. A search warrant is an order in writing, issued in the name secured from a person whom he considered reliable. To the
of the People of the Philippine Islands, signed by a judge or question What are your reason for applying for this search
a justice of the peace, and directed to a peace officer,
warrant, appearing in the affidavit, the agent answered: It
commanding him to search for personal property and bring it has been reported to me by a person whom I consider to be
before the court (section 95, General Orders. No. 58, as
reliable that there are being kept in said premises, books,
amended by section 6 of Act No. 2886). Of all the rights of a documents, receipts, lists, chits, and other papers used by
citizen, few are of greater importance or more essential to his him in connection with his activities as a money-lender,
peace and happiness than the right of personal security, and charging a usurious rate of interest, in violation of the law
that involves the exemption of his private affairs, books, and and in attesting the truth of his statements contained in the
papers from the inspection and scrutiny of others (In re
affidavit, the said agent states that he found them to be
Pacific Railways Commission, 32 Fed., 241; Interstate
correct and true to the best of his knowledge and belief.
Commerce Commission vs Brimson, 38 Law. ed., 1047;
Broyd vs. U. S., 29 Law. ed., 746; Caroll vs. U. S., 69 Law. Section 1, paragraph 3, of Article III of the Constitution,
relative to the bill of rights, provides that The right of the
ed., 543, 549). While the power to search and seize is
necessary to the public welfare, still it must be exercised and people to be secure in their persons, houses, papers, and
effects against unreasonable searches and seizures shall not
the law enforced without transgressing the constitutional
be violated, and no warrants shall issue but upon probable
rights or citizen, for the enforcement of no statue is of
cause, to be determined by the judge after examination under
sufficient importance to justify indifference to the basis
principles of government (People vs. Elias, 147 N. E., 472). oath or affirmation of the complainant and the witnesses he
may produce, and particularly describing the place top be
II. As the protection of the citizen and the maintenance of his searched, and the persons or things to be seized. Section 97
constitutional right is one of the highest duties and privileges of General Orders, No. 58 provides that A search warrant
of the court, these constitutional guaranties should be given a shall not issue except for probable cause and upon
liberal construction or a strict construction in favor of the
application supported by oath particularly describing the

place to be searched and the person or thing to be seized. It


will be noted that both provisions require that there be not
only probable cause before the issuance of a search warrant
but that the search warrant must be based upon an
application supported by oath of the applicant ands the
witnesses he may produce. In its broadest sense, an oath
includes any form of attestation by which a party signifies
that he is bound in conscience to perform an act faithfully
and truthfully; and it is sometimes defined asan outward
pledge given by the person taking it that his attestation or
promise is made under an immediate sense of his
responsibility to God (Bouviers Law Dictionary; State vs.
Jackson, 137 N. W., 1034; In re Sage, 24 Oh. Cir. Ct. [N. S.],
7; Pumphery vs. State, 122 N. W., 19; Priest vs. State, 6 N.
W., 468; State vs. Jones, 154 Pac., 378; Atwood vs. State,
111 So., 865). The oath required must refer to the truth of the
facts within the personal knowledge of the petitioner or his
witnesses, because the purpose thereof is to convince the
committing magistrate, not the individual making the
affidavit and seeking the issuance of the warrant, of the
existence of probable cause (U. S. vs. Tureaud, 20 Fed., 621;
U. S. vs. Michalski, 265 Fed., 8349; U. S. vs. Pitotto, 267
Fed., 603; U. S. vs. Lai Chew, 298 Fed., 652). The true test
of sufficiency of an affidavit to warrant issuance of a search
warrant is whether it has been drawn in such a manner that
perjury could be charged thereon and affiant be held liable
for damages caused (State vs. Roosevelt Country 20th Jud.
Dis. Ct., 244 Pac., 280; State vs. Quartier, 236 Pac., 746).

particular case is purely a judicial question, determinable


from a consideration of the circumstances involved,
including the purpose of the search, the presence or absence
or probable cause, the manner in which the search and
seizure was made, the place or thing searched, and the
character of the articles procured (Go-Bart Importing Co. vs.
U. S. 75 Law. ed., 374; Peru vs. U. S., 4 Fed., [2d], 881;U. S.
vs. Vatune, 292 Fed., 497; Angelo vs. U. S. 70 Law, ed., 145;
Lambert vs. U. S. 282 Fed., 413; U. S. vs. Bateman, 278
Fed., 231; Mason vs. Rollins, 16 Fed. Cas. [No. 9252], 2
Biss., 99).

upon the affidavit made by agent Mariano G. Almeda and


that he did not require nor take the deposition of any other
witness. Neither the Constitution nor General Orders. No. 58
provides that it is of imperative necessity to take the
deposition of the witnesses to be presented by the applicant
or complainant in addition to the affidavit of the latter. The
purpose of both in requiring the presentation of depositions
is nothing more than to satisfy the committing magistrate of
the existence of probable cause. Therefore, if the affidavit of
the applicant or complainant is sufficient, the judge may
dispense with that of other witnesses. Inasmuch as the
affidavit of the agent in this case was insufficient because his
In view of the foregoing and under the above-cited
knowledge of the facts was not personal but merely hearsay,
authorities, it appears that the affidavit, which served as the it is the duty of the judge to require the affidavit of one or
exclusive basis of the search warrant, is insufficient and
more witnesses for the purpose of determining the existence
fatally defective by reason of the manner in which the oath
of probable cause to warrant the issuance of the search
was made, and therefore, it is hereby held that the search
warrant. When the affidavit of the applicant of the complaint
warrant in question and the subsequent seizure of the books, contains sufficient facts within his personal and direct
documents and other papers are illegal and do not in any way knowledge, it is sufficient if the judge is satisfied that there
warrant the deprivation to which the petitioner was
exist probable cause; when the applicants knowledge of the
subjected.
facts is mere hearsay, the affidavit of one or more witnesses
having a personal knowledge of the fact is necessary. We
IV. Another ground alleged by the petitioner in asking that
conclude, therefore, that the warrant issued is likewise illegal
the search warrant be declared illegal and cancelled is that it because it was based only on the affidavit of the agent who
was not supported by other affidavits aside from that made
had no personal knowledge of the facts.
by the applicant. In other words, it is contended that the
search warrant cannot be issued unless it be supported by
V. The petitioner alleged as another ground for the
affidavits made by the applicant and the witnesses to be
declaration of the illegality of the search warrant and the
presented necessity by him. Section 1, paragraph 3, of
cancellation thereof, the fact that it authorized its execution
Article III of the Constitution provides that no warrants shall at night. Section 101 of General Orders, No. 58 authorizes
It will likewise be noted that section 1, paragraph 3, of
issue but upon probable cause, to be determined by the judge that the search be made at night when it is positively asserted
Article III of the Constitution prohibits unreasonable
searches and seizure. Unreasonable searches and seizures are after examination under oath or affirmation of the
in the affidavits that the property is on the person or in the
complainant and the witnesses he may produce. Section 98
a menace against which the constitutional guarantee afford
place ordered to be searched. As we have declared the
full protection. The term unreasonable search and seizure of General Orders, No. 58 provides that the judge or justice affidavits insufficient and the warrant issued exclusively
must, before issuing the warrant, examine under oath the
is not defined in the Constitution or in General Orders No.
upon it illegal, our conclusion is that the contention is
58, and it is said to have no fixed, absolute or unchangeable complainant and any witnesses he may produce and take
equally well founded and that the search could not legally be
their
depositions
in
writing.
It
is
the
practice
in
this
meaning, although the term has been defined in general
made at night.
jurisdiction to attach the affidavit of at least the applicant or
language. All illegal searches and seizure are unreasonable
complainant to the application. It is admitted that the judge
VI. One of the grounds alleged by the petitioner in support
while lawful ones are reasonable. What constitutes a
who issued the search warrant in this case, relied exclusively of his contention that the warrant was issued illegally is the
reasonable or unreasonable search or seizure in any

lack of an adequate description of the books and documents


to be seized. Section 1, paragraphs 3, of Article III of the
Constitution, and section 97 of General Orders, No. 58
provide that the affidavit to be presented, which shall serve
as the basis for determining whether probable cause exist
and whether the warrant should be issued, must contain a
particular description of the place to be searched and the
person or thing to be seized. These provisions are mandatory
and must be strictly complied with (Munch vs. U. S., 24 Fed.
[2d], 518; U. S. vs. Boyd, 1 Fed. [2d], 1019; U. S. vs.
Carlson, 292 Fed., 463; U. S. vs. Borkowski, 268 Fed., 408;
In re Tri-State Coal & Coke Co., 253 Fed., 605; People vs.
Mayen, 188 Cal., 237; People vs. Kahn, 256 Ill. App., 4125);
but where, by the nature of the goods to be seized, their
description must be rather generally, it is not required that a
technical description be given, as this would mean that no
warrant could issue (People vs. Rubio, 57 Phil. 284; People
vs. Kahn, supra). The only description of the articles given
in the affidavit presented to the judge was as follows: that
there are being kept in said premises books, documents,
receipts, lists, chits and other papers used by him in
connection with his activities as money-lender, charging a
usurious rate of interest, in violation of the law. Taking into
consideration the nature of the article so described, it is clear
that no other more adequate and detailed description could
have been given, particularly because it is difficult to give a
particular description of the contents thereof. The description
so made substantially complies with the legal provisions
because the officer of the law who executed the warrant was
thereby placed in a position enabling him to identify the
articles, which he did.
VII. The last ground alleged by the petitioner, in support of
his claim that the search warrant was obtained illegally, is
that the articles were seized in order that the Anti-Usury
Board might provide itself with evidence to be used by it in
the criminal case or cases which might be filed against him
for violation of the Anti-usury Law. At the hearing of the
incidents of the case raised before the court it clearly
appeared that the books and documents had really been

seized to enable the Anti-Usury Board to conduct an


investigation and later use all or some of the articles in
question as evidence against the petitioner in the criminal
cases that may be filed against him. The seizure of books and
documents by means of a search warrant, for the purpose of
using them as evidence in a criminal case against the person
in whose possession they were found, is unconstitutional
because it makes the warrant unreasonable, and it is
equivalent to a violation of the constitutional provision
prohibiting the compulsion of an accused to testify against
himself (Uy Kheytin vs. Villareal, 42 Phil,, 886; Brady vs. U.
S., 266 U. S., 620; Temperani vs. U. S., 299 Fed., 365; U. S.
vs. Madden, 297 Fed., 679; Boyd vs. U. S.,116 U. S., 116;
Caroll vs. U. S., 267 U. S., 132). Therefore, it appearing that
at least nineteen of the documents in question were seized
for the purpose of using them as evidence against the
petitioner in the criminal proceeding or proceedings for
violation against him, we hold that the search warrant issued
is illegal and that the documents should be returned to him.

orders which prejudiced him and are the subject matter of his
petition. Section 222 of the Code of Civil Procedure in fact
provides that mandamus will not issue when there is another
plain, speedy and adequate remedy in the ordinary course of
law. We are of the opinion, however, that an appeal from said
orders would have to lapse before he recovers possession of
the documents and before the rights, of which he has been
unlawfully deprived, are restored to him (Fajardo vs.
Llorente, 6 Phil. 426; Manotoc vs. McMicking and Trinidad,
10 Phil. 119; Cruz Herrera de Lukban vs. McMicking, 14
Phil. 641; Lamb vs. Phipps, 22 Phil. 456).

The Anti-Usury Board insinuates in its answer that the


petitioner cannot now question the validity of the search
warrant or the proceedings had subsequent to the issuance
thereof, because he has waived his constitutional rights in
proposing a compromise whereby he agreed to pay a fine of
P200 for the purpose of evading the criminal proceeding or
proceedings. We are of the opinion that there was no such
waiver, first, because the petitioner has emphatically denied
the offer of compromise and, second, because if there was a
compromise it reffered but to the institution of criminal
proceedings fro violation of the Anti-Usury Law. The waiver
would have been a good defense for the respondents had the
petitioner voluntarily consented to the search and seizure of
the articles in question, but such was not the case because
the petitioner protested from the beginning and stated his
protest in writing in the insufficient inventory furnished him
by the agents.

2. That since the provisions in question are drastic in their


form and fundamentally restrict the enjoyment of the
ownership, possession and use of the personal property of
the individual, they should be strictly construed;

Summarizing the foregoing conclusions, we hold:


1. That the provisions of the Constitution and General
Orders, No. 58, relative to search and seizure, should be
given a liberal construction in favor of the individual in
order to maintain the constitutional guaranties whole and in
their full force;

3. That the search and seizure made are illegal for the
following reasons: (a) Because the warrant was based solely
upon the affidavit of the petitioner who had no personal
knowledge of the facts of probable cause, and (b) because
the warrant was issued for the sole purpose of seizing
evidence which would later be used in the criminal
proceedings that might be instituted against the petitioner,
for violation of the Anti-Usury Law;

4. That as the warrant had been issued unreasonably, and as


it does not appear positively in the affidavit that the articles
were in the possession of the petitioner and in the place
indicated, neither could the search and seizure be made at
Said board alleges as another defense that the remedy sought night;
by the petitioner does not lie because he can appeal from the

5. That although it is not mandatory to present affidavits of


witnesses to corroborate the applicant or a complainant in
cases where the latter has personal knowledge of the facts,
when the applicants or complainants knowledge of the facts
is merely hearsay, it is the duty of the judge to require
affidavits of other witnesses so that he may determine
whether probable cause exists;
6. That a detailed description of the person and place to be
searched and the articles to be seized is necessary, but
whereby, by the nature of the articles to be seized, their
description must be rather general, but is not required that a
technical description be given, as this would mean that no
warrant could issue;

IN THE MATTER OF THE APPLICATION FOR A


WRIT OF HABEAS CORPUS, SIMON LUNA,
petitioner-appellant,
vs.
HON. LORENZO M. PLAZA, as Judge of the Municipal
Court of Tandag, Surigao del Sur; HON. SANTOS B.
BEBERINO as Provincial Fiscal of Surigao del Sur; and
THE PROVINCIAL WARDEN of Surigao del Sur,
respondents- appellees.
Sisenando Villaluz and Juan T. David for petitionerappellant.
Office of the Assistant Solicitor General Pacifico P. de
Castro and Solicitor Augusto M. Amores for other
respondents-appellees.
Provincial Fiscal Santos B. Beberno in his own behalf as
respondent-appellee.

questions were propounded by T-Sgt. Candido Patosa, and


that the answers were made by them. The affiants further
declared before respondent Judge that their answers were
true, and were freely and voluntarily made; that they fully
understood the questions and answers, and that they were
willing to sign their respective affidavits. The affiants signed
their respective affidavits in the presence of the respondent
Judge, who also signed after the usual procedure of
administering the oath.

Considering the answers of the affiants to the questions


contained in their sworn statements, together with the postmortem and autopsy report on the dead body of the victim
Jaime Diaz Ng, the certificate of death, the sketch showing
the position of the victim and the accused, and Exhibits 6, 7,
7. That the petitioner did not waive his constitutional rights
8, 12, and 13 of herein respondents, the respondent Judge
because the offer of compromise or settlement attributed to
opined that there was reasonable ground to believe that the
him, does not mean, if so made, that he voluntarily tolerated
crime of murder had been committed and the accused was
the search and seizure; and
ZALDIVAR, J.:
probably guilty thereof. Respondent Judge issued the order
and warrant of arrest, specifying therein that no bail should
8. That an appeal from the orders questioned by the
Appeal from the decision of the Court of First Instance of
be accepted for the provisional release of the accused. On
petitioner, if taken by him, would not be an effective, speedy Surigao del Sur, dated April 20, 1967, dismissing the petition February 20, 1967, upon motion of petitioner that he be
or adequate remedy in the ordinary course of law, and,
for a writ of habeas corpus, filed by herein petitioneradmitted to bail upon the ground that the evidence of guilt
consequently, the petition for mandamus filed by him, lies.
appellant Simon Luna hereinafter referred to simply as
was not strong, respondent Judge issued an order granting
petitioner who was charged with murder in Criminal Case bail, fixing it at P30,000.00; which order, however,
For the foregoing considerations, the search warrant and the No. 655-New of the same court.
respondent Judge later revoked, and petitioner was denied
seizure of June 3, 1936, and the orders of the respondent
bail.
court authorizing the relation of the books and documents,
The criminal action was commenced by T-Sgt. Candido
are declared illegal and are set aside, and it is ordered that
Patosa, PC investigator of Tandag, Surigao del Sur, by filing The case was subsequently remanded to the Court of First
the judge presiding over the Court of First Instance of
with respondent Municipal Judge Lorenzo M. Plaza, of the
Instance of Surigao del Sur, after petitioner filed a waiver of
Tayabas direct the immediate return to the petitioner of the
Municipal Court of Tandag, criminal case No. 1138 charging his right to preliminary investigation. On March 9, 1967
nineteen (19) documents designated on pages 1 to 4 of the
the accused, herein petitioner, with the crime of murder.
respondent Provincial Fiscal filed an information charging
inventory by Nos. 5, 10, 16, 23, 25,26, 27, 30, 31, 34, 36, 37, Supporting the complaint were sworn statements of the
herein petitioner with the crime of murder. The petitioner
38, 39, 40, 41, 42, 43 and 45, without special pronouncement witnesses for the prosecution, in the form of questions and
was detained in the provincial jail of Surigao del Sur under
as to costs. So ordered.
answers taken by T-Sgt. Patosa, and subscribed and sworn to the custody of respondent Provincial Warden.
before the respondent Judge at the time of the filing of the
complaint. The respondent Judge examined the prosecution On April 5, 1967, petitioner filed a petition for a writ of
G.R. No. L-27511 November 29, 1968
witnesses by reading to them "all over again the questions
habeas corpus with the Court of First Instance of Surigao del
and answers" in their statements in writing, and the
Sur, therein docketed as Special Proceedings No. 105-New,
witnesses-affiants declared before said Judge that the
claiming that he was being deprived of liberty without the

due process of law, on the ground that the imprisonment and


detention was the result of a warrant of arrest issued by
respondent Judge in violation of Republic Act No. 3828, and
praying for the annulment of the order for his arrest and his
discharge from confinement.

reduced to writing and form part of the records of the case.


The record of the instant case, according to petitioner, does
not show said examination was performed by respondent
Judge. Petitioner urges that the absence of any document in
the record that shows that respondent Judge had performed
the examination is positive proof that respondent Judge did
Herein respondents filed their answer, alleging that Republic not perform his duty, notwithstanding his testimony before
Act No. 3828 had been substantially complied with; that a
the Court of First Instance of Surigao del Sur, during the
motion to quash, and not a petition for habeas corpus was
hearing of this case, to the effect that he adopted the
the proper remedy; and that petitioner's application for bail
questions propounded to each of the prosecution witnesses
constituted a waiver of the right to question the validity of
by T-Sgt. Patosa. Petitioner maintains that this testimony,
the arrest.
being self-serving intended to cover up the failure to comply
with the law, should not have been believed by the Court of
After trial, the Court of First Instance of Surigao del Sur
First Instance, and said court thereby committed errors
rendered its decision, dated April 20, 1967, holding that
when, believing said testimony, it found that there had been
respondent Municipal Judge had substantially complied with substantial compliance with the requirement that the
Republic Act No. 3828, and consequently denied the
municipal judge should personally examine the witnesses.
application for the writ of habeas corpus, and dismissed the Petitioner further maintains that assuming that the adoption
case. Hence this appeal.
of the questions made by T-Sgt. Patosa constituted
substantial compliance with the requirement that the judge
Petitioner, in his assignment of errors, claims that the trial
should examine the witnesses by asking searching questions,
court erred, as follows:
still the second requirement, that of reducing to writing the
said procedure of adoption, has not been complied with; and
1. In giving absolute credence to the oral testimony so, Republic Act No. 3828 was still violated, and the
of the respondent Judge to the effect that he adopted issuance of the warrant of arrest was in violation of said Act
and made his own the questions and answers taken
and the Constitution and constituted denial of due process.
by T-Sgt. Patosa, PC Investigator, one of the
prosecution witnesses, because the records show the Petitioner contends that the trial court erred in giving
contrary;
absolute credence to the testimony of respondent Municipal
Judge. Regarding credibility of witnesses, this Court has
2. In denying the writ of habeas corpus and in
consistently held that, as a general rule, the lower court's
dismissing the petition.
findings as to the credibility of witnesses will not be
interfered with by appellate courts. Thus, in the case of
1. In support of his first assignment of error, petitioner
People vs. Sinaon1 this Court said:
contends that Republic Act No. 3828 imposes on a municipal
judge, before he can issue a warrant of arrest, two specific
Time and again, we have held that as a rule where
duties, to wit: (1) personally examine the complainant and
the issue is one of credibility of witnesses, appellate
witnesses with "searching questions and answers", which
courts will not generally disturb the findings of the
means that the judge must cross-examine them in case their
trial court, considering that it is in a better position
affidavits are presented; and (2) said examination must be
to decide the question, having seen and heard the

witnesses themselves and observed their deportment


and manner of testifying during the trial, unless there
is a showing that it has overlooked certain facts of
substance and value, that if considered, might affect
the result of the case.
Petitioner has appealed "from the decision/order" of the trial
court "to the Honorable Supreme Court of the Philippines,
on the ground that the same is contrary to law and the
Philippine Constitution" and prayed that "all the records of
the proceeding and the evidence, oral and documentary, be
transmitted or forwarded to the Honorable Supreme
Court ...".2 Since petitioner appealed directly to this Court he
must, therefore, raise only questions of law and he has
thereby waived the right to raise any question of fact, 3 and
the findings of facts of the trial court, under the rules and
precedents, must be deemed final and binding upon this
Court.4
The findings of facts of the trial court are found in the
following portion of the decision appealed from, to wit:
There is no dispute that there is a valid complaint
charging the accused Simon Luna, the herein
petitioner with the crime of Murder filed with the
respondent Judge authorized to conduct the
examination of the witnesses for the prosecution for
the purpose of determining the existence of probable
cause before the issuance of the corresponding
warrant of arrest; that the complaint is supported by
the statements of the witnesses under oath in writing
in the form of questions and answers and other
documents attached to the complaint; that before the
issuance of the corresponding warrant of arrest, the
respondent judge personally examined the witnesses
for the prosecution on their statements taken by TSgt. Candido Patosa by reading the questions and
answers all over again to the affiants who confirmed
to the respondent Judge that the statements
contained in their sworn statements are true; that

being satisfied that the questions and answers


contained in the sworn statements taken by T-Sgt
Patosa partake of the nature of his searching
questions and answers as required by law, the
respondent Judge adopted them as his own personal
examination of the witnesses for the purpose of
determining the existence of probable cause, the
order and the warrant of arrest were issued to take
the accused into custody for the commission of the
offense charged (Exhibits "H", "H-1", "I", and "I-1"petitioner); and that the petitioner waived his right to
the preliminary investigation (Exhibit "12"respondent) and applied to be admitted to bail.
Petitioner, however, claims that the failure of respondent
Judge to put in writing that he adopted the questions asked
by T-Sgt. Patosa and his failure to ask "searching questions"
violated Republic Act No. 3828.
Republic Act No. 3828, approved June 22, 1963, inserted in
section 87 (e) of the Judiciary Act of 1948 the following
paragraph:
No warrant of arrest shall be issued by any justice of
the peace in any criminal case filed with him unless
he first examines the witness or witnesses
personally, and the examination shall be under oath
and reduced to writing in the form of searching
questions and answers.
Before a municipal judge may issue a warrant of arrest, the
following conditions must first be fulfilled: (1) he must
examine the witnesses personally; (2) the examination must
be under oath; (3) the examination must be reduced to
writing in the form of searching questions and answers. Were
these conditions fulfilled in the instant case?
The first condition was fulfilled. The trial court found as a
fact that "the respondent judge personally examined the
witnesses for the prosecution ...;" that respondent judge

adopted as his own personal examination the questions asked


by T-Sgt. Patosa as appearing in the written statements,
which he read over again to the witnesses together with the
answers given therein, asking the witnesses whether said
answers were theirs, and whether the same answers were
true, to which the witness answered in the affirmative.
Republic Act No. 3828 does not prohibit the municipal
Judge from adopting the questions asked by a previous
investigator.
It appears that the sworn statements5 of the witnesses state at
the beginning that the sworn statement was "taken by T-Sgt.
Candido L. Patosa", and does not state that it was taken by
the respondent municipal Judge himself. This circumstance
is explained by the fact that said written statements already
taken by T-Sgt. Patosa were delivered to respondent
Municipal Judge who adopted the questions therein in his
examination, because he considered them searching
questions. Respondent Judge presumably did not consider it
necessary to change the introductory remarks in each of the
written statements. But that he made the examination
personally cannot be doubted; it is so stated in the order
dated February 18, 1967, which recites:
After examining the witness personally and under
oath there is reasonable ground to believe that an
offense for murder has been committed and that the
accused, Simon Luna, is probably guilty thereof.
(Exh. H)
The ruling in Doce vs. Branch II of the Court of First
Instance of Quezon, et al.,6 wherein this Court held that the
warrant of arrest issued therein was irregularly issued is not
applicable to the case at bar for the simple reason that the
facts are different. This Court in that case said:
There is merit in the assertion that the warrant of
arrest was irregularly issued. Section 87 of the
Judiciary Act as amended by Republic Act 3828
requires that the Municipal Judge issuing the same,

personally examine under oath the witnesses, and by


searching questions and answers which are to be
reduced to writing. Here, instead of searching
questions and answers, we have only the affidavits
of respondent and her one witness. Moreover, said
affidavits were sworn to before Judge Cabungcal,
not before Judge Juntereal who issued the warrant of
arrest.
In the instant case, as stated above, the respondent Municipal
Judge personally examined under oath the witnesses by
asking questions, that were adopted from a previous
investigation, and considered by him as sufficiently
searching and which questions and the answers thereto were
in writing and sworn to before him prior to his issuance of
the order of arrest.
The second condition required by Republic Act No. 3828 for
the issuance of a warrant of arrest was also fulfilled. The trial
court found that the complaint was "supported by statements
of the witnesses under oath." The record also shows the
following documents to have been subscribed and sworn to
before respondent Judge, namely: Exhibit B, sworn
statement of herein petitioner Simon Luna y Albay; Exhibit
C, sworn statement of Eusebio Corpuz; Exhibit D, sworn
statement of Bruno M. Zafra; Exhibit E, sworn statement of
Martiliano J. Bautista; Exhibit F, sworn statement of
Janedina Diaz y Bandoy.
The third condition required by Republic Act No. 3828 was
likewise fulfilled. The examination of the witnesses was
written down, in the form of searching questions and
answers. The term "searching questions and answers" means
only, taking into consideration the purpose of the preliminary
examination which is to determine "whether there is a
reasonable ground to believe that an offense has been
committed and the accused is probably guilty thereof so that
a warrant of arrest may be issued and the accused held for
trial",7 such questions as have tendency to show the
commission of a crime and the perpetrator thereof. What

would be searching questions would depend on what is


sought to be inquired into, such as: the nature of the offense,
the date, time, and place of its commission, the possible
motives for its commission; the subject, his age, education,
status, financial and social circumstances, his attitude toward
the investigation, social attitudes, opportunities to commit
the offense; the victim, his age, status, family
responsibilities, financial and social circumstances,
characteristics, etc. The points that are the subject of inquiry
may differ from case to case. The questions, therefore, must
to a great degree depend upon the Judge making the
investigation. At any rate, the court a quo found that
respondent judge was "satisfied that the questions and
answers contained in the sworn statements taken by T-Sgt.
Patosa partake of the nature of his searching questions and
answers as required by law," so the respondent Judge
adopted them.

Moreover, this Court has held that preliminary examination


is not an essential part of due process of law.8 Preliminary
examination may be conducted by the municipal judge, prior
to the issuance of the warrant of arrest, either in the
presence, or in the absence, of the accused. The record
shows that herein petitioner waived the preliminary
investigation before respondent Municipal Judge, and
instead, he filed a petition for bail. The petition for bail was
at first granted by respondent Judge, but later the order
granting bail was revoked. This conduct of petitioner
indicates that he had waived his objection to whatever
defect, if any, in the preliminary examination conducted by
respondent Judge prior to the issuance of the warrant of
arrest. Indeed, petitioner has no substantial much less
legal ground to complain that he was denied the due
process of law.

Provincial Warden by virtue of the order of arrest dated


February 18, 1967, and the order dated February 21, 1967, of
respondent Judge, to confine petitioner in the provincial jail.
It is not disputed by petitioner that respondent Judge had
jurisdiction to issue the warrant of arrest and the order of
commitment under the provisions of Section 47, Republic
Act No. 409, as amended by Republic Act No. 1201,
although petitioner did question the validity of the warrant of
arrest for allegedly having been issued in violation of
Republic Act No. 3828 which claim We have found to be
untenable. Consequently, the trial Judge did not commit an
error in denying the writ of habeas corpus prayed for.

At any rate, we believe that, if at all, the remedy available to


the petitioner herein, under the circumstances stated in this
opinion, is not a petition for a writ of habeas corpus but a
petition to quash the warrant of arrest or a petition for a
We find that the trial Judge committed no error when he held reinvestigation of the case by the respondent Municipal
Petitioner's further contention that the issuance of the
that, based upon the facts shown during the hearing of this
Judge or by the Provincial Fiscal.
warrant of arrest was a violation of the constitution and of
case, respondent Municipal Judge had substantially complied
We wish to stress, however, that what has been stated in this
procedural due process is likewise untenable. The
with the requirements of the law specifically Republic
Constitution, in Section 1(3), Article III, provides that no
Act 3828 before issuing the warrant of arrest in this case. opinion is certainly not intended to sanction the return to the
former practice of municipal judges of simply relying upon
warrant shall issue but upon probable cause, to be
2. In the light of what has been said above, it appears clear
affidavits or sworn statements that are made to accompany
determined by the judge after examination under oath or
that petitioner's second assignment of error, that the trial
the complaints that are filed before them, in determining
affirmation of the complainant and the witnesses he may
court erred in denying the writ of habeas corpus, is
whether there is a probable cause for the issuance of a
produce. The constitutional requirement of examination of
untenable. Moreover, Section 4 of Rule 102; of the Rules of warrant of arrest. That practice is precisely what is sought to
witnesses under oath was, as shown above, fulfilled. The
be voided by the amendment of Section 87 (c) of Republic
existence of probable cause depended to a large degree upon Court provides in part, as follows:
Act 296 (Judiciary Act of 1948) which requires that before a
the finding or opinion of the judge conducting the
SEC. 4. When writ not allowed or discharge
municipal judge issues a warrant of arrest he should first
examination. Respondent judge found that there was
authorized. If it appears that the person alleged to satisfy himself that there is a probable cause by examining
probable cause, as stated in his order of arrest, that "after
be restrained of his liberty is in the custody of an
the witnesses personally, and that the examination must be
examining the witnesses personally and under oath there is a
officer under process issued by a court or judge ...
under oath and reduced to writing in the form of searching
reasonable ground to believe that an offense of murder has
and
that
the
court
or
judge
had
jurisdiction
to
issue
questions and answers. It is obvious that the purpose of this
been committed and that the accused, Simon Luna, is
the process ... or make the order the writ, shall not be amendment is to prevent the issuance of a warrant of arrest
probably guilty thereof."
allowed....
against a person based simply upon affidavits of witnesses
Petitioner's last contention that the warrant of arrest issued
who made, and swore to, their statements before a person or
All the conditions, in the afore-quoted Section 4, set forth to persons other than the judge before whom the criminal
was a violation of procedural due process because of the
deny the writ, are present in the instant case. It is shown that complaint is filed. We wish to emphasize strict compliance
alleged defective preliminary examination has no leg to
petitioner is detained and is in the custody of the respondent by municipal or city judges of the provision of Section 87 (c)
stand on, in view of what we have hereinbefore stated.

of the Judiciary Act of 1948, as amended by Republic Act


3828, in order to avoid malicious and/or unfounded criminal
prosecution of persons.9
In the case now before Us, while it is true that the respondent
Municipal Judge did not himself personally cause to be
reduced to writing in the form of questions and answers the
examination of witnesses presented before him by the person
who filed the criminal complaint, We are satisfied that, as
shown by the evidence, respondent Judge had personally
examined the witnesses under oath and that the questions
asked by the Judge and the answers of the witnesses were
reflected in writings which were actually subscribed and
sworn to before him. Moreover, We are of the considered
view that no substantial right of the petitioner had been
violated because, as hereinbefore adverted to, petitioner
waived his right to preliminary investigation after he was
arrested, and he took the step of applying for bail before
respondent Municipal Judge. These acts of the petitioner
subsequent to his arrest, constitute an implied admission on
his part that here was a probable cause for the issuance of the
warrant of arrest against him. Those acts of the petitioner
constitute a waiver of whatever irregularity, if any there was,
that attended his arrest.10
WHEREFORE, the decision of the trial court dated April 20,
1967, appealed from, is affirmed. Costs against petitionerappellant. It is so ordered.
A.M. No. 71-MJ November 29, 1974
SOLEDAD AVILLAR DE MULATA, ANTONIO
ESTEBAN, and BUENAVENTURA BAYO, complainants,
vs.
JUDGE ELIAS C. IRIZARI, respondent.
RESOLUTION

ANTONIO, J.:p

fulfilled where the municipal judge examined under oath the


witnesses by asking questions that were adopted from a
These are two administrative complaints filed by Soledad
previous investigation and considered by him as sufficiently
Avillar and Antonio Esteban (Administrative Case No. L-2) searching and, which questions and the answers thereto,
and Buenaventura Bayo (Administrative Case No. L-3),
were in writing and sworn to before him prior to his issuance
respectively, against respondent Municipal Judge Elias C.
of the order of arrest. 2 Under Section 5, Rule 112, of the
Irizari of Hinatuan, Surigao del Sur. Both cases advert to
Revised Rules of Court, the accused is not entitled as a
respondent's conduct of the preliminary examination in two matter of right to be present during the preliminary
criminal cases filed with his court.
examination or to cross-examine the witnesses presented
against him before his arrest. 3 In Criminal Case No. 1074,
In Administrative Case No. L-2, respondent is charged with complainants not only failed to question in the aforesaid case
"grave abuse of discretion, favoritism and bias" for having
the validity of the preliminary examination, but also
conducted the preliminary examination in Criminal Case No. expressly waived their right to the second stage of the
1074 without giving the accused therein an opportunity to be preliminary investigation. This case was subsequently
heard because they were not granted the occasion to crosselevated to the Court of First Instance and docketed as
examine the complainant and her witnesses before issuing
Criminal Case No. L-125. On motion of the Provincial
the warrant for their arrest; that they were arrested and
Fiscal, the case against Antonio Esteban was dismissed,
detained in jail for fifteen (15) days without knowing why
while Soledad Avillar was found, after trial, guilty of the
they were arrested as neither of them were shown any
crime of Slight Physical Injuries.
warrant of arrest; and that they were "treated like criminals
of high degree" because they were first required to post bail Contrary to the claim of the complainants that they were first
for P20,000.00, "although later on thru the intervention of a required to post bail of P20,000.00 each, the amount of the
brother of theirs in the Iglesia ni Cristo," they were bailed
bail fixed for their provisional liberty as per Order of
out for P10,000.00.
respondent dated June 19, 1969 in Criminal Case No. 1074
(Exhibit "VII-A") is P5,000.00. We agree, therefore, with the
According to District Judge Otilio G. Abayan of the Court of recommendation of the Investigating Judge that on the basis
First Instance, Branch II, Lianga, Surigao del Sur, who was
of these facts, this charge should be dismissed.
designated to investigate these administrative cases,
Criminal Case No. 1074, which involved a complaint for
In Administrative Case No. L-3, respondent is charged with
"Assault Upon a Person in Authority with Physical Injuries", having arbitrarily dismiss a "Grave Coercion" case filed by
was filed by the Chief of Police of Hinatuan on July 19,
Buenaventura Bayo, a member of the Iglesia ni Cristo,
1969 against Soledad Avillar and Antonio Esteban. The
against Barrio Captain Monica Peas and fourteen (14)
Investigator found that respondent municipal judge issued
others. Complainant asserts that he was notified only two
the order for the arrest of the accused "after conducting a
days before the preliminary examination set on July 22,
preliminary examination consisting in the adoption of the
1969; that after he was asked a few questions, he was
affidavits of the prosecution witnesses and the propounding informed that the case was dismissed; that he was deprived
of additional questions to them." The requirement that the
of his right to be represented by his lawyer; and that the act
investigating judge must examine the witnesses personally,
of respondent in dismissing this case while ordering the
which examination shall be under oath and reduced to
arrest of the accused in Criminal Case No. 1074 (subject of
writing in the form of searching questions and answers, 1 is

Administrative Case No. L-2) shows his bias and prejudice


against members of the Iglesia ni Cristo.

that the offense has in fact been committed and that there is a
reasonable ground to believe that the accused committed the
same. This is in conformity with the constitutional
It was shown, however, that although the complaint was filed requirement that "no warrants shall issue but upon probable
on July 3, 1969, the preliminary examination was set for July cause, to be determined by the judge after examination under
21, 1969, but it was postponed to the following day.
oath or affirmation of the complainant and the witnesses he
Therefore, complainant had sufficient time and opportunity may produce." 4 The question as to whether "probable cause"
to consult his lawyers if he so desired.
exists or not, must, therefore, depend upon the judgment and
discretion of the judge issuing the warrant. We are not
The record also disclose that the dismissal of the case was
satisfied that in the circumstances attendant to the case,
made after respondent had propounded searching questions
respondent, in dismissing the complaint for "grave
to complainant Buenaventura Bayo (Exhibit "26") and his
coercion", has acted arbitrarily.
witnesses (Exhibits "19", "20", "21" and "22"). This
dismissal was predicated upon respondent's finding that the We do note, however, that respondent, upon accepting the
accused therein had acted without malice or criminal intent. complaint, failed to enter the same in his general docket
This conclusion is borne out by the admission of the
book, so that the case had no corresponding docket number,
complaining witness, Buenaventura Bayo, a member of the
before conducting the preliminary examination. The rules
Iglesia ni Cristo, during the preliminary examination to the
specifically require that the municipal judge must keep a
effect that he did not bury his dead child at the Roman
docket in which he shall enter all civil and criminal cases or
Catholic Cemetery of Barrio San Juan on the morning of
proceedings commenced before him. 5 It is a public record
June 14, 1969, because he was told by Monica Peas and
available during office hours for examination by any person,
Pelegrino Malinao that said cemetery is exclusively for the
upon his reasonable request, to ascertain the status of any
use of the members of said church. However, the same
given case pending therein. 6 Respondent is, therefore,
Monica Peas, with the assistance of Felix Sereno, chartered admonished to keep a faithful record of all proceedings
at their own personal expense a pump boat to bring the
before him in the dockets required by law to be kept for that
cadaver of his daughter to the Poblacion of Hinatuan for
purpose.
burial at the public cemetery on the afternoon of the same
day.
WHEREFORE, respondent is exonerated from the
aforementioned charges, with the above admonition.
It was also shown that Atty. Fermin B. Quejada, counsel of
the complaining witness, received a copy of this order of
G.R. Nos. L-25707&25753-25754 May 14, 1981
dismissal from respondent's court. Said counsel never
ANTONIO MARINAS, ANTONIO MONTANO and
bothered to file a motion for reconsideration or take any
GREGORIO RUPISAN petitioners,
action for the reinstatement of the case.
vs.
The purpose of the preliminary examination is to determine HON. ANDRES S. SIOCHI, Presiding Judge of the
whether or not there is sufficient reason to issue a warrant of Municipal Court of Pasig, Rizal, VICTORIA LASIN
arrest. Section 6 of Rule 112 of the Revised Rules of Court
VDA. DE ATIENZA and ROSARIO L. ATIENZA,
requires that the warrant of arrest shall be issued only when respondents.
the judge conducting the preliminary examination is satisfied

MELENCIO-HERRERA, J.:1wph1.t
Before us is a Petition for certiorari with Preliminary
injunction seeking to annul the proceedings held in Criminal
Cases Nos. 12943 and 12945 for Theft, and Criminal Case
No. 12944 for Grave Coercion, before the Municipal Court
of Pasig, Rizal; to annul the warrants of arrest issued in the
said cases; and to declare as unconstitutional and void
Section 5, Rule l l2 of the Rules of Court in so far as it
denies the accused the right of notice and opportunity to be
heard in the preliminary examination.
The present controversy arose out of the issuance by the
Municipal Court of Pasig, Rizal, of a Writ of Execution in
Civil Case No. 938 for Ejectment, entitled Jose C. Zulueta
vs. Gregorio Atienza. On December 13, 1965, petitioner
Antonio Marinas, Deputy Sheriff of Rizal, with his copetitioners Antonio Montano and Gregorio Rupisan enforced
said Writ of Execution by levying upon the personal
properties and chattels of private respondents Victoria Lasin
Vda. de Atienza and] Rosario L. Atienza, and taking out said
properties from their (respondents') rented house at #23
General Malvar St., Antonio Village, Pasig, Rizal.
Respondents were also ejected from said house. On the same
date, respondent Victoria Lasin Vda, de Atienza reported to
the police authorities of Pasig that her jewelry worth
P590.00 had been taken by petitioners without issuing any
receipt therefor, 1 and in connection therewith, she executed
a written Statement which was sworn to before Special
Counsel Lucila P. Alcoba. 2

On January 28, 1966, respondents re-entered the house they


had been ejected from after securing a Court Order for that
purpose. Respondent Rosario L. Atienza then discovered that
several pieces of her jewelry and other personal items, with a
total value of P1,018.00, were missing. She reported the loss
to the authorities on February 2, 1966, and her Statement
was taken. She subscribed and swore to the same before
respondent Municipal Judge Andres S. Siochi. 3

Case No. 12945 for Theft does not show the jurat on its face, Section 87, paragraph 4 of the Judiciary Act of 1948 (R.A.
but respondents state that it was also attested to by Lt.
296), as amended by Republic Acts Nos. 2613 and 3828,
Lontoc before respondent Judge and that this appears on the provides. 1wph1.t
dorsal side of the Complaint. On February 8, 1966, warrants
Municipal judges in the capitals of provinces
for the arrest of petitioners were issued by respondent Judge
and judges of city courts shall have like
in all three cases 9 after preliminary examination conducted
jurisdiction as the Court of First Instance to.
by him in Criminal Cases Nos. 12943 and 12944, and by
try parties charged with an offense
Special Counsel Lucila P. Alcoba in Criminal Case No.
committed within their respective
12945.
On February 3, 1966, respondents, armed with a Court Order
jurisdiction, in which the penalty provided
authorizing them to enter the premises of the said house, did Petitioners took exception to the issuance of the warrants of
by law does not exceed prision correccional
so again to get their remaining unlevied properties. They
arrest against them and instituted the present Petition raising
or imprisonment for not more than six years
claimed, however, that on the said date petitioners and their the following issues: 1wph1.t
or fine not exceeding six thousand pesos or
companions forcibly compelled them to deliver the unlevied
both, and in the absence of the district judge,
1. When Section 87, Republic Act No. 296,
personal properties found therein, hauled said articles into a
shall have like jurisdiction within the
as amended by Republic Act No. 3828,
truck and left. Private respondents reported the incident to
province as the Court of First Instance to
4
provides that when the penalty provided by
the police authorities at Pasig. Victoria Lasin executed a
hear application for bail.
5
law does not exceed prision correccional,
Statement alleging that the personal properties forcibly
then the Municipal Judge in the capitals of
Pursuant to the foregoing provision, both Criminal Cases
taken from them by petitioners, amounting to P2,645.00,
the provinces shall have "like jurisdiction as Nos. 12943 and 12945, for Theft of P590.00 and P1,018.00,
were not included in the levy. Her son, Tranquilino Atienza,
6
the Court of First Instance" to try the
respectively, fall under the concurrent jurisdiction of the
also executed an Affidavit corroborating her declaration.
offense, does the Municipal Court in such
Municipal Court of Pasig and the Court of First Instance of
Both Statements were subscribed and sworn to before
cases follow the procedure for Municipal
Rizal, as the penalty provided for said crimes, pursuant to
respondent Judge.
Courts or that for Courts of First Instance?
Article 309 (3) of the Revised Penal Code, is prision
On February 7, 1966, two separate charges for Theft,
correccional in its minimum and medium periods.
2.
Is
preliminary
investigation
a
part
of
due
docketed as Criminal Cases Nos. 12943 and 12945, were
process?
Criminal Case No. 12944 for Grave Coercion, with a
filed against petitioners and Carlos Quintana before the
penalty, under Article 286 of the Revised Penal Code, of
Municipal Court of Pasig, Rizal, respondent Judge,
3. Can there be due process without the
arresto mayor and a fine not exceeding P500.00, also falls
presiding. 7 A Complaint for Grave Coercion Crime Case No.
presence of the accused during the
under the concurrent jurisdiction of the Municipal Court of
12944) was also lodged against petitioners and three Does on
10
8
preliminary
investigation.
Pasig and the Court of First Instance of Rizal. 11
the same date. The three Complaints were filed by Lt. Jose
S. Lontoc, Chief of the Criminal Investigation Section of the
On February 23, 1966, we required respondents to file an
It is petitioners' submission that because of this concurrent
Police Department of Pasig, Rizal, for and on behalf of the
Answer,
and
ordered
the
issuance
of
a
Writ
of
Preliminary
jurisdiction, a Municipal Court acts in reality as a Court of
Chief of Police. These Complaints contained an annotation
First Instance and, consequently, it cannot issue warrants of
on the lower left hand corner reading: "APPROVED AFTER injunction restraining respondent Judge from enforcing the
warrants
of
arrest
issued
in
Criminal
Cases
Nos.
12943,
arrest without first giving the accused a chance to be heard;
PRELIMINARY EXAMINATION: (SGD) Lucila P. Alcoba,
12944 and 12945.
and that the Information filed should carry a certification
Special Counsel." The Complaints in Criminal Cases Nos.
under oath that defendant was given a chance to appear in
12943 and 12944 for Theft and Grave Coercion,
person at said examination and investigation. Continuing,
respectively, were subscribed and sworn to by Lt. Jose S.
petitioners argue that since Special Counsel Lucila P. Alcoba
Lontoc before respondent Judge. The Complaint in Criminal

of the Office of the Provincial Fiscal of Rizal, in Criminal


Cases Nos. 12943 and 12945, merely signed the Complaints
for these two cases below the notation, "Approved after
preliminary examination", her failure to make the
certification under oath to the effect that the accused were
given a chance to appear in person or by counsel at said
examination and investigation, was violative of the due
process clause, and, therefore, the warrants of arrest issued
thereafter should be quashed.
Section 14, Rule 112 of the Rules of Court, relied upon by
petitioners, provides: 1wph1.t
Section 14. Preliminary examination and
investigation by provincial or city fiscal or
by state attorney in cases cognizable by the
Court of First Instance. -Except when an
investigation has been conducted by a judge
of first instance, justice of the peace or other
officer in accordance with the provisions of
the preceding sections, no information for an
offense cognizable by the Court of First
Instance shall be filed by the provincial or
city fiscal, or state attorney, without first
giving the accused a chance to be heard in a
preliminary investigation conducted by him
or by his assistant by issuing a
corresponding subpoena. lf the accused
appears, the investigation shall be conducted
in his presence and he shall have the right to
be heard, and to cross-examine the
complainant and his witnesses. and to
adduce evidence in his favor. If he cannot be
subpoenaed, or if subpoenaed he does not
appear before the fiscal, the investigation
shall proceed without him.
The fiscal or state attorney shall certify
under oath in the information to be filed by
him that the defendant was given a chance to

appear in person or by counsel at said


investigation and examination.

What was conducted by the respondent Judge in these cases


is the preliminary examination before the issuance of a
warrant of arrest pursuant to section 1, Rule 112. The 1935
On the other hand, respondents contend that the governing
Constitution, in section l (3), Article III provides that no
proviso is the second paragraph of Section 10, Rule 112,
warrant shall be issued but upon probable cause to be
referring to the right of an accused to preliminary.
determined by the Judge after examination of witnesses
investigation after arrest, reading: "in cases triable in the
under oath or affirmation of the complaint and the witnesses
municipal or city courts. the accused shall not be entitled as he may produce. Conformably thereto, Section 87, paragraph
a matter of right to a preliminary investigation in accordance 3, of the Judiciary Act, as amended by Republic Act No.
with this section" and that this rule applies whether the case 3828, provides that: before a Municipal Judge may issue a
is within the exclusive original jurisdiction of the Municipal warrant of arrest, the following conditions must first be
Court or within its concurrent jurisdiction with the Court of fulfilled: (1) he must examine the witness or witnesses
First Instance.
personally; (2) the examination must be under oath; and (3)
the examination must be reduced to writing in the form of
The issue of whether or not an accused is entitled to appear
searching questions and answers.
and present evidence in a preliminary investigation in cases
falling within the concurrent jurisdiction of the Municipal
These requirements have been met in the three criminal
Court and the Court of First Instance has been squarely
cases involved herein. As explained by respondent Judge in
resolved in the negative by this Court, speaking through Mr. his Answer: 1wph1.t
Justice Claudio Teehankee, in the cases of People vs.
Abejuela and People vs. Endan, 12 reiterated in the case of
Before the warrants of arrest were issued by
Banzon vs. Cabato, 64 SCRA 419 (1975), which decisively
the respondent Judge in Criminal Cases Nos.
held, that even though the offense be one falling within the
12944 and 12945 (actually 12943 and
concurrent jurisdiction of the City Courts and Courts of First
12944), he first conducted, on February 8,
Instance, the accused is not entitled as a matter of right to be
1966, the necessary preliminary examination
heard in a preliminary investigation under section 10, Rule
required by Section l of Rule 112 by
112. The reason is because the case goes to trial already after
adopting, as his own questions, and by
13
the arrest of the accused and his delivery to the Court.
asking the complainants and their witnesses,
"The ensuing trial on the merits takes the place of
the same or Identical questions asked of
preliminary investigation, without needless waste or
them by the Investigating Police Officer in
duplication of time and effort, and a final verdict on the
their written statements before the said
innocence (or guilt) of the accused is thereupon rendered,
Police Investigator, Annexes "4", "5", and '7'
rather than an inconclusive dismissal of the charge by the
hereof, and thereafter the respondent Judge
fiscal in a preliminary investigation which would not
required them (the complainants and their
constitute jeopardy." 14 To reiterate and to re-state the rule,
witnesses) to subscribe before and make
therefore, there is no right of preliminary investigation in
oath to him as to the truth of the answers
cases triable by inferior Courts, without distinction as to
given by them to the Police Investigator as
whether such case be of their exclusive or concurrent
shown by the fact that in said Annexes "4",
15
jurisdiction.
"5", and "7", the deponents signed their
respective names twice, once before the

Investigating Police Officer and the second


therefore, must to a great degree depend
time before the respondent Judge who also
upon the Judge making the investigation. At
required them to take the jurat to the oath,
any rate, the court a quo found that
thereby complying to the requirements of
respondent Judge was satisfied that the
Section 87 of the Judiciary Act of 1948, as
questions and answers contained in the
amended, providing therein that 'no warrant
sworn statements taken by T-Sgt. Patosa
of arrest shall be issued by any justice of the
partake of the nature of his searching
peace in any criminal case filed with him
questions and answers as required by law,'
unless he first examines the witness or
so the respondent Judge adopted them. 17
witnesses personally, and the examination
shall be under oath and reduced to writing in In the language of this Court in De Mulata vs. Irizari, 61
SCRA 210, 213 (1974): 1wph1.t
the form of searching questions and
16
answers.
The requirement that the investigating judge
must examine the witnesses personally,
By "searching questions and answers" is meant: 1wph1.t
which examination shall be under oath and
the term "searching questions and answers"
reduced to writing in the form of searching
means only, taking into consideration the
questions and answers, is fulfilled where the
purpose of the preliminary examination
municipal judge examined under oath the
which is to determine 'whether there is a
witnesses by asking questions that were
reasonable ground to believe that an offense
adopted from a previous investigation and
has been committed and the accused is
considered by him as sufficiently searching
probably guilty thereof so that a warrant of
and, which questions and the answers
arrest may be issued and the accused held
thereto, were in writing and sworn to before
for trial', such questions as have tendency to
him prior to his issuance of the order of
show the commission of a crime and the
arrest.
perpetrator thereof. What would be
searching questions would depend on what In regards Criminal Case No. 12945 for Theft, respondent
Judge had this to say: 1wph1.t
is sought to be inquired into, such as: the
nature of the offense, the date, time, and
As regards Criminal Case No. 12943
place of its commission, the possible
(actually 12945), although, the respondent
motives for its commission the subject, his
Judge did not take the oath of the
age, education, status, financial and social
complainant and her witness on the
circumstances, his attitude toward the
statement given by them to the Police
investigation, social attitudes, opportunities
investigator, Annexes "I" and "2" hereof,
to commit the offense; the victim, his age,
Special Counsel Lucile P. Alcoba of the
status, family responsibilities, financial and
Office of the Provincial Fiscal of Rizal
social circumstances, characteristics, etc.
conducted the necessary preliminary
The points that are the subject of inquiry
examination required by Section l of Rule l l
may differ from case to case. The questions,

2 in that, as can be seen from said Annexes


'I' and '2', she asked the same or Identical
questions appearing in said annexes to the
deponents and adopted the questioning of
the Police Investigator as her own
interrogations of the complainant and her
witness, and thereafter she required them to
subscribe their respective names and to
swear before her as to the truth of the
answers given by them to each and every
question appearing in said Annexes '1' and
'2' and, although there was no certification,
in the exact form required by law, by Special
Counsel Lucile P. Alcoba that she conducted
the required preliminary examination of the
complainant and her witness, it is admitted
that her certification in the body of the
complaint stating "Approved after
preliminary examination", accompanied by
the oath taken by her before the respondent
Judge after making such certification, is a
substantial compliance to the requirements
of the law although it can be said that the
same is somewhat defective in form. (pp.
46-47, Rollo)
xxx xxx xxx
From the foregoing explanation, lt may be deduced that
respondent Judge was satisfied that the questions and
answers in a previous investigation by Special Counsel
Alcoba partook of the nature of his searching questions and
answers and made them his own. As held in Luna vs. Plaza,
supra, the Judiciary Act as amended by Republic Act No.
3828, does not prohibit the Municipal Judge from adopting
the questions asked by a previous investigator. For, in the
final analysis, whether or not probable cause exists or not
depends upon the judgment and discretion of the Judge
issuing the warrant of arrest (De Mulata vs. Irizari, supra). In
Criminal Case No. 12945 below, respondent Judge had

convinced himself that probable cause existed before he


issued the warrant of arrest. Under the attendant
circumstances, respondent Judge may not be said to have
acted arbitrarily.
We reiterate, however, the reminder in the Luna case
(supra), reading: 1wph1.t
We wish to stress, however, that what has
been stated in this opinion is certainly not
intended to sanction the return to the former
practice of municipal judges of simply
relying upon affidavits or sworn statements
that are made to accompany the complaints
that are filed before them, in determining
whether there is a probable cause for the
issuance of a warrant of arrest. That practice
is precisely what is sought to be voided by
the amendment of Section 87 (c) of
Republic Act. 296 (Judiciary Act of 1948)
which requires that before a municipal judge
issues a warrant of arrest he should first
satisfy himself that there is a probable cause
by examining the witnesses personally, and
that the examination must be under oath and
reduced to writing in the form of searching
questions and answers. It is obvious that the
purpose of this amendment is to prevent the
issuance of a warrant of arrest against a
person based simply upon affidavits of
witnesses who made, and swore to, their
statements before a person or persons other
than the judge before whom the criminal
complaint is filed. We wish to emphasize
strict compliance by municipal or city
judges of the provision of Section 87(c) of
the Judiciary Act of 1948, as amended by
Republic Act 3828, in order to avoid
malicious and/or unfounded criminal

prosecution of persons. (Luna vs. Plaza,


supra p. 323)
Petitioners further maintain that Section 5 of Rule 112 of the
Rules of Court, in so far as it authorizes the Municipal Court
to conduct a preliminary examination before the issuance of
a Warrant of Arrest without previous notice to the accused, is
unconstitutional as it violates the guarantee of equal
protection of the laws, and Section l (15), Art. III of the 1935
Constitution which states, "No person shall be held to
answer for a criminal offense without due process of law."
Section 5, Rule l l 2 provides: 1wph1.t
The municipal, the city judge, the fiscal or
the municipal mayor who conducts the
preliminary examination as provided in
these rules must take under oath, either in
the presence or in the absence of the
accused, the testimony of the complainant
and his witnesses. The testimony of the
complainant and his witnesses shall be
reduced to writing and signed by them.

From Section 5 of Rule 112, supra, it is clear that, unlike in


the preliminary investigation proper, an accused is not
entitled as a matter of right to be present, during the
preliminary examination nor to cross-examine the witnesses
presented against him before his arrest, the purpose of said
examination being merely to determine whether or not there
is sufficient reason to issue a warrant of arrest. 18 Section l
(3), Article III of the 1935 Constitution commanding the
determination of probable cause prior to issuance of a
warrant arrest, requires no notice to an accused. A
preliminary examination is generally a proceeding ex-parte
in which the person charged has no right to participate or be
present. The right to confrontation of witnesses neither
applies to a preliminary hearing. The reason therefor has
been explained thus: 1wph1.t
... It can not be seriously contended that an
accused person has a right to be present
during this stage of the proceedings. To hold
that he had such a right and to reverse a
judgment of conviction on this ground
would have the effect of destroying the very
purpose of that part of the criminal law. lt
would be against public policy. lt is
frequently essential that such investigations
be kept secret and that the accused should
have no suspicion of any complaint against
him, otherwise he might avoid punishment
for his crime by escaping before arrest. (U.S.
vs. Grant, et al., 18 Phil. 122, 147)

The preliminary examination referred to is defined, under


Section l of Rule 112, as a previous inquiry or examination
made before the arrest of the accused by a Judge or officer
authorized to conduct the same, with whom a Complaint or
Information has been filed imputing the commission of an
offense cognizable by the Court of First Instance, for the
purpose of determining whether there is a reasonable ground
to believe that an offense has been committed and the
accused is probably guilty thereof, so that a warrant of arrest
... it is often the only means of discovering
may be issued and the accused held for trial. This section
the persons who may reasonably be charged
does not refer to the preliminary investigation proper
with the crime so as to enable the fiscal to
provided for under Section 10, Rule 112, in which the
prepare his complaint or information, ...
accused is given access to the testimony and evidence
(People vs. Badilla, 48 Phil., 719, 731)
presented against him at the preliminary examination, and to
present evidence if he so desires.
While section l (3) Art. III of the 1935 Constitution does
require, before the issuance of a warrant of arrest, the
determination of probable cause by the Judge after

examination of witnesses he may produce, the curtailment of


the presence of an accused during that preliminary
examination entails no infringement of the constitutional
right to due process of law nor to equal protection of the
laws. Thus, in Manzano vs. Villa, 19 this Court categorically
held: 1wph1.t

which at times out- lasts the period of the


penalty provided by law for the offense,
besides the mental anguish suffered in
protracted litigations, are eliminated with the
assurance of a speedy and expeditious trial
for the accused, upon his arraignment
(without having to undergo the second stage
of the preliminary investigation), and of a
prompt verdict on his guilt or innocence. On
the other hand, the so-called first stage of
preliminary investigation or the preliminary
examination, conducted by the duly
authorized officer, as borne out by the
examination and sworn written statements of
the complainants and their witnesses,
generally offices to establish the existence of
reasonable ground to charge the accused
with having committed the offense
complained of.

WHEREFORE, the Petition is hereby denied and the Writ of


Preliminary Injunction heretofore issued is hereby lifted.
Costs against petitioners.
SO ORDERED.

Roan v. Gonzales, 145 SCRA 687 (1986)


Roan v. Gonzales, 145 SCRA 687 (1986) F: The challenged
SW was issued by the resp. judge on 5/10/84. The
petitioner''s house was searched 2 days later but none of the
articles listed in the warrant was discovered. The officers
conducting the search found 1 colt Magnum revolver & 18
live bullets w/c they confiscated. They are now the bases of
the charge against the petitioner. RULING: Search warrant
issued by resp. judge is hereby declared null and void and
accordingly set aside. The petitioner claims that no
depositions were taken by the resp. judge in accordance w/
Rule 126, Sec. 4 of the ROC, but this is not entirely true.
Depositions were taken of the complainant''s 2 witnesses in
addition to the affidavit executed by them. It is correct to
say, however, that the complainant himself was not subjected
to a similar interrogation. By his own accounts, all that resp.
Attention should also be called to the fact that neither the
judge did was question Capt. Quillosa on the contents of his
1935 nor the 1973 Constitution requires the holding of a
affidavit only "to ascertain among others, if he knew and
preliminary investigation. lt is settled doctrine that the right
understood the same," and only bec. "the application was not
hereto is of statutory character and may be invoked only
yet subscribed and sworn to." The suggestion is that he
when specifically created by statute. 20 lt is not a fundamental would not have asked any questions at all if the affidavit had
right and may be waived expressly or by silence. 21
already been completed when it was submitted to him. In
any case, he did not ask his own searching questions. He
limited himself to the contents of the affidavit. He did not
In a nutshell, the proceedings in these three criminal cases
take the applicant''s deposition in writing and attach them to
conformed to law and jurisprudence. But even conceding
the record, together w/ the affidavit presented to him. Such
that petitioners were entitled to a preliminary investigation,
written deposition is necessary in order that the Judge may
the proper forum before which absence thereof should have be able to properly determine the existence or non-existence
Neither can the withholding of the right of preliminary
been raised and ventilated was in the trial Court, not in an
of the probable cause, to hold liable for perjury the person
investigation from the accused in cases triable by inferior
appellate Court because the absence of preliminary
giving it if it will be found later that his declarations are
Courts be termed ' an unjust or unfair distinction, as
investigation does not go to the jurisdiction of the Court but false. (Mata v. Bayona.) The applicant was asking for the
issuance of the SW on the basis of mere hearsay and not of
explained in People vs. Abejuela, supra: 1wph1.t
merely to the regularity of the proceedings, and bearing in
mind that preliminary investigation can be waived, as in fact, info. personally known to him. His application, standing
alone, was insufficient to justify the issuance of the warrant
... The loss of time entailed in the conduct of it is frequently waived. 22
sought.
preliminary investigations, with the
consequent extension of deprivation of the
accused's liberty, in case he fails to post bail,
The preliminary examination conducted by
the municipal judge was essentially a
procedural matter and no substantial rights
of the accused were violated just because he
had not been given an opportunity to
examine the witnesses against him. The first
stage of the preliminary investigation is 'not
the occasion for full and exhaustive
presentation of parties' evidence but only
such as may engender well-grounded belief
that an offense has been committed and that
the accused is probably guilty thereof' The
proceeding is usually held ex-parte, for
under section 5 of Rule 112 all that is
required is for the judge conducting such
examination to 'take under oath, either in the
presence or absence of the accused, the
testimony of the complainant and his
witnesses,' said testimony to be reduced to
writing and signed by them. Hence, the
absence of the accused during the
preliminary examination was not a denial of
due process of law.

It was, therefore, necessary for the witnesses themselves, by


their own personal info., to establish the applicant''s claims.
Even assuming then that it would have suffied to take the
deposition only of the witnesses and not of the applicant
himself, there is still the question of the sufficiency of their
depositions. A study of the deposition taken from witnesess
Esmael Morada and Jesus Tohilida, who both claimed to be
"intelligence informers," shows that they were in the main a
mere restatement of their allegations in their affidavits,
except that they were made in the form of answers to the
questions put to them by the resp. judge. One may well
wonder why it did not occur to the resp. judge to ask how the
witness could be so certain even as to the caliber of the guns,
or how far he was from the window, or whether it was on the
first floor or second floor, or why his presence was not
noticed at all, or if the acts related were really done openly,
in the full view of the witnesses, considering that these acts
were against the law. These would have been judicious
questions but they were injudiciously omitted. Instead, the
declaration of the witnesses were readily accepted and the
warrant sought was issued forthwith. SOL-GEN ARGUES
THAT THE PETITIONER WAIVED WHATEVER
DEFECT WHEN THE PETITIONER VOLUNTARILY
SUBMITTED TO THE SEARCH AND MANIFESTED HIS
CONFORMITY IN WRITING. We do not agree. What we
see here is pressure exerted by the military authorities, who
practically coerced the petitioner to sign the supposed waiver
as guaran
Roan v. Gonzales, 145 SCRA 687 ( 1986)

to a similar interrogation. By his own accounts, all that resp.


judge did was question Capt. Quillosa on the contents of his
affidavit only to ascertain among others, if he knew and
understood the same, and only bec. the application was not
yet subscribed and sworn to. The suggestion is that he
would not have asked any questions at all if the affidavit had
already been completed when it was submitted to him. In
any case, he did not ask his own searching questions.
He limited himself to the contents of the affidavit. He did not
take the applicants deposition in writing and attach them to
the record, together w/ the affidavit presented to him. Such
written deposition is necessary in order that the Judge may
be able to properly determine the existence or non-existence
of the probable cause, to hold liable for perjury the person
giving it if it will be found later that his declarations are
false. (Mata v. Bayona.) The applicant was asking for the
issuance of the SW on the basis of mere hearsay and not of
info. personally known to him. His application, standing
alone, was insufficient to justify the issuance of the warrant
sought. It was, therefore, necessary for the witnesses
themselves, by their own personal info., to establish the
applicants claims.
Even assuming then that it would have suffied to take the
deposition only of the witnesses and not of the applicant
himself, there is still the question of the sufficiency of their
depositions. A study of the deposition taken from witnesess
Esmael Morada and Jesus Tohilida, who both claimed to be
F: The challenged SW was issued by the resp. judge on
intelligence informers, shows that they were in the main a
5/10/84. The petitioners house was searched 2 days later but
mere restatement of their allegations in their affidavits,
none of the articles listed in the warrant was discovered. The
except that they were made in the form of answers to the
officers conducting the search found 1 colt Magnum revolver
questions put to them by the resp. judge. One may well
& 18 live bullets w/c they confiscated. They are now the
wonder why it did not occur to the resp. judge to ask how the
bases of the charge against the petitioner. RULING: Search
witness could be so certain even as to the caliber of the guns,
warrant issued by resp. judge is hereby declared null and
or how far he was from the window, or whether it was on the
void and accordingly set aside. The petitioner claims that no
first floor or second floor, or why his presence was not
depositions were taken by the resp. judge in accordance w/
noticed at all, or if the acts related were really done openly,
Rule 126, Sec. 4 of the ROC, but this is not entirely true.
in the full view of the witnesses, considering that these acts
Depositions were taken of the complainants 2 witnesses in
were against the law. These would have been judicious
addition to the affidavit executed by them. It is correct to
questions but they were injudiciously omitted. Instead, the
say, however, that the complainant himself was not subjected

declaration of the witnesses were readily accepted and the


warrant sought was issued forthwith. SOL-GEN ARGUES
THAT THE PETITIONER WAIVED WHATEVER
DEFECT WHEN THE PETITIONER VOLUNTARILY
SUBMITTED TO THE SEARCH AND MANIFESTED HIS
CONFORMITY IN WRITING. We do not agree. What we
see here is pressure exerted by the military authorities, who
practically coerced the petitioner to sign the supposed waiver
as guaran

** Mata v Bayona 128 SCRA 388 (1984)


Facts: Soriano Mata was accused under Presidential Decree
(PD) 810, as amended by PD 1306, the information against
him alleging that Soriano Mata offered, took and arranged
bets on the Jai Alai game by selling illegal tickets known as
Masiao tickets without any authority from the Philippine
Jai Alai & Amusement Corporation or from the government
authorities concerned. Mata claimed that during the hearing
of the case, he discovered that nowhere from the records of
the said case could be found the search warrant and other
pertinent papers connected to the issuance of the same, so
that he had to inquire from the City Fiscal its whereabouts,
and to which inquiry Judge Josephine K. Bayona, presiding
Jufe of the City Court of Ormoc replied, it is with the
court. The Judge then handed the records to the Fiscal who
attached them to the records. This led Mata to file a motion
to quash and annul the search warrant and for the return of
the articles seized, citing and invoking, among others,
Section 4 of Rule 126 of the Revised Rules of Court. The
motion was denied by the Judge on 1 March 1979, stating
that the court has made a thorough investigation and
examination under oath of Bernardo U. Goles and Reynaldo
T. Mayote, members of the Intelligence Section of 352nd PC
Co./Police District II INP; that in fact the court made a
certification to that effect; and that the fact that documents
relating to the search warrant were not attached immediately
to the record of the criminal case is of no moment,
considering that the rule does not specify when these
documents are to be attached to the records. Matas motion
for reconsideration of the aforesaid order having been

denied, he came to the Supreme Court, with the petition for


certiorari, praying, among others, that the Court declare the
search warrant to be invalid for its alleged failure to comply
with the requisites of the Constitution and the Rules of
Court, and that all the articles confiscated under such
warrant as inadmissible as evidence in the case, or in any
proceedings on the matter.
Issue: Whether the judge must before issuing the warrant
personally examine on oath or affirmation the complainant
and any witnesses he may produce and take their depositions
in writing, and attach them to the record, in addition to any
affidavits presented to him.
Held: Under the Constitution no search warrant shall issue
but upon probable cause to be determined by the Judge or
such other responsible officer as may be authorized by law
after examination under oath or affirmation of the
complainant and the witnesses he may produce. More
emphatic and detailed is the implementing rule of the
constitutional injunction, The Rules provide that the judge
must before issuing the warrant personally examine on oath
or affirmation the complainant and any witnesses he may
produce and take their depositions in writing, and attach
them to the record, in addition to any affidavits presented to
him. Mere affidavits of the complainant and his witnesses
are thus not sufficient. The examining Judge has to take
depositions in writing of the complainant and the witnesses
he may produce and to attach them to the record. Such
written deposition is necessary in order that the Judge may
be able to properly determine the existence or nonexistence
of the probable cause, to hold liable for perjury the person
giving it if it will be found later that his declarations are
false. We, therefore, hold that the search warrant is tainted
with illegality by the failure of the Judge to conform with the
essential requisites of taking the depositions in writing and
attaching them to the record, rendering the search warrant
invalid.

moved to quash the warrant but his motion was denied.


HELD: The statements made in the affidavits are mere
conclusions of law and do not satisfy the requirement of
probable cause. The language used is all embracing as to
include all conceivable words and equipment of petitioner
regardless of whether they are legal or illegal. The search
warrant under consideration was in the nature of a general
warrant which is objectionable.
Corro v. Lising Philippine Times conclusions of law of
military officers will not satisfy probable cause requirement
for issuance of search warrants.

5. Typewriters, duplicating machines,


mimeographing and tape recording
machines, video machines and tapes
which have been used and are being used as instrument and
means of committing the crime of inciting to sedition
defined and penalized under Article 142 of the Revised Penal
Code, as amended by PD 1835 ... (p. 24, Rollo)

G.R. No. L-69899 July 15, 1985


ROMMEL CORRO, petitioner,
vs.
HON. ESTEBAN LISING Presiding Judge, Regional
Trial Court, Quezon City, Branch XCV HON. REMIGIO
ZARI Regional Trial Court, Quezon City, Branch 98;
CITY FISCAL'S OFFICE, Quezon City; LT. COL.
BERLIN A. CASTILLO and 1ST LT. GODOFREDO M.
IGNACIO, respondents,
Reynaldo L. Bagatsing for petitioner.
RELOVA, J.:
On September 29, 1983, respondent Regional Trial Court
judge Esteban Lising of Quezon City, upon application filed
by Lt. Col. Berlin Castillo of the Philippine Constabulary
Criminal Investigation Service, issued Search Warrant No.
Q-00002 authorizing the search and seizure of
1. Printed copies of Philippine Times;

Corro v. Lising 137 SCRA 341 (1985)


Corro v. Lising 137 SCRA 341 (1985) F: Respondent Judge
issued a search warrant for the seizure of articles allegedly
used by petitioner in committing the crime of sedition.
Seized were printed copies of the Philippine Times,
newspaper dummies, typewriters, mimeographing machines
and tape recorders, video machines and tapes. The petitioner

4. Subversive documents, articles, printed


matters, handbills, leaflets, banners;

2. Manuscripts/drafts of articles for


publication in the Philippine Times;
3. Newspaper dummies of the Philippine
Times;

On November 6, 1984, petitioner filed an urgent motion to


recall warrant and to return documents/personal properties
alleging among others that:
2. ... the properties seized are typewriters,
duplicating machines, mimeographing and
tape recording machines, video machines
and tapes which are not in any way,
inanimate or mute things as they are,
connected with the offense of inciting to
sedition.
3. More so, documents or papers seized
purporting to do the body of the crime has
been rendered moot and academic due to the
findings of the Agrava Board that a military
conspiracy was responsible for the slaying
of the late Senator Benigno Aquino, Jr. on
August 21, 1983 at the Manila International
Airport. The Agrava Board which has the
exclusive jurisdiction to determine the facts
and circumstances behind the killing had
virtually affirmed by evidence testamentary
and documentary the fact that soldiers killed
Benigno Aquino, Jr.
4. More so, the grave offense of libel, RTC,
Q.C. Branch XCV has dismissed said case

against the accused on all documents


Times at 610 Mezzanine Floor, Gochengco Building, T.M.,
pertinent and more so as we repeat, rendered Kalaw, Ermita, Manila.
moot and academic by the recent Agrava
In Our Resolution of February 19, 1985, respondents were
Report. (p. 27, Rollo)
required to file their comment. The plea for temporary
On January 28, 1985, respondent Judge Lising denied the
restraining order was granted and respondents City Fiscal's
motion in a resolution, pertinent portions of which state:
Office of Quezon City, Lt. Col. Berlin Castillo and 1st Lt.
Godofredo Ignacio were enjoined from introducing as
... The said articles presently form part of
evidence for the state the documents/properties seized under
the evidence of the prosecution and they are Search Warrant No. Q-00002 in Criminal Cage No. Q-29243
not under the control of the prosecuting arm (Sedition case against petitioner), pending before the
of the government. Under these
Regional Trial Court of Quezon City, Branch 98, effective
circumstances, the proper forum from which immediately and continuing until further orders from the
the petition to withdraw the articles should
Court.
be addressed, is the Office of the City Fiscal,
Quezon City and not with this Branch of the Respondents would have this Court dismiss the petition on
Court. It is to be further noted that it is not
the ground that (1) the present action is premature because
even with this Branch of the Court that the
petitioner should have filed a motion for reconsideration of
offense of inciting to sedition is pending. (p respondent Judge Lising's order of January 28, 1985; (2)
29, Rollo)
probable cause exists justifying the issuance of a search
warrant; (3) the articles seized were adequately described in
Hence, this petition for certiorari and mandamus, with
the search warrant; (4) a search was conducted in an orderly
application for preliminary injunction and restraining order
manner; (5) the padlocking of the searched premises was
to enjoin respondent Regional Trial Court, National Capital with the consent of petitioner's wife; (6) the findings of the
Region, Branch 98 from proceeding with the trial of
Agrava Board is irrelevant to the issue of the validity of the
Criminal Case No. S3-Q-29243, praying (a) that Search
search warrant; (7) press freedom is not an issue; and, (8) the
Warrant No. Q-00002 issued by respondent Judge Esteban
petition is barred by laches.
M. Lising be declared null and void ab initio and that a
mandatory injunction be issued directing respondents City
There is merit in the petition.
Fiscal's Office of Quezon City and Lt. Col. Berlin Castillo
and 1st Lt. Godofredo Ignacio jointly and severally to return Respondents contend that petitioner should have filed a
immediately the documents/properties illegally seized from motion for reconsideration of the order in question before
herein petitioner and that final injunction be issued enjoining coming to Us. This is not always so. When the questions
raised before the Supreme Court are the same as those which
respondents City Fiscal's Office of Quezon City, Lt. Col.
were squarely raised in and passed upon by the lower court,
Castillo and 1st Lt. Ignacio from utilizing said
the filing of the motion for reconsideration in said court
documents/properties as evidence in Criminal Case No.
before certiorari can be instituted in the Supreme Court is no
29243; and (b) that respondent PC-CIS officers Lt. Col.
Berlin A. Castillo and lst Lt. Godofredo Ignacio be directed longer a pre-requisite. As held in Bache & Co. (Phil.), Inc.
vs. Ruiz, 37 SCRA 823, (t)he rule requiring the filing of a
to reopen the padlocked office premises of the Philippine
motion for reconsideration before an application for a writ of

certiorari can be entertained was never intended to be


applied without considering the circumstances. The rule does
not apply where, the deprivation of petitioners' fundamental
right to due process taints the proceeding against them in the
court below not only with irregularity but also with nullity."
Likewise, in Pajo, et al. vs. Ago, et al., 108 Phil. 905 and in
Gonzales vs. Court of Appeals, 3 SCRA 465, this Court ruled
that "it is only when questions are raised for the first time
before the high court in a certiorari case that the writ shall
not issue, unless the lower court had first been given an
opportunity to pass upon the same." Further, in the case of
Matute vs. Court of Appeals, 26 SCRA 768, We held that
"while as a matter of policy a motion for reconsideration in
the lower court has often been considered a condition sine
qua non for the granting of a writ of certiorari, this rule does
not apply where the proceeding in which the error occurred
is a patent nullity or where 'the deprivation of petitioner's
fundamental right to due process ... taints the proceeding
against him in the court below not only with irregularity but
with nullity (Luzon Surety Co. v. Marbella et al., L-16038,
Sept. 30, 1960), or when special circumstances warrant
immediate and more direct action. ..." The records of this
petition clearly disclose that the issues herein raised have
already been presented to and passed upon by the court a
quo.
Section 3, Article IV of the 1973 Constitution provides:
SEC. 3. ...no search warrant or warrant of
arrest issue except upon probable cause to
be determined by the judge, or such other
responsible officer as may be authorized by
law, after examination under oath or
affirmation of the complainant and the
witnesses he may produce, and particularly
describing the place to be searched and the
persons or things to be seized.
and, Section 3, Rule 126 of the New Rules of Court, states
that:

SEC. 3. Requisites for issuing search


Penal Code as amended by Presidential
warrant. A search warrant shall not issue
Decree No. 1835; (p. 22, Rollo)
but upon probable cause in connection with
one specific offense to be determined by the and, the affidavit of Lt. Ignacio reads, among others
judge or justice of the peace after
... the said periodical published by Rommel
examination under oath or affirmation of the
Corro, contains articles tending to incite
complainant and the witnesses he may
distrust and hatred for the Government of
produce, and particularly describing the
the Philippines or any of its duly constituted
place to be searched and the persons or
authorities. (p. 23, Rollo)
things to be seized.
Probable cause may be defined as "such reasons, supported
by facts and circumstances, as will warrant a cautious man in
the belief that his actions, and the means taken in
prosecuting it, are legally just and proper (Burton vs. St.
Paul, M & M. Ry. Co., 33 Minn. 189, cited in U.S. vs.
Addison, 28 Phil. 566)." Thus, an application for search
warrant must state with particularly the alleged subversive
materials published or intended to be published by the
publisher and editor of the Philippine Times, Rommel Corro.
As We have stated in Burgos, Sr. vs. Chief of Staff of the
Armed Forces of the Philippines, 133 SCRA 800, "mere
generalization will not suffice." A search warrant should
particularly describe the place to be searched and the things
to be seized. "The evident purpose and intent of this
requirement is to limit the things to be seized to those, and
only those, particularly described in the search warrant- to
leave the officers of the law with no discretion regarding
what articles they should seize, to the end that unreasonable
searches and seizures may not be committed, that abuses
may not be committed Bache & Co. Phil. Inc. vs, Ruiz,
supra)." The affidavit of Col. Castillo states that in several
issues of the Philippine Times:
... we found that the said publication in fact
foments distrust and hatred against the
government of the Philippines and its duly
constituted authorities, defined and
penalized by Article 142 of the Revised

The above statements are mere conclusions of law and will


not satisfy the requirements of probable cause. They can not
serve as basis for the issuance of search warrant, absent of
the existence of probable cause. In fact, as a consequence of
the search warrant issued, the items confiscated from the
premises of the office of the Philippine Times at 610
Mezzanine Floor, Gochengco Bldg., T.M. Kalaw, Ermita,
Manila were the following:
1. One bundle of assorted negative;
2. One bundle of assorted lay out;
3. Three folders of assorted articles/writings
used by Philippine Times news and other
paraphernalias;
4. Four tape alleged speech of Mayor
Climaco, two alleged speeches of Aquino
and a speech of one various artist;
5. One bundle Dummies;
6. Ten bundles of assorted copies of
Philippine Times issued on different dates
(Nos. 6, 7, 8, 9, 10, 11, 12, 13, 14 & 15):
7. One Typewriter Remington Brand Long
Carriage with No. J-2479373;

8. OneTypewriterAdler-short with No.


9003011;
9. Three (3) bundles of Philippine Times
latest issue for Baguio City (p. 26, Rollo)
In Stonehill vs. Diokno, 20 SCRA 383, this Court held that
search warrants authorizing the seizure of books of accounts
and records "showing all the business transactions" of certain
persons, regardless of whether the transactions were legal or
illegal, contravene the explicit comment of the Bill of Rights
that the things to be seized should be particularly described
and defeat its major objective of eliminating general
warrants. In the case at bar, the search warrant issued by
respondent judge allowed seizure of printed copies of the
Philippine Times, manuscripts/drafts of articles for
publication, newspaper dummies, subversive documents,
articles, etc., and even typewriters, duplicating machines,
mimeographing and tape recording machines. Thus, the
language used is so all embracing as to include all
conceivable records and equipment of petitioner regardless
of whether they are legal or illegal. The search warrant under
consideration was in the nature of a general warrant which is
constitutionally objectionable.
Respondents do not deny the fact that the business office of
the "Philippine Times" of which petitioner was the publishereditor was padlocked and sealed. The consequence is, the
printing and publication of said newspaper were
discontinued. In Burgos, Sr. vs. Chief of Staff of the Armed
Forces of the Philippines, supra, We held that "[sluch closure
is in the nature of previous restraint or censorship abhorrent
to the freedom of the press guaranteed under the
fundamental law, and constitutes a virtual denial of
petitioners' freedom to express themselves in print. This state
of being is patently anathematic to a democratic framework
where a free, alert and even militant press is essential for the
political enlightenment and growth of the citizenry."

Finally, respondents argue that while the search warrant was


issued on September 29, 1983 and was executed on the very
same day, it was only on November 6, 1984, or one (1) year,
one (1) month and six (6) days when petitioner filed his
motion for the recall of the warrant and the return of the
documents/personal properties. Having failed to act
seasonably, respondents claim that petitioner is guilty of
laches.

WHEREFORE, Search Warrant No. Q-00002 issued by the be the only practical means of enforcing the
respondent judge on September 29, 1983 is declared null and constitutional injunction against unreasonable searches
void and, accordingly, SET ASIDE.
and seizures by outlawing all evidence illegally seized and
thereby removing the incentive on the part of state and
The prayer for a writ of mandatory injunction for the return police officers to disregard such basic rights. What the
of the seized articles is GRANTED and all properties seized plain language of the Constitution mandates is beyond the
thereunder are hereby ordered RELEASED to petitioner.
power of the courts to change or modify. All the articles
Further, respondents Lt. Col. Berlin A. Castillo and lst Lt.
thus seized fag under the exclusionary rule totally and
Godofredo M. Ignacio are ordered to RE-OPEN the
unqualifiedly and cannot be used against any of the three
Laches is the failure or neglect, for an unreasonable and
padlocked office premises of the Philippine Times at 610
petitioners.
unexplained length of time, to do that which by exercising
Mezzanine Floor, Gochengco Bldg., T.M. Kalaw, Ermita,
due diligence, could or should have been done earlier. The
Manila.
Nolasco vs. Pao
negligence or omission to assert a right within a reasonable
139 SCRA 152 (1985)
SO ORDERED.
time, warranting a presumption that the party entitled to
FACTS: The crime alleged is rebellion. Aguilar-Roque was
assert it either has abandoned it or declined to assert it
one of the accused of rebellion in a criminal case before a
(Tijam vs. Sibonghanoy, L-21450, April 15, 1968, 23 SCRA NOLASCO V. PAO - 147 SCRA 509
Special Military Commission. At that time, she was at large.
35).
FACTS:
The military authorities arrested her and Nolasco, who had
The case at bar is for the motion for partial
In his petition, Corro alleged that on October 1, 1983, less
no standing arrest against him, while the latter were onboard
reconsideration
of
both
petitioners
and
respondents
of
the
than forty-two (42) hours after the military operatives shut
a public vehicle. Consequently, the military authorities
SCs decision that the questioned search warrant by
down his newspaper on September 29, 1983, he was invited
searched the residence of Aguilar-Roque. They seized 428
petitioners is null and void, that respondents are enjoined
by the Director-General PC/INP, and subsequently detained. from introducing evidence using such search warrant,
documents and written materials, and additionally a portable
Thereafter, he was charged with the crime of inciting to
typewriter, and 2 wooden boxes.
but such personalities obtained would still be retained,
sedition before the City Fiscal's Office in Quezon City, and
without prejudice to petitioner Aguilar-Roque.
ISSUE: Whether or not the search and seizure fall under the
on October 7, 1983, a preventive detention action was served Respondents contend that the search warrant is valid and
that
it
should
be
considered
in
the
context
of
the
crime
rule of warrantless search incidental to a lawful arrest.
upon him. Consequently, he had to file a petition for habeas
of
rebellion,
where
the
warrant
was
based.
Petitioners
on
corpus. It was only on November 8, 1984 when this Court
the other hand, on the part of petitioner Aguilar-Roque,
HELD: Yes, the search and seizure fall under the rule of
issued its Resolution in G.R. No. 68976, entitled: In the
contend that a lawful search would be justified only by
warrantless search incidental to a lawful arrest. Considering
Matter of the Petition for Habeas Corpus of Rommel Corro a lawful arrest. And since there was illegal arrest of
that Aguilar-Roque has been charged with rebellion, which is
Angle Corro vs. Minister Juan Ponce Enrile, et al., releasing Aguilar-Roque, the search was unlawful and that the
a crime against public order, that the warrant for her arrest
Rommel Corro on recognizance of his lawyers, Attys.
personalities seized during the illegal search should be
has not been served for a considerable period of time; that
returned to the petitioner. The respondents, in defense,
Humberto B. Basco, Reynaldo Bagatsing and Edilberto
concede
that
the
search
warrants
were
null
and
void
but
the
she was arrested within the general vicinity of her dwelling;
Balce, In the same month, November 1984, petitioner filed
arrests
were
not.
and that the search of her dwelling was made within a half
his motion to recall warrant and to return the seized
hour of her arrest, the search and seizure conducted did not
documents. When respondent judge denied the motion, he
HELD:
need a search warrant, for possible effective results in the
came to Us.
"Any evidence obtained in violation of this . . . section shall
interest of public order.
be inadmissible for any purpose in any proceeding"
Considering the above circumstances, the claim that
(Sec. 4[2]). This constitutional mandate expressly adopting
G.R. No. L-64261 December 26, 1984
petitioner had abandoned his right to the possession of the
the exclusionary rule has proved by historical experience to
JOSE BURGOS, SR vs. THE CHIEF OF STAFF- AFP, ET
seized properties is incorrect.

AL
Facts:
Assailed in this petition for certiorari prohibition and
mandamus with preliminary mandatory and prohibitory
injunction is the validity of two [2] search warrants issued on
December 7, 1982 by Judge Ernani Cruz-Pano of the then
CFI of Rizal [Quezon City], under which the premises of the
"Metropolitan Mail" and "We Forum" newspapers,
respectively, were searched, and office and printing
machines, equipment, paraphernalia, motor vehicles and
other articles used in the printing, publication and
distribution of the said newspapers, as well as numerous
papers, documents, books and other written literature alleged
to be in the possession and control of petitioner Jose Burgos,
Jr. publisher-editor of the "We Forum" newspaper, were
seized.
Petitioners further pray that a writ of preliminary mandatory
and prohibitory injunction be issued for the return of the
seized articles, and that respondents be enjoined from using
the articles thus seized as evidence against petitioner Jose
Burgos, Jr. and the other accused in Criminal Case No. Q022782 of the Regional Trial Court of Quezon City, entitled
People v. Jose Burgos, Jr. et al.
Issue: Was the closure of WE Forum a case of prior
restraint?
Ruling:
Yes. As heretofore stated, the premises searched were the
business and printing offices of the "Metropolitan Mail" and
the "We Forum newspapers. As a consequence of the search
and seizure, these premises were padlocked and sealed, with
the further result that the printing and publication of said
newspapers were discontinued. Such closure is in the nature
of previous restraint or censorship abhorrent to the freedom
of the press guaranteed under the fundamental law, and
constitutes a virtual denial of petitioners' freedom to express
themselves in print. This state of being is patently
anathematic to a democratic framework where a free, alert
and even militant press is essential for the political
enlightenment and growth of the citizenry.

BURGOS, SR. V. CHIEF OF STAFF, AFP [133 SCRA


800; G.R. NO. 64261; 26 DEC 1984]
Facts: Petitioners assail the validity of 2 search warrants
issued on December 7, 1982 by respondent Judge Cruz-Pano
of the then Court of First Instance of Rizal, under which the
premises known as No. 19, Road 3, Project 6, Quezon City,
and 784 Units C & D, RMS Building, Quezon Avenue,
Quezon City, business addresses of the "Metropolitan Mail"
and "We Forum" newspapers, respectively, were searched,
and office and printing machines, equipment, paraphernalia,
motor vehicles and other articles used in the printing,
publication and distribution of the said newspapers, as well
as numerous papers, documents, books and other written
literature alleged to be in the possession and control of
petitioner Jose Burgos, Jr. publisher-editor of the "We
Forum" newspaper, were seized. As a consequence of the
search and seizure, these premises were padlocked and
sealed, with the further result that the printing and
publication of said newspapers were discontinued.
Respondents contend that petitioners should have filed a
motion to quash said warrants in the court that issued them
before impugning the validity of the same before this Court.
Respondents also assail the petition on ground of laches
(Failure or negligence for an unreasonable and unexplained
length of time to do that which, by exercising due diligence,
could or should have been done earlier. It is negligence or
omission to assert a right within a reasonable time,
warranting a presumption that the party entitled to assert it
either has abandoned it or declined to assert it). Respondents
further state that since petitioner had already used as
evidence some of the documents seized in a prior criminal
case, he is stopped from challenging the validity of the
search warrants.
Petitioners submit the following reasons to nullify the
questioned warrants:
1. Respondent Judge failed to conduct an examination under
oath or affirmation of the applicant and his witnesses, as
mandated by the above-quoted constitutional provision as
well as Sec. 4, Rule 126 of the Rules of Court.

2. The search warrants pinpointed only one address which


would be the former abovementioned address.
3. Articles belonging to his co-petitioners were also seized
although the warrants were only directed against Jose
Burgos, Jr.
4. Real properties were seized.
5. The application along with a joint affidavit, upon which
the warrants were issued, from the Metrocom Intelligence
and Security Group could not have provided sufficient basis
for the finding of a probable cause upon which a warrant
may be validly issued in accordance with Section 3, Article
IV of the 1973 Constitution.
Respondents justify the continued sealing of the printing
machines on the ground that they have been sequestered
under Section 8 of Presidential Decree No. 885, as amended,
which authorizes sequestration of the property of any person
engaged in subversive activities against the government in
accordance with implementing rules and regulations as may
be issued by the Secretary of National Defense.

Issue: Whether or Not the 2 search warrants were validly


issued and executed.

Held: In regard to the quashal of warrants that petitioners


should have initially filed to the lower court, this Court takes
cognizance of this petition in view of the seriousness and
urgency of the constitutional Issue raised, not to mention the
public interest generated by the search of the "We Forum"
offices which was televised in Channel 7 and widely
publicized in all metropolitan dailies. The existence of this
special circumstance justifies this Court to exercise its
inherent power to suspend its rules. With the contention

pertaining to laches, the petitioners gave an explanation


evidencing that they have exhausted other extra-judicial
efforts to remedy the situation, negating the presumption that
they have abandoned their right to the possession of the
seized property.

the facts within the personal knowledge of the petitioner or


his witnesses, because the purpose thereof is to convince the
committing magistrate, not the individual making the
affidavit and seeking the issuance of the warrant, of the
existence of probable cause." Another factor which makes
the search warrants under consideration constitutionally
objectionable is that they are in the nature of general
warrants. The description of the articles sought to be seized
under the search warrants in question are too general.

On the enumerated reasons:


1. This objection may properly be considered moot and
academic, as petitioners themselves conceded during the
hearing on August 9, 1983, that an examination had indeed
been conducted by respondent judge of Col. Abadilla and his With regard to the respondents invoking PD 885, there is an
witnesses.
absence of any implementing rules and regulations
promulgated by the Minister of National Defense.
2. The defect pointed out is obviously a typographical error. Furthermore, President Marcos himself denies the request of
Precisely, two search warrants were applied for and issued
military authorities to sequester the property seized from
because the purpose and intent were to search two distinct
petitioners. The closure of the premises subjected to search
premises. It would be quite absurd and illogical for
and seizure is contrary to the freedom of the press as
respondent judge to have issued two warrants intended for
guaranteed in our fundamental law. The search warrants are
one and the same place.
declared null and void.
3. Section 2, Rule 126, of the Rules of Court, does not
PEOPLE V. BURGOS - 144 SCRA 1
require that the property to be seized should be owned by the
FACTS:
person against whom the search warrant is directed. It may
or may not be owned by him.
Due to an information given by a person, who allegedly was
being forcibly recruited by accused to the NPA, the members
4. Petitioners do not claim to be the owners of the land
of the Constabulary went to the house of accused, asked
and/or building on which the machineries were placed. This about his firearm and documents connected to subversive
being the case, the machineries in question, while in fact
activities. Accused pointed to where his firearm was as well
bolted to the ground, remain movable property susceptible to as his other documents allegedly.
HELD:
seizure under a search warrant.

application
beyond the cases specifically provided by law. To do
so would infringe upon personal liberty and set back a
basic right so often violated and so deserving of full
protection.
G.R.No. 74869 July 6, 1988
PEOPLE OF THE PHILIPPINES, plaintiff-appellee,
vs.
IDEL AMINNUDIN y AHNI, defendant-appellant.
The Solicitor General for plaintiff-appellee.
Herminio T. Llariza counsel de-officio for defendantappellant.

CRUZ, J.:
The accused-appellant claimed his business was selling
watches but he was nonetheless arrested, tried and found
guilty of illegally transporting marijuana. The trial court,
disbelieving him, held it was high time to put him away and
sentenced him to life imprisonment plus a fine of
P20,000.00. 1

Idel Aminnudin was arrested on June 25, 1984, shortly after


disembarking from the M/V Wilcon 9 at about 8:30 in the
evening, in Iloilo City. The PC officers who were in fact
5. The broad statements in the application and joint affidavit The right of the person to be secure against any unreasonable waiting for him simply accosted him, inspected his bag and
finding what looked liked marijuana leaves took him to their
seizure of his body and any deprivation of liberty is a
are mere conclusions of law and does not satisfy the
headquarters for investigation. The two bundles of suspect
requirements of probable cause. Deficient of such particulars most basic and fundamental one. The statute or rule,
articles were confiscated from him and later taken to the NBI
as would justify a finding of the existence of probable cause, which allows exceptions to the requirement of warrants
laboratory for examination. When they were verified as
of arrest is strictly construed. Any exception must
said allegation cannot serve as basis for the issuance of a
marijuana leaves, an information for violation of the
search warrant and it was a grave error for respondent judge clearly fall within the situations when securing a
Dangerous Drugs Act was filed against him. 2 Later, the
warrant would be absurd or is manifestly unnecessary
to have done so. In Alvarez v. Court of First Instance, this
information was amended to include Farida Ali y Hassen,
Court ruled that "the oath required must refer to the truth of as provided by the Rule. We cannot liberally construe
who had also been arrested with him that same evening and
the rule on arrests without warrant or extend its

likewise investigated. 3 Both were arraigned and pleaded not


guilty. 4 Subsequently, the fiscal filed a motion to dismiss the
charge against Ali on the basis of a sworn statement of the
arresting officers absolving her after a 'thorough
investigation." 5 The motion was granted, and trial proceeded
only against the accused-appellant, who was eventually
convicted . 6

Iloilo City to sell watches but carried only two watches at the
time, traveling from Jolo for that purpose and spending
P107.00 for fare, not to mention his other expenses. 15
Aminnudin testified that he kept the two watches in a secret
pocket below his belt but, strangely, they were not
discovered when he was bodily searched by the arresting
officers nor were they damaged as a result of his
manhandling. 16 He also said he sold one of the watches for
According to the prosecution, the PC officers had earlier
P400.00 and gave away the other, although the watches
received a tip from one of their informers that the accusedbelonged not to him but to his cousin, 17 to a friend whose
appellant was on board a vessel bound for Iloilo City and
full name he said did not even know. 18 The trial court also
was carrying marijuana. 7 He was Identified by name. 8
rejected his allegations of maltreatment, observing that he
Acting on this tip, they waited for him in the evening of June had not sufficiently proved the injuries sustained by him. 19
25, 1984, and approached him as he descended from the
gangplank after the informer had pointed to him. 9 They
There is no justification to reverse these factual findings,
detained him and inspected the bag he was carrying. It was
considering that it was the trial judge who had immediate
found to contain three kilos of what were later analyzed as
access to the testimony of the witnesses and had the
10
marijuana leaves by an NBI forensic examiner, who
opportunity to weigh their credibility on the stand. Nuances
testified that she conducted microscopic, chemical and
of tone or voice, meaningful pauses and hesitation, flush of
chromatographic tests on them. On the basis of this finding, face and dart of eyes, which may reveal the truth or expose
the corresponding charge was then filed against Aminnudin. the lie, are not described in the impersonal record. But the
trial judge sees all of this, discovering for himself the truant
In his defense, Aminnudin disclaimed the marijuana,
fact amidst the falsities.
averring that all he had in his bag was his clothing consisting
of a jacket, two shirts and two pairs of pants. 11 He alleged
The only exception we may make in this case is the trial
that he was arbitrarily arrested and immediately handcuffed. court's conclusion that the accused-appellant was not really
His bag was confiscated without a search warrant. At the PC beaten up because he did not complain about it later nor did
headquarters, he was manhandled to force him to admit he
he submit to a medical examination. That is hardly fair or
was carrying the marijuana, the investigator hitting him with realistic. It is possible Aminnudin never had that opportunity
a piece of wood in the chest and arms even as he parried the as he was at that time under detention by the PC authorities
blows while he was still handcuffed. 12 He insisted he did not and in fact has never been set free since he was arrested in
even know what marijuana looked like and that his business 1984 and up to the present. No bail has been allowed for his
was selling watches and sometimes cigarettes. 13 He also
release.
argued that the marijuana he was alleged to have been
There is one point that deserves closer examination,
carrying was not properly Identified and could have been
however, and it is Aminnudin's claim that he was arrested
any of several bundles kept in the stock room of the PC
and searched without warrant, making the marijuana
headquarters. 14
allegedly found in his possession inadmissible in evidence
The trial court was unconvinced, noting from its own
against him under the Bill of Rights. The decision did not
examination of the accused that he claimed to have come to even discuss this point. For his part, the Solicitor General

dismissed this after an all-too-short argument that the arrest


of Aminnudin was valid because it came under Rule 113,
Section 6(b) of the Rules of Court on warrantless arrests.
This made the search also valid as incidental to a lawful
arrest.
It is not disputed, and in fact it is admitted by the PC officers
who testified for the prosecution, that they had no warrant
when they arrested Aminnudin and seized the bag he was
carrying. Their only justification was the tip they had earlier
received from a reliable and regular informer who reported
to them that Aminnudin was arriving in Iloilo by boat with
marijuana. Their testimony varies as to the time they
received the tip, one saying it was two days before the arrest,
20
another two weeks 21 and a third "weeks before June 25." 22
On this matter, we may prefer the declaration of the chief of
the arresting team, Lt. Cipriano Querol, Jr., who testified as
follows:
Q You mentioned an
intelligence report, you
mean with respect to the
coming of Idel Aminnudin
on June 25, 1984?
A Yes, sir.
Q When did you receive this
intelligence report?
A Two days before June 25,
1984 and it was supported
by reliable sources.
Q Were you informed of the
coming of the Wilcon 9 and
the possible trafficking of
marijuana leaves on that
date?

A Yes, sir, two days before


June 25, 1984 when we
received this information
from that particular
informer, prior to June 25,
1984 we have already
reports of the particular
operation which was being
participated by Idel
Aminnudin.

A Previous to June 25, 1984


we received reports on the
activities of Idel
Aminnudin.

Q You said you received an


intelligence report two days
before June 25, 1984 with
respect to the coming of
Wilcon 9?

Q From whom did you get


that information?

A Yes, sir.
Q Did you receive any other
report aside from this
intelligence report?
A Well, I have received also
other reports but not
pertaining to the coming of
Wilcon 9. For instance,
report of illegal gambling
operation.
COURT:
Q Previous to that particular
information which you said
two days before June 25,
1984, did you also receive
daily report regarding the
activities of Idel Aminnudin

Q What were those


activities?
A Purely marijuana
trafficking.

A It came to my hand which


was written in a required
sheet of information, maybe
for security reason and we
cannot Identify the person.
Q But you received it from
your regular informer?

Q And this information


respecting Idel Aminnudin's
coming to Iloilo with
marijuana was received by
you many days before you
received the intelligence
report in writing?
A Not a report of the
particular coming of
Aminnudin but his
activities.
Q You only knew that he
was coming on June
25,1984 two days before?
A Yes, sir.
Q You mean that before
June 23, 1984 you did not
know that minnudin was
coming?

A Yes, sir.
ATTY. LLARIZA:
Q Previous to June 25,
1984, you were more or less
sure that Idel Aminnudin is
coming with drugs?
A Marijuana, sir.

A Before June 23,1984, I, in


my capacity, did not know
that he was coming but on
June 23, 1984 that was the
time when I received the
information that he was
coming. Regarding the
reports on his activities, we
have reports that he was
already consummated the
act of selling and shipping
marijuana stuff.
COURT:

Q And as a result of that


report, you put him under
surveillance?

A Because we were very


very sure that our operation
will yield positive result.

A Yes, sir.

Q Is that your procedure


that whenever it will yield
positive result you do not
need a search warrant
anymore?

Q In the intelligence report,


only the name of Idel
Aminnudin was mentioned?

expediency could not be invoked to dispense with the


obtention of the warrant as in the case of Roldan v. Arca, 24
for example. Here it was held that vessels and aircraft are
subject to warrantless searches and seizures for violation of
the customs law because these vehicles may be quickly
moved out of the locality or jurisdiction before the warrant
can be secured.

The present case presented no such urgency. From the


conflicting declarations of the PC witnesses, it is clear that
A Yes, sir.
A Search warrant is not
they had at least two days within which they could have
23
necessary.
obtained a warrant to arrest and search Aminnudin who was
Q Are you sure of that?
coming to Iloilo on the M/V Wilcon 9. His name was known.
That last answer is a cavalier pronouncement, especially as it The vehicle was Identified. The date of its arrival was
A On the 23rd he will be
comes from a mere lieutenant of the PC. The Supreme Court certain. And from the information they had received, they
coming with the woman.
cannot countenance such a statement. This is still a
could have persuaded a judge that there was probable cause,
government of laws and not of men.
indeed, to justify the issuance of a warrant. Yet they did
Q So that even before you
nothing. No effort was made to comply with the law. The
received the official report
The mandate of the Bill of Rights is clear:
Bill of Rights was ignored altogether because the PC
on June 23, 1984, you had
lieutenant who was the head of the arresting team, had
already gathered
Sec. 2. The right of the people to be secure
determined on his own authority that a "search warrant was
information to the effect
in their persons, houses, papers and effects
not necessary."
that Idel Aminnudin was
against unreasonable searches and seizures
coming to Iloilo on June 25,
of whatever nature and for any purpose shall In the many cases where this Court has sustained the
1984?
be inviolable, and no search warrant or
warrantless arrest of violators of the Dangerous Drugs Act, it
warrant of arrest shall issue except upon
has always been shown that they were caught red-handed, as
A Only on the 23rd of June.
probable cause to be determined personally a result of what are popularly called "buy-bust" operations of
by the judge after examination under oath or the narcotics agents. 25 Rule 113 was clearly applicable
Q You did not try to secure
affirmation of the complainant and the
because at the precise time of arrest the accused was in the
a search warrant for the
witnesses he may produce, and particularly act of selling the prohibited drug.
seizure or search of the
describing the place to be searched and the
subject mentioned in your
persons or things to be seized.
In the case at bar, the accused-appellant was not, at the
intelligence report?
moment of his arrest, committing a crime nor was it shown
In the case at bar, there was no warrant of arrest or search
that he was about to do so or that he had just done so. What
A No, more.
warrant issued by a judge after personal determination by
he was doing was descending the gangplank of the M/V
him of the existence of probable cause. Contrary to the
Wilcon 9 and there was no outward indication that called for
Q Why not?
averments of the government, the accused-appellant was not his arrest. To all appearances, he was like any of the other
caught in flagrante nor was a crime about to be committed or passengers innocently disembarking from the vessel. It was
had just been committed to justify the warrantless arrest
only when the informer pointed to him as the carrier of the
allowed under Rule 113 of the Rules of Court. Even
marijuana that he suddenly became suspect and so subject to

apprehension. It was the furtive finger that triggered his


arrest. The Identification by the informer was the probable
cause as determined by the officers (and not a judge) that
authorized them to pounce upon Aminnudin and
immediately arrest him.

including the basest of criminals. The Constitution covers


with the mantle of its protection the innocent and the guilty
alike against any manner of high- handedness from the
authorities, however praiseworthy their intentions.

Those who are supposed to enforce the law are not justified
Now that we have succeeded in restoring democracy in our
in disregarding the rights of the individual in the name of
country after fourteen years of the despised dictatorship,
order. Order is too high a price for the loss of liberty. As
when any one could be picked up at will, detained without
Justice Holmes, again, said, "I think it a less evil that some
charges and punished without trial, we will have only
criminals should escape than that the government should
ourselves to blame if that kind of arbitrariness is allowed to play an ignoble part." It is simply not allowed in the free
return, to once more flaunt its disdain of the Constitution and society to violate a law to enforce another, especially if the
the individual liberties its Bill of Rights guarantees.
law violated is the Constitution itself.
While this is not to say that the accused-appellant is
innocent, for indeed his very own words suggest that he is
lying, that fact alone does not justify a finding that he is
guilty. The constitutional presumption is that he is innocent,
and he will be so declared even if his defense is weak as long
as the prosecution is not strong enough to convict him.
Without the evidence of the marijuana allegedly seized from
Aminnudin, the case of the prosecution must fall. That
evidence cannot be admitted, and should never have been
considered by the trial court for the simple fact is that the
marijuana was seized illegally. It is the fruit of the poisonous
tree, to use Justice Holmes' felicitous phrase. The search was
not an incident of a lawful arrest because there was no
warrant of arrest and the warrantless arrest did not come
under the exceptions allowed by the Rules of Court. Hence,
the warrantless search was also illegal and the evidence
obtained thereby was inadmissible.

We find that with the exclusion of the illegally seized


marijuana as evidence against the accused-appellant, his
guilt has not been proved beyond reasonable doubt and he
must therefore be discharged on the presumption that he is
innocent.
ACCORDINGLY, the decision of the trial court is
REVERSED and the accused-appellant is ACQUITTED. It
is so ordered.
G.R. No. 72301

July 31, 1987

ROLANDO PONSICA, ROGELIO ARNAIZ, FR. NICO


HOFSTEDE, BERNARDINO PATIGAS, ZACHEUS
ROJO, GODOFREDO RETIRACTON, LORETO
BERING, ROGELIO ARTAJO, JOVITO MARATAS,
CARLOS ALLEONES, MILO PICCIO, ADOLFO
MAGUATE, GONZALO CASTILLA, RONNIE
DESUYO, FEDERICO AYO, ROLANDO BERNABE,
The Court strongly supports the campaign of the government MARIANO REYES, DANIEL GEMPESALA,
against drug addiction and commends the efforts of our law- WILFREDO SARATOBIAS, MILDRED SAGUIRE,
CRESENCIO ENCARGUEZ, JOHN BUSTAMANTE,
enforcement officers against those who would inflict this
JOHN DOE and RICHARD DOE, petitioners,
malediction upon our people, especially the susceptible
youth. But as demanding as this campaign may be, it cannot vs.
be more so than the compulsions of the Bill of Rights for the HON. EMILIO M. IGNALAGA, Presiding Judge,
Municipal Trial Court of Escalante, Negros Occidental,
protection of the liberty of every individual in the realm,

MAYOR BRAULIO LUMAYNO, CAPT. MODESTO


SAN-SON, CAPT. RAFAEL JUGAN, respondents.
NARVASA, J.:
The chief issue raised by the petitioners in this case is
whether or not Section 143 of the Local Government Code1
granting power to the municipal mayor to conduct
preliminary investigations and order the arrest of the
accused, was repealed by the 1985 Rules on Criminal
Procedure promulgated by this Court; and is, in addition,
unconstitutional as vesting the power to conduct preliminary
investigations in an official who cannot be deemed a
"neutral and detached magistrate" within the contemplation
of Section 3, Article IV of the 1973 Constitution. The issue is
hereby resolved adversely to the petitioners, with the
stressed qualification that the mayor's power to order arrest
ceased to exist as of February 2, 1987 when the new
Constitution was ratified by the Filipino people, and that, in
any event, the investigation actually conducted by
respondent mayor in the case at bar was fatally defective.
Shortly after noon on September 20, 1985, an attempt was
made by firemen and soldiers to disperse a crowd of
demonstrators massed in front of the Municipal Building of
Escalante, Negros Occidental, with the use first, of water
spewed from fire hoses, and later, tear gas. Eventually there
was gunfire. Within moments, rallyists lay dead on and by
the National Road. The fatalities numbered fifteen (15),
according to the military officers; twenty-nine (29),
according to the demonstrators.

In the afternoon of that day, Escalante Town Mayor Braulio


Lumayno, in view of the absence of the Municipal Circuit
Court Judge (Emilio Ignalaga), took cognizance of a
complaint filed by the Military Station Commander charging
some of the rallyists with the felony of inciting to sedition,
and after avowedly conducting an investigation of the
witnesses presented by the complainant, issued an order for
the arrest of certain of the demonstrators. His order reads as
follows:

3) 3-page document, "Searching Questions and


knives; a "pogakhang" with 2 live cartridges; 2 grenades;
Answers" signed by Capt. Sanson, dated. and sworn and several empty shells of different caliber. Hours later,
to before Mayor Lumayno on, September 20, 1985; 5 another corpse, Identified as that of a demonstrator, too, was
brought to the PC Headquarters.
4) 3-page sworn statement of Godofredo Hoyo-A y
Jayme, General Manager of the BalintawakThe petitioners however give a different version of the facts.
6
Escalante Water District;
What happened, according to them, was that at 9 o'clock in
the morning on that day, a group of demonstrators,
7
5) Affidavit of Leopoldo Villalon;
"composed mostly of laid-off sugar field workers," gathered
in front of Escalante Municipal Hall "in the exercise of their
6) Affidavit of Elpidio Carbajosa; 8 and
It appearing that the Presiding Municipal Circuit
constitutionally guaranteed right to freedom of expression
Court Judge, Hon. Emilio M. Ignalaga, is on official
and to assemble peacefully to petition the government for
9
7)
Affidavit
of
Eduardo
Flores.
leave of absence and, in the interest of justice, the
redress of grievances."10 About an hour afterwards, fire
undersigned has to urgently act on the complaint
trucks arrived one after another, as well as jeeploads of
The
gist
of
the
testimony
of
Capt.
Sanson
and
the
other
filed by the Station Commander, against the abovesoldiers and CHDF members, in full combat gear, Shortly
affiants is that on September 20, 1985, the demonstrators,
named accused for "INCITING TO SEDITION"
after noon, after "going thru the motions of negotiating with
numbering "about 1,000, " had blockaded the main highway the demonstrators," the military officers ordered the crowd to
and, on the basis of the evidence submitted after a
searching question and answer were conducted and, in front of the Escalante Municipal Building, by massing
disperse; but without warning, fire hoses were trained on and
themselves on the road as well as by piling stones, coconut
being satisfied that said crime has been committed,
sprayed water at the demonstrators. When the rallyists did
trunks
and
pieces
of
wood
in
the
middle
of
the
highway.
in order not to frustrate the ends of justice, it is
not budge, tear gas canisters were thrown at them. A
They
were
also
"shouting
invectives,
seditious
and
scurrilous
necessary that the above-named accused be placed
demonstrator picked up a canister and threw it at an "empty
words against the government." Negotiations with Ponsica,
under custody. Let therefore, a warrant of arrest be
space in the plaza" The soldier and CHDF members
Chairman
of
the
Escalante
Chapter
of
"BAYAN"
(Bagong
issued for said above-named accused.
thereupon fired indiscriminately at the crowd, killing 29 and
Alyansang Makabayan), to have the road cleared having
injuring at least 30 persons.11
been unavailing, firemen on firetrucks began to train a
Bail recommended: P12,000.00. 2
"torrent of water" from their fire hoses on the demonstrators. After the Mayor had referred the case to Municipal Trial
The rallyists retaliated by hurling stones at the firemen. One Court Judge Ignalaga on September 24, 1985,12 an
In the record of the Court a quo appear the following inter
of them "approached the security of the firetruck and stabbed "URGENT MOTION TO QUASH WARRANT OF
alia:
him." Others climbed aboard the trucks and tried to grab the ARREST" was filed on September 26, 1985 by petitioners'
1) Complaint for "Inciting to Sedition" (RPC 142, as firehoses and firearms of the officers. At this point, on Capt. counsel on the ground that a mayor no longer has authority
Sanson's orders, his "back-up teams" of soldiers commenced to conduct preliminary investigations or issue warrants of
and by PD 183 and PD 1974), signed by a Capt.
to throw tear gas at the crowd. One of the demonstrators
Jugan, and sworn to before Mayor Lumayno on
arrest that authority having been "withheld in the 1985 New
picked up a tear gas canister and hurled it back at the
Sept. 20, 1985, bearing the stamped notation of
Rules on Criminal Procedure."13 This was opposed by the
3
soldiers. At the same time gunfire from "different assorted
filing with the MTC: "9/24/85, 4:00 PM:"
Station Commander14 who invoked Section 143 of the Local
firearms" emanated from the rallyists; and some of the shots Government Code providing that "(i)n case of temporary
2) 2-page Affidavit of Capt. Sanson, dated, and
hit the blinker of a firetruck and the headlight of another. The absence of the Judge assigned to the municipality, the mayor
sworn to before Actg.. City Fiscal Abros (Cadiz
soldiers shot back. This exchange of gunfire resulted in
may conduct the preliminary examination in criminal cases
4
City) on Sept. 20, 1985;
"fourteen (14) demonstrators killed on the spot. " Recovered when, in his opinion the investigation cannot be delayed
at the scene were a rifle; a U.S. 45 cal. pistol; 2 "homemade" without prejudice to the interest of justice." A reply was filed
pistols; 14 steel arrows and 4 assorted slings; 30 assorted
by the petitioners after their receipt of the opposition "only

last October 8, 1985."15 They contended that the "power of


the municipal mayor to conduct preliminary investigation
and issue a warrant of arrest under the 1964 Revised Rules
of Court** (had been) impliedly repealed by the 1985 New
Rules on Criminal Procedure;" that "the 1985 New Rules on
Criminal Procedure being a special law, controls over
provisions of the Local Government Code (BP 337, 1983),
which is a general law;" and in any case, "subject t warrants
of arrest should be reviewed and revoked as done without
observance of legal requisites."
By Resolution dated Oct. 11, 1985,16 the Judge confirmed the
mayor's arrest order. He opined that in the absence of the
judge, the mayor still has authority to conduct preliminary
investigations and issue arrest warrants, since Rule 112, Sec.
2 (d), of the 1985 Rules, includes as among those authorized
to conduct preliminary investigations, "Such other officers as
may be authorized by law;" and the Local G government
Code of 1983, Section 143, grants a town mayor authority to
conduct preliminary examinations in case of the temporary
absence of the judge when such investigation cannot be
delayed without prejudice to the interest of justice. The
Judge declared that in the case at bar, the mayor had
conducted the examination personally, and having in the
exercise of his discretion found probable cause, issued the
warrants of arrest in question; and conceding arguendo
irregularity in that the preliminary examination was
conducted without according the parties the assistance of
lawyers (contrary to par. 2, Sec. 143, Chap. 3, Title 2, Book
11, Local Government Code), this does not render the
proceedings void because at any rate, the mayor had duly
observed the uniform procedure under PD 91 (citing: People
v. Paran, 52 Phil. 712; Hashim v. Boncan, 71 Phil. 216; Lino
v. Fugoso, 77 Phil. 933).
It is Mayor Lumayno's order for the petitioners' arrest of
September 20, 1985, and Jude Ignalaga's Order of October
11, 1985 "validating that order of arrest" that the petitioners
would have this Court nullify and perpetually enjoin.17

In their petition for certiorari and prohibition filed on


October 15, 1985, and their Memorandum of January 21,
1986,18 they assert that:
1) while Section 3,Rule 112 of the l964 Rules of
Court,empowers the municipal mayor, "in case of
temporary absence of both the municipal and the
auxiliary municipal judges from the municipality,
town or place wherein they exercise their
jurisdiction, to make the preliminary examination in
criminal cases when such examination cannot be
delayed without prejudice to the interest of justice,"
that power has been removed from him by the 1985
New Rules on Criminal Procedure which "do not
mention the Municipal Mayor as among the officers
authorized to conduct preliminary investigation
(Section 2, Rule 112), much less to issue an order of
arrest (Section 6, Ibid.)19

3) the Mayor's examination "falls short of the


requirements of "searching questions and answers;"
the statements of the witnesses supposedly
interrogated by the mayor, are either merely
conclusions of law or sterile as regards seditious
utterances, hence, probable cause was non-existent 23
and
4) Article 142 of the Revised Penal Code, as
amended-under which the petitioners are charged is
based on the US Sedition Act of 1978, which has
been declared by the US Supreme Court as
"repugnant to the constitutional guarantee of
freedom of speech and expression (New York Times
Co. v. Sullivan, 376 U.S. 254);" hence, said Article
142 is also fatally flawed and therefore, the warrant.
of arrest in question was in effect issued for a "crime
which in the context of the constitutionally
guaranteed freedom of speech and expression does
not exist."24

2) Section 143 of BP Blg. 337 (Local Government


Code) under which the order of arrest in question
is presumably made to rest-cannot withstand the
In his comment filed with this Court,25 Judge Ignalaga argues
"constitutional test" of Section 3, Article IV,20 which that
safeguards the right against unreasonable searches
1) The validity of the preliminary examination on
and seizures, and requires the interposition of a
the basis of which the arrest warrants were issued,
"judge, or such other responsible officer as may be
may not be raised for the first time in the Supreme
authorized by law," meaning "a neutral and detached
Court, but should first be ventilated before the RTC
magistrate competent to determine probable cause
having cognizance of the crime.26 In any case, upon
(Shadwick v. City of Tampa, 40 LW 4758; Castillo v.
the considerations set out in his resolution of
Jias, 62 SCRA 124; Ang Tibay v. CIR, 69 Phil. 635;
October 11, 1985, the preliminary examination in
Zambales Chromite, etc. v. C.A., 934 SCRA 2617);"
question is valid.27
and since a mayor is obviously not such a magistrate
(U.S. v. Chadwick, 433 U.S. 197), the orders of
2) The petitioners raise factual issues which are best
arrest at bar are constitutionally infirm; 21 moreover,
left for determination by the RTC, the Supreme
said Section 143 of the Local Government Code is
Court not being a trier of facts.28
"merely a rule of procedure** (and is thus) deemed
to have been superseded by the New Rules of
3) The Mayor in fact followed the procedure in the
Criminal Procedure;22
constitution prior to issuing the arrest warrants. 29

4) The constitutionality of PD 1974 should be


assailed by separate petition.30
The Solicitor General at the time, Estelito Mendoza, also
filed a comment on December 6, 1985.31 The comment
addressed itself only to the issue of "the validity of the law
authorizing municipal mayors to issue warrants of arrest and
the law punishing inciting to sedition," in view of the fact
that the respondents had presented their own separate
comments on the petition.32 The comment points out that:
1) Section 3, Article IV, of the 1973 Constitution,
which mentions a "judge, or such other responsible
officer as may be authorized by law" as vested with
competence to conduct preliminary investigations, is
an innovation. In implementation thereof, BP Blg.
337 was enacted on February 10, 1983, empowering
mayors to conduct preliminary investigations (Sec.
143). This is a recognition of the truism that the
determination of probable cause is but a quasi
judicial function Ocampo v. US, 58 LED 1231).33
Petitioners' American authorities are not applicable:
the mayor is the highest official in the municipality;
he exercises only general supervision over the police
but is not directly involved in police work; the old
rules precisely expressed a recognition of the
capability of i mayors to determine probable cause,
and the omission of mayors in the 1985 Rules
simply means that the determination of officers who
may be authorized to conduct preliminary
investigations was deemed best left to legislation.34
2) The Philippine sedition law is not akin to the US
Sedition Law; in any event our own sedition law has
passed the test of constitutionality (Espuelas v.
People, 90 Phil. 524).35
Solicitor General Sedfrey A. Ordonez subsequently declared
that he "stands by the constitutionality of the statutes the
petitioners question and, therefore, sustains the position

taken by his predecessor in office;"36 this, in response to this


of the defendant and to grant him bail in the manner
Court's resolution dated May 15, 1986, requiring the parties
and cases provided for in Rule 114. 38
to state whether supervening events had transpired materially
The Rules of 1940 contained an Identical provision, in
affecting the case.37
Section 3, Rule 108.
It is clear from the outset that the issue before this Court
The 1985 Rules on Criminal Procedure did not reproduce
does not involved the adjudgment of the guilt or innocence
of the soldiers in the tragic and regrettable killings in front of this provision, and did not include the mayor in the
enumeration of the officers authorized to conduct
the Escalante Town Hall in the early afternoon of that day,
preliminary investigations, those listed being judges of
the twentieth of September, 1985, an event that caused a
municipal trial courts and municipal circuit trial courts; city
great outcry of lamentation and condemnation throughout
or provincial fiscals and their assistants; national and
the land. This is a matter that should be and is in fact now
regional state prosecutors; and "such other officers as may
subject of a separate criminal proceeding. Neither is the
propriety of the victims' exercise of their constitutional rights be authorized by law." 39
of free speech and free assembly for redress of grievances in
SEC. 6. When warrant of arrest may issue.
the premises at issue here. The basic question before the
Court is divorced of the drama and the passion of those
(a) By the Regional Trial Court. Upon the filing
issues; it deals mainly with the dry, unexciting, but
of an information, the Regional Trial Court may
nonetheless important matter of whether or not the municipal
issue a warrant for the arrest of the accused
mayor has the power to conduct preliminary investigations
in the light of the 1985 amendments of the rules governing
(b) By the Municipal Trial Court. If the municipal
criminal procedure in the Rules of Court. The answer to the
trial judge conducting the preliminary investigation
question entails a re-examination and analysis of the relevant
is satisfied after an examination in writing and under
legal provisions.
oath of the complainant and his witnesses in the
form of searching questions and answers, that a
The 1964 Rules of Court explicitly gave the mayor authority
probable cause exists and that there is a necessity of
to conduct preliminary investigations.
placing the respondent under immediate custody in
order not to frustrate the ends of justice, he shall
SEC. 3. Preliminary examination by the municipal
issue a warrant of arrest. (n). 40
mayor. In case of temporary absence of both the
municipal and the auxiliary municipal judges from
It is of course the deletion in the 1985 rules of reference to
the municipality, town, or place wherein they
exercise their jurisdiction, the municipal mayor shall the municipal mayor as among those authorized to conduct
make the preliminary examination in criminal cases preliminary investigations and order arrests upon which the
petitioners chiefly rely as basis for their impugnation of
when such examination cannot be delayed without
Mayor Lumayno's preliminary investigation and order of
prejudice to the interest of justice. He shall make a
arrest based thereon.
report of any preliminary examination so made to
the municipal or to the auxiliary municipal judge
immediately upon the return of one or the other. He
shall have authority in such cases to order the arrest

The matter is however treated of not solely by the Rules of


Court but also by the 1973 Constitution, Section 3, Article
IV whereof reads:
SEC. 3. The right of the people to be secure in their
persons, houses, papers, and effects against
unreasonable searches and seizures of whatever
nature and for any purpose shall not be violated, and
no search warrant or warrant of arrest shall issue
except upon probable cause to be determined by the
judge, or such other responsible officer as may be
authorized by law, after examination under oath or
affirmation of the complainant and the witnesses he
may produce, and particularly describing the place to
be searched, and the persons or things to be seized. 41
Parenthetically, it may be noted that Section 3, Rule III of
the 1935 Constitution mentioned only "the judge" as having
power to determine probable cause and issue search and
arrest warrants.
The matter is further dealt with in Section 143 of Batas
Pambansa Bilang 337, otherwise known as the Local
Government Code, which took effect on February 10, 1983.
This section evidently deems the mayor a "responsible
officer" in contemplation of the cited constitutional
provision, and explicitly authorizes him to conduct
preliminary examination in criminal cases and order the
arrest of the accused upon probable cause.
SEC. 143. Authority of the Mayor to Conduct
Preliminary Examination.
(1) In case of temporary absence of the judge
assigned to the municipalities the mayor may
conduct the preliminary examination in criminal
cases when, in his opinion the investigation cannot
be delayed without prejudice to the interest of
justice.

(2) No examination shall be conducted unless the


parties are assisted by lawyers.
(3) In cases where he may conduct preliminary
examination, the mayor shall upon probable cause
after examination of witnesses, have authority order
the arrest of the accused and to grant him bail in the
manner and cases provided in the Revised Rules of
Court and order his provisional release.
(4) The mayor shall make a report of any
preliminary examination so made immediately after
the return of the judge assigned in the area, or upon
the designation of his replacement. 42
The 1973 Constitution plainly and unmistakably grants to
the legislature the power to determine which " responsible
officers, " aside from judges, may issue warrants of arrest
after examination under oath or affirmation of the
complainant and the witnesses he may produce. And as
plainly and unmistakably, the legislature, the Batas
Pambansa, has in the Local Government Code made the
determination that the mayor is such a "responsible officer
"and has in consequence authorized him to conduct
preliminary investigations in criminal cases and order the
arrest of the accused upon a finding of probable cause.
The first issue raised by the petitioners that in
September ,1985 the mayor no longer had power to conduct
preliminary investigations and issue arrest warrants 43
must therefore be resolved against them. The argument that
Section 143 of the Local Government Code is just "a rule of
procedure merely having incorporated Rule 112, Section 2 of
the old rules of Criminal Procedure," and should therefore be
deemed "superseded by the New Rules of Criminal
Procedure pursuant to the power of this ** Court to
promulgate rules of procedure (Article X, Section 5 151,
Constitution **)"44 cannot be sustained. However superior
the Supreme Court may be to the other branches of
government in the realm of adjudication, its power to ordain

rules of court was at the time inferior to the lawmaking


power of the legislature.45 It is true that the 1935 Constitution
repealed all procedural laws then in force "as statutes" and
declared them to be "rules of court;" but this was only so that
they could be subject to repeal or modification by the
Supreme Court, which was given the power to promulgate
rules of procedure,46 and has since been exercising such
power by promulgating the Rules of Court of 1940 and of
1964, and the 1985 Rules on Criminal Procedure, etc. But
the 1935 and 1973 Constitutions explicitly conferred on the
legislature the power to repeal, alter or supplement those
rules of court, although it would appear that that power is no
longer granted to it by the 1987 Charter.47
Now, the conditions under which the mayor could conduct
preliminary investigations are also clearly indicated by the
law, to wit:
1) in case of the temporary absence of the judge
assigned to the municipality; and
2) in his (the mayor's) opinion, the investigation
cannot be delayed without prejudice to the interests
of justice.
On these occasions, the mayor may order the arrest of the
accused upon a finding of probable cause after searching
examination of the complainant and the witnesses the latter
may present.
In the case at bar, there is no showing that these conditions
have not been met. Indeed, quite apart from the presumption
that official duty has been regularly performed. 48 and the
affirmative statements of respondent official that on that
fateful 20th day of September, 1985, Judge Ignalaga was in
Manila"49 on official leave or absence,"50 the petitioners have
not put the fulfillment of those conditions at issue at all, or
the matter of whether or not the accused were "assisted by
lawyers" in the course of the investigation.51

The petitioners also challenge the constitutionality of Section


143 of the Local Government Code, it being claimed that the
mayor has been thereby authorized to conduct pre
investigations and issue warrants of arrest, although he can
not be deemed a "neutral and detached magistrate" within
the contemplation of Section 3, Article IV of the 1973
Constitution.52
The competence of the Batasang Pambansa to decide and
declare by statute which "responsible officers," aside from
judges, should be entrusted with the authority to conduct
preliminary investigations and issue arrest warrants cannot
be doubted in view of the clear language of the
Constitution.53 And the determination by the Batasan that a
municipal mayor is one such "responsible officer" who may
properly be entrusted with the function of conducting
preliminary investigations and ordering arrests of suspects
upon probable cause, can not be subject of judicial review,
absent any indication that the legislative proceedings leading
to that statutory determination are void on account of some
grave cause. Certainly, the wisdom of the statute, or the
validity of the reasons underlying it, or the adequacy of the
statistics, facts and circumstances considered by the
legislature in its enactment, are beyond the sphere of inquiry
of the courts54 The Batasan was apparently of the belief that
a municipal mayor could be sufficiently objective and
impartial as to be relied upon to conduct preliminary
investigations and issue orders of arrest in the exceptional
situation when the judge assigned in the municipality was
absent. The petitioners disagree. They contend that the
mayor cannot in the very nature of things be "neutral and
detached." The disagreement, and the fact that plausible
reasons may be adduced by one side or the other on the
proposition does not make the question a justiciable one. The
theory advocated by the petitioners that the mayor's "deep
involvement in law enforcement functions is likely to color
his judgment as a trier of probable cause,"55 does not induce
persuasion. In the first place the premise cannot be
conceded. While it is true that the mayors do "exercise
general supervision over units and elements of the INP

stationed or assigned in their respective jurisdictions," they


are not themselves directly involved in police work and
cannot in any sense be described, as the petitioners do, as
being deeply involved in law enforcement functions. And
even if that "deep involvement" be conceded, it does not
follow that this would necessarily preclude their assuming
"the cold neutrality of an impartial judge" in conducting
preliminary investigations of persons suspected of crimes.

of the Municipal Trial Courts and Municipal Circuit Trial


Courts; (3) national and regional state prosecutors; and (d)
such other officers as may be authorized by law.57 But only
"the judge" may issue search and arrest warrants after due
determination of probable cause.58
The petitioners' assault, on the other hand, upon the
constitutionality of Article 141 of the Revised Penal Code,
defining and penalizing the felony of inciting to sedition,
upon the claim that it was "borrowed" from the U.S. Sedition
Act of 1798 which in turn has been struck down as
inconsistent with the First Amendment of the American
Constitution,59 is repelled by the Solicitor General's
arguments.60

But it must be emphasized here and now that what has just
been described is the state of the law as it was in September,
1985. The law has since been alter. No longer does the
mayor have at this time the power to conduct preliminary
investigations, much less issue orders of arrest. Section 143
of the Local Government Code, conferring this power on the
In the first place, ** our law on inciting to sedition is
mayor has been abrogated, rendered functus officio by the
not akin to the US Sedition Act of 1798 which was
1987 Constitution which took effect on February 2, 1987, the
imposed on the American colonies by their British
date of its ratification by the Filipino people. Section 2,
ruler. With the success of the American revolution,
Article III of the 1987 Constitution pertinently provides that
the 1798 Sedition Act naturally ceased to have effect
"no search warrant or warrant of arrest shall issue except
as it would be utterly incongrous to punish those
upon probable cause to be determined personally by the
who sought the overthrow of the British government
judge after examination under oath or affirmation of the
in America.
complainant and the witnesses he may produce, and
particularly describing the place to be searched and the
xxx
xxx
xxx
person or things to be seized. " The constitutional
proscription has thereby been manifested that thenceforth,
xxx
xxx
xxx
the function of determining probable cause and issuing, on
the basis thereof, warrants of arrest or search warrants, may
To annul our law on sedition is to give license to
be validly exercised only by judges, this being evidenced by
those who seek the application of lawless methods in
the elimination in the present Constitution of the phrase,
the advancement of their political views. Our
"such other responsible officer as may be authorized by law"
constitution surely does not contemplate this.
found in the counterpart provision of said 1973
56
Constitution, who, aside from judges, might conduct
Finally, the petitioners postulate that in the determination of
preliminary investigations and issue warrants of arrest or
the existence of probable cause from the constitutional
search warrants.
aspect, it is required that: "(1) The judge (or) officer must
examine the ** witnesses personally; (2) The examination
As the law now stands, the mayor may no longer conduct
must be under oath; and (3) The examination must be
preliminary investigation, the authority to do so being
reduced to writing in the form of searching questions and
limited under Section 2, Rule 1 1 2 of the Rules of Court to
answers. (Marinas v. Siochio, 104 SCRA 403)."
(1) provincial or city fiscals and their assistants; (2) judges

In the light of these principles they contend: firstly, that the


mayor's questioning of the witnesses was not "searching"
enough; and secondly, that the witnesses' testimony does not
establish prima facie the commission of the felony of
inciting to sedition.

Marcos, pamatukan ang NSL, ibagsak ang military


kagwalaon ang CHDF, suklan nato ang gobyerno," a
woman speaker had said the same thing, and a third speaker,
petitioner Daniel Gempesala, also stated: Lumpagon and
dictatorial na regimen ni Marcos, kag ibagsak ang
Gobyerno;"66 and they had elicited a response from the
The fact is that is shown by the record, questions about the
people there who had raised their right hands and repeated
material events were in truth propounded by Mayor
the shouted "slogan."67 Carbajosa in his turn declared that
61
Lumayno to the chief witness. Capt. Sanson; and no proof the demonstrators ** were shouting for trouble and ready to
to the contrary has been submitted. The circumstance that the make revolution with the use of arms."68
answers given by Capt. Sanson to the mayor's questions are
closely reflective of the contends of his affidavit should not Another affiant, Eduardo Flores, stated that the
come as a surprise and cannot, without more, be taken as
demonstrators were unruly, "tumultuously shouting seditious
debilitating or nullifying the interrogation.
words against the government and shouting for revolution."69

WHEREFORE, the writs of certiorari and prohibition are


granted. The order of respondent Mayor Lumayno issued on
September 20, 1985 and the resolution promulgated by
respondent Judge Ignalaga on October 11, 1985 are annulled
and set aside, and the respondents are perpetually forbidden
to enforce or in any way implement the orders for the arrest
of any of the petitioners. No costs.
ROGELIO ABERCA, et al. vs. FABIAN VER, et al.
L-69866
April 15, 1988
FACTS:

The petitioner's second point is much more substantial and is


decisive of the controversy. The petitioners are correct in
their claim that Capt. Sanson's testimony62 does not in truth
contain any facts demonstrating the actuality of the crime of
inciting to sedition, which is the crime for which arrest
warrants were issued by Mayor Lumayno. The facts recited
by Capt. Sanson may perhaps warrant a charge of traffic
obstruction, or grave coercion, or malicious mischief, or
illegal possession of firearms or deadly weapons, or maybe,
attempted murder or homicide. But it is barren of facts to
support an accusation of inciting to sedition.

The sworn statement of the only other witness, Leopoldo


Sometime in the early 1980s, various Intelligence
Villalon,70 is totally innocuous as far as proving the elements
units
of
the
AFP known as Task Force Makabansa (TFM)
of inciting to sedition is concerned.
were ordered by respondents then Maj. Gen. Fabian Ver to
conduct pre-emptive strikes against known communistThe evidence can not justify the action taken by the
terrorist (CT) underground houses in view of increasing
respondent Mayor and Judge. The Court thus declares as
reports about CT plans to sow disturbances in Metro
sorely inadequate and mortally defective the avowed
Manila. In compliance thereof, the TFM raided several
evidentiary foundation for Mayor Lumayno's finding of
probable cause respecting the commission by the petitioners places, employing in most cases defectively issued judicial
of the crime of inciting to sedition. The affiants declarations search warrants. During these raids, certain members of the
in their sworn statements which might otherwise be pertinent raiding TFM confiscated a number of purely personal items
to the offense, are generalities, mere conclusions of their, not belonging to the 20 petitioners. Petitioners were arrested
without proper arrest warrants issued by the courts. For
The other evidence on record exhibits the same barreness.
positive averments of particular facts within their personal
some period after their arrest, they were arrested without
knowledge. 'They do not Identify the specific persons
Two witnesses, Godofredo Hoyo-A and Elpidio Carbajosa,
supposed to have perpetrated the crime charged, except two. denied visits of relatives and lawyers; interrogated in
advert in their affidavits63 to the shouting of "seditious
But even the Identification of these two is of no moment. For violation of their rights to silence and counsel, through
words," etc. Hoyo-A is quoted as deposing that "the
except as regards Capt. Sanson, whose testimony, to repeat, threats, torture and other forms of violence in order to obtain
incriminatory information or confessions and in order to
demonstrators were already unruly ** (and kept) on
is in any case ineffectual to prove the precise offense
punish them.
shouting anti-government slogans, ** invectiues and
ascribed to the petitioners, there had been no searching
64
seditious words against the government," that certain of
interrogation by Mayor Lumayno of the witnesses as
Plaintiffs then filed an action for damages before
the petitioners were Identified as being "against the
required by the Constitution. Hence, whatever credit could
the RTC of Quezon City against respondents-officers of the
government, ** and using the (Mt. Carmel) School in
possibly have been accorded to the affidavit of Hoyo-A
65
AFP headed by Ver. Respondents, in their motion to dismiss,
doctrining (sic) the students against the government," that which ventures to quote the exact words allegedly shouted
claimed that (1) the wrti of habeas corpus was suspended,
petitioner Rogelio Arnaiz had delivered a speech in which he by petitioners Arnaiz and Gempesala, and an unidentified
thus giving credence to petitioners detention; (2)
had said, "Rumpagon ang dictatorial na regimen ni Pres.
woman was thereby effectively foreclosed.

respondents were immune from liability for acts done in the arrest or detention. What is suspended is merely the right of
performance of their official duties, and that (3) the the individual to seek release from detention through the writ
complaint did not state a cause of action against of habeas corpus as a speedy means of obtaining his liberty.
respondents.
Moreover, as pointed out by petitioners, their right
On November 8, 1983, the RTC granted the and cause of action for damages are explicitly recognized in
motion to dismiss the case. A motion to set aside the order PD 1755 which amended Art. 1146 of the Civil Code by
dismissing the complaint, and a supplemental motion for adding the following text: However, when the action (for
reconsideration were filed by petitioners. On May 11, 1984, injury to the rights of the plaintiff or for quasi-delict) arises
the trial court, without acting on the motion to set aside the from or out of any act, activity or conduct of any public
Order of Nov. 8, 1983, declared the finality of said Order officer involving the exercise of powers or authority arising
against petitioners. After their motion for reconsideration from martial law including the arrest, detention and/or trial
was denied by the RTC, petitioners then filed the instant of the plaintiff, the same must be brought within one year.
petition for certiorari, on March 15, 1985, seeking to annul
Even assuming that the suspension of the PWHC
and set aside the respondent courts resolutions and order.
suspends petitioners right of action for damages for illegal
ISSUES:
arrest and detention, it does not and cannot suspend their
rights and causes of action for injuries suffered because of
(1) Whether or not the suspension of the privilege of the respondents confiscation of their private belongings, the
writ of habeas corpus bars a civil action for damages for violation of their right to remain silent and to counsel and
illegal searches conducted by military personnel and other their right to protection against unreasonable searches and
violations of rights and liberties guaranteed under the seizures and against torture and other cruel and inhuman
Constitution;
treatment.
(2)
Whether or not respondents may invoke state
immunity from suit for acts done in the performance of
official duties and functions;
(3)
Whether or not a superior officer, under the notion of
respondeat superior, be answerable for damages jointly and
severally with his subordinates, to the person whose
constitutional rights and liberties have been violated.
HELD:
(1) The suspension of the privilege of the writ of
habeas corpus (PWHC) does not destroy petitioners right
and cause of action for damages for illegal arrest and
detention and other violations of their constitutional rights.
The suspension does not render valid an otherwise illegal

liberties of the individual citizen enshrined and protected by


the Constitution.
Article 32 of the Civil Code, which renders any
public officer or employees, or any private individual, liable
in damages for violating the constitutional rights and
liberties of another, does not exempt the respondents from
responsibility. Only judges are excluded from liability under
the said article, provided their acts or omissions do not
constitute a violation of the Revised Penal Code or other
penal statute.

This is not say that military authorities are


restrained from pursuing their assigned task or carrying out
their mission with vigor, to protect the Philippines from its
enemies, whether of the left or of the right, or from within or
without, seeking to destroy or subvert our democratic
institutions and imperil their very existence. What is meant
is that in carrying out their task and mission, constitutional
and legal safeguards must be observed; otherwise, the very
fabric of our faith will start to unravel. In the battle of
competing ideologies, the struggle of mind is just as vital as
the struggle of arms. The linchpin in that psychological
struggle is faith in the rule of law. Once that faith is lost or
The question became moot and academic since the compromised, the struggle may well be abandoned.
suspension of the PWHC had been lifted with the issuance of
then Pres. Corazon Aquino of Proclamation No. 2 on March
(3) The doctrine of respondeat superior is not
25, 1986.
applicable in this case. It has been generally limited in its
application to principal and agent or to master and servant
(2) It may be that the respondents, as members of relationships. No such relationship exists superiors of the
the AFP, were merely responding to their duties, as they military and their subordinates. However, the decisive factor
claim, to prevent or suppress lawless violence, insurrection, in this case is the language of Art. 32, Civil Code; the law
rebellion and subversion in accordance with Proclamation speaks of an officer or employee or person directly or
No. 2054 of Pres. Marcos, despite the lifting of Martial Law indirectly responsible for the violation of the constitutional
on January 27, 1981, and in pursuance of such objective, to rights and liberties of another. Thus, it is not the actor alone
launch pre-emptive strikes against alleged CT underground who must answer for damages under Art. 32; the person
houses. But this cannot be construed as a blanket license or indirectly responsible has also to answer for the damages or
roving commission untrammeled by any constitutional injury caused to the aggrieved party. Art. 32 makes the
restraint, to disregard or transgress upon the rights and persons who are directly as well as indirectly responsible for
the transgression joint tortfeasors.

Valmonte v. De Villa, 170 SCRA 256 (1989)


F: On 1/20/87, the NCRDC was activated w/ the mission of
conducting security operations w/in its area or responsibility
and peripheral areas, for the purpose of establishing an
effective territorial defense, maintaining peace and order,
and providing an atmosphere conducive to the social,
economic and political dev''t of the NCR. As part of its duty
to maitain peace and order, the NCRDC installed
checkpoints in various parts of Valenzuela and MM.
Petitioners aver that, bec. of the institution of said
checkpoints, the Valenzuela residents are worried of being
harassed and of their sarety being placed at the arbitrary,
capricious and whimsical disposition of the military
manning the checkpoints, considering that their cars and
vehicles are being subjected to regular searches and checkups, especially at night or at dawn, w/o a SW and/ or court
order. Their alleged fear for their safety increased when
Benjamin Parpon, was gaunned down allegedly in cold
blood by members of the NCRDC for ignoring and/ or
continuing to speed off inspite of warning shots fired in the
air. HELD: Petitioner''s concern for their safety and
apprehension at being harassed by the military manning the
checkpoints are not sufficient grounds to declare the
checkpoints per se, illegal. No proof has been presented
before the Court to show that, in the course of their routine
checks, the military, indeed, committed specific violations of
petitioners'' rights against unlawful search and seizure of
other rights. The constitutional right against unreasonable
searches and seizures is a personal right invocable only by
those whose rights have been infringed, or threatened to be
infringed.
Not all searches and seizures are prohibited. Those w/c are
reasonable are not forbidden. The setting up of the
questioned checkpoints may be considered as a security
measure to enable the NCRDC to pursue its mission of
establishing effective territorial defense and maintaining
peace and order for the benfit of the public. Checkpoints
may not also be regarded as measures to thwart plots to
destabilize the govt, in the interest of public security.
Between the inherent right of the state to protect its existence
and promote public welfare and an individual''s right against
a warrantless search w/c is, however, reasonably conducted,
the former should prevail. True, the manning of checkpoints
by the military is susceptible of abuse by the military in the
same manner that all governmental power is susceptible of

abuse. But, at the cost of occasional inconveninece,


discomfort and even irritation to the citizen, the checkpoints
during these abnormal times, when conducted w/in
reasonable limits, are part of the price we pay for an orderly
society and a peaceful community.
** Valmonte v De Villa 185 SCRA 665 (1989)F:
On 1/20/87, the NCRDC was activated w/ the mission of
conducting security operations w/in its area or responsibility
and peripheral areas, for the purpose of establishing an
effective territorial defense, maintaining peace and order,
and providing an atmosphere conducive to the social,
economic and political dev't of the NCR. As part of its duty
to maitain peace and order, the NCRDC installed
checkpoints in various parts of Valenzuela and MM.
Petitioners aver that, bec. of the institution of said
checkpoints, the Valenzuela residents are worried of being
harassed and of their sarety being placed at the arbitrary,
capricious and whimsical disposition of the military manning
the checkpoints, considering that their cars and vehicles are
being subjected to regular searches and check-ups, especially
at night or at dawn, w/o a SW and/ or court order. Their
alleged fear for their safety increased when Benjamin
Parpon, was gaunned down allegedly in cold blood by
members of the NCRDC for ignoring and/ or continuing to
speed off inspite of warning shots fired in the air.
HELD: Petitioner's concern for their safety and
apprehension at being harassed by the military manning the
checkpoints are not sufficient grounds to declare the
checkpoints per se, illegal. No proof has been presented
before the Court to show that, in the course of their routine
checks, the military, indeed, committed specific violations of
petitioners' rights against unlawful search and seizure of
other rights.
The constitutional right against unreasonable
searches and seizures is a personal right invocable only by
those whose rights have been infringed, or threatened to be
infringed.
Not all searches and seizures are prohibited. Those
w/c are reasonable are not forbidden.
The setting up of the questioned checkpoints may be
considered as a security measure to enable the NCRDC to
pursue its mission of establishing effective territorial defense
and maintaining peace and order for the benfit of the public.

Checkpoints may not also be regarded as measures to thwart


plots to destabilize the govt, in the interest of public security.
Between the inherent right of the state to protect its
existence and promote public welfare and an individual's
right against a warrantless search w/c is, however,
reasonably conducted, the former should prevail.
True, the manning of checkpoints by the military is
susceptible of abuse by the military in the same manner that
all governmental power is susceptible of abuse. But, at the
cost of occasional inconveninece, discomfort and even
irritation to the citizen, the checkpoints during these
abnormal times, when conducted w/in reasonable limits, are
part of the price we pay for an orderly society and a peaceful
community.

** People v Malmstedt 198 SCRA 401 (1991)


Facts: In an information filed against the accused- appellant
Mikael Malmstead was charged before the RTC of La
Trinidad, Benguet, for violation of Section 4, Art. II of
Republic Act 6425, as amended, otherwise known as the
Dangerous Drugs Act of 1972, as amended.
Accused Mikael Malmstedt, a Swedish national, entered the
Philippines for the third time in December 1988 as a tourist.
He had visited the country sometime in 1982 and 1985.
In the evening of 7 May 1989, accused left for Baguio City.
Upon his arrival thereat in the morning of the following day,
he took a bus to Sagada and stayed in that place for two (2)
days. Then in the 7 in the morning of May 11, 1989, the
accused went to Nangonogan bus stop in Sagada.
At about 8: 00 o'clock in the morning of that same day (11
May 1989), Captain Alen Vasco, the Commanding Officer of
the First Regional Command (NARCOM) stationed at Camp
Dangwa, ordered his men to set up a temporary checkpoint
at Kilometer 14, Acop, Tublay, Mountain Province, for the
purpose of checking all vehicles coming from the Cordillera
Region. The order to establish a checkpoint in the said area
was prompted by persistent reports that vehicles coming
from Sagada were transporting marijuana and other
prohibited drugs. Moreover, information was received by the
Commanding Officer of NARCOM, that same morning that
a Caucasian coming from Sagada had in his possession

prohibited drugs. The group composed of seven (7)


NARCOM officers, in coordination with Tublay Police
Station, set up a checkpoint at the designated area at about
10:00 o'clock in the morning and inspected all vehicles
coming from the Cordillera Region.

derivative of marijuana. Thus, an information was filed


against accused for violation of the Dangerous Drugs Act.

(a) When, in his presence, the person to be arrested has


committed is actually committing, or is attempting to
commit an offense;

ACCUSEDS DEFENSE

During the arraignment, accused entered a plea of "not


The two (2) NARCOM officers started their inspection from guilty." For his defense, he raised the issue of illegal search
the front going towards the rear of the bus. Accused who was of his personal effects. He also claimed that the hashish was
the sole foreigner riding the bus was seated at the rear
planted by the NARCOM officers in his pouch bag and that
thereof.
the two (2) travelling bags were not owned by him, but were
merely entrusted to him by an Australian couple whom he
During the inspection, CIC Galutan noticed a bulge on
met in Sagada. He further claimed that the Australian couple
accused's waist. Suspecting the bulge on accused's waist to
intended to take the same bus with him but because there
be a gun, the officer asked for accused's passport and other
were no more seats available in said bus, they decided to
identification papers. When accused failed to comply, the
take the next ride and asked accused to take charge of the
officer required him to bring out whatever it was that was
bags, and that they would meet each other at the Dangwa
bulging on his waist. The bulging object turned out to be a
Station.
pouch bag and when accused opened the same bag, as
ordered, the officer noticed four (4) suspicious-looking
The trial court found the guilt of the accused Mikael
objects wrapped in brown packing tape, prompting the
Malmstedt established beyond reasonable doubt.
officer to open one of the wrapped objects. The wrapped
objects turned out to contain hashish, a derivative of
Seeking the reversal of the decision of the trial court finding
marijuana.
him guilty of the crime charged, accused argues that the
search of his personal effects was illegal because it was
Thereafter, accused was invited outside the bus for
made without a search warrant and, therefore, the prohibited
questioning. But before he alighted from the bus, accused
drugs which were discovered during the illegal search are
stopped to get two (2) travelling bags from the luggage
not admissible as evidence against him.
carrier. Upon stepping out of the bus, the officers got the
bags and opened them. A teddy bear was found in each bag.
Feeling the teddy bears, the officer noticed that there were
Issue: Whether or Not the contention of the accused is valid,
bulges inside the same which did not feel like foam stuffing. and therefore the RTC ruling be reversed.
It was only after the officers had opened the bags that
accused finally presented his passport.
Held: The Constitution guarantees the right of the people to
Accused was then brought to the headquarters of the
be secure in their persons, houses, papers and effects against
NARCOM at Camp Dangwa, La Trinidad, Benguet for
unreasonable searches and seizures. However, where the
further investigation. At the investigation room, the officers search is made pursuant to a lawful arrest, there is no need to
opened the teddy bears and they were found to also contain
obtain a search warrant. A lawful arrest without a warrant
hashish. Representative samples were taken from the hashish may be made by a peace officer or a private person under the
found among the personal effects of accused and the same
following circumstances.
were brought to the PC Crime Laboratory for chemical
analysis.
Sec. 5 Arrest without warrant; when lawful. A peace
officer or a private person may, without a warrant, arrest a
In the chemistry report, it was established that the objects
person:
examined were hashish. a prohibited drug which is a

(b) When an offense has in fact just been committed, and he


has personal knowledge of facts indicating that the person to
be arrested has committed it; and
(c) When the person to be arrested is a prisoner who has
escaped from a penal establishment or place where he is
serving final judgment or temporarily confined while his
case is pending, or has escaped while being transferred from
one confinement to another.
Accused was searched and arrested while transporting
prohibited drugs (hashish). A crime was actually being
committed by the accused and he was caught in flagrante
delicto. Thus, the search made upon his personal effects falls
squarely under paragraph (1) of the foregoing provisions of
law, which allow a warrantless search incident to a lawful
arrest. While it is true that the NARCOM officers were not
armed with a search warrant when the search was made over
the personal effects of accused, however, under the
circumstances of the case, there was sufficient probable
cause for said officers to believe that accused was then and
there committing a crime.
Probable cause has been defined as such facts and
circumstances which could lead a reasonable, discreet and
prudent man to believe that an offense has been committed,
and that the objects sought in connection with the offense are
in the place sought to be searched. Warrantless search of the
personal effects of an accused has been declared by this
Court as valid, because of existence of probable cause,
where the smell of marijuana emanated from a plastic bag
owned by the accused, 10 or where the accused was acting
suspiciously, 11 and attempted to flee.
The appealed judgment of conviction by the trial court is
hereby affirmed. Costs against the accused-appellant.

Alih vs Castro
GL. 69401

This case was in explaining Sec 3 of Art 2 of the 1987


Constitution regarding the supremacy of the civilians.

THE PEOPLE OF THE PHILIPPINES, plaintiffappellee,


vs.
ARNULFO CENDANA y REYES, accused-appellant.

Edmundo M. Manaois for accused-appellant.

there, wilfully, unlawfully


and feloniously shoot one
Dominador Manongdo y
Loresco, inflicting upon him
a gunshot wound which
caused his instantaneous
death, to the damage and
prejudice of his heirs.

CORTES, J.:

Contrary to Article 248 of


the Revised Penal Code.
[Record, p. 1].

Facts;
The Solicitor General for plaintiff-appellee.
Zona was conducted by the men of Maj. Gen Castro in a
compoud where petioners reside and conducted illegal
search and thereafter seized guns from them. The order was
carried on by his Castro's men with the contention that the
petitioners are involved in the latest killing of the town's
mayor Cesar Climaco.
Issue;
Is the warrantless search and seizure legal?
Held;
The Supreme Court declared those seized in custodia legis
and declared that the operation conducted by Maj. Gen.
Castro was ILLEGAL. The respondents have all the time to
obtain a search warrant granted that they have about 10 trial
courts. The SC also held the protection of the petitioner's
human rights as stated in Art IV Sec 3 and 4 of the 1973
Constitution regarding illegal search and seizure. The
presumption of innocence of the petitioners should be
observed and that they cannot be subjected to selfincriminating instances like paraffin tests, photographing and
finger printing.
As penned by J. Cruz in this case, "The Constitution is a law
for rulers and people, equally in war and in peace, and
covers with the shield of its protection all classes of men, at
all times and under all circumstances. No doctrine, involving
more pernicious consequences, was ever invented by the wit
of man than that any of its provisions can be suspended
during any of the great exigencies of government." G.R. No.
84715 October 17, 1990

In the early morning of November 24, 1986, the Police


Station at Sta. Barbara, Pangasinan received a report that a
man was found dead on the field near the ricemill of a Mrs.
Thelma Bautista at Barangay Ventinilla West, Sta. Barbara,
Pangasinan. Upon investigation, policemen found the
cadaver, Identified to be that of Dominador Manongdo, lying
prostrate on the ground with one gunshot wound on the head.

Upon arraignment, accused-appellant pleaded not guilty. The


pre-trial conference was terminated on November 2, 1987
after which, trial proceeded.

Subsequently, accused-appellant Arnulfo Cendana y Reyes


was apprehended and later charged with the crime of murder
"thru illegally possessed firearm" on the basis of the
following information:

WHEREFORE, the Court


finds accused Arnulfo
Cendana y Reyes guilty
beyond reasonable doubt of
the crime of Murder thru
Illegally Possessed Firearm,
and considering that the
crime of Murder was
committed with the use of
an unlicensed firearm
(shotgun, Exhibit "H"), the
accused is sentenced to
suffer the penalty of
Reclusion Perpetua with all
the accessory penalties
provided by law, to
indemnify the heirs of the
deceased Dominador
Manongdo y Loresco the

That on or about November


23, 1986, in the evening at a
ricefield in barangay
Ventinilla West,
municipality of Sta.
Barbara, province of
Pangasinan, Philippines and
within the jurisdiction of
this Honorable Court, the
above-named accused,
armed with an unlicensed
shotgun, with intent to kill,
with treachery and evident
premeditation, did then and

The trial judge convicted accused-appellant in a decision


dated June 3, 1988, the dispositive portion reading as
follows:

sum of P30,000.00, and to


pay the costs.
The firearm is ordered
confiscated and forfeited in
favor of the Government,
and the Clerk of this Court
is ordered to turn over the
firearm to the nearest
Constabulary Command.
[Record, p. 121].
Not agreeing with the findings of the trial court, accusedappellant appeals to this Court raising the following
assignment of errors:
I. The trial court erred in convicting the
accused even without proof beyond
reasonable doubt.
II. The trial court erred in not appreciating
the defense of the accused.
III. The trial court erred in convicting the
accused based on shaky and crude
circumstantial evidence. [Rollo, p. 43].
At the outset, it is noted that the Solicitor General filed a
manifestation recommending the acquittal of accusedappellant. He asserts that the alleged admission made by the
accused-appellant and the gun seized from him are both
inadmissible in evidence for having been obtained in
violation of accused-appellant's constitutional rights. He
further asserts that after the exclusion of such evidence, the
remaining evidence of the prosecution is clearly insufficient
to support a conviction based on proof beyond reasonable
doubt [Rollo, pp. 84-85].
We agree.

To support its judgment of conviction, the trial court relied


mainly on the testimonies of the Police Station Commander
Sgt. Amadeo Asuncion, Pat. Alden Poserio, and Pat.
Fernando Quinto, who were the police officers investigating
the case. Their testimonies sought to prove the following:
After the cadaver was brought to the morgue and later to a
funeral parlor for autopsy, they went back to the place where
the body was found and after interviewing some people,
received information that accused-appellant was seen before
the incident carrying a gun [TSN, November 3, 1987, p. 8].
When they went to accused-appellant's house, he was not
around and so they picked up one of his brothers, Antonio
Cendana, who informed them that accused-appellant was at
Pogo District in Dagupan City [TSN, November 3, 1987, pp.
8-9; February 1, 1988, pp. 5-6]. Not knowing where such
place was, they forced Antonio Cendana to accompany them
to the house where accused-appellant could be found [TSN,
November 3, 1987, p. 9; February 1, 1988, p. 6]. Upon their
arrival, Sgt. Asuncion and Pat. Quinto entered the house
while Pat. Poserio remained outside [TSN, November 3,
1987, p. 10]. They were able to arrest accused-appellant who
afterwards admitted to them that he was the one who shot the
victim [TSN, February 1, 1988, p. 7]. They also recovered a
homemade shotgun which, according to Pat. Quinto, was
voluntarily handed to them by accused-appellant [TSN,
November 4, 1987, p. 4] but according to Sgt. Asuncion was
handed to them by a woman relative of accused-appellant
upon the latter's instructions [TSN, February 1, 1988, p. 7].
They then proceeded to the NBI, Dagupan City where
accused-appellant was subjected to a paraffin test and the
recovered firearm surrendered for ballistics examination
[TSN, November 3, 1987, p. 11; February 1, 1988, p. 8].
From the NBI, they proceeded to the Sta. Barbara Police
Station where accused was thereafter detained [TSN,
November 3, 1987, p. 12; February 1, 1988, p. 8]. No written
statement was taken from accused-appellant [TSN,
November 3, 1987, p. 12].
From the foregoing narration of events, we note the
following:

(1) That there was no eye witness to the killing of the victim
Dominador Manongdo;
(2) That the accused-appellant was apprehended by the
police investigators on the basis of information obtained
from unidentified persons that accused-appellant was seen
carrying a gun before the incident.
(3) That the accused-appellant was apprehended by the
police officers without any warrant of arrest;
(4) That the shotgun was recovered without a search warrant
from the house where accused-appellant was arrested; and
(5) That the alleged statement made by accused-appellant to
the police officers admitting to the commission of the
offense and made after his arrest, was used as the main basis
for his conviction.
I
Section 5, Rule 113 of the 1985 Rules of Criminal Procedure
enumerates the instances when a peace officer or a private
person may arrest a person without a warrant:
(a) When, in his presence, the person to be
arrested has commited is actually
committing, or is attempting to commit an
offense;
(b) When an offense has in fact just been
committed, and he has personal knowledge
of facts indicating that the person to be
arrested has committed it; and
(c) When the person to be arrrested is a
prisoner who has escaped from a penal
establishment or place where he is serving
final judgment or temporarily confined
while his case is pending, or has escaped

while being transferred from one


confinement to another.
The facts of the case do not warrant the applicability of
paragraphs (a) and (c). Moreover, in paragraph (b), the only
instance under which accused-appellant's case could possibly
fall, what is essential is that the person making the arrest has
personal knowledge of the facts indicating that the arrestee is
responsible for an offense which has just been committed
[People v. Burgos, G.R. No. 68955, September 4, 1986, 144
SCRA 1]. Accused-appellant was arrested one day after the
killing of the victim and only on the basis of information
obtained by the police officers from unnamed sources. These
abovementioned circumstances clearly belie a lawful
warrantless arrest.
Considering that the arrest of accused-appellant herein was
unlawful, any search conducted on his person or place of
arrest which is an incident thereof, was also unlawful
[People v. Burgos, supra]. Perforce, any evidence recovered
during the unlawful search, being made without a warrant,
becomes inadmissible in evidence against accused-appellant
and the shotgun which was allegedly the fatal weapon cannot
be presented against him [Nolasco v. Pano, G.R. No. 69803,
January 30, 1987, 147 SCRA 509].
Section 20, Article IV of the 1973 Constitution ordains that:
No person shall be compelled to be a
witness against himself Any person under
investigation for the commission of an
offense shall have the right to remain silent
and to counsel, and to be informed of such
right. No force, violence, threat,
intimidation, or any other means which
vitiates the free will shall be used against
him. Any confession obtained in violation of
this section shall be inadmissible in
evidence.

The Court elaborated on the scope of this right in the case of


Morales, Jr. v. Enrile [G.R. No. 61016, April 26, 1983, 121
SCRA 538], thus:

This procedure served as the guideline in subsequent cases


[People v. Ramos, G.R. No. 59318, May 16, 1983, 122
SCRA 312; People v. Galit, G.R. No. 51770, March 20,
1985, 135 SCRA 465]. In the present case, if is clear from
At the time a person is arrested, it shall be
the record that at the time that accused-appellant was
the duty of the arresting officer to inform
arrested, he was not apprised of the right to remain silent and
him of the reason for the arrest and he must to counsel, and to be informed of such rights, before he
be shown the warrant of arrest, if any. He
supposedly admitted to the killing of the deceased. As can be
shall be informed of his constitutional rights gleaned from the testimony of Sgt. Asuncion:
to remain silent and to counsel, and that any
statement he might make could be used
Q Were you able to locate
against him. The person arrested shall have
Arnulfo Cendana?
the right to communicate with his lawyer, a
A Yes, sir.
relative, or anyone he chooses by the most
expedient means-by telephone if possible-or
Q Where?
by letter or messenger. It shall be the
responsibility of the arresting officer to see
A At the house of one of his
to it that this is accomplished. No custodial
relatives, sir.
investigation shall be conducted unless it be
in the presence of counsel engaged by the
Q What happened when you
person arrested, by any person on his behalf,
arrived in that house?
or appointed by the court upon petition
either of the detainee himself or by anyone
A When we arrived in the
on his behalf. The right to counsel may be
house I asked one of the
waived but the waiver shall not be valid
occupants of the house the
unless made with the assistance of counsel.
whereabouts of Arnulfo
Any statement obtained in violation of the
Cendana and I told one of
procedure herein laid down, whether
my men to surround the
exculpatory or inculpatory, in whole or in
house. I tried to went (sic)
part, shall be inadmissible in evidence. [At
up to the house but I saw
p. 554]
Arnulfo Cendana trying to
escape so I told him "agka
ombabatik ta paltogen taka"
which means don't run or
else I will shoot you''.
Q What happened when you
warned Arnulfo Cendana
not to escape?

A He stopped and he raised


his hands.

The trial court took into account the following reports as


clear indications that it was accused-appellant who fired the
shotgun which killed Dominador Manongdo: (1) the
Q What happened next?
postmortem report, as testified to by the Municipal Health
Officer, Dr. Leonard Carbonell, to the effect that "the cause
A I asked him if he was the of death of the deceased is intracranial injuries with
one who is responsible
intracranial hemorrhage secondry to gunshot wound" [TSN,
about the incident, the
November 5, 1987, p. 43]; (2) Chemistry Report No. C-86killing incident. At first he
1205, as testified to by Ma. Carina Javier, an NBI Forensic
denied it but later on when I Chemist, to the effect that the examination of the paraffin
talked to him in calm
casts of accused-appellant yielded positive results for specks
manner he admitted that he in both hands, indicating that gunpowder nitrates were
was responsible and then I
present [TSN, December 22, 1987, pp. 53-54]; and (3) the
asked him where is the gun ballistics report, as testified to by Irineo Ordiano, Jr., an NBI
that he used.
Senior Ballistician, to the effect that the homemade shotgun
allegedly recovered from the accused-appellant is
Q What did he tell you?
serviceable [TSN, January 20, 1988, p. 64].
A At first he denied it but
later on he told to one of his
relatives to get the gun, sir.
[TSN, February 1, 1988, pp.
6-7; Emphasis supplied]
The failure of the police investigators to apprise accusedappellant of his constitutional rights makes inadmissible
their testimonies that the accused-appellant admitted to the
commission of the offense and pointed to the location of the
shotgun [Art. III, sec. 12, par. 3, Constitution].
With the exclusion of these alleged admission of the
accused-appelant, we must look into the other evidence
taken against the accused-appellant which led to a judgment
of conviction against him by the trial court. Only if this is
sufficient to convince the Court without any reasonable
doubt that he committed the offense charged can the
judgment of conviction rendered against him be affirmed.
II.

A careful analysis of the postmortem report will show that


the gunshot wound sustained by the deceased Dominador
Manongdo is indeed compatible with what could be inflicted
by a shotgun. However, such a finding assumes no
significance in connection with a finding of the guilt of
accused-appellant unless the firing of the shotgun could
positively be linked to him.
Neither is the ballistics report any support to the conclusion
that accused-appellant is guilty of the commission of the
offense. All that the NBI ballistics report establishes is that
the shotgun which was submitted to it for examination is still
serviceable. Although this could have been done facilely by
the crime laboratory, there was no finding as to whether or
not the shotgun has just been recently fired. Therefore,
instead of bolstering the case for the prosecution, it instead
raises the doubt that the tested shotgun may not have been
the same shotgun which was used to shoot at and kill the
deceased Manongdo.

paraffin casts taken from his hands. If this was supported by


other evidence before the Court, then the presence of
gunpowder nitrates on the hands of accused-appellant could
have been a strong indication that it was he who fired the
gun which killed the deceased. However, this does not obtain
in the case at bar. Likewise of note is the plausible reason
preferred by the accused-appellant that he tested positive for
gunpowder burns because he took a turn at firing at coconut
fruits with an armalite of a military man who was likewise a
guest at a party given by his kumpare [TSN, April 6, 1988, p.
7].
The absence of any eyewitness to the commission of the
offense and the exclusion of the admissions allegedly made
by accused-appellant and the fruits thereof, calls into
application Rule 133, section 5 of the Rules of Court which
states:
Sec. 5. Circumstantial evidence, when
sufficient. Circumstantial evidence is
sufficient for conviction if:
(a) There is more than one circumstance;
(b) The facts from which the inferences are
derived are proven; and
(c) The combination of all the circumstances
is such as to produce a conviction beyond
reasonable doubt.
There being only one circumstance indicative of the guilt of
the accused, i.e. the paraffin casts of accused-appellant's
hands yielded for gunpowder burns the above requisites are
not satisfied. The case of the prosecution definitely fails.

WHEREFORE, finding that the guilt of the accusedappellant has not been proved beyond reasonable doubt, the
The only other evidence left with which to establish the guilt decision of the lower court is reversed and the accusedof accused-appellant is the positive result obtained from the appellant is hereby ACQUITTED.

SO ORDERED.

Contrary to law [Rollo, p. 4].

G.R. No. 87783 August 6, 1990


THE PEOPLE OF THE PHILIPPINES, plaintiffappellee,
vs.
ADELINA CASTILLER y CASTRO, defendant-appellant.
The Solicitor General for plaintiff-appellee.
Public Attorney's Office for defendant-appellant.

CORTES, J.:
Appellant Adelina Castiller y Castro was charged with and
convicted of violation of Section 4, Art. II of Rep. Act No.
6425, as amended, otherwise known as the Dangerous Drugs
Act, under an information which read:
That on or about the 17th day of April, 1988,
in the Municipality of Taguig, Metro
Manila, Philippines and within the
jurisdiction of this Honorable Court, the said
accused, without having been authorized by
law, did then and there wilfully, unlawfully
and feloniously sell, deliver and give away
to another two (2) foils of dried marijuana
fruiting tops and was found to be in
possession of the following: one (1)
Ajinomoto Plastic bag containing 545 grams
of dried marijuana fruiting tops, ten (10)
sticks of marijuana cigarette wrapped with
white paper and five (5) newspaper
wrappers each with dried marijuana fruiting
tops having a total weight of 20.77 grams,
which is a prohibited drug.

"customer" was in fact a police officer, scampered into the


back of her store and locked herself inside even before Pat.
Upon arraignment, appellant, assisted by counsel, entered a Mendibel could hand over the twenty-peso bill in payment of
plea of "not guilty" to the offense charged. Trial ensued with the foils of marijuana [TSN, August 10, 1988, p. 15]. Pat.
the prosecution and the defense presenting their own
Mendibel immediately gave the pre-arranged signal to
witnesses and evidence to support their respective versions
inform his teammates that the operation had indeed yielded
of the events leading to the arrest of the accused-appellant.
marijuana, and to get their assistance in arresting the
appellant [TSN, October 7, 1988, p. 3]. The police officers
According to the prosecution witnesses, composed mainly of knocked at the door but appellant refused to open the door.
police officers, at around nine o'clock in the morning of
Later, however, she voluntarily opened the door when the
April 18, 1988, the anti-narcotics intelligence division of the police officers, still knocking, asked permission to be
Taguig Police Station received information from an
allowed inside her store [TSN, August 10, 1988, pp. 16-17].
undisclosed caller that marijuana was being sold by an old
woman in a small store in North Daang Hari Street, Taguig
When the police officers went inside, appellant pointed to a
[TSN, June 29, 1988, p. 4; TSN, October 7, 1988, p. 2].
large gray container placed beside some cases of softdrink
Immediately, Capt. Ferdinand Santos organized a team
[TSN, August 10, 1988, p. 4]. They found a hole at the
composed of police officers Felixberto Maog (team leader), bottom of the large gray plastic container, and stored inside
Jesus Chan, Joselito Lintad, Jessie Pasion, Ruel Viring, and were a plastic "Ajinomoto" bag containing substances which
Carlos Mendibel in order to conduct a buy-bust operation
appeared to be marijuana leaves, ten (10) sticks of what
[TSN, June 29, 1988, p. 4]. Pat. Mendibel, assigned to act as appeared to be marijuana cigarettes, five (5) foils of what
poseur-buyer, was briefed by Capt. Santos separately from
appeared to be marijuana leaves wrapped in newspaper, six
the other members of the team who were to serve as backup (6) packs of "Capitol" brand rolling paper and a red
[TSN, August 2, 1988, pp. 3-4], and was later handed a
leatherette bag [Exhibits "G", "C", "D", "E", "H"-1" to "H-6"
twenty-peso bill to be used to purchase marijuana in the
and "H" respectively].
buybust operation [TSN, June 29, 1988, p. 3; TSN, August
10, 1988, p. 14].
Appellant and the confiscated evidence were brought to the
police station. Pat. Santiago Villa, the officer assigned to
Around noontime, the plain clothes team arrived at North
conduct the investigation, informed appellant of her
Daang Hari. Indeed, they found an old woman in a small
constitutional rights. Stating only her name and her desire to
store in No. 303 North Daang Hari, which Pat. Jessie Pasion consult her lawyer [TSN, August 25, 1988, p. 5], appellant
pointed out to Mendibel as the subject of the operation. The opted to remain silent.
other members of the team in the meantime positioned
themselves some fifteen (15) meters away from the store
The seized specimen, along with a letter-request dated April
[TSN, October 7, 1988, p. 2]. Pat. Mendibel went to the
18, 1988 signed by Capt. Ferdinand Santos, were then
store, and said to the appellant: "La paiskor ng dalawang
brought to the PC Crime Laboratory at Camp Crame so that
foil" [TSN, July 15, 1988, p. 3]. Appellant went inside the
tests could be conducted on the substances which appeared
store and came back with two (2) foils of marijuana which
to be marijuana.
she gave to Pat. Mendibel. Taking the marijuana foils, Pat.
The laboratory analysis conducted by P/Capt. Lina C.
Mendibel introduced himself as a police officer [TSN, July
Sarmiento found the specimens to be "positive for marijuana,
15, 1988, pp. 3-4]. Appellant, upon learning that her

a prohibited drug" [Chemistry Report No. D-348-88, Exhibit


"A"; and Certificate of Laboratory Result, Exhibit "J"] i.e., a
plastic bag containing 545 grams dried marijuana fruiting
tops, ten (10) sticks of marijuana cigarettes wrapped with
white paper, and the five (5) newspaper wrappers each with
dried marijuana fruiting tops having a total weight of 20.77
grams.

On the other hand, Rebecca De los Santos testified that in


the evening of April 17, 1988, she saw a certain woman
leave her water container at appellant's store, and heard the
name "Magda" mentioned [TSN, February 14, 1989, p. 2].

Pursuant to Article 45 of the Revised Penal


Code, the gray plastic container (Exhibit
"G"), the red leatherette bag (Exhibit "H")
and six (6) packs of rolling paper (Exhibits
"H-1" to "H-6") are hereby ordered
confiscated and forfeited in favor of the
Government.

The trial court, giving credence to the evidence of the


prosecution, found that appellant was caught in flagrante
delicto delivering marijuana. Furthermore, the trial court
On the other hand, the defense presented a different version held that appellant's possession of considerable quantities of
SO ORDERED.
of the events that transpired.
marijuana indicates the intention to sell, distribute or deliver
the same, and that she was "really engaged in the illicit trade Appellant assails the decision of the trial court in this appeal,
According to appellant, she lived in a squatter area where the of marijuana." The dispositive portion of the judgment of
assigning the following errors:
only source of water was an artesian well located one house conviction reads:
I
away from her store [TSN, February 14, 1989, p. 41. People
usually had to queue up in order to get water from said well.
WHEREFORE, the Court hereby sentences
THE TRIAL COURT ERRED IN GIVING
Those living nearby habitually left their water containers at
the accused, ADELINA CASTILLER Y
CREDENCE TO THE TESTIMONIES OF
appellant's store which they later claimed when the need
CASTRO to suffer the penalty of reclusion
THE PROSECUTION WITNESSES AND
arose [TSN, February 14, 1989, p. 4]. Appellant testified that
perpetua with all its accessory penalties, to
IN TOTALLY DISREGARDING THE
on the night before the alleged buy-bust operation took
pay a fine of P20,000 and to pay the costs.
VERSION OF THE DEFENSE.
place, one "Magda" was the last person to leave her water
In the service of her sentence, the accused
container at the store [TSN, January 10, 1989, p. 2]. The next
II
shall be credited in full with the period of
day, at around 11:30 in the morning, a group of men,
her preventive imprisonment.
strangers to appellant, approached her in her store. One of
THE TRIAL COURT ERRED IN
them suddenly held her at her right side and informed her
CONVICTING THE ACCUSED DESPITE
Pursuant to Section 20, Article II of
that she would be brought to the police precinct. Appellant
HER UNLAWFUL ARREST AND
Republic Act No. 6425, as amended, let the
protested and asked if he had a warrant, but the man did not
ILLEGAL SEARCH AND SEIZURE
545 grams of dried marijuana fruiting tops,
answer her [TSN, January 10, 1989, p. 3]. The others went
UPON HER PREMISES.
ten (10) sticks of marijuana cigarette and
inside her store and began searching the place. Appellant
another 20.77 grams of dried marijuana
was brought to the police precinct while the others continued
III
fruiting tops in five (5) newspaper wrappers
searching her store [TSN, January 10, 1989, pp. 3-4].
subject matter of this case be turned over to
Appellant denied knowledge of the articles inside the gray
THE TRIAL COURT ERRED IN FINDING
the Dangerous Drugs Board Custodian, NBI,
container which was seized from her store but she admitted
THE ACCUSED-APPELLANT GUILTY
to be disposed of according to law.
that she recognized the gray container to be Magda's [TSN,
BEYOND REASONABLE DOUBT OF
January 10, 1989, p. 4].
THE CRIME OF VIOLATION OF
SECTION 4, ARTICLE II OF R.A. 6425,
Two other witnesses were presented for the defense.
AS AMENDED. [Appellants Brief, p. 1.]
Carmelita Ramos testified that, like some five other
neighbors of appellant, she usually left her water container at
the latter's store, and claimed it when she needed to fetch
water from the well again [TSN, February 14, 1989, pp. 3-4].

Ultimately, this case presents only one issue: whether or not


appellant was proven guilty of the crime charged beyond
reasonable doubt.

Patog, G.R. No. 69620, September 24, 1986, 144 SCRA


429].

Appellant underscores in its arguments the admission by the


police officers that at the time they set out to conduct the
operation, they did not know who their exact "target" was,
On the other hand, the defense did not put a substantial
having acted solely on a tip given by a caller who merely
After a thorough perusal of the evidence of both parties, we defense other than the denial of the accused. It is well-settled described the marijuana seller to be an old woman in a store
find that indeed, as sufficiently established by the
that mere denials cannot prevail against positive
on North Daang Hari, but gave no specific name. From this
prosecution, appellant is guilty of the crime charged.
identification of appellant as the seller of the prohibited
allegation, the defense concluded that no buy-bust operation
substances [People v. Khan, G.R. No. 71863, May 23, 1988, had in fact taken place, and that the prosecution merely
The testimonies of the prosecution witnesses substantially
161 SCRA 406; People v. Paco, G.R. No. 76893, February
concocted the story in order to justify an unlawful arrest
covered the circumstances of each and every stage of the
27, 1989, 170 SCRA 681].
resulting from an "intense enthusiasm to respond to the
crime, and the resulting arrest, search and seizure. The buyunidentified caller" [Appellant's Brief, pp. 7-8].
bust operation team leader (Pfc Felixberto Maog) testified as Furthermore, the testimonies of defense witnesses Rebecca
to the circumstances leading to and the formation of the
de los Santos and Carmelita Ramos do not buttress
This contention is devoid of merit.
entrapment scheme. The poseur- buyer (Pat. Mendibel)
appellant's case. At best, they present minor details which
positively identified appellant as the woman who delivered
neither substantiate appellant's denial of the charges nor
The buy-bust operation is a recognized means of entrapment
two (2) foils of marijuana to him, and along with a member strengthen the claim of an arrest and search totally wanting
for the apprehension of drug pushers [People v. Gatong-o
of the "back-up" (Pat. Jesus Chan), testified as to the
in legal basis.
G.R. No. 78698, December 29, 1988, 168 SCRA 716]. This
incidental search conducted on the store, and the prohibited
is so because the very characteristic of prohibited drugs i.e.
Curiously, appellant never made any effort to locate
drugs thereon. The case investigator (Pat. Santiago Villa)
their being easily concealed and transferred without threat of
"Magda", the alleged owner of the container filled with the
testified on the investigation made on appellant and the
detection in small and handy quantities, allows its sale, use
examination of the seized articles after the arrest, search and prohibited substances, in order to have her arrested.
and delivery with relative facility. Contrary to appellant's
Certainly in a case where her own life and liberty were at
seizure, and the forensic chemist (P/Capt. Lina Sarmiento)
contention, no surveillance of the area or the subject of the
stake, the victim of a wrongful accusation would have
found the seized articles submitted for examination to be
entrapment is necessary where the police officers had, as in
earnestly, nay even desperately, sought ways to vindicate
positive for marijuana.
this case, reasonable ground to believe that the informer and
herself by at least assisting the police in order to have the
the information given were reliable, and that a crime is
We find their testimonies to be clear, lucid, straightforward
true offender apprehended. Appellant's inaction becomes all indeed being perpetrated. The buy-bust operation was
and uncontradicted on all material points.
the more baffling considering that, by her own admission,
formed by the police officers precisely to test the veracity of
she had known "Magda" for about a year, they lived on the
the tip and in order to apprehend the perpetrator, if he in fact
There is nothing in the record to suggest that the police
same street (North Daang Hari), "Magda" fetched water at
commits the offense, before he further endangers society.
officers were compelled by any motive other than to
the artesian well just one house away from appellant's store Besides, the tip given describing an old woman keeping a
accomplish their mission to capture appellant in the
everyday, and even bought things from the store [TSN,
store at North Daang Hari as a drug pusher sufficiently and
execution of the crime. The presumption being that police
January 10, 1989, pp. 2, 6].
accurately points to the appellant. There was no arbitrariness
officers perform their duties regularly in the absence of any
on the part of the poseur-buyer in approaching appellant,
evidence to the contrary [Rule 13], Section 5 (m), Rules of
Collaterally, appellant would now attack the decision by
who exactly fit every detail of the description given.
Court; People v. Natipravat, G.R. No. 69876, November 13, putting at issue the validity of the buy-bust operation, the
1986, 145 SCRA 483; People v. De Jesus, G.R. Nos. 71942- failure of prosecution to produce the "buy-bust money", and The defense further belabors its theory that no entrapment
3, November 13, 1986, 145 SCRA 521; People v. Claudia
the legality of the arrest and the incidental search and
was effected by pointing to the fact that the twenty-peso bill
G.R. No. 72564, April 15, 1988, 160 SCRA 6461, their
seizure.
allegedly used in the buy-bust operation and later submitted
testimonies are entitled to full faith and credence [People v.
in evidence was not even marked, thereby rendering the

alleged buy-bust operation all the more dubious [Appellant's


Brief, p. 8].
Again, we find no merit in this argument.
Although the alleged buy-bust money was indeed unmarked,
nevertheless, this fact alone will not suffice to weaken the
prosecution's case. Pat. Mendibel testified that he was
informed by his immediate superior who handed him the
money that it was marked [TSN, June 29, 1988, p. 5; TSN,
August 10, 1988, p. 14]. Apparently however, Mendibel,
who joined the police service barely four months earlier
[TSN, June 29, 1988], failed to personally check whether it
was marked. As correctly noted by the Solicitor General,
Mendibel was still a "tyro" in such operations, this being his
first drugs case [Appellee's Brief, p. 14; Rollo, p. 84], and he
was obviously unfamiliar with the manner in which buy-bust
operations are conducted.
Be that as it may, the failure to mark the bill is not fatal to
the case because the Dangerous Drugs Act punishes "any
person who, unless authorized by law, shall sell, administer,
deliver, give away to another, distribute, dispatch in transit or
transport any prohibited drug, or shall act as a broker in any
such transactions. ... [Emphasis supplied]." In this case, the
police officer sought to buy two sticks of marijuana and his
offer was accepted by appellant who produced and delivered
the same. The crime was consummated by the mere delivery
of the prohibited goods even without money changing hands.
As to the issue of the legality of appellant's warrantless arrest
and the warrantless search, we find that both arrest and
incidental search were made well within the bounds of the
law.
The 1985 Rules on Criminal Procedure provide that:
Sec. 5. Arrest without warrant; when lawful.
A peace officer or a private person may,
without a warrant, arrest a person:

a) When, in his presence the person to be


arrested has committed, is actually
committing, or is attempting to commit an
offense;
b) When an offense has in fact just been
committed and he has personal knowledge
of facts indicating that the person to be
arrested has committed it;

Therefore, the entry by the law enforcers into the store in


order to effect appellant's arrest was perfectly lawful.
As to the validity of the search conducted upon the premises
of appellant's store and the consequent seizure of
incriminating evidence found therein, this Court finds that
both search and seizure were valid.

That searches and seizures must be supported by a valid


warrant is not an absolute rule [Manipon Jr. v.
xxx xxx xxx
Sandiganbayan, G.R. No. 58889, July 31, 1986, 143 SCRA
267]. Among the exceptions granted by law is a search
In cases falling under paragraphs (a) and (b) incidental to a lawful arrest under Section 12, Rule 126 of
hereof, the person arrested without a warrant the Rules on Criminal Procedure, which provides that "[a]
shall be forthwith delivered to the nearest
person lawfully arrested may be searched for dangerous
police station or jail, and he shall be
weapons or anything which may be used as proof of the
proceeded against in accordance with Rule
commission of an offense, without a search warrant." In such
112, Section 7. [Rule 113, Section 5; 1985
an instance, a contemporaneous search may be conducted
Rules on Criminal Procedure.]
upon the person of the arrestee and the immediate vicinity
where the arrest was made [Nolasco v. Pao, G.R. No.
Both of the above-cited instances of a lawful warrantless
69803, January 30, 1987, 147 SCRA 5091, as was done in
arrest are attendant in this case. Appellant was caught in
this case. The inclusion of the seized items in the evidence
flagrante delicto delivering to the poseur-buyer two (2)
for the prosecution cannot be challenged as they were seized
sticks of marijuana. The offense was committed in the
in conformity with the provision on lawful searches.
presence of the police officer, and therefore the latter had
personal knowledge of the commission of the offense. Under All told, this Court is convinced that appellant Adelina
the circumstances, appellant's arrest was lawfully effected
Castiller had indeed committed the offense as charged. The
without need of a warrant.
assailed decision must be upheld. The trial court aptly quoted
the dictum of this Court in the case of People v. Abedes
Furthermore, where, as in this case, the person to be arrested [G.R. No. 73399, November 28, 1986, 146 SCRA 132]: "A
attempts to evade the same, the Rules on Criminal Procedure drug pusher is a killer without mercy. He poisons the mind
allow a peace officer, in order to make a lawful warrantless
and deadens the body. He deserves no mercy." Our society
arrest, to "... break into any building or enclosure in which
has very right to be protected from the potential and actual
the person to be arrested is or is reasonably believed to be, if harm wrought by prohibited drugs.
he is refused admittance thereto, after he has announced his
authority and purpose" [Section 11, Rule 113, 1985 Rules on WHEREFORE, the decision appealed from is hereby
Criminal Procedure]. In the case at bar, the police officers
AFFIRMED.
did not have to break into the premises since appellant
voluntarily allowed the law officer inside the store.
SO ORDERED.

G.R. No. 76547 July 30, 1990


PEOPLE OF THE PHILIPPINES, plaintiff-appellee,
vs.
ALBERT OLAES y AMOROSO, accused-appellant.
The Solicitor General for plaintiff-appellee.
Romeo C. Alinea for accused-appellant.

REGALADO, J.:
The decision 1 of the Regional Trial Court, Branch LXXII in
Olongapo City, in Criminal Case No. 3602 involving a
violation of Section 4, Article II of Republic Act No. 6425
(Dangerous Drugs Act of 1972) has been brought to us on
appeal 2 by accused-appellant who was sentenced to suffer
the penalty of life imprisonment and to pay a fine of
P30,000.00 and the costs. The confiscation and immediate
destruction of six (6) lids of marijuana subject matter of the
case was also ordered. 3
In an information dated March 27, 1978, appellant was
charged with unlawfully selling six (6) lids of marijuana. 4
On March 16, 1979, appellant, assisted by his counsel de
oficio, was duly arraigned and, having pleaded not guilty, 5
trial on the merits ensued.
Based on the evidence for the prosecution, the court a quo
narrated the case for the People as follows:
That at 12:50 in the afternoon of June 15,
1977, said officers were at their office at the
CANU investigating Manuelito Bernardo,
whom they arrested earlier at No. 41 Harris
St., East Bajac-Bajac, Olongapo City for
possession of several lids of marijuana.
Bernardo informed them that the lids of

marijuana confiscated from him came from


marijuana which were confiscated from the
one alias "Abet" and his brother alias
kitchen of Benjamin Olaes are covered and
"Bonjing". They made Bernardo agree to act
stated in the Receipt for Property Seized
as buyer and to go to the house which,
(Exhibit "I"). 6
according to Bernardo, was the source of the
On the other hand, the decision makes the factual recital that
confiscated marijuana. After giving

instructions to Bernardo about the prearranged signal, they all proceeded near the
The defense version per testimony of
house of "Abet" located at No. 116 Jones
accused Alberto Olaes tends to show that on
Street, Olongapo City. Bernardo entered the
June 15, 1977, at around two o'clock in the
compound bringing with him marked money
afternoon, he was at their house located at
amounting to P300.00 (These were earlier
116 Jones Street Kalalake, Olongapo City.
xeroxed after the signature of Pacifico
While at Room 5 of said address, a certain
Mugar was affixed).itc-asl
Eling carrying a bag came to their house
asking for his brother Benjamin Olaes who
After consummating the deal on the
was not home at that time. Eling was
marijuana, Bernardo came out and
carrying a bag of dog food. Upon being told
surrendered the same to the officers.
that Benjamin was out, Eling asked to be
Immediately, they entered "Abet's" house
accompanied to Room 2 also located at 116
and found him in the living room with some
Jones Street, Olongapo City, where he left
other persons. "Abet" was confronted,
the bag in his brother's room, particularly in
searched, and in his person was found the
the "lababo". Thereafter, he and Eling went
P300.00 marked money and lids of
out of the room and he went home to his
marijuana. "Abet" was interrogated on the
residence at Room 5, 116 Jones Street,
spot as to the whereabouts of other
Olongapo City. He executed a sworn
marijuana if any. They proceeded next door
statement in connection with this case. He
at his brother Benjamin Olaes' kitchen
does not know any person by the name of
where they recovered sixty more lids of
Lito. He affixed his signature on his
marijuana. Thereafter, "Abet", which (sic)
statement because he was mauled by the
was later identified as accused Albert Olaes
investigator and was threatened to be killed.
was brought to the CANU office for
investigation. Albert Olaes was fingerOn cross-examination, he stated that he does
printed by C2C Armando Cases and the
not know who among the investigators beat
booking sheet and arrest report were
him up and neither does he know who
prepared (Exhibit "G"). The sworn statement
arrested him. All his personal circumstances
of Olaes was taken by Pacifico Mugar
stated in his statement are true. He also
(Exhibit "K"). Pfc. Abello together with
declared that Eling came from Saletran,
Capitulo, Elgar, Macomb and Cases
Dasmarinas. Cavite. 7
executed a Joint Affidavit in connection with
this case (Exhibit "H"). The sixty lids of

Opining that the testimony of appellant is incredible and


relying heavily on the testimonies of the prosecution
witnesses, as well as the extrajudicial confession executed by
appellant "corroborated by evidence of corpus delicti," the
court below rendered its judgment of conviction stated at the
outset of this decision. Hence, this appeal where appellant
submits as alleged reversible errors (1) the conviction based
on an extrajudicial confession extracted in violation of due
process and the bill of rights of the fundamental law: (2) the
conviction under Section 4, Article II of Republic Act No.
6425 (selling) even if one of the elements of the offense is
missing; and (3) the resolution of doubt on the defense
theory by reason of non-corroboration. 8
We find for appellant and decree his acquittal.
Firstly, the extrajudicial confession 9 executed by appellant
on June 15, 1977 is inadmissible in evidence. An
examination thereof shows that he was informed of his
constitutional rights to remain silent and to be assisted by
counsel during said investigation. He was also asked if he
was waiving his right to be assisted by counsel and he
answered in the affirmative. However, while the rights of a
person under custodial investigation may be waived,
nonetheless the waiver must be made not only voluntarily,
knowingly and intelligently but in the presence and with the
assistance of counsel. 10
In the case at bar, the waiver was made without the
assistance of counsel. 11 This omission alone is sufficient to
invalidate the confession. 12 While the trial court observed
that the narration of the accused in his extrajudicial
confession is complete in every detail and did not show any
sign of suspicious circumstance to indicate that there was
pressure of restraints exerted upon his person, 13 having been
made without the mandatory assistance of counsel the same
is inadmissible in evidence regardless of the absence of
coercion or even if it had been voluntarily given. 14

Secondly, this case exemplifies the instance where the nonpresentation of the supposed poseur-buyer is fatal to the
prosecution's case. The records show that the alleged sales
transaction took place inside the house of appellant. In other
words, the transaction was supposedly witnessed only by the
poseur-buyer, Manuelito Bernardo. Only he has personal
knowledge of such transaction which is the subject matter of
this prosecution. In People vs. Ramos, 15 where the alleged
informant and poseur-buyer was one and the same person,
we stressed that without the testimony of said poseur-buyer,
there is no convincing evidence pointing to the accused as
having sold marijuana. In this case, the police officers did
not see the actual sale of marijuana. For the culprit to be
convicted, the element of sale must be unequivocably
established. Yet, the alleged poseur-buyer in the "buy-bust"
operation, the only one who allegedly dealt directly with
appellant in the purchase of marijuana, was not presented at
all at the trial. Under such circumstances, we have repeatedly
held that the failure of the prosecution to present the alleged
buyer was a fatal blow to the case against the accused. 16
The other prosecution witnesses admitted that it was only
when the alleged poseur-buyer left the house of appellant
and gave the signal indicating the consummation of the
transaction that they started to approach the house of
appellant and entered the same. Thereafter, they allegedly
searched the person of appellant and proceeded to the
adjoining room where they claimed to have recovered other
prohibited drugs. 17 We have carefully reviewed the records
and find that the testimonies of the prosecution witnesses do
not inspire belief.
While suspiciously dovetailing on certain aspects in their
testimonies, the members of said law enforcement team,
despite the fact that they claim to have been together before,
during and after the operation, enmeshed themselves in
significant inconsistencies and contradictions.
Thus, for instance, Cpl. Ernesto Abello claimed that the
marked money which was used to purchase the marijuana

was zeroxed before it was given to the poseur-buyer who


later allegedly paid it to appellant. 18 On the other hand, Pfc.
Jaime Capitulo who allegedly recovered the marked money
from appellant testified that it was after such money was
retrieved and the team had returned to their office that he
turned over the money to their superior, Capt. Aldaba, and it
was then that zerox copies of the money were made
presumably for purposes of evidence, 19 only to subsequently
reverse himself by claiming that the same was zeroxed
before they were given to the poseur-buyer. 20 It will be
noted that it is this evidence, with the confusing testimony
thereon, by which the prosecution seeks to link appellant to
the supposed sale.
Now, the poseur-buyer, according to the prosecution
witnesses, was supposed to give them a pre-arranged signal
after he had made the purchase of the marijuana. He is
alleged to have done so but, as to the signal pre-arranged and
given, the versions of said witnesses are in hopeless disarray.
According to Cpl. Abello, Bernardo "would be coming out of
the backdoor and he will be combing his hair. 21 On the other
hand, Sgt. Glenn Logan testified that the prearranged signal
was that Bernardo would put out his handkerchief and wipe
his face. 22 These were all contradicted in the testimony of
Pfc. Jaime Capitulo who claimed that the informant was able
to give the pre-arranged signal because the investigating
team had with them an electric device with a receiver that
received Bernardo's signal which could be heard by
everyone in the group. 23 How this would be possible and
why the witnesses are at loggerheads hereon, the prosecution
does not say.
Again, Sgt. Logan declared that the search was conducted in
the residence of appellant resulting in the seizure of sixty
(60) lids of marijuana. 24 On the contrary, Pat. Abella clearly
explained that said sixty (60) lids of marijuana were hidden
and recovered in the kitchen of the house of Benjamin Olaes,
a brother of appellant, which was a separate residence. 25
These conflicting testimonies, which further discredit said
witnesses, are apart from the consideration that appellant

herein is charged not with illegal possession but with the sale The foregoing considerations cannot but rule out a verdict of
of marijuana.
guilty, there being an exiguity of any other independent
incriminating evidence, aside from appellant's uncounselled
We also do not lose sight of the fact that without the
and inadmissible extrajudicial confession. While the theory
testimony of the supposed poseur-buyer proving the alleged of the defense is not totally convincing, the Court cannot tip
sale of marijuana inside appellant's house, the unlawful
the scales of justice against him in the face of the cardinal
intrusion into the sanctity of appellant's abode and the
and long entrenched rule that the prosecution must rely on
unreasonable search and seizure proscribed by the
the strength of its own evidence and not on the weakness of
Constitution are clearly established. It is undisputed that the that of the defense.
police operatives did not have either a search warrant or a
warrant of arrest. 26 The searches on the person of appellant One final observation. What cannot escape our attention is
and of his house were not also incidental to a lawful arrest.
the penalty imposed by the trial court, that is, life
The police officers admittedly did not have personal
imprisonment and a fine of P30,000.00. The court a quo
knowledge at all of what actually transpired inside the
obviously failed to consider that the alleged crime was
appellant's house. They only learned of the alleged
committed on June 15, 1977. Under Section 4 of Republic
consummation of the illicit transaction when they were
Act No. 6425, prior to its amendment on February 17, 1980
supposedly given a signal by their so-called poseur-buyer
by Presidential Decree No. 1675, the act of selling marijuana
after the latter left appellant's house. However, what they
was punishable by imprisonment ranging from twelve (12)
reportedly learned from said poseur-buyer was indubitably
years and one (1) day to twenty (20) years and a fine ranging
hearsay as the latter was never called to appear and testify at from P12,000.00 to P20,000.00. 31 Due to the urgent
the trial. Since what was conducted was a warrantless search necessity of reinforcing the drive against dangerous drugs by
and the arrest of appellant was unlawful, any evidence
making "drug-pushing" a capital offense, Presidential Decree
obtained from him is also inadmissible in evidence.
No. 1675 was subsequently passed providing for the penalty
of life imprisonment to death and a fine ranging from
It is further undisputed that the six (6) lids of marijuana
P20,000.00 to P30,000.00. Obviously, this amendment
supposedly bought by the poseur-buyer was taken by the
having taken effect after the alleged commission of the crime
police officers not from appellant but from said poseurin the case at bar, it could not have been given retroactive
27
buyer. Appellant's disclaimer of ownership of the
effect.
prohibited drug should have cautioned and alerted the
prosecution to the fact that the testimony of their alleged
WHEREFORE, the guilt of accused-appellant not having
poseur-buyer was not merely corroborative nor cumulative
been proved with the requisite quantum of evidence, the
but direct and material to the defense of appellant who
appealed decision is REVERSED and he is hereby
28
claims innocence of the offense imputed to him. It is also ACQUITTED on reasonable doubt, with costs de officio.
significant that the identity of the informer and/or poseurbuyer was already known during the trial. 29 Despite all these, SO ORDERED.
the prosecution opted not to present him for reasons which
remain unknown. Such unexplained failure to present this
vital witness gives rise to the presumption that, if he had
Papa vs.
been presented, his testimony would probably not have
30
M
supported the case of the prosecution.

ago
Facts:Petitioner Martin Alagao, head of the counter-intelligence unit of
the Manila Police Department, acting upona reliable information
received on November 3,
1
966 to theeffect that a certain shipment of personal effects,
allegedlymisdeclared and undervalued, would be released thefollowing
day from the customs zone of the port of Manilaand loaded on two
trucks, and upon orders of petitioner Ricardo Papa, Chief of Police of
Manila and a duly deputizedagent of the Bureau of Customs, conducted
surveillance atgate No.
1
of the customs zone.When the trucks left gate No.
1
at about 4:3
0
in theafternoon of November 4,
1
966, elements of the counter-intelligence unit went after the trucks and
intercepted them atthe Agrifina Circle, Ermita, Manila. The load of the
two trucks,consisting of nine bales of goods, and the two trucks,
wereseized on instructions of the Chief of Police. Uponinvestigation, a
person claimed ownership of the goods andshowed to the policemen a
"Statement and Receipts of Duties Collected on Informal Entry No.
1
47-55
01
", issued bythe Bureau of Customs in the name of a certain
BienvenidoNaguit.The respondent Mago, filed a petition for
mandamusand certiorari before the CFI Manila contending that
thesearch and seizure is illegal for lack of a valid warrant.Moreover, she
also contends that such articles sought fromher is not included by the law
for prohibited importation andthat it no longer under the control of the
Tariff and Customscode for it (articles) were already sold to the
petitioner.She also contends that the search seizureconducted by the
respondents are illegally being madeoutside the jurisdiction of the BOC
and that the subsequentsearch warrant issued by the collector of customs
is not validbeing not issued by a judge.The respondent Mago filed an exparte motion torelease the confiscated articles upon her posting a
bond.This motion was then granted by the respondent
JudgeJarencio.Issue:Was the seizure of the goods unlawful? And that
theBOC has no jurisdiction over the articles sought?Held:No. it is a valid
seizure.The Chief of the Manila Police Department, RicardoG. Papa,
having been deputized in writing by theCommissioner of Customs,
could, for the purposes of theenforcement of the customs and tariff laws,

effect searches,seizures, and arrests, and it was his duty to make


seizure,among others, of any cargo, articles or other movableproperty
when the same may be subject to forfeiture or liablefor any fine imposed
under customs and tariff laws. He couldlawfully open and examine any
box, trunk, envelope or other container wherever found when he had
reasonable cause tosuspect the presence therein of dutiable articles
introducedinto the Philippines contrary to law; and likewise to
stop,search and examine any vehicle, beast or person
reasonablysuspected of holding or conveying such article as aforesaid.It
cannot be doubted, therefore, that petitioner Ricardo G.Papa, Chief of
Police of Manila, could lawfully effect thesearch and seizure of the goods
in question. The Tariff andCustoms Code authorizes him to demand
assistance of anypolice officer to effect said search and seizure, and the
latter has the legal duty to render said assistance. This was whathappened
precisely in the case of Lt. Martin Alagao who, withhis unit, made the
search and seizure of the two trucksloaded with the nine bales of goods in
question at the Agrifina Circle. He was given authority by the Chief of
Policeto make the interception of the cargo.Petitioner Martin Alagao and
his companionpolicemen had authority to effect the seizure without
anysearch warrant issued by a competent court.The Tariff and Customs
Code does not require saidwarrant in the instant case. The Code
authorizes personshaving police authority under Section 22
0
3 of the Tariff andCustoms Code to enter, pass through or search any
land,inclosure, warehouse, store or building, not being a dwellinghouse;
and also to inspect, search and examine any vesselor aircraft and any
trunk, package, box or envelope or anyperson on board, or stop and
search and examine anyvehicle, beast or person suspected of holding or
conveyingany dutiable or prohibited article introduced into
thePhilippines contrary to law, without mentioning the need of asearch
warrant in said cases.It is our considered view, therefore, that except inthe
case of the search of a dwelling house, personsexercising police authority
under the customs law may effectsearch and seizure without a search
warrant in theenforcement of customs laws.
Note:
The Bureau of Customs has the duties, powers and jurisdiction, among
others,
(1)
to assess and collect all lawfulrevenues from imported articles, and all
other dues, fees,charges, fines and penalties, accruing under the tariff
andcustoms laws;
(2)
to prevent and suppress smuggling andother frauds upon the customs;
and
(3)

to enforce tariff andcustoms laws.The goods in question were imported


from Hongkong, asshown in the "Statement and Receipts of Duties
Collected onInformal Entry." As long as the importation has not
beenterminated the imported goods remain under the jurisdictionof the
Bureau of Customs.
Importationis deemed terminated only upon thepayment of the duties,
taxes and other charges upon thearticles, or secured to be paid, at the port
of entry and thelegal permit for withdrawal shall have been granted.. the
payment of the duties, taxes, fees and other charges must be paid in full.
Roldan vs. Arca [GR L-25434, 25 July 1975]
First Division, Makasiar (J): 4 concur, 1 took no part
Facts:
On 3 April 1964, Morabe, De Guzman & Company filed
with the Court of First Instance(CFI) of Manila a civil case
(56701) against Fisheries Commissioner Arsenio N. Roldan,
Jr., for the recovery of fishing vessel Tony Lex VI which had
been seized and impounded by theFisheries Commissioner
through the Philippine Navy. On 10 April 1964, the company
prayed for a writ of preliminary mandatory injunction with
the CFI, but said prayer was denied. On 28 April1964, the
CFI set aside its order of 10 April 1964 and granted the
companys motion for reconsideration praying for
preliminary mandatory injunction. Thus, the company
took possession of the vessel Tony Lex VI from the
Philippine Fisheries Commission adn thePhilippine Navy by
virtue of the said writ. On 10 December 1964, the CFI
dismissed Civil Case56701 for failure of the company to
prosecute as well as for failure of the Commission and
the Navy to appear on the scheduled date of hearing. The
vessel, Tony Lex VI or Srta. Winniehowever, remained in the
possession of the company.On 20 July 1965, the Fisheries
Commissioner requested the Philippine Navy to
apprehendvessels Tony Lex VI and Tony Lex III, also
respectively called Srta. Winnie and Srta. Agnes, for alleged
violations of some provisions of the Fisheries Act and the
rules and regulations promulgated thereunder. On August 5
or 6, 1965, the two fishing boats were actually seized
for illegal fishing with dynamite. Fish caught with dynamite
and sticks of dynamite were then foundaboard the two
vessels. On 18 August 1965, the Fisheries Commissioner
requested the PalawanProvincial Fiscal to file criminal
charges against the crew members of the fishing vessels. On

30September 1965, there were filed in the CFI of Palawan a


couple of informations, one against thecrew members of
Tony Lex III, and another against the crew members of Tony
Lex VI bothfor violations of Act 4003, as amended by
Commonwealth Acts 462, 659 and 1088, i.e., for illegal
fishing with the use of dynamite. On the same day, the Fiscal
filed an ex parte motion tohold the boats in custody as
instruments and therefore evidence of the crime, and cabled
theFisheries Commissioner to detain the vessels. On October
2 and 4, likewise, the CFI of Palawanordered the Philippine
Navy to take the boats in custody. On 2 October 1965, the
company fileda complaint with application for preliminary
mandatory injunction (Civil Case 62799) with theCFI of
Manila against the Commission and the Navy. Among
others, it was alleged that at thetime of the seizure of the
fishing boats in issue, the same were engaged in legitimate
fishingoperations off the coast of Palawan; that by virtue of
the offer of compromise dated 13September 1965 by the
company to the Secretary of Agriculture and Natural
Resources, thenumerous violations of the Fishery Laws, if
any, by the crew members of the vessels weresettled. On 18
October 1965, Judge Francisco Arca issued an order granting
the issuance of thewrit of preliminary mandatory injunction
and issued the preliminary writ upon the filing by
thecompany of a bond of P5,000.00 for the release of the two
vessels. On 19 October 1965, theCommission and the Navy
filed a motion for reconsideration of the order issuing the
preliminarywrit on 18 October 1965 on the ground, among
others, that on 18 October 1965 the Philippine Navy received
from the Palawan CFI two orders dated October 2 and 4,
1965 requiring thePhilippine Navy to hold the fishing boats
in custody and directing that the said vessels should not be
released until further orders from the Court, and that the
bond of P5,000.00 is grosslyinsufficient to cover the
Governments losses in case the two vessels, which are
worthP495,000.00, are placed beyond the reach of the
Government, thus frustrating their forfeiture asinstruments of
the crime. On 23 November 1965, Judge Arca denied the
said motion for reconsideration. The Commission and the
Navy filed a petition for certiorari and prohibition with
preliminary injunction to restrain Judge Arca from enforcing
his order dated 18 October 1965,and the writ of preliminary
mandatory injunction thereunder issued.
Issue:

Whether the Fisheries Commissioner and the Navy can


validly direct and/or effect theseizure of the vessels of the
company for illegal fishing by the use of dynamite and
without therequisite licenses.
Held:
Section 4 of Republic Act 3512 approved on 20 March 1963
empowers the FisheriesCommissioner to carry out the
provisions of the Fisheries Act, as amended, and all rules
andregulations promulgated thereunder, to make searches
and seizures personally or through hisduly authorized
representatives in accordance with the Rules of Court, of
explosives such asdynamites and the like; including fishery
products, fishing equipment, tackle and other thingsthat are
subject to seizure under existing fishery laws; and to
effectively implement theenforcement of existing fishery
laws on illegal fishing. Paragraph 5 of Section 4 of the
sameRepublic Act 3512 likewise transferred to and vested in
the Philippine Fisheries Commission allthe powers,
functions and duties heretofore exercised by the Bureau of
Customs, Philippine Navy and Philippine Constabulary over
fishing vessels and fishery matters. Section 12 of
theFisheries Act, otherwise known as Republic Act 4003, as
amended, prohibits fishing withdynamites or other
explosives which is penalized by Section 76 thereof by a
fine of not lessthan P1,500.00 nor more than P5,000.00, and
by imprisonment for not less than one (1) year andsix (6)
months nor more than five (5) years, aside from the
confiscation and forfeiture of allexplosives, boats, tackles,
apparel, furniture, and other apparatus used in fishing in
violation of said Section 12 of this Act. Section 78 of the
same Fisheries Law provides that in case of asecond
offense, the vessel, together with its tackle, apparel, furniture
and stores shall be forfeitedto the Government. The second
paragraph of Section 12 also provides that the
possessionand/or finding, of dynamite, blasting caps and
other explosives in any fishing boat shallconstitute a
presumption that the said dynamite and/or blasting caps and
explosives are beingused for fishing purposes in violation of
this Section, and that the possession or discover in
anyfishing boat or fish caught or killed by the use of
dynamite or other explosives, under experttestimony, shall
constitute a presumption that the owner, if present in the
fishing boat, or thefishing crew have been fishing with
dynamite or other explosives. Under Section 78 of
theFisheries Act, as amended, any person, association or

corporation fishing in deep sea fisherywithout the


corresponding license prescribed in Sections 17 to 22 Article
V of the Fisheries Actor any other order or regulation
deriving force from its provisions, shall be punished for
eachoffense by a fine of not more than P5,000.00, or
imprisonment, for not more than one year, or both, in the
discretion of the Court; Provided, That in case of an
association or corporation, thePresident or manager shall be
directly responsible for the acts of his employees or laborers
if it is proven that the latter acted with his knowledge;
otherwise the responsibility shall extend only asfar as fine is
concerned: Provided, further, That in the absence of a known
owner of the vessel,the master, patron or person in charge of
such vessel shall be responsible for any violation of thisAct:
and Provided, further, That in case of a second offense, the
vessel together with its tackle,apparel, furniture and stores
shall be forfeited to the Government. Under Section 13
of Executive Order 389 of 23 December 1950, reorganizing
the Armed Forces of the Philippines,the Philippine Navy has
the function, among others, to assist the proper
governmental agenciesin the enforcement of laws and
regulations pertaining to Fishing. Section 2210 of the Tariff
andCustoms Code, as amended by PD 34 of 27 October
1972, authorized any official or person
exercising police authority under the provisions of the Code,
to search and seize any vessel or air craft as well as any
trunk, package, bag or envelope on board and to search any
person on boardfor any breach or violation of the customs
and tariff laws. Herein, when the Philippine Navy,upon
request of the Fisheries Commissioner, apprehended on
August 5 or 6, 1965 the fishing boats Tony Lex III and Tony
Lex VI, otherwise known respectively as Srta. Agnes and
Srta.Winnie, these vessels were found to be without the
necessary license in violation of Section 903of the Tariff and
Customs Code and therefore subject to seizure under Section
2210 of the sameCode, and illegally fishing with explosives
and without fishing license required by Sections 17and 18 of
the Fisheries Law. Search and seizure without search warrant
of vessels and air craftsfor violations of the customs laws
have been the traditional exception to the
constitutionalrequirement of a search warrant, because the
vessel can be quickly moved out of the locality
or jurisdiction in which the search warrant must be sought
before such warrant could be secured;hence it is not

practicable to require a search warrant before such search or


seizure can beconstitutionally effected. The same exception
should apply to seizures of fishing vessels breaching our
fishery laws: They are usually equipped with powerful
motors that enable them toelude pursuing ships of the
Philippine Navy or Coast Guard.
G.R. No. L-25434 July 25, 1975
HONORABLE ARSENIO N. ROLDAN, JR., in his
capacity as Acting Commissioner, Philippine Fisheries
Commission, and THE PHILIPPINE NAVY, petitioners,
vs.
HONORABLE FRANCISCO ARCA, as Presiding Judge
of the Court of First Instance of Manila (Branch 1) and
MORABE, DE GUZMAN & COMPANY, respondents.
Office of the Solicitor General Arturo A. Alafriz and
Solicitor Augusto M. Amores for petitioners.
J. C. Yuseco and A.R. Narvasa for private respondent.

MAKASIAR, J.:
A petition for certiorari and prohibition with preliminary
injunction to restrain respondent Judge from enforcing his
order dated October 18, 1965, and the writ of preliminary
mandatory injunction thereunder issued.
On April 3, 1964, respondent company filed with the Court
of First Instance of Manila a civil case docketed as No.
56701 against petitioner Fisheries Commissioner Arsenio N.
Roldan, Jr., for the recovery of fishing vessel Tony Lex VI
(one of two fishing boats in question) which had been seized
and impounded by petitioner Fisheries Commissioner
through the Philippine Navy.

On April 10, 1964, respondent company prayed for a writ of 659 and 1088, i.e., for illegal fishing with the use of
preliminary mandatory injunction with respondent court, but dynamite. On the same day, the Fiscal filed an ex parte
said prayer was, however, denied.
motion to hold the boats in custody as instruments and
therefore evidence of the crime (p. 54, rec.), and cabled the
On April 28, 1964, the Court of First Instance of Manila set Fisheries Commissioner to detain the vessels (p. 56, rec.).
aside its order of April 10, 1964 and granted respondent
company's motion for reconsideration praying for
On October 2 and 4, likewise, the Court of First Instance of
preliminary mandatory injunction. Thus, respondent
Palawan ordered the Philippine Navy to take the boats in
company took Possession of the vessel Tony Lex VI from
custody.
herein petitioners by virtue of the abovesaid writ.
On October 2, 1965, respondent company filed a complaint
On December 10, 1964, the Court of First Instance of Manila with application for preliminary mandatory injunction,
dismissed Civil Case No. 56701 for failure of therein
docketed as Civil Case No. 62799 with the Court of First
petitioner (respondent company herein) to prosecute as well Instance of Manila against herein petitioners. Among others,
as for failure of therein defendants (petitioners herein)to
it was alleged that at the time of the seizure of the fishing
appear on the scheduled date of hearing. The vessel, Tony
boats in issue, the same were engaged in legitimate fishing
Lex VI or Srta. Winnie however, remained in the possession operations off the coast of Palawan; that by virtue of the
of respondent company.
offer of compromise dated September 13, 1965 by
respondent company to the Secretary of Agriculture and
On July 20, 1965, petitioner Fisheries Commissioner
Natural Resources, the numerous violations of the Fishery
requested the Philippine Navy to apprehend vessels Tony
Laws, if any, by the crew members of the vessels were
Lex VI and Tony Lex III, also respectively called Srta.
settled.
Winnie and Srta. Agnes, for alleged violations of some
provisions of the Fisheries Act and the rules and regulations On October 9, 1965, petitioners, represented by the Solicitor
promulgated thereunder.
General, opposed the above-mentioned complaint, alleging
among others, that: (1) the issuance of the writ would disrupt
On August 5 or 6, 1965, the two fishing boats were actually the status quo of the parties and would render nugatory any
seized for illegal fishing with dynamite. Fish caught with
decision of the respondent court favorable to the defendant;
dynamite and sticks of dynamite were then found aboard the (2) that the vessels, being instruments of a crime in criminal
two vessels.
cases Nos. 3416 and 3417 filed with the Court of First
Instance of Palawan, the release of the vessels sans the
On August 18, 1965, the Fisheries Commissioner requested corresponding order from the above-mentioned court would
the Palawan Provincial Fiscal to file criminal charges against deprive the same of its authority to dispose of the vessels in
the crew members of the fishing vessels.
the criminal cases and the Provincial Fiscal would not be
able to utilize said vessels as evidence in the prosecution of
On September 30, 1965, there were filed in the court of First said cases; (3) that as petitioners herein were in possession
Instance of Palawan a couple of informations, one against
of one of the vessels in point, they cannot now be deprived
the crew members of Tony Lex III, and another against the
of the legal custody thereof by reason of the dismissal of
crew members of Tony Lex VI both for violations of Act Civil Case No. 56701; (4) that petitioner Fisheries
No. 4003, as amended by Commonwealth Acts Nos. 462,
Commissioner has the power to seize and detain the vessels

pursuant to Section 5 of Republic Act No. 3215 in relation to


Sections 903 and 2210 of the Revised Tariff and Customs
Code; (5) that respondents herein have not exhausted
administrative remedies before coming to court; (6) that the
compromise agreement approved by the Secretary of
Agriculture and Natural Resources and indorsed to the
Fisheries Commissioner is never a bar to the prosecution of
the crime perpetrated by the crew members of the vessels
belonging to respondent company.
And again, on October 15, 1965, herein petitioners filed their
memorandum praying for the denial of the application for
preliminary mandatory injunction. On the same day, October
15, 1965, herein petitioners filed an urgent motion to submit
additional documentary evidence.
On October 18, 1965, herein petitioners, as defendants in
said Civil Case No. 62799, filed their answer to the
complaint with affirmative defenses, reiterating the grounds
in their opposition to the issuance of a writ of preliminary
mandatory injunction and adding that herein private
respondent admitted committing the last violation when it
offered in its letter dated September 21, 1965 to the Acting
Commissioner of Fisheries, to compromise said last
violation (Exh. 12, pp. 60-61, rec.).
On said day, October 18, 1965, the respondent Judge issued
the challenged order granting the issuance of the writ of
preliminary mandatory injunction and issued the preliminary
writ upon the filing by private respondent of a bond of
P5,000.00 for the release of the two vessels(pp. 95-102,
rec.).
On October 19, 1965, herein petitioners filed a motion for
reconsideration of the order issuing the preliminary writ on
October 18, 1965 on the ground, among others, that on
October 18, 1965 the Philippine Navy received from the
Palawan Court of First Instance two orders dated October 2
and 4, 1965 requiring the Philippine Navy to hold the fishing
boats in custody and directing that the said vessels should

not be released until further orders from the Court, and that
the bond of P5,000.00 is grossly insufficient to cover the
Government's losses in case the two vessels, which are worth
P495,000.00, are placed beyond the reach of the
Government, thus frustrating their forfeiture as instruments
of the crime (pp. 103-109, rec.).1wph1.t
On November 23, 1965, respondent Judge denied the said
motion for reconsideration (p. 110, rec.).
WE rule that the respondent Judge of the Manila Court of
First Instance acted without jurisdiction and with grave
abuse of discretion when he issued on October 18, 1965 the
order directing the issuance of a writ of preliminary
mandatory injunction and when he refused to reconsider the
same.

Instance expressly direct the Philippine Navy "to hold in


custody" the two vessels and that "same should not be
released without prior order or authority from this Court"
(pp. 108, 109, rec.). Only the Palawan court can order the
release of the two vessels. Not even the Secretary of
Agriculture and Natural Resources nor the Fisheries
Commissioner can direct that the fishing boats be turned
over to private respondent without risking contempt of court.
The grave abuse of discretion committed by the respondent
Judge was heightened by the fact that he did not reconsider
his order of October 18, 1965 after he was informed by
petitioners in their motion for reconsideration filed on
October 19, 1965 that the Palawan Court of First Instance
had already issued the two orders dated October 2 and 4,
1965 directing the Philippine Navy to hold in custody the
fishing boats until further orders.

I
It is basic that one court cannot interfere with the judgments,
When the respondent Judge issued the challenged order on
orders or decrees of another court of concurrent or
October 18, 1965 and the writ of preliminary mandatory
coordinate jurisdiction having equal power to grant the relief
injunction pursuant thereto, the fishing vessels were already sought by injunction; because if coordinate courts were
under the jurisdiction of the Court of First Instance of
allowed to interfere with each other's judgments, decrees or
Palawan by virtue of its orders of October 2 and 4, 1965,
injunctions, the same would obviously lead to confusion and
upon motion of the Provincial Fiscal (pp. 54, 55, rec.),
might seriously hinder the administration of justice
directing the Philippine Navy to detain (pp. 108, 109, rec.)
(Ongsinco, etc. vs. Tan, et al., 97 Phil. 330; PNB vs.
said vessels, which are subject to forfeiture as instruments of Javellana, 92 Phil. 525; Montesa vs. Manila Cordage
the crime, to be utilized as evidence in Criminal Cases Nos. Company, 92 Phil. 25; Hubahib vs. Insular Drug Company,
3416 and 3417 for illegal fishing pending in said court (pp.
64 Phil. 119; Hacbang, et al. vs. The Leyte Auto Bus
54-55, rec.). The said vessels were seized while engaging in Company, et al., G.R. No. L-17907, May 30, 1963, 8 SCRA,
prohibited fishing within the territorial waters of Palawan
103, 107-109; NPC vs. Hon. Jesus de Vera, G.R. No. L(pp. 45, 48,-53, rec.) and hence within the jurisdiction of the 15763, Dec. 22, 1961, 3 SCRA, 646, 648; Cabigao vs. del
Court of First Instance of Palawan, in obedience to the rule
Rosario, 44 Phil. 182; Araneta & Uy vs. Commonwealth
that "the place where a criminal offense was committed not Insurance Company, 55 OG 431; Moran, Comments on the
only determines the venue of the action but is an essential
Rules of Court, Vol. III, 1970 ed., p. 64).
element of jurisdiction"(Lopez vs. Paras, L-25795, Oct. 29,
1966, 18 SCRA 616, 619). The jurisdiction over the vessels As early as October 2 and 4, 1965, the two boats were
acquired by the Palawan Court of First Instance cannot be
already in custodia legis under the sole control of the
interfered with by another Court of First Instance. The orders Palawan Court of First Instance. The Manila Court of First
of October 2 and 4, 1965 by the Palawan Court of First
Instance cannot interfere with and change that possession

(Hacbang vs. Leyte Bus Co., Inc., supra; NPC vs. Hon. Jesus
de Vera, supra).
It is immaterial that the vessels were then in the Philippine
Navy basin in Manila; for the same in no way impugns the
jurisdiction already vested in the Palawan court, which has
custody thereof through the Philippine Navy. This is
analogous to the situation in Colmenares versus Villar (L27124, May 29, 1970, 33 SCRA 186, 188-9), wherein We
ruled "where the illegal possession of firearms was
committed in the town where the Court sits, the fact that the
firearms were confiscated from the accused in another town
does not affect the jurisdiction of the Court" (pp. 186, 189).
It is likewise of no moment that the herein respondents were
not notified by the herein petitioners of the seizure of the
questioned vessels by the Philippine Navy, because such
previous notice is not required by law.
II
The dismissal on December 10, 1964 of the first Civil Case
No. 56701 by the Court of First Instance of Manila had the
necessary effect of automatically dissolving the writ of
preliminary mandatory injunction issued therein on April 28,
1964, directing the return of fishing vessel Tony Lex VI (pp.
156-157, rec.). Such a preliminary writ, like any other
interlocutory order, cannot survive the main case of which it
was but an incident; because "an ancillary writ of
preliminary injunction loses its force and effect after the
dismissal of the main petition" (National Sugar Workers'
Union, etc., vs. La Carlota Sugar Central, et al., L-23569,
May 25, 1972, 45 SCRA 104, 109; Lazaro vs. Mariano, 59
Phil. 6Z7, 631; Saavedra vs. Ibaez, 56 Phil. 33, 37; Hi Caiji
vs. Phil. Sugar Estate and Development Company, 50 Phil.
592, 594).1wph1.t
Moreover, the writ of preliminary injunction issued on April
28, 1964 in Civil Case No. 56701 was directed against the
detention of the vessel Tony Lex VI for violations committed

prior to August 5, 1965, and therefore cannot and does not


extend to the seizure and detention of said vessel for
violations on August 5 or 6, 1965, which violations were not
and could not possibly be the subject-matter of said Civil
Case No. 56701 which was filed on April 3, 1964 (p. 12,
rec.).
III
Herein petitioners can validly direct and/or effect the seizure
of the vessels of private respondent for illegal fishing by the
use of dynamite and without the requisite licenses.
Section 4 of Republic Act No. 3512 approved on March 20,
1963 empowers the Fisheries Commissioner to carry out the
provisions of the Fisheries Act, as amended, and all rules and
regulations promulgated thereunder, to make searches and
seizures personally or through his duly authorized
representatives in accordance with the Rules of Court, of
"explosives such as ... dynamites and the like ...; including
fishery products, fishing equipment, tackle and other things
that are subject to seizure under existing fishery laws"; and
"to effectively implement the enforcement of existing fishery
laws on illegal fishing."
Paragraph 5 of Section 4 of the same Republic Act 3512
likewise transferred to and vested in the Philippine Fisheries
Commission "all the powers, functions and duties heretofore
exercised by the Bureau of Customs, Philippine Navy and
Philippine Constabulary over fishing vessels and fishery
matters ..."
Section 12 of the Fisheries Act, otherwise known as
Republic Act No. 4003, as amended, prohibits fishing with
dynamites or other explosives which is penalized by Section
76 thereof "by a fine of not less than P1,500.00 nor more
than P5,000.00, and by imprisonment for not less than one
(1) year and six (6) months nor more than five (5) years,
aside from the confiscation and forfeiture of all explosives,
boats, tackles, apparel, furniture, and other apparatus used

in fishing in violation of said Section 12 of this Act." Section assist the proper governmental agencies in the enforcement
78 of the same Fisheries Law provides that "in case of a
of laws and regulations pertaining to ... fishing ..." (46 OG
second offense, the vessel, together with its tackle, apparel,
5905, 5911).
furniture and stores shall be forfeited to the Government."
Section 2210 of the Tariff and Customs Code, as amended by
The second paragraph of Section 12 also provides that "the
PD No. 34 of October 27, 1972, authorized any official or
possession and/or finding, of dynamite, blasting caps and
person exercising police authority under the provisions of
other explosives in any fishing boat shall constitute a
the Code, to search and seize any vessel or air craft as well
presumption that the said dynamite and/or blasting caps and as any trunk, package, bag or envelope on board and to
explosives are being used for fishing purposes in violation of search any person on board for any breach or violation of the
this Section, and that the possession or discovery in any
customs and tariff laws.
fishing boat or fish caught or killed by the use of dynamite
or other explosives, under expert testimony, shall constitute a When the Philippine Navy, upon request of the Fisheries
presumption that the owner, if present in the fishing boat, or Commissioner, apprehended on August 5 or 6, 1965 the
fishing boats Tony Lex III and Tony Lex VI, otherwise
the fishing crew have been fishing with dynamite or other
known respectively as Srta. Agnes and Srta. Winnie, these
explosives." (Emphasis supplied).
vessels were found to be without the necessary license in
Under Section 78 of the Fisheries Act, as amended, any
violation of Section 903 of the Tariff and Customs Code and
person, association or corporation fishing in deep sea fishery therefore subject to seizure under Section 2210 of the same
without the corresponding license prescribed in Sections 17 Code, and illegally fishing with explosives and without
to 22 Article V of the Fisheries Act or any other order or
fishing license required by Sections 17 and 18 of the
regulation deriving force from its provisions, "shall be
Fisheries Law (pp. 46-47, rec.).1wph1.t
punished for each offense by a fine of not more than
The operation of the fishing boat Tony Lex III was
P5,000.00, or imprisonment, for not more than one year, or
both, in the discretion of the Court; Provided, That in case of suspended pursuant to the order dated January 28, 1964
an association or corporation, the President or manager shall issued by the Commissioner of Fisheries pending the final
determination of the case against it for illegal fishing with
be directly responsible for the acts of his employees or
explosives on January 21, 1964 (p. 34, rec.) and remained
laborers if it is proven that the latter acted with his
knowledge; otherwise the responsibility shall extend only as suspended until its apprehension on August 5 or 6, 1965 (p.
46, rec.).
far as fine is concerned: Provided, further, That in the
absence of a known owner of the vessel, the master, patron
For illegal fishing with explosives on March 23, 1963, the
or person in charge of such vessel shall be responsible for
any violation of this Act: and Provided, finally, That in case renewal of the fishing boat license of Tony Lex VI was
suspended for one year from the time said boat was moored
of a second offense, the vessel together with its tackle,
at Pier 14 at North Harbor, Manila, without prejudice to the
apparel, furniture and stores shall be forfeited to the
institution of a criminal case against its owner and/or
Government" (Emphasis supplied).
operator, pursuant to the order dated May 19, 1964 issued by
Under Section 13 of Executive Order No. 389 of December the Commissioner of Fisheries (pp. 35-36, rec.), the motion
23, 1950, reorganizing the Armed Forces of the Philippines, for reconsideration of which order was denied by the
the Philippine Navy has the function, among others, "to

Commissioner of Fisheries in an order dated August 17,


1964 (pp. 41-42, rec.).
For illegal fishing with dynamite on March 28, 1963, the
operation of Tony Lex VI was suspended by the
Commissioner of Fisheries in an order dated April 1, 1963
(p. 62, rec.).

As heretofore intimated, the two fishing boats were


apprehended on numerous occasions for fishing with
dynamite from March 28, 1963 to March 11, 1964, which
violations private respondent, as owner-operator, sought to
compromise by offering to pay a fine of P21,000.00 for all
said prior violations.

Such previous violations of Sections 12, 17 and 18 of the


For illegal fishing again with explosives on April 25, 1963,
Fisheries Act committed by the two fishing boats, Tony Lex
the fishing boat Tony Lex VI together with its tackle,
III and Tony Lex VI, from March 28, 1963 until August 5 or
apparel, furniture and all other apparatus used in fishing was 6, 1965, rendered the said vessels subject to forfeiture under
ordered confiscated and forfeited in favor of the Government Sections 76 and 78 of the Fisheries Act, as amended.
and a fine in the amount of P5,000.00 was imposed on its
owners-operators, without prejudice to the filing of the
Search and seizure without search warrant of vessels and air
necessary criminal action, pursuant to the order of June 2,
crafts for violations of the customs laws have been the
1964 of the Commissioner of Fisheries(pp. 37-38, rec.).
traditional exception to the constitutional requirement of a
search warrant, because the vessel can be quickly moved out
Again, for comitting the same violation on June 19, 1963, a of the locality or jurisdiction in which the search warrant
fine in the amount of P5,000.00 was imposed on the owners- must be sought before such warrant could be secured; hence
operators of fishing boat Tony Lex VI pursuant to the order
it is not practicable to require a search warrant before such
of June 4, 1964 issued by the Commissioner of Fisheries (pp. search or seizure can be constitutionally effected (Papa vs.
39-40, rec.)..
Mago, L-27360, Feb. 28, 1968, 22 SCRA 857, 871-74;
Magoncia vs. Palacio, 80 Phil. 770, 774; Caroll vs. U.S. 267,
It appears, therefore, that since January 28, 1964, the fishing pp. 132, 149, 158; Justice Fernando, The Bill of Rights, 1972
boat Tony Lex III was suspended from operating and was
ed., p. 225; Gonzales, Philippine Constitutional Law, 1966
ordered moored at Pier 14, North Harbor, Manila (pp. 34,
ed., p. 300).
46-47, rec.); and that the fishing vessel Tony Lex VI was
suspended for one year from May 24, 1964 and was actually The same exception should apply to seizures of fishing
ordered forfeited to the Government pursuant to the order of vessels breaching our fishery laws. They are usually
June 2, 1964 for repeated violations of Section 12 of the
equipped with powerful motors that enable them to elude
Fisheries Act (pp. 37- 38. rec.).1wph1.t As a matter of
pursuing ships of the Philippine Navy or Coast Guard.
fact, when apprehended on August 5 or 6, 1965, both vessels
were found to be without any license or permit for coastwise Another exception to the constitutional requirement of a
trade or for fishing and unlawfully fishing with explosives,
search warrant for a valid search and seizure, is a search or
for which reason their owners and crew were accordingly
seizure as an incident to a lawful arrest (Alvero vs. Dizon, 76
indicted by the Provincial Fiscal of Palawan for illegal
Phil. 637; Justice Fernando, The Bill of Rights, 1972 ed., p.
fishing with dynamite and without the requisite license (pp. 224). Under our Rules of Court, a police officer or a private
48-53, rec.).
individual may, without a warrant, arrest a person (a) who
has committed, is actually committing or is about to commit
an offense in his presence; (b) who is reasonably believed to

have committed an offense which has been actually


committed; or (c) who is a prisoner who has escaped from
confinement while serving a final judgment or from
temporary detention during the pendency of his case or while
being transferred from one confinement to another (Sec. 6,
Rule 113, Revised Rules of Court). In the case at bar, the
members of the crew of the two vessels were caught in
flagrante illegally fishing with dynamite and without the
requisite license. Thus their apprehension without a warrant
of arrest while committing a crime is lawful. Consequently,
the seizure of the vessel, its equipment and dynamites
therein was equally valid as an incident to a lawful arrest.
The alleged compromise approved by the Secretary of
Agriculture and Natural Resources on September 13, 1965
(pp. 63-64, 158-159, rec.) cannot be invoked by the
respondents because the said compromise referred to about
thirty violations of the fisheries law committed by the private
respondent from March 28, 1963 to March 11, 1964. The
violations by the two vessels of private respondent by reason
of which these vessels were apprehended and detained by the
Philippine Navy upon request of the Commissioner of
Fisheries, were committed on August 5 or 6, 1965.
Moreover, the power to compromise would exist only before
a criminal prosecution is instituted; otherwise the
Department Secretary or any of his sub-alterns can render
criminal prosecutions for violations of the fisheries law a
mere mockery. It is not in the public interest nor is it good
policy to sustain the viewpoint that the Department Secretary
can compromise criminal cases involving public, not private,
offenses after the indictment had been instituted in court.
The fishing vessels together with all their equipment and the
dynamites found therein are not only evidence of the crime
of illegal fishing but also subject to forfeiture in favor of the
Government as instruments of the crime (Art. 45, Revised
Penal Code, Sec. 78, Act No. 4003, as amended). Section
80(j) of Act No. 4003, as amended, precludes such a
compromise the moment the Fisheries Commissioner
decides to prosecute the criminal action in accordance with

Sections 76 and 78 of the other penal provisions of the


fisheries law. Furthermore, any compromise shall be upon
the recommendation of the Fisheries Commission (Section
80[i], Act No. 4003), which did not recommend such a
compromise for the violation on August 5 or 6, 1965 of
Section 12 in relation to Sections 76 and 78 of Act No. 4003,
as amended. On the contrary, the Fisheries Commissioner
requested the Provincial Fiscal to institute the criminal cases
(pp. 43-45, rec.) and the Provincial Fiscal filed the
corresponding informations docketed as Criminal Cases Nos.
3416 and 3417 on September 30, 1965 against the owners
and the members of the crew of the vessels (pp. 48-53, rec.).
It should be noted that in the first indorsement dated
September 13, 1965 of the Secretary of Agriculture and
Natural Resources approving the compromise fine of
P21,000.00 for the various violations committed previous to
August 5 or 6, 1965 (pp. 34-42, 47, 58-64, 149-155, 158159, rec.), the Department Secretary "believes that the offer
made by the company was an implied admission of
violations of said provisions of the Fisheries Law and
regulations, ..." (pp. 63, 158, rec.). The said approval was
granted after the private respondent filed a motion for
reconsideration of the indorsement dated March 5, 1965 of
the Secretary of Agriculture and Natural Resources
disapproving the offer by private respondent to pay the fine
by way of compromise.

They can also fall under the term fishing equipment


employed in Section 4 of Republic Act No. 3512; because a
fishing equipment is never complete and cannot be
effectively used in off-shore or deep-sea fishing without the
fishing boat or fishing vessel itself. And these two vessels of
private respondent certainly come under the term fishing
vessels employed in paragraph 5 of Section 4 of the same
Republic Act 3512 creating the Fisheries Commission.

PEOPLE OF THE PHILIPPINES, petitioner,


vs.
COURT OF FIRST INSTANCE OF RIZAL, BRANCH
IX, QUEZON CITY, presided by HON. ULPIANO
SARMIENTO, JESSIE HOPE and MONINA MEDINA,
respondents.

Hence, no useful purpose can be served in trying to


distinguish between boat and vessel with reference to Tony
Lex III and Tony Lex VI. As a matter of fact, the accepted
definition of vessel includes "every description of water
craft, large or small, used or capable of being used as a
means of transportation on water" (Cope versus Vallete, etc.,
199 U.S. 625; U.S. vs. Holmes, 104 Fed. 884; Charles
Barnes Co. vs. One Dredge Boat, 169 Fed. 895; and Yu Con
vs. Ipil, 41 Phil. 780).

GUERRERO, J.:

The word boat in its ordinary sense, means any water craft
(Monongahela River Construction, etc. vs. Hardsaw, 77 NE
363, 365). The fishing boats Tony Lex III and Tony Lex VI
are likewise vessels within the meaning of the term vessel
used in Sections 903 and 2210 of the Tariff and Customs
Code.

WHEREFORE, THE PETITION IS HEREBY GRANTED


AND THE ORDER OF RESPONDENT JUDGE DATED
There can be no dispute that the term fishing boat (employed OCTOBER 18, 1965, THE WRIT OF PRELIMINARY
MANDATORY INJUNCTION ISSUED THEREUNDER
in the second paragraph of Section 12 of the Fisheries Act
AND THE ORDER DATED NOVEMBER 23, 1965, ARE
applies to the vessels Tony Lex III and Tony Lex VI. Even
HEREBY SET ASIDE AS NULL AND VOID, WITH
private respondent refers to said fishing boats as fishing
COSTS AGAINST PRIVATE RESPONDENT.
vessels "engaged in fishing operations" or "in commercial
fishing" in paragraph IV of its complaint in Civil Case No.
62799 (p. 18, rec.), as well as in its various communications People v. CFI of Rizal search of moving vehicle Carroll
doctrine search of moving vehicles or automobiles no
to the Fisheries Commissioner (pp. 60-61, 65, 82,
search warrant needed
rec.).1wph1.t The two fishing vessels Tony Lex III and
Tony Lex VI likewise fall under the term vessel used in
Sections 17, 76 and 78, as well as the term boats utilized in G.R. No. L-41686 November 17, 1980
the second paragraph of Section 76 of the Fisheries Act.

This original petition for certiorari seeks to nullify the Order


dated August 20, 1975 issued by District Judge Ulpiano
Sarmiento in Criminal Case No. Q-3781 which stalled the
prosecution of respondents Sgt. Jessie C. Hope and Monina
Medina for the alleged violation of section 3601 1 of the
Tariff and Customs Code. The order declared as inadmissible
in evidence the allegedly smuggled articles obtained by
apprehending agents in the course of a warrantless search
and seizure. Dispositively, the order decreed:
WHEREFORE, in accordance with Article
IV, Sec. 4, paragraph 2 of the present
Constitution, the boxes and the watches and
bracelets contained therein seized from the
car of the accused Sgt. Jessie C. Hope, are
hereby declared inadmissible in evidence in
this case; likewise, the pictures taken of said
items attempted to be presented as evidence
in the instant case is hereby declared in
admissible as evidence against the accused.
SO ORDERED.
The records disclose that one week before February 9, 1974,
the Regional Anti-Smuggling Action Center (RASAC) was
informed by an undisclosed Informer that a shipment of
highly dutiable goods would be transported to Manila from
Angeles City on a blue Dodge car. Spurred by such lead,
RASAC Agents Arthur Manuel and Macario Sabado, on the

aforesaid date and upon order of the Chief of Intelligence


and Operations Branch, RASAC-MBA, Col. Antonio Abad,
Jr., stationed themselves in the vicinity of the toll gate of the
North Diversion Road at Balintawak, Quezon City.
At about 6:45 A.M. of the same day, a light blue Dodge car
with Plate No. 21-87-73, driven by Sgt. Jessie Hope who
was accompanied by Monina Medina approached the exit
gate and after giving the toll receipt sped away towards
Manila. The RASAC agents gave a chase and overtook Sgt.
Hope's car. Agent Sabado blew his whistle and signaled Sgt.
Hope to stop but the latter instead of heeding, made a U-turn
back to the North Diversion Road, but he could not go
through because of the buses in front of his car. At this point,
the agents succeeded in blocking Sgt. Hope's car and the
latter stopped. Manuel and Sabado who were in civilian
clothes showed their Identification cards to respondents and
introduced themselves as RASAC agents.

(7) more in the baggage compartment which was opened on


orders of Col. Abad. On the same order of the intelligence
officer, the boxes were opened before the presence of
respondents Hope and Medina, representatives of the Bureau
of Internal Revenue, Bureau of Customs, P.C., COSAC and
photographers of the Department of National Defense. The
contents of the boxes revealed some "4,441 more or less
wrist watches of assorted brands; 1,075 more or less watch
bracelets of assorted brands" (based on a later inventory),
supposedly untaxed.

back the boxes to Manila for a consideration of P1,000.00


without disclosing the contents thereof which claimant
simply represented as PX goods; that when he bought the
watches from Buenafe, he presumed that the corresponding
duties have already been paid, only to be surprised later on
when he was informed that the same were seized for nonpayment of taxes.

On the other hand, respondent Hope testified to the effect


that at the time of apprehension, he had no knowledge of the
contents of the boxes, and granting that he had such
As consequence, thereof, ASAC Chairman General Pelagio knowledge, he never knew that these are untaxed
Cruz requested the Bureau of Customs to issue a Warrant of commodities that he consented to transport said boxes from
Seizure and Detention against the articles including the
Angeles City to Manila in his car upon request of his girl
Dodge car. The Collector of Customs did issue the same on friend Monina as a personal favor; that he was not present
February 12, 1974. It was admitted, however, that when the when the boxes were loaded in his car nor was he ever told
apprehending agents arrested respondents and brought them of their contents on the way. On the part of respondent
together with the seized articles to the ASAC Office in Camp Monina Medina, she testified that what she did was only in
Aguinaldo, the former were not armed with a warrant of
compliance with the agreement with Mr. Del Rosario to
The Agents saw four (4) boxes on the back seat of the Dodge arrest and seizure.
transport the boxes and deliver them to a certain Mr. Peter at
and upon inquiry as to what those boxes were, Sgt. Hope
the Tropical Hut who will in turn give her the contracted
answered "I do not know." Further, respondents were asked In conjunction with the Warrant of Seizure and Detention
price; that Mr. Del Rosario did not reveal the contents of the
where they were bringing the boxes, to which respondent
issued by the Collector of Customs, seizure proceedings
boxes which she came to know of only when the boxes were
Medina replied that they were bringing them (boxes) to the
were instituted and docketed as Seizure Identification No.
opened at Camp Aguinaldo. As there was not enough
Tropical Hut at Epifanio de los Santos. Agent Sabado
14281 against the wrist watches and watch bracelets
evidence to controvert the testimonies of respondents and the
boarded the Dodge car with respondents while Agent
pursuant to Section 2530 (m) 1 of the Tariff and Customs narration of claimant Antonio del Rosario, the Collector of
Manuel took their own car and both cars drove towards
Code, and Seizure Identification No. 14281-A against the
Customs issued his decision in the seizure cases on April 1,
2
Tropical Hut making a brief stop at the Bonanza where
Dodge car pursuant to Section 2530(k) of the same Code.
1975 declaring that the seized articles including the car are
Agent Manuel called up Col. Abad by telephone.
not subject of forfeiture. The dispositive portion of this
During the hearing of the aforesaid cases, respondents
decision reads:
Arriving at the Tropical Hut, the party, together with Col.
disclaimed ownership of the seized articles. Ownership was
Abad who had joined them waited for the man who
instead claimed by one Antonio del Rosario who intervened
WHEREFORE, by virtue of Section 2312 of
according to Monina Medina was supposed to receive the
in the proceedings. The claimant-intervenor testified that he
the Tariff and Customs Code, it is hereby
boxes. As the man did not appear, Col. Abad "called off the bought the watches and bracelets from Buenafe Trading as
ordered and decreed that the subject motor
mission" and brought respondents and their car to Camp
evidenced by a sales invoice certified to be authentic by the
vehicle, one (1) Dodge, Model 1965, Motor
Aguinaldo arriving there at about 9:00 A.M. (Respondents'
BIR Revenue Regional Office No. 6 of Quezon City, which
No. 33859, Serial No. W357348361, File
Memorandum, records, pp. 180-183).
transaction was entered in the book of accounts of aforesaid
No. 2B-1884, with Plate No. EH 21-87, '73
claimant; that the same articles were brought to a buyer in
covered by Seizure Identification No.
An inspection of Sgt. Hope's car at Camp Aguinaldo yielded Angeles City, but when the sale failed to materialize,
14281-A be, as it is hereby declared released
eleven (11) sealed boxes, four (4) on the rear seat and seven claimant contracted respondent Monina Medina to transport
to its registered owner, Jessie C. Hope, upon

proper Identification. Relative to Seizure


Identification No. 14281, it is further
ordered and decreed that the subject matter
thereof to wit: 4,606 pcs. of assorted brands
of wrist watches, 1,399 pieces of assorted
brands of wrist bracelets and 100 pcs. of
tools be, as they are hereby likewise
declared released to the rightful owner
thereof, Antonio del Rosario, upon payment
of the levitable duties, taxes and other
charges due thereon plus a fine equivalent to
100% of the duties and taxes thereof.
Furthermore, should claimant-intervenor fail
to pay the assessable duties, taxes and other
charges owing from the aforestated articles
within 30 days from the time this decision
becomes final and unappealable, the same
shall be deemed abandoned in favor of the
government to be disposed of in the manner
provided for by law.
Meanwhile, on March 14, 1974, after the requisite
preliminary investigation, the City Fiscal of Quezon City,
finding the existence of a prima facie case against
respondents Hope and Medina, filed Criminal Case No. Q3781 in the Court of First Instance of Rizal (Quezon City).
Upon arraignment on April 23, 1974, respondents pleaded
not guilty. Trial commenced on January 28, 1975 and while
the prosecution through its first witness, Agent Macario
Sabado, was adducing as evidence the pictures of the eleven
(11) boxes containing the assorted watches and watch
bracelets, counsel for respondents objected to the
presentation of the pictures and the subject articles on the
ground that they were seized without the benefit of warrant,
and therefore inadmissible in evidence under Section 4(2),
Article IV of the New Constitution. After the parties have
argued their grounds in their respective memoranda,
respondent trial court issued the questioned order of August
20, 1975 as cited earlier. The prosecutions motion for
reconsideration was denied on September 30, 1975. Hence,

this petition which was treated as a special civil action in


Our Resolution of May 5, 1976.

and tariff laws as charged in the criminal complaint.


Respondents argue further that the interception of accused
Jessie Hope's car by RASAC Agents while in the course of a
The substantive issue as urged in the petition is whether or
normal trip without any order of the court and without
not the seizure of the merchandise in a moving vehicle by
having shown that the interception was necessary in the
authorized agents commissioned to enforce customs laws
interest of national security, public safety or public health, is
without warrant of seizure breaches the constitutional
an impairment of the liberty of travel under section 5, Article
immunity against unreasonable search and seizure and
IV of the 1973 Constitution. Finally, they claim that the
therefore, such merchandise are inadmissible in evidence.
agents had one week's time before the date of apprehension
Corollary to the issue is, has the trial court gravely abused its to secure the necessary warrant but since they failed to get
discretion in finding the affirmative?
this court order, the search of Hope's car and the spontaneous
seizure of the boxes loaded therein and the contents thereof
The State holds on the proposition that the rules governing
is a violation of the constitutional guarantee against
search and seizure had been liberalized when a moving
"unreasonable searches and seizure of whatever nature and
vehicle is the object of the search and the necessity of a prior for any purpose" under section 3, Article IV of the
warrant has been relaxed on the ground of practicality,
fundamental law.
considering that before a warrant could be obtained, the
place, things and persons to be searched must be described to We find for petitioner. The opposing counsel's attempt to
the satisfaction of the issuing judge a requirement which draw an Identity between the seizure cases and the present
borders on impossibility in the case of smuggling effected by criminal action to the ultimate end that the decision in the
the use of a moving vehicle that can transport contraband
former should be made decisive of the issue of criminal
from one place to another with impunity. Petitioner
liability must be overruled. It is not accurate to say that the
vigorously contends that contraband may be seized without Collector of Customs made no findings that the articles were
necessity of a search warrant since the Constitution does not smuggled. In fact, what the Collector stated was that the
guaranty immunity to smugglers and that a warrantless
prosecution failed to present the quantum of evidence
seizure of contraband in a moving vehicle is justified by the sufficient to warrant the forfeiture of the subject articles
traditional exception attached to the Fourth Amendment of
(Pages 128 and 130 of Annex "E", Records, p. 109). In a
the U.S. Constitution, and such exception must be adopted in general sense, this does not necessarily exclude the
interpreting the relevant provision in the new Philippine
possibility of smuggling. But if the aim of a confirmation
Constitution.
that the goods are indeed smuggled, is to draw an inference
to tie up respondents' criminal liability, the Collector is not
As counter argument, respondents maintain that the decision duty bound, nor is there any need for him to arrive at such a
of the Collector of Customs in their seizure cases which has conclusion. It is quite clear that seizure and forfeiture
now become final and unappealable has made no
proceedings under the tariff and customs laws are not
pronouncement that the subject articles are smuggled items. criminal in nature as they do not result in the conviction of
More so, the decision has entirely cleared respondents of any the offender nor in the imposition of the penalty provided for
liability or responsibility in the alleged smuggling activity
in section 3601 of the Code 3. As can be gleaned from
and as a consequence, the decision has the direct effect of
Section 2533 of the code, seizure proceedings, such as those
deciding finally that the watches and bracelets are not
instituted in this case, are purely civil and administrative in
smuggled and that respondents have not violated the customs character, the main purpose of which is to enforce the

administrative fines or forfeiture incident to unlawful


importation of goods or their deliberate possession. The
penalty in seizure cases is distinct and separate from the
criminal liability that might be imposed against the indicted
importer or possessor and both kinds of penalties may be
imposed. 4

power granted to persons having police authority under


Section 2203 of the Code, who in order to discharge their
official duties more effecttively

In the case at bar, the decision of the Collector of Customs,


as in other seizure proceedings, concerns the res rather than
the persona. The proceeding is a probe on contraband or
illegally imported goods. These merchandise violated the
revenue law of the country, and as such, have been prevented
from being assimilated in lawful commerce until
corresponding duties are paid thereon and the penalties
imposed and satisfied either in the form of fines or of
forfeiture in favor of the government who will dispose of
them in accordance with law. The importer or possessor is
treated differently. The fact that the administrative penalty
befalls on him is an inconsequential incidence to criminal
liability. By the same token, the probable guilt cannot be
negated simply because he was not held administratively
liable. The Collector's final declaration that the articles are
not subject to forfeiture does not detract his findings that
untaxed goods were transported in respondents' car and
seized from their possession by agents of the law. Whether
criminal liability lurks on the strength of the provision of the
Tariff and Customs Code adduced in the information can
only be determined in a separate criminal action.
Respondents' exoneration in the administrative cases cannot
deprive the State of its right to prosecute. But under our
penal laws, criminal responsibility, if any, must be proven
not by preponderance of evidence but by proof beyond
reasonable doubt.
Considering now the critical area of the dispute, under the
law, the authority of persons duly commissioned to enforce
tariff and customs laws is quite exceptional when it pertains or,
to the domain of searches and seizures of goods suspected to
have been introduced in the country in violation of the
customs laws. This Court had occasion to recognize this

... may at anytime enter, pass through, or


search any land or inclosure of any
warehouse, store or other building not being
a dwelling house. (Section 2208, emphasis
supplied)
... (to) go aboard any vessel or aircraft
within the limits of any collection district,
and to inspect, search and examine said
vessel or aircraft and any trunk, package,
box or envelope on board, and search any
person on board the said vessel or aircraft
and to this end to hail and stop such vessel
or aircraft if under way. to use all necessary
force to compel compliance; and if it shall
appear that any breach or violation of the
customs and tariff laws of the Philippines
has been committed, whereby or in
consequence of which such vessels or
aircrafts, or the article, or any part thereof,
on board of or imported by such vessel or
aircrafts, is hable to forfeiture to make
seizure of the same or any part thereof.
The power of search herein above given
shall extend to the removal of any false
bottom, partition, bulkhead or other
obstruction, so far as may be necessary to
enable the officer to discover whether any
dutiable or forfeitable articles may be
concealed. (Section 2210)

... (to) open and examine any box, trunk,


envelope or other container wherever found

when he has reasonable cause to suspect the


presence therein of dutiable or prohibited
article or articles introduced into the
Philippines contrary to law, and likewise to
stop, search and examine any vehicle, beast
or person reasonably suspected of holding
or conveying such article as aforesaid
(Section 2211, emphasis supplied)
As enunciated in the leading case of Papa v. Mago 5, in the
exercise of the specific functions aforecited, the Code does
not mention the need of a search warrant unlike Section 2209
which explicitly provides that a "dwelling house may be
entered and searched only upon warrant issued by a judge
(or justice of the peace), upon swom application showing
probable cause and particularly describing the place to be
searched and person or thing to be seized." Aware of this
delineation, the Court in that case expressed the considered
view that "except in the case of the search of a dwelling
house, persons exercising police authority under the customs
law may effect search and seizure without a search warrant
in the enforcement of customs laws.
The rationale of the Mago ruling was nurtured by the
traditional doctrine in Carroll v. United States 6 wherein an
imprimatur against constitutional infirmity was stamped in
favor of a warrantless search and seizure of such nature as in
the case at bar. On this stable foundation We refute the
constitutional charge of respondents that the warrantless
seizure violated Article IV, Section 3 of the 1973
Constitution, which finds origin in the Fourth Amendment of
the American Constitution 7
The Carroll doctrine arose from the indictment and
conviction of George Carroll and partner for transporting in
an automobile intoxicating liquor in violation of the National
Prohibition Act. They assailed the conviction on the ground
that the trial court admitted in evidence two of the sixtyeight bottles found by searching the automobile and eventual
seizure of the same allegedly in violation of the 4th

Amendment, and therefore that the use of the liquor as


evidence was improper. 8 To paraphrase the significant views
of Mr. Chief Justice Taft, the legislative history of the Act
clearly established the intent of Congress to make a
distinction between the necessity for a search warrant in the
search of private dwellings and that of automobiles and other
road vehicles in the enforcement of the Act. This distinction
is consistent with the 4th Amendment since the latter does
not denounce an searches or seizures, but only such as are
unreasonable. Searches and seizures without warrant are
valid if made upon probable cause, that is, upon a belief
reasonably arising out of circumstances known to the seizing
officer, that an automobile or other vehicle contains that
which by law is subject to seizure and destruction. 9
Similarly, other statutes of the Union such as the Act of
1789, Act of August 4, 1790, and Act of March 3, 1815,
among others, construed in the light of the 4th Amendment
had recognized the distinctive feature of a warrantless search
of a ship motorboat, wagon, or automobile for contraband
goods where it is not practicable to secure a warrant because
the vehicle can be quickly moved out of the locality or
jurisdiction in which the warrant must be sought. 10 In such a
situation, what appears to the measure of legality of the
seizure was formulated in this sense: "that the seizing officer
shall have reasonable or probable cause for believing that the
automobile which he stops and seizes has contraband liquor
therein which is being illegally transported. " Therein the
guarantee of the 4th Amendment was fulfilled. Where
seizure is impossible except without warrant, the seizing
officer acts unlawfully and at his peril unless he can show
the court probable cause. 11

their duty. The records hardly reveal anything certain and


confirmatory of the report during the said period except the
general knowledge that some highly dutiable goods would
be transported from Angeles City to Manila in a blue Dodge
automobile. Not even the trial court has made any findings
that ASAC has established with exactitude the place to be
searched and the person or thing to be seized. Lacking this
essential determination, the agents could not have possibly
secured a valid warrant even if they had foreseen its
compelling necessity. For one thing, the information could
have been just another false alarm. Providentially, however,
things turned out differently when in the morning of
February 9, 1974, the undisclosed Informer himself went
along with the agents to the rendezvous point where at the
appointed time he positively Identified an approaching car as
the one described by him a week earlier to be the suspected
carrier of untaxed merchandise. Clearly therefore, the agents
acted not on the basis of a mere hearsay but on a confirmed
information worthy of belief and probable cause enough for
them to adopt measures to freeze the fleeting event.

The purpose of the constitutional guarantee against


unreasonable searches and seizures is to prevent violations of
private security in person and property and unlawful
invasion of the sanctity of the home by officers of the law
acting under legislative or judicial sanction and to give
remedy against such usurpation when attempted. 14 The right
to privacy is an essential condition to the dignity and
happiness and to the peace and security of every individual,
whether it be of home or of persons and correspondence. 15
The constitutional inviolability of this great fundamental
right against unreasonable searches and seizures must be
deemed absolute as nothing is more closer to a man's soul
than the serenity of his privacy and the assurance of his
personal security. Any interference allowable can only be for
the best of causes and reasons. We draw from the context of
the Constitution that an intended search or seizure attains a
high degree of propriety only when a probable cause duly
determined is branded on a warrant duly issued by a judge or
other responsible person as may be authorized by law. Not
invariably, however, the reasonableness or unreasonableness
of the interference is not wholly defendent on the presence
We need not argue that the subjective phase of the police
of a warrant or the lack of it. In the ordinary cases where
action taken by the ASAC Agents to effect the apprehension warrant is indispensably necessary, the mechanics prescribed
of the suspected violators can be anything less than the
by the Constitution and reiterated in the Rules of Court must
ensuing interception and stoppage of respondents' vehicle
be followed and satisfied. But We need not argue that there
after a short chase. Neither can We sustain the argument that are exceptions. Thus, in the extraordinary events where
in doing so, the agents violated respondents' constitutional
warrant is not necessary to effect a valid search or seizure, or
"liberty of travel". To recall again Mr. Chief Justice Taft:
when the latter cannot be performed except without warrant,
"(B)ut those lawfully within the country, entitled to use the
what constitutes a reasonable or unreasonable search or
public highways, have a right to free passage without
seizure becomes purely a judicial question, determinable
interruption or search unless there is known to a competent
from the uniqueness of the circumstances involved,
official authorized to search, probable cause for believing
including the purpose of the search or seizure, the presence
The counsel for the State is candid enough to admit that the that their vehicles are carrying contraband or illegal
or absence of probable cause, the manner in which the search
13
Anti-Smuggling Action Center tries its best to follow-up the merchandise." What followed next in the scene was a
and seizure was made, the place or thing searched and the
more promising tips and information from informers, but
simple inquiry as to the contents of the boxes seen inside the character of the articles procured. 16
ever often, the information proves false or the smugglers are car. Respondents' baffled denial of knowledge thereof could
forewarned. 12 It is quite true the ASAC received one such
not but only heighten the suspicion of a reasonable and
The ultimate question then, if any, that should confront the
information several days or a week before the encounter; but inquisitive mind. Thus, the probable cause has not been any actuations of the ASAC Agents in this case is whether the
the fact that its agents failed to obtain a warrant in spite of
less mitigated.
warrantless search and seizure conducted by them is lawful
the time allowance is not a sign that they have been remiss in
or not. We have already seen that what they did was a

faithful performance of a duty authorized under the Tariff


and Customs Code directing them as authorized agents to
retrieve articles reasonably suspected of having been
possessed, issued or procured in violation of the tariff laws
for which the government has a direct interest. The official
capacity of the agents has never been questioned by
respondents. Neither did respondents raise an issue on the
constitutionality of the law giving the agents the power to act
as mandated. There 'is no question that the Agents have not
exceeded their authority nor have they acted so licentiously
to bear upon respondents moral embarrassment or substantial
prejudice beyond what is necessary. The purpose of the
search and seizure is more than clear to Us, hence, We rule
out the suspicion that the intention is only to elicit evidence
to be used against respondents.
We do not see strong justification for the trial court's failure
to recognize the circumstances at bar as among the "rare
cases" which it admittedly conceded to be exempted from
the requirement of a warrant. 17 The lapse lies on the dismal
gap in the trial court's developmental treat- ment of the law
on arrest, search and seizure. It missed the vital distinction
emphatically laid down in Boyd v. United States 18 which was
cited in Carroll with "particular significance and
applicability." Thus, We quote Mr. Justice Bradley in Boyd:
... The search and seizure of stolen or
forfeited goods, or goods liable to duties
and concealed to avoid the payment thereof,
are totally different things from a search for
and seizure of a man's private books and
papers for the purpose of obtaining
information therein contained, or of using
them as evidence against him, The two
things differ in toto coelo. In the one case,
the government is entitled to the possession
of the property; in the other it is not. The
seizure of stolen goods is authorized by the
common law; and the seizure of goods
forfeited for a breach of the revenue laws or

concealed to avoid the duties payable on


them, has been authorized by English
statutes for at least two centuries past; and
the like seizure have been authorized by our
revenue acts from the commencement of the
government. The first statute passed by
Congress to regulate the collection of duties,
the Act of July 31, 1789. 1 State at L. 29, 43,
chap. 5, contains provisions to this effect. As
this act was passed by the same Congress
which proposed for adoption the original
Amendments to the Constitution, it is clear
that the members of that body did not regard
searches and seizures of this kind as
'unreasonable' and they are not embraced
within the prohibition of the Amendment. So
also the supervision authorized to be
exercised by officers of the revenue over the
manufacture of custody of excisable articles,
and the entries thereof in books required by
law to be kept for their inspection, are
necessarily excepted out of the category of
unreasonable searches and seizures. So also
the laws which provide for the search and
seizure of articles and things which it is
unlawful for a person to have in his
possession for the purpose of issue or
disposition, such as counterfeit coin, lottery
tickets, implements of gambling, etc. are not
within this category. Commonwealth v.
Dana, 2 Met 329. Many other things of this
character might be enumerated. (Emphasis
supplied).
Recently, in Viduya v. Berdiago 19 " this Court reiterated the
controlling force of the Papa v. Mago ruling hereinbefore
cited and the persuasive authority of the leading decision in
Carroll v. U.S., supra, and in explaining the rationale of the
doctrine significantly said that "(i)t is not for this Court to do
less than it can to implement and enforce the mandates of the

customs and revenue laws. The evils associated with tax


evasion must be stamped out without any disregard, it is
to be affirmed, of any constitutional right ...
The circumstances of the case at bar undoubtedly fall
squarely within the privileged area where search and seizure
may lawfully be effected without the need of a warrant. The
facts being no less receptive to the applicability of the classic
American ruling, the latter's force and effect as well as the
Mago decision must be upheld and reiterated in this petition.
the find that the constitutional guarantee has not been
violated and the respondent court gravely erred in issuing the
order of August 20, 1975 declaring as inadmissible evidence
the items or articles obtained and seized by the apprehending
agents without any search warrant, as well as the pictures of
said items attempted to be presented as evidence against the
accused.
Notwithstanding the reversal and setting aside of the order of
respondent judge assailed herein, thereby allowing the
introduction and admission of the subject prohibited articles
in the trial of the accused Jessie C. Hope and Monina
Medina for alleged smuggling, in the interest of speedy
justice, the prosecution is directed forthwith to re-assess and
re-evaluate the evidence at its disposal, considering the lapse
of time since the trial commenced on June 28, 1975 and was
thus delayed due to the filing of the instant certiorari petition
and that on April 1, 1975, after seizure proceedings initiated
by the Collector of Customs, the said articles were ordered
released upon payment of the leviable duties, taxes and other
charges due thereon plus a fine equivalent to 100% of the
duties and taxes thereof. After such re-assessment and reevaluation, the prosecution must promptly take the necessary
action on the premises for the protection of the rights and
interests of all parties concerned.
WHEREFORE, the Order appealed from is hereby set aside
and the case is ordered remanded for further trial and
reception of evidence without excluding the articles subject
of the seizure or for such action as the prosecution may take

after the re-assessment and re-evaluation of its evidence as


hereinabove directed.

W/N petitioner, in the discharge of his official


function, lay himself open to a criminal prosecution
for usurpation of judicial functions

more easily understandable why that person, TeofilaIbaez, who could


be aptly described as the wrong person atthe wrong place and at the
wrong time, would have signifiedher consent readily and
immediately.Under the circumstances, that was the most prudentcourse
This judgment is immediately executory.
of action. It would save her and even petitioner Velasco himself from
HELD:
any gossip or innuendo. Nor could theofficers of the law be blamed if
SO ORDERED.
they would act on theappearances. There was a person inside who from
It is undeniable that petitioner, as Acting Collector of
allindications was ready to accede to their request. Evencommon
Customs for the Port of Manila, had the requisite authority
Pacis vs. Pamaran
courtesy alone would have precluded them frominquiring too closely as
for the issuance of the contested warrant of seizure and
to why she was there. Under all thecircumstances, therefore, it can
detention for the automobile owned by respondent Ricardo
readily be concluded thatthere was consent sufficient in law to dispense
Santos. What was done by him certainly could not be the
56 SCRA 16 (1974)
with the needfor a search warrant. The petition cannot, therefore, prevail.
basis of a prosecution for the usurpation of judicial
Lopez vs. Commissioner of Customs [GR L-27968, 3 December 1975]
functions. The remedy of prohibition lies.
Second Division, Fernando (J): 4 concur, 1 took no part
Power of Acting Commissioner of Customs to issue Lopez and Velasco vs. Commissioner of Customs
Facts:
Confusing
case
where
it
is
adverse
to
later
jurisprudence
that
a
waiver
of
a warrant of seizure and detention
right can only be waived by the person whoseright against unreasonable M/V Jolo Lema had been under strict surveillance by the
combined team of agents of the NBI, PC, RASAC, and City
search and seizure was invaded.
FACTS:
Facts: After the surveillance conducted by the respondentsNBI officers, Police of Davao prior to its apprehension at a private wharf
inBatjak, Sasa, Davao City. M/V [Jolo Lema] was skippered
NBI and PC Davao officers went to the room(Rm 22
Respondent Ricardo Santos is the owner of a Mercury
(sic) by Capt. Aquilino Pantinopleand chartered by Mr.
0
automobile, model 1957, brought into the country without
Tomas Velasco. During the period from the latter part of
)
rented
by
the
petitioner,
Tomas
Velasco,
to
searchand
seized
articles
payment of customs duty and taxes because its original
August toSeptember 18, 1966, the said vessel was in
owner, Donald James Hatch, was tax-exempt. Santos later on papers and documents including a .45 calpistol, that became evidence
Indonesian waters where it loaded copra and coffee beans
that commodities confiscated inthe wharf (MV Jojo Lema) allegedly
paid P311.00 for customs duty and taxes.
from Taruna, Pitta, and Mangenito, all of Indonesia. In its
smuggled fromIndonesia to the country (sacks of coffee beans and
trip to Indonesia it broughtvarious merchandise from the
On July 22, 1964, Acting Collector of Customs Pedro Pacis copra).The search was without a search warrant, however,the officers
Philippines which were exchanged and/or bartered for copra
have
successfully
confiscated
the
articles
byvirtue
of
the
consent
of
the
was informed by the General Affairs Administration of the
andcoffee beans and subsequently taken to Davao City. Said
petitioners
wife
(Teofila
ibanez)who
also
is
an
occupant
of
the
room
Department of National Defense that the automobile was a
vessel passed Marore, Indonesia on 18
hot car. By virtue thereof, Pacis, through his subordinates, rented by the petitioner.The petitioner contends that the consent given
looked into the records of his office and found that although byIbanez cannot be regarded since she was not the legal wifeof the
petitioner Velasco, but a certain Corazon Velasco.The decision of CTAis September 1966 on its a way to Tahuna, Indonesia before
the amount of P311.00 was already paid for customs duty,
adverse by the petitioner,affirmed by the SC.Issue:Was the consent valid proceeding to Davao City where it wasapprehended on 19
the amount collectible on the said car should be P2,500.00,
to justify the warrantlesssearch and seizure?Held: As far as the decision September 1966. At about 3:00 p.m. of the said day, when
more or less.
is concerned, yes, theconsent given by Ibanez is a valid so as to dispense the vessel wassearched and after Captain Pantinople
informed the team that Velasco, the charterer of the
Based on such discrepancy, he instituted seizure proceedings thenecessity of a search warrant.The court ruled that the mere fact that
vessel,had other documents showing that vessel came from
Ibanez ispresent in a room rented by the petitioner, her consent
and issued a warrant of seizure and detention. The
Indonesia carrying smuggled copra andcoffee, a combined
wouldlead to belief that her consent as an alleged wife of thepetitioner
automobile was also taken by the Department of National
team of Constabulary and Regional Anti-Smuggling Center
and that it would be an act on behalf of thepetitioner.
Defense agents and brought to the General Affairs
operativesheaded by Earl Reynolds, Senior NBI Agent of
Note:
Administration for compound.
Davao, proceeded to the Velascos room at theSkyroom
Exact decision penned by J. LaurelThere was an attempt on the part of
Hotel in Davao City, to ask for said document. Velasco was
In answer, Santos filed a criminal complaint against Pacis for petitioners tocounteract the force of the above recital by an affidavit of
oneCorazon Y. Velasco, who stated that she is the legal wife of petitioner not inside the hotel roomwhen they entered the room. There
usurpation of judicial functions with the City Fiscal of
Tomas Velasco, and another by such petitioner himself reiterating such a are conficting claims whether the manicurist Teofila
Manila, Manuel Pamaran, alleging that Pacis did not have
Ibaezor whether Velascos wife, who was allegedly inside
fact and that the person who waspresent at his hotel room was one
authority to issue such warrant of seizure and detention.
the room at that time, voluntarily allowedthe police officers
Teofila Ibaez, "amanicurist by occupation." Their effort appurtenant
thereto isdoomed to failure. If such indeed were the case, then it ismuch to enter; and whether the police officers forcibly opened
ISSUE:

luggages and boxesfrom which only several documents and


papers were found, then seized, confiscated and took away
the same, or whether Mrs. Velasco volunteered to open the
suitcases and baggages of Velasco and delivered the
documents and things contained therein to Reynolds. The
Collector of Customs of Davao seized 1,480 sacks of copra
and 86 sacks of coffee from the M/V motor vesselJolo Lema.
The seizure was declared lawful by the Court of Tax
Appeals, and its decision wasaffirmed by the Supreme Court
on 29 November 1974 in Nasiad vs. Court of Tax Appeals
(GR L-29318, November 29, 1974, 61 SCRA 238). In the
present special civil action for certiorari, prohibition and
mandamus; the only question left then is whether the search
conducted by a partyheaded by Reynolds without the search
warrant for the hotel room of Velasco, who entered into
acontract with Jose G. Lopez, the awardee of such Philippine
Reparations Commission vessel, for its operation and use
ostensibly for fishing, is violative of such constitutional
provision.
Issue:
Whether there was consent on the part of the person who
was the occupant of the hotelroom then rented by Velasco.
Held:
There was an attempt on the part of Lopez and Velasco to
counteract the force of therecital of the written statement of
Teofila Ibaez (allegedly wife of Tomas Velasco) by
anaffidavit of one Corazon Y. Velasco, who stated that she is
the legal wife of Velasco, and another by Velasco himself;
reiterating that the person who was present at his hotel room
was oneTeofila Ibaez, a manicurist by occupation. If such
indeed were the case, then it is much moreeasily
understandable why that person, Teofila Ibaez, who could
be aptly described as thewrong person at the wrong place
and at the wrong time, would have signified her consent
readilyand immediately. Under the circumstances, that was
the most prudent course of action. It wouldsave her and even
Velasco himself from any gossip or innuendo. Nor could the
officers of thelaw be blamed if they would act on the
appearances. There was a person inside who from
allindications was ready to accede to their request. Even
common courtesy alone would have precluded them from
inquiring too closely as to why she was there. Under all the
circumstances,therefore, it can readily be concluded that
there was consent sufficient in law to dispense with theneed
for a search warrant.

Whether the warrantless seizure incidental to the buy-bust


operation violates Beltransconstitutional rights against
People vs. dela Cruz [GR 83260, 18 April 1990]
unreasonable search and seizure.
Second Division, Regalado (J): 4 concur
Held:
Facts:
A buy-bust operation is the method employed by peace
After receiving a confidential report from Arnel, their
informant, a buy-bust operationwas conducted by the 13th officers to trap and catch amalefactor in flagrante delicto. It
Narcotics Regional Unit through a team composed of T/Sgt. is essentially a form of entrapment since the peace
JaimeRaposas as Team Leader, S/Sgt. Rodelito Oblice, Sgt. officer neither instigates nor induces the accused to commit a
crime. Entrapment is the employment of such ways and
Dante Yang, Sgt. Vicente Jimenez, P/Pfc.Adolfo Arcoy as
means for the purpose of trapping or capturing a lawbreaker
poseur-buyer and Pat. Deogracias Gorgonia at Maliclic St.,
from whose mind thecriminal intent originated. Oftentimes,
Tondo, Manila ataround 2:30 p.m. of 4 May 1987 to catch
the pusher/s. P/Pfc. Adolfo Arcoy acted as the poseur- buyer it is the only effective way of apprehending a criminal inthe
with Arnel as his companion to buy marijuana worth P10.00 act of the commission of the offense. While it is conceded
from the two accused, Juande la Cruz and Reynaldo Beltran. that in a buy-bust operation, thereis seizure of evidence from
ones person without a search warrant, needless to state a
At the scene, it was Juan de la Cruz whom Arcoy
firstnegotiated with on the purchase and when Arcoy told De searchwarrant is not necessary, the search being incident to a
la Cruz that he was buying P10.00worth of marijuana, De la lawful arrest. A peace officer may,without a warrant, arrest a
Cruz instructed Reynaldo Beltran to give one aluminum foil person when, in his presence, the person to be arrested has
committed,is actually committing or is attempting to commit
of marijuana which Beltran got from his pants pocket and
an offense. It is a matter of judicial experiencethat in the
delivered it to Arcoy. After ascertainingthat the foil of
arrest of violators of the Dangerous Drugs Act in a buy-bust
suspected marijuana was really marijuana, Arcoy gave the
operation, the malefactorswere invariably caught redprearranged signal tohis teammates by scratching his head
handed. There being no violation of the constitutional right
and his teammates who were strategically positioned in
thevicinity, converged at the place, identified themselves as againstunreasonable search and seizure, the confiscated
articles are admissible in evidence.
NARCOM agents and effected thearrest of De la Cruz and
Beltran. The P10.00 marked bill used by Arcoy was found in
the possession of Juan de la Cruz together with two
NOLASCO VS PANO147 SCRA 509 (1987)Facts
aluminum foils and containing marijuana. Juande la Cruz y
: The case at bar is for the motion for partial reconsideration
Gonzales and Reynaldo Beltran y Aniban were charged in
of both petitioners and respondents of the SCs decision that
Criminal Case 87-54417of the Regional Trial Court (RTC)
the questioned search warrant by petitioners is null and void,
of Manila with violation of Section 4, Art. II, in relation
that respondents areenjoined from introducing evidence
toSection 21, Article IV of Republic Act 6425, as amended. using such search warrant, but such personalities obtained
The court, on 15 March 1988, foundDela Cruz and Beltran
wouldstill be retained, without prejudice to petitioner
guilty beyond reasonable doubt and sentenced each of them Aguilar-Roque. Respondents contend that the searchwarrant
to suffer the penalty of reclusion perpetua, with the
is valid and that it should be considered in the context of the
accessory penalties provided by law; to pay a fine
crime of rebellion, where thewarrant was based. Petitioners
of P20,000.00, without subsidiary imprisonment in case of
on the other hand, on the part of petitioner Aguilar-Roque,
insolvency, and each to pay one-half of the costs. From this contend thata lawful search would be justified only by a lawful arrest.
decision, de la Cruz and Beltran appealed. In a letter of the
And since there was illegal arrest of Aguilar-Roque, the search was
Warden, ManilaCity Jail, dated 3 March 1989, the Court was unlawful and that the personalities seized during the illegal
informed of the death of de la Cruz on 21 February1989.
searchshould be returned to the petitioner. The respondents,
Thus, the criminal case against de la Cruz was dismissed in in defense, concede that the search warrants werenull and
the Supreme Court resolutionof 25 September 1989. The
void but the arrests were not.
present appellate proceeding is limited only to Beltran.
Issue
Issue:
: WON the articles seized were illegally obtained.

Ruling
: Yes.
RD
: "Any evidence obtained in violation of this . . . section shall
be inadmissible for any purpose in anyproceeding" (Sec.
4[2]). This constitutional mandate expressly adopting the
exclusionary rule hasproved by historical experience to be
the only practical means of enforcing the
constitutionalinjunction against unreasonable searches and
seizures by outlawing all evidence illegally seized
andthereby removing the incentive on the part of state and
police officers to disregard such basic rights.What the plain
language of the Constitution mandates is beyond the power
of the courts to change ormodify. All the articles thus seized
fag under the exclusionary rule totally and unqualifiedly and
cannotbe used against any of the three petitioners.
People vs Anita Claudio
G.R. No. 72564
April 15, 1988

is a well-established rule that alibi cannot prevail over


positive testimony. The judgment appealed from is
AFFIRMED.
Sec. 4, Art II of R.A. 5425

The provision provides the Sale, Administration, Delivery


Distribution and Transportation of Prohibited Drugs where
the penalty of life imprisonment to death and a fine ranging
from twenty thousand to thirty thousand pesos shall be
imposed upon any person who, unless authorized by law,
shall sell, administer, deliver, give away to another,
distribute, dispatch in transit or transport any prohibited
drug, or shall act as a broker in any of such transactions.
Although the accused contends that she may not be
convicted of this provision, the court held that contention is
without merit. A closer perusal of the subject provision
shows that it is not only delivery which is penalized but also
the sale, administration, distribution and transportation of
Facts:
prohibited drugs. Claudio was caught transporting 1.1 kilos
On or about 21 July 1981, in the Olongapo City, Philippines, of marijuana, thus the lower court did not err in finding her
guilty of violating Sec. 4.
the above-named ACCUSED without being lawfully
authorized, did then and there willfully, unlawfully and
As held in the case of People v. Toledo, (140 SCRA 259,
knowingly transport 1.1 kilos of Marijuana dried leaves,
267) "the possession of such considerable quantity as three
which are prohibited drugs for the purpose of selling the
plastic bags of marijuana leaves and seeds coupled with the
same from Baguio City to Olongapo City.
fact that he is not a user of prohibited drugs cannot indicate
anything except the intention of the accused to sell,
Issues:
distribute and deliver said marijuana.
a)Whether or not the accused is also liable Sec. 4, Art. II of
Rule 126, Sec. 12
R.A. 6425 aside from Sec. 8, Art. II of the same Act?
b)Whether warrantless search, seizure, and apprehension is
unlawful under Rule 126, Sec.12?
Held:
Yes. In the case at bar, alibi does not deserve much credit as
it was established only by the accused herself. Moreover, it

flagrante delicto. The warrantless search being an incident to


a lawful arrest is in itself lawful. (Nolasco v. Pano, 147
SCRA 509). Therefore, there was no infirmity in the seizure
of the 1.1 kilos of marijuana.

The provision provides the Search incident to lawful arrest


where a person lawfully arrested may be searched for
dangerous weapons or anything which may be used as proof
of the commission of an offense, without a search warrant in
paragraph (12a). Thus, appellant Claudio was caught
transporting prohibited drugs. Pat. Daniel Obia did not need
a warrant to arrest Claudio as the latter was caught in

PEOPLE VS. DEL ROSARIO [234 SCRA 246; G.R. NO.


109633; 20 JUL 1994]
Wednesday, February 04, 2009 Posted by Coffeeholic Writes
Labels: Case Digests, Political Law
Facts: Accused was charged and convicted by the trial court
of illegal possession of firearms and illegal possession and
sale of drugs, particularly methamphetamine or shabu. After
the issuance of the search warrant, which authorized the
search and seizure of an undetermined quantity of
methamphetamine and its paraphernalias, an entrapment
was planned that led to the arrest of del Rosario and to the
seizure of the shabu, its paraphernalias and of a .22 caliber
pistol
with
3
live
ammunition.
Issue: Whether or Not the seizure of the firearms was
proper.
Held: No. Sec 2 art. III of the constitution specifically
provides that a search warrant must particularly describe the
things to be seized. In herein case, the only objects to be
seized that
the
warrant
determined was
the
methamphetamine and the paraphernalias therein. The
seizure
of
the
firearms
was
unconstitutional.
Wherefore the decision is reversed and the accused is
acquitted.

** Harvey v Santiago 162 SCRA 840 (1988)


Facts: This is a petition for Habeas Corpus. Petitioners are
the following: American nationals Andrew Harvey, 52 and
Jonh Sherman 72. Dutch Citizen Adriaan Van Den Elshout,

58. All reside at Pagsanjan Laguna respondent


Commissioner Miriam Defensor Santiago issued Mission
Orders to the Commission of Immigration and Deportation
(CID) to apprehended petitioners at their residences. The
Operation Report read that Andrew Harvey was found
together with two young boys. Richard Sherman was found
with two naked boys inside his room. While Van Den
Elshout in the after Mission Report read that two children
of ages 14 and 16 has been under his care and subjects
confirmed being live-in for sometime now.

seizures by CID agents.


(3) Whether or Not the writ of Habeas Corpus may be
granted to petitioners.

Held: While pedophilia is not a crime under the Revised


Penal Code, it violates the declared policy of the state to
promote and protect the physical, moral, spiritual and social
well being of the youth. The arrest of petitioners was based
on the probable cause determined after close surveillance of
3 months. The existence of probable cause justified the arrest
Seized during the petitioners apprehension were rolls of
and seizure of articles linked to the offense. The articles were
photo negatives and photos of suspected child prostitutes
seized as an incident to a lawful arrest; therefore the articles
shown in scandalous poses as well as boys and girls engaged are admissible evidences (Rule 126, Section12 of Rules on
in sex. Posters and other literature advertising the child
Criminal Procedure).
prostitutes were also found.
The rule that search and seizures must be supported by a
Petitioners were among the 22 suspected alien pedophiles.
valid warrant of arrest is not an absolute rule. There are at
They were apprehended 17 February1988 after close
least three exceptions to this rule. 1.) Search is incidental to
surveillance for 3 month of the CID in Pagsanjan, Laguna.
the arrest. 2.) Search in a moving vehicle. 3.) Seizure of
17 of the arrested aliens opted for self-deportation. One
evidence in plain view. In view of the foregoing, the search
released for lack of evidence, another charged not for
done was incidental to the arrest.
pedophile but working with NO VISA, the 3 petitioners
chose to face deportation proceedings. On 4 March1988,
The filing of the petitioners for bail is considered as a waiver
deportation proceedings were instituted against aliens for
of any irregularity attending their arrest and estops them
being undesirable aliens under Sec.69 of
from questioning its validity. Furthermore, the deportation
Revised Administrative Code.
charges and the hearing presently conducted by the Board of
Special Inquiry made their detention legal. It is a
Warrants of Arrest were issued 7March1988 against
fundamental rule that habeas corpus will not be granted
petitioners for violation of Sec37, 45 and 46
when confinement is or has become legal, although such
of Immigration Act and sec69 of
confinement was illegal at the beginning.
RevisedAdministrative Code. Trial by the Board of Special
Inquiry III commenced the same date. Petition for bail was
The deportation charges instituted by the Commissioner
filed 11March 1988 but was not granted by the
ofImmigration are in accordance with Sec37 (a) of
Commissioner of Immigration. 4 April1988 Petitioners filed the PhilippineImmigration Act of 1940 in relation to sec69
a petition for Writ of Habeas Corpus. The court heard the
of the RevisedAdministrative code. Section 37 (a) provides
case on oral argument on 20 April 1988.
that aliens shall be arrested and deported upon warrant of the
Commissioner ofImmigration and Deportation after a
Issues:
determination by the Board of Commissioners of the
existence of a ground for deportation against them.
(1) Whether or Not the Commissioner has the power to arrest Deportation proceedings are administrative in character and
and detain petitioners pending determination of existence of never construed as a punishment but a preventive measure.
probable cause.
Therefore, it need not be conducted strictly in accordance
with ordinary Court proceedings. What is essential is that
(2) Whether or Not there was unreasonable searches and
there should be a specific charge against the alien intended to

be arrested and deported. A fair hearing must also be


conducted with assistance of a counsel if desired.
Lastly, the power to deport aliens is an act of the State and
done under the authority of the sovereign power. It a police
measure against the undesirable aliens whose continued
presence in the country is found to be injurious to the public
good and tranquility of the people.
EXC: determination by administrative officials
CASES: Morano vs, Vivo, 20 SCRA
562
Facts:
Petitioners are Chinese nationals (Chan Sau Wah from
Fukein, with a minor childfrom prior marriage, Fu Yan Fun)
who were granted a temporary visitors visa as momimmigrant for 2 months upon posting P4k cash bond to visit
a cousin in the Philippines.She soon married to Esteban
Morano, a Filipino Citizen, on January 24, 1962 and
gaveb i r t h t o a c h i l d , E s t e b a n M o r a n o , J r . A f t e r
s e v e r a l e x t e n s i o n s t o p r o l o n g s t a y i n Philippines,
their visas expired on Sept. 10, 1962 and were ordered by
Commissioner of Immigration (COI) on Aug. 31, 1962 thru a
letter, to leave the country on or before Sept.1 0 , 1 9 6 2
with warning of issuance of warrant of
a r r e s t f o r f a i l u r e t o l e a v e a n d confiscation of
bond.Petitioners then filed with the CFI of Manila
for Mandamus to compel COI to cancel their ACR, to
stop issuing arrest warrant, and preliminary injunction from
confiscatingtheir bond. They argue that Chan Sau Wah
became a Filipino Citizen upon marriage toE s t e b a n
Morano by virtue of Section 15 of
Co mmo nwe a lt h Ac t No. 473
( R e v i s e d Naturalization Act). Likewise, it argues
that Section 37 of the Naturalization Law
isunconstitutional for allowing the COI to issue warrant of
arrest and effecting deportationwithout judicial intervention
enshrined in the Constitution. CFI decided partly
againstpetitioners, thus, COI and petitioners both appealed to
SC.
Issues:

Whether or not the marriage of Chan Sau Wah to Esteban


Moranomakes her a Filipino citizen.


Whether or not Section 37 of the Naturalization Law
empowering theCOI to issue a warrant of arrest, and deport
upon a warrant on deportation casesis unconstitutional for
are covered by the Constitutional mandate on searchesand
seizures without judicial intervention required under the
Constitution onsearches and seizures.
Ruling:Citizenship.
Marriage to a Filipino citizen does not
ipso facto
make her a Filipinocitizen. She must show that she
possess all the qualifications, and none of
thedisqualifications required by the Naturalization Law
requiring as follows:

Valid marriage; and

Alien woman herself might be lawfully naturalizedIn the


additional stipulation of facts of July 3, 1963, petitioners
admit that Chan Sau Wahis not possessed of all the
qualifications required by the Naturalization Law. Thus,
shedid not become a Filipino citizen.
Searches and seizures.
Power to deport aliens is an attribute of sovereignty planted
onthe accepted maxim of international law, that every
sovereign nation has the power, asinherent in
sovereignty, and essential to self-preservation, to
forbid the entrance of foreigners within its dominions.
Section 1 (3), Article III of the Constitution, does notrequire
judicial intervention in the execution of a final
order of deportation issued inaccordance with law. The
constitutional limitation contemplates an order of arrest in
theexercise of judicial power as a step preliminary or
incidental to prosecution or proceedingfor a given offense or
administrative action, not as a measure indispensable to
carry out avalid decision by a competent official, such as a
legal order of deportation, issued by theCommissioner of
Immigration, in pursuance of a valid legislation.Petition
for mandamus and prohibition with respect to
petitioners Chan Sau Wah ishereby denied; and
judgment declaring her a citizen of the Philippines, directing
COI tocancel her Alien Certificate of Registration and other
immigration papers, and declaringthe preliminary injunction
with respect to her permanent, are all hereby set aside; and
inall other respects, the decision appealed is hereby affirmed.

G.R. No. L-22554 August 29, 1975


DELFIN LIM and JIKIL TAHA, plaintiffs-appellants,
vs.
FRANCISCO PONCE DE LEON AND ORLANDO
MADDELA, defendants-appellees.
Ricardo L. Manalilig for plaintiffs-appellants.
Iigo R. Pea for defendants-appellees.

On May 14, 1962, after conducting a preliminary


investigation, Fiscal Francisco Ponce de Leon in his capacity
as Acting Provincial Fiscal of Palawan, filed with the Court
of First Instance of Palawan the corresponding information
for Robbery the Force and Intimidation upon Persons against
Jikil Taha. The case was docketed as Criminal Case No.
2719.
On June 15, 1962, Fiscal Francisco Ponce de Leon, upon
being informed that the motor launch was in Balabac,
Palawan, wrote the Provincial Commander of Palawan
requesting him to direct the detachment commander-in
Balabac to impound and take custody of the motor launch. 1

On June 26, 1962, Fiscal Ponce de Leon reiterated his


request to the Provincial Commander to impound the motor
launch, explaining that its subsequent sale to a third party,
Appeal on a question of law from the decision of the Court
plaintiff-appellant Delfin Lim, cannot prevent the court from
of First Instance of Palawan in Civil Case No. 416, entitled
taking custody of the same. 2 So, on July 6, 1962 upon order
"Delfin Lim and Jikil Taha vs. Francisco Ponce de Leon and of the Provincial Commander, defendant-appellee Orlando
Orlando Maddela", dismissing the complaint of the plaintiffs Maddela, Detachment Commander of Balabac, Palawan,
and ordering them to pay each of the defendants jointly and seized the motor launch "SAN RAFAEL" from plaintiffseverally the sum of P500.00 by way of actual damages;
appellant Delfin Lim and impounded it.
P500.00 by way of attorney's fees; and P1,000.00 by way of
exemplary damages.
On July 15, 1962 plaintiff-appellant Delfin Lim pleaded with
Orlando Maddela to return the motor launch but the latter
On April 29, 1961, plaintiff-appellant Jikil Taha sold to a
refused. Likewise, on September 20, 1962, Jikil Taha
certain Alberto Timbangcaya of Brooke's Point, Palawan a
through his counsel made representations with Fiscal Ponce
motor launch named M/L "SAN RAFAEL". A year later or
de Leon to return the seized property to plaintiff-appellant
on April 9, 1962 Alberto Timbangcaya filed a complaint with Delfin Lim but Fiscal Ponce de Leon refused, on the ground
the Office of the Provincial Fiscal of Palawan alleging that
that the same was the subject of a criminal offense.
after the sale Jikil Taha forcibly took away the motor launch
from him.
All efforts to recover the motor launch going to naught,
plaintiffs-appellants Delfin Lim and Jikil Taha, on November
19, 1962, filed with the Court of First Instance of Palawan a
complaint for damages against defendants-appellees Fiscal
Francisco Ponce de Leon and Orlando Maddela, alleging that
on July 6, 1962 Orlando Maddela entered the premises of
Delfin Lim without a search warrant and then and there took
away the hull of the motor launch without his consent; that
MARTIN, J.:

he effected the seizure upon order of Fiscal Ponce de Leon


who knew fully well that his office was not vested with
authority to order the seizure of a private property; that said
motor launch was purchased by Delfin Lim from Jikil Taha
in consideration of Three Thousand Pesos (P3,000.00), Two
Thousand Pesos (P2,000.00) of which has been given to Jikil
Taha as advance payment; that as a consequence of the
unlawful seizure of the motor launch, its sale did not
materialize; and that since July 6, 1962, the said motor
launch had been moored at the Balabac Bay, Palawan and
because of exposure to the elements it had become worthless
and beyond repair. For the alleged violation of their
constitutional rights, plaintiffs-appellants prayed that
defendants-appellees be ordered to pay jointly and severally
each of them the sum of P5,750.00 representing actual,
moral and exemplary damages and attorney's fees.

On September 13, 1965, the trial court rendered its decision,


upholding the validity of the seizure of the motor launch on
the ground that "the authority to impound evidences or
exhibits or corpus delicti in a case pending investigation is
inherent in the Provincial Fiscal who controls the
prosecution and who introduces those exhibits in the court."
Accordingly, the trial court dismissed the complaint of
plaintiffs-appellants and ordered them to pay jointly and
severally each of the defendants-appellees the amount of
P500.00 by way of actual damages another amount of
P500.00 for attorney's fees and P1,000.00 as exemplary
damages.

under oath or affirmation of the complainant


and the witnesses he may produce, and
particularly describing the place to be
searched, and the persons or things to be
seized. 3

A cursory reading of the above provision easily brings into


focus the unreasonableness of the seizure of the
aforementioned motor launch. A search and seizure to be
reasonable, must be effected by means of a valid search
warrant. And for a search warrant to be valid: (1) it must be
issued upon probable cause; (2) the probable cause must be
determined by the judge himself and not by the applicant or
Hence, this appeal.
any other person; (3) in the determination of probable cause,
the judge must examine, under oath or affirmation, the
Two vital issues call for resolution by this Court. First,
complainant and such witnesses as the latter may produce;
whether or not defendant-appellee Fiscal Ponce de Leon had and (4) the warrant issued must particularly describe the
the power to order the seizure of the motor launch in
In their answer, defendants-appellees denied the material
place to be searched and persons or things to be seized. 4
question without a warrant of search and seizure even if the Thus in a long line of decisions, this Court has declared
allegations of the complaint and as affirmative defenses
alleged that the motor launch in question which was sold by same was admittedly the corpus delicti of the crime. Second, invalid search warrants which were issued in utter disregard
whether or not defendants-appellees are civilly liable to
Jikil Taha to Alberto Timbangcaya on April 29, 1961 was
of the constitutional injunction. 5
plaintiffs-appellants for damages allegedly suffered by them
sometime in April 1962, forcibly taken with violence upon
granting that the seizure of the motor launch was unlawful.
Defendants-appellees admitted that when Orlando Maddela
persons and with intent to gain by Jikil Taha from Alfredo
entered the premises of Delfin Lim and impounded the
Timbangcaya without the latter's knowledge and consent,
motor launch he was not armed with a search warrant; that
thus giving rise to the filing of a criminal charge of robbery The gravamen of plaintiffs-appellants' argument is that the
taking
of
the
motor
launch
on
July
6,
1962
by
Orlando
he effected the seizure of the motor launch in the absence of
against Jikil Taha; that Fiscal Ponce de Leon, in his capacity
Maddela upon the order of Fiscal Ponce de Loon was in
and without the consent of Delfin Lim. There can be no
as Acting Provincial Fiscal of Palawan ordered Orlando
violation of the constitutional guarantee against
question that without the proper search warrant, no public
Maddela to seize and impound the motor launch "SAN
unreasonable
searches
and
seizures
since
it
was
done
without
official has the right to enter the premises of another without
RAFAEL", for being the corpus delicti of the robbery; and
a warrant.
his consent for the purpose of search and seizure. 6 And since
that Orlando Maddela merely obeyed the orders of his
in the present case defendants-appellees seized the motor
superior officer to impound said launch. By way of
The pertinent provision of the Constitution then in force
launch without a warrant, they have violated the
counterclaim, defendants-appellees alleged that because of
reads:
constitutional right of plaintiffs-appellants against
the malicious and groundless filing of the complaint by
unreasonable search and seizure.
plaintiffs-appellants, they were constrained to engage the
3) The right of the people to be secure in
services of lawyers, each of them paying P500.00 as
their persons, houses, papers and effects
Defendants-appellees however would want to justify the
attorney's fees; and that they suffered moral damages in the
against
unreasonable
searches
and
seizures
seizure of the motor launch even without a warrant because
amount of P5,000.00 each and actual damages in the amount
shall not be violated, and no warrants shall
of Fiscal Ponce de Leon's alleged inherent power to order the
of P500.00 each. They also prayed that each of them
issue
but
upon
probable
cause,
to
be
seizure of a personal property which is the corpus delicti of a
awarded exemplary damages in the amount of P1,000.00.
determined by the judge after examination
crime, he being a quasi judicial officer who has the control

of the prosecution and the presentation of the evidence in the


criminal case. They argue that inasmuch as the motor launch
in question was allegedly stolen by Jikil Taha from
Timbangcaya, Fiscal Ponce de Leon could order its seizure
even without a search warrant. We cannot agree. Under the
old Constitution 7 the power to issue a search warrant is
vested in a judge or magistrate and in no other officer and no
search and seizure can be made without a proper warrant. At
the time the act complained of was committed, there was no
law or rule that recognized the authority of Provincial Fiscals
to issue a search warrant. In his vain attempt to justify the
seizure of the motor launch in question without a warrant
Fiscal Ponce de Leon invoked the provisions of Republic Act
No. 732, which amended Sections 1674 and 1687 of the
Revised Administrative Code. But there is nothing in said
law which confers upon the provincial fiscal; the authority to
issue warrants, much less to order without warrant the
seizure of a personal property even if it is the corpus delicti
of a crime. True, Republic Act No. 732 has broadened the
power of provincial fiscals to conduct preliminary
investigations, but said law did not divest the judge or
magistrate of its power to determine, before issuing the
corresponding warrant, whether or not probable cause exists
therefor. 8

indignity of a search for the evidence of


crime, without a legal warrant procured for
that purpose. No amount of incriminating
evidence whatever its source, will supply the
place of such warrant. At the closed door of
the home be it palace or hovel even
bloodhounds must wait till the law, by
authoritative process, bids it open.
(Emphasis supplied.)

following rights and liberties of another


person shall be liable to the latter for
damages.
xxx xxx xxx
(9) The rights to be secure in one's person,
house, papers, and effects against
unreasonable searches and seizures.

xxx xxx xxx


Defendant-appellee Fiscal Ponce de Leon would also invoke
lack of time to procure a search warrant as an excuse for the
The indemnity shall include moral damages.
seizure of the motor launch without one. He claimed that the
Exemplary damages may also be
motor launch had to be seized immediately in order to
adjudicated.
preserve it and to prevent its removal out of the locality,
since Balabac, Palawan, where the motor launch was at the
ART. 2219. Moral damages may be
time, could only be reached after three to four days' travel by
12
recovered in the following and analogous
boat. The claim cannot be sustained. The records show that
13
cases:
on June 15, 1962 Fiscal Ponce de Leon made the first
request to the Provincial Commander for the impounding of
xxx xxx xxx
the motor launch; and on June 26, 1962 14 another request
was made. The seizure was not effected until July 6, 1962. In
(6) Illegal search;
short, Fiscal Ponce de Leon had all the time to procure a
search warrant had he wanted to and which he could have
xxx xxx xxx
taken in less than a day, but he did not. Besides, there is no
Moreover, under Sections 2 and 3 of Rule 122 of the Rules
basis for the apprehension that the motor launch might be
(1) Acts and action referred to in Articles 21,
of Court 9 which complement the constitutional provision
moved out of Balabac because even prior to its seizure the
26, 27, 28, 29, 30, 32, 34 and 35.
earlier cited, two principles are made clear, namely: (1) that motor launch was already without its engine. 15 In sum, the
in the seizure of a stolen property search warrant is still
fact that there was no time to secure a search warrant would
Pursuant to the foregoing provisions, a person whose
necessary; and (2) that in issuing a search warrant the judge not legally justify a search without one. 16
constitutional rights have been violated or impaired is
alone determines whether or not there is a probable cause.
As to whether or not they are entitled to damages, plaintiffs- entitled to actual and moral damages from the public officer
The fact that a thing is a corpus delicti of a crime does not
justify its seizure without a warrant. As held in U.S. v. de los appellants anchor their claim for damages on Articles 32 and or employee responsible therefor. In addition, exemplary
damages may also be awarded. In the instant case, plaintiff2219 of the New Civil Code which provide in part as
Reyes and Esguerra, 10 citing McClurg v. Brenton: 11
appellant Delfin Lim claimed that he purchased the motor
follows:
launch from Jikil Taha in consideration of P3,000.00, having
The mere fact that a man is an officer,
ART. 32. Any public officer or employee, or given P2,000.00 as advanced payment; that since or seizure
whether of high or low degree, gives him no
on July 6, 1962 the motor launch had been moored at
any private individual, who directly or
more right than is possessed by the ordinary
Balabac Bay and because of exposure to the elements it has
indirectly obstructs, defeats, violates or in
private citizen to break in upon the privacy
become worthless at the time of the filing of the present
any manner impedes or impairs any of the
of a home and subject its occupant to the

action; that because of the illegality of the seizure of the


motor launch, he suffered moral damages in the sum of
P1,000.00; and that because of the violation of their
constitutional rights they were constrained to engage the
services of a lawyer whom they have paid P1,500.00 for
attorney's fees. We find these claims of Delfin Lim amply
supported by the evidence and therefore should be awarded
the sum of P3,000.00 as actual damages; P1,000.00 as moral
damages and P750.00 for attorney's fees. However, with
respect co plaintiff Jikil Taha, he is not entitled to recover
any damage which he alleged he had suffered from the
unlawful seizure of the motor launch inasmuch as he had
already transferred the ownership and possession of the
motor launch to Delfin Lim at the time it was seized and
therefore, he has no legal standing to question the validity of
the seizure. Well settled is the rule that the legality of a
seizure can be contested only by the party whose rights have
been impaired thereby, and that the objection to an unlawful
search and seizure is purely personal and cannot be availed
of by third parties. 17 Consequently, one who is not the
owner, lessee, or lawful occupant of the premise searched
cannot raise the question of validity of the search and
seizure. 18 Jikil Taha is not without recourse though. He can
still collect from his co-plaintiff, Delfin Lim the unpaid
balance of P1,000.00.
Defendant-appellee Fiscal Ponce de Leon wanted to wash
his hands of the incident by claiming that "he was in good
faith, without malice and without the slightest intention of
inflicting injury to plaintiff-appellant, Jikil Taha" 19 when he
ordered the seizure of the motor launch. We are not prepared
to sustain his defense of good faith. To be liable under
Article 32 of the New Civil Code it is enough that there was
a violation of the constitutional rights of the plaintiffs and it
is not required that defendants should have acted with malice
or bad faith. Dr. Jorge Bocobo, Chairman of the Code
Commission, gave the following reasons during the public
hearings of the Joint Senate and House Committees, why
good faith on the part of the public officer or employee is
immaterial. Thus:

DEAN BOCOBO. Article 32, regarding


article. Precisely, the opening object of the
individual rights; Attorney Cirilo Paredes
article is to put an end to abuses which are
proposes that Article 32 be so amended as to
justified by a plea of good faith, which is in
make a public official liable for violation of
most cases the plea of officials abusing
another person's constitutional rights only if
individual rights. 20
the public official acted maliciously or in
But defendant-appellee Orlando Maddela cannot be held
bad faith. The Code Commission opposes
accountable because he impounded the motor launch upon
this suggestion for these reasons:
the order of his superior officer. While a subordinate officer
The very nature of Article 32 is that the
may be held liable for executing unlawful orders of his
wrong may be civil or criminal. It is not
superior officer, there are certain circumstances which would
necessary therefore that there should be
warrant Maddela's exculpation from liability. The records
malice or bad faith. To make such a requisite show that after Fiscal Ponce de Leon made his first request
would defeat the main purpose of Article 32 to the Provincial Commander on June 15, 1962 Maddela was
which is the effective protection of
reluctant to impound the motor launch despite repeated
individual rights. Public officials in the past orders from his superior officer. 21 It was only after he was
have abused their powers on the pretext of
furnished a copy of the reply of Fiscal Ponce de Leon, dated
justifiable motives or good faith in the
June 26, 1962, to the letter of the Provincial Commander,
performance of their duties. Precisely, the
justifying the necessity of the seizure of the motor launch on
object of the Article is to put an end to
the ground that the subsequent sale of the launch to Delfin
official abuse by the plea of good faith. In
Lim could not prevent the court from taking custody of the
the United States this remedy is in he nature same, 22 that he impounded the motor launch on July 6, 1962.
of a tort.
With said letter coming from the legal officer of the
province, Maddela was led to believe that there was a legal
Mr. Chairman, this article is firmly one of
basis and authority to impound the launch. Then came the
the fundamental articles introduced in the
order of his superior officer to explain for the delay in the
New Civil Code to implement democracy.
seizure of the motor launch. 23 Faced with a possible
There is no real democracy if a public
disciplinary action from his Commander, Maddela was left
official is abusing, and we made the article
with no alternative but to seize the vessel. In the light of the
so strong and so comprehensive that it
above circumstances. We are not disposed to hold Maddela
concludes an abuse of individual rights even answerable for damages.
if done in good faith, that official is liable.
As a matter of fact, we know that there are
IN VIEW OF THE FOREGOING, the decision appealed
very few public officials who openly and
from is hereby reversed and another one entered declaring
definitely abuse the individual rights of the the seizure illegal and ordering defendant-appellee Fiscal
citizens. In most cases, the abuse is justified Francisco Ponce de Leon to pay to plaintiff-appellant Delfin
on a plea of desire to enforce the law to
Lim the sum of P3,000.00 as actual damages, plus P1,000.00
comply with one's duty. And so, if we
moral damages, and, in addition, P750.00 for attorney's fees.
should limit the scope of this article, that
With costs against defendant-appellee Fiscal Ponce de Leon.
would practically nullify the object of the

SO ORDERED.
SALAZAR VS. ACHACOSO [183 SCRA 145; G.R. NO.
81510; 14 MAR 1990]
Wednesday, February 04, 2009 Posted by Coffeeholic Writes
Labels: Case Digests, Political Law
Facts: Rosalie Tesoro of Pasay City in a sworn statement
filed with the POEA, charged petitioner with illegal
recruitment. Public respondent Atty. Ferdinand Marquez sent
petitioner a telegram directing him to appear to the POEA
regarding the complaint against him. On the same day, after
knowing that petitioner had no license to operate a
recruitment agency, public respondent Administrator Tomas
Achacoso issued a Closure and Seizure Order No. 1205 to
petitioner. It stated that there will a seizure of the documents
and paraphernalia being used or intended to be used as the
means of committing illegal recruitment, it having verified
that petitioner has (1) No valid license or authority from
the Department of Labor and Employment to recruit and
deploy workers for overseas employment; (2) Committed/are
committing acts prohibited under Article 34 of the New
Labor Code in relation to Article 38 of the same code. A
team was then tasked to implement the said Order. The
group, accompanied by mediamen and Mandaluyong
policemen, went to petitioners residence. They served the
order to a certain Mrs. For a Salazar, who let them in. The
team confiscated assorted costumes. Petitioner filed with
POEA a letter requesting for the return of the seized
properties, because she was not given prior notice and
hearing. The said Order violated due process. She also
alleged that it violated sec 2 of the Bill of Rights, and the
properties were confiscated against her will and were done
with
unreasonable
force
and
intimidation.
Issue: Whether or Not the Philippine Overseas Employment
Administration (or the Secretary of Labor) can validly issue
warrants of search and seizure (or arrest) under Article 38 of
the
Labor
Code
Held: Under the new Constitution, . . . no search warrant or
warrant of arrest shall issue except upon probable cause to

be determined personally by the judge after examination


under oath or affirmation of the complainant and the
witnesses he may produce, and particularly describing the
place to be searched and the persons or things to be seized.
Mayors and prosecuting officers cannot issue warrants of
seizure or arrest. The Closure and Seizure Order was based
on Article 38 of the Labor Code. The Supreme Court held,
We reiterate that the Secretary of Labor, not being a judge,
may no longer issue search or arrest warrants. Hence, the
authorities must go through the judicial process. To that
extent, we declare Article 38, paragraph (c), of the Labor
Code, unconstitutional and of no force and effect The
power of the President to order the arrest of aliens for
deportation is, obviously, exceptional. It (the power to order
arrests) cannot be made to extend to other cases, like the one
at bar. Under the Constitution, it is the sole domain of the
courts. Furthermore, the search and seizure order was in the
nature of a general warrant. The court held that the warrant is
null and void, because it must identify specifically the things
to
be
seized.
WHEREFORE, the petition is GRANTED. Article 38,
paragraph (c) of the Labor Code is declared
UNCONSTITUTIONAL and null and void. The respondents
are ORDERED to return all materials seized as a result of
the implementation of Search and Seizure Order No. 1205.
Salazar v. Achacoso, 183 SCRA 145 F: Pursuant to the powers vested by
PD 1920 and EO 1022, POEA Administrator Achacoso ordered the
closure of the recruitment agency of Horty Salazar, having verified that
she hadno license to operate a recruitment agency. He further ordered the
seizure of the documents and paraphernalias,being used or intended to be
used as the means of commiting illegal recruitment. This order was
enforced on 26January 1988. Petitioner filed this suit for prohibition.
Issue: May the POEA (or the Sec. of Labor) validly issuewarrants of
serach and seizure (or arrest ) under Art. 38 of the Labor Code?HELD:
NO.The provisions of PD 1920and EO 1022, now embodied in Art. 38
of the Labor Code, are the dying vestiges of authoritarian rule in its
twilightsmoments. Under Art. III, Sec 2 of the 1987 Constitution, it is
only judges and no other, who may issue warrants of arrest and search.
The exception is in cases of deportation of illegal and undesirable aliens,
whom the President of the Commissioner of Immigration may order
arrested, following a final order of deportation, for the purpose
of deportation. The Sec. of Labor , not being a judge. may no longer
issue search or arrest warrants. Hence, theauthorities must go through the
judicial process. To that extent, we declare Art. 38, par. C of the Labor

Code,unconstitutional and of no force and effect.&n bsp; a. Existence of


probable cause. Probable cause is such facts andcircumstances as would
reasonably make a prudent man believe that a crime have been
committed and that thedocuments or things sought to be searched and
seized are in the possession of the person against whom the warrantis
sought. Without probable cause, there can be no valid search warrant.
See Pasionvda. de Garcia v. Locsin, 65Phil. 689, (1938)
G.R. No. 83578 March 16, 1989
THE PRESIDENTIAL ANTI-DOLLAR SALTING
TASK FORCE, petitioner,
vs.
HONORABLE COURT OF APPEALS, HONORABLE
TEOFILO L, GUADIZ, JR.,Presiding Judge,
REGIONAL TRIAL COURT, Branch 147: NCR
(MAKATI), and KARAMFIL IMPORT-EXPORT CO.,
INC., respondents.
K. V. Faylona & Associates for respondents.

SARMIENTO, J.:
The petitioner, the Presidential Anti-Dollar Salting Task
Force, the President's arm assigned to investigate and
prosecute so-called "dollar salting" activities in the country
(per Presidential Decree No. 1936 as amended by
Presidential Decree No. 2002), asks the Court to hold as null
and void two Resolutions of the Court of Appeals, dated
September 24, 1987 1 and May 20, 1988, 2 reversing its
Decision, dated October 24, 1986. 3 The Decision set aside
an Order, dated April 16, 1985, of the Regional Trial Court, 4
as well as its Order, dated August 21, 1985. The Resolution,
dated September 24, 1987 disposed of, and granted, the
private respondent Karamfil Import-Export Co., Inc.'s
motion for reconsideration of the October 24, 1986 Decision;
the Resolution dated May 20, 1988, in turn, denied the
petitioner's own motion for reconsideration.
The facts are not in controversy. We quote:

On March 12, 1985, State Prosecutor Jose B.


1) Competency of this Court to act on
Rosales, who is assigned with the
petition filed by the petitioners;
Presidential Anti-Dollar Salting Task Force
2) Validity of the search warrants issued by
hereinafter referred to as PADS Task Force
respondent State Prosecutor;
for purposes of convenience, issued search
warrants Nos. 156, 157, 158, 159, 160 and
3) Whether or not the petition has become
161 against the petitioners Karamfil Importmoot and academic because all the search
Export Co., Inc., P & B Enterprises Co.,
warrants sought to be quashed had already
Inc., Philippine Veterans Corporation,
been implemented and executed. 8
Philippine Veterans Development
Corporation, Philippine Construction
Development Corporation, Philippine Lauan On April 16, 1985, the lower court issued the first of its
challenged Orders, and held:
Industries Corporation, Inter-trade
Development (Alvin Aquino), Amelili U.
WHEREFORE, in view of all the foregoing,
Malaquiok Enterprises and Jaime P. Lucman
the Court hereby declares Search Warrant
Enterprises.
Nos. 156, 157, 158, 159, 160, and 161 to be
null and void. Accordingly, the respondents
The application for the issuance of said search warrants was
are hereby ordered to return and surrender
filed by Atty. Napoleon Gatmaytan of the Bureau of Customs
immediately all the personal properties and
who is a deputized member of the PADS Task Force.
documents seized by them from the
Attached to the said application is the affidavit of Josefin M.
petitioners by virtue of the aforementioned
Castro who is an operative and investigator of the PADS
search warrants.
Task Force. Said Josefin M. Castro is likewise the sole
deponent in the purported deposition to support the
SO ORDERED. 9
application for the issuance of the six (6) search warrants
involved in this case. The application filed by Atty.
Gatmaytan, the affidavit and deposition of Josefin M. Castro On August 21, 1985, the trial court denied reconsideration.
are all dated March 12, 1985. 5

On April 4, 1986, the Presidential Anti-Dollar Salting Task


Force went to the respondent Court of Appeals to contest, on
certiorari, the twin Order(s) of the lower court.

Shortly thereafter, the private respondent (the petitioner


below) went to the Regional Trial Court on a petition to
enjoin the implementation of the search warrants in question.
In ruling initially for the Task Force, the Appellate Court
6 On March 13, 1985, the trial court issued a temporary
held:
restraining order [effective "for a period of five (5) days
notice " 7 ] and set the case for hearing on March 18, 1985.
Herein petitioner is a special quasi-judicial
body with express powers enumerated under
In disposing of the petition, the said court found the material
PD 1936 to prosecute foreign exchange
issues to be:
violations defined and punished under P.D.
No. 1883.

The petitioner, in exercising its quasijudicial powers, ranks with the Regional
Trial Courts, and the latter in the case at bar
had no jurisdiction to declare the search
warrants in question null and void.
Besides as correctly pointed out by the
Assistant Solicitor General the decision of
the Presidential Anti-Dollar Salting Task
Force is appealable to the Office of the
President.10
On November 12, 1986, Karamfil Import-Export Co., Inc.
sought a reconsideration, on the question primarily of
whether or not the Presidential Anti-Dollar Salting Task
Force is "such other responsible officer' countenanced by the
1973 Constitution to issue warrants of search and seizure.
As we have indicated, the Court of Appeals, on Karamfil's
motion, reversed itself and issued its Resolution, dated
September 1987, and subsequently, its Resolution, dated
May 20, 1988, denying the petitioner's motion for
reconsideration.
In its petition to this Court, the petitioner alleges that in so
issuing the Resolution(s) above-mentioned, the respondent
Court of Appeals "committed grave abuse of discretion
and/or acted in excess of its appellate jurisdiction," 11
specifically:
a) In deviating from the settled policy and
rulings of the Supreme Court that no
Regional Trial Courts may countermand or
restrain the enforcement of lawful writs or
decrees issued by a quasi-judicial body of
equal and coordinate rank, like the PADS
Task Force;
b) For resorting to judicial legislation to
arrive at its erroneous basis for

reconsidering its previous Decision dated


October 24, 1986 (see Annex "I") and thus
promulgated the questioned Resolutions
(Annexes "A" and "B"), which violated the
constitutional doctrine on separation of
powers;

said presidential body be said to be "such other responsible


officer as may be authorized by law" to issue search warrants
under the 1973 Constitution questions we take up
seriatim.**

jurisdiction. An examination of the Presidential Anti-Dollar


Salting Task Force's petition shows indeed its recognition of
judicial review (of the acts of Government) as a basic
privilege of the courts. Its objection, precisely, is whether it
is the Regional Trial Court, or the superior courts, that may
undertake such a review.

In submitting that it is a quasi-judicial entity, the petitioner


states that it is endowed with "express powers and functions
c) In not resolving directly the other
under PD No. 1936, to prosecute foreign exchange violations Under the Judiciary Reorganization Act of 1980, 17 the
important issues raised by the petitioner in
as defined and punished under PD No. 1883." 13 "By the
Court of Appeals exercises:
its Petition in CA-G.R. No. 08622-SP
very nature of its express powers as conferred by the laws,"
(3) Exclusive appellate jurisdiction over all
despite the fact that petitioner has
so it is contended, "which are decidedly quasi-judicial or
final judgments, decisions, resolutions,
demonstrated sufficiently and convincingly discretionary function, such as to conduct preliminary
orders or awards of Regional Trial Court and
that respondent RTC, in issuing the
investigation on the charges of foreign exchange violations,
quasi-judicial agencies, instrumentalities,
questioned Orders in Special Proceeding No. issue search warrants or warrants of arrest, hold departure
boards or commissions, except those falling
M-624 (see Annexes "C" and 'D"),
orders, among others, and depending upon the evidence
within the appellate jurisdiction of the
committed grave abuse of discretion and/or presented, to dismiss the charges or to file the corresponding
Supreme Court in accordance with the
acted in excess of jurisdiction:
information in court of Executive Order No. 934, PD No.
Constitution, the provisions of this Act, and
1936 and its Implementing Rules and Regulations effective
1. In ruling that (a) the description of the
of subparagraph (1) of the third paragraph
August 26, 1984), petitioner exercises quasi-judicial power
things to be seized as stated in the contested or the power of adjudication ." 14
and subparagraph (4) of the fourth paragraph
search warrant were too general which
of Section 17 of the Judiciary Act of 1948.
allegedly render the search warrants null and The Court of Appeals, in its Resolution now assailed, 15 was
18
void; (b) the applications for the contested
of the opinion that "[t]he grant of quasi-judicial powers to
search warrants actually charged two
petitioner did not diminish the regular courts' judicial power xxx xxx xxx
offenses in contravention of the 2nd
of interpretation. The right to interpret a law and, if
Under the present Constitution, with respect to its provisions
paragraph, Section 3, Rule 126 of the Rules necessary to declare one unconstitutional, exclusively
of Court; and (c) this case has not become
pertains to the judiciary. In assuming this function, courts do on Constitutional Commissions, it is provided, in part that:
moot and academic, even if the contested
not proceed on the theory that the judiciary is superior to the
... Unless otherwise provided by this
search warrants had already been fully
two other coordinate branches of the government, but solely
Constitution or by law, any decision, order,
implemented with positive results; and
on the theory that they are required to declare the law in
or ruling of each Commission may be
every case which come before them." 16
brought to the Supreme Court on certiorari
2. In ruling that the petitioner PADS Task
by the aggrieved party within thirty days
Force has not been granted under PD 1936
This Court finds the Appellate Court to be in error, since
from receipt of a copy thereof. 19
'judicial or quasi-judicial jurisdiction. 12
what the petitioner puts to question is the Regional Trial
Court's act of assuming jurisdiction over the private
We find, upon the foregoing facts, that the essential
respondent's petition below and its subsequent countermand On the other hand, Regional Trial Courts have exclusive
questions that confront us are- (i) is the Presidential Antiof the Presidential Anti-Dollar Salting Task Force's orders of original jurisdiction:
Dollar Salting Task Force a quasi-judicial body, and one co- search and seizure, for the reason that the presidential body,
(6) In all cases not within the exclusive
equal in rank and standing with the Regional Trial Court, and as an entity (allegedly) coordinate and co-equal with the
jurisdiction of any court, tribunal, person or
accordingly, beyond the latter's jurisdiction; and (ii) may the Regional Trial Court, was (is) not vested with such a

body exercising judicial or quasi-judicial


functions. 20
xxx xxx xxx
Likewise:
... The Supreme Court may designate certain
branches of the Regional Trial Court to
handle exclusively criminal cases, juvenile
and domestic relations cases, agrarian case,
urban land reform cases which do not fall
under the jurisdiction of quasi- judicial
bodies and agencies and/or such other
special cases as the Supreme Court may
determine in the interest of a speedy and
efficient administration of justice. 21
xxx xxx xxx

two, also known as the "Minimum Wage


Law"; the Department of Labor under
Section 23 of Republic Act Numbered Eight
hundred seventy-five, also known as the
"Industrial Peace Act"; the Land
Registration Commission; the Securities and
Exchange Commission; the Social Security
Commission; the Civil Aeronautics Board;
the Patent Office and the Agricultural
Inventions Board, may appeal therefrom to
the Court of Appeals, within the period and
in the manner herein provided, whether the
appeal involves questions of fact, mixed
questions of fact and law, or questions of
law, or all three kinds of questions. From
final judgments or decisions of the Court of
Appeals, the aggrieved party may appeal by
certiorari to the Supreme Court as provided
in Rule 45 of the Rules of Court. 24

Under our Resolution dated January 11, 1983: 22

Because of subsequent amendments, including the abolition


of various special courts, 25 jurisdiction over quasi-judicial
... The appeals to the Intermediate Appellate bodies has to be, consequently, determined by the
Court [now, Court of Appeals] from quasicorresponding amendatory statutes. Under the Labor Code,
judicial bodies shall continue to be governed decisions and awards of the National Labor Relations
by the provisions of Republic Act No. 5434 Commission are final and executory, but, nevertheless,
insofar as the same is not inconsistent with
'reviewable by this Court through a petition for certiorari and
the provisions of B.P. Blg. 129. 23
not by way of appeal." 26

The pertinent provisions of Republic Act No. 5434 are as


follows:
SECTION 1. Appeals from specified
agencies. Any provision of existing law or
Rule of Court to the contrary
notwithstanding, parties aggrieved by a final
ruling, award, order, decision, or judgment
of the Court of Agrarian Relations; the
Secretary of Labor under Section 7 of
Republic Act Numbered Six hundred and

Under the Property Registration Decree, decisions of the


Commission of Land Registration, en consults, are
appealable to the Court of Appeals. 27
The decisions of the Securities and Exchange Commission
are likewise appealable to the Appellate Court, 28 and so are
decisions of the Social Security Commission.29
As a rule, where legislation provides for an appeal from
decisions of certain administrative bodies to the Court of
Appeals, it means that such bodies are co-equal with the

Regional Trial Courts, in terms of rank and stature, and


logically, beyond the control of the latter.
As we have observed, the question is whether or not the
Presidential Anti-Dollar Salting Task Force is, in the first
place, a quasi-judicial body, and one whose decisions may
not be challenged before the regular courts, other than the
higher tribunals the Court of Appeals and this Court.
A quasi-judicial body has been defined as "an organ of
government other than a court and other than a legislature,
which affects the rights of private parties through either
adjudication or rule making." 30 The most common types of
such bodies have been listed as follows:
(1) Agencies created to function in situations
wherein the government is offering some
gratuity, grant, or special privilege, like the
defunct Philippine Veterans Board, Board on
Pensions for Veterans, and NARRA, and
Philippine Veterans Administration.
(2) Agencies set up to function in situations
wherein the government is seeking to carry
on certain government functions, like the
Bureau of Immigration, the Bureau of
Internal Revenue, the Board of Special
Inquiry and Board of Commissioners, the
Civil Service Commission, the Central Bank
of the Philippines.
(3) Agencies set up to function in situations
wherein the government is performing some
business service for the public, like the
Bureau of Posts, the Postal Savings Bank,
Metropolitan Waterworks & Sewerage
Authority, Philippine National Railways, the
Civil Aeronautics Administration.

(4) Agencies set up to function in situations


wherein the government is seeking to
regulate business affected with public
interest, like the Fiber Inspections Board, the
Philippine Patent Office, Office of the
Insurance Commissioner.
(5) Agencies set up to function in situations
wherein the government is seeking under the
police power to regulate private business
and individuals, like the Securities &
Exchange Commission, Board of Food
Inspectors, the Board of Review for Moving
Pictures, and the Professional Regulation
Commission.
(6) Agencies set up to function in situations
wherein the government is seeking to adjust
individual controversies because of some
strong social policy involved, such as the
National Labor Relations Commission, the
Court of Agrarian Relations, the Regional
Offices of the Ministry of Labor, the Social
Security Commission, Bureau of Labor
Standards, Women and Minors Bureau. 31
As may be seen, it is the basic function of these bodies to
adjudicate claims and/or to determine rights, and unless its
decision are seasonably appealed to the proper reviewing
authorities, the same attain finality and become executory. A
perusal of the Presidential Anti-Dollar Salting Task Force's
organic act, Presidential Decree No. 1936, as amended by
Presidential Decree No. 2002, convinces the Court that the
Task Force was not meant to exercise quasi-judicial
functions, that is, to try and decide claims and execute its
judgments. As the President's arm called upon to combat the
vice of "dollar salting" or the blackmarketing and salting of
foreign exchange, 32 it is tasked alone by the Decree to
xxx xxx xxx
handle the prosecution of such activities, but nothing more.
We quote:

SECTION 1. Powers of the Presidential


Anti-Dollar Salting Task Force.-The
Presidential Anti-Dollar Salting Task Force,
hereinafter referred to as Task Force, shall
have the following powers and authority:
a) Motu proprio or upon complaint, to
investigate and prosecute all dollar salting
activities, including the overvaluation of
imports and the undervaluation of exports;
b) To administer oaths, summon persons or
issue subpoenas requiring the attendance
and testimony of witnesses or the production
of such books, papers, contracts, records,
statements of accounts, agreements, and
other as may be necessary in the conduct of
investigation;
c) To appoint or designate experts,
consultants, state prosecutors or fiscals,
investigators and hearing officers to assist
the Task Force in the discharge of its duties
and responsibilities; gather data, information
or documents; conduct hearings, receive
evidence, both oral and documentary, in all
cases involving violation of foreign
exchange laws or regulations; and submit
reports containing findings and
recommendations for consideration of
appropriate authorities;
d) To punish direct and indirect contempts
with the appropriate penalties therefor under
Rule 71 of the Rules of Court; and to adopt
such measures and take such actions as may
be necessary to implement this Decree.

f. After due investigation but prior to the


filing of the appropriate criminal charges
with the fiscal's office or the courts as the
case may be, to impose a fine and/or
administrative sanctions as the
circumstances warrant, upon any person
found committing or to have committed acts
constituting blackmarketing or salting
abroad of foreign exchange, provided said
person voluntarily admits the facts and
circumstances constituting the offense and
presents proof that the foreign exchange
retained abroad has already been brought
into the country.
Thereafter, no further civil or criminal action
may be instituted against said person before
any other judicial regulatory or
administrative body for violation of
Presidential Decree No. 1883.
The amount of the fine shall be determined
by the Chairman of the Presidential AntiDollar Salting Task Force and paid in Pesos
taking into consideration the amount of
foreign exchange retained abroad, the
exchange rate differentials, uncollected taxes
and duties thereon, undeclared profits,
interest rates and such other relevant factors.
The fine shall be paid to the Task Force
which shall retain Twenty percent (20 %)
thereof. The informer, if any, shall be
entitled to Twenty percent (20 %) of the
fine. Should there be no informer, the Task
Force shall be entitle to retain Forty percent
(40 %) of the fine and the balance shall
accrue to the general funds of the National
government. The amount of the fine to be
retained by the Task Force shall form part of

its Confidential Fund and be utilized for the


operations of the Task Force . 33

It will not do to say that the fact that the Presidential Task
Force has been empowered to issue warrants of arrest,
search, and seizure, makes it, ergo, a "semi-court". Precisely,
The Court sees nothing in the aforequoted provisions (except it is the objection interposed by the private respondent,
with respect to the Task Force's powers to issue search
whether or not it can under the 1973 Charter, issue such
warrants) that will reveal a legislative intendment to confer it kinds of processes.
with quasi-judicial responsibilities relative to offenses
punished by Presidential Decree No. 1883. Its undertaking,
It must be observed that under the present Constitution, the
as we said, is simply, to determine whether or not probable
powers of arrest and search are exclusive upon judges. 35 To
cause exists to warrant the filing of charges with the proper
that extent, the case has become moot and academic.
court, meaning to say, to conduct an inquiry preliminary to a Nevertheless, since the question has been specifically put to
judicial recourse, and to recommend action "of appropriate
the Court, we find it unavoidable to resolve it as the final
authorities". It is not unlike a fiscal's office that conducts a
arbiter of legal controversies, pursuant to the provisions of
preliminary investigation to determine whether or not prima the 1973 Constitution during whose regime the case was
facie evidence exists to justify haling the respondent to
commenced.
court, and yet, while it makes that determination, it cannot be
Since the 1973 Constitution took force and effect and until it
said to be acting as a quasi-court. For it is the courts,
ultimately, that pass judgment on the accused, not the fiscal. was so unceremoniously discarded in 1986, its provisions
conferring the power to issue arrest and search warrants
It is not unlike the Presidential Commission on Good
upon an officer, other than a judge, by fiat of legislation have
Government either, the executive body appointed to
been at best controversial. In Lim v. Ponce de Leon, 36 a
investigate and prosecute cases involving "ill-gotten wealth". 1975 decision, this Court ruled that a fiscal has no authority
It had been vested with enormous powers, like the issuance
to issue search warrants, but held in the same vein that, by
of writs of sequestration, freeze orders, and similar
virtue of the responsible officer" clause of the 1973 Bill of
processes, but that did not, on account thereof alone, make it Rights, "any lawful officer authorized by law can issue a
a quasi-judicial entity as defined by recognized authorities. It search warrant or warrant of arrest.37 Authorities, however,
cannot pronounce judgement of the accused's culpability, the have continued to express reservations whether or not fiscals
jurisdiction to do which is exclusive upon the
may, by statute, be given such a power. 38
Sandiganbayan. 34
Less than a year later, we promulgated Collector of Customs
If the Presidential Anti-Dollar Salting Task Force is not,
v. Villaluz, 39 in which we categorically averred: Until now
hence, a quasi-judicial body, it cannot be said to be co-equal only the judge can issue the warrant of arrest." 40 "No law or
or coordinate with the Regional Trial Court. There is nothing presidential decree has been enacted or promulgated vesting
in its enabling statutes that would demonstrate its standing at the same authority in a particular responsible officer ." 41
par with the said court.
Apparently, Villaluz had settled the debate, but the same
In that respect, we do not find error in the respondent Court question persisted following this Courts subsequent rulings
of Appeal's resolution sustaining the assumption of
upholding the President's alleged emergency arrest powers .
jurisdiction by the court a quo.
42 [Mr. Justice Hugo Gutierrez would hold, however, that a
Presidential Commitment Order (PCO) is (was) not a species

of "arrest" in its technical sense, and that the (deposed) Chief


Executive, in issuing one, does not do so in his capacity as a
"responsible officer" under the 1973 Charter, but rather, as
Commander-in-Chief of the Armed Forces in times of
emergency, or in order to carry out the deportation of
undesirable aliens.43 In the distinguished Justice's opinion
then, these are acts that can be done without need of judicial
intervention because they are not, precisely, judicial but
Presidential actions.]
In Ponsica v. Ignalaga,44 however, we held that the mayor
has been made a "responsible officer' by the Local
Government Code, 45 but had ceased to be one with the
approval of the 1987 Constitution according judges sole
authority to issue arrest and search warrants. But in the same
breath, we did not rule the grant under the Code
unconstitutional based on the provisions of the former
Constitution. We were agreed, though, that the "responsible
officer" referred to by the fundamental law should be one
capable of approximating "the cold neutrality of an impartial
judge." 46
In striking down Presidential Decree No. 1936 the
respondent Court relied on American jurisprudence, notably,
Katz v. United States, 47 Johnson v. United States, 48 and
Coolidge v. New Hampshire 49 in which the American
Supreme Court ruled that prosecutors (like the petitioner)
cannot be given such powers because of their incapacity for
a "detached scrutiny" 50 of the cases before them. We affirm
the Appellate Court.
We agree that the Presidential Anti-Dollar Salting Task Force
exercises, or was meant to exercise, prosecutorial powers,
and on that ground, it cannot be said to be a neutral and
detached "judge" to determine the existence of probable
cause for purposes of arrest or search. Unlike a magistrate, a
prosecutor is naturally interested in the success of his case.
Although his office "is to see that justice is done and not
necessarily to secure the conviction of the person accused,"
51 he stands, invariably, as the accused's adversary and his

accuser. To permit him to issue search warrants and indeed,


warrants of arrest, is to make him both judge and jury in his
own right, when he is neither. That makes, to our mind and
to that extent, Presidential Decree No. 1936 as amended by
Presidential Decree No. 2002, unconstitutional.

The Court joins the Government in its campaign against the


scourge of "dollar- salting", a pernicious practice that has
substantially drained the nation's coffers and has seriously
threatened its economy. We recognize the menace it has
posed (and continues to pose) unto the very stability of the
country, the urgency for tough measures designed to contain
if not eradicate it, and foremost, the need for cooperation
from the citizenry in an all-out campaign. But while we
support the State's efforts, we do so not at the expense of
fundamental rights and liberties and constitutional
safeguards against arbitrary and unreasonable acts of
Government. If in the event that as a result of this ruling, we
prove to be an "obstacle" to the vital endeavour of stamping
out the blackmarketing of valuable foreign exchange, we do
not relish it and certainly, do not mean it. The Constitution
simply does not leave us much choice.

Facts:Sometime July of
1
99
1
, the narcotics division of theWPD conducted a by bust operation in
Zamora andPandacan Streets in Manila after the confirmation of
drugpushing reports in the same area.During the operation in the area,
It is our ruling, thus, that when the 1973 Constitution spoke
after the policeofficers saw the accused selling something to a buyer,
of "responsible officer" to whom the authority to issue arrest
theyapproached Espano and frisked him. The officers seized twoplastic
and search warrants may be delegated by legislation, it did
tea bags of marijuana from the accused. They thenlater asked the accused
if there are more marijuana in him,the Espano replied that there are more
not furnish the legislator with the license to give that
in his house. Thus,the arresting officers went to Espanos house
authority to whomsoever it pleased. It is to be noted that the
andsubsequently confiscated ten more plastic tea bags
Charter itself makes the qualification that the officer himself
of marijuana.Espano was brought to the police, and charged inviolation
must be "responsible". We are not saying, of course, that the
with RA 6425. The RTC Br.
Presidential Anti-Dollar Salting Task Force (or any similar
1
prosecutor) is or has been irresponsible in discharging its
of Manila convictedhim in the charges whish was later affirmed
duty. Rather, we take "responsibility", as used by the
in toto
Constitution, to mean not only skill and competence but
by therespondent appellate court. Hence, this petition for review. As
defense, the petitioner contends that the drugsconfiscated are
more significantly, neutrality and independence comparable WHEREFORE, the petition is DISMISSED. No costs. SO
inadmissible evidence against him beingsearch and seized
to the impartiality presumed of a judicial officer. A
ORDERED.
illegally.Issue:Was the contention correct?Held:Partly, the drugs
prosecutor can in no manner be said to be possessed of the
confiscated in his person isadmissible, however, the articles sought in his
latter qualities.
residencecannot be admitted as evidence by the prosecution
beingunlawfully seized.Rule
According to the Court of Appeals, the implied exclusion of Rule 126, Sec. 2. Personal property to be seized.
11
prosecutors under the 1973 Constitution was founded on the
3 Section 5(a) of the Rules of Court providesthat an officer may without
A search warrant may be issued for the search and
a warrant arrest a person when inhis presence:(a) Has committed(b) Is
requirements of due process, notably, the assurance to the
seizure
of
personal
property:
actually committing(c) Or is about to commit a crimeIn the case at bar,
respondent of an unbiased inquiry of the charges against him
(a) Subject of the offense;
petitioner's arrest falls squarely under the aforecited rule. He was caught
prior to the arrest of his person or seizure of his property. We
in
add that the exclusion is also demanded by the principle of
(b) Stolen or embezzled and other proceeds or fruits of
flagranti
separation of powers on which our republican structure rests. the offense; and
as a result of abuy-bust operation conducted by police officers on the
Prosecutors exercise essentially an executive function (the
basisof information received regarding the illegal trade of drugswithin the
petitioner itself is chaired by the Minister, now Secretary, of (c) Use or intended to be used as the means of committing area of Zamora and Pandacan Streets, Manila.The police officer saw
an offense.
Trade and Industry), since under the Constitution, the
petitioner handing over something toan alleged buyer. After the buyer
left, they searched him anddiscovered two cellophanes of marijuana. His
President has pledged to execute the laws. 52 As such, they
arrest was,therefore, lawful and the two cellophane bags of
cannot be made to issue judicial processes without
marijuanaseized were admissible in evidence, being the fruits of
E
unlawfully impinging the prerogative of the courts.
thecrime. As for the ten cellophane bags of marijuana found
spano vs.
atpetitioner's residence, however, the same inadmissiblein evidence.The
C
At any rate, Ponsica v. Ignalaga should foreclose all
1
ourtof
questions on the matter, although the Court hopes that this
9
A
disposition has clarified a controversy that had generated
8
ppeals
often bitter debates and bickerings.

7 Constitution guarantees freedom againstunreasonable searches and


seizures under Article III,Section 2 which provides:
The right of the people to be secure in their persons,houses, papers and
effects against unreasonablesearches and seizures of whatever nature and
for anypurposes shall be inviolable, and no search warrant or warrant of
arrest shall issue except upon probable causeto be determined personally
by the judge after examination under oath or affirmation of the
complainantand the witnesses he may produce, and
particularlydescribing the place to be searched and the persons or things
to be seized." An exception to the said rule is a warrantless
searchincidental to a lawful arrest for dangerous weapons or anything,
which may be used as proof of the commission of an offense. It may
extend beyond the person of the onearrested to include the premises or
surroundings under hisimmediate control. In this case, the ten cellophane
bags of marijuana seized at petitioner's house after his arrest atPandacan
and Zamora Streets do not fall under the saidexceptions.The articles
seized from petitioner during his arrestwere valid under the doctrine of
search made incidental to alawful arrest. The warrantless search made in
his house,however, which yielded ten cellophane bags of
marijuanabecame unlawful since the police officers were not armedwith
a search warrant at the time. Moreover, it was beyondthe reach and
control of petitioner.

Sec. 5. Arrest without warrant, when lawful. A peace


officer or private person may, without warrant, arrest a
person:
(a)
When in his presence, the person to be
arrested has committed, is actually committing, or is
attempting to commit an offense;
(b)
When an offense has in fact just been
committed, and he has personal knowledge of facts
indicating that the person to be arrested has committed it x x
x
It is the considered view of the Court that there was no need
for Luciano and Caraan to be armed with a warrant of
arrest when they arrested Marquez and the accused since
they had personal knowledge of the actual commission of
the crime viz: They were eyewitnesses to the illegal
exchange of marijuana and P190.00 between Marquez
and accused who were caught in flagrante delicto. The
facts and circumstances attendant precisely fall under Sec. 5,
(a), Rule 113 of the Rules on Criminal Procedure. The
subsequent arrest of Marquez and accused were made under
the principle of "hot pursuit". The recovery of the marijuana
from Marquez and the P190.00 from accused by the said
People vs. Bati (G.R. No. 87429, August 27, 1990)
police officers were not violative of their constitutional
Facts: By the word of their civilian informer, Patrolmen Jose rights since Marquez and the accused voluntarily
Luciano, Angelito Caraan, Nelson Dimatulac and Democrito surrendered them to the police officers. But even for the sake
Cuenca immediately proceeded to the vicinity an alleged
of argument that the recovery of the marijuana and peso bills
buy-and-sell of marijuana was taking place. They saw
were against the consent of Marquez and accused, still, the
Marquez giving something to Bati, who, thereafter, handed a search on their persons was incidental to their valid
wrapped object, which turned out to be marijuana worth
warrantless arrest. For, the rule that searches and seizures
P190, to Marquez who then inserted the object inside the
must be supported by a valid warrant is not an absolute
front of his pants in front of his abdomen while Bati, on his rule. There are at least three exceptions: (1) search
part, placed the thing given to him inside his pocket.
incidental to an arrest, (2) search of a moving vehicle and
Marquez was arrested on the spot. Both Bati and Marquez
(3) seizure of evidence in plain view. In the case at bar, the
were brought to the Police station where they admitted they searches made on Marquez and accused were incidental to
were in the buying and selling of the confiscated marijuana. their valid arrest.
Issue: Appellant contends that the arrest was not valid as the
requirements for a warrantless arrest were not complied
with.
Held: This contention is without merit.
Section 5 Rule 113 of the Rules in Criminal Procedure
clearly provides:

Having caught the appellant in flagrante as a result of the


buy-bust operation, the policemen were not only authorized
but were also under obligation to apprehend the drug pusher
even without a warrant of arrest And since appellant's arrest
was lawful, it follows that the search made incidental to the
arrest was also valid.
Supreme Court Poetry: The law is severe because those
who are caught in the strangle hold of prohibited drugs not

only slide into the ranks of the living dead, what is worse,
they become a grave menace to the safety of the law-abiding
members of society.

Josefina Garcia-Padilla vs Minister of Defense Juan


Ponce Enrile et al
Reversal of the Lansang Doctrine & Reinstatement of the
Montenegro Doctrine
In July 1982, Sabino Padilla, together w/ 8 others who were
having a conference in a house in Bayombong, NV, were
arrested by members of the PC. The raid of the house was
authorized by a search warrant issued by Judge Sayo.
Josefina, mother of Sabino, opposed the arrest averring that
no warrant of arrest was issued but rather it was just a
warrant of arrest hence the arrest of her son and the others
was w/o just cause. Sabino and companions together with 4
others were later transferred to a facility only the PCs know.
Josefina petitioned the court for the issuance of the writ of
habeas corpus.
ISSUE: Whether or not the arrests done against Sabino et al
is valid.
HELD: In a complete about face, the SC decision in the
Lansang Case was reversed and the ruling in the Barcelon
Case & the Montenegro Case was again reinstated. The
questioned power of the president to suspend the privilege of
the WoHC was once again held as discretionary in the
president. The SC again reiterated that the suspension of the
writ was a political question to be resolved solely by the
president. It was also noted that the suspension of the
privilege of the writ of habeas corpus must, indeed, carry
with it the suspension of the right to bail, if the governments
campaign to suppress the rebellion is to be enhanced and
rendered effective. If the right to bail may be demanded
during the continuance of the rebellion, and those arrested,
captured and detained in the course thereof will be released,

they would, without the least doubt, rejoin their comrades in


the field thereby jeopardizing the success of government
efforts to bring to an end the invasion, rebellion or
insurrection.

NOTE: This ruling was abrogated by Sec 18, Art 7 of the


1987 Constitution which expressly constitutionalized the
Lansang Doctrine. Note as well that under Art 3 (Sec 13) of
the Constitution it is stated that the right to bail shall not be
impaired even if the privilege of the writ of habeas corpus is
suspended.
GARCIA PADILLA VS ENRILE

Facts:
The records show that nine (9) of the fourteen (14)
detainees herein were arrested on July 6, 1982 at about 1:45
p.m. when three (3) teams of the PC/INP of Bayombong,
Nueva after securing a Search Warrant conducted a raid at
the residence of Dra. Aurora Parong. Apprehended during
the said raid were Dra. Aurora Parong, Benjamin Pineda,
Sabino Padilla, Francisco Divinagracia, Zenaida Mallari,
Letty Ballogan, Norberto Portuguese, and Mariano Soriano
who were then having a conference in the dining room of
Dra. Parong's residence which had been doing on since
10:00 a.m. of that same day.
The other four (4) detainees herein, namely: Imelda de los
Santos, Eufronio Ortiz, Jr., Juanita Granada, and Bienvenida
Garcia, were arrested on the following day, July 7, 1982 by
the same PC teams. On July 15, 1982, Tom Vasquez was
arrested, and his Volkswagen car, bearing Plate No. DAP
347, was seized by the PC authorities.
The herein fourteen (14) detainees were all detained at the

PC/INP Command Headquarters, Bayombong, Nueva


Vizcaya from July 6, 1982 until their transfer on the morning
of August 10, 1982 to an undisclosed place reportedly to
Camp Crame, Quezon City, to Echague, Isabela, and to
Tuguegarao, Cagayan.

forcible overthrow of the government and duty constituted


authorities. " LOI 1211 does not, in any manner, limit the
authority of the President to cause the arrest and detention of
persons engaged in, or charged with the crimes or offenses
mentioned in said Proclamation in that he (President) would
subject himself to the superior authority of the judge who,
Hence, this petition for the writ of habeas corpus and
under normal judicial processes in the prosecution of the
mandamus filed by Josefina Garcia-Padilla, mother of
common offenses, is the one authorized to issue a judicial
detained petitioner Sabino G. Padilla, Jr. on August 13, 1982. warrant after a preliminary investigation is conducted with a
The mandamus aspect of the instant petition has, however,
finding of probable cause. Those who would read such an
become moot and academic, and whereabouts of petitioners intention on the part of the President in issuing LOI 1211
having already become known to petitioner Josefina Garcia- seems to do so in their view that LOI forms part of the law
Padilla.
of the land under the 1976 amendment of the Constitution.
They would then contend that a PCO issued not in
compliance with the provisions of the LOI would be an
illegality and of no effect.
Ruling:
To form part of the law of the land, the decree, order or LOI
The last question relates to the legality of the Presidential
must be issued by the President in the exercise of his
Commitment Order (PCO) issued by the President on July
extraordinary power of legislation as contemplated in
12, 1982, tested by the conformity of its issuance to the
Section 6 of the 1976 amendments to the Constitution,
procedure laid down under LOI 1211, petitioners insisting
whenever in his judgment, there exists a grave emergency or
that the LOI limits the authority of the President to cause the a threat or imminence thereof, or whenever the interim
arrest and detention of persons engaged in or charged with,
Batasan Pambansa or the regular National Assembly fails or
the crimes mentioned in Proclamation No. 2045. They
is unable to act adequately on any matter for any reason that
contend that the procedure prescribed in the LOI not having in his judgment requires immediate action. There can be no
been observed, the PCO issued thereunder did not validate
pretense, much less a showing, that these conditions
the initial illegal arrest of the herein petitioners as wen as
prompted the President to issue LOI 1211. Verily, not all LOI
their continued detention.
issued by the President should be dignified into forming part
of the law of the land.
By its very nature, and clearly by its language, LOI 1211 is a
People vs. Maspil,
mere directive of the President as Commander-in-Chief of
the police had also been tipped off by an informant thatt h e
the Armed Forces of the Philippines to his subordinates or
implementing officers for the ultimate objective of providing a c c u s e d M aspil and Basking would be transporting
guidelines in the arrest and detention of the persons covered a large volume
of marijuana to Baguio City. The operatives intercepted a
by Presidential Proclamation No. 2045. The purpose is "to
Sarao type jeep driven by Maspil andBagking as his
insure protection to individual liberties without sacrificing
companion. Upon inspection, the jeep was found
the requirements of public order and safety and the
loaded with two plasticbags., which when opened
effectiveness of the campaign against those seeking the
contained several bundles of suspected dried marijuana
leaves.Maspil and Basking were arrested and the

suspected marijuana leaves were confiscated. The


Supreme Court held that this case falls squarely to the
exceptions of requiring a searchwarrant. It also falls to the
exceptions enumerated in Rule 113, Sec. 5 which state:Sec.
5.
Arrest without warrant; when lawful.
A

The Solicitor General for plaintiff-appellee.


Peter C. Fianza for defendants-appellants.

GUTIERREZ, JR., J.:

That on or about the 1st day of November,


1986, at Sayangan, Municipality of Atok,
Province of Benguet, Philippines, and within
the jurisdiction of this Honorable Court, the
above-named accused, conspiring,
confederating and mutually aiding each
other, and without any authority of law, did
then and there willfully, unlawfully and
knowingly transport and carry in transit
from Sinto, Bauko, Mt. Province to Atok,
Benguet One Hundred Eleven Kilos and
Nine Grams (111.9 kilos), more or less, of
dried marijuana leaves which are sources of
dangerous and prohibited drugs and from
which dangerous and prohibited drugs may
be derived and manufactured, in violation of
the said law. (Rollo, p. 11)

peace officer or a private person may, withouta warrant,


This petition is an appeal from the decision of the Regional
arrest a person:(a) When, in his presence, the person to be
arrested has committed, is actually committing,or is
Trial Court of Baguio City, Branch 5, the dispositive portion
attempting to commit an offense.It is because according to
of which reads:
the Court, the appellants (Maspil and Basking) were caught
inflagrante delicto since they were transporting the
WHEREFORE, the Court finds and declares
prohibited drugs at the time of the arrest.(Emphasis
the accused MOISES MASPIL, JR. y
Supplied)There are also several cases
WAYWAY and SALCEDO BAGKING y
such People vs Ramos, Amelia Roque vs de Villa,
ALTAKI, guilty beyond reasonable doubt of
Anonuevovs Ramos and People vs Tangliben,
the crime of illegal transportation of
where in the court rulings did not strictly observed therule on
personal knowledge of facts constituting a probable cause
marijuana as charged and hereby sentences
under Rule 113, Sec. 5 (a)and (b). The fact that the search
EACH of them to suffer LIFE
The narration of facts by the trial court is as follows:
yielded possession of illegal articles was included as
IMPRISONMENT; to pay a fine of
a justification for a warrantless arrest under Section 5(a)
P20,000.00, without subsidiary
According to Jerry Valeroso, Sgt. Amador
although the arresting officer at thetime of arrest has no
imprisonment in case of insolvency; and to
Ablang and Sgt. Florentino Baillo, all
personal knowledge of a crime being committed.In the case
pay
their
proportionate
shares
in
the
costs.
members of the First Narcotics Regional
at hand, the Makati Anti-illegal Drugs Unit caught the
respondents in flagrantedelicto and that is delivering or
Unit of the Narcotics Command stationed in
transporting the prohibited drugs at the time of the arrest.
The confiscated marijuana (Exhibits "B",
Baguio City, (See also Exhibit "I") on
Theitems confiscated were inside the white crosswind,
"B-1 " to "B-23"; "C" to "C-16", "D", "D-1"
October 30, 1986, they established a
Respondent 1 was driving whererespondent 2 is his
to "D-20"; "E", "E-1", to "E-14", "F", "F-1",
checkpoint in front of the Municipal Hall at
companion. The fact that the respondents were caught in
"G", "G-l") are hereby declared forfeited in
Sayangan, Atok, Benguet, which is along the
flagrante delicto is already a sufficient ground for the Makati
favor
of
the
Government
and
upon
the
Halsema Highway, to check on vehicles
Anti-Illegal Drug Unit to arrest them.And thus, there is a
finality of this decision, the Branch Clerk of
proceeding to Baguio City because their
probable cause for the Makati Anti-illegal Drugs Unit to
Court
is
directed
to
turn
over
the
same
to
the
arrest therespondents.
Commanding Officer, Maj. Basilio Cablayan
Dangerous Drugs Board (NBI), through the
had been earlier tipped off by some
Chief, PC Crime Laboratory, Regional Unit
confidential informers that the herein
G.R. No. 85177 August 20, 1990
No. 1, Camp Dangwa, La Trinidad, Benguet,
accused Maspil and Basking would be
for disposition in accordance ' with law.
transporting a large volume of marijuana to
PEOPLE OF THE PHILIPPINES, plaintiff-appellee,
(Rollo, pp. 25-26)
Baguio City. The informers went along with
vs.
the operatives to Sayangan.
MOISES MASPIL, JR. y WAYWAY and SALCEDO
In Criminal Case No. 4263-R, the information filed against
BAGKING y ALTAKI, defendants-appellants.
the two accused alleged:
At about 2:00 o'clock in the early morning
of November 1, the operatives intercepted a

Sarao type jeep driven by Maspil with


Bagking as his companion. Upon inspection,
the jeep was found loaded with two (2)
plastic sacks (Exhibits "B" and "D"), one (1)
jute sack (Exhibit "C") and three (3) big
round tin cans (Exhibits "E", "F" and "G")
which, when opened contained several
bundles of suspected dried marijuana leaves
(Exhibits "B-1", to "B-23", "C-1" to "C-16",
D-1" to "D-20", "E-1" to "E-14","F-1" and
"G-1").
Maspil and Basking were arrested and the
suspected marijuana leaves were
confiscated.
The confiscated items were later on referred
to the PC Crime Laboratory, Regional Unit
1, for examination (Exhibit "A"). Forensic
Chemist Carlos V. Figuerroa performed the
requested examination and determined that
the specimen, with an aggregate weight of
115.66 kilos, were positive to the standard
tests for marijuana.
The accused admitted that the marijuana
dried leaves were indeed confiscated from
the jeep being then driven by Maspil with
Bagking as his helper. However, they
claimed that the prohibited drugs belonged
to two of their passengers who loaded them
in the jeep as paying cargo for Baguio City
without the accused knowing that they were
marijuana.
The accused declared that on October 31,
1986, at the burned area along Lakandula
Street, Baguio City, a certain Mrs. Luisa
Mendoza hired the jeep of Maspil to
transport her stock of dried fish and canned

goods contained in cartons to Abatan,


Buguias, Benguet, because her own vehicle
broke down. They left Baguio City at about
1:00 o'clock in the afternoon (11:30 in the
morning, according to Bagking) with Mrs.
Mendoza, her helper and salesgirls on board
the jeep with Maspil as driver and Bagking
as his own helper. They arrived at Abatan at
about 6:00 o'clock in the evening.
After unloading their cargo, Maspil and
Bagking repaired to a restaurant for their
dinner before undertaking the trip back to
Baguio City. While thus eating, they were
approached by two persons, one of whom
they would learn later on to be a certain
Danny Buteng. Buteng inquired if they were
going to Baguio City and upon being given
an affirmative answer, he said that he would
ride with them and that he has some cargo.
Asked what the cargo was, Buteng replied
that they were flowers in closed tin cans and
sealed sacks for the commemoration of All
Souls Day in Baguio City. After Buteng had
agreed to Maspil's condition that he would
pay for the space to be occupied by his
cargo, Buteng himself and his companion
loaded the cargo and fixed them inside
Maspil's jeep.
Maspil and Bagking left Abatan at about
7:00 o'clock that same evening of October
31. Aside from Buteng and companion they
had four other passengers. These four other
passengers alighted at Natubleng, Buguias,
Benguet.
Upon reaching Sayangan, Atok, Benguet,
Maspil stopped at the Marosan Restaurant
where they intended to take coffee. Their

remaining passengers-Buteng and


companion alighted and went to the
restaurant. However, a soldier waved at
Maspil to drive to where he was, which
Maspil did. The soldier secured Maspil's
permission to inspect their cargo after which
he grabbed Maspil on the latter's left
shoulder and asked who owned the cargo.
Maspil told the soldier that the cargo
belonged to their passengers who went to
the restaurant. The soldier called for his
companions and they went to look for
Maspil's passengers in the restaurant. Later
on, they returned and placed Maspil and
Bagking under arrest since their cargo
turned out to be marijuana.
Lawrence Balonglong, alias Banawe, a radio
reporter of DZWX Bombo Radio who was
invited by Lt Valeroso to witness the
operation, affirmed the unsuccessful pursuit
of the alleged two companions of Maspil
and Bagking. He recalled that he was
awakened from his sleep at the townhall in
Sayangan after the arrest of Maspil and
Bagking. When he went to the scene, the
NARCOM operatives boarded the jeep of
Maspil to chase the two companions of
Maspil and Bagking. Balonglong climbed on
top of the jeep with his camera to join the
chase. They proceeded toward the direction
of Bontoc but failed to catch anyone. Hence,
they returned.
Thereupon, Maspil and Bagking were taken
to the townhall where they were allegedly
maltreated to admit ownership of the
confiscated marijuana. At about 4:00 o'clock
in the afternoon of November 1, the soldiers
took them away from Sayangan to be

transferred to their station at Baguio City.


On their way, particularly at Km. 32 or 34,
they met Mike Maspil, an elder brother of
Moises Maspil, and the soldiers called for
him and then Lt. Valeroso and his men
mauled him on the road.
Mike testified that between 3:00 and 4:00
o'clock in the afternoon of November 1, he
was informed by a neighbor that his brother
Moises was detained at the Atok Municipal
Jail. So he called for Jose Pos-el and James
Longages, his driver and helper,
respectively, to go along with him to see
Moises. They rode in his jeep. On the way,
they met the group of Lt. Valeroso. For no
apparent reason, Lt. Valeroso boxed and
kicked him several times. Thereafter, Lt.
Valeroso placed him under arrest together
with his driver and helper. They were all
brought to a shoe store on Gen. Luna Road,
Baguio City, together with Moises and
Bagking. There, Lt. Valeroso got his wallet
containing P21000 and Seiko wrist watch
but the receipt (Exhibit "3") was issued by a
certain Miss Pingil, a companion of
Valeroso. He was released after nine days.
He then went to Lt. Valeroso to claim his
wallet, money and watch but he was told
that they were with Miss Pingil. However,
when he went to Miss Pingil, the latter said
that the items were with Lt. Valeroso. He
sought the assistance of then Tourism
Deputy Minister Honorato Aquino who
assigned a lawyer to assist him. The lawyer
advised him to file a case against Lt.
Valeroso but because of the intervening
congressional elections, the matter has never
been pursued. (Rollo, p. 21-24)

The appellants raise the following assignment of errors in


their appeal, to wit:
I
THAT THE TRIAL COURT ERRED IN
NOT FINDING THAT THE ALLEGED
MARIJUANA AS CHARGED IN THE
INFORMATION IS DIFFERENT FROM
THAT PRESENTED FOR LABORATORY
EXAMINATION.
II
THAT THE TRIAL COURT ERRED IN
FINDING THAT THERE WERE ONLY
TWO OCCUPANTS, THE APPELLANTS,
IN THE VEHICLE WHERE THE
ALLEGED MARIJUANA WAS
CONFISCATED
III.
THAT THE TRIAL COURT ERRED IN
FINDING THAT THE ACCUSED KNEW
THAT THE CARGO THEY WERE
TRANSPORTING WAS MARIJUANA.
IV
THAT THE TRIAL COURT ERRED IN
ADMITTING IN EVIDENCE THE
ALLEGED CONFISCATED
MARIJUANA.
V
THAT THE TRIAL COURT ERRED IN
SHIPPING FROM THE PROSECUTION
THE BURDEN OF PROVING THE

COMMISSION OF THE OFFENSE


CHARGED TO THE APPELLANTS TO
PROVE THEIR INNOCENCE." (Rollo, p.
40)
The main defense of the appellants is their claim that the
prohibited drugs belonged to their two passengers who
loaded them in the jeep as paying cargo without the
appellants knowing that the cargo was marijuana.
In the second and third assignment of errors, the appellants
claim that the trial court erred in not appreciating their
version of the facts.
The appellants state that the trial court's reliance on Sgt.
Baillo's testimony that they were the only ones in the jeep
cannot be given credence as Sgt. Baillo's testimony is full of
inconsistencies.
The appellants cite Sgt. Baillo's inconsistencies as to the
time of the arrest whether morning or afternoon, the time the
checkpoint was removed and the person who were with him
at the time of arrest.
It has been ruled that inconsistencies in the testimonies of
the prosecution witnesses not on material points is not fatal.
Moreover, minor inconsistencies are to be expected but must
be disregarded if they do not affect the basic credibility of
the evidence as a whole. (People v. Marcos, G. R. No.
83325, May 8, 1990).
The defense even state that there were l lot of policemen
(T.S.N., December 1, 1987, p. 22) and it was but natural that
there would be confusion on who was there at the time of the
arrest.

The trial court gave credence to the positive and categorical


statement of Sgt. Baillo that there were only two occupants,
and these were the appellants inside the jeepney at the time
(T.S.N., June 30, 1987, p. 18). We see no cogent reason to
reverse this finding of fact.

11:00, 1guess, p.m., they


awakened me so I went and
I saw these two guys being
apprehended by the Narcom
operative.

In their brief, the appellants even admit that "he


(Balonglong) did not see the passengers" and it was just his
impression that there were other people present. (Appellant's
Brief, p. 7)
The appellants maintain that they did not know what was in
the cargo. Their main concern was in going back to Baguio
City and they saw no need to question their two passengers
on why flowers were being kept in closed cans and sacks.
They were apprehended after midnight. They traversed a
lonely and reputedly dangerous portion of the mountain
highway.

There is nothing in the records to suggest that the arrest was


motivated by any reason other than the desire of the police
officers to accomplish their mission. Courts generally give
full faith and credit to police officers when the facts and
circumstances surrounding their acts sustain the presumption
that they have performed their duties in a regular manner.
(Rule 131, Section 5 (m), Rules of Court; People v. Marcos,
supra; People v. Yap and Mendoza, G.R. Nos. 87088-89,
May 9,1990)

Court:

The appellants put forward the testimony of Lawrence


Balonglong which corroborates and affirms their stand that
there were, indeed, passengers in the jeepney.

A Already apprehended.

However, a close perusal of said testimony reveals no such


corroboration. The pertinent portions of Balonglong's
testimony is as follows:

Q And when you saw these


persons, what did you do, if It is likewise incredible that the appellants did not show the
any?
slightest curiousity as to why flowers were being kept in
closed tin cans and sealed sacks and cellophane. On the other
A What I recall is that when hand, the appellants had clear knowledge that Luisa
I went to the road, where
Mendoza was transporting cartons containing dried fish and
these two guys were
canned goods on the trip out of Baguio. It is contrary to
apprehended, the operatives human experience that the appellants would inquire about
boarded the same jeep and I the name of the passenger and the cargo she was loading on
even climbed the jeep . . .
their jeep and not doing the same about another who would
on top of the jeep holding
transport goods on a midnight trip.
my camera and tape
recorder and we . . . I don't Well-settled is the rule that evidence to be believed, must not
know ... they chased,
only proceed from the mouth of a credible witness but it
according to the operatives, must be credible itself. No better test has yet been found to
they chased two
measure the value of a witness than its conformity to the
companions of the two
knowledge and common experience of mankind. (People v.
arrested guys. (T.S.N., May Maribung, 149 SCRA 292, 297 [1987] ; People v. Aldana,
11, 1988, p. 4)
G.R. No. 81817, July 27, 1989; People v. Pascua, G.R. No.
82303, December 21, 1989).

xxx xxx xxx


Q Where were you when
these two persons were
apprehended?
A I was in the Municipal
Hall asleep, sir.
Q How did you know then
that these people were
apprehended?
A It is like this, sir, on the
night of October 31, 1 was
then asleep and at around

Q You saw them being


apprehended?
A No, sir ... I saw them
there.
Q Already apprehended?

Atty. Fianza:

The appellants' version is not believable. It is inconceivable


that the appellants would not even bother to ask the names of
the strangers who approached them in a restaurant at night
wanting to hire their jeepney, considering that they were
familiar with the Identity of the passenger, Luisa Mendoza,
who hired them to transport her goods to Abatan, Buguias,
Benguet.

The appellants further allege that if, indeed they knew about
the contents of their cargo, they would have adopted means
to prevent detection or to evade arrest.

Q Front seat of what?

At the time the appellants were being motioned by the


policemen to come nearer the checkpoint, there was no way
that the appellants could have evaded the arrest without
putting their lives in jeopardy. They decided to just brazen it
out with police and insist on their version of the story.

Q And did you ask or see


what was inside the jeep?

A The jeep, sir.

Another ground stated by the appellants for the


inadmissibility in evidence of the confiscated marijuana is
that the marijuana allegedly seized from them was a product
of an unlawful search without a warrant.
In the case of Valmonte v. de Villa, G.R. No. 83988,
September 29, 1989, the Court held that:

A Yes.
xxx xxx xxx
Q And what were those?

As for the other assigned errors, the appellants in the first


assigned error, contend that since there is a discrepancy of
3.76 between the number of kilos stated in the information
(111.9 kilos) and in the report (115.66 kilos) of the forensic
chemist, it is very likely that the marijuana presented as
evidence was not the one confiscated from the appellants or
even if they were the same, it could have already been
tampered with. The appellants conclude that the marijuana
then, cannot be admitted as evidence.
The marijuana examined by the forensic chemist, which was
contained in three big round tin cans, two jute sacks (there
was really only one jute sack colored light green which was
confiscated but since one of the plastic sacks [green]
appeared to be tattered, some of its contents were transferred
to a white jute sack), (T.S.N., June 23, 1987, p. 5) and two
plastic bags colored yellow and green (T.S.N., June 23,
1987, p. 3), was positively Identified to be the same as those
confiscated from the appellants. This is very clear from the
testimony of Lt. Valeroso who stated:
xxx xxx xxx
Q When you went down,
where were these two
suspects, as you said?
A They were sitted (sic) at
the front seat.

A It was all suspected


marijuana dried leaves
contained in three big cans,
one sack colored green, two
sacks colored yellow and
green. (Italics supplied,
T.S.N., September 16, 1987,
P. 4)
Lt. Valeroso further testified that Exhibits "B" (yellow
plastic big), "C" light green jute sack, "D" (green plastic
bag), "E" (one big can), "F" (second can), "G" (third can)
were, indeed, the same articles which he saw at the back of
the jeepney of the appellants. (T.S.N., September 16, 1987,
p. 5)

True, the manning of checkpoints by the


military is susceptible of abuse by the men
in uniform, in the same manner that all
government power is susceptible of abuse.
But at the cost of occasional inconvenience,
discomfort and even irritation to the citizen,
the checkpoints during these abnormal
times, when conducted within reasonable
limits are part of the price we pay for an
orderly society and a peaceful community.

The search was conducted within reasonable limits. There


was information that a sizeable volume of marijuana will be
transported to take advantage of the All Saints Day holiday
wherein there will be a lot of people going to and from
Baguio City (T.S.N., September 16, 1987, p. 6). In fact,
during the three day (October 30, 1986 to November 1,
One of the appellants, Moises Maspil, even admitted that the 1986) duration of the checkpoint, there were also other drug
articles Identified by Lt. Valeroso in his testimony were
related arrests made aside from that of the two appellants.
indeed, the same articles confiscated from their jeepney at
Sayangan, Atok, Benguet. (T.S.N., February 24, 1988, pp.
But even without the Valmonte ruling, the search would still
34-35)
be valid. This case involves a search incident to a lawful
arrest which is one of the exceptions to the general rule
Moreover, the words "more or less" following the weight in requiring a search warrant. This exception is embodied in
kilos of the marijuana in the questioned information declare Section 12 of Rule 126 of the 1985 Rules on Criminal
that the number of kilos stated therein is just an
Procedure which provides:
approximation. It can therefore be a little lighter or heavier.
The slight discrepancy is not material.
Sec. 12. Search incident to lawful arrest. A
person lawfully arrested may be searched for
dangerous weapons or anything which may

be used as proof of the commission of an


offense, without a search warrant.

the arrival. A jeepney cannot be equated with a passenger


ship on the high seas. The ruling in the Aminnudin case, is
not applicable to the case at bar.

and Rule 113, Section 5 (11) which state:


As for the fifth and last assigned error we agree with the
Sec. 5. Arrest without warrant; when lawful. Solicitor General that:
A peace officer or a private person may,
without a warrant, arrest a person:
Examination of the testimonies of appellants
show that they admit the fact that the
(a) When, in his presence, the person to be
confiscated marijuana was taken from their
arrested has committed, is actually
jeep while they were transporting it from
committing, or is attempting to commit an
Abatan, Buguias, Benguet to Baguio City.
offense.
This being so, the burden of the prosecution
to prove illegal transportation of prohibited
This case falls squarely within the exceptions. The appellants
drugs punished under Section 4 of RA 6425,
were caught in flagrante delicto since they were transporting
as amended, has been satisfactorily
the prohibited drugs at the time of their arrest. (People v.
discharged. The rule in civil as well as in
Tangliben, G.R. No. 63630, April 6, 1990) A crime was
criminal cases is that each party must prove
actually being committed.
his own affirmative allegations. The
prosecution avers the guilt of the accused
The appellants, however, cite the case of People V.
who is presumed to be innocent until the
Aminnudin, (163 SCRA 402 [1988]. In said case, the PC
contrary is proved. Therefore, the
officers received information that the accused-appellant, on
prosecution must prove such guilt by
board a vessel bound for Iloilo City, was carrying marijuana.
establishing the existence of all the elements
When the accused-appellant was descending the gangplank,
of the crime charged. But facts judicially
the PC officers detained him and inspected the bag that he
known, presumed, admitted or confessed
was carrying and found marijuana. The Court ruled that
need not be proved. (Rule 129, Sec. 4, Rules
since the marijuana was seized illegally, it is inadmissible in
on Evidence) (Appellee's Brief, p. 26-27)
evidence.
WHEREFORE, the guilt of the appellants having been
There are certain facts of the said case which are not present proved beyond reasonable doubt, the appealed decision is
in the case before us. In the Aminnudin case, the records
hereby AFFIRMED.
showed that there was sufficient time and adequate
information for the PC officers to have obtained a warrant.
SO ORDERED.
The officers knew the name of the accused, that the accused
was on board M/V Wilcon 9, bound to Iloilo and the exact
date of the arrival of the said vessel.
On the other hand, in this case there was no information as to
the exact description of the vehicle and no definite time of

POSADAS VS. COURT OF APPEALS [188 SCRA 288;


G.R. NO. 89139; 2 AUG 1990]
Friday, February 06, 2009 Posted by Coffeeholic Writes
Labels: Case Digests, Political Law
Facts: Members of the Integrated National Police (INP) of
the Davao Metrodiscom assigned with the Intelligence Task
Force, Pat. Ursicio Ungab and Pat. Umbra Umpar conducted
surveillance along Magallanes Street, Davao City. While in
the vicinity of Rizal Memorial Colleges they spotted
petitioner carrying a "buri" bag and they noticed him to be
acting suspiciously. They approached the petitioner and
identified themselves as members of the INP. Petitioner
attempted to flee but his attempt to get away was
unsuccessful. They then checked the "buri" bag of the
petitioner where they found one (1) caliber .38 Smith &
Wesson revolver with Serial No. 770196, two (2) rounds of
live ammunition for a .38 caliber gun, a smoke (tear gas)
grenade, and two (2) live ammunitions for a .22 caliber gun.
They brought the petitioner to the police station for further
investigation. In the course of the same, the petitioner was
asked to show the necessary license or authority to possess
firearms and ammunitions found in his possession but he
failed to do so. He was then taken to the Davao Metrodiscom
office and the prohibited articles recovered from him were
indorsed to M/Sgt. Didoy the officer then on duty. He was
prosecuted for illegal possession of firearms and
ammunitions in the Regional Trial Court of Davao City.
Issue: Whether or Not the warantless search is valid.
Held: In justifying the warrantless search of the buri bag
then carried by the petitioner, argues that under Section 12,
Rule 136 of the Rules of Court a person lawfully arrested
may be searched for dangerous weapons or anything used as
proof of a commission of an offense without a search
warrant. It is further alleged that the arrest without a warrant
of the petitioner was lawful under the circumstances.
in the case at bar, there is no question that, indeed, it is
reasonable considering that it was effected on the basis of a
probable cause. The probable cause is that when the
petitioner acted suspiciously and attempted to flee with the

buri bag there was a probable cause that he was concealing Criminal Case 87-54417of the Regional Trial Court (RTC)
something illegal in the bag and it was the right and duty of of Manila with violation of Section 4, Art. II, in relation
the
police
officers
to
inspect
the
same. toSection 21, Article IV of Republic Act 6425, as amended.
The court, on 15 March 1988, foundDela Cruz and Beltran
It is too much indeed to require the police officers to search guilty beyond reasonable doubt and sentenced each of them
the bag in the possession of the petitioner only after they to suffer the penalty of reclusion perpetua, with the
shall have obtained a search warrant for the purpose. Such an accessory penalties provided by law; to pay a fine
exercise may prove to be useless, futile and much too late. of P20,000.00, without subsidiary imprisonment in case of
insolvency, and each to pay one-half of the costs. From this
Clearly, the search in the case at bar can be sustained under decision, de la Cruz and Beltran appealed. In a letter of the
the exceptions heretofore discussed, and hence, the Warden, ManilaCity Jail, dated 3 March 1989, the Court was
constitutional guarantee against unreasonable searches and informed of the death of de la Cruz on 21 February1989.
seizures has not been violated.
Thus, the criminal case against de la Cruz was dismissed in
the Supreme Court resolutionof 25 September 1989. The
present appellate proceeding is limited only to Beltran.
Issue:
Whether the warrantless seizure incidental to the buy-bust
operation violates Beltransconstitutional rights against
People vs. dela Cruz [GR 83260, 18 April 1990]
unreasonable search and seizure.
Second Division, Regalado (J): 4 concur
Held:
Facts:
A buy-bust operation is the method employed by peace
After receiving a confidential report from Arnel, their
officers to trap and catch amalefactor in flagrante delicto. It
informant, a buy-bust operationwas conducted by the 13th
is essentially a form of entrapment since the peace
Narcotics Regional Unit through a team composed of T/Sgt.
officer neither instigates nor induces the accused to commit a
JaimeRaposas as Team Leader, S/Sgt. Rodelito Oblice, Sgt.
crime. Entrapment is the employment of such ways and
Dante Yang, Sgt. Vicente Jimenez, P/Pfc.Adolfo Arcoy as
means for the purpose of trapping or capturing a lawbreaker
poseur-buyer and Pat. Deogracias Gorgonia at Maliclic St.,
from whose mind thecriminal intent originated. Oftentimes,
Tondo, Manila ataround 2:30 p.m. of 4 May 1987 to catch
it is the only effective way of apprehending a criminal inthe
the pusher/s. P/Pfc. Adolfo Arcoy acted as the poseur- buyer
act of the commission of the offense. While it is conceded
with Arnel as his companion to buy marijuana worth P10.00
that in a buy-bust operation, thereis seizure of evidence from
from the two accused, Juande la Cruz and Reynaldo Beltran.
ones person without a search warrant, needless to state a
At the scene, it was Juan de la Cruz whom Arcoy
searchwarrant is not necessary, the search being incident to a
firstnegotiated with on the purchase and when Arcoy told De
lawful arrest. A peace officer may,without a warrant, arrest a
la Cruz that he was buying P10.00worth of marijuana, De la
person when, in his presence, the person to be arrested has
Cruz instructed Reynaldo Beltran to give one aluminum foil
committed,is actually committing or is attempting to commit
of marijuana which Beltran got from his pants pocket and
an offense. It is a matter of judicial experiencethat in the
delivered it to Arcoy. After ascertainingthat the foil of
arrest of violators of the Dangerous Drugs Act in a buy-bust
suspected marijuana was really marijuana, Arcoy gave the
operation, the malefactorswere invariably caught redprearranged signal tohis teammates by scratching his head
handed. There being no violation of the constitutional right
and his teammates who were strategically positioned in
againstunreasonable search and seizure, the confiscated
thevicinity, converged at the place, identified themselves as
articles are admissible in evidence.
NARCOM agents and effected thearrest of De la Cruz and
Beltran. The P10.00 marked bill used by Arcoy was found in
the possession of Juan de la Cruz together with two
aluminum foils and containing marijuana. Juande la Cruz y
Gonzales and Reynaldo Beltran y Aniban were charged in

GO VS. COURT OF APPEALS [206 SCRA 138; G.R.


NO. 101837; 11 FEB 1992]
Wednesday, February 04, 2009 Posted by Coffeeholic Writes
Labels: Case Digests, Political Law
Facts: Petitioner, while traveling in the wrong direction on a
one-way street, almost had a collision with another vehicle.
Petitioner thereafter got out of his car, shot the driver of the
other vehicle, and drove off. An eyewitness of the incident
was able to take down petitioners plate number and reported
the same to the police, who subsequently ordered a manhunt
for petitioner. 6 days after the shooting, petitioner presented
himself in the police station, accompanied by 2 lawyers, the
police detained him. Subsequently a criminal charge was
brought against him. Petitioner posted bail, the prosecutor
filed the case to the lower court, setting and commencing
trial without preliminary investigation. Prosecutor reasons
that the petitioner has waived his right to preliminary
investigation as bail has been posted and that such situation,
that petitioner has been arrested without a warrant lawfully,
falls under Section 5, Rule 113 and Section 7, Rule 112 of
The 1985 Rules of Criminal Procedure which provides for
the rules and procedure pertaining to situations of lawful
warrantless arrests. Petitioner in his petition for certiorari
assails such procedure and actions undertaken and files for a
preliminary
investigation.
Issues:
(1) Whether or Not warrantless arrest of petitioner was
lawful.
(2) Whether or Not petitioner effectively waived his right to
preliminary
investigation.
Held: Petitioner and prosecutor err in relying on Umil v.
Ramos, wherein the Court upheld the warrantless arrest as
valid effected 1 to 14 days from actual commission of the
offenses, which however constituted continuing crimes,
i.e. subversion, membership in an outlawed organization, etc.
There was no lawful warrantless arrest under Section 5, Rule
113. This is because the arresting officers were not actually

there during the incident, thus they had no personal


knowledge and their information regarding petitioner were
derived from other sources. Further, Section 7, Rule 112,
does
not
apply.
Petitioner was not arrested at all, as when he walked in the
police station, he neither expressed surrender nor any
statement that he was or was not guilty of any crime. When a
complaint was filed to the prosecutor, preliminary
investigation should have been scheduled to determine
probable cause. Prosecutor made a substantive error,
petitioner is entitled to preliminary investigation, necessarily
in a criminal charge, where the same is required appear
thereat. Petition granted, prosecutor is ordered to conduct
preliminary investigation, trial for the criminal case is
suspended pending result from preliminary investigation,
petitioner is ordered released upon posting a bail bond.
Go vs. Court of Appeals [GR 101837, 11 February 1992]
En Banc, Feliciano (J): 5 concur
Facts:
On 2 July 1991, Eldon Maguan was driving his car along
Wilson St., San Juan, MetroManila, heading towards P.
Guevarra St. Rolito Go y Tambunting entered Wilson St.,
where it isa one-way street and started traveling in the
opposite or wrong direction. At the corner of Wilson and J.
Abad Santos Sts., Gos and Maguans cars nearly bumped
each other. Go alightedfrom his car, walked over and shot
Maguan inside his car. Go then boarded his car and left
thescene. A security guard at a nearby restaurant was able to
take down Gos car plate number. The police arrived shortly
thereafter at the scene of the shooting and there retrieved an
empty shelland one round of live ammunition for a 9mm
caliber pistol. Verification at the LandTransportation Office
showed that the car was registered to one Elsa Ang Go. The
following day,the police returned to the scene of the shooting
to find out where the suspect had come from;they were
informed that Go had dined at Cravings Bake Shop shortly
before the shooting. The police obtained a facsimile or
impression of the credit card used by Go from the cashier of
the bake shop. The security guard of the bake shop was
shown a picture of Go and he positivelyidentified him as the
same person who had shot Maguan. Having established that
the assailantwas probably Go, the police launched a manhunt
for Go. On 8 July 1991, Go presented himself before the
San Juan Police Station to verify news reports that he was

being hunted by the police;he was accompanied by two (2)


lawyers. The police forthwith detained him. An eyewitness
tothe shooting, who was at the police station at that time,
positively identified Go as the gunman.That same day, the
police promptly filed a complaint for frustrated homicide
against Go with theOffice of the Provincial Prosecutor of
Rizal. First Assistant Provincial Prosecutor Dennis
VillaIgnacio (Prosecutor) informed Go, in the Presence of
his lawyers. that he could avail himself of his right to
preliminary investigation but that he must first sign a waiver
of the provisions of Article 125 of the Revised Penal Code.
Go refused to execute any such waiver. On 9 July
1991,while the complaint was still with the Prosecutor, and
before an information could be filed incourt, the victim,
Eldon Maguan, died of his gunshot wound(s). Accordingly,
on 11 July 1991,the Prosecutor, instead of filing an
information for frustrated homicide, filed an information
for murder before the Regional Trial Court. No bail was
recommended. At the bottom of theinformation, the
Prosecutor certified that no preliminary investigation had
been conducted because the accused did not execute and sign
a waiver of the provisions of Article 125 of theRevised Penal
Code. In the afternoon of 11 July 1991, Gos counsel filed
with the prosecutor an

which granted bail: petitioner was given 48 hours from


receipt of the Order to surrender himself: (2) recalling
andcancelling the 16 July 1991 Order which granted leave to
the Prosecutor to conduct preliminaryinvestigation: (3)
treating Gos omnibus motion for immediate release and
preliminaryinvestigation dated 11 July 1991 as a petition for
bail and set for hearing on 23 July 1991. On 19July 1991, Go
filed a petition for certiorari, prohibition and mandamus
before the Supreme Courtassailing the 17 July 1991 Order.
Go also moved for suspension of all proceedings in the
case pending resolution by the Supreme Court of his
petition: this motion was, however, denied byJudge Pelayo.
On 23 July 1991, Go surrendered to the police. By a
Resolution dated 24 July1991, the Supreme Court remanded
the petition for certiorari, prohibition and mandamus to
theCourt of Appeals. On 16 August 1991, Judge Pelayo
issued an order in open court setting Gosarraignment on 23
August 1991. On 19 August 1991, Go filed with the Court of
Appeals amotion to restrain his arraignment. On 23 August
1991, Judge Pelayo issued a CommitmentOrder directing the
Provincial Warden of Rizal to admit Go into his custody at
the RizalProvincial Jail. On the same date, Go was arraigned.
In view, however, of his refusal to enter a plea, the trial court
entered for him a plea of not guilty. The trial court then set
the criminal casefor continuous hearings on 19, 24 and 26
September; on 2, 3, 11 and 17 October; and on 7, 8, 14,15,
omnibus motion for immediate release and proper
preliminary investigation, alleging that thewarrantless arrest 21 and 22 November 1991. On 27 August 1991. Go filed a
petition for habeas corpus in theCourt of Appeals. On 30
of Go was unlawful and that no preliminary investigation
had been conducted before the information was filed. On 12 August 1991, the Court of Appeals issued the writ of habeas
corpus.The petition for certiorari, prohibition and
July 1991, Go filed an urgent ex-parte motion for
mandamus, on the one hand, and the petition for habeas
specialraffle in order to expedite action on the Prosecutors
corpus, upon the other, were subsequently consolidated in
bail recommendation. The case was raffledto the sala of
Judge Benjamin V. Pelayo (Branch 168, RTC of Pasig City), the Court of Appeals. TheCourt of Appeals, on 2 September
who, on the samedate, approved the cash bond posted by Go 1991, issued a resolution denying Gos motion to restrain
hisarraignment on the ground that motion had become moot
and ordered his release. Go was in fact released thatsame
day. On 16 July 1991, the Prosecutor filed with the Regional and academic. On 19 September 1991,trial of the criminal
case commenced. On 23 September 1991, the Court of
Trial Court a motion for leaveto conduct preliminary
Appeals rendered aconsolidated decision dismissing the 2
investigation and prayed that in the meantime all
proceedings in the court be suspended. On the said date, the petitions on the grounds that Gos warrantless arrest
wasvalid and Gos act of posting bail constituted waiver of
trial court issued an Order 9 granting leave to
any irregularity attending his arrest,among others. On 3
conduct preliminary investigation and cancelling the
October 1991, the prosecution presented three (3) more
arraignment set for 15 August 1991 until after
witnesses at the trial.Gos Counsel also filed a Withdrawal
the prosecution shall have concluded its preliminary
of Appearance with the trial court, with Gosconformity. On
investigation. On 17 July 1991, however, theJudge motu
proprio issued an Order, (1) recalling the 12 July 1991 Order 4 October 1991, Go filed the present petition for Review on

Certiorari. On 14October 1991, the Court issued a


Resolution directing Judge Pelayo to held in abeyance
thehearing of the criminal case below until further orders
from the Supreme Court.
Issue:
Whether Go was arrested legally without warrant for the
killing of Maguan, and is thusnot entitled to be released
pending the conduct of a preliminary investigation.

that Section 7 of Rule 112 is also notapplicable. Indeed, Go


was not arrested at all. When he walked into the San Juan
Police Station,accompanied by two (2) lawyers, he in fact
placed himself at the disposal of the policeauthorities. He did
not state that he was surrendering himself, in all
probability to avoid theimplication he was admitting that he
had slain Eldon Maguan or that he was otherwise guilty of
acrime. When the police filed a complaint for frustrated
homicide with the Prosecutor, the latter should have
immediately scheduled a preliminary investigation to
Held:
determine whether there was probable cause for charging Go
Gos warrantless arrest or detention does not fall within
in court for the killing of Eldon Maguan. Instead, as
the terms of Section 5 of Rule113 of the 1985 Rules on
Criminal Procedure which provides that A peace officer or notedearlier, the Prosecutor proceeded under the erroneous
a private person may, without a warrant, arrest a person: (a) supposition that Section 7 of Rule 112 wasapplicable and
required Go to waive the provisions of Article 125 of the
When, in his presence, the person to becreated has
Revised Penal Code as acondition for carrying out a
committed, is actually committing, or is attempting to
commit an offense; (b) Whenan offense has in fact just been preliminary investigation. This was substantive error, for Go
wasentitled to a preliminary investigation and that right
committed, and he has personal knowledge of facts
indicatingthat the person to be arrested has committed it; and should have been accorded him without anyconditions.
Moreover, since Go had not been arrested; with or without a
(c) When the person to be arrested is a prisoner who has
warrant, he was alsoentitled to be released forthwith subject
escaped from a penal establishment or place where he is
serving final judgmentor temporarily confined while his case only to his appearing at the preliminary investigation.
is pending or has escaped while being transferred fromone
confinement to another. In cases falling under paragraphs (a)
and (b) hereof, the personarrested without a warrant shall be PEOPLE VS. MARTI [193 SCRA 57; G.R. NO. 81561;
18 JAN 1991]
forthwith delivered to the nearest police station or jail, and
Tuesday, February 03, 2009 Posted by Coffeeholic Writes
heshall be proceeded against in accordance with Rule 112,
Section 7. Gos arrest took place 6days after the shooting Labels: Case Digests, Political Law
of Maguan. The arresting officers obviously were not
Facts: Accused-appellant went to a forwarding agency to
present, withinthe meaning of Section 5(a), at the time Go
send four packages to a friend in Zurich. Initially, the
had allegedly shot Maguan. Neither could thearrest
effected 6 days after the shooting be reasonably regarded as accused was asked by the proprietress if the packages can be
examined. However, he refused. Before delivering said
effected when [theshooting had] in fact just been
committed within the meaning of Section 5 (b). Moreover, packages to the Bureau of Customs and the Bureau of Posts,
the husband of the proprietress opened said boxes for final
noneof the arresting officers had any personal
knowledge of facts indicating that Go was thegunman who inspection. From that inspection, included in the standard
operating procedure and out of curiosity, he took several
had shot Maguan. The information upon which the police
grams
of
its
contents.
acted had been derivedfrom statements made by alleged
eyewitnesses to the shooting one stated that Go was
He brought a letter and the said sample to the National
thegunman another was able to take down the alleged
Bureau of Investigation. When the NBI was informed that
gunmans cars plate number which turnedout to be
the rest of the shipment was still in his office, three agents
registered in Gos wifes name. That information did not,
however, constitute personalknowledge. It is thus clear to went back with him. In their presence, the husband totally
opened the packages. Afterwards, the NBI took custody of
the Court that there was no lawful warrantless arrest of Go
said packages. The contents , after examination by forensic
withinthe meaning of Section 5 of Rule 113. It is clear too

chemists, were found to be marijuana flowering tops.


The appellant, while claiming his mail at the Central Post
Office, was invited by the agents for questioning. Later on,
the trial court found him guilty of violation of the Dangerous
Drugs
Act.
Issues:
(1) Whether or Not the items admitted in the searched
illegally
searched
and
seized.
(2) Whether or Not custodial investigation properly applied.
(3) Whether or Not the trial court not give credence to the
explanation of the appellant on how said packages came to
his
possession.
Held: No. The case at bar assumes a peculiar character
since the evidence sought to be excluded was primarily
discovered and obtained by a private person, acting in a
private capacity and without the intervention and
participation of State authorities. Under the circumstances,
can accused/appellant validly claim that his constitutional
right against unreasonable searches and seizure has been
violated. Stated otherwise, may an act of a private individual,
allegedly in violation of appellant's constitutional rights, be
invoked against the State. In the absence of governmental
interference, the liberties guaranteed by the Constitution
cannot be invoked against the State. It was Mr. Job Reyes,
the proprietor of the forwarding agency, who made
search/inspection of the packages. Said inspection was
reasonable and a standard operating procedure on the part of
Mr. Reyes as a precautionary measure before delivery of
packages to the Bureau of Customs or the Bureau of Posts.
Second, the mere presence of the NBI agents did not convert
the reasonable search effected by Reyes into a warrantless
search and seizure proscribed by the Constitution. Merely to
observe and look at that which is in plain sight is not a
search. Having observed that which is open, where no
trespass has been committed in aid thereof, is not search.
No. The law enforcers testified that accused/appellant was

informed of his constitutional rights. It is presumed that they


have regularly performed their duties (See. 5(m), Rule 131)
and their testimonies should be given full faith and credence,
there
being
no
evidence
to
the
contrary.

packages. The contents, after examination by


forensicchemists, were found to be marijuana flowering
tops.The appellant, while claiming his mail at the Central
Post Office, was invited by theagents for questioning. Later
on, the trial court found him guilty of violation of
No. Appellant signed the contract as the owner and shipper theDangerous Drugs Act.
thereof giving more weight to the presumption that things ISSUES:
which a person possesses, or exercises acts of ownership 1.
over, are owned by him (Sec. 5 [j], Rule 131). At this point,
appellant is therefore estopped to claim otherwise.
Whether or not the items admitted in the searched illegally
searched and seized.2.
PEOPLE OF THE PHILIPPINES,
plaintiff-appellee vs
Whether or not custodial investigation was not properly
applied.3.
A
NDRE M
Whether or not the trial court did not give credence to the
A
explanation of the appellanton how said packages came to
RTI,
his possession.
accused-appellant
HELD:
.
1.
G
.
No. The case at bar assumes a peculiar character since the
evidence sought to beexcluded was primarily discovered and
R
obtained by a private person, acting in a private capacity and
.
without the intervention and participation of State
No
authorities.Under the circumstances, can accused/appellant
.
validly claim that his constitutionalright against
81561 January 18, 1991
unreasonable searches and seizure has been violated. Stated
otherwise,may an act of a private individual, allegedly in
F
violation of appellant's constitutionalrights, be invoked
AC
against the State. In the absence of governmental
TS:
Accused-appellant went to a forwarding agency to send four interference, theliberties guaranteed by the Constitution
cannot be invoked against the State. It wasMr. Job Reyes, the
packages to a friend inZurich. Initially, the accused was
proprietor of the forwarding agency, who made
asked by the proprietress if the packages can beexamined.
However, he refused. Before delivering said packages to the search/inspectionof the packages. Said inspection was
Bureau of Customs and the Bureau of Posts, the husband of reasonable and a standard operating procedureon the part of
the proprietress opened said boxes for final inspection. From Mr. Reyes as a precautionary measure before delivery of
that inspection, included in the standard operating procedure packages tothe Bureau of Customs or the Bureau of Posts.
andout of curiosity, he took several grams of its contents.He Second, the mere presence of the NBIagents did not convert
the reasonable search effected by Reyes into a
brought a letter and the said sample to the National Bureau
of Investigation. When the NBI was informed that the rest of warrantlesssearch and seizure proscribed by the Constitution.
Merely to observe and look at thatwhich is in plain sight is
the shipment was still in his office, three agents went back
not a search. Having observed that which is open, where
with him. In their presence, the husband totally opened the
notrespass has been committed in aid thereof, is not search.
packages. Afterwards,the NBI took custody of said
2.

No. The law enforcers testified that accused/appellant was


informed of hisconstitutional rights. It is presumed that they
have regularly performed their duties(See. 5(m), Rule 131)
and their testimonies should be given full faith and
credence,there being no evidence to the contrary.3.
No. Appellant signed the contract as the owner and shipper
thereof giving moreweight to the presumption that things
which a person possesses, or exercises acts of ownership
over, are owned by him (Sec. 5 [j], Rule 131). At this point,
appellant istherefore estopped to claim otherwise.
Horacio Morales Jr vs Minister of Defense Juan
Ponce Enrile et al
17 11 2010

Habeas Corpus The Right to Bail


In April 1982, Morales and some others were arrested while
driving a motor vehicle in Laong-Laan St, QC. They were
charged in CFI Rizal for rebellion punishable under the RPC.
Morales alleged that they were arrested without any warrant
of arrest; that their constitutional rights were violated, among
them the right to counsel, the right to remain silent, the right
to a speedy and public trial, and the right to bail.
Respondents countered that the group of Morales were
already under surveillance for some time before they were
arrested and that the warrantless arrest done is valid and at
the same time the privilege of the writ of habeas corpus was
already suspended.
ISSUE: Whether or not Morales et al can post bail.
HELD: Normally, rebellion being a non-capital offense is
bailable. But because the privilege of the writ of habeas
corpus remains suspended with respect to persons at present
detained as well as other who may hereafter be similarly
detained for the crimes of insurrection or rebellion,

subversion, conspiracy or proposal to commit such crimes,


and for all other crimes and offenses committed by them in
furtherance of or on the occasion thereof, or incident thereto,
or in connection therewith, the natural consequence is that
the right to bail for the commission of anyone of the said
offenses is also suspended. To hold otherwise would defeat
the very purpose of the suspension. Therefore, where the
offense for which the detainee was arrested is anyone of the
said offenses he has no right to bail even after the charges
are filed in court. The crimes of rebellion, subversion,
conspiracy or proposal to commit such crimes, and crimes or
offenses committed in furtherance thereof or in connection
therewith constitute direct attacks on the life of the State.
Just as an individual has right to self-defense when his life is
endangered, so does the State. The suspension of the
privilege of the writ is to enable the State to hold in
preventive imprisonment pending investigation and trial
those persons who plot against it and commit acts that
endanger the States very existence. For this measure of selfdefense to be effective, the right to bail must also be deemed
suspended with respect to these offenses. However, there is a
difference between preventive and punitive imprisonment.
Where the filing of charges in court or the trial of such
charges already filed becomes protracted without any
justifiable reason, the detention becomes punitive in
character and the detainee regains his right to freedom. Quite
notable in this case however is that the 2nd division of the SC
reiterated the Lansang Doctrine as opposed to what they
ruled in the Garcia-Padilla Case.
PEOPLE OF THE PHILIPPINES, Plaintiff-Appellee,
vs.RUBEN BURGOS y TITO, Defendant-Appellant
G . R . No.L-68955 September 4, 1986
FACTS:
Cesar Masamlok surrendered to the authorities at the Davao
del Sur Constabulary Head Quarters.He testified that Ruben
Burgos forcibly recruited him as member of NPA. Burgos threatened
himwith the use of a firearm. Masamlok attended the seminar where
Burgos spoke about hismembership with the NPAand the
organizations desire to overthrow the government.Pursuant to this
information, PC-INP members went to house of accused. Accused was

plowingthe field when they arrived. Pat. Bioco called the accused and
asked him about the firearm.Accused denied possession of said
firearm but later, his wife pointed to a place below their
housewhere a gun was buried in the ground. After the recovery of said
firearm, accused pointed to astock pile of cogon where the officers
recovered:1.maroon notebook 2.

Firearm in Furtherance of Subversion is the testimony of Cesar


Masamlok. In the instant case, Masamlok's testimonywas totally
uncorroborated. Considering that Masamlok surrendered to the military
certainlyhis fate depended on how eagerly he cooperated with the
authorities
G.R. Nos. 95122-23 May 31, 1991

pamphlets: Ang Bayan, Pahayagan ng Paritdo Komunista ng Pilipinas


etc.Accused admitted that firearm was issued to him by Nestor Jimines,
team leader of sparrow unit.
ISSUE:
1.

BOARD OF COMMISSIONERS (COMMISSION ON


IMMIGRATION AND DEPORTATION), BOARD OF
SPECIAL INQUIRY, COMMISSIONER ANDREA D.
DOMINGO, ASSOCIATE COMMISSIONER JORGE V.
SARMIENTO, ACTING ASSOCIATE
Whether or not the warrantless arrest of Ruben Burgos was lawful?2.
COMMISSIONER REGINO R. SANTIAGO,
Whether or not the search of his house and the subsequent confiscation MEMBERS OF THE BOARD OF SPECIAL INQUIRY,
of a firearm anddocuments allegedly found therein conducted in a lawful ESTANISLAO CANTA, LEO MAGAHOM and
and valid manner?3.
BENJAMIN KALAW, petitioners,
vs.
Whether or not the evidence sustaining the crime charged meet the test of HON. JOSELITO DELA ROSA, Presiding Judge, RTC
proving guilt beyond reasonable doubt?
Manila, Branch 29, WILLIAM T. GATCHALIAN,
HELD:
respondents.
1.
BOARD OF COMMISSIONERS (COMMISSION ON
IMMIGRATION AND DEPORTATION), BOARD OF
SPECIAL INQUIRY, COMMISSIONER ANDREA D.
DOMINGO, ASSOCIATE COMMISSIONER JORGE V.
SARMIENTO, ACTING ASSOCIATE
COMMISSIONER REGINO R. SANTIAGO,
MEMBERS OF THE BOARD OF SPECIAL INQUIRY,
ESTANISLAO CANTA, LEO MAGAHOM and
No. If an arrest without a warrant is unlawful at the moment it is made, BENJAMIN KALAW, petitioners,
generally nothing thathappened or is discovered afterwards can make it vs.
lawful. Neither can it be presumed thatthere was a waiver, or that consent HON. TERESITA DIZON CAPULONG, Presiding
was given by the accused to be searched simply becausehe failed to
Judge, RTC Branch 172, Valenzuela, Metro Manila, DEE
object.3.
HUA T. GATCHALIAN, SHERWING T.
GATCHALIAN, KENNETH T. GATCHALIAN,
No. The Constitution itself mandates that any evidence obtained in
REXLON T. GATCHALIAN, and WESLIE T.
violation of right isinadmissible in evidence. With the extra-judicial
GATCHALIAN, respondents.
confession, the firearm, and the allegedsubversive documents
inadmissible in evidence against the accused-appellant, the
G.R. Nos. 95612-13 May 31, 1991
onlyremaining proof to sustain the charge of Illegal Possession of
No. Under Section 6(a) of Rule 113, the officer arresting a person who
has just committed, iscommitting, or is about to commit an offense must
have
personal
knowledge of that fact. Theoffense must also be committed in his
presence or within his view. In this case, the accusedwas arrested on the
sole basis of Masamlok's verbal report. Masamlok led the authorities
tosuspect that the accused had committed a crime.2.

WILLIAM T. GATCHALIAN, petitioner,


vs.
BOARD OF COMMISSIONERS (COMMISSION ON
IMMIGRATION AND DEPORTATION), et al.,
respondents.

On December 13, 1990, petitioners filed their comment to


respondent Gatchalian's counter-petition. The Court
considers the comment filed by respondent Gatchalian as
answer to the petition and petitioners' comment as answer to
the counter-petition and gives due course to the petitions.

The Solicitor General for petitioners.

There is no dispute as to the following facts:

edesma, Saludo & Associates for respondent William


Gatchalian.
Cervo and Tanay Law Office for respondent T.D. Capulong,
D.H.T. Gatchalian, et al.

BIDIN, J.:p
This is a petition for certiorari and prohibition filed by the
Solicitor General seeking 1) to set aside the
Resolution/Temporary Restraining Order dated September 7,
1990, issued by respondent Judge de la Rosa in Civil Case
No. 90-54214 which denied petitioners' motion to dismiss
and restrained petitioners from commencing or continuing
with any of the proceedings which would lead to the
deportation of respondent William Gatchalian, docketed as
D.C. No. 90-523, as well as the Order of respondent Judge
Capulong dated September 6, 1990 in Civil Case No. 3431V-90 which likewise enjoined petitioners from proceeding
with the deportation charges against respondent Gatchalian,
and 2) to prohibit respondent judges from further acting in
the aforesaid civil cases.
On October 23, 1990, respondent Gatchalian filed his
Comment with Counter-Petition, docketed as G.R. Nos.
96512-13, alleging lack of jurisdiction on the part of
respondent Board of Commissioners, et al., over his person
with prayer that he be declared a Filipino citizen, or in the
alternative, to remand the case to the trial court for further
proceedings.

On July 12, 1960, Santiago Gatchalian, grandfather of


William Gatchalian, was recognized by the Bureau of
Immigration as a native born Filipino citizen following the
citizenship of his natural mother, Marciana Gatchalian
(Annex "1", counter-petition). Before the Citizenship
Evaluation Board, Santiago Gatchalian testified that he has
five (5) children with his wife Chu Gim Tee, namely: Jose
Gatchalian, Gloria Gatchalian, Francisco Gatchalian, Elena
Gatchalian and Benjamin Gatchalian (Annex "2", counterpetition).
On June 27, 1961, William Gatchalian, then a twelve-year
old minor, arrived in Manila from Hongkong together with
Gloria, Francisco, and Johnson, all surnamed Gatchalian.
They had with them Certificates of Registration and Identity
issued by the Philippine Consulate in Hongkong based on a
cablegram bearing the signature of the then Secretary of
Foreign Affairs, Felixberto Serrano, and sought admission as
Filipino citizens. Gloria and Francisco are the daughter and
son, respectively, of Santiago Gatchalian; while William and
Johnson are the sons of Francisco.
After investigation, the Board of Special Inquiry No. 1
rendered a decision dated July 6, 1961, admitting William
Gatchalian and his companions as Filipino citizens (Annex
"C", petition). As a consequence thereof, William Gatchalian
was issued Identification Certificate No. 16135 by the
immigration authorities on August 16, 1961 (Annex "D",
petition).
On January 24, 1962, the then Secretary of Justice issued
Memorandum No. 9 setting aside all decisions purporting to

have been rendered by the Board of Commissioners on


appeal or on review motu proprio of decisions of the Board
of Special Inquiry. The same memorandum directed the
Board of Commissioners to review all cases where entry was
allowed on the ground that the entrant was a Philippine
citizen. Among those cases was that of William and others.
On July 6, 1962, the new Board of Commissioners, after a
review motu proprio of the proceedings had in the Board of
Special Inquiry, reversed the decision of the latter and
ordered the exclusion of, among others, respondent
Gatchalian (Annex "E", petition). A warrant of exclusion
also dated July 6, 1962 was issued alleging that "the decision
of the Board of Commissioners dated July 6, 1962 . . . has
now become final and executory (Annex "F", petition).
The actual date of rendition of said decision by the Board of
Commissioners (whether on July 6, 1962 or July 20, 1962)
became the subject of controversy in the 1967 case of
Arocha vs. Vivo (21 SCRA 532) wherein this Court sustained
the validity of the decision of the new Board of
Commissioners having been promulgated on July 6, 1962, or
within the reglementary period for review.
Sometime in 1973, respondent Gatchalian, as well as the
others covered by the July 6, 1962 warrant of exclusion,
filed a motion for re-hearing with the Board of Special
Inquiry where the deportion case against them was assigned.
On March 14, 1973, the Board of Special Inquiry
recommended to the then Acting Commissioner Victor
Nituda the reversal of the July 6, 1962 decision of the then
Board of Commissioners and the recall of the warrants of
arrest issued therein (Annex "5", counter-petition).
On March 15, 1973, Acting Commissioner Nituda issued an
order reaffirming the July 6, 1961 decision of the Board of
Special Inquiry thereby admitting respondent Gatchalian as a
Filipino citizen and recalled the warrant of arrest issued
against him (Annex "6", counter-petition).

On June 7, 1990, the acting director of the National Bureau


of Investigation wrote the Secretary of Justice
recommending that respondent Gatchalian along with the
other applicants covered by the warrant of exclusion dated
July 6, 1962 be charged with violation of Sec. 37 (a), pars. 1
and 2, in relation to Secs. 45 (c), and (d) and (e) of
Commonwealth Act No. 613, as amended, also known as the
Immigration Act of 1940 (Annex "G", petition).
On August 1, 1990, the Secretary of Justice indorsed the
recommendation of the NBI to the Commissioner of
Immigration for investigation and immediate action (Annex
"20", counter-petition).
On August 15, 1990, petitioner Commissioner Domingo of
the Commission of Immigration and Deportation * issued a
mission order commanding the arrest of respondent William
Gatchalian (Annex "18", counter-petition). The latter
appeared before Commissioner Domingo on August 20,
1990 and was released on the same day upon posting
P200,000.00 cash bond.
On August 29, 1990, William Gatchalian filed a petition for
certiorari and prohibition with injunction before the
Regional Trial Court of Manila, Br. 29, presided by
respondent Judge dela Rosa, docketed as Civil Case No. 9054214.

alleged, among others, that petitioners acted without or in


excess of jurisdiction in the institution of deportation
proceedings against William. On the same day, respondent
Capulong issued the questioned temporary restraining order
restraining petitioners from continuing with the deportation
proceedings against William Gatchalian.
The petition is anchored on the following propositions: 1)
respondent judges have no jurisdiction over petitioners
(Board of Commissioners, et al.,) and the subject matter of
the case, appellate jurisdiction being vested by BP 129 with
the Court of Appeals; 2) assuming respondent judges have
jurisdiction, they acted with grave abuse of discretion in
preempting petitioners in the exercise of the authority and
jurisdiction to hear and determine the deportation case
against respondent Gatchalian, and in the process determine
also his citizenship; 3) respondent judge dela Rosa gravely
abused his discretion in ruling that the issues raised in the
deportation proceedings are beyond the competence and
jurisdiction of petitioners, thereby disregarding the cases of
Arocha vs. Vivo and Vivo vs. Arca (supra), which put finality
to the July 6, 1962 decision of the Board of Commissioners
that respondent Gatchalian is a Chinese citizen; and 4)
respondent judge Capulong should have dismissed Civil
Case No. 3431-V-90 for forum-shopping.

In his counter-petition, William Gatchalian alleges among


others that: 1) assuming that the evidence on record is not
On September 4, 1990, petitioners filed a motion to dismiss sufficient to declare him a Filipino citizen, petitioners have
Civil Case No. 90-54214 alleging that respondent judge has no jurisdiction to proceed with the deportation case until the
no jurisdiction over the Board of Commissioners and/or the courts shall have finally resolved the question of his
Board of Special Inquiry. Nonetheless, respondent judge dela citizenship; 2) petitioners can no longer judiciously and
Rosa issued the assailed order dated September 7, 1990,
fairly resolve the question of respondent's citizenship in the
denying the motion to dismiss.
deportation case because of their bias, pre-judgment and
prejudice against him; and 3) the ground for which he is
Meanwhile, on September 6, 1990, respondent Gatchalian's sought to be deported has already prescribed.
wife and minor children filed before the Regional Trial Court
of Valenzuela, Metro Manila, Br. 172, presided by
For purposes of uniformity, the parties herein will be referred
respondent judge Capulong Civil Case No. 3431-V-90 for
to in the order the petitions were filed.
injunction with writ of preliminary injunction. The complaint

Petitioners argue that under Sec. 9 (3) of BP 129, it is the


Court of Appeals which has exclusive appellate jurisdiction
over all final judgments or orders of quasi-judicial agencies,
boards or commissions, such as the Board of Commissioners
and the Board of Special Inquiry.
Respondent, on the other hand, contends that petitioners are
not quasi-judicial agencies and are not in equal rank with
Regional Trial Courts.
Under Sec. 21 (1) of Batas Pambansa Blg. 129, the Regional
Trial Courts have concurrent jurisdiction with this Court and
the Court of Appeals to issue "writs of certiorari,
prohibition, mandamus, quo warranto, habeas corpus and
injunction which may be enforced in any part of their
respective regions, . . ." Thus, the RTCs are vested with the
power to determine whether or not there has been a grave
abuse of discretion on the part of any branch or
instrumentality of the government.
It is true that under Sec. 9 (3) of Batas Pambansa Blg. 129,
the Court of Appeals is vested with
(3) Exclusive appellate jurisdiction over all
final judgments, decisions, resolutions,
order, or awards of Regional Trial Courts
and quasi-judicial agencies,
instrumentalities, board or commission,
except those falling within the appellate
jurisdiction of the Supreme Court in
accordance with the Constitution, the
provisions of this Act, and of sub-paragraph
(1) of the third paragraph of and subparagraph (4) of the fourth paragraph of
Section 17 of the Judiciary Act of 1948.
It does not provide, however, that said exclusive appellate
jurisdiction of the Court of Appeals extends to all quasijudicial agencies. The quasi-judicial bodies whose decisions
are exclusively appealable to the Court of Appeals are those

which under the law, Republic Act No. 5434, or their


enabling acts, are specifically appealable to the Court of
Appeals (Presidential Anti-Dollar Salting Task Force vs.
Court of Appeals, 171 SCRA 348 [1989]; Lupangco vs.
Court of Appeals, 160 SCRA 848 [1988]). Thus, under
Republic Act No. 5434, it is specifically provided that the
decisions of the Land Registration Commission (LRC), the
Social Security Commission (SSC), Civil Aeronautics Board
(CAB), the Patent Office and the Agricultural Invention
Board are appealable to the Court of Appeals.
In the Presidential Anti-Dollar Salting Task Force (supra),
this Court clarified the matter when We ruled:
Under our Resolution dated January 11,
1983:
. . . The appeals to the
Intermediate Appellate
Court (now Court of
Appeals) from quasijudicial bodies shall
continue to be governed by
the provisions of Republic
Act No. 5434 insofar as the
same is not inconsistent
with the provisions of B.P.
Blg. 129.
The pertinent provisions of Republic Act
No. 5434 are as follows:
Sec. 1. Appeals from
specified agencies. Any
provision of existing law or
Rules of Court to the
contrary notwithstanding,
parties aggrieved by a final
ruling, award, order, or
decision, or judgment of the

Court of Agrarian Relations;


the Secretary of Labor
under Section 7 of Republic
Act Numbered Six hundred
and two, also known as the
"Minimum Wage Law"; the
Department of Labor under
Section 23 of Republic Act
Numbered Eight hundred
seventy-five, also known as
the "Industrial Peace Act";
the Land Registration
Commission; the Social
Security Commission; the
Civil Aeronautics Board;
the Patent Office and the
Agricultural Inventions
Board, may appeal
therefrom to the Court of
Appeals, within the period
and in the manner herein
provided, whether the
appeal involves questions of
fact, mixed questions of fact
and law, or questions of law,
or all three kinds of
questions. From final
judgments or decisions of
the Court of Appeals, the
aggrieved party may appeal
by certiorari to the Supreme
Court as provided under
Rule 45 of the Rules of
Court.
Because of subsequent amendments,
including the abolition of various special
courts, jurisdiction over quasi-judicial
bodies has to be, consequently, determined
by the corresponding amendatory statutes.

Under the Labor Code, decisions and awards


of the National Labor Relations Commission
are final and executory, but, nevertheless,
reviewable by this Court through a petition
for certiorari and not by way of appeal.
Under the Property Registration Decree,
decision of the Commission of Land
Registration, en consulta, are appealable to
the Court of Appeals.
The decisions of the Securities and
Exchange Commission are likewise
appealable to the Appellate Court, and so are
decisions of the Social Security
Commission.
As a rule, where legislation provides for an
appeal from decisions of certain
administrative bodies to the Court of
Appeals, it means that such bodies are coequal with the Regional Trial Courts, in
terms of rank and stature, and logically,
beyond the control of the latter. (Emphasis
supplied)
There are quasi-judicial agencies, as the National Labor
Relations Commissions, whose decisions are directly
appealable to this Court. It is only when a specific law, as
Republic Act No. 5434, provides appeal from certain bodies
or commissions to the Court of Appeals as the Land
Registration Commission (LRC), Securities and Exchange
Commission (SEC) and others, that the said commissions or
boards may be considered co-equal with the RTCs in terms
of rank, stature and are logically beyond the control of the
latter.
However, the Bureau of Immigration (or CID) is not among
those quasi-judicial agencies specified by law whose
decisions, orders, and resolutions are directly appealable to

the Court of Appeals. In fact, its decisions are subject to


judicial review in accordance with Sec. 25, Chapter 4, Book
VII of the 1987 Administrative Code, which provides as
follows:
Sec. 25. Judicial Review.(1) Agency
decisions shall be subject to judicial review
in accordance with this chapter and
applicable laws.
xxx xxx xxx
(6) The review proceeding shall be filed in
the court specified in the statute or, in the
absence thereof, in any court of competent
jurisdiction in accordance with the
provisions on venue of the Rules of Court.
Said provision of the Administrative Code, which is
subsequent to B.P. Blg. 129 and which thus modifies the
latter, provides that the decision of an agency like the Bureau
of Immigration should be subject to review by the court
specified by the statute or in the absence thereof, it is subject
to review by any court of competent jurisdiction in
accordance with the provisions on venue of the Rules of
Court.
B.P. Blg. 129 did not intend to raise all quasi-judicial bodies
to the same level or rank of the RTC except those
specifically provided for under the law as aforestated. As the
Bureau of Immigration is not of equal rank as the RTC, its
decisions may be appealable to, and may be reviewed
through a special civil action for certiorari by, the RTC (Sec.
21, (1) BP 129).
True, it is beyond cavil that the Bureau of Immigration has
the exclusive authority and jurisdiction to try and hear cases
against an alleged alien, and in the process, determine also
their citizenship (Lao Gi vs. Court of Appeals, 180 SCRA
756 [1989]). And a mere claim of citizenship cannot operate

to divest the Board of Commissioners of its jurisdiction in


deportation proceedings (Miranda vs. Deportation Board, 94
Phil. 531 [1954]).
However, the rule enunciated in the above-cases admits of an
exception, at least insofar as deportation proceedings are
concerned. Thus, what if the claim to citizenship of the
alleged deportee is satisfactory? Should the deportation
proceedings be allowed to continue or should the question of
citizenship be ventilated in a judicial proceeding? In Chua
Hiong vs. Deportation Board (96 Phil. 665 [1955]), this
Court answered the question in the affirmative, and We
quote:

can be availed of only after the Deportation


Board has unjustly trampled upon it,
besmirching the citizen's name before the
bar of public opinion? (Emphasis supplied)

The doctrine of primary jurisdiction of petitioners Board of


Commissioners over deportation proceedings is, therefore,
not without exception (Calacday vs. Vivo, 33 SCRA 413
[1970]; Vivo vs. Montesa, 24 SCRA 155 [1967]). Judicial
intervention, however, should be granted only in cases where
the "claim of citizenship is so substantial that there are
reasonable grounds to believe that the claim is correct. In
other words, the remedy should be allowed only on sound
discretion of a competent court in a proper proceeding (Chua
When the evidence submitted by a
Hiong vs. Deportation Board, supra; Co. vs. Deportation
respondent is conclusive of his citizenship,
Board, 78 SCRA 107 [1977]). It appearing from the records
the right to immediate review should also be that respondent's claim of citizenship is substantial, as We
recognized and the courts should promptly
shall show later, judicial intervention should be allowed.
enjoin the deportation proceedings. A
citizen is entitled to live in peace, without
In the case at bar, the competent court which could properly
molestation from any official or authority,
take cognizance of the proceedings instituted by respondent
and if he is disturbed by a deportation
Gatchalian would nonetheless be the Regional Trial Court
proceeding, he has the unquestionable right and not the Court of Appeals in view of Sec. 21 (1), BP 129,
to resort to the courts for his protection,
which confers upon the former jurisdiction over actions for
either by a writ of habeas corpus or of
prohibition concurrently with the Court of Appeals and the
prohibition, on the legal ground that the
Supreme Court and in line with the pronouncements of this
Board lacks jurisdiction. If he is a citizen
Court in Chua Hiong and Co cases.
and evidence thereof is satisfactory, there is
Ordinarily, the case would then be remanded to the Regional
no sense nor justice in allowing the
Trial Court. But not in the case at bar. Considering the
deportation proceedings to continue,
voluminous pleadings submitted by the parties and the
granting him the remedy only after the
evidence presented, We deem it proper to decide the
Board has finished its investigation of his
controversy right at this instance. And this course of action is
undesirability.
not without precedent for "it is a cherished rule of procedure
. . . And if the right (to peace) is precious
for this Court to always strive to settle the entire controversy
and valuable at all, it must also be protected in a single proceeding leaving no root or branch to bear the
on time, to prevent undue harassment at the seeds of future litigation. No useful purpose will be served if
hands of ill-meaning or misinformed
this case is remanded to the trial court only to have its
administrative officials. Of what use is this
decision raised again to the Court of Appeals and from there
much boasted right to peace and liberty if it to this Court" (Marquez vs. Marquez, 73 Phil. 74; Keramic

Industries, Inc. vs. Guerrero, 61 SCRA 265 [1974]) Alger


Electric, Inc. vs. Court of Appeals (135 SCRA 37 [1985]),
citing Gayos vs. Gayos (67 SCRA 146 [1975]).
In Lianga Bay Logging Co., Inc. vs. Court of Appeals (157
SCRA 357 [1988]), We also stated:
Remand of the case to the lower court for
further reception of evidence is not
necessary where the court is in a position to
resolve the dispute based on the records
before it. On many occasions, the Court, in
the public interest and the expeditious
administration of justice, has resolved
actions on the merits instead of remanding
them to the trial court for further
proceedings, such as where the ends of
justice would not be subserved by the
remand of the case or when public interest
demands an early disposition of the case or
where the trial court had already received all
the evidence of the parties (Quisumbing vs.
CA, 112 SCRA 703; Francisco, et al., vs.
The City of Davao, et al., supra; Republic
vs. Security Credit & Acceptance Corp., et
al., 19 SCRA 58; Samal vs. CA, supra;
Republic vs. Central Surety & Insurance
Co., 25 SCRA 641).
Likewise in Tejones vs. Gironella (159 SCRA 100 [1988]),
We said:
Sound practice seeks to accommodate the
theory which avoids waste of time, effort
and expense, both to the parties and the
government, not to speak of delay in the
disposal of the case (cf. Fernandez vs.
Garcia, 92 Phil. 592, 297). A marked
characterstic of our judicial set-up is that
where the dictates of justice so demand . . .

the Supreme Court should act, and act with


finality (Li Siu Liat vs. Republic, 21 SCRA
1039, 1046, citing Samal vs. CA, 99 Phil.
230 and US vs. Gimenez, 34 Phil. 74.)
(Beautifont, Inc. vs. Court of appeals, et al.,
Jan. 29, 1988; See also Labo vs.
Commission on Elections, 176 SCRA 1
[1989]).

of Pedro Gatchalian. Well settled is the rule that a person not


party to a case cannot be bound by a decision rendered
therein.
Neither can it be argued that the Board of Commissioners'
decision (dated July 6, 1962) finding respondent's claim to
Philippine citizenship not satisfactorily proved, constitute
res judicata. For one thing, said decision did not make any
categorical statement that respondent Gatchalian is a
Chinese. Secondly, the doctrine of res judicata does not
apply to questions of citizenship (Labo vs. Commission on
Elections (supra); citing Soria vs. Commissioner of
Immigration, 37 SCRA 213; Lee vs. Commissioner of
Immigration, 42 SCRA 561 [1971]; Sia Reyes vs.
Deportation Board, 122 SCRA 478 [1983]).

Respondent Gatchalian has adduced evidence not only


before the Regional Trial Court but also before Us in the
form of public documents attached to his pleadings. On the
other hand, Special Prosecutor Renato Mabolo in his
Manifestation (dated September 6, 1990; Rollo, p. 298,
counter-petition) before the Bureau of Immigration already
stated that there is no longer a need to adduce evidence in
In Moy Ya Lim vs. Commissioner of Immigration (41 SCRA
support of the deportation charges against respondent. In
292 [1971]) and in Lee vs. Commissioner of Immigration
addition, petitioners invoke that this Court's decision in
Arocha vs. Vivo and Vivo vs. Arca (supra), has already settled (supra), this Court declared that:
respondent's alienage. Hence, the need for a judicial
(e)verytime the citizenship of a person is
determination of respondent's citizenship specially so where
material or indispensable in a judicial or
the latter is not seeking admission, but is already in the
administrative case, whatever the
Philippines (for the past thirty [30] years) and is being
corresponding court or administrative
expelled (Chua Hiong vs. Deportation Board, supra).
authority decides therein as to such
citizenship is generally not considered as res
According to petitioners, respondent's alienage has been
adjudicata, hence it has to be threshed out
conclusively settled by this Court in the Arocha and Vivo
again and again as the occasion may
cases, We disagree. It must be noted that in said cases, the
demand.
sole issue resolved therein was the actual date of rendition of
the July 6, 1962 decision of the then board of
An exception to the above rule was laid by this Court in
Commissioners, i.e., whether the decision was rendered on
July 6, 1962 or on July 20, 1962 it appearing that the figure Burca vs. Republic (51 SCRA 248 [1973]), viz:
(date) "20" was erased and over it was superimposed the
We declare it to be a sound rule that where
figure "6" thereby making the decision fall within the onethe citizenship of a party in a case is
year reglementary period from July 6, 1961 within which the
definitely resolved by a court or by an
decision may be reviewed. This Court did not squarely pass
administrative agency, as a material issue in
upon any question of citizenship, much less that of
the controversy, after a full-blown hearing
respondent's who was not a party in the aforesaid cases. The
with the active participation of the Solicitor
said cases originated from a petition for a writ of habeas
General or his authorized representative, and
corpus filed on July 21, 1965 by Macario Arocha in behalf

this finding or the citizenship of the party is


affirmed by this Court, the decision on the
matter shall constitute conclusive proof of
such party's citizenship in any other case or
proceeding. But it is made clear that in no
instance will a decision on the question of
citizenship in such cases be considered
conclusive or binding in any other case or
proceeding, unless obtained in accordance
with the procedure herein stated.
Thus, in order that the doctrine of res judicata may be
applied in cases of citizenship, the following must be
present: 1) a person's citizenship must be raised as a material
issue in a controversy where said person is a party; 2) the
Solicitor General or his authorized representative took active
part in the resolution thereof, and 3) the finding or
citizenship is affirmed by this Court.
Gauged by the foregoing, We find the pre-conditions set
forth in Burca inexistent in the Arocha and Vivo cases relied
upon by petitioners. Indeed, respondent William Gatchalian
was not even a party in said cases.
Coming now to the contention of petitioners that the arrest of
respondent follows as a matter of consequence based on the
warrant of exclusion issued on July 6, 1962, coupled with
the Arocha and Vivo cases (Rollo, pp. 33), the Court finds the
same devoid of merit.
Sec. 37 (a) of Commonwealth Act No. 613, as amended,
otherwise known as the Immigration Act of 1940, reads:
Sec. 37. (a) The following aliens shall be
arrested upon the warrant of the
Commissioner of Immigration or of any
other officer designated by him for the
purpose and deported upon the warrant of
the Commissioner of Immigration after a
determination by the Board of

Commissioner of the existence of the ground suspects, William Gatchalian included. Paragraphs 1 and 3 of
for deportation as charged against the alien. the mission order directs the Intelligence Agents/Officers to:
(Emphasis supplied)
xxx xxx xxx
From a perusal of the above provision, it is clear that in
1. Make a warrantless arrest under the Rules
matters of implementing the Immigration Act insofar as
of Criminal Procedure, Rule 113, Sec. 5, for
deportation of aliens are concerned, the Commissioner of
violation of the Immigration Act, Sec. 37,
Immigration may issue warrants of arrest only after a
para. a; Secs. 45 and 46 Administrative
determination by the Board of Commissioners of the
Code;
existence of the ground for deportation as charged against
the alien. In other words, a warrant of arrest issued by the
xxx xxx xxx
Commissioner of Immigration, to be valid, must be for the
sole purpose of executing a final order of deportation. A
3. Deliver the suspect to the Intelligence
warrant of arrest issued by the Commissioner of Immigration
Division and immediately conduct custodial
for purposes of investigation only, as in the case at bar, is
interrogation, after warning the suspect that
null and void for being unconstitutional (Ang Ngo Chiong
he has a right to remain silent and a right to
vs. Galang, 67 SCRA 338 [1975] citing Po Siok Pin vs.
counsel; . . .
Vivo, 62 SCRA 363 [1975]; Vivo vs. Montesa, 24 SCRA
155; Morano vs. Vivo, 20 SCRA 562; Qua Chee Gan vs.
Hence, petitioners' argument that the arrest of respondent
Deportation Board, 9 SCRA 27 [1963]; Ng Hua To vs.
Galang, 10 SCRA 411; see also Santos vs. Commissioner of was based, ostensibly, on the July 6, 1962 warrant of
exclusion has obviously no leg to stand on. The mission
Immigration, 74 SCRA 96 [1976]).
order/warrant of arrest made no mention that the same was
As We held in Qua Chee Gan vs. Deportation Board (supra), issued pursuant to a final order of deportation or warrant of
exclusion.
"(t)he constitution does not distinguish warrants between a
criminal case and administrative proceedings. And if one
But there is one more thing that militates against petitioners'
suspected of having committed a crime is entitled to a
determination of the probable cause against him, by a judge, cause. As records indicate, which petitioners conveniently
why should one suspected of a violation of an administrative omitted to state either in their petition or comment to the
counter-petition of respondent, respondent Gatchalian, along
nature deserve less guarantee?" It is not indispensable that
the alleged alien be arrested for purposes of investigation. If with others previously covered by the 1962 warrant of
exclusion, filed a motion for re-hearing before the Board of
the purpose of the issuance of the warrant of arrest is to
Special Inquiry (BSI) sometime in 1973.
determine the existence of probable cause, surely, it cannot
pass the test of constitutionality for only judges can issue the
On March 14, 1973, the Board of Special Inquiry, after
same (Sec. 2, Art. III, Constitution).
giving due course to the motion for re-hearing, submitted a
memorandum to the then Acting Commissioner Victor
A reading of the mission order/warrant of arrest (dated
Nituda (Annex "5", counter-petition) recommending 1 the
August 15, 1990; Rollo, p. 183, counter-petition) issued by
the Commissioner of Immigration, clearly indicates that the reconsideration of the July 6, 1962 decision of the then
Board of Commissioners which reversed the July 6, 1961
same was issued only for purposes of investigation of the

decision of the then Board of Special Inquiry No. 1 and 2 the


lifting of the warrants of arrest issued against applicants. The
memorandum inferred that the "very basis of the Board of
Commissioners in reversing the decision of the Board of
Special Inquiry was due to a forged cablegram by the then
Secretary of Foreign Affairs, . . ., which was dispatched to
the Philippine Consulate in Hong Kong authorizing the
registration of applicants as P.I. citizens." The Board of
Special Inquiry concluded that "(i)f at all, the cablegram
only led to the issuance of their Certificate(s) of Identity
which took the place of a passport for their authorized travel
to the Philippines. It being so, even if the applicants could
have entered illegally, the mere fact that they are citizens of
the Philippines entitles them to remain in the country."

by the Bureau of Immigration in an Order


citizenship of respondent William Gatchalian that is in issue
dated July 12, 1960. (Annex "37", Comment and addressed for determination of the Court in this case.
with Counter-Petition).
Furthermore, petitioners' position is not enhanced by the fact
Nonetheless, in said order it was found that the applicants
that respondent's arrest came twenty-eight (28) years after
therein have not satisfactorily proven that they are the
the alleged cause of deportation arose. Section 37 (b) of the
children and/or grandchildren of Santiago Gatchalian. The
Immigration Act states that deportation "shall not be effected
status of Santiago Gatchalian as a Filipino was reiterated in . . . unless the arrest in the deportation proceedings is made
Arocha and Arca (supra) where advertence is made to the
within five (5) years after the cause of deportation arises." In
"applicants being the descendants of one Santiago
Lam Shee vs. Bengzon (93 Phil. 1065 [1953]), We laid down
Gatchalian, a Filipino." (at p. 539).
the consequences of such inaction, thus:

There should be no question that Santiago Gatchalian,


grandfather of William Gatchalian, is a Filipino citizen. As a
matter of fact, in the very order of the BOC of July 6, 1962,
which reversed the July 6, 1961 BSI order, it is an accepted
fact that Santiago Gatchalian is a Filipino. The opening
paragraph of said order states:

The dissenting opinions of my esteemed brethrens, Messrs.


Justices F.P. Feliciano and H.G. Davide, Jr., proposing to reopen the question of citizenship of Santiago Gatchalian at
this stage of the case, where it is not even put in issue, is
quite much to late. As stated above, the records of the
Bureau of Immigration show that as of July 20, 1960,
Santiago Gatchalian had been declared to be a Filipino
citizen. It is a final decision that forecloses a re-opening of
the same 30 years later. Petitioners do not even question
Santiago Gatchalian's Philippine citizenship. It is the

In the sworn statement of Santiago Gatchalian before the


Philippine Consul in Hongkong in 1961 (Annex "1" to the
Comment of petitioners to Counter-Petition), he reiterated
his status as a Philippine citizen being the illegitimate child
On March 15, 1973, then Acting Commissioner Nituda
of Pablo Pacheco and Marciana Gatchalian, the latter being a
issued an Order (Annex "6", counter-petition) which
Filipino; that he was born in Manila on July 25, 1905; and
affirmed the Board of Special Inquiry No. 1 decision dated
that he was issued Philippine Passport No. 28160 (PA-No.
July 6, 1961 admitting respondent Gatchalian and others as
A91196) on November 18, 1960 by the Department of
Filipino citizens; recalled the July 6, 1962 warrant of arrest
Foreign Affairs in Manila. In his affidavit of January 23,
and revalidated their Identification Certificates.
1961 (Annex "5", counter-petition), Santiago reiterated his
The above order admitting respondent as a Filipino citizen is claim of Philippine citizenship as a consequence of his
the last official act of the government on the basis of which
petition for cancellation of his alien registry which was
respondent William Gatchalian continually exercised the
granted on February 18, 1960 in C.E.B. No. 3660-L; and that
rights of a Filipino citizen to the present. Consequently, the
on July 20, 1960, he was recognized by the Bureau of
presumption of citizenship lies in favor of respondent
Immigration as a Filipino and was issued Certificate No. 1William Gatchalian.
2123.

The claim to Filipino citizenship of


abovenamed applicants is based on the
citizenship of one Santiago Gatchalian
whose Philippine citizenship was recognized

There is however an important circumstance


which places this case beyond the reach of
the resultant consequence of the fraudulent
act committed by the mother of the minor
when she admitted that she gained entrance
into the Philippines by making use of the
name of a Chinese resident merchant other
than that of her lawful husband, and that is,
that the mother can no longer be the subject
of deportation proceedings for the simple
reason that more than 5 years had elapsed
from the date of her admission. Note that the
above irregularity was divulged by the
mother herself, who in a gesture of sincerity,
made an spontaneous admission before the
immigration officials in the investigation
conducted in connection with the landing of
the minor on September 24, 1947, and not
through any effort on the part of the
immigration authorities. And considering
this frank admission, plus the fact that the
mother was found to be married to another
Chinese resident merchant, now deceased,
who owned a restaurant in the Philippines
valued at P15,000 and which gives a net
profit of P500 a month, the immigration
officials then must have considered the
irregularity not serious enough when, inspire

of that finding, they decided to land said


minor "as a properly documented preference
quota immigrant" (Exhibit D). We cannot
therefore but wonder why two years later the
immigration officials would reverse their
attitude and would take steps to institute
deportation proceedings against the minor.
Under the circumstances obtaining in this
case, we believe that much as the attitude of
the mother would be condemned for having
made use of an improper means to gain
entrance into the Philippines and acquire
permanent residence there, it is now too
late, not to say unchristian, to deport the
minor after having allowed the mother to
remain even illegally to the extent of
validating her residence by inaction, thus
allowing the period of prescription to set in
and to elapse in her favor. To permit his
deportation at this late hour would be to
condemn him to live separately from his
mother through no fault of his thereby
leaving him to a life of insecurity resulting
from lack of support and protection of his
family. This inaction or oversight on the part
of immigration officials has created an
anomalous situation which, for reasons of
equity, should be resolved in favor of the
minor herein involved. (Emphasis supplied)
In the case at bar, petitioners' alleged cause of action and
deportation against herein respondent arose in 1962.
However, the warrant of arrest of respondent was issued by
Commissioner Domingo only on August 15, 1990 28 long
years after. It is clear that petitioners' cause of action has
already prescribed and by their inaction could not now be
validly enforced by petitioners against respondent William
Gatchalian. Furthermore, the warrant of exclusion dated July
6, 1962 was already recalled and the Identification certificate

of respondent, among others, was revalidated on March 15,


1973 by the then Acting Commissioner Nituda.

(c) Obtains, accepts or uses any immigration


document, knowing it to be false; or

It is also proposed in the dissenting opinions of Messrs.


Justices Feliciano and Davide, Jr., that the BOC decision
dated July 6, 1962 and the warrant of exclusion which was
found to be valid in Arocha should be applicable to
respondent William Gatchalian even if the latter was not a
party to said case. They also opined that under Sec. 37 (b) of
the Immigration Act, the five (5) years limitation is
applicable only where the deportation is sought to be
effected under clauses of Sec. 37 (b) other than clauses 2, 7,
8, 11 and 12 and that no period of limitation is applicable in
deportations under clauses 2, 7, 8, 11 and 12.

(d) Being an alien, enters the Philippines


without inspection and admission by the
immigration officials, or obtains entry into
the Philippines by wilful, false, or
misleading representation or wilful
concealment of a material fact; or

The Court disagrees. Under Sec. 39 of the Immigration Act,


it is reiterated that such deportation proceedings should be
instituted within five (5) years. Section 45 of the same Act
provides penal sanctions for violations of the offenses
therein enumerated with a fine of "not more than P1,000.00
and imprisonment for not more than two (2) years and
deportation if he is an alien." Thus:

(f) In any immigration matter shall


knowingly make under oath any false
statement or representations; or

(e) Being an alien shall for any fraudulent


purpose represent himself to be a Philippine
citizen in order to evade any requirement of
the immigration laws; or

(g) Being an alien, shall depart from the


Philippines without first securing an
immigration clearance certificates required
by section twenty-two of this Act; or

Penal Provisions
Sec. 45. Any individual who
(a) When applying for an immigration
document personates another individual, or
falsely appears in the name of deceased
individual, or evades the immigration laws
by appearing under an assumed name;
fictitious name; or

(h) Attempts or conspires with another to


commit any of the foregoing acts, shall be
guilty of an offense, and upon conviction
thereof, shall be fined not more than one
thousand pesos, and imprisoned for not
more than two years, and deported if he is
an alien. (Emphasis supplied)
Such offenses punishable by correctional penalty prescribe
in 10 years (Art. 90, Revised Penal Code); correctional
penalties also prescribe in 10 years (Art. 92, Revised Penal
Code).

(b) Issues or otherwise disposes of an


immigration document, to any person not
authorized by law to receive such document;
It must be noted, however, that under Sec. 1, Act No. 3326
or
[1926], as amended, (Prescription for Violations Penalized
by Special Acts and Municipal Ordinances) "violations

penalized by special acts shall, unless otherwise provided in


such acts, prescribe in accordance with the following
rules: . . .c) after eight years for those punished by
imprisonment for two years or more, but less than six years; .
. ."

In relation to Sec. 37 (b) of the Immigration Act, the rule,


therefore, is:

1. Deportation or exclusion proceedings should be initiated


within five (5) years after the cause of deportation or
exclusion arises when effected under any other clauses other
Consequently, no prosecution and consequent deportation for than clauses 2, 7, 8, 11 and 12 and of paragraph (a) of Sec.
violation of the offenses enumerated in the Immigration Act 37 of the Immigration Act; and
can be initiated beyond the eight-year prescriptive period,
2. When deportation or exclusion is effected under clauses 2,
the Immigration Act being a special legislation.
7, 8, 11 and 12 of paragraph (a) of Sec. 37, the prescriptive
The Court, therefore, holds that the period of effecting
period of the deportation or exclusion proceedings is eight
deportation of an alien after entry or a warrant of exclusion
(8) years.
based on a final order of the BSI or BOC are not
In the case at bar, it took petitioners 28 years since the BOC
imprescriptible. The law itself provides for a period of
prescription. Prescription of the crime is forfeiture or loss of decision was rendered on July 6, 1962 before they
commenced deportation or exclusion proceedings against
the rights of the State to prosecute the offender after the
respondent William Gatchalian in 1990. Undoubtedly,
lapse of a certain time, while prescription of the penalty is
petitioners' cause of action has already prescribed. Neither
the loss or forfeiture by the government of the right to
may an action to revive and/or enforce the decision dated
execute the final sentence after the lapse of a certain time
July 6, 1962 be instituted after ten (10) years (Art. 1144 [3],
(Padilla, Criminal Law, Vol. 1, 1974, at p. 855).
Civil Code).
"Although a deportation proceeding does not partake of the
nature of a criminal action, however, considering that it is a Since his admission as a Filipino citizen in 1961, respondent
harsh and extraordinary administrative proceeding affecting William Gatchalian has continuously resided in the
Philippines. He married Ting Dee Hua on July 1, 1973
the freedom and liberty of a person, the constitutional right
(Annex "8", counter-petition) with whom he has four (4)
of such person to due process should not be denied. Thus,
minor children. The marriage contract shows that said
the provisions of the Rules of Court of the Philippines
respondent is a Filipino (Annex "8"). He holds passports and
particularly on criminal procedure are applicable to
earlier passports as a Filipino (Annexes "9", "10" & "11",
deportation proceedings." (Lao Gi vs. Court of Appeals,
counter-petition). He is a registered voter of Valenzuela,
supra). Under Sec. 6, Rule 39 of the Rules of Court, a final
Metro Manila where he has long resided and exercised his
judgment may not be executed after the lapse of five (5)
right of suffrage (Annex 12, counter-petition). He engaged in
years from the date of its entry or from the date it becomes
final and executory. Thereafter, it may be enforced only by a business in the Philippines since 1973 and is the
director/officer of the International Polymer Corp. and
separate action subject to the statute of limitations. Under
Art. 1144 (3) of the Civil Code, an action based on judgment Ropeman International Corp. as a Filipino (Annexes, "13" &
"14", counter-petition). He is a taxpayer. Respondent claims
must be brought within 10 years from the time the right of
that the companies he runs and in which he has a controlling
action accrues.
investment provides livelihood to 4,000 employees and
approximately 25,000 dependents. He continuously enjoyed

the status of Filipino citizenship and discharged his


responsibility as such until petitioners initiated the
deportation proceedings against him.
"The power to deport an alien is an act of the State. It is an
act by or under the authority of the sovereign power. It is a
police measure against undesirable aliens whose presence in
the country is found to be injurious to the public good and
domestic tranquility of the people" (Lao Gi vs. Court of
Appeals, supra). How could one who has helped the
economy of the country by providing employment to some
4,000 people be considered undesirable and be summarily
deported when the government, in its concerted drive to
attract foreign investors, grants Special Resident Visa to any
alien who invest at least US$50,000.00 in the country? Even
assuming arguendo that respondent is an alien, his
deportation under the circumstances is unjust and unfair, if
not downright illegal. The action taken by petitioners in the
case at bar is diametrically opposed to settled government
policy.
Petitioners, on the other hand, claim that respondent is an
alien. In support of their position, petitioners point out that
Santiago Gatchalian's marriage with Chu Gim Tee in China
as well as the marriage of Francisco (father of William)
Gatchalian to Ong Chiu Kiok, likewise in China, were not
supported by any evidence other than their own self-serving
testimony nor was there any showing what the laws of China
were. It is the postulate advanced by petitioners that for the
said marriages to be valid in this country, it should have been
shown that they were valid by the laws of China wherein the
same were contracted. There being none, petitioners
conclude that the aforesaid marriages cannot be considered
valid. Hence, Santiago's children, including Francisco,
followed the citizenship of their mother, having been born
outside of a valid marriage. Similarly, the validity of the
Francisco's marriage not having been demonstrated, William
and Johnson followed the citizenship of their mother, a
Chinese national.

After a careful consideration of petitioner's argument, We


find that it cannot be sustained.

Consequently, the testimonies/affidavits of Santiago


Finally, respondent William Gatchalian belongs to the class
Gatchalian and Francisco Gatchalian aforementioned are not of Filipino citizens contemplated under Sec. 1, Article IV of
self-serving but are competent proof of filiation (Art. 172
the Constitution, which provides:
In Miciano vs. Brimo (50 Phil. 867 [1924]; Lim and Lim vs. [2], Family Code).
Collector of Customs, 36 Phil. 472; Yam Ka Lim vs.
Sec. 1. The following are citizens of the
Collector of Customs, 30 Phil. 46 [1915]), this Court held
Philippine law, following the lex loci celebrationis, adheres
Philippines:
that in the absence of evidence to the contrary, foreign laws to the rule that a marriage formally valid where celebrated is
(1) Those who are citizens of the Philippines
on a particular subject are presumed to be the same as those valid everywhere. Referring to marriages contracted abroad,
at the time of the adoption of this
of the Philippines. In the case at bar, there being no proof of Art. 71 of the Civil Code (now Art. 26 of the Family Code)
Constitution. . . .
Chinese law relating to marriage, there arises the
provides that "(a)ll marriages performed outside of the
presumption that it is the same as that of Philippine law.
Philippines in accordance with the laws in force in the
country where they were performed, and valid there as such, This forecloses any further question about the Philippine
citizenship of respondent William Gatchalian.
The lack of proof of Chinese law on the matter cannot be
shall also be valid in this country . . ." And any doubt as to
blamed on Santiago Gatchalian much more on respondent
the validity of the matrimonial unity and the extent as to how
The Court is not unaware of Woong Woo Yiu vs. Vivo (13
William Gatchalian who was then a twelve-year old minor.
far the validity of such marriage may be extended to the
The fact is, as records indicate, Santiago was not pressed by consequences of the coverture is answered by Art. 220 of the SCRA 552 [1965]) relied upon by petitioners. The ruling
arrived thereat, however, cannot apply in the case at bar for
the Citizenship Investigation Board to prove the laws of
Civil Code in this manner: "In case of doubt, all
China relating to marriage, having been content with the
presumptions favor the solidarity of the family. Thus, every the simple reason that the parties therein testified to have
been married in China by a village leader, which
testimony of Santiago that the Marriage Certificate was lost intendment of law or facts leans toward the validity of
undoubtedly is not among those authorized to solemnize
or destroyed during the Japanese occupation of China.
marriage, the indissolubility of the marriage bonds, the
marriage as provided in Art. 56 of the Civil Code (now Art.
Neither was Francisco Gatchalian's testimony subjected to
legitimacy of children, the community of property during
the same scrutiny by the Board of Special Inquiry.
marriage, the authority of parents over their children, and the 7, Family Code).
Nevertheless, the testimonies of Santiago Gatchalian and
validity of defense for any member of the family in case of
Francisco Gatchalian before the Philippine consular and
unlawful aggression." (Emphasis supplied). Bearing in mind Premises considered, the Court deems it unnecessary to
resolve the other issues raised by the parties.
immigration authorities regarding their marriages, birth and the "processual presumption" enunciated in Miciano and
relationship to each other are not self-serving but are
other cases, he who asserts that the marriage is not valid
WHEREFORE, G.R. Nos. 95122-23 is DISMISSED for lack
admissible in evidence as statements or declarations
under our law bears the burden of proof to present the
of merit; G.R. Nos. 95612-13 is hereby GRANTED and
regarding family reputation or tradition in matters of
foreign law.
respondent William Gatchalian is declared a Filipino citizen.
pedigree (Sec. 34, Rule 130). Furtheremore, this salutary
rule of evidence finds support in substantive law. Thus, Art. Having declared the assailed marriages as valid, respondent Petitioners are hereby permanently enjoined from continuing
with the deportation proceedings docketed as DC No. 90-523
267 of the Civil Code provides:
William Gatchalian follows the citizenship of his father
for lack of jurisdiction over respondent Gatchalian, he being
Francisco, a Filipino, as a legitimate child of the latter.
Art. 267. In the absence of a record of birth, Francisco, in turn is likewise a Filipino being the legitimate a Filipino citizen; Civil Cases No. 90-54214 and 3431-V-90
authentic document, final judgment or
child of Santiago Gatchalian who (the latter) is admittedly a pending before respondent judges are likewise DISMISSED.
possession of status, legitimate filiation may Filipino citizen whose Philippine citizenship was recognized Without pronouncement as to costs.
be proved by any other means allowed by
by the Bureau of Immigration in an order dated July 12,
SO ORDERED.
the Rules of Court and special laws. (See
1960.
also Art. 172 of the Family Code)

PEOPLE VS. SUCRO [195 SCRA 388; G.R. No. 93239;


PROBABLE CAUSE existed. The police officers have
18 Mar 1991]
personal knowledge of the actual commission of the crime
Wednesday, February 04, 2009 Posted by Coffeeholic Writes from the surveillance of the activities of the accused. As
police officers were the ones conducting the surveillance, it
is presumed that they are regularly in performance of their
Labels: Case Digests, Political Law
duties.
Facts: Pat. Fulgencio went to Arlie Regalados house at C.
Quimpo to monitor activities of Edison SUCRO (accused). ** People v Sucro 195 SCRA 388 (1991)
Sucro was reported to be selling marijuana at a chapel 2
meters away from Regalados house. Sucro was monitored to Facts: Pat. Fulgencio went to Arlie Regalados house at C.
have talked and exchanged things three times. These Quimpo to monitor activities of Edison SUCRO (accused).
activities are reported through radio to P/Lt. Seraspi. A third Sucro was reported to be selling marijuana at a chapel 2
buyer was transacting with appellant and was reported and
meters away from Regalados house. Sucro was monitored to
later identified as Ronnie Macabante. From that moment,
P/Lt.Seraspi proceeded to the area. While the police officers have talked and exchanged things three times. These
were at the Youth Hostel in Maagama St. Fulgencio told Lt. activities are reported through radio to P/Lt. Seraspi. A third
Seraspi to intercept. Macabante was intercepted at Mabini buyer was transacting with appellant and was reported and
and Maagama crossing in front of Aklan Medical center. later identified as Ronnie Macabante. From that moment,
Macabante saw the police and threw a tea bag of marijuana P/Lt.Seraspi proceeded to the area. While the police officers
on the ground. Macabante admitted buying the marijuana were at the Youth Hostel in Maagama St. Fulgencio told Lt.
from
Sucro
in
front
of
the
chapel.
Seraspi to intercept. Macabante was intercepted at Mabini
The police team intercepted and arrested SUCRO at the and Maagama crossing in front of Aklan Medical center.
corner of C. Quimpo and Veterans. Recovered were 19 sticks Macabante saw the police and threw a tea bag of marijuana
and 4 teabags of marijuana from a cart inside the chapel and on the ground. Macabante admitted buying the marijuana
another
teabag
from
Macabante.
from Sucro in front of the chapel.
Issues:
(1) Whether or Not arrest without warrant is lawful.
(2) Whether or Not evidence from such arrest is admissible.
Held: Search and seizures supported by a valid warrant of
arrest is not an absolute rule. Rule 126, Sec 12 of Rules of
Criminal Procedure provides that a person lawfully arrested
may be searched for dangerous weapons or anything, which
may be used as proff of the commission of an offense,
without a search warrant.(People v. Castiller) The failure of
the police officers to secure a warrant stems from the fact
that their knowledge required from the surveillance was
insufficient to fulfill requirements for its issuance. However,
warantless search and seizures are legal as long as

without a search warrant.(People v. Castiller) The failure of


the police officers to secure a warrant stems from the fact
that their knowledge required from the surveillance was
insufficient to fulfill requirements for its issuance. However,
warantless search and seizures are legal as long as
PROBABLE CAUSE existed. The police officers have
personal knowledge of the actual commission of the crime
from the surveillance of the activities of the accused. As
police officers were the ones conducting the surveillance, it
is presumed that they are regularly in performance of their
duties.

PEOPLE VS. SOLAYAOGR 119220, SEPT. 20, 1996,


SECOND DIVISIONJUSTICE ROMEROFACTS:
Nilo Solayao was charged before the Regional Trial Court of
Naval, Biliran, Branch 16, with the crime of illegal
possession of firearm and ammunition

defined and penalized under Presidential Decree No. 1866.SPO3 Jose Nio stated that he and other operatives went to
Barangay Caulangohan, Caibiran, Biliran.They were to
conduct an intelligence patrol as required of them by their
intelligence officer to verifyreports on the presence of armed
The police team intercepted and arrested SUCRO at the
persons roaming around the barangays of Caibiran.- The
corner of C. Quimpo and Veterans. Recovered were 19 sticks
team of Police Officer Nio proceeded to Barangay Onion
and 4 teabags of marijuana from a cart inside the chapel and where they met the group of accused-appellant Nilo Solayao
another teabag from Macabante.
numbering five. The former became suspicious when they
observed that thelatter were drunk and that Solayao was
wearing a camouflage uniform or a jungle suit.
Issues:
Solayaoscompanions, upon seeing the government agents,
fled.- Confiscated from Solayao is a homemade firearm
(1) Whether or Not arrest without warrant is lawful.
called Latong. Solayao admitted that he had nopermission to
possess the same. Thereupon, SPO3 Nio confiscated the
(2) Whether or Not evidence from such arrest is admissible. firearm and turned him over tothe custody of the policeman
of Caibiran who subsequently investigated him and charged
Held: Search and seizures supported by a valid warrant of
him withillegal possession of firearm.- Solayao was found
guilty, then he appealed to the court against the admissibility
arrest is not an absolute rule. Rule 126, Sec 12 of Rules of
Criminal Procedure provides that a person lawfully arrested of the firearm asevidence as it was the product of an
may be searched for dangerous weapons or anything, which unlawful warrantless search.
ISSUE:
may be used as proff of the commission of an offense,

WON
Sa dahilang napatunayan ng Hukuman na
t
sina Danilo Punzalan, Vget Ison at Guilbert
he trial court erred in admitting in evidence the homemade
Cuison ay nagkasala sa salang sinadyang
firearm
pagpatay, ayon sa Articulo 248 ng Binagong
.
Kodigo Penal sila ay pinapatawan ng
HELD:
parusang KAMATAYAN. Magbabayad sila
Firearm is admissible as evidence.
ng danyos perwisyong P12,000.00 sa mga
RATIO:
naulila ng nasawi at P5,000.00 bayad
The case at bar constitutes an instance where a search and
pinsalang moral at P5,000.00 bayad
seizure may be effected without firstmaking an arrest. There
was justifiable cause to "stop and frisk" accused-appellant
pinsalang di dapat pamarisan at hati-hati sila
when his companionsfled upon seeing the government
sa gastos ng usapin.
agents. Under the circumstances, the government agents
could notpossibly have procured a search warrant first.
IPINAGUUTOS. 1
Suspicion also arouse when the group was spotteddressed in
camouflage.Also the officers were precisely in the area to
conduct an operation to verify the intelligencereport and stop The original information, dated July 19, 1978 and filed wih
the trial court on July 27, 1978 charged only three (3)
any potential disturbance in the area.
accused, namely, Ramon Jumawan, Danilo Punzalan and
G.R. No. L-54562 August 6, 1987
Vget Ison, with murder qualified by treachery, abuse of
superior strength and cruelty. About four (4) months later, on
THE PEOPLE OF THE PHILIPPINES, plaintiffNovember 21, 1978, the information was amended to
appellee,
include Guilbert Cuison as a fourth accused as a result of a
vs.
preliminary investigation conducted by the Fiscal pursuant to
DANILO PUNZALAN, VGET ISON and GUILBERT
the Order of the trial court dated August 11, 1978 issued
CUISON, defendants-appellants.
upon motion of the private prosecutor on the strength of a
supplementary sworn statement of one Arsenio Telmo dated
August 9, 1978 (Exh. 1) claiming that Guilbert Cuison was
GANCAYCO, J.:
one of those he saw hitting the late Cpl. Maximo de los
Santos with a chair.
This is an automatic review of the decision dated September
7, 1979, rendered by Hon. Judge Onofre L. Vinaluz, Circuit Upon arraignment, the three (3) accused, Danilo Punzalan,
Criminal Court, Pasig, Metro Manila in Criminal Case No.
Vget Ison and Guilbert Cuison pleaded not guilty to the
CCC-VII-2639-Rizal, convicting all three (3) accusedcharge and thereafter entered into trial. The other accused,
appellants of the crime of murder as charged in the
Ramon Jumawan, was never arrested and remained at large.
information and imposing upon them the death penalty for
After trial, the lower court returned a verdict of guilty and
the fatal shooting of Cpl. Maximo de los Santos of the
sentenced the three (3) accused to death as earlier stated.
Paraaque police force in Paranaque, Metro Manila, on June
11, 1978. The dispositive portion of the decision in the
As set forth in the People's brief, the facts attendant to the
vernacular reads in full as follows:
commission of the crime charged are as follows:

In the early evening of June 11, 1978, Cpl.


Maximo delos Santos of the Paraaque
Police Department, was inside the Nordel
Restaurant in Sucat Road, Paranaque, Metro
Manila. While he was there, a group of men
composed of Ramon Jumawan, who remains
at large, Danilo Punzalan, Vget Ison,
Guilbert Cuison, Reynaldo Dandan,
Angelito Bernardo, Cesar Llamas, Wally or
Walfrido Punzalan, Benjamin Arciaga and
Leon Allan: gue occupied a round table at
the restaurant drinking beer. Sometime later,
Cpl. delos Santos approached the group
saying: "Mga padres, pulis ako, ibig ko lang
alamin sa inyo kung sino ang may baril."
Whereupon, Wally Punzalan who was seated
across the table, pulled out a .45 cal. pistol
which his brother, Danilo, had earlier
slipped to him under the table. Cpl. delos
Santos drew his service revolver and fired
twice at Wally Punzalan before the latter
could fire at him. Accused Ramon Jumawan
then hit the policeman on the head with a
chair. The policeman fell to the floor, face
down. Appellants Danilo Punzalan and
Guilbert Cuison followed suit hitting Cpl.
delos Santos repeatedly with chairs on the
head, back and arms. Appellant Vget Ison
joined in and hit their victim with a beer
bottle (Tsn, Jan. 10, 1979, pp. 6-20; Tsn,
July 19, 1979, pp. 14-16; Exhs. E, F). The
attack was so violent that the chairs and beer
bottles which appellants used in hitting the
policeman were broken. The broken chairs
and fragments of broken glass were strewn
on the floor (Tsn, Dec. 7, 1978, pp. 67-76;
Exhs. C to C-16).
As the policeman lay prostrate and helpless
on the floor, Ramon Jumawan grabbed the

victim's service revolver and shot the latter


four (4) times. All shots found their mark
(Tsn, Jan. 10, 1979, pp. 16.17; Tsn, Dec.
7,1978, pp. 28-33; Exh. A).
After the group saw that Cpl. Santos was
already dead the group left together (Tsn,
Jan. 10, 1979, p. 19). They helped each
other in bringing the wounded Wally
Punzalan to the hospital where he expired
five days later (Tsn, July 17, 1979, pp. 7578, 151-152). The jacket of Danilo Punzalan
was left on the floor (Tsn, Dec. 7, 1978, pp.
80-81; Exh. C-4. 2
Three separate briefs by different lawyers were filed for the
appellants: the first for all the appellants; the second for
appellants Vget Ison and Guilbert Cuison; and the third, for
appellant Vget Ison only. The defense advanced the
following version of the incident:
The undisputed facts are that at about 9:30
P.M. June 11, 1978, a group of ten (10) male
persons was * at the Nordel Restaurant, Dr.
Santos Street, Sucat, Paraaque, Metro
Manila, for food and beer. The group
included the four accused, Danilo Punzalan,
Ramon Jumawan Vget Ison, and Guilbert
Cuison. Wally Punzalan, brother of Danilo
Punzalan was also there. The owner of the
restaurant, Adelaida Borinaga, was also
present as was one Elizabeth Mendoza,
waitress, who waited on the group. The ten
persons seated themselves about a round
table at the center of the eating place as they
partook of the food and drinks.
Cpl. Maximo de los Santos, a traffic
policeman of Paranaque, was also in the
restaurant, in civilian clothes, drinking beer,

seated at the counter some five meters away


from the table of the group. He ordered a
bottle of beer from Adelaida Borinaga.

With that backdrop, We shall now discuss the various


assignments of errors raised by the defense which are
summarized as follows:

All went well until Cpl. de los Santos


suddenly left his place at the counter, went
to the table occupied by the accusedappellants and their companions, and with
his .38 caliber pistol shot Wally Punzalan
twice at close range felling the latter. In the
ensuing rapid succession of action Ramon
Jumawan hit Cpl. de los Santos on the head
with a chair and gained possession of the
policeman's gun and used it in shooting the
policeman in turn three or four times.

1. The trial court erred in finding the


existence of treachery and conspiracy
among the accused;

The policeman sustained four gunshot


wounds three of which caused his death. The
fourth one was a mere grazing wound. He
also had a lacerated wound on the head and
other wounds and some contusions and
abrasions. He died on the spot where he fell.
The necropsy report (Exhibit A) gave the
cause of death as severe hemorrhage
secondary to gunshot wounds. He was
brought to the Olivares General Hospital
nearby where he expired six days later, on
June 17, 1978, from peritonitis generalized,
secondary to gunshot wounds according to
the necropsy report (Exhibit 6). 3

4. The trial court erred in failing to acquit all


the defendants for failure of the prosecution
to establish their guilt beyond reasonable
doubt.

It appears from the foregoing narration that contrary to the


trial court's findings that the gunplay was started by one of
the members of the group. ("isa sa mga nakaupo ang
nagbunot ng .45 cal. at binaril ang pulis ... ) what actually
transpired was that it was the policeman (victim) who first
shot Wally Punzalan.

2. The trial court erred in not crediting the


defendants with the justifying circumstance
of self-defense or defense of relative;
3. The trial court erred in giving credence to
the testimony of the prosecution witness,
Arsenio Telmo;

Anent the first assigned error, appellants maintain that the


trial court erred in finding the existence of conspiracy and
treachery among them.
As provided in paragraph 2, Article 8 of the Revised Penal
Code, conspiracy exists when two or more persons come to
an agreement concerning the commission of a felony and
decide to commit it. Since in conspiracy, the act of one
becomes the act of all, all persons taking part in the crime
are held guilty as principals. Because of its far-reaching
consequences, the same degree of proof required for
establishing the crime is required to support a finding of the
presence of conspiracy. Thus, it must be shown to exist as
clearly and convincingly as the commission of the offense
itself, 4 in order to uphold the fundamental principle that no
one shall be found guilty of a crime except upon proof
beyond reasonable doubt. 5

While it may be true that direct proof is not essential to


prove conspiracy, 6 and it may be shown by acts and
circumstances from which may logically be inferred the
existence of a common design among the accused to commit
the offense charged, nonetheless, the evidence to prove the
same must be positive and convincing. 7

The fact that the appellants struck and hit the victim with the
chairs and beer bottles did not make them conspirators
thereby. The evidence shows that appellants acted on
impulse, without prior deliberation, planning or design. The
appellants must have felt threatened by the deceased who
killed their companion. Hence their spontaneous response to
the real and determined acts of the victim must have been
A close scrutiny of the records in the present case shows that motivated by a desire to repel the aggression if not to prevent
there is not a shred of evidence to establish the existence of any further harm the victim may inflict on any of them.
conspiracy between appellants. From the evidence presented
by the prosecution it is clear that the incident at the Nordel
The simultaneous attack of appellants cannot be considered
Restaurant on June 11, 1978 was unplanned and
as amounting to a conspiracy to kill the deceased. Neither
spontaneous.
joint and simultaneous action nor relationship is per se a
sufficient indicium of conspiracy. 10 A common design must
First, admittedly, appellants were together drinking and
further be shown to have motivated such action. 11 Indeed, it
eating in the company of other persons before the incident
is the gunshot wounds that led to the death of the victim. The
happened. This circumstance alone does not suffice to prove injuries inflicted by appellants were superficial to serious but
the existence of a common criminal design. Mere suspicion, not fatal. 12 In the absence of clear proof that the killing was
speculation, relationship or association and companionship
in fact envisaged by them, they cannot be held responsible
do not establish conspiracy, for proof thereof must be
for the death of the victim.
positive and convincing. 8 In the instant case, it is neither
alleged nor shown that appellants' meeting at the Nordel
With respect to treachery, the attendance of this aggravating
Restaurant at that particular time was planned. Nor is there
circumstance is found in the concurrence of two conditions:
evidence that any of the appellants knew that the victim was (1) the employment of means, method or manner of
going to be in that particular place or that such meeting was execution which would insure the offender's safety from any
purposely sought for by them. No doubt the encounter was
defensive or retaliatory act on the part of the offended party,
purely accidental. In fact, had the victim not approached the which means that no opportunity is given to the latter to do
appellants, no untoward incident could have happened.
so, 13 and (2) that such means, method or manner of
execution was deliberately or consciously chosen. 14 Thus, it
Second, it appears that it was the deceased Maximo de los
is not enough that the means, method or form of execution
Santos who approached the table of appellants and was the
tends directly and specially to facilitate the commission of
first to fire the two shots at Wally Punzalan, when he
the offense without danger to the offender arising from the
attempted to draw his gun. The fact that the attack was
defense or retaliation that might be made by the offended
commenced by Maximo de los Santos negates the theory that party. It is further required, for treachery to be appreciated,
appellants conspired to kill him. lt was only after Wally
that such means, method or form was deliberated upon or
Punzalan was gunned down that appellants reacted and
consciously adopted by the offender. 15
ganged up on the victim. Ramon Jumawan hit the victim at
the back so his gun fell. Jumawan then picked up the gun
The first of the two conditions aforestated appears to be
9
which he used in killing the victim.
present in the instant case. It has been adequately established
that appellants delivered blows to the victim when the latter

was down on the floor after being struck by Ramon


Jumawan which fact insured their safety from any
defensive or retaliatory act from the victim. However, the
second condition is wanting.
From the rapid succession of events, it appears that the
means employed were not deliberated upon by appellants.
The act of the appellants in assaulting the victim appears to
be impelled by the instinct of self-preservation if not
retaliation for the shooting of their comrade. Such deliberate
or conscious choice of the means was held non-existent
where, as in this case, the attack was the product of an
impulse of the moment, 16 and where the defendants did not
make any preparation to assault the victim. 17 The decision
to attack the victim was accidental. 18 The attack arose from
a chance encounter and quarrel. 19 Treachery therefore is not
present in this case.
Anent the second assigned error, appellants maintain that the
trial court erred in not crediting the appellants with the
justifying circumstance of self-defense or defense of relative.
Self-defense 20 and defense of the rights of another 21 are
recognized circumstances justifying an offense and
exempting the perpetrator from criminal liability.
It appears that all the elements of self-defense and defense of
a stranger are present under the facts and circumstances of
the case at bar.
It was shown that the policeman was the aggressor. The
assault was sudden and totally unexpected so that he
succeeded in pumping two bullets into the chest and stomach
of his unsuspecting victim Wally Punzalan, giving the latter
no chance or time to defend himself or evade the aggression.
After gunning down Wally Punzalan, the policeman's
aggression did not stop there. While the policeman stood
there menacingly with his gun, the peril to the life of Wally
Punzalan and all his companions continued. Faced with this

real and imminent danger to their own life, and in obedience


to the dictates of the instinct of self- preservation, appellants
together with Ramon Jumawan, mauled the policeman to
immobilize him. And when the latter was disarmed,
Jumawan picked the gun and shot him four times. Although
this Court cannot in all reason condone the shooting of a
person who is already helpless and lying prostrate on the
floor, so that Jumawan who is at large must be held to
account for his acts, as to the appellants, this Court is
mindful of the following discourse it made on the subject,
thus:
The law does not require, and it would be
too much to ask of the ordinary man, that
when he is defending himself from a deadly
assault, in the heat of an encounter at close
quarters, he should so mete out his blows
that upon a calm and deliberate review of
the incident, it will not appear that he
exceeded the precise limits of what was
absolutely necessary to put his antagonist
hors de combat; or that he struck one blow
more than was absolutely necessary to save
his own life; or that he failed to hold his
hand so as to avoid inflicting a fatal wound
where a less severe stroke might have served
his purpose. Of course, the victim of an
unlawful aggression may not lawfully
exceed the bounds of rational necessity in
repelling the assault. But the measure of
rational necessity in cases of this kind is to
be found in the situation as it appears to the
victim of the assault at the time when the
blow is struck; and the courts should not and
will not, in the light of after events or fuller
knowledge, hold the victims of such deadly
assaults at close quarters, to so strict a
degree of accountability that they will
hesitate to put forth their utmost effort in

their own defense when that seems to them


to be reasonably necessary. 22
The reasonableness of the means employed
to prevent an aggression depends upon the
nature and quality of the weapon used by the
aggressor, his physical condition, his size,
his character and the surrounding
circumstances vis-a-vis those of the person
defending himself. It is also well-settled that
in emergencies which imperil the life and
limb of a person, human nature acts not
upon processes of formal reason but in
obedience to the imperious dictates of the
instinct of self-preservation ... the protective
mantle of the law shields not only him who
repels actual aggression but as well as him
who prevents an aggression that is real and
imminent. And the killing of the aggressor
would be justified at a time when all the
elements of self-defense are present. 23

that the companions of the slain Wally Punzalan were not


motivated by revenge, resentment or other evil motive in
defending themselves and Wally Punzalan.
The appellants having acted in self-defense and/or in defense
of their companion, acted justifiably and are hereby exempt
from criminal liability.
As to the third assigned error, the testimony of prosecution
witness Arsenio Telmo is sought to be discredited because he
gave his statement implicating appellants to the police only
four (4) days after the incident.

The initial reluctance of witnesses in this country to


volunteer information about a criminal case and their
unwillingness to be involved in or dragged into a criminal
investigation is common and has been judicially declared not
to affect credibility. 25 Telmo, a witness to the incident, was
actually asked to go to the police headquarters on the night
of June 11, 1978 but he refrained because he was scared.
However, he readily gave his name, address and residence
certificate to the police. There is nothing in the record that
would prove he had an ax to grind against appellants. He is
In emergencies of this kind, human nature
does not act upon processes of formal reason therefore a disinterested witness. Also, his presence at the
night of the incident was confirmed by the restaurant owner.
but in obedience to the instinct of self26
preservation, and when it is apparent that a
person has reasonably acted upon this
The fact that it was only on August 9, 1978 that Telmo
instinct, it is the duty of the courts to
executed a supplemental affidavit implicating Guilbert
sanction the act and to hold the actor
24
Cuison for the first time is explained by the fact that it was
irresponsible in law for the consequences.
only on that date that he was made to confront Cuison when
the latter gave himself up to the authorities at Camp Crame.
Except for the shooting of the victim by Jumawan, We find
On that occasion, Telmo readily Identified Cuison as one of
that the appellants employed reasonable means to repel the
those who hit the victim. 27
aggression of the policeman.
The final requisite for complete self-defense, lack of
sufficient provocation of the person defending himself, is
also present. The evidence shows that Wally Punzalan and
his companions had not done anything to invite the ire of the
policeman and provoke the latter's aggression. It also appears

Anent the last assigned error, from our view of the evidence,
the participation of herein appellants in mauling the
policeman cannot be denied. However, as earlier discussed,
their acts were justified.

From the evidence on record, this Court is convinced that


herein appellants did not intend to kill the victim; that the
circumstances where they found themselves provoked the
action they took; their friend dead, shot by another who
happened to be a policeman, who at that time was in plain
clothes but was holding a gun. It is easy to imagine and feel
the intense passion, fear and apprehension in everyone's
heart at a time like that. In appellants' eyes, he was a
common enemy, killer of their friend and a possible threat to
their lives. Accused appellants cannot therefore be faulted
since self- preservation is still the first law of nature.

absence of a search warrant.Issue:Was the search unlawful?Held:Yes.


The search n the house of the appellant isillegal because of absence of
search warrant and evenprobable cause for the issuance of the same.In
the testimony of the witness by the prosecution,he has no personal
knowledge making its testimonieshearsay and weak to establish the
existence of a probablecause.Moreover, the search conducted is not one
beingqualified as a valid search without warrant(
1
) Search incidental to an arrest.(2) Customs search.(3) Consented
search(4) Search of a moving vehicle(5) Stop frisk(6) Plain view
search.
(
A
pply the
Considering the above disquisitions, this Court is of the
M
opinion that appellants should be acquitted of the charges
endoza doctrine- incidental discovery of incri
against them.
m
inating evidence to qualify as a plain view search
WHEREFORE, the judgment of conviction is hereby set
)Moreover, the rights granted under the Bill of Rights ispersonal and that
aside and appellants are acquitted of the crime charged with cannot be waived by anyone else rather than the person whose rights is
invaded or one whoexpressly to do so in his behalf.In the case at bar, the
costs de oficio. Their immediate release from custody is
prosecution did notestablished that Luz Tancianco was authorized by
ordered, unless they are otherwise detained for some other
theappellant to allow the officers to enter the appellants houseand seized
legal cause. However, the authorities must exert all efforts
the effects of the appellant upon seeing it inpresumption that it is
for the apprehension and prosecution of Ramon Jumawan for connected with the crime of subversion.Thereby violating the appellants
this killing.
rights of privacy and securityof house and effects.Thus, the prosecutions
evidences are weak andthat the search being illegal, the articles sought
SO ORDERED.
areinadmissible as evidences being a fruit of poisonous tree.

caliber .45 firearm and other items. After the raid,the group
proceeded to Bonuan, Dagupan City, and put under
surveillance the rented apartmentof Rosemarie Aritumba,
sister of Berlina Aritumba whom they earlier arrested. They
interviewedLuzviminda Morados, a visitor of Rosemarie
Aritumba. She stated that she worked with
BernieMendoza/Basilio Damaso. She guided the group to the
house rented by Damaso(@Mendoza).When they reached
the house, the group found that it had already vacated by the
occupants.Since Morados was hesitant to give the new
address of Damaso (@Mendoza), the group lookedfor the
Barangay Captain of the place and requested him to point out
the new house rented byDamaso (@Mendoza). The group
again required Morados to go with them. When they
reachedthe house, the group saw Luz Tanciangco outside.
They told her that they already knew that shewas a member
of the NPA in the area. At first, she denied it, but when she
saw Morados sherequested the group to go inside the house.
Upon entering the house, the group, as well as theBarangay
Captain, saw radio sets, pamphlets entitled Ang Bayan,
xerox copiers and acomputer machine. They also found
persons who were companions of Luz Tanciangco
(namely,Teresita Calosa, Ricardo Calosa, Marites Calosa,
Eric Tanciangco and Luzviminda Morados).The group
requested the persons in the house to allow them to look
around. When LuzTanciangco opened one of the rooms, they
saw books used for subversive orientation, one M-14rifle,
bullets and ammunitions, Kenwood radio, artificial beard,
maps of the Philippines,Zambales, Mindoro and Laguna and
other items. They confiscated the articles and brought themto
People vs. Damaso [GR 93516, 12 August 1992]
their headquarters for final inventory. They likewise brought
First Division, Medialdea (J): 3 concur
the persons found in the house tothe headquarters for
Facts:
People vs. Damaso
investigation. Said persons revealed that Damaso
On 18 June 1988, Lt. Candido Quijardo, a Philippine
Facts: After a sequence of arrest among members of theNPA, the
(@Mendoza) was thelessee of the house and owned the
Constabulary officer connected withthe 152nd PC Company items confiscated therefrom. Thus, Basilio Damaso,
Philippine Constabulary officers of LingayenPangasinan, the
at Lingayen, Pangasinan, and some companions were sent to wasoriginally charged in an information filed before the
apprehended NPA(Luz Tancianco)members pointed out to the PC the
house of the AppellantDamaso where the same is leasing.Being pointed verify the presence of CPP/NPA members in Barangay
Regional Trial Court of Dagupan City withviolation of
out that the appellant is the lessee of the house, the police went inside and Catacdang, Arellano-Bani, Dagupan City. In said
Presidential Decree 1866 in furtherance of, or incident to, or
eventually saw Min connection with thecrime of subversion, together with
place,
the
group
apprehended
Gregorio
Flameniano,
Berlina
1
Luzviminda Morados y Galang @ Ka Mel, Teresita Calosa
Aritumba, Revelina Gamboa andDeogracias Mayaoa. When yMacabangon @ Ka Tessie, Ricardo Calosa y Perez @ Ka
4rifles, radio sets, subversive materials and pamphlets, maps,computer
Ric, Marites Calosa y Evangelista @Ka Tess, Eric
machines, bullets and ammunitions. These articlesare confiscated to use interrogated, the persons apprehended revealed that there
as evidence against the accused inthe crime of Subversion.He was then was anunderground safehouse at Gracia Village in Urdaneta, Tanciangco y Capira @ Ka Ric and Luz Tanciangco y
Pangasinan. After coordinating with theStation Commander Pencial @ Ka Luz. Suchinformation was later amended to
convicted by the RTC Dagupan in thecrime of subversion, hence this
of Urdaneta, the group proceeded to the house in Gracia
exclude all other persons except Damaso from the
appeal.The appellant contends that the seizure in his houseis illegal for
Village. Theyfound subversive documents, a radio, a 1 x 7

criminalcharge. Upon arraignment, Damaso pleaded not


guilty to the crime charged. Trial on the meritsensued. The
prosecution rested its case and offered its exhibits for
admission. The defensecounsel interposed his objections to
the admissibility of the prosecutions evidence on grounds
of its being hearsay, immaterial or irrelevant and illegal for
lack of a search warrant; and thereafter,manifested that he
was not presenting any evidence for the accused. On 17
January 1990, thetrial court rendered its decision, finding
Damaso guilty beyond reasonable doubt, sentencing thelatter
to suffer the penalty of Reclusion Perpetua and to pay the
costs of the proceedings. Damasoappealed.
Issue:
Whether there was waiver on the part of Damaso to allow
the warrantless search of hishouse.
Held:
Damaso was singled out as the sole violator of PD 1866, in
furtherance of, or incident to,or in connection with the crime
of subversion. There is no substantial and credible evidence
to

sufficient importance to justify indifference to the basic


principles of government. As aconsequence, the search
conducted by the authorities was illegal. It would have been
different if the situation here demanded urgency which could
have prompted the authorities to dispense witha search
warrant. But the record is silent on this point. The fact that
they came to Damasoshouse at nighttime, does not grant
them the license to go inside his house.

[G.R. No. 104378. August 20, 1996]


PEOPLE OF THE PHILIPPINES, plaintiff-appellee, vs.
DANILO JUATAN y CAPSA, accused-appellant.

The prosecution made the following rendition of what it


submitted to be the facts.
The Western Police District Command, based in the City of
Manila, received word from a confidential informant and
some barangay officials that Danilo Juatan had been dealing
in prohibited drugs. On orders of P/Maj. Cipriano Herrera,
Jr., of the Narcotics Section, a police team, led by Lt.
Enrique Sy, was organized to conduct a one-week
surveillance on Juatan. When its surveillance showed
positive results, the police team decided to conduct a buybust operation.

On 05 July 1991, at around one o'clock in the morning, the


8-member police teamiv[4] proceeded to Instruccion Street,
Sampaloc, Manila. Pat. Ernesto Yamson was designated to
DECISION
be the poseur-buyer, while Pat. Eduardo Sison and the others
were assigned to secure the area. Pat. Amelito Lopez placed
VITUG, J.:
himself around seven meters away from the house of the
v
i
In its decision, [1] dated 03 February 1992, in Criminal Case accused. [5]
No. 91-96313, the Regional Trial Court of Manila, Branch
establish the fact that the appellant is allegedly the same
45, pronounced the accused, DANILO JUATAN Y CAPSA, The actual operation started with the informant calling the
person as the lessee of the house wherethe M-14 rifle and
"guilty beyond reasonable doubt" of violating Section 15 of accused and telling him that Yamson wanted to buy "drugs."
other subversive items were found or the owner of the said
Juatan met Yamson and the informant at the corner of an
Republic Act No. 6425, as amended by Presidential Decree
items. Evenassuming for the sake of argument that Damaso 1683, and sentenced him to suffer the penalty of life
alley, near Maceda and Instruccion streets, around ten meters
is the lessee of the house, the case against himstill will not
away from Juatan's house. Yamson gave Juatan a P500-bill
imprisonment and to pay a fine of P20,000.00.
prosper, the reason being that the law enforcers failed to
marked with his initials on the upper right hand portion of
vi
comply with therequirements of a valid search and seizure
over to Yamson a plastic
Juantan was charged, on 09 July 1991, with the commission the bill; [6] in turn, Juatan handed
vii
proceedings. The constitutional immunity fromunreasonable
container
measuring
2"
x
2
1/2."
[7]
At that precise
of the above offense in an information that read:
searches and seizures, being a personal one cannot he waived
moment, Yamson raised his right hand to signal his
by anyone except the person whose rights are invaded or one
companions that the deal had been made. Sensing that
"That on or about July 5, 1991, in the City of Manila,
who is expressly authorized to do so in his or her .
something was amiss, Juatan fled. He was about to get into
Philippines, the said accused, not having been authorized by
Therecords show that Damaso was not in his house at that
his house when the police apprehended him. Upon being
law to sell, dispense, deliver, transport or distribute any
time Luz Tanciangco and Luz Morados,his alleged helper,
searched, Juatan's right side pocket yielded the marked
regulated drug, did then and there wilfully, unlawfully and
allowed the authorities to enter it. There is no evidence that
P500-bill.viii[8] Juatan was accompanied by his wife to the
feloniously sell or offer for sale white crystalline substance
would establishthe fact that Luz Morados was indeed
police headquarters.ix[9] The booking and information sheet
known as `Shabu' containing methamphetamine
Damasos helper or if it was true that she was his helper,that
bearing Juatan's signaturex[10] described him to be a 5'6" tall
hydrochloride, which is a regulated drug.
Damaso had given her authority to open his house in his
taxi driver, with tattoo marks of the "Sigue-Sigue"
absence. The prosecution likewisefailed to show if Luz
commando.xi[11]
ii
"Contrary
to
law."
[2]
Tanciangco has such an authority. Without this evidence, the
authoritiesintrusion into Damasos dwelling cannot be given
At around nine o'clock that same morning, the members of
The accused entered a plea of not guilty; iii[3] forthwith, the
any color of legality. While the power tosearch and seize is
police team executed a joint affidavit of arrest. xii[12] Sgt.
trial
proceeded.
necessary to the public welfare, still it must be exercised and
Antonio T. Taca, signing for Maj. Cipriano Herrera, Jr., sent
the law enforcedwithout transgressing the constitutional
a letter-request to the Chemistry Section of the Criminal
rights of the citizens, for the enforcement of no statute isof
Investigation Laboratory for the laboratory examination of

the contents of the plastic bag taken from Juatan. xiii[13] The
request, along with the specimen, was received in the late
afternoon of 05 July 1991.xiv[14] On 08 July 1991,xv[15] the
item was turned over to the forensic chemist, Renee Eric
Checa, of the Chemistry Section. Measuring the specimen,
Checa found it to weigh 0.395 gram.xvi[16] Using the thinlayer chromatography, Checa specifically identified the
article to be shabu or methamphetamine hydrochloride.xvii
[17]

The defense presented three other witnesses. Jesus Lingat, a


32-year-old driver who resided just a house away from the
Juatan residence, testified that at around 1:30 a.m. of 05 July
1991, he and his brother saw around nine armed men
entering Juatan's house and later taking appellant with them.
The witness did not see Juatan's wife.xxiv[24] Ludovico
Munsayac, a 35-year old merchant and member of the
barangay council, said that he personally knew Juatan to be
just a taxi driver. Nonilon Reyes, barangay captain of
Barangay 524, Zone 52, Sampaloc, Manila, stated that
Juatan used to be a neighbor, and that, in connection with the
At the police headquarters, Juatan admitted that he was a
drug user but, after being apprised of his constitutional rights latter's application with the "Operation Tulong of DZRH," he
had issued a certification to the effect that Juatan was a taxi
to counsel and to remain silent, appellant decided not to
xviii
driver "ready to lend a helping hand to his neighbor."xxv[25]
make any further statement. [18] On 08 July 1991, Sgt.
Taca referred the case to the inquest fiscal xix[19] who
suggested that the accused be detained.xx[20]
On rebuttal, the prosecution presented Pat. Eduardo Sison
who denied the allegations made against him by Juatan and
his wife.
In his testimony, Juatan said that he was arrested by
policemen Edwin Sison, Ernesto Yamson and Amelito
Lopez. Surprised by the arrest, Juatan asked the police
The primary issue in this appeal, as so aptly pointed out by
officers what the problem was. He was told to produce a
appellant's counsel himself, is one of credibility.xxvi[26] It has
certain Boy Chua whom he did not know at all, thus
long been the entrenched rule in criminal jurisprudence that
prompting him to remark, "Edwin (referring to Pat. Sison),
appellate courts should defer to the findings of the trial court
xxi
personal na yata ang ginagawa mo sa akin." [21] When
considering the latter's vantage position in ascertaining that
Juatan asked to be shown either a search warrant or a
question of credibility.xxvii[27] Unless it is fairly evident that
warrant of arrest, he, instead, got a punch on the face from
the trial court, we consistently have said before, overlooked
Sison. The other police officers stopped Sison from doing
certain facts of substance and value that, if considered, could
any possible further harm on the accused. The group made a affect the outcome of the case, the trial court's own appraisal
search. Finding nothing, they dragged Juatan out of his
on the issue should be held to stand.xxviii[28] We have
house and brought him, along with his wife, to the police
examined the areas suggested by appellant but, in sum, they
headquarters.xxii[22]
appear to us to be matters peripheral in nature more than of
substance. For one, appellant ascribes ulterior motives on
the part of the arresting officers; however, he did not give
Juatan's wife, 38-year-old Aurora, declared that she was
doing her laundry, at about one o'clock in the morning of 05 any clear account or details of this charge.
July 1991, when armed men forcibly entered their house.
When she started to complain, the men simply told her to
Appellant underscores the absence of a warrant of arrest or
keep quiet. She followed the group upstairs; she was told
search warrant despite the fact that the police has had him
that they were looking for her husband and a certain Boy
subjected, prior to his apprehension, to a week-long
Chua. When she told them that her husband was asleep in
surveillance. In fine, he contends that the police could have
the room, the men went in and immediately handcuffed him. easily procured a "warrant before proceeding with the buyIt only angered the policemen when she demanded to be
bust operation."xxix[29] A buy-bust operation is far variant
shown a warrant. At the police headquarters, she recognized from an ordinary arrest; it is a form of entrapment which has
one of the apprehending police officers to be Pat. Sison. xxiii
repeatedly been accepted to be a valid means of arresting
[23]
violators of the Dangerous Drugs Law. In a buy-bust
operation the violator is caught in flagrante delicto and the

police officers conducting the operation are not only


authorized but duty-bound to apprehend the violator and to
search him for anything that may have been part of or used
in the commission of the crime.xxx[30]
Appellant has a violated Section 15 of the Dangerous Drugs
Law, as amended, which prescribes the penalty of reclusion
perpetua to death. Pursuant, however, to the Court's ruling in
People vs. Simon,xxxi[31] the penalty, considering that only
0.395 gram of shabu is involved in the prohibited sale,
should now be prision correccional conformably with the
amendatory law. Applying the Indeterminate Sentence Law,
and there being neither an aggravating nor mitigating
circumstance, the imposable penalty could be within the
range of arresto mayor, as minimum, to prision correccional
in its medium period, as maximum.
WHEREFORE, the Decision of the trial court finding
appellant Danilo Juatan y Capsa guilty beyond reasonable
doubt of violating Section 15 of the Dangerous Drugs Law,
as amended, is hereby affirmed subject to the modification
that he, instead, is meted the indeterminate sentence of six
(6) months of arresto mayor, as minimum penalty, to four (4)
years and two (2) months of prision correccional medium, as
maximum penalty. Considering that appellant has been
imprisoned for more than five (5) years or beyond the
maximum penalty herein imposed upon him, his
IMMEDIATE RELEASE from confinement is hereby
ordered unless his continued detention can be justified by
any other valid reason or cause. Costs against appellant.

G.R. No. 135053


PEOPLE OF THE PHILIPPINES, appellee,
vs.
BENJAMIN GALVEZ, appellant.
PANGANIBAN, J.:
The mandatory procedure laid down by jurisprudence and
the Rules of Court should be meticulously observed by trial
courts in accepting a plea of guilt in a case involving a
capital offense. There should be no doubt that the accused
might have misunderstood the nature of the charges and the

consequences thereof. Otherwise, the plea would be set


aside for having been improvidently made.
, J.:

beside each other on the bed. Their bed was beside an


aparador and Cristina slept on the side nearest to the
aparador. Appellants bed was beside the door and was just
near the bed where Cristina and Melowin were sleeping.
Three (3) of their brothers were also asleep in the house,
namely, Melchor, 13 years old, Alvin, 12 years old, and
Jesus, 9 years old, while their 19 year-old brother was at a
neighbors house watching television. Their mother, Marilyn
Galvez, was not with them as she was in Hongkong at that
time.

The mandatory procedure laid down by jurisprudence and


the Rules of Court should be meticulously observed by trial
courts in accepting a plea of guilt in a case involving a
capital offense. There should be no doubt that the accused
might have misunderstood the nature of the charges and the
consequences thereof. Otherwise, the plea would be set aside
for having been improvidently made.
"That night, appellant, who was drunk, arrived and then ate.
After eating, he went to lie down on his bed. After a lapse of
The Case
thirty (30) minutes, appellant went to lie down on the bed
where Cristina and Melowin were sleeping. He positioned
For automatic review is the July 30, 1998 Decision1 of the
himself on the right side of Cristina, slowly lifting and
Regional Trial Court (RTC) of Bayombong, Nueva Vizcaya moving Melowin, who was asleep beside Cristina, away
(Branch 27) in Criminal Case No. 3299, convicting
from her sisters side. Then, he slowly removed Cristinas
Benjamin Galvez of qualified rape and sentencing him to
shorts and shirt and began touching her on all parts of her
death. The assailed Decision disposed as follows:
body. He positioned himself on top of her but she kicked
him, causing him to be thrown against the aparador.
"WHEREFORE, finding the accused Benjamin Galvez y
Apparently hurt and angered, he pulled his samurai from
Domingo GUILTY beyond reasonable doubt of committing under his mat and pointed it towards the left front portion
rape against his own daughter, 16 years old at the time of the between her breast and her armpit. Scared, she was
rape, he is hereby sentenced to die by lethal injection; to pay immobilized. She knew of her fathers capacity to kill her
the victim the sum of P50,000.00 as civil indemnity and to
because she had previously witnessed how he almost killed
pay the costs of the suit."2
their mother when the latter was still with them before she
left for Hongkong. Appellant completely undressed himself,
The Facts
started to kiss her, and forcibly inserted his sexual organ into
hers and did a pumping motion. Before leaving her to sleep
In its Brief,3 the Office of the Solicitor General presents the on his bed, he threatened to kill her if she would report what
following narration of facts:
he did to her, warning that he would also include the one
whom she would report to. She felt pain on her body,
"Cristina Galvez was born on May 10, 1981. She, her father, including her private parts. She also saw a small quantity of
Benjamin Galvez, the appellant herein, together with her five blood coming from her vagina.
(5) siblings, resided at Tuao South, Bagabag, Nueva Vizcaya.
"Sometime in the third week of April 1997, about 6:00
oclock in the evening, Cristina, who was then sixteen (16)
years old, and her youngest sister, Melowin, were sleeping

struggle and he successfully had sexual intercourse with her.


He thereafter regularly raped her about 3 to 4 times a week,
usually after a one (1) day interval. This went on until
Cristina became pregnant and gave birth to a baby boy on
January 23, 1998. The baby boy was named Christian."4
(Citations omitted)
In an Information dated February 20, 1998, appellant was
charged with multiple rape, defined and penalized under
Republic Act No. 8353. He allegedly committed the crime as
follows:
"That sometime in the third week of April, 1997, in the
evening and several times thereafter, in Barangay Tuao
South, Municipality of Bagabag, Province of Nueva Vizcaya,
Philippines and within the jurisdiction of this Honorable
Court, the above-named accused, with lewd design, with the
use of force, threat and intimidation, did then and there
wilfully, unlawfully and feloniously have carnal knowledge
of his daughter Cristina Galvez y Tomboc, 16 years old at
the time, against the latters will and without her consent, to
her damage and prejudice including her parents."5
During the arraignment, read and explained to appellant in
Ilocano a dialect he spoke and understood were the
charges for ten counts of rape. Assisted by Atty. Renato
Mercado, he pleaded not guilty to the charges.6 However, on
May 14, 1998, appellant, this time with the assistance of
Atty. Ruby Rosa Espino,7 changed his plea to that of guilt.
In accordance with the RTCs Order dated May 14, 1998, an
inquiry into the voluntariness and full comprehension of his
plea was conducted. After hearing evidence for the
prosecution, the lower court rendered the assailed Decision.
Appellant did not present any evidence on his behalf. Neither
did his counsel present in his Brief any counter-statement of
the facts.8

"On April 28, 1997, appellant repeated the dastardly act on


his daughter, Cristina. This time, he already held the
samurai, pointing it towards her neck when he went near her. Ruling of the Trial Court
Afraid, she had to allow appellant to undress her without a

The automatically appealed Decision states that the trial was


conducted pursuant to People v. Alicando,9 which held that
"a conviction in capital offenses cannot rest alone on a plea
of guilt after a free and intelligent plea of guilt, the trial
court must require the prosecution to prove the guilt of the
appellant and the precise degree of his culpability beyond
reasonable doubt."10 As appellant absolutely refused to offer
any evidence in his own favor, the trial court decided the
case solely on the basis of the evidence presented by the
prosecution.
According full faith and credence to the testimony of the
victim, the RTC found that she had been raped by her father.
It based its conclusion on the following: (1) she cried several
times during her testimony; (2) no daughter, especially one
as young as she was, would have charged her own father
with so serious an offense that prescribed the death penalty,
if she had not indeed been raped; (3) appellant was accorded
the chance to refute the claim of his daughter, yet he did not;
besides, a young girl like her would not have submitted to
the advances of her own father, unless these were made
through force and intimidation; and (4) appellant entered a
plea of guilt.11
However, the RTC convicted him of only one count of rape
perpetrated in the third week of April, 1997, because the
Information had alleged only one incident of the crime. The
allegation that the victim was raped several times after the
third week of April 1997 was deemed "too indefinite to give
the accused the opportunity to prepare his defense."
Hence, this automatic appeal.12
The Issue
Appellant prays for the remand of the case to the court of
origin for proper arraignment and trial based on this sole
assignment of error:

"The court a quo gravely erred in not applying the


Moreover, as prescribed in Aranzado, the searching inquiry
safeguards set forth under Rule 116, 1985 Rules on Criminal to be conducted by the trial court should consist of the
Procedure."13
following:
This Courts Ruling

"(1) Ascertain from the accused himself (a) how he was


brought into the custody of the law; (b) whether he had the
We agree with appellant his plea of guilt was
assistance of a competent counsel during the custodial and
improvident.
preliminary investigations; and (c) under what conditions he
was detained and interrogated during the investigations.
Main Issue:
These the court shall do in order to rule out the possibility
that the accused has been coerced or placed under a state of
Proper Procedure When the Accused Pleads Guilty in a Case duress either by actual threats of physical harm coming from
Involving a Capital Offense
malevolent or avenging quarters.
Citing People v. Bello,14 appellant argues that the trial court
failed to observe the mandatory procedure for accepting a
positive plea to a charge punishable by death.
The stringent procedure governing the reception of a plea of
guilt, especially in a case involving the death penalty, is
imposed upon the trial judge in order to leave no room for
doubt on the possibility that the accused might have
misunderstood the nature of the charge and the consequences
of the plea.15

(2) Ask the defense counsel a series of questions as to


whether he had conferred with, and completely explained to,
the accused the meaning and consequences of a plea of
guilty.
(3) Elicit information about the personality profile of the
accused, such as his age, socio-economic status, and
educational background, which may serve as a trustworthy
index of his capacity to give a free and informed plea of
guilty.

In People v. Aranzado,16 the Court, citing Section 3, Rule


(4) Inform the accused [of] the exact length of imprisonment
11617 of the Rules of Court, set the following guidelines for or nature of the penalty under the law and the certainty that
receiving a plea of guilt in a case involving a capital offense: he will serve such sentence. Not infrequently indeed an
accused pleads guilty in the hope of a lenient treatment or
"(1) The court must conduct a searching inquiry into the
upon bad advice or because of promises of the authorities or
voluntariness and full comprehension of the consequences of parties of a lighter penalty should he admit guilt or express
the plea;
remorse. It is the duty of the judge to see to it that the
accused does not labor under these mistaken impressions.
(2) The court must require the prosecution to present
evidence to prove the guilt of the accused and the precise
(5) Require the accused to fully narrate the incident that
degree of his culpability; and
spawned the charges against him or make him reenact the
manner in which he perpetrated the crime, or cause him to
(3) The court must ask the accused if he desires to present
supply missing details of significance."19
evidence in his behalf and allow him to do so if he
desires."18

Appellants re-arraignment on May 14, 1999 miserably fell


short of these guidelines, as shown by the pertinent portion
of the transcript of stenographic notes, which we quote
hereunder:
"PROS. CASTILLO:
By way of dialogue with the defense counsel the accused is
willing to enter a plea of guilty for the ten (10) counts of
rape, your Honor.
COURT:
Why dont you arraign him? Alright, the previous plea of not
guilty is now withdrawn to give way to the plea of guilty by
the accused for 10 counts of rape but as the Court had
already observed we have to conduct the re-arraignment of
this case. Will you please arraign him.

It is clear from the foregoing that the trial judge did not
conduct a "searching inquiry" into the voluntariness of
appellants plea of guilt and full comprehension thereof. He
asked no questions on the subjects mentioned in Aranzado.
His purported compliance with Alicando was more like a
monologue, or a warning at best, rather than a searching
inquiry. He did not inquire into appellants personality
profile age, socio-economic status or educational
background.21 His Honor did not even require an answer to
his question on whether appellant realized that the death
penalty would result from the latters plea. No response from
appellant was given or recorded.
Moreover, there is no showing that the lawyer explained to
appellant the consequences of the latters plea probable
conviction and death sentence. Equally important, the trial
judge should have asked why the plea of appellant was
changed. The former obtained none of the information
required in Aranzado. Hence, there is no basis to conclude
that the latter voluntarily and intelligently pleaded guilty to
the charges against him.

remanded the case for re-arraignment of the accused who


had been charged with the rape and murder of a nine-yearold girl, because his counsel had declined to present
evidence for his client, banking on the mitigating
circumstance of the plea of guilt. This Court clarified that
under no circumstance would an admission of guilt in that
case affect or reduce the death sentence because it was a
single indivisible penalty which is applied regardless of any
mitigating or aggravating circumstance attending the crime.
In the instant case, the Court also notes that "guilty" was not
the original plea of appellant; hence, careful effort should
have been exerted by the court below to inquire into why he
changed his plea. In addition, he refused to present evidence
in his defense. This should have again prompted the trial
judge to probe more deeply, following the guidelines in
Aranzado.

A plea of guilt is improvidently accepted where no effort is


made to explain to the accused that, in a case involving a
capital offense, such plea may result in the imposition of the
death penalty.26 The same is true when the requirements in
In Bello, the Court remarked that there were cases when the Aranzado are not satisfied.27 Recently, in People v.
INTERPRETER:
accused would plead guilty in the hope of a lenient treatment Bernas,28 the Court set aside a death sentence and remanded
or because of promises from the authorities or parties that an the case to the trial court, because the Aranzado guidelines
The accused pleaded guilty.
expression of remorse would result in a lighter penalty.22
on how to conduct a "searching inquiry" had not been
followed.
COURT:
Where the punishment to be inflicted is death, it is not
enough that the information be read to the accused or even
WHEREFORE, the automatically appealed Decision is SET
I would like to ask the accused if he understands from his
translated into the dialect they speak. This is because the
ASIDE. Criminal Case No. 3299 is REMANDED to the
counsel, the circumstances in this case because the victim
implementation of such penalty is irrevocable, and
court of origin for re-arraignment and further proceedings to
here is his own daughter and she is below 18 years of age. In experience has shown that innocent persons have at times
be conducted with all deliberate speed, in accordance with
accordance with the heinous offense law, the Court will have pleaded guilty.23 The trial court must avoid improvident
this Decision. No costs.
to impose on him the penalty of death. Is this clear to the
pleas of guilt, since the accused might be admitting their
accused? At any rate we will conduct a trial to find out if
SO ORDERED.
guilt and thus forfeiting their lives and liberties without
there is sufficient evidence to convict you so that your rights having fully understood the meaning, significance or
will be protected you are given a chance to prove your
CALLANTA VS. VILLANUEVA [77 SCRA 377; G.R.
consequences of their pleas.24
innocence latter to refute the evidence of guilt beyond
NOS. 24646 & L-24674; 20 JUN 1977]
Thursday, February 12, 2009 Posted by Coffeeholic Writes
reasonable doubt."20
What is apparent here is that appellant was not properly
Labels: Case Digests, Political Law
advised by his counsel. In People v. Sevilleno,25 the Court
(The accused was arraigned by reading to him the
Information in Ilocano dialect which the accused speaks and
understands.)

Facts: Two complaints for grave oral defamation were filed


against Faustina Callanta. The City Judge of Dagupan City,
Felipe Villanueva, denied the motions to quash the
complaints. Thus, petitioner Callanta brought the suits for
certiorari in the Supreme Court. Petitioner questions the
validity of the issuance of warrant of arrest by respondent,
arguing that the City Fiscal should have conducted the
preliminary investigation. According to petitioners counsel,
there was jurisdictional infirmity. After the issuance of the
warrants of arrest and the bail fixed at P600, petitioner
posted the bail bond, thus obtaining her provisional liberty.
The City Fiscal in this case did not disagree with the judges
investigation, and agreed with the complaints filed.
Issue: Whether or Not petitioners contentions are to be
given
merit.

1 Respondent Judge denies the motions to quash two


complaints for grave oral defamation against petitioner.

ground that it shouldh a v e b e e n t h e C i t y


F i s c a l w h o c o n d u c t e d t h e preliminary
investigation.- After the warrants were issued (with bail
pegged atP600), Callanta posted the required bail
2 Petitioners contest the validity of the arrest warrants
bonds andwas granted her provisional liberty.- T h e
issued by respondent Judge on the ground that it should have
City Fiscal had manifested his intent
been the City Fiscal who should have conducted the
t o prosecute the case.- February 25, 1965 After the Court
preliminary investigation.
had conductedp r e l i m i n a r y i n v e s t i g a t i o n a n d
h a d a c q u i r e d jurisdiction over the case, the Court
referred the caseto the Fiscal.- March 4, 1965 The
3 After the warrants were issued however, petitioner
arraignment was postponedbecause the Fiscal was still
posted bail thus obtaining her provisional liberty.
doing his investigation.- I n t h e p r o c e e d i n g s o f A p r i l
20, 1965, the Fiscalentered his appearance for
Issue: WON warrant of arrest may be invalidated after
t h e g o v e r n m e n t a n d manifested that he was ready for
posting of bail
trial.
ISSUE
Held: NO
WON Callanta can contest the validity of his arrest
HELD
1
Zacarias v. Cruz: Posting of a bail bond constitutes
NO
waiver of any irregularity attending the arrest of a person, Ratio
Posting of a bail bond constitutes waiver of any
estops him from discussing the validity of his arrest.
irregularity attending the arrest of a person andestops him
from discussing the validity of his arrest.
2
People v. Obngayan: Where the accused has filed bail
Reasoning
and waived the preliminary investigation proper, he has
- In the case of
waived whatever defect, if any, in the preliminary
Luna vs. Plaza
examination conducted prior to the issuance of the warrant , the Court held thatwhere petitioner has filed an
of arrest.
application for bail andw a i v e d t h e p r e l i m i n a r y
i n v e s t i g a t i o n p r o p e r , h e waived his objection to
whatever defect, if any, in thepreliminary examination
PETITION DISMISSED
conducted, prior to theissuance of a warrant of arrest.

Held: Based on many precedent cases of the Supreme Court,


where the accused has filed bail and waived the preliminary
investigation proper, he has waived whatever defect, if any,
in the preliminary examination conducted prior to the
issuance of the warrant of arrest. In the case at bar, it is
futile for the petitioner to question the validity of the
issuance of the warrant of arrest, because she posted the bail
bond. Petitioner also erred in arguing that only the City
Fiscal can conduct a preliminary investigation. According to
the Charter of the City of Dagupan, the City Court of
Dagupan City may also conduct preliminary investigation
for any offense, without regard to the limits of punishment,
and may release, or commit and bind over any person
charged with such offense to secure his appearance before
the proper court. Petition for certiorari is denied. CALLANTAv VILLANUEVA
Restraining order issued by the Court is lifted and set aside. 77 SCRA 377FERNANDO; June 20, 1977
NATURE
Original petitions in the Supreme Court,
certiorariwith preliminary injunction
Callanta v Villanueva (77 SCRA 377)
FACTS
November 10, 2010
- Judge Villanueva of Dagupan refused to grant
themotions to quash two complaints for oral
defamationagainst Callanta.- Callantas counsel argued
that there was an issuewith regard to the validity of
Villanuevas issuance of the warrants of arrest on the
Facts:

- This doctrine has been upheld in a number of


casesincluding
People vs. Olandar, Zacarias vs. Cruz,Bermejo vs.
Barrios, People vs. La Caste, Manzano vsVilla
andPeople vs. Obngayan
which stated thatw h e r e t h e a c c u s e d h a s f i l e d
bail and waived thepreli minar y investigation
proper, he has waived w h a t e v e r d e f e c t , i f
a n y , i n t h e p r e l i m i n a r y examination
conducted prior to the issuance of the warrant of
arrest.- T h e c i t y f i s c a l h a d b e e n q u i t e
a c t i v e i n t h e investigation and in the prosecution of the

accused.It was he who manifested his readiness to appear


inthe trial.
Obiter
- With regard to the issue of whether or not the
onlyp e r s o n v e s t e d w i t h a u t h o r i t y t o
c o n d u c t a preliminary investigation is the city fiscal,
the Charterof the City of Dagupan provides that the City
Courto f D a g u p a n C i t y m a y a l s o c o n d u c t
p r e l i m i n a r y investigation for for any offense,
without regard tothe limits of punishment and may release
or commitany person charged with such offense to secure
hisappearance before the proper court.
Dispositive
W H E R E F O R E , t h e s e p e t i t i o n s f o r certiorari
are dismissed. The restraining order issuedby this Court is
lifted and set aside. Costs against petitioner.
SEPARATE OPINIONAQUINO [concurring]
- Sec. 77 of the Dagupan City charter
e x p r e s s l y empowers its city court (formerly municipal
court) toconduct preliminary investigation for any
offense,without regard to the limits of punishment.Every justice of the peace, municipal
j u d g e (meaning city judge), city or provincial
fiscal, shallhave authority to conduct preliminary
examination orinvestigation in accordance with these
rules of alloffenses alleged to have been committed within
hismunicipality, city or province, cognizable by
theCourt of First Instance (Sec. 87 of the Judiciary
Lawand Sec. 2, Rule 112)
G.R. No. 103964 August 1, 1996
PEOPLE OF THE PHILIPPINES, plaintiff-appellee,
vs.
NARCISO NAZARENO, RAMIL REGALA,
ORLANDO HULAR and MANUEL LAUREAGA,
accused-appellants.

MENDOZA, J.:p

This is an appeal from the decision, 1 dated May 28,


1991, of the Regional Trial Court of Makati (Branch
136), finding accused-appellants Narciso Nazareno
and Ramil Regala guilty of murder for the killing of
Romulo "Molet" Bunye II in Muntinlupa, Metro
Manila on December 14, 1988 and sentencing them
to suffer the penalty of reclusion perpetua. In
addition, the two were ordered to pay jointly and
severally to the heirs of the deceased the amount of
P50,000.00. Two others, accused with them, Manuel
Laureaga and Orlando Hular, were acquitted.
The evidence for the prosecution shows that on
December 14, 1988, between 8:00 a.m. and 9:00
a.m., Romulo Bunye II took a tricycle (referred to in
the record as "stainless" tricycle evidently because
its body was made of stainless steel), which was
driven by Fernando Hernandez. Unknown to Bunye
was that two men were waiting outside his house
and that the two hailed another tricycle in order to
follow him.
Bunye alighted at the corner of T. Molina and
Mendiola Streets in Alabang, Muntinlupa and
crossed to the left side of the street . Shortly after,
the tricycle, driven by Rogelio de Limos, arrived and
stopped in front of Hernandez's "stainless" tricycle.
One of the men jumped out of the tricycle and shot
Bunye at the back of the head. When Bunye fell face
down, the assailant fired another shot at Bunye's
head. Then, the other man approached Bunye and
shot him also in the head.
Rogelio de Limos and Fernando Hernandez, the
tricycle drivers, executed sworn affidavits relating
what they had witnessed. 2 The two described the
assailants and stated that they could recognize the
killers if they saw them again. There was another
witness, a woman, who was also a passenger of the

"stainless" tricycle on which Bunye rode but her


identity had remained unknown.
The autopsy report on the victim showed that he
died of gunshot wounds in the head. 3
On December 28, 1988, Ramil Regala, Narciso
Nazareno, Orlando Hular and Manuel Laureaga
were arrested. Regala and Nazareno were put in a
police line-up. They were identified and pointed to
as the assailants by the tricycle drivers Hernandez
and de Limos. Hernandez and de Limos executed
additional sworn affidavits.
Ramil Regala executed affidavits, dated December
28, 1988 4 and January 2, 1989, 5 admitting
participation in the slaying of Bunye and pointing to
Narciso Nazareno and a certain Rey Taling as his coconspirators. He claimed that they had been hired by
Orlando "Boy" Hular to kill the victim and told that
they would be paid P30,000.00 by Manuel Laureaga.
His affidavits were corroborated by Orlando Hular
who, in an affidavit, executed on the same day,
December 28, 1988, 6 stated that it was Laureaga
who wanted Bunye killed, apparently in connection
with Bunye's job as administrator of the public
market in Alabang.
However, Regala and Hular subsequently recanted.
Regala claimed that he had been tortured. 7 On the
other hand, Hular claimed that, although he was not
tortured, he admitted to the crime and signed the
affidavit because he was afraid he would also be
tortured. 8 Narciso Nazareno also claimed to have
been tortured to admit to the crime but refused to
sign any written statement. 9
The trial court ruled the confessions of Regala and
Hular to be in admissible. However, it held Regala
and Nazareno guilty on the basis of their positive

identification by Hernandez and de Limos during the


police line-up on December 28, 1988 and their
testimony in court. The trial court stated:
As between the aforecited testimonies of
Rogelio de Limos and Hernandez on one
hand and the testimonies of Narciso
Nazareno and Ramil Regala on the other, the
Court would place its reliance on the
testimonies of the prosecution witnesses,
because firstly, there is no showing in the
record that Rogelio de Limos and Fernando
Hernandez are manufactured evidence. As a
matter of fact, none of the defense witnesses
had ever made such an imputation; and
neither did the defense lawyers do so in their
extensive memoranda. Secondly, it is a wellsettled doctrine in this jurisdiction that as
between positive testimonies and denials,
the Court should place more weight on the
former (People v. Mostoles, Jr. 124 SCRA
906). Thirdly, the testimonies of Narciso
Nazareno and Ramil Regala are in the nature
of alibis, and it is also settled that because
they can easily be concocted, the Courts
should exercise extreme caution in accepting
them as defense (People vs. Bagsica, 65
SCRA 400).
Orlando Hular and Manuel Laureaga were acquitted
for lack of evidence against them. 10
Hence this appeal by Nazareno and Regala.
In his brief, accused-appellant Narciso Nazareno
assigns the following errors:
THE LOWER COURT ERRED IN
FAILING TO CONSIDER THE
VIOLATION OF THE ACCUSED-

APPELLANT'S CONSTITUTIONAL
RIGHT TO DUE PROCESS AS A FATAL
FLAW IN HIS PROSECUTION AND
SUBSEQUENT CONVICTION.
THE LOWER COURT ERRED IN
FAILING TO ACQUIT THE ACCUSEDAPPELLANT ON REASONABLE
DOUBT.
Accused-appellant Ramil Regala, on the other hand,
contends:
THE TRIAL COURT ERRED IN NOT
CONSIDERING THE UNLAWFUL
ARREST OF RAMIL REGALA AS A
GROSS VIOLATION OF HIS
CONSTITUTIONAL RIGHT TO DUE
PROCESS.
THE TRIAL COURT ERRED IN NOT
CONSIDERING THE TESTIMONY OF
THE EXPERT WITNESS FROM THE
NATIONAL BUREAU OF
INVESTIGATION IN DETERMINING
THE PROBABILITY OF GUILT OF
APPELLANT.
THE TRIAL COURT ERRED IN
RELYING ON THE INCREDIBLE
TESTIMONIES OF FERNANDO
HERNANDEZ AND ROGELIO DE
LIMOS IN CONVICTING HEREIN
APPELLANT.
We have reviewed the record and the evidence, and
we find accused-appellants' contentions to be
without merit.

First. Accused-appellants claim that their arrests


without warrant were illegal and justify the
nullification of the proceedings of the trial court.
The contention is untenable. The warrantless arrest
of accused-appellant Narciso Nazareno was upheld
by this Court in 1990 in a petition for habeas corpus.
It appears that, on January 9, 1989, Nazareno filed a
motion for bail. 11 As the trial court denied his
motion, a petition for habeas corpus was filed on his
behalf with this Court. It was alleged that Nazareno's
arrest was illegal because it was made without
warrant fourteen days after the killing of Romulo
Bunye II. This Court dismissed the petition in its
decision of July 9, 1990. 12 He filed a motion for
reconsideration which the Court also denied on the
ground that the warrantless arrest was in accordance
with Rule 113, 5(b) of the Revised Rules of
Criminal Procedure. 13 The question which Nazareno
raises has thus been settled long ago in a final
decision of this Court.
Furthermore, Nazareno and Regala waived
objections based on the alleged irregularity of their
arrest, considering that they pleaded not guilty to the
charges against them and participated in the trial.
Any defect in their arrest must be deemed cured
when they voluntarily submitted to the jurisdiction
of the court. 14 For the legality of an arrest affects
only the jurisdiction of the court over the person of
the accused. 15 Consequently, if objections based on
this ground are waived, the fact that the arrest was
illegal is not a sufficient cause for setting aside an
otherwise valid judgment rendered after a trial, free
from error. 16 The technicality cannot render the
subsequent proceedings void and deprive the State
of its right to convict the guilty when all the facts on
record point to the culpability of accused. 17
Second. Accused-appellants argue that the trial court
erred in giving credence to the testimony of

Hernandez and de Limos. Accused-appellant


Nazareno claims that the decision of the trial court
does not contain an analysis of the testimonies of
Hernandez and de Limos and suggests that the
killing of Bunye was executed by professionals and
not by a simple fruit vendor. Nazareno claims that
the witnesses were reluctant, evasive and fearful and
that they never had the opportunity to fully observe
the incident as there was traffic and they merely had
a side view of the assailants. Nazareno claims that
the evidence of torture and maltreatment and the
other circumstances were indications of
manipulation and manufacture of evidence to frame
him. 18 Similarly, accused-appellant Regala claims
that the testimonies of Hernandez and de Limos
were confused and confusing. He then suggests that
the manner the killing was perpetuated shows that it
was done by professional assassins. 19
These arguments are without merit. Far from being
confused, the testimonies of Hernandez and de
Limos were straightforward and unwavering and
justified the trial court in giving them full faith and
credit. The accused-appellants were positively
identified by Hernandez and de Limos under
circumstances which were ideal for identification.
The incident happened in daylight and only two
meters away from them. 20 They did not only see the
assailants but they also witnessed the whole
incident.
The testimonies of Hernandez and de Limos during
direct and cross examinations corroborate each other
on the material facts. A summary of Hernandez's
direct examination 21 reveals that on December 14,
1988, between 8:00 to 9:00 in the morning, a woman
took a ride in his tricycle, followed by a man whom
he identified as Molet Bunye, who asked to be taken
to Purok 6, T. Molina St. Upon reaching his
destination, Bunye alighted and walked across the

street. Hernandez said that just then he noticed a


man approach Bunye, point a gun and fire at him. He
saw the face of the assailant. He then saw the
assailant position himself near Bunye's head and fire
another shot when Bunye fell. Hernandez said he got
confused and afraid and, as his other passenger was
screaming, he tried to turn his tricycle around to
leave. As he was doing so, he noticed accusedappellant Ramil Regala on his right approach Bunye
and fire a shot at the victim. When asked if he could
identify the assailants, Hernandez answered "yes."
When asked to point to the assailants, he identified
accused-appellant Narciso Nazareno as the first one
who shot Bunye and Ramil Regala as the second
one.
In his cross-examination, defense counsel tried to
show that Hernandez did not see what really
happened. This is not so. His testimony on cross
examination, slightly edited, follows: 22
Q On Dec. 14, 1988, you
were driving your tricycle.
Is that correct?
A Yes, sir.
Q You know that Rogelio de
Limos was also driving his
tricycle on the morning of
Dec. 28, 1988?
A Yes, sir.
Q While you were driving
your tricycle on that faithful
morning, who was ahead
you or Mr. de Limos?
A I.

Q After you took the


woman and the man who
was shot, did you know that
Mr. de Limos was following
you?
A I don't know.
Q How do you know that
you were ahead?
A I learn[ed] it later on
when Mr. de Limos
overtook my tricycle?
Q In what place, when [did]
Mr. de Limos overtake your
tricycle?
A Also at T. Molina Street,
purok 6.
Q When Mr. de Limos
overtook your tricycle,
where you on stop position
or were you still running?
A Already stopped.
Q In other words you
stopped your tricycle and
that was the time when Mr.
de Limos went ahead or
overtook your tricycle?
A Yes, sir.
Q To what side of you, left
side or right side, did Mr. de
Limos pass?

A To my left.
Q He stopped his tricycle in
front of yours?
A Yes, sir.
Q When Mr. de Limos
stopped his tricycle in front
of yours, how far was his
tricycle from the front side
of your tricycle?
A Almost touching the front
side of my tricycle.
Q Can you tell us, how
many minutes after you
have stopped when Mr. de
Limos overtook you?

tricycle in relation to yours?


Was it directly in front of
you or was it towards your
left side?

enough space for a person


to pass?

A His tricycle stopped


directly in front of me, but a
little sway to the right.

Q Your two (2) passengers,


the man and the woman
who alighted your tricycle
first?

Q Would you say that the


distance of your tricycle
when both of you stopped
was about one (1) foot
only?
A About two (2) to three (3)
feet.
Q Is it not correct to say that
the tricycle touched each
other at that time?

A I don't remember.
Q Maybe five (5) minutes
after you have stopped that
Mr. de Limos arrived?
A I can not estimate.
Q Was it very short period
or was it in reasonable
period of time after you
stopped when Mr. de Limos
arrived?

A I was surprised when he


suddenly overtook me.
Q So your previous
statement is not correct that
the tricycle almost touched
each other, because they
were between two (2) to
three (3) feet?

A Yes, sir.

A The man (Bunye).


Q After Molet Bunye
alighted from your tricycle,
he walked to the left,
crossed the street crossing
in front of your tricycle?
A After Molet Bunye
alighted from my tricycle,
he walked in front of me
and paid his fare. After that
he walked across the street
towards the left.
xxx xxx xxx
Q Was Molet Bunye able to
reach the other side of T.
Molina Street?
A Yes, sir.

A Only for a while.

A Because I don't remember


the distance because the
incident happened a long
time ago.

Q When Mr. de Limos


stopped his tricycle, what
was the position of his

Q When the two (2)


tricycles [were] already on
stopped position, was there

Q This tricycle of Mr. de


Limos, tell us when the
tricycle arrived was it after
Molet Bunye paid his fare
to you or after?

A While Molet Bunye was


paying the fare, that was the
time when Mr. de Limos
overtook me.
Q Molet Bunye was still
near you or beside you
when the tricycle of Mr. de
Limos arrived?
A Yes, sir.
Q After paying his fare, you
said, Molet Bunye walked
to the other side of the
street. How long did it take
Molet Bunye to go to the
other side of the street?
A I don't remember.
Q When the tricycle of Mr.
de Limos overtook you, you
did not know that that was
Mr. de Limos?
A I noticed him when he
was already in front of me,
and he was at the back.
Q When Molet Bunye paid
his fare to you, you backtract your tricycle because
you were preparing to turn
around?
A Yes, sir. I back-up my
tricycle a little so that I'll be
prepared to leave the place.

Q Were you supposed to


leave the place after
embarking Molet Bunye,
because you were to bring
another lady to another
destination?
A Yes, sir.
Q When you back-up to
which direction were you
looking?

A Yes, sir, but he did not


hear me.
Q How far was he when you
call[ed] him?
A About three (3) to four (4)
meters.
Q And he was walking
away from you?
A Yes, sir.

A To the direction of Molet


Bunye. I was looking to the
direction of Molet Bunye,
because I was about to call
him because he still had
change, and I was about to
return it back to Molet
Bunye.
Q When you were backingup your tricycle preparing to
leave the place, you were
looking at Molet Bunye?
A Yes, sir.
Q Did you not look behind,
so that you will be able to
see him?
A I look[ed] at my side
mirror.
Q Did you call Molet
Bunye, so that he can hear
you, because you wanted to
give his change?

xxx xxx xxx


Q How much [did] Molet
Bunye give you, when he
paid his fare to you?
A Two pesos (2.00).
Q Why did you not give his
change when he was still
near you?
A Because he was in hurry.
Q In other words, when he
alighted he did not approach
you anymore but he just
handed to you his two
(2.00) pesos?
A Yes, sir, he handed his
two (2.00) pesos to me. He
said "BOY BAYAD KO."
(Witness demonstrating by
stretching his left hand
forward.)

Q And you immediately


received the money?
A Yes, sir.
Q And Molet Bunye
proceeded to walk away
from you?
A Yes, sir.
Q When was it when the
tricycle of Mr. de Limos
overtook you, was it when
Molet Bunye paid his fare
to you or after Molet Bunye
had paid?

A Like this (witness


demonstrating stood up
stretching his right hand
forward as if holding a gun
and poked it at the back of
the head of the interpreter).

A Yes, sir.
Q Assuming that the Court
Interpreter is Molet Bunye,
please demonstrate which
part of the head was the gun
pointed [at]?
A (Witness stood up
demonstrating by pointing
somewhere above the neck
or maybe center of the neck
of the interpreter.)

A I can not remember.


Q You said that the first
person who shot Molet
Bunye was Narciso
Nazareno, is that correct?
A Yes, sir.
Q Did you see that?
A Yes, sir. I saw that the gun
was poked at the head.
Q And you heard the shot?
A Yes, sir.
Q Can you demonstrate how
the gun was pointed to the
head of Molet Bunye?

Q The man whom you said


pointed a gun at the head of
Molet Bunye, he was also
walking away from you, is
that correct?

Q He was not walking


towards you?
A No, sir. Away from me.
Q So that you were from the
position from where you
were, you were still in your
tricycle?
A Yes, sir.

Q Can you tell us, how tall


is Molet Bunye in relation
to our Court Interpreter
here?
A Shorter than the
interpreter.
Q When the gun which you
said was pointed at the head
of Molet Bunye, Molet
Bunye was walking away
from you? Is that correct?
A Yes, sir.

Q From that position you


can see Molet Bunye and
the person whom you said
fired a shot?
A Yes, sir.
Q The man whom you said
fired the first shot was also
walking when he fired the
first shot?
A The man who was
holding a gun stopped first
and he fired it.

Q He was walking. Right?


A Yes, sir.

Q Tell us how far was this


man who fired the first shot
to Molet Bunye when the
first shot was fired?

A About one (1) meter.


Q What did you see after
the first shot was fired?
A He positioned himself
near the head of Molet
Bunye, and he was already
facing me and he fired
another shot.
Q At that time if your
narration is correct Bunye
was already lying on the
ground?

of Molet Bunye, and it was


fired, Molet Bunye was
lying on the ground, and the
man fired another shot.
During this period were you
looking at Molet Bunye?
A Yes, sir, because I was
shocked, surprised at what
had happened so I looked at
Molet Bunye.
Q Can you tell us in seconds
or minutes, how long did
this happening last?

A I also noticed that the


other person was
approaching.
Q How far where you
positioned from Molet
Bunye when he fell to the
ground?
A About three (3) to four (4)
meters.
Q Do you know the width
of T. Molina Street?
A No, sir.

A Yes, sir.

A I can not estimate.

Q So the first shot was fired


when Molet Bunye was
walking?

Q All the while you were


looking at Molet Bunye
while this was happening?

A Yes, sir.

A Yes, sir.

Q That was to the head?

Q Where you then worried


about the change of Molet
Bunye while you were
looking at him?

Q At that time you claimed


that you were at the right
side of the road?
A Yes, sir.

A Yes, sir.
Q Do you know what part
of the body of Molet Bunye
was hit by the shot?
A I don't remember what
part of the body was hit, but
he fired it at the head.

A Yes, sir. It is like that.


Q After the second shot did
you turn your tricycle and
went home?
A Yes, sir.

xxx xxx xxx


Q And you went home?
Q All the while when the
gun was pointed at the head

Q And the incident


happened at the edge of the
left side?
A Yes, sir.
Q And you said that the
distance between your
tricycle and the edge of T.
Molina is only four (4)
meters?
A My estimate is more or
less three (3) to four (4)
meters.

Q The second person, did


you see him approach Molet
Bunye?
A Yes, sir.
Q What did he do with
Molet Bunye?
A I also noticed that the
second person also pointed
a gun at the head of Molet
Bunye.
Q The second man did he
fire a shot?

Narciso Nazareno and Ramil Regala. De Limos said


that he was told by Nazareno to follow the
"stainless" tricycle ahead and that upon overtaking it
as it stopped at the corner of T. Molina St. and
Mendiola St., he also stopped his vehicle. One of the
passengers, Nazareno, jumped out of the tricycle,
pulled something from his waist and fired at Bunye.
When Bunye fell down, Nazareno fired another shot
at the head of Bunye, after which Ramil Regala got
off the tricycle, approached Bunye and also shot him
in the head. Accused-appellants then boarded the
tricycle and ordered him to take them to the rotunda,
where they alighted from the tricycle and fled.
De Limos was unshaken by questions asked during
the cross examination. 23 His testimony, slightly
edited, follows:

A Yes, sir.
Q To the head of Molet
Bunye?
A Yes, sir.
Q Do you know if he hit the
head?
A Yes, sir.
Q How about the face? Do
you know if he hit the face?
A It was pocked very near
the head.
Hernandez's testimony was corroborated by Rogelio
de Limos. De Limos testified that on December 14,
1988, between 8:00 to 9:00 in the morning, two men,
standing in front of Bunye's house on Ilaya Street,
hailed his tricycle. He identified in court the two as

Q When you were operating


your tricycle last Dec. 14,
1988, you were flagged by
two (2) persons who
became your passengers.
Tell us where is this
stainless tricycle at the time
you were flagged down in
relation to your tricycle?
A The stainless tricycle was
already running.
Q When you stopped your
tricycle and you picked up
these two (2) persons, how
far away was the stainless
tricycle from you?
A I don't remember.

Q Can we say about twenty


(20) meters away and still
running at the time you stop
your tricycle?
A Not more than twenty
(20).
Q What would be your
estimate if it is less than
twenty (20) meters away?
A I am not sure of my
estimate.
Q When these two (2)
passengers got in your
vehicle and at the moment
you started your tricycle,
how far away is this first
tricycle from you?
A It is not far.
Q Give us your estimate in
meters the distance between
you when you started your
tricycle and the stainless
tricycle that you were to
follow?
A About more or less
twenty (20) meters.
Q Were their other vehicles
including tricycle[s] that
were between you and the
stainless tricycle when you
commence running?

A I don't remember, sir.


Q So that the space between
you and the first tricycle
was totally clear of vehicle?

Q When the first tricycle


already stopped you were
able to get near that
vehicle?

Q And the passenger of the


stainless vehicle got off?

A Yes, sir.

Q Who got off their


respective vehicle[s] first,
your passenger or the
passenger of the stainless
tricycle?

A Yes, sir.
Q How many meters did
this first tricycle travel from
where you picked up your
passengers until it stopped
and unloaded its
passengers?
A More or less 200 meters.
Q What street were you
traveling at that time?
A Ilaya Street, Alabang
Muntinlupa.

Q But at any time when you


were running and traveling
along Ilaya Street you were
able to get [to] this stainless
tricycle?

A Yes, sir.
Q How far away from your
starting point where you
able to approach the vehicle
that you are going to
follow?
A Near the corner of
Mendiola and Timolin
Street.

A The passenger of the


stainless tricycle.

A No, sir.
Q When your first
passenger alighted from
your vehicle you were still
running . . . your vehicle
was still running. Is that
correct?
A Yes, sir.

Q When you were travelling


along Ilaya Street following
this stainless vehicle were
you exactly behind?

A Yes, sir.

Q Where was this stainless


vehicles situated at that
moment when your first
passenger alighted from
your vehicle?

Q When the passenger of


the stainless tricycle got off,
the tricycle was on your
right side. Is that correct?
A No, sir. The stainless
tricycle was behind me.
Q Directly behind your
tricycle?
A Yes, sir.
Q How far away was it from
you?

A The tricycle was behind


me.

A About one (1) foot behind


me.

Q You mean to say that you


overtook the stainless
tricycle?

Q At that moment when


your first passenger alighted
from your tricycle where
was the victim in relation to
where you were situated at
that moment?

A Yes, sir.

A He was in front of me
walking.

shot where was the stainless


tricycle?

Q So you can see him


directly in front of you
walking?

A He was still behind me.

A Yes, sir.
Q Your tricycle was on the
right portion of the left side
of the street. Is that correct?
A Yes, sir.
Q And the victim was
walking also on the right
portion of the street of the
road?
A At the left side.

Q How did you know that


the vehicle was still behind
you?
A Because I saw and I could
not leave the place.
Q When did you see the
stainless tricycle, was it still
behind you in relation to the
first shot?
A At the time when I heard
the first shot.

Q How far away from you


was the victim when you
said he was shot for the first
time?

Q When you heard the first


shot. You want to impress
[to] us that you looked back
and you saw the stainless
tricycle still there behind
your tricycle?

A More or less two (2)


meters.

A Because the tricycle could


not run or move.

A Because the tricycle was


"NAKATOTOK" behind me
and I could not move my
tricycle.
Q It was only your
impression that the stainless
tricycle was still there but
you did not see it actually?
A No, sir. I see.
Q When did you see it?
A At that time because I
could not overtake coz of
that incident.
Q After the first shot, did
you see the stainless tricycle
still there?
A Yes, sir.
Q You mean you looked
back?
A Yes, sir.
Q Why did you look back?

Q Will you please clarify.


Was he walking in front of
you or to the left of you
when he was first shot?

Q But when the first shot


was fired you did not look
back?

A Because the passenger of


the stainless tricycle
scramble[d].

A I did not.
A To the left.
Q At that moment, meaning,
when the victim was first

Q So you did not really


know that the tricycle was
still there?

Q After the second shot was


the stainless tricycle still
there?
A Yes, sir.

Q Did you look back?


A Yes, sir, through my side
view.

Q How about to his rear was


he blocked by any other
vehicle?

Q You claim that your two


(2) passengers rode your
vehicle again after the
shooting. Is that correct?

A No, sir.
Q Did you look back and
see that stainless tricycle
still there after the second
shot?
A I saw the driver of the
stainless tricycle moving his
tricycle.
Q You said there were three
(3) shots fired. After the
third shot where was the
stainless tricycle?
A After the third shot, the
driver of the stainless
tricycle was still moving his
tricycle back and forth to
get out of the place.
Q Was he able to get out?

A Yes, sir.
Q Of your knowledge he
was free to move out by
backing away from your
vehicle?
A Yes, sir.
A I don't remember.
Q You claimed that
immediately after the
second shot was fired, this
stainless tricycle was
already moving his vehicle
back and forth to get out of
the place. Is that correct?

Q How far was it after the


third shot was fired that
these passengers boarded
your vehicle again?
A Minutes only.

A Yes, sir.

Q How many minutes?

Q How far was this stainless


vehicle from the victim
when the latter fell down
the street after the first shot?

A More or less five (5)


minutes.

A Yes, sir.
A I don't remember.
Q How long after the third
shot was he able to get out
from where he was?

Q Who boarded your


vehicle first, the first one
who alighted or the second
one who alighted?

Q When your second


passenger got off you did
not see him get off?

A I don't remember.

Q What were their


instructions to your when
they boarded?
A Before answering your
question I made [a] mistake
on the preceding question.
Not minutes only seconds.

A I did not.
Q Was this stainless tricycle
blocked by your tricycle so
that he can not get out.

Q You felt movement of


your tricycle?

A Yes, sir.

A Yes, sir.

Q Will you now answer my


question?
A They told me to bring
them to "Rotonda."

Q In what dialects or
language was the instruction
given to you?

Q How many minutes?

A Tagalog.

Q How many seconds?

Q Was that all they said to


you during that occasion
when they boarded your
vehicle for the second time?

A More or less ten (10)


seconds.

A Seconds only.

Q Did your passenger pay


you his fare?

A Yes, sir.
A No, sir.
Q From the scene of the
shooting to where you
discharged your passengers,
how far was it?
A I can not estimate the
distance.
Q You were driving your
tricycle along that road for
many years already?

Q Did you not run after him


to be able to collect the
fare?
A No, sir.
Q When your two (2)
passengers alighted, where
did you go next?
A I went home.

inconsistencies on minor points which do not affect


the truth of the testimonies. They are discrepancies
to be expected from uncoached witnesses. 26 What is
important is that the testimonies corroborate each
other on important and relevant details concerning
the principal occurrence. 27
Third. Accused-appellant Nazareno makes much of
the fact that he was recognized by Hernandez only in
the police line-up. Nazareno claims that Hernandez
was his classmate in the elementary grade in one
subject 28 and therefore should have been able to
identify and name him on December 14, 1988, when
Hernandez gave his first sworn statement. It is
contended that because Hernandez did not then
recognize Nazareno shows that Hernandez's
testimony was "manufactured."
Other than Nazareno's claim, however, there is no
other evidence showing that he and Hernandez were
classmates. This matter was brought up only once
during Hernandez's cross examination in the hearing
for bail, where the following appears: 29
ATTY. MATUNOG:

A Yes, sir.
Q Why did you say that you
can not estimate the
distance between these two
places?
A I am not sure.
Q How many minutes did it
take you to drive them to
the place where they
alighted from your vehicle?
A Few minutes only.

As we have held in other cases, 24 testimonies given


in simple, straightforward manner, giving details of
the incident that could not have been merely
concocted, indicate sincerity in the narration of
events of the incident and truth as to what actually
happened, specially, if those giving them have not
been shown to have any improper motive to testify
falsely against the accused. The weight to be given
to testimonies of such witnesses depends chiefly
upon their observation and means of knowing the
facts testified to by them. 25 Accused-appellants point
to inconsistencies in the testimonies regarding
distances and time lapse as an indication that the
testimonies are lacking in veracity. These are

Q Mr. Hernandez is it
correct that you signed a
statement dated 28
December 1988 before
Capt. Jose Manuel?
FISCAL:
Admitted.
Q On Exhibit "C" [which]
appears to be signed by you
question No. 9. The
question raised to you,

YONG TAONG BUMARIL


DITO KAY MOLET,
NAKIKILALA MO BA
ITO? and your answer:

The best evidence is the


statement.

YAON ISA LANG HO.


(affiant pointing to the
person of Narciso Nazareno
y Barro 22 taong gulang,
binata, walang trabaho,
tubong Catanduanes at
kasalukuyang naninirahan
sa Prk. 6-B, Alabang
Muntinlupa, MM.)

MAY ANSWER.

Court:

A When I looked to my left,


I saw him.
Q Since when have you
known the accused
Nazareno?
A Only on that day.

Is this correct? This was the


answer you gave?
ATTY. BAUTISTA:
May it please be placed on
record that the witness
refused to answer.
FISCAL:
May I remind the witness he
is under oath.
COURT:
MAY ANSWER.
A Nazareno (pointing to
Nazareno). Yes, sir.
Q Why do you know the
accused Nazareno?
FISCAL:

After all, as Nazareno claimed, they were classmates


in only one subject in the elementary grades and that
had been eight years before the incident.

Q You do not remember you


were a classmate of
Nazareno sometime in
1979-1980?
A I cannot remember sir. I
don't remember.
There is nothing in the sworn statements of
Hernandez nor in his testimony in court which
shows that he ever claimed to know Nazareno
because they had been classmates eight years before
he identified him in the police line-up. He said he
only came to know Nazareno at the police line-up on
December 28, 1988. He was able to identify
Nazareno because he remembered his face as that of
the one who shot Bunye on December 14, 1988.
Indeed, Hernandez should first have been asked
whether he and Nazareno had not been classmates in
the elementary grades. There was no basis for
defense counsel's questions premised on this fact.
Be that as it may, even if they had been classmates,
it is possible Hernandez had forgotten Nazareno.

Fourth. Accused-appellant Nazareno assails the


testimony of the eyewitnesses as contrary to the
evidence. He contends that Bunye was 5'6" tall,
while he is only 5'4". Considering the downward
angle of the bullet wound at the back of Bunye's
head, he theorizes that the assailant must have been
taller than Bunye or Bunye must have been shot
while he was in a kneeling position.
This contention is without merit. In the first place,
there is no basis for the claim that Bunye was 5'6"
tall and Nazareno only 5'4". Bunye's cadaver was
not measured. Nothing in the record shows how the
figure 5'6" was reached. His death certificate, 30 the
Certificate of Post-Mortem Examination 31 and the
autopsy report 32 do not contain data as to his height.
It appears that the basis of accused-appellant
Nazareno in claiming that Bunye was 5'6" is the
testimony of the wife of the victim, Evelyn Bunye,
who said during her cross examination that barefoot,
the height of her husband was 5'6." Her only basis
for saying so was "because he was [her] husband." 33
Nor was there evidence as to Nazareno's height. It is
only in his appellant's brief that his height is said to
be 5'4." This Court then, for lack of any basis,
cannot give credence to this assertion.
In the second place, only accused-appellants claim
that, on the basis of the autopsy report, the trajectory
of the bullet was "sharply downwards" 34 and that
therefore the conclusion was that the gun was fired
from a certain position in such a way that "(1) the
gun was aimed at the head of victim; (2) the gun was
positioned with the barrel pointed downwards not
upwards and neither laterally; (3) the gun was held
at a point approximately five (5) feet four (4) inches

from the ground; and (4) was fired while at that


position." 35
The autopsy report does not say what accusedappellants say. It reads: 36
Wounds Gunshot:
I. Entrance, oval, edges inverted, 0.9 x 1.0
cm. in size, located at the scalp, parietal
region, along the posterior median line, 13.0
cm. above and 5.0 cm. behind the left
external auditory meatus, directed forwards,
downwards and laterally, involving the skin
and underlying soft tissues, fracturing the
parietal bone along the posterior median
line, perforating the left side of the parietal
bone along the posterior median line,
perforating the left sides, of the parietal and
temporal lobes of the brain, making an Exit,
roughly oval in shape, edges averted, located
at the face, left side, 1.0 cm. below and 2.5
cm. in front of the left external auditory
meatus.
II. Entrance, oval, edges inverted, 0.9 x 1.0
cm. in size, located at the face, right side,
3.0 cm. above and 5.5 cm. in front of the
right external auditory meatus, directed
backwards, downwards and from right to
left, involving the skin and underlying soft
tissues, fracturing the right side, of the
temporal bone, perforating the right side of
the temporal lobe of the brain, piercing the
left side of occipital lobe of the brain, with
two (2) fragments lodged and recovered in
that area.
There are no findings in the report regarding the
relative positions of the victim and the accused-

appellants. The medico-legal officer, when asked if


he could determine the position of the victim from
the gunshot wound located at the back, said, "Since
the entry of the gunshot wound is at the back,
naturally it is in the back." 37 The answer is vague
and does not shed light on the positions of those
involved.
No further clarification was made as accusedappellants' counsels waived the right to cross
examine the medico-legal officer. Evidence could
have been presented showing that the road was level
and not sloping or that the angle could not have been
caused by the recoil affecting the assailant. As it is,
the mere assertion of disparity in height cannot
persuade this Court to do away with the positive
identification of the accused-appellants by the
witnesses.
Moreover, the position of a victim when he was shot
vis-a-vis an assailant is difficult to ascertain
considering the mobility of the head. 38 It is entirely
possible that the victim's head in this case was
slightly inclined upward. As the autopsy report
shows, the trajectory of the bullet was "directed
forwards, downwards and laterally" and not, as
accused-appellants claim, "sharply downwards."
This downward angle could be due to the bullet
hitting a specific bone (scalp) causing a slight
deflection. 39
On the other hand, accused-appellant Ramil Regala
contends that the failure of the prosecution to
investigate Rey Taling, his alleged companion, and
Mang Doming, Mang Romy, and Nick Pealosa, as
those who supplied the guns, raises doubt as to
accused-appellant's guilt. However, the manner by
which the prosecution of a case is handled is within
the sound discretion of the prosecutor and the non-

inclusion of other guilty parties is irrelevant to the


case against an accused. 40
Fifth. Indeed, accused-appellants' defense consists of
denial and alibi. Nazareno claims that at the time of
the incident he was in the market selling fruits, while
Regala claims that he was at home in Cavite. 41
Accused-appellant Regala failed to present any
witness to corroborate his alibi, while accusedappellant Nazareno presented his mother Gloria
Nazareno who testified that she asked her son to go
to the market to tend their fruit stand because she
wanted to stay at home to rest. 42 His mother could
not thus testify positively whether he was in the
market at the time of the killing. Nor were accusedappellants strangers to each other. Regala hauled
fruits for Nazareno's family. 43 This circumstance
rules out the possibility raised by the defense that the
accused-appellants had simply been picked up at
random 44 by the police. Bare denial and alibi are
insufficient to overcome the positive identification
given by the prosecution witnesses. As the trial court
held, between the positive declarations of the
prosecution witnesses and the negative statements of
the accused, the former deserve more credence and
weight. 45
But we find that the qualifying circumstance of
evident premeditation was not proven and therefore
should not be appreciated. There is neither evidence
of planning or preparation to kill nor of the time
when the plot was conceived. 46 However, the
qualifying circumstance of treachery was correctly
appreciated by the trial court to qualify the killing to
murder. The testimonies of the prosecution witnesses
show that Bunye was not in a position to defend
himself and that the manner by which the killing was
done shows that his assailants consciously and
deliberately adopted a particular method or form of
attack to insure the accomplishment of their purpose.

7 The attack was sudden and unexpected, giving


likewise motu proprio addressed by the Court in this
Bunye no chance to defend himself, thereby insuring Decision.
the execution of the crime without risk to its
Challenged in the instant amended petition is the Decision 1
perpetrators.
of respondent Sandiganbayan 2 in Criminal Case No. 8496
WHEREFORE, the judgment of the Regional Trial
promulgated on June 19, 1987 convicting petitioner of
Court of Makati, Branch 136, being in accord with
brigandage, and the Resolution 3 promulgated on July 27,
the law and the evidence, is AFFIRMED.
1987 denying his motion for reconsideration.

Highway of said municipality, at the point of


their guns, and then take, rob and carry away
with them the following, to wit:

SO ORDERED.

3) Social Security System Pension Checks


and Vouchers

G.R. No. 79543 October 16, 1996


JOSE D. FILOTEO, JR., petitioner,
vs.
SANDIGANBAYAN and THE PEOPLE OF THE
PHILIPPINES, respondents.

PANGANIBAN, J.:p
A person under investigation for the commission of an
offense is constitutionally guaranteed certain rights. One of
the most cherished of these is the right "to have competent
and independent counsel preferably of his choice". The 1987
Constitution, unlike its predecessors, expressly covenants
that such guarantee "cannot be waived except in writing and
in the presence of counsel". In the present case, petitioner
claims that such proscription against an uncounselled waiver
of the right to counsel is applicable to him retroactively, even
though his custodial investigation took place in 1983 long
before the effectivity of the new Constitution. He also
alleges that his arrest was illegal, that his extrajudicial
confession was extracted through torture, and that the
prosecution's evidence was insufficient to convict him.
Finally, though not raised by petitioner, the question of what
crime - brigandage or robbery was committed is

The Facts
Petitioner Jose D. Filoteo, Jr. was a police investigator of the
Western Police District in Metro Manila, an old hand at
dealing with suspected criminals. A recipient of various
awards and commendations attesting to his competence and
performance as a police officer, he could not therefore
imagine that one day he would be sitting on the other side of
the investigation table as the suspected mastermind of the
armed hijacking of a postal delivery van.
Along with his co-accused Martin Mateo, Jr. y Mijares,
PC/Sgt. Bernardo Relator, Jr. y Retino, CIC Ed Saguindel y
Pabinguit, Ex-PC/Sgt. Danilo Miravalles y Marcelo and
civilians Ricardo Perez, Reynaldo Frias, Raul Mendoza,
Angel Liwanag, Severino Castro and Gerardo Escalada,
petitioner Filoteo was charged in the following Information:

1) Postal Delivery Truck


2) Social Security System Medicare Checks
and Vouchers

4) Treasury Warrants
5) Several Mail Matters from abroad
in the total amount of P253,728.29 more or
less, belonging to US Government
Pensionados, SSS Pensionados, SSS
Medicare Beneficiaries and Private
Individuals from Bulacan, Pampanga,
Bataan, Zambales and Olongapo City, to the
damage and prejudice of the owners in the
aforementioned amount.
Contrary to law

On separate dates, accused Filoteo, Mateo, Saguindel,


That on or about the 3rd day of May, 1982, Relator and Miravalles, assisted by their respective counsel,
in the municipality of Meycauayan, province pleaded not guilty. Their co-accused Perez, Frias, Mendoza,
Liwanag, Castro and Escalada were never arrested and
of Bulacan, Philippines, and within the
jurisdiction of this Honorable Court, the said remained at large. Accused Mateo escaped from police
custody and was tried in absentia in accordance with Article
accused, two of whom were armed with
guns, conspiring, confederating together and IV, Section 19 of the 1973 Constitution. Accused Saguindel
and Relator failed to appear during the trial on February 21,
helping one another, did then and there
1985 and on March 31, 1986, respectively, and were thus
wilfully, unlawfully and feloniously with
ordered arrested but remained at large since then. Like in the
intent of gain and by means of violence,
case of Mateo, proceedings against them were held in
threat and intimidation, stop the Postal
absentia. 5 Only Filoteo filed this petition, after the
Delivery Truck of the Bureau of Postal
while it was travelling along the MacArthur

respondent Court rendered its assailed Decision and


Resolution.
Before trial commenced and upon the instance of the
prosecution for a stipulation of facts, the defense admitted
the following: 6

accused Filoteo and Miravalles presented their respective


testimonies plus those of Gary Gallardo and Manolo
Almogera. Filoteo also submitted his Exhibits 1-14-Filoteo,
but Miravalles filed no written evidence. Thereafter, the
prosecution proffered rebuttal evidence and rested with the
admission of Exhibits A-16-a, A-31 and L.

ordered the postal employees to disembark from the van. As


he stepped out of the van, Miranda took the ignition key with
him, but when threatened, he surrendered it to one of the car
passengers. 13 The three postal employees were then ordered
to board the Benz.

As he was about to enter the car, Bautista looked back and


saw one of the malefactors, who turned out to be Reynaldo
Frias, going up the van. Inside the car, the three delivery
At about 6:30 in the morning of May 3, 1982, Bureau of Post employees were ordered to lower their heads. They sat
mail van no. MVD 02 left San Fernando, Pampanga to pick between two of their captors at the back of the car while two
up and deliver mail matters to and from Manila. On board
others were in front. Later, Nerito Miranda asked permission
the vehicle were Nerito Miranda, the driver, and two couriers to straighten up as he was feeling dizzy for lack of air. As he
named Bernardo Bautista and Eminiano Tagudar who were
stretched, he caught a glimpse of the pimply face of the man
seated beside the driver. They arrived at around 9:40 that
to his left. He also recognized the driver who had glanced
morning at the Airmail Distribution Center of the Manila
back. These men turned out to be Angel Liwanag and
7
International Airport where they were issued waybills for
Reynaldo Frias, respectively. 14
the sacks of mail they collected. They then proceeded to the
Central Post Office where they likewise gathered mail
As the car started moving, Bautista complained about feeling
8
matters including 737 check letters sent by the United
"densely confined." We was allowed to raise his head but
States Embassy. All the mail matters were placed inside the with eyes closed. However, he sneaked a look and
delivery van, and its door padlocked.
recognized the driver of the car as Raul Mendoza and the
fellow beside him who poked a "balisong" at him as Angel
As they had to deliver mail matters to several towns of
Liwanag. The man in uniform on the front seat was Eddie
Bulacan, they took the MacArthur Highway on the return
Saguindel. Earlier, as he was about to enter the car, Bautista
trip to Pampanga. When they reached Kalvario,
looked back and recognized Frias. 15 These incidents yielded
Meycauayan, Bulacan at about 4:30 in the afternoon, an old the pieces of information critical to the subsequent
blue Mercedes Benz sedan 9 overtook their van and cut
identification of Mendoza, Liwanag, Saguindel and Frias in
across its path. The car had five (5) passengers three
the line-up of suspects at Camp Crame later on.
seated in front and two at the back. The car's driver and the
passenger beside him were in white shirts; the third man in
The car seemed to move around in circles. When it finally
front and the person immediately behind him were both clad came to a stop, the captured men discovered that they were
in fatigue uniforms, while the fifth man in the back had on a along Kaimito Road in Kalookan City They were made to
long-sleeved shirt. 10
remove their pants and shoes and then told to run towards
the shrubs with their heads lowered. Upon realizing that the
Two of the car passengers aimed an armalite and a hand gun hijackers had left, they put on their pants and reported the
The prosecution sought to prove its case with the testimonies at driver Nerito Miranda as someone uttered, "Are you not
incident to the Kalookan Police Station.
11
going to stop this truck?" Frightened, Miranda pulled over
of Bernardo Bautista, Rodolfo Miranda, Capt. Rosendo
6
The Security and Intelligence Unit of the Bureau of Posts
Ferrer, M/Sgt. Noel Alcazar and Capt. Samuel Pagdilao, Jr. - and stopped the van's engine. Alighting from the car, the
12
armed group identified themselves as policemen. They
recovered the postal van at the corner of Malindang and
a and the submission of Exhibits A to K. In their defense,
The existence of the bound record of
Criminal Case No. 50737-B-82, consisting
of 343 pages from the Bulacan CFI (Exhibit
A); in 1982 or thereabouts, accused
Bernardo Relator was a PC Sergeant at
Camp Bagong Diwa, Bicutan, Metro
Manila; as such PC Sergeant, accused
Relator was issued a service revolver, Smith
& Wesson Revolver, 32 (sic), with Serial
No. 11707 (Exhibit B) and holster (Exhibit
B-1) with six (6) live ammo (Exhibit B-2);
in 1982 or thereabouts, accused Eddie
Saguindel was a PC Constable First Class;
on May 30, 1982, accused Saguindel,
together with accused Relator and Danilo
Miravalles, a former PC Sergeant, was
invited for investigation in connection with
the hijacking of a delivery van by the
elements of the Special Operations Group,
PC, and the three availed of their right to
remain silent and to have counsel of their
choice, as shown by their Joint Affidavit
(Exhibit A-20); and the existence of the
sworn statement executed by accused Martin
Mateo (Exhibit A-11) as well as the
Certification dated May 30, 1982, subject to
the qualification that said document was
made under duress.

Evidence for the Prosecution

Angelo Streets, La Loma, Quezon City on May 4, 1982.


Discovered missing were several mail matters, 16 including
checks and warrants, along with the van's battery, tools and
fuel. 17

accompany Frias to Obrero Tondo while he escorted


Alcantara to their headquarters at Camp Crame. On the way
to the headquarters, Alcantara denied participation in the
hijacking although he admitted living with Martin Mateo
who allegedly was in possession of several checks. Alcantara
In a letter-request dated May 6, 1982 to then Col. Ramon
was turned over to the investigation section of the SOG for
Montao, then Postmaster General Roilo S. Golez sought the further questioning.
assistance of the Special Operations Group (SOG) of the
Philippine Constabulary in the investigation of the hijacking Meanwhile, Lt. Pagdilao's group was able to corner Ricardo
incident. 18 Responding to the request, the SOG, which was
Perez in his house in Tondo. Confronted with the hijacking
tasked to detect, investigate and "neutralize" criminal
incident, Perez admitted participation therein and expressed
syndicates in Metro Manila and adjacent provinces,
disappointment over his inability to dispose of the checks
organized two investigative teams. One group was led by
even after a month from the hijacking. He surrendered the
Capt. Rosendo Ferrer and the other by 1st Lt. Samuel
checks in his possession to Lt. Pagdilao.'s. 19
Pagdilao. Initially, they conducted a "massive intelligence
build-up" to monitor the drop points where the stolen checks An hour and a half later, Capt. Ferrer received information
over their two-way radio that Ricardo Perez and Raul
could be sold or negotiated.
Mendoza were in Lt. Pagdilao's custody. Capt. Ferrer
On May 28, 1982, the SOG received a tip from a civilian
ordered that, instead of returning to headquarters, Lt.
informer that two persons were looking for buyers of stolen Pagdilao and his companions should meet him in Quirino,
checks. Capt. Ferrer requested the informer to arrange a
Novaliches to apprehend Martin Mateo. They met at the
meeting with them. The meeting materialized at about 9:00
designated place and proceeded to Gulod, Novaliches
P.M. of May 29, 1982 at the Bughaw Restaurant in Cubao,
arriving there at about 10:30 P.M. of May 29, 1982.
Quezon City. With cash on hand, Capt. Ferrer posed as the
buyer. The informer introduced him to Rey Frias and Rafael Walking atop a ricefield dike to the house of Mateo, they
Alcantara. Frias in turn showed Capt. Ferrer a sample Social noticed two men heading in their direction. Perez identified
them as Martin Mateo and Angel Liwanag. The latter threw
Security System (SSS) pension check and told him that the
something into the ricefield which, when retrieved, turned
bulk of the checks were in the possession of their
out to be bundles of checks wrapped in cellophane inside a
companions in Obrero, Tondo, Manila. After some
negotiations, they agreed to proceed to Tondo. Then as they plastic bag. 20 As the two were about to board the SOG
teams's car, Mateo said, "Sir, Kung baga sa basketball, talo
boarded a car, Capt. Ferrer introduced himself and his
companions as lawmen investigating the hijacking incident. na kami. Ibibigay ko yong para sa panalo. Marami pa akong
tseke doon sa bahay ko, sir, kunin na natin para di na natin
Shocked and distressed, Frias calmed down only when
babalikan." 21 Capt. Ferrer accompanied Mateo to his house
assured that his penalty would be mitigated should he
cooperate with the authorities. Frias thus volunteered to help where they retrieved several other checks in another plastic
crack the case and lead the SOG team to Ricardo Perez and bag.
Raul Mendoza.
On the way to the SOG headquarters in Camp Crame, Mateo
Capt. Ferrer instructed Lt. Pagdilao, his assistant operations and Liwanag admitted participation in the postal hijacking.
At a confrontation with Perez and Mendoza, all four of them
officer who was in another car during the mission, to

pointed to petitioner, Jose D. Filoteo, Jr., as the mastermind


of the crime.
Consequently, Capt. Ferrer directed Lt. Pagdilao to
accompany Mateo to the house of petitioner in Tondo,
Manila. The lawmen found petitioner at home. Upon being
invited to Camp Crame to shed light on his participation in
the hijacking, petitioner was dumbfounded (" parang
nagulat). Pursuant to standard operating procedure in arrests,
petitioner was informed of his constitutional rights, 22
whereupon they proceeded to Camp Crame. However, the
group, including petitioner, returned to the latter's place to
recover the loot. It was "in the neighborhood," not in
petitioner's house, where the authorities located the checks. 23
The authorities confronted Filoteo about his participation in
the hijacking, telling him that Frias, Mendoza and Perez had
earlier volunteered the information that petitioner furnished
the Benz used in the hijacking. Thereupon, Filoteo admitted
involvement in the crime and pointed to three other soldiers,
namely, Eddie Saguindel, Bernardo Relator and Jack
Miravalles (who turned out to be a discharged soldier), as his
confederates. At 1:45 in the afternoon of May 30, 1982,
petitioner executed a sworn statement in Tagalog before
M/Sgt. Arsenio C. Carlos and Sgt. Romeo P. Espero which,
quoted in full, reads as follows:

BA
BA
LA

Na
ko
g
ipa
lam
sa
iyo
Pa
olm

an
Filo
teo,
na
ang
dahi
lan
ng
pags
isiy
asat
na
ito
ay
tung
kol
sa
isan
g
kaso
ng
Rob
bery
-inBan
d/Hi
Jack
ing
na
nag
ana
p
noo
ng
ika3 ng
May
o

198
2
doo
n sa
Mey
caua
yan,
Bul
acan
,
mga
ban
dan
g
alas
4:00
ng
hap
on,
hum
igitkum
ulan
g,
kun
g
saan
g
mar
ami
ng
tsek
eng
US,
tsek
e ng
BIR
at

iba
pa
g
mg
pe
on
na
tse
e
an
na
aw
mu
as
iyo
Na
ko
rin
ibi
ay
sa
iyo
an
ba
ala
ali
sun
od
sa
mg
isi
asa
dn
Se
ion
20
Ar
cle
IV

ng
Bag
ong
Sali
gan
g
Bata
s ng
Rep
ubli
ka
ng
Pili
pina
s,
kag
aya
ng
mga
sum
usu
nod:
a. Na ikaw ay may karapatang tumahimik;
b. Na ikaw ay may karapatang kumuha ng
isang abugadong sarili mong pili upang may
magpapayo sa iyo habang ikaw ay
sinisiyasat;
c. Na ikaw ay may karapatang huwag
sumagot sa mga katanungang maaring
makasira sa iyo sa dahilang anumang iyong
isasalaysay ay maaring gamitin pabor or
laban sa iyo sa kinauukulang hukuman;
d. Na kung ikaw ay walang maibabayad sa
isang abugado, ako mismo ang makipagugnayan sa CLAO-IBP upang ikaw ay

magkaroon ng isang abugadong walang


bayad.
1. TANONG: Ang mga
bagay-bagay bang akin
nang naipaliwanag sa iyo ay
iyong lubos na naiintindihan
at nauunawaan?
SAGOT: Opo.
2. T: Handa mo bang
lagdaan ang ilalim ng
katanungan at sagot na ito
bilang katibayan na iyo
ngang naiintindihan ang
iyong mga karapatan at
gayun na rin sa dahilan ng
pagsisiyasat na ito, at ikaw
din ay nakahanda ngang
magbigay ng isang malaya
at kusang-loob na salaysay,
sumagot sa mga katanungan
at sumusumpang lahat ng
iyong isasalaysay ay
pawang mga katotohanan
lamang?

3. T: Maari bang sabihin


mong mull ang iyong buong
pangalan, edad at iba pang
bagay-bagay na maaring
mapagkakikilalanan sa iyo?
S: Jose Filoteo y Diendo,
30-anyos, may-asawa, isang
Patrolman ng Western
Police District,
Metropolitan Police Force
na kasalukuyang nakatalaga
sa General Assignment
Section, Investigation
Division ng naturang
Distrito ng Pulisya at
kasalukuyang nakatira sa
No. 810 Cabesas St.,
Dagupan, Tondo, Manila.
4. T: Kailan ka pa naappoint sa service bilang
isang Kabatas?
S: Noon pong October
1978, hindi ko maalaala ang
exactong petsa, noong ako
ay mapasok sa serbisyo.

S: Opo, pipirma ako Ser.


5. T: Kailan ka pa naman
na-assign sa GAS, WPD,
MPF?

MGA SAKSI:

S: Noon lamang pong


January 1982.

(Sgd.) (Sgd.)
ROMEO P. ESPERO
THERESA L. TOLENTINO
Ssg., PC C1C, WAC (PC)

6. T: Patrolman Filoteo,
ikaw ba ay tubong saang
bayan, lungsod or
lalawigan?

S: Pagkakaalam ko sa
tatay ko ay Bulacan
samantalang ang aking ina
naman ay Bisaya, pero ako
ay ipinanganak na sa
Maynila noon July 17,
1951.
7. T: Ano naman ang
natapos mong kurso sa pagaaral?
S: Undergraduate ako ng
BS Criminology sa PCCr,
dahil hindi ko natapos ang
second semester ng 4th year
ko.
8. T: Ano naman ang
iyong specific designation
sa GAS, ID, WPD-MPF?
S: Sa Follow-Up Unit
ako.
9. T: At bilang miyembro
ng follow-up unit no GAS,
ano naman ang iyong mga
specific duties?
S: Kami po ang
magsasagawa ng follow-up
kung may mga at-large sa
mga suspects namin sa mga
kasong hawak ng
investigation.
10. T: Noong ika-3 ng
Mayo 1982, mga bandang
alas-4:00 ng hapon humigit-

kumulang, saan ka naroroon


at ano ang iyong ginagawa?
S: Nasa Plaza Lawton ho
kami, eh, at inaantay na
namin iyong hi-nayjack
namin na Philippine Mail
delivery van.
11. T: Wika mo'y kami,
sinu-sino ang tinutukoy
mong mga kasamahan?
S: Si Carding Perez, ho;
si Junior ho (Affiant pointed
to Martin Mateo, Jr. who
was seated in the
investigation room and
asked the name and was
duly answered: Martin
Mateo, Jr.); si Rey Frias,
Raul Mendoza; Angelo
Liwanag at ang mga taga
LRP ng PC Brigade na sina
Sgt. Ed Saguindel, Sgt. Dan
Miravales at isa pang
Sergeant na ang alam ko
lang sa kanya ay JUN ang
tawag namin. Walo (8)
(corrected and initialled by
affiant to read as "SIYAM
[9]") kaming lahat doon
noon at ang mga gamit
naman naming kotse noon
ay ang kotse ng kumpare
kong si Rudy Miranda na
isang Mercedes Benz na
may plakang NMJ-659
kung saang ang driver
namin noon ay si Raul

Mendoza (corrected and


initialled by affiant to read
as "AKO") at ang mga
kasama naman naming
sakay ay sina Angelo
Liwanag, Sgt. Ed Saguindel
at Sgt. Jun na parehong
taga-LRP (affiant added and
initialled this additional
fact: "AT RAUL
MENDOZA"). Ang isang
kotse namang gamit namin
ay pag-aari daw ng pinsan
ni Carding Perez na kanya
na rin mismong minaneho
na isang Lancer na dirtywhite ang kulay at ang mga
sakay naman ni Carding
Perez ay sina Junior Mateo,
Rey Frias at Sgt. Dan
Miravalles ng LRP rin. Pero
may kasama pa kaming
contact ni Carding Perez na
taga-loob ng Post Office na
sina Alias NINOY na isang
dispatcher at Alias JERRY,
dahil ang mastermind dito
sa trabahong ito ay si
Carding PEREZ at kami
naman ng mga sundalong
taga-LRP ay kanila lamang
inimporta upang umeskort
sa kaniia sa pag-hijack ng
delivery van.
12. T: Anong oras naman
noong umalis ang delivery
van ng Post Office
patungong norte?

S: Kung hindi ako


nagkakamali ay nasa
pagitan na noon ng alas4:00 hanggang alas-5:00 ng
hapon.
13. T: Isalaysay mo nga
ng buong-buo kung ano ang
mga naganap noong hapon
na iyon?
S: Noon pong lumakad
na ang delivery van ng
Central Post Office,
sinundan na namin, una ang
van, sumunod ang Lancer at
huli ang Mercedes Benz
namin. Pagdating namin sa
Malinta, Valenzuela Metro
Manila ay nagpalit kami ng
puwesto sa pagsunod, van
naman ngayon, sunod ang
Mercedes Benz at huli na
ang Lancer. Noong
makapasok na kami ng
boundary ng Meycauayan,
Bulacan ay kumuha na kami
ng tiyempo at noon
makatiyempo kami ay kinat
namin ang delivery van.
Tumigil naman ito at
bumaba kaagad sina Sgt. Ed
Saguindel at Sgt. Jun ng
LRP datiil sila noon ang
may hawak ng kanilang
Armalite Rifle pero may
service pa silang maiksing
baril. Pinababa nila ang
tatlong maydala ng delivery
van at pinasakay sa

Mercedes Benz, habang


nakatutok ang kanilang mga
baril sa kanila. Ako naman
ay bumaba na sa aming
kotse at sumakay ng
delivery van at ako na
mismo ang nagmaneho at
sinamahan naman ako nina
Junior Mateo at si Rey
Frias, tatlo (3) rin kaming
pumalit sa puwesto noong
tatlong (3) taga-Post Office
na maydala ng delivery van.
Nag-Utturn (sic) kami
ngayon at ibinalik na namin
sa Manila ang van. Iyong
Mercedes Benz na
minaneho pa rin ni Raul
Mendoza ay dumeretso pa
norte samantalang ang
Lancer naman ay nag-Uturn din at sumunod sa
amin. Noong makarating na
kami sa Malinta,
Valenzuela, Metro Manila
ay inunahan na kami ng
Lancer at iyon na nga,
parang follow the leader na
dahil siya na noon ang
aming guide.
14. T: Ipagpatuloy mo
ang iyong pagsasalaysay?
S: Dumeretso kami
ngayon sa may Obrero, sa
bahay mismo nina Carding
Perez, at noong nakarating
na kami roon ay iniyatras ko
na ang van sa kaniling

garahe at doon ay ibinaba


namin lahat ang mga duffle
bag, hindi ko na ho alam
kung ilan lahat iyon, na
siyang laman ng delivery
van at pagkatapos ay umalis
kaming muli ng mga
kasama ko rin sa van
papuntang Quezon City
kung saan namin inabandon
ang delivery van. Sa Retiro
ho yata iyong lugar na iyon,
kung hindi ako
nagkakamali.
15. T: Ano ang mga
sumunod na nangyari?
S: Sumakay kami
ngayon ng taksi at bumalik
na kami kina Carding Perez
sa may bahay nila sa
Obrero, Tondo, Manila at
inabutan na namin sila na
nagkakarga na noong mga
duffle bag sa (sic), madilim
na ho noon, sa isang
kotseng mamula-mula o
orange na Camaro at isa
pang Mercedes Benz na
brown, dahil ang Lancer ay
isinoli na raw nila sa mayari. Dinala nila ngayon ang
mga duffle bag sa Bocaue,
Bulacan, iyon kasi ang
usapan namin noon dahil
sumilip lamang ako noon at
kasama ko si Carding Perez,
kami naman ngayon ay
pumunta sa bahay nina

Rudy Miranda sa San


Marcelino, Malate, Manila
na sakay ng isang Toyota
Corona na brown na si
Carding Perez ang
nagmaneho. Pagdating
namin doon sa kina Rudy
Miranda ay naroon na rin
noon ang Mercedes Benz na
ginamit namin, pero wala na
ang crew ng delivery van
dahil ibinaba at iniwanan
daw nila sa Caloocan City.
Ang naroroon na lamang
noon ay sina Angelo
Liwanag, si Raul Mendoza,
si Sgt. Ed Saguindel at si
Sgt. Jun na parehong tagaLRP. Naiwan na noon ang
Mercedes Benz namin doon
kina Rudy Miranda at
iniwan na rin ang susi doon
sa kamag-anak, dahil hindi
nila alam ang trabahong ito.
Sumakay na iyong apat
naming kasama sa Toyota
Corona na sakay namin at
inihatid namin sina Sgt.
Saguindel at Sgt. Jun doon
sa tinitirhan nitong huling
nabanggit na sundalo doon
sa malapit sa Del Pan
Bridge sa may Recto
Avenue sa San Nicolas yata
iyon sa Manila. Kami
naman ngayong apat, sina
Carding Perez, Angelo
Liwanag at si Raul
Mendoza ay tumuloy na sa

Bocaue, Bulacan. Dumaan


kami sa North Diversion
Road at paglabas namin sa
exit papuntang Bocaue,
Bulacan ay hindi na
kalayuan doon, hindi ko
alam ang lugar pero alam
kong puntahan. Bahay daw
yata ng kamag-anak ni
Carding Perez iyon pero
hindi ko alam ang pangalan.
Naroon na ngayon ang
buong tropa, maliban sa
mga dalawang sundalong
naihatid na namin sa may
Manila, at may mga
nadagdag pang ibang
mukha pero hindi ko ito
mga kakilala. Si JACK o
Sgt. Dan Miravalles ay
naroon din noon. Kumain
kami, pagkatapos ay
nagbukasan na ng mga
duffle bag. Iyon na nga,
nakita na namin ang mga
tsekeng ito, (Affiant pointed
to the checks he voluntarily
surrendered) at aming
inihiwalay ngayon sa mga
sulat na naroon na sinunog
lahat pagkatapos doon sa
bahay ni Junior Mateo sa
Novaliches. Di magdamag
ngayon ang trabaho namin,
kinabukasan ay kanyakanyang uwian na,
pagkatapos ay pahinga.
Kinabukasan mull, gabi,
inilipat na namin doon sa

bahay ni Junior Mateo ang


mga tsekeng ito (Affiant
again referred to said
checks). Isinakay namin
noon sa isang cargo truck na
pag-aari din daw nina
Carding. lyong mga tsekeng
iyan ngayon ay nakalagay
noon doon sa isang
sikretong compartment sa
gitna ng truck, doon ba sa
may chassis. Sikretong
compartment iyon, na
mahirap mahalata.
16. T: Ikaw ba naman ay
mayroong dalang baril noon
at kung ganoon, sabihin mo
nga kung anong uring baril
iyon?
S: Wala po akong baril,
Ser.
17. T: Paano naman
napunta ang mga tsekeng ito
(the checks recovered from
the Affiant was referred to)
sa iyo?
S: E, di ganoon na nga
he, habang tumatagal ay
umiinit ang situwasyon sa
aming grupo, dahil iyong
partehan sana namin ay
puro pangako ang nangyari.
Kaya napagpasyahan namin
na hatiin na lamang iyong
mga tseke upang walang
onsehan sa amin. Ito ngayon

ay parte namin nina Sgt. Ed


Saguindel, Sgt. Dan
Miravalles Alias JACK at ni
Sgt. Jun, dahil noong una ay
doon muna sa amin ito
nakatago (The checks
recovered from the Affiant
was referred to). Pero
habang tumatagal ay umiinit
at nalaman namin pati na
may alarma na, kaya't
inilipat namin doon sa may
Raxa Bago sa may likod ng
Alhambra Cigar & Cigarette
Factory sa Tondo, Manila at
akin munang ipinatago sa
isang kumare ko doon,
pansamantala, pero hindi
alam nitong kumare ko ang
laman noon dahil mahigpit
kong ipinagbilin na huwag
nilang bubuksan. Doon na
rin namin kinuha iyon
noong isurender ko ang mga
tsekeng ito kagabi, at
hanggang sa kinuha na
namin ang supot na ito (the
checks placed in a plastic
bag was again referred to)
ay wala pa rin kamalaymalay ang kumare ko.
18. T: Iyong sinasabi
mong mga kontak nina
Carding Perez sa Central
Post Office, mga kakilala
mo rin ba ang mga ito?
S: Iyong araw lamang na
iyon ko sila nakita, dahil

maghapon ko noon silang


nakikita, itong si Alias
NINOY lamang ang
dispatcher, dahil palabaslabas siya noon at
nakikipag-usap kina
Carding Perez, Raul
Mendoza at saka si Rey
Frias. Makikilala ko itong si
Alias NINOY kung makita
ko siyang muli.
19. T: Sino naman ang
kumontak sa iyo upang
sumama sa trabahong ito?

WAKAS NG SALAYSAY: .
. . ./ac

MGA SAKSI SA LAGDA:


(Sgd.)
SSG ROMEO P. ESPERO PC
(Sgd.)
C1C THERESA TOLENTINO WAC (PC) 24

Petitioner executed two other documents on the same day,


May 30, 1982. One was a certification stating that he
voluntarily surrendered "voluminous assorted US checks and
S: Si Junior Mateo po,
vouchers," that because of the "large number of pieces" of
ipinakilala niya ako kina
checks, he affixed his signature upon the middle portion of
Carding at sa buong tropa
the back of each check "to serve as identification in the
na namin.
future, prior to the completion of its proper inventory and
listing conducted by elements of SOG" in his presence, and
20. T: Pansamantala ay
that he "guided the elements of SOG" to the residence of
wala na muna akong
Rodolfo C. Miranda, the owner of the sky-blue Mercedes
itatanong pa sa iyo,
Benz car which was surrendered to the SOG Headquarters. 25
mayroon ka bang nais na
idagdag, bawasin o palitan The other document was a sworn statement wherein
petitioner attested to his waiver of the provisions of Article
kaya sa salaysay na ito?
125 of the Revised Penal Code and the following facts: (a)
that he was apprised of his constitutional rights under
S: Wala na po.
Section 20, Article IV of the (1973) Constitution, that he
21. T: Handa mo bang
understood all his rights thereunder, and that the
lagdaan ang iyong salaysay investigators offered him counsel from the CLAO-IBP but
na ito bilang patotoo sa
he refused to avail of the privilege; (b) that he was arrested
katotohanan nito nang hindi by SOG men in his house at around 11:00 p.m. of May 29,
ka pinilit, sinaktan or
1982" sa dahilang ako ay kasangkot sa pagnanakaw ng mga
pinangakuan kaya ng
US Treasury Warrants, SSS Pension Checks and Vouchers at
anuman upang lumagda
SSS Medicare Checks and Vouchers mula sa delivery van ng
lamang?
Philippine Mail;" (c) that the SOG men confiscated from
him numerous checks and a Mercedes Benz 200 colored skyS: Opo.
blue, and (d) that he was not hurt or maltreated nor was

anything taken from him which was not duly receipted


for. 26

ito yong baril na nagamit." 32 The three suspects were


brought to Camp Crame for further investigation. Thereafter,
Capt. Ferrer submitted an after-operations report about their
mission and executed jointly with Lt. Pagdilao an affidavit
on the same matter. 33

and directed them to proceed to Camp Crame. At the office


of the SOG, they were told to go over some pictures for
identification of the culprits. The three recognized and
As certified to by petitioner (in the above described
pointed to the suspects in a line-up. Tagudar identified
document), he led the SOG operatives to the house of
Saguindel and Liwanag. 38 Miranda pointed at Frias and
Rodolfo Miranda on Singalong where the latter admitted that
Liwanag 39 while Bautista identified Frias, Mendoza and
petitioner was his friend. He denied, however, having
Aside from petitioner, Liwanag, Mateo and Perez executed
Liwanag. 40 Petitioner himself, when told to identify his
34
knowledge that his car was used in the hijacking until the
sworn statements. Prior to doing so, they waived their right alleged cohorts, pointed to Severino Castro as their contact at
authorities came to his house. According to Miranda, he was to counsel. Liwanag and Mateo admitted their participation
the post office. 41 Five of the suspects who were not
made to believe that his car would be used for surveillance
and implicated petitioner in the crime. Perez, on the other
identified in the line-up were however implicated by
purposes because petitioner's jeep was not available. The car hand, denied having driven a Lancer car in the hijacking and Liwanag, Mateo and petitioner.
was not returned until the evening following that when it
stated that he was implicated in the crime only because in
2
was borrowed. 7 After the trip to Miranda's house,
one drinking spree with petitioner, Mateo and one alias
SOG Chief Investigator Jorge C. Mercado filed a complaint
petitioner informed the investigators that some more checks "Buro" during that month of May, they had a heated
for robbery-in-band (hijacking) before the Municipal Court
could be recovered from his kumare. Said checks were
altercation. Like petitioner, Liwanag and Mendoza certified of Meycauayan, Bulacan against petitioner and ten (10)
retrieved and turned over to headquarters along with the car that they voluntarily surrendered vouchers and checks which others, namely, Mateo, Saguindel, Relator, Miravalles, Perez,
surrendered by Miranda who later executed a sworn
were part of their loot in the hijacking; they also executed
Frias, Mendoza, Liwanag, Castro and Escalada (Criminal
28
statement dated May 31, 1992 at the SOG.
waivers under Article 125 of the Revised Penal Code. For his Case No. 7885). 42
part, Relator executed a certification to the effect that he
Upon learning of the whereabouts of Miravalles, Eddie
On August 8, 1983, the Information previously referred to
voluntarily surrendered his .32 caliber Smith & Wesson
Saguindel and Bernardo Relator, the team of Capt. Ferrer
and aforequoted was filed with the Sandiganbayan and
service revolver used in the commission of the crime. In
proceeded to Taguig, Metro Manila in the afternoon of May spite of the fact that his father-in-law was a lawyer,
docketed as Criminal Case No. 8496.
30, 1982. They met Miravalles along the way to his house.
petitioner did not manifest that he needed the assistance of
On September 20, 1983, Sandiganbayan Associate Justice
Informed by Capt. Ferrer that six of his companions were
counsel. During the taking of his statement, petitioner was
Romeo M. Escareal issued orders for the arrest of the
already under custody and that they implicated him as one of visited by Jimmy Victorino and another comrade from the
accused 43 and fixed bail at P13,000.00 each. Saguindel and
their confederates, Miravalles reacted by saying, "Sir, ang
General Assignment Section of the WPD.
Relator filed a motion to quash the Information asserting that
hihina kasi ng mga loob niyan, eh." 29
For their part, Relator, Saguindel and Miravalles executed a under the Articles of War and Section 1 of P.D. 1850, they
Capt. Ferrer later asked Miravalles to bring him to Eddie
should be tried by a court martial. 44 The Sandiganbayan
joint affidavit 35 manifesting their option to avail of their
Saguindel. At the barracks of the Long Range Patrol in
denied the motion on January 3, 1984 45 on the ground that
right to remain silent until such time as they would have
Bicutan, Metro Manila, Saguindel voluntarily accepted the
courts martial could no longer exercise jurisdiction over
retained a counsel of their choice. Frias and Mendoza
invitation to proceed to the SOG headquarters, after
them by virtue of their separation from military service.
executed a similar joint affidavit. 36 Severino Castro, the
Miravalles initially informed him of the facts obtained
postal employee implicated, also chose to remain silent as he
Evidence for the Defense
during the investigation. Saguindel was heard saying, "Hindi wanted to testify in court. However, he linked to the crime a
na kami interesado, sir, sa mga tsekeng iyan kasi isang
certain Gerardo Escalada, a former clerk of the Central Post
30
buwan na hindi pa nabebenta." With Miravalles and
Office and son of a director of the Bureau of Posts in Region Testifying in his own defense, petitioner alleged that as a
patrolman since August 21, 1978 assigned to the
Saguindel, Capt. Ferrer and his team moved on to Binondo, I. 37
Investigation Division or the Detective Bureau of the WPD
Manila to look for Bernardo Relator. When they found him
to which the General Assignment Section belonged, he was
at home, Relator excused himself, went upstairs, returned
On May 31, 1982, then Postmaster General Golez
31
with a .32 caliber revolver with six bullets and said, "Sir,
summoned postal employees Miranda, Bautista and Tagudar the recipient of several awards and recognitions starting with

ranking fifth in the Final Order of Merit in the basic course


for police officers. 46 He also claimed to have received a
loyalty medal for meritorious service above the call of
duty 47 and several commendations 48 for the distinguished
performance of his duties. On that fateful date of May 3,
1982, he was a member of the Special Task Force Unit
covering the tourist belt area.
Of the ten other accused in this case, petitioner admitted
knowing only Martin Mateo whose name appeared in the
initial follow-up operation he allegedly participated in
regarding a P250,000 qualified theft case on May 16, 1980 at
the Shemberg Marketing Corporation. 49 Although a suspect,
Mateo was not charged in the information subsequently filed
in that case. Sometime in March 1981, Mateo visited
petitioner at the police headquarters seeking assistance in his
bid to lead a new life. Considering Mateo's familiarity with
underworld characters, petitioner readily made him an
informer who was paid from time to time out of the police
intelligence fund. Mateo proved to be an effective informer.
In fact, he allegedly supplied vital information on the
identities and whereabouts of suspects in robbery cases at the
La Elegancia Jewelry Store, at the Likha Antique and Crafts,
50
and in an alleged racket in Aranque Market in Manila
involving jewelries.
As such informer, Mateo became accustomed to borrowing
petitioner's owner-type jeep whenever he was given an
assignment. In one instance however, petitioner saw Mateo
using his jeep with some male companions. Because Mateo
denied the occurrence of the incident, petitioner from then
on refused to lend his jeep to Mateo. Instead, Mateo was
given an allowance to cover his traveling expenses.
About a month prior to May 3, 1982, petitioner met Mateo
and requested the latter to give him a good project as he was
working for his transfer to the Metrocom Intelligence
Security Group (MISG). On May 2, 1982, Mateo urged
petitioner to lend him his jeep in order that he could followup a bank robbery case. That same evening, petitioner

approached his kumpare, accused Rodolfo Miranda, to


borrow the latter's old Mercedes Benz since, if the jeep was
used, Mateo could be identified as an informer. Petitioner
left his jeep with Miranda and "went around boasting of the
Mercedes Benz." 51

board two cars. When petitioner noticed that they were not
heading for Miranda's place, he clutched the hand of Lt.
Pagdilao, pleading for pity and thinking that he was about to
be "salvaged". Lt. Pagdilao however informed him that they
would be dropping by petitioner's house first per the
investigator's information that more checks could be
Mateo took the Benz in the morning of May 3, 1982.
recovered thereat. A warrantless search was then allegedly
Petitioner advised him to return the car between the hours of conducted in petitioner's house but nothing was found.
two and three in the afternoon at the Lakan Beer House at
Suddenly, someone from the other car came out of a nearby
the corner of Rizal Avenue and Zurbaran Streets in Sta. Cruz, house owned by Mateo and reported that they had recovered
Manila where petitioner was to meet his friend Manolo
some checks. Thereafter, they proceeded to the house of
Almoguera who would be celebrating his birthday there.
Miranda who was also invited for questioning. The latter
Petitioner met Almoguera and company at around 3:30 in the surrendered his Benz to the group.
afternoon. He waited for Mateo until shortly before 5:00 in
the afternoon when he was constrained to leave without
At the SOG headquarters in Camp Crame, petitioner was
seeing Mateo because he had to attend a mandatory regular
repeatedly coaxed to admit participation in the hijacking. As
troop formation at 5:00 P.M. at the police headquarters.
he vehemently denied the accusation against him, someone
From there, petitioner proceeded to his area of responsibility blindfolded him from behind, led him outside and loaded
in the tourist belt. He returned to the beer house at about
him in a car. He was taken to an unidentified place and made
6:00 in the evening hoping to find Mateo and the
to lie flat on his back. An object was tied to his small finger
automobile. A little before 8:00 o'clock, someone informed
to electrocute him. While a wet handkerchief was stuffed in
him that Mateo had finally arrived. Petitioner went out and
his mouth, someone mounted his chest and applied the
scolded Mateo for being late; the latter apologized and said
"water cure" ("tinutubig") through his nose. Because these
that his surveillance bore good results. Petitioner then
ordeals were simultaneously carried out, petitioner felt
returned the car to Miranda, through the latter's cousin.
unbearable pain. He sought permission to get in touch with
his father-in-law, Atty. Felix Rosacia, but his request was
At around 11:00 in the evening of May 29, 1982, Mateo,
denied. They urged him to cooperate otherwise something
escorted by a group of military men, went to petitioner's
terrible would happen to him.
house at 810 Cabezas St., Tondo, Manila. The group refused
to give any reason for their visit but arrested him. Wearing
Meanwhile, petitioner's wife reported to the WPD General
only short pants, petitioner was made to board a car where
Assignment Section her husband's forcible abduction by
he was handcuffed. The men asked him about the Benz and armed men whom she mistook for CIS agents. A check with
the identities of his companions in an alleged hijacking
the CIS yielded negative results. Thereafter, Lt. Reynaldo
incident. Petitioner admitted having knowledge of the exact Dator went to the SOG where he was informed that
location of the car but denied participation in the crime.
petitioner was being investigated but no details were given
Nobody apprised him of his constitutional rights to remain
thereon pending clearance with superior officers. 53
52
silent and to be assisted by counsel.
Consequently, a newspaper carried an item on the SOG's
refusal to allow petitioner's co-police officers to see him in
Petitioner was then instructed to accompany Lt. Pagdilao to his detention cell. 54
the residence of Miranda to get the Benz. They were on

Among his comrades, only Jimmy Victorino, formerly of the


WPD who was transferred to the SOG, was able to visit him.
Petitioner revealed to Victorino the maltreatment done him
but the latter expressed helplessness about it. In fact,
Victorino advised him to just cooperate so that the SOG
would not incriminate him (" para hindi ka pag-initan dito").
55
The advice came after petitioner was warned that he, like
Pat. Serrano of the WPD, would be liquidated by the SOG, 56
should he refuse to cooperate. Later, Mateo came to
petitioner's cell and confided that he had been similarly
maltreated and forced to implicate petitioner.
After Mateo left, a prepared statement was shown and read
to petitioner. Because its contents were false, petitioner
refused to sign it. Placing his arm around petitioner, a certain
Capt. Lagman told petitioner that he thought they had an
understanding already. Petitioner later discovered that
Lagman was not member of the military but an "agent" of
the SOG, and a member of the "Contreras gang". Petitioner
was therefore constrained to sign the statement because of
his excruciating experience ("hirap na hirap"). He however
admitted having read the document before affixing his
signature thereto and initialing the corrections therein. The
waiver under Article 125 of the Revised Penal Code and the
certification he executed were allegedly also obtained by
duress. Although he picked out one Severino Castro in a
police line-up, he did not even know Castro. He implicated
Castro because he was threatened by a certain Boy Zapanta.
Petitioner filed a complaint for grave coercion and
maltreatment against Lt. Rosendo Ferrer and several John
Does. On August 4, 1982, Asst. City Fiscal Emelita H.
Garayblas recommended its dismissal for petitioner's failure
to appear despite subpoenas and to answer clarificatory
questions as well as to authenticate his statement. 57
However, petitioner swore that he never received the
subpoenas.
Petitioner's alibi was supported by Manolo Almoguera
whose birthday on May 3, 1995 was the reason for the

celebration at the Lakan Beer House. While his baptismal


certificate indicated that he was born on May 4, 1956, 58 a
joint affidavit 59 also attested that his birth date was actually
May 3, 1956. Gary Gallardo, the owner of the beer house,
corroborated Almoguera's testimony as to petitioner's alleged
presence during the birthday celebration.

mail van which was hijacked had been


recovered, as well as most of the checks and
warrants which were surrendered by some of
the accused, without prejudice to the
institution of the proper civil action to
recover damages should proof thereof be
available.

The Respondent Court's Decision


On June 18, 1987, the Sandiganbayan rendered the herein
questioned 51-page Decision, the dispositive portion of
which reads:

Consequently, it is hereby ordered that


Exhibits B, B-l and B-2, which are the .32
Cal. Revolver, Smith and Wesson, Serial No.
11707, its holster and six (6) live
ammunition respectively, which were
surrendered by accused Relator, and
Exhibits J, J-l to J-5, consisting of 187, 222,
215, 197, 194 and 22 pieces, respectively, of
Social Security System and Medicare checks
and vouchers, be returned to the Firearm and
Explosives Unit (FEU), PC, Camp Crame,
Quezon City and the Social Security
System, respectively, upon proper receipts.

WHEREFORE, judgment is hereby


rendered finding accused Jose Filoteo, Jr. y
Diendo, Martin Mateo, Jr. y Mijares,
Bernardo Relator, Jr. y Retino and Eddie
Saguindel y Pabinguit GUILTY as coprincipals beyond reasonable doubt of the
violation of Section 2 (e), in relation to
Section 3 (b) of Presidential Decree No.
532, otherwise known as the Anti-Piracy and
Anti-Highway Robbery Law of 1974 and
Let copies of this decision be furnished the
hereby sentences each of said accused to
Postmaster-General, Central Post Office,
suffer the indeterminate penalty ranging
Liwasang Bonifacio, Metro Manila and the
from TWELVE (12) YEARS and ONE (1)
Commanding General and Chief, PC-INP,
DAY as minimum, to THIRTEEN (13)
Camp Crame, Quezon City for their
YEARS, ONE (1) MONTH and ELEVEN
information and guidance with respect to the
(11) DAYS as maximum, both of reclusion
other accused who are still at-large.
temporal, and to pay their proportionate
share of the costs of the action. Accused
SO ORDERED.
Danilo Miravalles y Marcelo is hereby
Petitioner's motion for reconsideration of said Decision was
acquitted, with costs de oficio, for
denied by the Sandiganbayan in its challenged Resolution of
insufficiency of evidence.
July 27, 1987. Hence, the instant alternative petition for
No civil indemnity is hereby awarded due to certiorari and/or review on certiorari charging the
the complete dearth of any proof as to the
Sandiganbayan with having gravely abused its discretion
actual damages suffered by the Bureau of
amounting to lack or excess of jurisdiction and with
Posts or the owners of the pilfered mail
reversible error in arriving at said Decision.
matters, and it further appearing that the

The Issues
The amended petition raises the following:
Assignments of Error
and/or
Excess of Jurisdiction/Grave Abuse of
Discretion

petitioner and three of the other accused


and in finding the testimonies and
investigation reports relative thereto.
"credible and unrefuted", said findings
being, insofar as petitioner is concerned,
absolutely without any basis in the evidence
and in fact contrary to the prosecution's
only evidence that has some measure of
competency and admissibility.

xxx xxx xxx


Fourth
First
The respondent court erred and gravely
abused its discretion as well as exceeded its
jurisdiction when it made its determination
of the alleged guilt of petitioner on the basis
of mere preponderance of evidence and not
proof beyond reasonable doubt.
Second
The respondent court erred and gravely
abused its discretion as well as exceeded its
jurisdiction in finding that petitioner's
having borrowed the Mercedes Benz car
utilized by the other accused in the
hijacking of the mail van indubitably
established his direct participation and/or
indispensable cooperation in the said
hijacking, the same being in gross disregard
of basic Rules of Law.
Third
The respondent court erred and gravely
abused its discretion as well as exceeded its
jurisdiction in finding that the voluminous
SSS Medicare and Pension Checks were
confiscated from and surrendered by

The respondent court erred and gravely


abused its discretion in finding that dorsal
portions of the checks and warrants
allegedly taken from petitioner were signed
by him to indicate his admission of
accountability therefor and that his
signatures thereon confirm the confiscation
from and/or surrender by him of said checks,
said findings being absolutely without any
support in the evidence.
Fifth
The respondent court erred and gravely
abused its discretion as well as exceeded its
jurisdiction in admitting and considering
against petitioner his alleged extra judical
confession, despite petitioner's
uncontradicted testimony and documentary
proof that he was made to give or sign the
same through torture, maltreatment,
physical compulsion, threats and
intimidation and without the presence and
assistance of counsel, his request for which
was refused, in gross violation of
Constitutional Provisions and the prevailing
jurisprudence.

Sixth
The respondent court erred and gravely
abused its discretion as well as exceeded its
jurisdiction in finding that petitioner's
participation in the hijacking of the mail
van is indubitably established "by the
manner by which the SOG operatives
succeeded in ferreting out the members of
the hijacking syndicate one by one through
patient sleuthing" and in finding that they
did so "without resorting to extra-legal
measures" and that "no evidence having
been adduced to show that they were
actuated by improper motives to testify
falsely against the herein accused, then their
testimonies should be accorded full
credence".
Seventh
The respondent court erred and gravely
abused its discretion as well as exceeded its
jurisdiction in finding that "even setting
aside the inter-locking confessional
statements of Filoteo, Mateo and
Liwanag, . . substantial and sufficient
evidence exist which indubitably prove the
guilt of Filoteo" (Petitioner).
Eighth
Insofar as petitioner is concerned, the
respondent court erred and gravely abused
its discretion as well as exceeded its
jurisdiction in finding that "accused
Filoteo's ( petitioner's) and Mateo's
[alleged] unexplained possession of the
stolen checks raised the presumption that
"they were responsible for the robbery in

question", petitioner's alleged possession


not being borne out but disputed by the
prosecution's own evidence.
Ninth
The respondent court erred and gravely
abused its discretion as well as exceeded its
jurisdiction in finding that "accused
Filoteo's denials and alibi cannot be
entertained for being quite weak and
implausible". The truth of the matter being
that they should have been sustained since
petitioner was not identified by direct
victims-eyewitnesses as among those who
participated in or were present at the hijack
and none of the checks and treasury
warrants were found in his possession or
retrieved from him.

the use of pejorative words, and without


stating the legal basis of such rejection, the
various vital factual points raised by
petitioner, in gross violation of the express
mandate of the 1987 Constitution.
The Court believes that the above "errors" may be condensed
into four:
(1) Are the written statements, particularly the extra-judicial
confession executed by the accused without the presence of
his lawyer, admissible in evidence against him?
(2) Were said statements obtained through torture, duress,
maltreatment and intimidation and therefore illegal and
inadmissible?
(3) Was petitioner's warrantless arrest valid and proper?

in accordance with Rule 45 of the Rules of


Court. And Rule 45 of the Revised Rules of
Court provides, in Section 2, that only
questions of law may be raised in the
Petition for Review and these must be
distinctly set forth. Thus, in principle,
findings of fact of the Sandiganbayan are
not to be reviewed by this Court in a petition
for review on certiorari. There are, of
course, certain exceptions to this general
principle. Here, reading petitioner's Petition
for Review and Memorandum in the most
favorable possible light, petitioner may be
seen to be in effect asserting that the
Sandiganbayan misapprehended certain
(f)acts in arriving at its factual conclusions.

As amended by Republic Act No. 7975, Section 7 of P.D.


No. 1606 expressly provides that "(d)ecisions and final
(4) Is the evidence of the prosecution sufficient to find the
orders of the Sandiganbayan shall be appealable to the
petitioner guilty beyond reasonable doubt?
Tenth
Supreme Court by petition for review on certiorari raising
pure questions of law in accordance with Rule 45 of the
The Court's Ruling
The respondent court erred and gravely
Rules of Court." However, in exceptional cases, this Court
abused its discretion as well as exceeded its
has taken cognizance of questions of fact in order to resolve
Preliminary
Issue:
Rule
4
or
Rule
65?
jurisdiction in finding that the participation
legal issues, as where there was palpable error or grave
of petitioner in the criminal conspiracy has
misapprehension of facts by the lower court. Criminal cases
been proven beyond reasonable doubt by the Before ruling on the foregoing issues, it is necessary to dwell elevated by convicted public officials from the
on the procedural aspects of the case. Petitioner, a
evidence of record and that said evidence
Sandiganbayan deserve the same thorough treatment by this
"segurista", opted to file an (amended) "alternative petition" Court as criminal cases involving ordinary citizens simply
"not only confirms the conspiracy between
for certiorari under Rule 65 and for review on certiorari
[him and the other accused] as easily
because the constitutional presumption of innocence must be
under Rule 45 of the Rules of Court. We however hold that
discernible from their conduct before,
overcome by proof beyond reasonable doubt. In all criminal
the instant petition must be considered as one for review on cases, a person's life and liberty are at stake. 61
during and after the commission of the
certiorari under Rule 45. In Jariol, Jr. vs. Sandiganbayan, 60
offense, but also their participation and/or
this Court clearly ruled:
indispensable cooperation".
Eleventh
The respondent Court erred and gravely
abused its discretion as well as exceeded its
jurisdiction in cavalierly rejecting, through

Presidential Decree No. 1486, as amended


by P.D. No. 1606, which created the
Sandiganbayan, specified that decisions and
final orders of the Sandiganbayan shall be
subject to review on certiorari by this Court

As a petition for review under Rule 45 is the available


remedy, a petition for certiorari under Rule 65 would not
prosper. Basic it is that certiorari is invocable only where
there is no other plain, speedy or adequate remedy. For
waffling on procedural matters, petitioner could have lost
this battle through a summary dismissal of his "alternative"
petition. But in view of the importance of the issues raised,
the Court decided to take cognizance of the matter.
First Issue: Uncounselled Waiver
On the merits of the petition, we find that the pivotal issue
here is the admissibility of petitioner's extrajudicial
confession which lays out in detail his complicity in the
crime.
Petitioner contends that respondent Court erred in admitting
his extrajudicial confession notwithstanding uncontradicted
testimony and documentary proof that he was made to sign
the same through torture, maltreatment, physical
compulsion, threats and intimidation and without the
presence and assistance of counsel. He also claims that in
executing the extrajudicial confession, he was denied the
right to counsel in the sameway that his waiver of the said
right was likewise without the benefit of counsel. Petitioner
therefore questions the respondent Court's admission
evidence of his extrajudicial confession on the strength of
cases 62 upholding the admissibility of extrajudicial
confessions notwithstanding the absence of counsel
"especially where the statements are replete with details and
circumstances which are indicative of voluntariness." We
shall first tackle the issue of his uncounselled waiver of his
right to counsel.
The pertinent provision of Article IV, Section 20 of the 1973
Constitution reads as follows:
No person shall be compelled to be a
witness against himself. Any person under
investigation for the commission of an

offense shall have the right to remain silent


Obviously, the 1973 Constitution did not
and to counsel and to be informed of such
contain the right against an uncounselled
rights. No force, violence, threat,
waiver of the right to counsel which is
intimidation, or any other means which
provided under paragraph 1, Section 12,
vitiate the free will shall be used against
Article III of the 1987 Constitution, above
him. Any confession obtained in violation of
underscored.)
this section shall be inadmissible in
In the landmark case of Magtoto vs. Manguera, 63 the Court
evidence.
categorically held that the aforequoted provisions of the
In comparison, the relevant rights of an accused under
1973 Constitution (which were not included in the 1935
Article III, Section 12 of the 1987 Constitution are, inter
Charter) must be prospectively applied. This Court said:
alia, as follows:
We hold that this specific portion of this
(1) Any person under investigation for the
constitutional mandate has and should be
commission of an offense shall have the
given a prospective and not a retrospective
right to be informed of his right to remain
effect. Consequently, a confession obtained
silent and to have competent and
from a person under investigation for the
independent counsel preferably of his own
commission of an offense, who has not been
choice. If the person cannot afford the
informed of his right (to silence and) to
services of counsel, he must be provided
counsel, is inadmissible in evidence if the
with one. These rights cannot be waived
same had been obtained after the effectivity
except in writing and in the presence of
of the New Constitution on January 17,
counsel.
1973. Conversely, such confession is
admissible in evidence against the accused,
(2) No torture, force, violence, threat,
if the same had been obtained before the
intimidation; or any other means which
effectivity of the New Constitution, even if
vitiate the free will shall be used against
presented after January 17, 1973, and even if
him. Secret detention places, solitary,
he had not been informed of his right to
incommunicado, or other similar forms of
counsel, since no law gave the accused the
detention are prohibited.
right to be so informed before that date.
(3) Any confession or admission obtained in By parity of reasoning, the specific provision of the 1987
violation of this or Section 17 hereof shall
Constitution requiring that a waiver by an accused of his
be inadmissible in evidence against him.
right to counsel during custodial investigation must be made
with the assistance of counsel may not be applied
(4) The law shall provide for penal and civil retroactively or in cases where the extrajudicial confession
sanctions for violations of this section as
was made prior to the effectivity of said Constitution.
well as compensation to and rehabilitation of Accordingly, waivers of the right to counsel during custodial
victims of torture or similar practices and
investigation without the benefit of counsel during the
their families." (emphasis supplied.
effectivity of the 1973 Constitution should, by such

argumentation, be admissible. Although a number of cases


held that extrajudicial confessions made while the 1973
Constitution was in force and effect, should have been made
with the assistance of counsel, 64 the definitive ruling was
enunciated only on April 26, 1983 when this Court, through
Morales, Jr. vs. Enrile, 65 issued the guidelines to be
observed by law enforcers during custodial investigation.
The Court specifically ruled that "(t)he right to counsel may
be waived but the waiver shall not be valid unless made with
the assistance of counsel. 66 Thereafter, in People vs.
Luvendino, 67 the Court through Mr. Justice Florentino P.
Feliciano vigorously taught:

Philippines," and Article 4 of the same Code which states


that "(l)aws shall have no retroactive effect unless the
contrary is provided," the principle of prospectivity of
statutes, original or amendatory, shall apply to judicial
decisions, which, although in themselves are not laws, are
nevertheless evidence of what the law means. 69

Petitioner's contention that Article III, Section 12 of the 1987


Constitution should be given retroactive effect for being
favorable to him as an accused, cannot be sustained. While
Article 22 of the Revised Penal Code provides that "(p)enal
laws shall have a retroactive effect insofar as they favor the
person guilty of a felony who is not a habitual criminal,"
. . . The doctrine that an uncounseled waiver what is being construed here is a constitutional provision
of the right to counsel is not to be given
specifically contained in the Bill of Rights which is
legal effect was initially a judge-made one
obviously not a penal statute. A bill of rights is a declaration
and was first announced on 26 April 1983 in and enumeration of the individual rights and privileges
Morales vs. Enrile and reiterated on 20
which the Constitution is designed to protect against
March 1985 in People vs. Galit. . . .
violations by the government, or by individuals or groups of
individuals. It is a charter of liberties for the individual and a
While the Morales-Galit doctrine eventually limitation upon the power of the state. 70 Penal laws, on the
became part of Section 12(1) of the 1987
other hand, strictly and properly are those imposing
Constitution, that doctrine affords no
punishment for an offense committed against the state which
comfort to appellant Luvendino for the
the executive of the state has the power to pardon. In other
requirements and restrictions outlined in
words, a penal law denotes punishment imposed and
Morales and Galit have no retroactive effect enforced by the state for a crime or offense against its law. 71
and do not reach waivers made prior to 26
April 1983 the date of promulgation of
Hence, petitioner's vigorous reliance on People vs. Sison 72 to
Morales.
make his extrajudicial confession inadmissible is misplaced.
In that case, the extrajudicial confession was executed on
Pursuant to the above doctrine, petitioner may not claim the May 19, 1983, clearly after the promulgation of Morales on
benefits of the Morales and Galit rulings because he
April 26, 1983.
executed his extrajudicial confession and his waiver to the
right to counsel on May 30, 1982, or before April 26, 1983. The admissibility of petitioner's uncounselled waiver of the
The prospective application of "judge-made" laws was
right to counsel notwithstanding, the Court has still to
68
underscored in Co vs. Court of Appeals where the Court
determine whether such waiver was made voluntarily and
ruled thru Chief Justice Andres R. Narvasa that in
intelligently. 73 The waiver must also be categorical and
accordance with Article 8 of the Civil Code which provides definitive, 74 and must rest on clear evidence. 75
that "(j)udicial decisions applying or interpreting the laws or
the Constitution shall form part of the legal system of the

In his affidavit of May 30, 1982 waiving the provisions of


Article 125 of the Revised Penal Code, 76 petitioner stated
that:
. . . matapos akong mapagpaliwanagan ng
mga imbestigador ng Special Operations
Group, PC/INP Central Anti-Organized
Crime Task Force, Camp Crame, Quezon
City ng aking mga karapatan alinsunod sa
mga isinasaad ng Section 20, Article IV ng
Bagong Saligang Batas ng Republika ng
Pilipinas ay malaya at kusang-loob na
nagsasalaysay ng mga sumusunod kahit na
walang abugadong magpapayo sa akin sa
pagsasagawa nito sa dahilang alam at
nauunawaan ko ang aking ginagawa at wala
naman akong isasalaysay kung hindi mga
katotohanan lamang, bagama't ako ay inalok
ng mga imbestigador na ikuha ng isang
abugadong walang bayad mula sa CLAOIBP na akin namang tinanggihan:
xxx xxx xxx
Na ako ay hindi sinaktan a minaltrato
gayunding walang kinuha mula sa akin na
hindi niresibohan;
xxx xxx xxx
Sgt. Arsenio Carlos, investigating officer, testified that he
apprised petitioner of his right to counsel even in waiving
the same right 77 but petitioner did not even inform him that
his father-in-law was a lawyer. Although allowed to talk for
thirty minutes with Jimmy Victorino, who was his comrade
at the WPD General Assignment Section, 78 still, petitioner
did not invoke his right to counsel.
It should be emphasized that petitioner could not have been
ignorant of his rights as an accused. He was a fourth year

criminology student and a topnotch student in the police


basic course. 79 Having been in the police force since 1978,
with stints at the investigation division or the detective
bureau, he knew the tactics used by investigators to
incriminate criminal suspects. 80 In other words, he was
knowledgeable on the matterof extrajudicial confessions.
The Second Issue: Confession Extracted Through
Torture?

Besides, the question of whether petitioner was indeed


subjected to torture or maltreatment is a factual question
addressed primarily to trial courts, the findings of which are
binding on this Court whose function, as afore-discussed, is
principally to review only of questions of law. Moreover, we
have pored over the assailed Decision and we are satisfied
that respondent Court performed its duty in evaluating the
evidence. More on this later.
The Third Issue: Illegal Arrest?

Petitioner's claim that he was tortured into signing the


confession appears incredible, or at least susceptible to
serious doubts. The allegation of torture was negated by the
medical report 81 showing no evidence of physical injuries
upon his person. As correctly observed by the Solicitor
General, there is no reason to maltreat him in particular
when the record shows that the investigating team respected
the right of the other suspects to remain silent. When he was
presented before Judge Mariano Mendieta of the municipal
court in Meycauayan, petitioner even waived his right to
present evidence 82 instead of impugning his confession on
account of the torture allegedly inflicted upon him. If indeed
he had been tortured, he would have revived the case he filed
against his alleged torturers upon learning of its dismissal.
Furthermore, an examination of his signatures in the
different documents on record bearing the same discloses an
evenness of lines and strokes in his penmanship which is
markedly consistent in his certification, extrajudicial
confession and waiver of detention. Human experience has
proven that the lines and strokes of a person's handwriting
reflect his disposition at a certain given time. In the present
case, no handwriting expert is needed to declare that
petitioner's signatures were written voluntarily and not under
compulsion of fear immediately after he had been subjected
to maltreatment. In view of the foregoing, his extrajudicial
confession is presumed to have been voluntarily made, in the
absence of conclusive evidence showing that petitioner's
consent in executing the same had been vitiated. 83

Petitioner questions the manner of his arrest, stating that the


arresting officers "invited" him without a warrant of arrest
and brought him to Camp Crame where he was allegedly
subjected to torture almost a month after the commission of
the crime. 84 Petitioner's claim is belatedly made. He should
have questioned the validity of his arrest before he entered
his plea in the trial court. On this point, this Court explained
in People vs. Lopez, Jr.: 85
Finally, it is much too late for appellant to
raise the question of his arrest without a
warrant. When accused-appellant was
arrested and a case was filed against him, he
pleaded not guilty upon arraignment,
participated in the trial and presented his
evidence. Appellant is thus estopped from
questioning the legality of his arrest. It is
well-settled that any objection involving a
warrant of arrest or procedure in the
acquisition by the court of jurisdiction over
the person of an accused must be made
before he enters his plea, otherwise the
objection is deemed waived. Besides, this
issue is being raised for the first time by
appellant. He did not move for the quashal
of the information before the trial court on
this ground. Consequently, any irregularity
attendant to his arrest, if any, was cured
when he voluntarily submitted himself to the

jurisdiction of the trial court by entering a


plea of not guilty and by participating in the
trial. Moreover, the illegal arrest of an
accused is not sufficient cause for setting
aside a valid judgment rendered upon a
sufficient complaint after trial free from
error.
The only move petitioner made in regard to his arrest was to
file a complaint for "grave coercion, grave threat &
maltreatment" which was docketed as I.S. No. 82-12684
before the Fiscal's Office of Quezon City. 86 The complaint
was an offshoot of his alleged maltreatment in the hands of
the SOG upon his arrest. However, as stated above, he did
not lift a finger to revive it upon its dismissal.
The Fourth Issue: Sufficiency of the Prosecution's
Evidence
Contrary to petitioner's claim, his culpability has been
proven beyond reasonable doubt. He borrowed a car to use
in the hijacking knowing fully well that his owner-type jeep
would give away his identity. Hecould not be identified by
the postal employees in the postal van simply because after
overtaking said vehicle and forcing its driver to pull over, he
gave up driving the Mercedes Benz where the postal
employees were made to ride, and commandeered the van.
That the checks were not found in his own home is of no
moment. Before the arrest and upon learning that the
authorities had begun to nail down the identities of the
malefactors, hehad entrusted them to his "kumare". It was
petitioner himself who led the team of Lt. Pagdilao back to
his place after he had admitted to Sgt. Arsenio Carlos that his
share of the checks were in the possession of his "kumare" in
the neighborhood. 87
In view of these facts, it is beyond dispute that petitioner was
a direct participant in the commission of the crime. His alibi
has been correctly considered by the Sandiganbayan to be
weak and implausible. The distance between Kalvario,

Meycauayan, Bulacan and downtown Manila where


in a better position to weigh conflicting
petitioner claimed to have been at the crucial time was
testimonies. They heard the witnesses
between fifteen (15) to twenty (20) kilometers, which,
themselves and observed their deportment
through first-class roads, could be negotiated during that
and manner of testifying. . . . 90
time in approximately thirty (30) minutes. It could not
So overwhelming is the prosecution's evidence that
therefore have been physically impossible for him to be at
the crime scene or its immediate vicinity when the crime was respondent Court opined that even without the "inter-locking
confessions of Filoteo, Mateo and Liwanag" the remaining
committed. 88
evidence would still be sufficient for conviction. 91 Said the
Having already ruled on the admissibility of petitioner's
respondent tribunal:
confession, this Court holds that the full force of the totality
However, even setting aside the interof the prosecution's evidence proves his guilt well beyond
locking confessional statements of Filoteo,
reasonable doubt. Weighing heavily against the defense is
Mateo and Liwanag, we are of the
the well-settled doctrine that findings of facts of the trial
considered opinion that substantial and
courts in this case, the Sandiganbayan itself
sufficient evidence exist which indubitably
particularly in the assessment of the credibility of witnesses,
prove the guilt of Filoteo, Relator, Mateo
is binding upon this Court, absent any arbitrariness, abuse or
and Saguindel who had submitted
palpable error.
themselves to the jurisdiction of this Court.
. . . It is well-settled that this Court will not
As above-stated, Filoteo was responsible for
interfere with the judgment of the trial court
securing the use of the Mercedes Benz car
in passing on the credibility of the witnesses,
used by the co-conspirators in the hiunless there appears in the record some fact
jacking. Together with Mateo, Liwanag and
or circumstance of weight and influence
Mendoza, he surrendered voluminous
which has been overlooked or the
assorted checks which were part of the loot.
significance of which has been
Relator admitted that his service firearm was
misapprehended or misinterpreted. The
used by him in the hi-jacking, which firearm
reason for this is that the trial court is in a
was identified by prosecution witnesses
better position to decide the question, having
Miranda and Bautista. Saguindel was
heard the witnesses themselves and
identified in line-ups at the SOG office as
observed their deportment and manner of
the suspect clad in fatigue uniform and
testifying during the trial. 89
carrying an Armalite rifle by prosecution
witnesses Tagudar and Bautista. All three (3)
The doctrine is firmly settled that the trial
accused, namely, Mateo, Relator and
court's conclusion on issues of credibility is
Saguindel also jumped bail during the trial
accorded with highest respect by the
and did not offer any evidence to refute the
appellate courts (People v. Dominguez, 217
evidence presented by the prosecution
SCRA 170). Appellate courts will generally
against them. Such flight to evade
respect the findings of trial courts on the
prosecution constitutes an implied admission
credibility of witnesses since trial courts are
of guilt.

Moreover, accused Filoteo's and Mateo's


unexplained possession of the stolen checks
raises the presumption that they were
responsible for the robbery in question. It is
a rule established by an abundance of
jurisprudence that when stolen property is
found in the possession of one, not the
owner, without a satisfactory explanation of
his possession, he will be presumed the
thief. This rule is in accordance with the
disputable presumption "that a person found
in possession of a thing taken in the doing of
a recent wrongful act is the taker and doer of
the whole act." In the instant case, said
accused has not given such satisfactory
explanation, much more so when their
possession had been positively established
by the testimonies of prosecution witnesses
Capt. Ferrer and Sgt. Carlos and by
accused's own signatures at the back of said
checks.
Furthermore, accused Filoteo's denials and
alibi cannot be entertained for being quite
weak and implausible. His claim that he
merely borrowed the Mercedes Bent car
from Rodolfo Miranda to help out his coaccused Mateo, who had been utilized by the
police as an "informer" and was following
up tips in certain unsolved cases, appears to
be incredible and fantastic. He also claimed
that he could not have participated in the hijack because after giving the car to Mateo in
the morning of May 2, 1982, he waited at
the corner of Zurbaran St. and Avenida Rizal
between 2-3:00 o'clock p.m. of the same day
and then went to the WPD headquarters to
attend the police formation at around 5:00
o'clock p.m. when Mateo failed to show up.
Thereafter, he tried to show through his

witnesses Gary Gallardo and Manolo


Almogera that he was with them between
3:00 o'clock to 4:45 o'clock p.m., then from
6:00 o'clock to 8:30 o'clock p.m. and,
finally, from 10:45 o'clock p.m. to 11:00
o'clock of the same date. It was through said
witnesses that he tried to establish his
whereabouts between 4:30 o'clock to 7:30
o'clock p.m. of May 2, 1982, the period
from the time the mail van was hi-jacked up
to when postal employees Bautista, Miranda
and Tagudar were brought to Caloocan City
and freed by their captors. Such alibi,
however, fails to show that it was physically
impossible for him to be present at the scene
of the hi-jacking. We take judicial notice
that the distance between the crime scene
and down-town Manila is some 15-20
kilometers and negotiable over first- class
roads in some thirty (30) minutes.
We are likewise convinced that there is sufficient evidence of
conspiracy as convincing as the evidence of the participation
of each of the accused. As ratiocinated in the assailed
Decision: 92
The participation of accused Filoteo, Mateo,
Relator and Saguindel in the criminal
conspiracy have (sic) been proved beyond
reasonable doubt by the evidence on record
and which evidence not only confirms the
existence of the conspiracy between them as
easily discernible from their conduct before,
during and after the commission of the
offense, but also their participation therein
as co-principals by direct participation
and/or indispensable cooperation. Their
concerted efforts were performed with
closeness and coordination indicating their
common purpose. Hence, there being

collective criminal responsibility, the act of


one is the act of all, and each of the
participants are responsible for what the
others did in all the stages of execution of
the offense.
Final Question: Brigandage or Robbery?
The Court believes that, though not raised as an issue and
though not argued by the parties in their pleadings, the
question of which law was violated by the accused should be
discussed and passed upon. In fact, petitioner should have
brought up such question as it may benefit him with a
reduced penalty.
The respondent Court convicted the accused of brigandage
punishable under Presidential Decree No. 532. 93
Justifying the above disposition, the assailed Decision
ratiocinates:
Accused herein are charged with the
violation of Presidential Decree No. 532,
otherwise known as the Anti-Piracy and
Anti-Highway Robbery Law of 1974. Under
said decree, with respect to the highway
robbery aspect, the offense is committed on
a "Philippine Highway" which under
Section 2 (c) thereof has been defined as
"any road, street, passage, highway and
bridges or any part thereof, or railway or
railroad within the Philippines, used by
persons or vehicles, or locomotives or trains
for the movement or circulation of persons
or transportation of goods, articles or
property or both", while under Section 2 (e)
thereof "Highway Robbery/ Brigandage" has
been defined as the "the seizure of any
person for ransom, extortion or other
unlawful purposes or the taking away of

property of another by means of violence


against or intimidation of persons nor force
upon things or other unlawful means,
committed by any person on any Philippine
Highway". (Emphasis supplied)
The offense described in the information and
established by the evidence presented by the
prosecution properly falls within the ambit
of the aforesaid special law. Therein, it was
conclusively proven that a postal van
containing mail matters, including checks
and warrants, was hi-jacked along the
national highway in Bulacan by the accused,
with the attendant use of force, violence and
intimidation against the three (3) postal
employees who were occupants thereof,
resulting in the unlawful taking and
asportation of the entire van and its contents
consisting of mail matters. Also the evidence
further showed that the crime was
committed by the accused who were PC
soldiers, policeman (sic) and private
individuals in conspiracy with their coaccused Castro and Escalada who were
postal employees and who participated in
the planning of the crime. Accordingly, all
the essential requisites to constitute a
consummated offense under the law in point
are present. (Emphasis in the original text.)
Obviously, the Court a quo labored under the belief that
because the taking or robbery was perpetrated on a national
highway (McArthur Highway), ergo, Presidential Decree No.
532, otherwise known as the Anti-Piracy and Anti-Highway
Robbery Law of 1974, must have been the statute violated.
Such reasoning has already been debunked by this Court in
the case of People vs. Isabelo Puno, 94 where it was ruled in
unmistakable language that it takes more than the situs of the

robbery to bring it within the ambit of PD 532. Said the


Court through Mr. Justice Florenz D. Regalado:
The following salient distinctions between
brigandage and robbery are succinctly
explained in a treatise on the subject and are
of continuing validity:
The main object of the
Brigandage Law is to
prevent the formation of
bands of robbers. The heart
of the offense consists in the
formation of a band by
more than three armed
persons for the purpose
indicated in art. 306. Such
formation is sufficient to
constitute a violation of art.
306. It would not be
necessary to show, in a
prosecution under it, that a
member or members of the
band actually committed
robbery or kidnapping or
any other purpose attainable
by violent means. The crime
is proven when the
organization and purpose of
the band are shown to be
such as are contemplated by
art. 306. On the other hand,
if robbery is committed by a
band, whose members were
not primarily organized for
the purpose of committing
robbery or kidnapping, etc.,
the crime would not be
brigandage, but only
robbery. Simply because

robbery was committed by a


band of more than three
armed persons, it would not
follow that it was
committed by a band of
brigands. In the Spanish text
of art. 306, it is required that
the band "sala a los campos
para dedicarse a robar."
(Emphasis ours.)
In fine, the purpose of brigandage, is inter
alia, indiscriminate highway robbery. If the
purpose is only a particular robbery, the
crime is only robbery, or robbery in band if
there are at least four armed participants.
The martial law legislator, in creating and
promulgating Presidential Decree No. 532
for the objectives announced therein, could
not have been unaware of that distinction
and is presumed to have adopted the same,
there being no indication to the contrary.
This conclusion is buttressed by the rule on
contemporaneous construction, since it is
one drawn from the time when and the
circumstances under which the decree to be
construed originated. Contemporaneous
exposition or construction is the best and
strongest in the law.
Further, that Presidential Decree No. 532
punishes as highway robbery or brigandage
only acts of robbery perpetrated by outlaws
indiscriminately against any person or
persons on Philippine highways as defined
therein, and not acts of robbery committed
against only a predetermined or particular
victim, is evident from the preambular
clauses thereof, to wit:

WHEREAS, reports from


law-enforcement agencies
reveal that lawless elements
are still committing acts of
depredation upon the
persons and properties of
innocent and defenseless
inhabitants who travel from
one place to another,
thereby disturbing the
peace, order and tranquility
of the nation and stunting
the economic and social
progress of the people:
WHEREAS, such acts of
depredations constitute . . .
highway
robbery/brigandage which
are among the highest forms
of lawlessness condemned
by the penal statutes of all
countries:
WHEREAS, it is imperative
that said lawless elements
be discouraged from
perpetrating such acts of
depredations by imposing
heavy penalty on the
offenders, with the end in
view of eliminating all
obstacles to the economic,
social, educational and
community progress of the
people; (Emphasis
supplied.)
Indeed, it is hard to conceive of how a single
act of robbery against a particular person

chosen by the accused as their specific


victim could be considered as committed on
the "innocent and defenseless inhabitants
who travel from one place to another," and
which single act of depredation would be
capable of "stunting the economic and social
progress of the people" as to be considered
"among the highest forms of lawlessness
condemned by the penal statutes of all
countries, and would accordingly constitute
an obstacle "to the economic, social,
educational and community progress of the
people, such that said isolated act would
constitute the highway robbery or
brigandage contemplated and punished is
said decree. This would be an exaggeration
bordering on the ridiculous.
From the above, it is clear that a finding of brigandage or
highway robbery involves not just the locus of the crime or
the fact that more than three (3) persons perpetrated it. It is
essential to prove that the outlaws were purposely organized
not just for one act of robbery but for several indiscriminate
commissions thereof. In the present case, there had been no
evidence presented that the accused were a band of outlaws
organized for the purpose of "depredation upon the persons
and properties of innocent and defenseless inhabitants who
travel from one place to another." What was duly proven in
the present case is one isolated hijacking of a postal van.
There was also no evidence of any previous attempts at
similar robberies by the accused to show the "indiscriminate"
commission thereof. 95
Upon the other hand, the Information did not specifically
mention P.D. 532. 96 The facts alleged therein and proven by
the evidence constitute the offense of robbery defined in Art.
293 in relation to Art. 295 and punished by Art. 244, par. 5,
all of the Revised Penal Code. 97 From the facts, it was duly
proven that:

* personal property (treasury warrants, checks, mail, van,


tools, etc.)

defined in Arts. 293 and 295 and penalized


under Art. 294, paragraph 5, of the Revised
Penal Code Code IMPOSING on him an
indeterminate sentence of four (4) years and
two (2) months of prision correccional, as
minimum, to ten (10) years of prision mayor
as maximum, and to pay his proportionate
share of the costs of the action.

* belonging to another were


* unlawfully taken by the accused
* with intent to gain (animo lucrandi)
* with intimidation against three persons (Art. 293)

All other parts of the disposition are hereby AFFIRMED.

* in an uninhabited place, or

SO ORDERED.

* by an band, or
* by attacking a moving motor vehicle

G.R. No. 94494 March 15, 1996

* on a highway; and

PEOPLE OF THE PHILIPPINES, plaintiff-appellee,


vs.
DIONISIO LAPURA y CAJAN, accused-appellant.

* the intimidation was made with the use of firearms (Art.


295)

Hence, the offender shall be punished by the maximum


period of the penalty provided under paragraph 5 of Art. 294, VITUG, J.:p
which is, " prision correccional in its maximum period to
prision mayor in its medium period".
On appeal is the decision, 1 dated 04 June 1990, of the
Regional Trial Court of Manila, Branch 12, in Criminal Case
Effectively, the penalty imposed by the Court a quo should
No. 88-61209, convicting herein appellant Dionisio Lapura y
be lightened. However, such lighter penalty shall benefit
Cajan of murder and imposing on him the penalty of
only herein petitioner and not his co-accused who did not
reclusion perpetua. The trial court has reserved to the heirs
contest or appeal the Sandiganbayan's Decision.
of the victim, Petronilo Lim, the right to file a separate suit
for civil indemnification.
WHEREFORE, the petition is DENIED, but the first
paragraph of the dispositive portion of the assailed Decision The information, dated 01 March 1988, which opened the
is partially MODIFIED to read as follows:
criminal case against appellant read:
WHEREFORE, judgment is hereby
rendered finding accused Jose Filoteo, Jr. y
Diendo GUILTY beyond reasonable doubt
as co-principal in the crime of robbery as

That on or about February 19, 1988, in the


City of Manila, Philippines, the said
accused, conspiring and confederating
together with others whose true names,

identities and present whereabouts are still


unknown and helping one another, did then
and there wiffully, unlawfully and
feloniously, with intent to kill, evident
premeditation and treachery, attack, assault
and use personal violence upon one
Petronilo Lim, by then and there shooting
him several times with a .45 caliber pistol
hitting him on left anterior lumbar and left
thigh, thereby inflicting upon said Petronilo
Lim multiple gunshot wounds which were
the direct and immediate cause of his death
thereafter.
Contrary to law.

Following his arraignment on 13 April 1988, at which the


accused pleaded not guilty to the charge, a petition for bail
was heard. On 14 September 1988, the trial court rejected
the petition. A motion for the reconsideration of the denial
order, itself, was later denied.
The prosecution's evidence would tend to establish that at
approximately 7:30 in the morning of 19 February 1988,
Petronilo Lim, said to be a special agent of the Criminal
Investigation Service, 3 was on board his car, with his sister,
driving along Honorio Lopez Blvd., Balut, Tondo, Manila.
Just as he started slowing down the car before turning left to
Infanta Street, two persons suddenly came forward and fired
at him.
Edgardo Samson, a 27-year-old bicycle maker, then working
for the Teen's Bicycle Enterprises at Honorio Lopez Blvd.,
was only around ten (10) meters away from the shooting
incident. Instinctively turning his head to where the sound of
gunshots emanated, he saw two persons a hunchback
(kuba) who was positioned at the front right side of the car
and another person at the left side of the vehicle.

The victim, now all bloodied, got out of the car and fired
back using his "baby" armalite. Instantly, a person who was
wearing a white undershirt, "maong" pants and white shoes,
grabbed the armalite but one Ambet Zabala immediately
grappled for its possession. Ambet succeeded in recovering
the armalite which he turned over to "Amang" Manalo. The
man in "maong" pants fled towards nearby San Rafael
Village.
On 26 February 1988, Samson executed a sworn statement
before Senior investigation Agent Jesus Caizares and Sgt.
Feliciano Garcia at the Special Investigation Unit, Criminal
Investigation Service Command, Camp Crame, Quezon City.
Later that day, he identified appellant in a police line-up to
be the person who positioned himself at the left side of the
victim's car and who fired a .45 caliber pistol at the victim.
He executed another statement to this effect before
Caizares.

The defense interposed alibi. Appellant, then 32 years of age


and a combo drummer by profession, testified that at the
time the shooting took place he was sleeping at his sister's
house in 1039 Int. 17, P. Vargas St., Tondo, Manila. He woke
up rather late that day for he had performed the night before
at the North Mall in Caloocan City. His sister, Adelaida
Lapura Cuison, corroborating appellant's alibi. testified that
on 19 February 1988, when she left the house at about 6:30
a.m., her brother was still sleeping in their living room and,
coming back to the house around thirty minutes later, she
found him still asleep. He did not wake up until around ten
o'clock that morning.

Appellant was arrested by police officers in the afternoon of


25 February 1988 at the house of a fellow musician; Danilo
Cabrera, in Mata Street, Divisoria, Manila. Appellant and
Cabrera, along with another musician friend, Reynaldo
Eliezer, were brought to Station 1 at North Bay, Tondo,
Manila, where statements were taken. They were later
The victim, 55-year-old Petronilo Lim, died of three gunshot brought inside the office of Col. Maganto where appellant
wounds two (2) at the left anterior lumbar area which
was informed that his being a suspect in the killing of
lacerated the small intestine the liver, the kidney and the
Petronilo Lim was because he resembled the cartograph of
ascending colon, and one (1) "thru and thru" at the left thigh. the killer. Cabrera added that they were watching television
4
According to Dr. Marcial Ceido, who conducted the
when the arresting policemen suddenly entered their house
autopsy, the victim must have been fired at while his body
shouting, "You are NPA's, no one must move. 7
was inclined towards the right 5 and that, because there was
"tattooing" on his left posterior forearm, the muzzle of the
After evaluating the evidence presented before it, the trial
6
gun must have been pointed at close range.
court gave credence to the prosecution's case, particularly to
the eyewitness account of Samson, and accordingly rendered
judgment convicting appellant of murder.
In this appeal, it is initially argued that the trial court should
have outrightly dismissed the case against appellant
considering (a) that the information was filed without the
written authority of the city fiscal or prosecutor and (b) that
the certification of the investigating fiscal appended to the
information was defective for (i) not being under oath, (ii)
having failed to state that a preliminary investigation under
Section 3, Rule 112, of the Rules of Court was conducted,
and (iii) not having stated that the accused was duly

informed of the complaint and given the opportunity to


present controverting evidence.
The sufficiency of the allegations found in the complaint,
conformably with Section 6, Rule 110, of the Rules of Court,
8
has not been questioned; what, instead, is being assailed
centers on the supposed failure of the investigating
prosecutor to obtain the prior written authority of the city
prosecutor in the manner required under Section 4, 9 Rule
112, of the Rules of Court, before the filing of the case. This
assertion contradicts the certification of the investigating
fiscal attesting to the fact that the information has been duly
filed under the authority of the City Fiscal; viz:
I hereby certify that an ex-parte
investigation in this case has been conducted
by me in accordance with law; that there is
reasonable ground to believe that the offense
charged has been committed; that the
accused is probably guilty thereof and that
the filing of this information is with the
prior authority and approval of the City
Fiscal. 10 (Emphasis supplied.)
Absent convincing evidence to the contrary, the
presumption of regularity in the performance of
official functions has to be upheld. Moreover, this
matter should have been raised below in a proper
motion to quash 11 that appellant could have done but
did not.
Relative to the claim that the certification did not fully
comply with the requirements of Section 4, 12 Rule 112, of
the Rules of Court, we need merely to reiterate the settled
rule that such certification is not an indispensable part of, let
alone invalidate even by its absence, an information. 13 In
People vs.
Marquez, 14 the Court has had occasion to explain:

. . . . It should be observed that section 3


[now section 4] of Rule 110 defines an
information as nothing more than "an
accusation in writing charging a person with
an offense subscribed by the fiscal and filed
with the court." Thus, it is obvious that such
certification is not an essential part of the
information itself and its absence cannot
vitiate it as such. True, as already stated,
section 14 of Rule 112 enjoin that "no
information . . . shall be filed, without first
giving the accused a chance to be heard in a
preliminary investigation," but, as can be
seen, the injunction refers to the non-holding
of the preliminary investigation, not the
absence of the certification. In other words,
what is not allowed is the filing of the
information without a preliminary
investigation having been previously
conducted, and the injunction that there
should be a certification is only a
consequence of the requirement that a
preliminary investigation should first be
conducted.
As the Court has also said in Pecho vs.
Sandiganbayan 15
If the absence of a certification would not
even invalidate the information, then its
presence, although deficient because of
some missing clauses or phrases required
under Section 4, Rule 112 of the Rules of
Court, can do nothing worse than the former.
In passing, the question of whether or not a preliminary
investigation has been properly conducted is itself one that
should be interposed prior to an arraignment. 16 It does not
here appear that appellant did before entering his plea of "not
guilty" to the charge.

On the merits of the case, appellant faults the trial court for
believing the testimony of Samson despite supposed
contradictions and inconsistencies of the witness. A close
look at the records betrays any validity to the allegation.
First of all, in his; first sworn statement, Samson identified
the three perpetrators 17 of the crime; thus:
16. T Maari mo bang
masabi ang mga anyo ng
tatlong lalake na tinutukoy
mo?
S Yon nasa harap ng kotse
ay medyo kuba at
katamtaman ang katawan at
kulay ng balat, at iyon nasa
gawing kaliwa ng kotse ay
mahaba ang buhok, may
bigote, mataas at regular
ang lake ng kanyang
katawan at katamtaman ang
kulay ng balat at iyon
dumamput no Armalite ni
Mr. Lim at (sic) payat at
mababa at kayumanggi ang
kulay. 18
Then, in his second sworn statement, Samson
pointed to appellant:
04 T Ihinaharap namin sa
iyo ngayon ang anim na
lalaki na nadito (sic) sa loob
ng tanggapan ng Special
Investigation Unit ng CIS
Camp Crame, Quezon City
maari mo bang makilala at
maituro sa mga lalaki na
ihinarap sa iyo kung meron

man sa kanilaang bumaril


kay Petronilo Lim?
S Mayroon pong isa yong
lalaking may bigote na
nakasuot ng puting T-shirt at
maong na pantalon na
ikalawa sa aking gawing
kaliwa. (Affiant pointing to
the person of Dionisio
Lapura y Cajan in a Police
line-up composed of six
persons)
05 T Ito bang tao na
itinuturo mo ngayon ay
nasisiguro mo siya na isa sa
tatlo na magkakasama na
bumaril kay Petronilo Lim
na tinutukoy mo sa iyong
salaysay?
S Opo siya ang isa sa tatlo
na tinutukoy ko na bumaril
kay Mr. Petronilo Lim at
siya iyong bumaril na nasa
gawing kaliwa ng
kotse. 19 (Emphasis
supplied.)
Most importantly, on the witness stand, Samson,
although visibly shaken 20 and notwithstanding the
vigorous and dramatic 21 cross-examination by
defense counsel, still stood by his statement that
appellant was the assailant who fired from the left
side of the victim's car.
The defense could not attach any evil motive on the part of
Samson that might have impelled him to testify falsely
against appellant. 22 Absent the most compelling reason, it is

inconceivable why Samson would openly concoct a story


that would send an innocent man to jail. His credibility has
likewise been tested and favorably evaluated by the trial
court, a matter which cannot just be ignored.

chance to look for a lawyer


to assist him during that
line-up as part of the
investigation?

Appellant contends that he has not been provided with the


assistance of coirnsel at the "police line-up." He suggests
that the Court take another look at its rulings on the right of
an accused to counsel during a "police line-up." 23

FISCAL

Upon his arrest, appellant Lapura refused to make any


statement. 24 During the investigation that followed,
appellant's counsel, Atty. Orlando Salatandre, was in the
investigation room, a fact which, unexpectedly, he himself
elicited from Agent Caizares on cross-examination. Thus:

Objection, Your Honor, the


question is premised on the
fact; it's one of the
constitutional rights of the
accused to be assisted by
counsel during the police
line up
ATTY. SALATANDRE.

Q But do I recall from you


that you have noticed this
Samson pinpointing one of
them?
A Yes, sir, that is part of his
statement, the identification
of the suspect.
Q Despite the fact you were
busy during that line up?

If Your Honor please, the


line-up is already part of the
investigation, Your Honor
please. Then, the accused
therefore, must be assisted
by counsel because who
knows it's done without
prejudice to the procedure,
Your Honor please.
COURT

A I was busy supervising


the investigation but the
Special Investigation Unit is
only a small room that you
could notice what is
happening within.
Q Considering the fact you
were present, did you notice
the accused before he was
included in this line up
whether he was given a

Let the witness answer.


A If I am not mistaken, I do
not only remember exactly
the time element. If I
remember correctly on that
same night you were there,
you appeared there.
ATTY. SALATANDRE

Yes, I was there. 25


Given the above circumstances, it would be wrong
to conclude that Atty. Salatandre was unable to
provide legal assistance to appellant. Moreover, the
inadmissibility of a police line-up identification of
an uncounseled accused should not necessarily
foreclose the admissibility of an independent incourt identification. In this instance, Samson, in
open court, pointed to appellant and identified him
again to be the person dressed in white T-shirt, pants
and shoes who had come running towards him and
the bridge after shooting Mr. Lim. 26
Proof of motive, contrary to appellant's allegation, is not
crucial where the identity of the accused, such as here, has
been amply established. 27
Appellant contends that People vs. Opida, 28 where the Court
exonerated the appellant for nonobservance of certain of his
constitutional rights as an accused, should also be applied in
acquiring him. He alleges that
Like accused-appellants Olpida (sic) and
Marcelo in that case, the constitutional
rights of herein appellant have been grossly
violated. From the very time that he was
arrested without warrant on February 25,
1988 and detained at WPD station, Tondo,
Manila in connection with the murder of
Petronilo Lim, five days after its
perpetration on the flimsy ground that his
face resembles that of man drawn in a
cartograph prepared by the police, perhaps
based on the descriptions of people who
may have witnessed the crime, to the filing
of the fatally flawed information on March
2, 1988 or 7 days after his unlawful arrest,
he was deprived of his constitutional rights
against unreasonable search and seizure, of

his right against arbitrary or unlawful arrest,


of his right to remain silent and to counsel
and to be informed of said rights, of his right
to a preliminary investigation in a crime
cognizable by the Regional Trial Court and
finally of his right to have a valid
information against him filed within the
period allowed by law implied in Article 125
of the Revised Penal Code the provisions of
which he had never waived. 29

Revised Penal Code, its medium Period of reclusion


perpetua. 34
WHEREFORE, the Decision in Criminal Case No. 88-61209
of the Regional Trial Court of Manila, Branch 12, convicting
appellant Dionisio Lapura y Cajan of the crime of murder for
the killing of Petronilo Lim and imposing on him the penalty
of reclusion perpetua is hereby AFFIRMED. Costs against
appellant.
SO ORDERED.

We agree with the Solicitor General that Opida is


predicated on two vital premises; viz: (a) the trial
judge's palpable partiality, as well as the irregular
manner in which he conducted his interrogation of
the accused and their witness, and (b) the admission
of an extrajudicial confession despite strong
evidence of manhandling by the police. These
circumstances do not obtain in this case. During
trial, while the judge did propound questions to the
witnesses, they clearly appear, however, to be in the
main clarificatory and certainly not adversarial in
character. 30 Relative to his alleged warrantless
arrest, he has waived, by filing a petition for bell,
any irregularity attendant thereto. 31 Indeed, by his
application for bail and by entering a plea of not
guilty and then submitting to the proceedings below,
appellant must be deemed to have foregone his right
to preliminary investigation and to question any
irregularity that might have attended such
investigation. 32
The trial court correctly appreciated the aggravating
circumstance of treachery. Treachery attended the killing
since the attack was perpetrated suddenly and without
warning of any kind. 33 Evident premeditation, although
alleged in the information, was not at all established. Absent
any mitigating or aggravating circumstance, the penalty due
for imposition would be, in the penalty of reclusion
temporal maximum to death prescribed by Article 248 of the

G.R. No. 116011 March 7, 1996


PEOPLE OF THE PHILIPPINES, plaintiff-appellee,
vs.
RHODESA* SILAN y BORQUE, alias "Dada," and
VIRGILIO GARCIA, alias "Billy Garcia," accusedappellants.

REGALADO, J.:p
Accused-appellants were prosecuted for the special complex
crime of robbery with homicide in Criminal Case No. 93016
of the Regional Trial Court, Branch 163, of Pasig, Metro
Manila upon an indictment filed on June 17, 1992 and
alleging
That on or about the 1st day of June, 1992,
in the Municipality of Marikina, Metro
Manila, Philippines and within the
jurisdiction of this Honorable Court, the
above-named accused, conspiring and
confederating together with John @ "Tol",

whose true nam(e), identity and present


whereabouts (are) still unknown and all of
them mutually helping and assisting one
another, by means of force, violence and
intimidation employed upon the person of
Evangeline Gargantos, did then and there
willfully, unlawfully and feloniously take,
steal and carry away complainant's personal
belongings and valuables all amounting to
P6,500.00, to the damage and prejudice of
the offended party in the aforementioned
amount of P6,500.00; that on the occasion of
the said robbery, the above-named accused,
conspiring and confederating together with
John Doe @ "Tol", whose true name,
identity and present whereabout(s) (are) still
unknown and they all mutually helping and
assisting one another, with evident
premeditation and treachery, with the use of
superior strength and during nighttime,
while armed with bladed weapon, with
intent to kill, did then and there willfully,
unlawfully and feloniously attack assault,
strangle with a cord of an electric stove, and
stab said Evangeline Gargantos, thereby
inflicting upon her fatal injuries which
caused her death. 1
They were both found guilty as charged in the decision of
said court, dated October 21, 1993, which adjudged as
follows:
WHEREFORE, foregoing premises
considered, this Court finds both accused,
Rhodesa Silan and Virgilio Garcia guilty as
principal(s) of the crime of Robbery with
homicide defined in Article 293 and
penalized under Article 294. And,
considering the aggravating circumstance of
the crime having been committed in the

dwelling of the offended party and there


being no mitigating circumstance on record
this Court sentences them to suffer the
penalty of reclusion perpetua with its
accessories provided for by law, to restitute
to the heirs of Evangeline Gargantos the
value of the article stolen from her house in
the amount of P6,500.00; to indemnify said
heirs (in) the sum of P50,000.00 and to pay
the cost.
If accused has signed a written conformity
to abide with the rules on convicted
prisoner, their detention service if any
should be deducted from this sentence
computed as provided for by law. 2
We have carefully and thoroughly reviewed the records of
this case, especially the transcripts of the stenographic notes
taken at the trial, the documentary evidence, and the
respective presentations of the parties, and we find that these
findings of the court below which were adopted in the
People's brief provide us sufficient and accurate bases for
this appellate review:
Investigation conducted by police
investigator Ricardo Domingo revealed that
Rhodesa Silan was seen by Carol
Concepcion about 2:00 P.M. on either 31
May or 1 June 1992. Silan inquired from
Carol as to whether or not there was anyone
in the house of her auntie. Carol told her that
there is nobody there because they are (o)n
vacation. Silan told Carol that she would
like to get something inside the house.
About 7:00 o'clock in the evening of the
same day Carol while fetching water, again
saw Silan with a male companion going to
the house of Evangeline Gargantos at the
back. Carol did not recognize the male

companion of Silan (TSN page 4, dated 23


Sept. 1992).
Investigator Domingo took the statement of
Rhodesa Silan who at the time was assisted
by Atty. Salvador Navarro. (p. 5, November
4, 1992, TSN). In her extra-judicial
statement (Exh. "H") Silan stated that she
went to the house of her auntie Vangie in the
late afternoon of 1 June 1992. (p. 4, July 1,
1993, TSN). Her companions in going to
Vangie's house were Virgilio "Billy" Garcia
and a certain Tol. (p. 4, ibid.) They went to
Vangie's house to get her things so they can
sell them as Billy needed money. (p. 4, ibid.)
Upon reaching the house of Vangie they
found it closed. They went to the back
portion where Billy used a screw driver in
destroying the padlock of the kitchen door.
(p. 5, ibid.) Once the kitchen door was
opened, they entered the house, Silan going
to her room while Billy and Tol went
upstair(s). (p. 5, ibid.) While Silan was
arranging her things she heard a loud sound
of objects falling. She went up the house and
found that an electric fan fell on the floor.
She saw Billy and Tol searching something.
(p. 5, ibid.) She also notice (sic) that the
room was in disarray. It was at this juncture
that Billy told her to get things that she
wants. She then got two jackets, coleman,
colored blue dress of Remia, perfume,
make-up kit, t-shirt, girdle of Remia and
plate with saucer. When they went down
from the second floor of the house Billy
talked with her and forced her to go to her
former room where he forced her to have
sex with him. It was at that instance when
her auntie Vangie arrived and knocked at the
door. (p. 6, ibid.) Billy then ordered her to

stand up in one corner and told her he will


take care of things. He then put off the light
and opened the door and allowed her auntie
to come in. (p. 6, ibid.) After her auntie
entered the house there was a commotion.
(p. 6, ibid.) She heard her auntie shouting
"Who are you, why did you enter the house"
followed by shouts for help "Aling Lina,
Aling Lina, tulungan ninyo ako, may
gustong pumatay sa akin". (p. 6, ibid.) Soon
thereafter silence followed. Then Billy
returned for her and pulled her out the
house. (p. 7, ibid.)
Some of the articles taken by Silan from the
house of Evangeline Gargantos were
recovered from the house were she is now
living with her live-in partner. Said articles
were identified in Court by SPO1 Celso
Cruz, the evidence custodian to whom the
recovered articles were turned over after the
same were recovered. (p. 3, October 22,
1992, TSN) Remia Gargantos Gillo, a niece
of the victim who also lived with Vangie in
#29 Sable St., SSS Village, identified the
blouse, the t-shirt, a bottle of perfume and
powder case as that belongin(g) to the
family of the deceased. (p. 6, October 22,
1992, TSN)

sufficiently established by the prosecution. 4 Parenthetically,


appellant Silan also presented herself as the sole witness in
her defense.
Appellant Garcia has a more extended assignment of errors
in his brief, 5 contending that the court below erred (1) in
relying on the testimony and extrajudicial confession of
appellant Silan in convicting him; (2) in failing to consider
that appellant Silan had sufficient motive to implicate him in
the crime charged; (3) in quickly dismissing his defense of
alibi; and (4) in not considering the illegality of his arrest by
the police.
We absolve the trial court of the aforestated errors imputed
to it and which we shall discuss seriatim conjointly with our
reasons for rejecting the same, starting with the submissions
of the lady appellant first.
1. Appellant Silan leads off with the argument that her only
intention in going to the house of her aunt, the victim
Evangeline Gargantos, was to get some of her things which
she had left behind when she left the place. She claimed that
she intended to sell the same and give the money to appellant
Garcia because of his threat to kill her brother if she did not
do so. She denied that there was any prior agreement
between her and appellant Garcia or his companion, called
"Tol," to rob and kill her aunt, and that those crimes were
committed by the latter two without her participation or
consent.

saw Garcia and Tol taking things from the room of her aunt,
she asked them to desist but they refused. According to her,
this proved that she did not consent to those larcenous acts of
her companions.
We believe otherwise. Her simple admonition, even if true, is
not evidence of a true intent to prevent her associates from
taking valuables from the house. She could and should have
insisted, for apparently there was no danger to her at that
moment, but she did not do so. Further belying her supposed
solicitude is the fact that she readily accepted the white bag
containing part of the items taken from the room of the
victim. If she really had no intention to gain, she should have
returned the white bag which she admittedly opened and
wherein she discovered valuables belonging to her aunt and
the latter's two
sons. 6
On the contrary, the extrajudicial admissions in her sworn
statement 7 which she voluntarily executed on June 15, 1992
at the Marikina Police Station, with the legal safeguards
discussed hereinafter, clearly spell out in detail that she and
her aforesaid confederates planned the robbery. Her act of
taking things when she was allegedly instructed by appellant
Garcia to take whatever she liked necessarily made her a
principal by direct participation through confederacy in the
robbery.

What also caught our attention is the testimony of


prosecution witness Carol Concepcion that she saw and
We
reject
this
hand
washing
attempt.
It
is
established
that
the
talked to appellant Silan not only on June 1, 1992 but also
Atty. Salvador Navarro affirmed before the
prior thereto, that is, in all probability on May 31, 1992
Court the fact that he assisted Rhodesa Silan three of them first agreed to meet at a specific place on an
appointed time and, from there, they then proceeded together likewise near the very house of the victim at Marikina. On
when she gave her extra-judicial statement
May 31, 1992 when witness Concepcion saw appellant Silan
before police investigator Ricardo Domingo. to the victim's house at 29 Sable Street, SSS Village,
3
Marikina, Metro Manila. The events that transpired after
at around 2:00 P.M., the latter asked her whether there was a
(pp. 5-7, ibid) (pp. 2-3, Decision)
they had forced their way into the house clearly revealed
man in the house, informing the former that she was going
how they cooperated with each other to successfully loot the inside the house to get a few things which she had left
It is appellant Silan's position, as articulated in her lone
assignment of error, that the lower court erred in convicting house and escape undetected through the back door. When
behind. 8
appellant Silan was in her former room, she said she heard
her of the crime of robbery with homicide without proof of
noises coming from upstairs and that when she went up and
conspiracy between her and appellant Garcia having been

Viewed from the events that transpired thereafter, her


testimony that the reason why she went with her companions
to the house of her aunt on June 1, 1992 was just to get her
things is extremely doubtful and obviously contrived for she
could have taken everything that she left behind the first
time that witness Carol Concepcion saw her on May 31,
1992. It would, therefore, not be far-fetched to assume that
during this first meeting with Carol Concepcion at the
residence of Evangeline Gargantos, appellant Silan was
actually conducting a surveillance of the place prior and
consequent to the plan to ransack the same on June 1, 1992.
It is likewise plausible to assume that she had reason to be
angry with her aunt for she was practically thrown out of the
house, because of her having stolen the camera of the latter
and pawning the same, aside from previous similar incidents.
In her testimony, she admitted that the reason why she left
the house was because she was forced out due to the
argument with her aunt regarding that camera. 9 It is apparent
that she had every opportunity to return the things which she
took from her aunt's house but she did not.

original plan of the group to commit robbery, there being


conspiracy among the three malefactors, the crime of one is
the crime of all. Reiterating our previous holdings on this
issue, People vs. Nunag, et al. 10 stressed the rule that
whenever homicide has been committed as a consequence or
on the occasion of the robbery, all those who took part as
principals in the robbery will also be held guilty as principals
for the special complex crime of robbery with homicide,
although they did not actually take part in the homicide.
2. We are not at all impressed by appellant Garcia's
exculpatory attempt to attribute his being enmeshed in this
criminal case to the extrajudicial statement of appellant
Silan. He assails the same for allegedly having been obtained
by the police authorities without appellant Silan having been
provided with counsel. Ironically, this claim was likewise
belatedly made by Silan in a tragicomic deviation after she
had voluntarily narrated in her aforesaid statement the details
of her participation in the events that took place in the
victim's residence.

How appellant Silan came to give that statement, and how it


With regard to the killing of her aunt, it is indeed out of the
was reduced to its present form as submitted in evidence, is
ordinary for appellant Silan not to even bother to find out
vividly reported in the testimony of SP04 Ricardo S.L.
what happened to her own aunt when, in fact, the last that
Domingo, the police investigator. 11 The statement was taken
she heard from the victim was the latter's cries for help after down in the presence of this appellant's own mother,
appellant Garcia and Tol went to intercept her at the door.
Catalina Silan, as indicated therein and whose signature is
Appellant Silan may not have participated in the actual
affixed to the pages thereof. Said appellant was assisted by
killing of Evangeline Gargantos for in truth there is no
Atty. Salvador Navarro, whose appearance and attendance
evidence specifically showing who actually tied the electric during the interrogation and preparation of this document is
cord around the victim's neck and stabbed her twice.
also specifically mentioned therein and on the first page
However, the series of events prior to the June 1, 1992
whereof he also affixed his signature. On top of that, said
incident, the testimony of witness Carol Concepcion, and the Atty. Navarro testified in court 12 on his assistance given to
manifest concerted acts of appellant Silan and her two
appellant Silan throughout the entire proceeding and how the
confederates as recounted by the appellant Silan herself,
constitutional injunctions mentioned in the sworn statement
irresistibly suggest that appellant Silan did participate in the were duly observed.
planning and commission of the composite crime charged.
Although the killing of Evangeline Gargantos may just have
been an unfortunate complication and was not part of the

And, if that would not be enough, appellant Silan herself


took the witness stand and affirmed to the trial court her
participation in the crime, virtually reiterating what she
stated in her extrajudicial statement, thus:
Q Ms. witness, on June 1,
1992 at about 5:30 P.M.,
you remember where you
were?
A Yes, ma'm.
Q Where were you?
A At Caloocan, Quezon
City in Bonifacio.
Q What were you doing in
Bonifacio?
A I was with "Billy" Garcia
and his friend (whom) he
used to call Tol.
xxx xxx xxx
Q What happened?
A He (was) asking for (a)
sum of money.
Q Did you give him money?
A No, ma'm
Q Why?
A Because I (did) not have
money.

Q So, when you did not


give him money, what did
you do?
A I remembered that when I
left Sable Street, and I have
few belongings left behind
and then he asked me if we
can go there?
Q What is that place?
A Sable St., SSS Village,
Marikina, Metro Manila.
Q Who owns that house at
SSS Village?
A My auntie.
xxx xxx xxx
Q Were you able to go to
that house?

xxx xxx xxx

A Not immediately, ma'm.

Q Were you able to reach


that house of Mrs.
Gargantos?

Q Why?
A Because the door (was)
locked.

A Yes, ma'm.
Q What time did you arrive?
A At around 7:00 o'clock in
the evening, ma'm.
Q Upon reaching the house,
what did you do, the three
of you?

Q (How) were you able to


unlock the door?
A He brought out a screw
driver and unscrew(ed) the
lock.
Q The door was destroyed,
(wasn't) it?

A When we reached the


house the door was close(d)
and I told them, we just
leave the house.

A The padlock only was


destroyed.

Q Did you leave the house?

A When we entered the


house, I went to my room.

A Yes, ma'm.

A No, ma'm, because he


was insisting to enter the
house.

Q You said you went to that


SSS Village, who were your
companion(s)?

Q Who (was) insisting?

Q Did you enter the house?

Q How about the other 2,


where did they go?
A They also entered the
house, and went upstair(s).

A Billy Garcia.
Q How did you know?

A Billy Garcia.
Q What did he tell you?
Q Only?
A And a certain friend of
hi(s) which I don't know the
name. He just called him
"Tol."

A He told us, we just enter


so that we can get my
belongings.
Q So, did you enter the
house?

A Because when I (was) in


my room downstair(s), I
heard a noise upstair(s), as
if something fell, so I went
upstair(s) and saw that the
door was also opened and
unlocked.

Q What did you notice?


A Bill(y) Garcia and his
friend (were) already
upstair(s) and the room was
in disarray.

A Billy Garcia handed me a


white bag and we were
about to leave.
Q You were about to leave,
what happened?

Q What is the commotion


all about?
A There was something
happening in the house.
Q What did you hear?

Q What did these two


person do?

A When we were about to


leave, my auntie came.

A As if they are searching


for something.

Q How did you know?

Q What did you do, when


you saw them as if they are
searching for something?
A I tried to stop them.
Q And what was their
reply?
A Billy Garcia said, "Don't
make any noise. Huwag
kang makialam dito, kami
na ang bahala dito."

A She was knocking at the


door.
Q Who open(ed) the door?
A Before she enter(ed),
Billy Garcia turned off the
light in the house.

A My auntie shouting.
Q Shouting of what, what
was her statement?
A She was shouting, "Aling
Lina, Aling Lina, please
help me."
Q What did you do, whe(n)
she was asking for help?
A None, ma'm.

Q After it was turned off,


what happened next?

Q Why?

A He told me not to panic


because we can get out of
the house.

A Because I ha(d) a mixed


emotion at that time. I (was)
afraid and nervous.

Q (Was) your auntie able to


enter the house?

xxx xxx xxx

Q What did they do?


A They kept on searching
and I went downstair(s), I
continued arranging my
belongings.
Q After you were through in
arranging your belongings,
can you please tell us what
happened next?

A Yes, ma'm.
Q What happened, if you
know?
A When I was in my room I
heard there was a
commotion, something
happening.

Q You said there was a


commotion and your auntie
shouting for help, what
happened after that?
A She (was) also shouting
that somebody was trying to
kill her and then after a few
moments, there was silence.
And Billy Garcia went to

my room and told me we


were leaving.
Q So, after Billy Garcia
entered your room, you left
the house?
A Yes, ma'm.
Q Where did you pass?
A The back door.
Q Do you know what
happened to your auntie
before leaving the house?
A No, ma'm.
Q You said a while ago that
your auntie (was) dead,
when for the first time did
you learn that she was
dead?

cross-examination, as in fact she was cross-examined


thereon, the initial objections of appellant Garcia have been
rendered academic.
There being no legal obstacle, on either constitutional or
evidentiary grounds, to our considering the inculpatory
statements of appellant Silan both against her and appellant
Garcia, we find said testimony of the former to be credible
just as the trial court also lent its imprimatur thereto.
Appellant Garcia's involvement and incrimination in this
case, contrary to his desperate asseverations, rests on a firm
basis. Significantly, the brief of appellee even confines its
arguments to the representations made in appellant Garcia's
brief, but does not even bother to make any substantial
comment on that of appellant Silan.
Appellant Garcia contends that the trial court failed to
consider the fact that appellant Silan had a sufficient motive
to implicate him in the offense with which they both now
stand charged. He refers to the amorous relationship between
them and later, between her and his brother, both of which
associations "turned sour." We likewise reject this
speculative hypothesis.

As the prosecution points out, the trial court relied upon the
testimony of appellant Silan not only because of her
extrajudicial statement but also because she categorically
testified that appellant Garcia was one of the culprits in the
offense charged. It was convinced, and so are we, that her
Give or take a few passages, the aforequoted testimony of
aforesaid testimony, minus the understandable attempts at
appellant Silan is a repetition of what she declared in her
justification of her acts and mitigation of her liability, was
sworn statement aforementioned. The same, however, is
credible and can be fully appreciated against appellant
further challenged by appellant Garcia as being inadmissible Garcia. He completely failed to present any evidence on the
in evidence as an extrajudicial statement which did not have alleged ill motive of appellant Silan. Surprisingly, he even
all the requisites for its admission as an exception to the rule admits in his brief that, on the matter of the unrequited
of res inter alios acta. Although we do not agree with his
relationships between Silan and him or his brother, "there is
strained dissertation thereon, it would be pointless to further no direct relation that would convince any court that this
dwell on that objection since the contents of the same have
could have spur(r)ed Silan in implicating (him)" in the
been reproduced as judicial admissions by appellant Silan's offense charged and now under review. 14
testimony in court. Her admissions having been laid open for
A Few days before my
arrest. 13 (Corrections in
parentheses supplied)

Appellant Garcia further insists that although he was


identified by appellant Silan as her co-conspirator and as one
of the perpetrators of the offense, no prosecution witness
made that positive identification, hence his alibi should be
sustained. We do not believe that there is any logic to what
he would now require in addition to the eyewitness account
of his own co-conspirator declared in open court subject to
such cross-examination as the defense desired to conduct.
We accordingly quote with approval what the trial court
observed on this point:
As to the accused Virgilio Garcia, the extrajudicial confession of her co-accused
Rhodesa Silan, implicating him in the
commission of the crime charged is
admissible against him if such extra-judicial
confession is used only as circumstantial
evidence to show the probability of his
participation (People vs. Victor, 181 SCRA
818). But in the case at bar, in addition to
the extra-judicial confession of Rhodesa
Silan, she also testified in open court
pointing to her co-accused Virgilio Garcia as
her companion in entering the house of the
victim by opening its back door with the use
of screw driver. She also pointed to Garcia
as the person who ransacked the bedroom of
the house of the victim. She likewise pointed
to Garcia as the person who ordered her to
stand by in her former room while he opens
the door of the kitchen and allowed the
victim to enter the house. She further stated
that after the victim entered the kitchen
door, there was a commotion and she heard
the victim shouting for help and that when
the victim became silent Garcia returned to
where she was, pulled her and together they
left the place. 15

Finally, appellant Garcia bewails and assigns as reversible


error the failure of the trial court to take into account his
supposed illegal arrest which later culminated in the filing of
the charges against him. We note, however, that he never
objected thereto nor placed that matter in issue when,
instead, he entered his plea on arraignment and went to trial.
As the Solicitor General points out, even assuming that he
was illegally arrested, this will not affect his culpability since
an allegation of a warrantless arrest cannot deprive the State
of its right to convict the guilty when all the facts on record
point to his culpability. 16
We also agree with the advertence to our pronouncement in
the aforecited case of Briones which could very well be said
of this incident in the case at bar:
Immediately after their arrest, appellants . . .
could have objected to the legality thereof
due to the failure of the police officer to
secure first a warrant for their arrest. Not
only that, without having questioned the
legality of their arrest, they even pleaded, on
arraignment, to the information filed against
them. Appellants' acts constitute a clear
waiver of their right against unlawful
restraint of liberty. Besides, it would be
impractical, if not ridiculous, to order the
court a quo to set the appellants free then
issue a warrant for their arrest, and try them
all over again when appellants themselves
have waived their right to object to such
irregularity and when their objection is truly
based on overwhelming evidence.
ON THE FOREGOING CONSIDERATIONS, the appealed
judgment of the court a quo is hereby AFFIRMED in toto,
with costs against accused-appellants Rhodesa Silan y Roque
and Virgilio Garcia.
SO ORDERED.

1.In the instant case, the lower court lifted the three
questioned search warrants against the
privaterespondents on the ground that it acted on the
20thCentury Fox v CA
application for the issuance of the said search warrantsand
Date (19 August 1988) | Ponente: Gutierrez Jr.
granted it on the
Overview: Search warrants were recalled because
misrepresentations of applicant NBI and its witnesses
the NBI and witnesses misrepresented that they
that infringement of copyright or a piracy of a particular film
had personal knowledge of the piracy.
have been committed- A s f o u n d o u t b y t h e c o u r t ,
Statement of the Case
the NBI agents who acted as witnesses
-The lower court later on lifted the 3 search
did not havepersonal knowledge of the subject matter of their testimony
warrants and ordered the NBI to return the
which was the allegedcommission of the offense by the
properties thatwere seized.- C A d i s m i s s e d
private respondents.M R s .
Only the petitioner's counsel who was also a witness during the
Statement of Facts
application for the issuance of the search warrants stated that
-August 26, 1985: a letter-complaint by petitioner
he had personal knowledge
20thCentury Fox Film Corporation through
counselsought the National Bureau of Investigation's (NBI) that theconfiscated tapes owned by the private respondents
were pirated tapes taken from master tapes belonging to the
assistance in the conduct of searches and seizuresin
petitioner.
connection with the latter's anti-film piracy campaign.However, the lower court did not give much credenceto his testimony
Specifically, the letter-complaint alleged that
in view of the fact that the master tapes of the allegedly
certain videotape outlets all over Metro Manila are
piratedtapes were not shown to the court during the application.
engagedin the unauthorized sale and renting out of
copyrighted films in videotape form which constitute
The essence of a copyright infringement is the similarity or
aflagrant violation of Presidential Decree No. 49(otherwise
at least substantialsimilarity of the purported pirated works to the
known as the Decree on the Protection of Intellectual
copyrighted work
Property).-Acting on the letter-complaint, the NBI
. Hence, the applicant
conductedsurveillance and investigation of the
outlets pinpointedbythe petitioner and subsequently filed must present to the court the copyrighted films to compare
them with the purchasedevidence of the video tapes
three (3) applicationsfor search warrants.- S e p t e m b e r 4 ,
allegedly pirated to determine whether the latter is
1985: the lower court issued the desired
anunauthorized reproduction of the former.
s e a r c h w a r r a n t s . -The NBI accompanied by the
This linkage of the copyrighted films to thepirated films
petitioner's agents, raided the video outlets and
seized the itemsdescribed therein.-An inventory of the must be established to satisfy the requirements of probable
cause. Mereallegations as to the existence of the copyrighted
items seized was made and left with the private
films cannot serve as basis for theissuance of a search
respondents.-The lower court later on lifted the 3
warrant.-Search warrant must contain a specific
search warrants and ordered the NBI to return the
description of the articles to be seized.
properties thatwere seized.
Generalwarrants are constitutionally objectionable.
Applicable Laws:
Judgment:
Section 2, Article Ill, 1987 Constitution.
Petition dismissed.
Issues:
1.Was there grave abuse of discretion on the part of
the lower court when it lifted the search warrants
itearlier issued against the private respondents? No.
Columbia Pictures Entertainment vs Court of Appeals
Rationale
On February 9, 2012

ISSUE: Whether or not the 20th Century Fox ruling may be


applied retroactively in this case.

demonstration involving the direct use of the senses of the


presiding magistrate. Such auxiliary procedure, however,
Intellectual Property Law on Copyright Requirements
does not rule out the use of testimonial or documentary
Before a Search Warrant May Be Issued in Copyright Cases HELD: No. In 1986, obviously the 1988 case of 20th
evidence, depositions, admissions or other classes of
Piracy
Century Fox was not yet promulgated. The lower court could evidence tending to prove the factum probandum, especially
not possibly have expected more evidence from the VRB
where the production in court of object evidence would
In 1986, the Video Regulatory Board (VRB) applied for a
and Columbia Pictures in their application for a search
result in delay, inconvenience or expenses out of proportion
warrant against Jose Jinco (Jingco), owner of Showtime
warrant other than what the law and jurisprudence, then
to is evidentiary value.
Enterprises for allegedly pirating movies produced and
existing and judicially accepted, required with respect to the
owned by Columbia Pictures and other motion picture
finding of probable cause.
In fine, the supposed pronouncement in said case regarding
companies. Jingco filed a motion to quash the search warrant
the necessity for the presentation of the master tapes of the
but the same was denied in 1987. Subsequently, Jinco filed
The Supreme Court also revisited and clarified the ruling in copy-righted films for the validity of search warrants should
an Urgent Motion to Lift the Search Warrant and Return the the 20th Century Fox Case. It is evidently incorrect to
at most be understood to merely serve as a guidepost in
Articles Seized. In 1989, the RTC judge granted the motion. suggest, as the ruling in 20th Century Fox may appear to do, determining the existence of probable cause in copy-right
The judge ruled that based on the ruling in the 1988 case of that in copyright infringement cases, the presentation of
infringement cases where there is doubt as to the true nexus
20th Century Fox Film Corporation vs CA, before a search
master tapes of the copyright films is always necessary to
between the master tape and the pirated copies. An objective
warrant could be issued in copyright cases, the master copy meet the requirement of probable cause for the issuance of a and careful reading of the decision in said case could lead to
of the films alleged to be pirated must be attached in the
search warrant. It is true that such master tapes are object
no other conclusion than that said directive was hardly
application for warrant.
evidence, with the merit that in this class of evidence the
intended to be a sweeping and inflexible requirement in all
ascertainment of the controverted fact is made through
or similar copyright infringement cases.

i
ii
iii
iv
v
vi
vii
viii
ix
x
xi
xii
xiii
xiv
xv
xvi
xvii
xviii
xix
xx
xxi

xxii
xxiii
xxiv
xxv
xxvi
xxvii
xxviii
xxix
xxx
xxxi

You might also like